*NURSING > TEST BANK > NURSING 100 MATERNAL CHILDNURSING CARE 6TH EDITION PERRY TEST BANK CHAPTER 1-49,100% CORRECT (All)

NURSING 100 MATERNAL CHILDNURSING CARE 6TH EDITION PERRY TEST BANK CHAPTER 1-49,100% CORRECT

Document Content and Description Below

NURSING 100 MATERNAL CHILDNURSING CARE 6TH EDITION PERRY TEST BANK CHAPTER 1-49 Chapter 01: 21st Century Maternity Nursing Perry: Maternal Child Nursing Care, 6th Edition MULTIPLE CHOICE ... 1. When providing care for a pregnant woman, the nurse should be aware that one of the most frequently reported maternal medical risk factors is: a. diabetes mellitus. b. mitral valve prolapse (MVP). c. chronic hypertension. d. anemia. ANS: A The most frequently reported maternal medical risk factors are diabetes and hypertension associated with pregnancy. Both of these conditions are associated with maternal obesity. There are no studies that indicate MVP is among the most frequently reported maternal risk factors. Hypertension associated with pregnancy, not chronic hypertension, is one of the most frequently reported maternal medical risk factors. Although anemia is a concern in pregnancy, it is not one of the most frequently reported maternal medical risk factors in pregnancy. PTS: 1 DIF: Cognitive Level: Knowledge OBJ: Nursing Process: Assessment MSC: Client Needs: Physiologic Integrity 2. To ensure optimal outcomes for the patient, the contemporary maternity nurse must incorporate both teamwork and communication with clinicians into her care delivery. The SBAR technique of communicatioNnUiRsSaInNGeaTsBy.C-tOo-Mremember mechanism for communication. Which of the following correctly defines this acronym? a. Situation, baseline assessment, response b. Situation, background, assessment, recommendation c. Subjective background, assessment, recommendation d. Situation, background, anticipated recommendation ANS: B The situation, background, assessment, recommendation (SBAR) technique provides a specific framework for communication among health care providers. Failure to communicate is one of the major reasons for errors in health care. The SBAR technique has the potential to serve as a means to reduce errors. PTS: 1 DIF: Cognitive Level: Comprehension OBJ: Nursing Process: Assessment, Planning MSC: Client Needs: Safe and Effective Care Environment 3. The role of the professional nurse caring for childbearing families has evolved to emphasize: a. providing care to patients directly at the bedside. b. primarily hospital care of maternity patients. c. practice using an evidence-based approach. d. planning patient care to cover longer hospital stays. ANS: C Professional nurses are part of the team of health care providers who collaboratively care for patients throughout the childbearing cycle. Providing care to patients directly at the bedside is one of the nurse's tasks; however, it does not encompass the concept of the evolved professional nurse. Throughout the prenatal period, nurses care for women in clinics and physician's offices and teach classes to help families prepare for childbirth. Nurses also care for childbearing families in birthing centers and in the home. Nurses have been critically important in developing strategies to improve the well-being of women and their infants and have led the efforts to implement clinical practice guidelines using an evidence-based approach. Maternity patients have experienced a decreased, rather than an increased, length of stay over the past two decades. PTS: 1 DIF: Cognitive Level: Comprehension OBJ: Nursing Process: Implementation MSC: Client Needs: Safe and Effective Care Environment 4. A 23-year-old African-American woman is pregnant with her first child. Based on the statistics for infant mortality, which plan is most important for the nurse to implement? a. Perform a nutrition assessment. b. Refer the woman to a social worker. c. Advise the woman to see an obstetrician, not a midwife. d. Explain to the woman the importance of keeping her prenatal care appointments. ANS: D Consistent prenatal care is the best method of preventing or controlling risk factors associated with infant mortality. Nutritional status is an important modifiable risk factor, but a nutrition assessment is not the most important action a nurse should take in this situation. The patient may need assistance from a socialNwUoRrSkIeNrGaTtBs.oCmOeMtime during her pregnancy, but a referral to a social worker is not the most important aspect the nurse should address at this time. If the woman has identifiable high risk problems, her health care may need to be provided by a physician. However, it cannot be assumed that all African-American women have high risk issues. In addition, advising the woman to see an obstetrician is not the most important aspect on which the nurse should focus at this time, and it is not appropriate for a nurse to advise or manage the type of care a patient is to receive. PTS: 1 DIF: Cognitive Level: Application OBJ: Nursing Process: Planning MSC: Client Needs: Health Promotion and Maintenance 5. During a prenatal intake interview, the nurse is in the process of obtaining an initial assessment of a 21-year-old Hispanic patient with limited English proficiency. It is important for the nurse to: a. use maternity jargon in order for the patient to become familiar with these terms. b. speak quickly and efficiently to expedite the visit. c. provide the patient with handouts. d. assess whether the patient understands the discussion. ANS: D Nurses contribute to health literacy by using simple, common words; avoiding jargon; and evaluating whether the patient understands the discussion. Speaking slowly and clearly and focusing on what is important increase understanding. Most patient education materials are written at too high a level for the average adult and may not be useful for a patient with limited English proficiency. PTS: 1 DIF: Cognitive Level: Application OBJ: Nursing Process: Evaluation MSC: Client Needs: Health Promotion and Maintenance 6. When managing health care for pregnant women at a prenatal clinic, the nurse should recognize that the most significant barrier to access to care is the pregnant woman's: a. age. b. minority status. c. educational level. d. inability to pay. ANS: D The most significant barrier to health care access is the inability to pay for services; this is compounded by the fact that many physicians refuse to care for women who cannot pay. Although adolescent pregnant patients statistically receive less prenatal care, age is not the most significant barrier. Significant disparities in morbidity and mortality rates exist for minority women; however, minority status is not the most significant barrier to access of care. Disparities in educational level are associated with morbidity and mortality rates; however, educational level is not the most significant barrier to access of care. PTS: 1 DIF: Cognitive Level: Knowledge OBJ: Nursing Process: Assessment MSC: Client Needs: Safe and Effective Care Environment 7. When the nurse is unsure about how to perform a patient care procedure, the best action would be to: a. ask another nurse. b. discuss the procedure with theNpUaRtiSeInNtG'sTpBh.CyOsiMcian. c. look up the procedure in a nursing textbook. d. consult the agency's procedure manual and follow the guidelines for the procedure. ANS: D It is always best to follow the agency's policies and procedures manual when seeking information on correct patient procedures. These policies should reflect the current standards of care and state guidelines. Each nurse is responsible for her own practice. Relying on another nurse may not always be safe practice. Each nurse is obligated to follow the standards of care for safe patient care delivery. Physicians are responsible for their own patient care activity. Nurses may follow safe orders from physicians, but they are also responsible for the activities that they as nurses are to carry out. Information provided in a nursing textbook is basic information for general knowledge. Furthermore, the information in a textbook may not reflect the current standard of care or individual state or hospital policies. PTS: 1 DIF: Cognitive Level: Application OBJ: Nursing Process: Implementation MSC: Client Needs: Physiologic Integrity 8. From the nurse's perspective, what measure should be the focus of the health care system to reduce the rate of infant mortality further? a. Implementing programs to ensure women's early participation in ongoing prenatal care. b. Increasing the length of stay in a hospital after vaginal birth from 2 to 3 days. c. Expanding the number of neonatal intensive care units (NICUs). d. Mandating that all pregnant women receive care from an obstetrician. ANS: A Early prenatal care allows for early diagnosis and appropriate interventions to reduce the rate of infant mortality. An increased length of stay has been shown to foster improved self-care and parental education. However, it does not prevent the incidence of leading causes of infant mortality rates, such as low birth weight. Early prevention and diagnosis reduce the rate of infant mortality. NICUs offer care to high risk infants after they are born. Expanding the number of NICUs would offer better access for high risk care, but this factor is not the primary focus for further reduction of infant mortality rates. A mandate that all pregnant women receive obstetric care would be nearly impossible to enforce. Furthermore, certified nurse-midwives (CNMs) have demonstrated reliable, safe care for pregnant women. PTS: 1 DIF: Cognitive Level: Comprehension OBJ: Nursing Process: Implementation MSC: Client Needs: Health Promotion and Maintenance 9. Alternative and complementary therapies: a. replace conventional Western modalities of treatment. b. are used by only a small number of American adults. c. recognize the value of patients' input into their health care. d. focus primarily on the disease an individual is experiencing. ANS: C Many popular alternative healing modalities offer human-centered care based on philosophies that recognize the value of the patient's input and honor the individual's beliefs, values, and desires. Alternative and complementary therapies are part of an integrative approach to health care. An increasing number of AmNeUriRcSaInNGadTuBl.tCsOaMre seeking alternative and complementary health care options. Alternative healing modalities offer a holistic approach to health, focusing on the whole person, not just the disease. PTS: 1 DIF: Cognitive Level: Comprehension OBJ: Nursing Process: Planning MSC: Client Needs: Physiologic Integrity 10. A 38-year-old Hispanic woman delivered a 9-pound, 6-ounce girl vaginally after being in labor for 43 hours. The baby died 3 days later from sepsis. On what grounds would the woman potentially have a legitimate legal case for negligence? a. She is Hispanic. b. She delivered a girl. c. The standards of care were not met. d. She refused fetal monitoring. ANS: C Not meeting the standards of care is a legitimate factor for a case of negligence. The patient's race is not a factor for a case of negligence. The infant's gender is not a factor for a case of negligence. Although fetal monitoring is the standard of care, the patient has the right to refuse treatment. This refusal is not a case for negligence; however, informed consent should be properly obtained, and the patient should sign an against medical advice form for refusal of any treatment that is within the standard of care. PTS: 1 DIF: Cognitive Level: Comprehension OBJ: Nursing Process: Planning MSC: Client Needs: Health Promotion and Maintenance 11. A newly graduated nurse is attempting to understand the reason for increasing health care spending in the United States. Her research finds that these costs are much higher compared with other developed countries as a result of: a. a higher rate of obesity among pregnant women. b. limited access to technology. c. increased usage of health care services along with lower prices. d. homogeneity of the population. ANS: A Health care is one of the fastest growing sectors of the U.S. economy. Currently, 17.5% of the gross domestic product is spent on health care. Higher spending in the United States compared with 12 other industrialized countries is related to higher prices and readily accessible technology along with greater obesity rates among women. More than one third of women in the United States are obese. Of the U.S. population, 16% is uninsured and has limited access to health care. Maternal morbidity and mortality are directly related to racial disparities. PTS: 1 DIF: Cognitive Level: Analysis OBJ: Nursing Process: Planning MSC: Client Needs: Safe and Effective Care Environment 12. The term used to describe legal and professional responsibility for practice for maternity nurses is: a. collegiality. b. ethics. c. evaluation. d. accountability. ANS: D NURSINGTB.COM Accountability refers to legal and professional responsibility for practice. Collegiality refers to a working relationship with one's colleagues. Ethics refers to a code to guide practice. Evaluation refers to examination of the effectiveness of interventions in relation to expected outcomes. PTS: 1 DIF: Cognitive Level: Understanding OBJ: Nursing Process: Evaluation MSC: Client Needs: Health Promotion and Maintenance 13. Through the use of social media technology, nurses can link with other nurses who may share similar interests, insights about practice, and advocate for patients. The most concerning pitfall for nurses using this technology is: a. violation of patient privacy and confidentiality. b. institutions and colleagues may be cast in an unfavorable light. c. unintended negative consequences for using social media. d. lack of institutional policy governing online contact. ANS: A The most significant pitfall for nurses using this technology is the violation of patient privacy and confidentiality. Furthermore, institutions and colleagues can be cast in unfavorable lights with negative consequences for those posting information. Nursing students have been expelled from school and nurses have been fired or reprimanded by their Board of Nursing for injudicious posts. The American Nurses Association has published six principles for social networking and nurses. All institutions should have policies guiding the use of social media, and nurses should be familiar with these guidelines. PTS: 1 DIF: Cognitive Level: Analysis OBJ: Nursing Process: Implementation MSC: Client Needs: Safe and Effective Care Environment 14. An important development that affects maternity nursing is integrative health care, which: a. seeks to provide the same health care for all racial and ethnic groups. b. blends complementary and alternative therapies with conventional Western treatment. c. focuses on the disease or condition rather than the background of the patient. d. has been mandated by Congress. ANS: B Integrative health care tries to mix the old with the new at the discretion of the patient and health care providers. Integrative health care is a blending of new and traditional practices. Integrative health care focuses on the whole person, not just the disease or condition. U.S. law supports complementary and alternative therapies but does not mandate them. PTS: 1 DIF: Cognitive Level: Understanding OBJ: Nursing Process: Implementation MSC: Client Needs: Health Promotion and Maintenance NURSINGTB.COM 15. The nurse caring for a pregnant patient should be aware that the U.S. birth rate shows which trend? a. Births to unmarried women are more likely to have less favorable outcomes. b. Birth rates for women 40 to 44 years old are beginning to decline. c. Cigarette smoking among pregnant women continues to increase. d. The rates of maternal death owing to racial disparity are elevated in the United States. ANS: A Low-birth-weight infants and preterm birth are more likely because of the large number of teenagers in the unmarried group. Birth rates for women in their early 40s continue to increase. Fewer pregnant women smoke. In the United States, there is significant racial disparity in the rates of maternal death. PTS: 1 DIF: Cognitive Level: Comprehension OBJ: Nursing Process: Assessment MSC: Client Needs: Safe and Effective Care Environment 16. Maternity nursing care that is based on knowledge gained through research and clinical trials is: a. derived from the Nursing Intervention Classification. b. known as evidence-based practice. c. at odds with the Cochrane School of traditional nursing. d. an outgrowth of telemedicine. ANS: B Evidence-based practice is based on knowledge gained from research and clinical trials. The Nursing Intervention Classification is a method of standardizing language and categorizing care. Dr. Cochrane systematically reviewed research trials and is part of the evidence-based practice movement. Telemedicine uses communication technologies to support health care. PTS: 1 DIF: Cognitive Level: Comprehension OBJ: Nursing Process: Assessment MSC: Client Needs: Psychosocial Integrity 17. The level of practice a reasonably prudent nurse provides is called: a. the standard of care. b. risk management. c. a sentinel event. d. failure to rescue. ANS: A Guidelines for standards of care are published by various professional nursing organizations. Risk management identifies risks and establishes preventive practices, but it does not define the standard of care. Sentinel events are unexpected negative occurrences. They do not establish the standard of care. Failure to rescue is an evaluative process for nursing, but it does not define the standard of care. PTS: 1 DIF: Cognitive Level: Comprehension OBJ: Nursing Process: Diagnosis MSC: Client Needs: Safe and Effective Care Environment 18. While obtaining a detailed historyNfUroRmSINaGwToBm.CaOnMwho has recently emigrated from Somalia, the nurse realizes that the patient has undergone female genital mutilation (FGM). The nurse's best response to this patient is: a. “this is a very abnormal practice and rarely seen in the United States.” b. “do you know who performed this so that it can be reported to the authorities?” c. “we will be able to restore your circumcision fully after delivery.” d. “the extent of your circumcision will affect the potential for complications.” ANS: D “The extent of your circumcision will affect the potential for complications” is the most appropriate response. The patient may experience pain, bleeding, scarring, or infection and may require surgery before childbirth. With the growing number of immigrants from countries where FGM is practiced, nurses will increasingly encounter women who have undergone the procedure. Although this practice is not prevalent in the United States, it is very common in many African and Middle Eastern countries for religious reasons. Responding with, “This is a very abnormal practice and rarely seen in the United States” is culturally insensitive. The infibulation may have occurred during infancy or childhood. The patient will have little to no recollection of the event. She would have considered this to be a normal milestone during her growth and development. The International Council of Nurses has spoken out against this procedure as harmful to a woman's health. PTS: 1 DIF: Cognitive Level: Application OBJ: Nursing Process: Planning MSC: Client Needs: Safe and Effective Care Environment 19. To ensure patient safety, the practicing nurse must have knowledge of the current Joint Commission's “Do Not Use” list of abbreviations. Which of the following is acceptable for use? a. q.o.d. or Q.O.D. b. MSO4 or MgSO4 c. International Unit d. Lack of a leading zero ANS: C The abbreviations “i.u.” and “I.U.” are no longer acceptable because they could be misread as “I.V.” or the number “10.” The abbreviation “q.o.d. or Q.O.D.” should be written out as “every other day.” The period after the “Q” could be mistaken for an “I”; the “o” could also be mistaken for an “i.” With MSO4 or MgSO4, it is too easy to confuse one medication for another. These medications are used for very different purposes and could put a patient at risk for an adverse outcome. They should be written as morphine sulfate and magnesium sulfate. The decimal point should never be missed before a number to avoid confusion (i.e., 0.4 rather than .4). PTS: 1 DIF: Cognitive Level: Application OBJ: Nursing Process: Assessment MSC: Client Needs: Psychosocial Integrity 20. Healthy People 2020 has established national health priorities that focus on a number of maternal-child health indicators. Nurses are assuming greater roles in assessing family health and providing care across the perinatal continuum. Therefore, it is important for the nurse to be aware that significant progress has been made in: a. the reduction of fetal deaths and use of prenatal care. b. low birth weight and preterm bNiUrtRhS. INGTB.COM c. elimination of health disparities based on race. d. infant mortality and the prevention of birth defects. ANS: A Trends in maternal child health indicate that progress has been made in relation to reduced infant and fetal deaths and increased prenatal care. Notable gaps remain in the rates of low birth weight and preterm births. According to the March of Dimes, persistent disparities still exist between African-Americans and non-Hispanic Caucasians. Many of these negative outcomes are preventable through access to prenatal care and the use of preventive health practices. This demonstrates the need for comprehensive community-based care for all mothers, infants, and families. PTS: 1 DIF: Cognitive Level: Knowledge OBJ: Nursing Process: Implementation MSC: Client Needs: Safe and Effective Care Environment MULTIPLE RESPONSE 1. Which interventions would help alleviate the problems associated with access to health care for maternity patients? (Select all that apply.) a. Provide transportation to prenatal visits. b. Provide child care so that a pregnant woman may keep prenatal visits. c. Mandate that physicians make house calls. d. Provide low-cost or no-cost health care insurance. e. Provide job training. ANS: A, B, D Lack of transportation to visits, lack of child care, and lack of affordable health insurance are prohibitive factors associated with lack of prenatal care. House calls are not a cost-effective approach to health care. Although job training may result in employment and income, the likelihood of significant changes during the time frame of the pregnancy is remote. PTS: 1 DIF: Cognitive Level: Implementation OBJ: Nursing Process: Planning MSC: Client Needs: Health Promotion and Maintenance MATCHING Medical errors are a leading cause of death in the United States. The National Quality Forum has recommended numerous safe practices that nursing can promote to reduce errors. Match each safe practice with the correct statement. a. Ask the patient to “teach back.” b. Comply with CDC guidelines. c. Ensure that information is documented in a timely manner. d. Promote interventions that will reduce patient risk. e. Reduce exposure to radiation. 1. Hand hygiene 2. Informed consent 3. Culture measurement, feedback, and intervention 4. Pediatric imaging 5. Patient care information NURSINGTB.COM 1. ANS: B PTS: 1 DIF: Cognitive Level: Application OBJ: Nursing Process: Implementation MSC: Client Needs: Safe and Effective Care Environment NOT: The National Quality Forum updated its publication Safe Practices for Better Healthcare in 2010, outlining 24 safe practices that should be used in all health care settings to reduce the risk of harm from the environment of care, processes, and systems. These are only a few of the recommended practices; however, nurses should be familiar with these guidelines. 2. ANS: A PTS: 1 DIF: Cognitive Level: Application OBJ: Nursing Process: Implementation MSC: Client Needs: Safe and Effective Care Environment NOT: The National Quality Forum updated its publication Safe Practices for Better Healthcare in 2010, outlining 24 safe practices that should be used in all health care settings to reduce the risk of harm from the environment of care, processes, and systems. These are only a few of the recommended practices; however, nurses should be familiar with these guidelines. 3. ANS: D PTS: 1 DIF: Cognitive Level: Application OBJ: Nursing Process: Implementation MSC: Client Needs: Safe and Effective Care Environment NOT: The National Quality Forum updated its publication Safe Practices for Better Healthcare in 2010, outlining 24 safe practices that should be used in all health care settings to reduce the risk of harm from the environment of care, processes, and systems. These are only a few of the recommended practices; however, nurses should be familiar with these guidelines. 4. ANS: E PTS: 1 DIF: Cognitive Level: Application OBJ: Nursing Process: Implementation MSC: Client Needs: Safe and Effective Care Environment NOT: The National Quality Forum updated its publication Safe Practices for Better Healthcare in 2010, outlining 24 safe practices that should be used in all health care settings to reduce the risk of harm from the environment of care, processes, and systems. These are only a few of the recommended practices; however, nurses should be familiar with these guidelines. 5. ANS: C PTS: 1 DIF: Cognitive Level: Application OBJ: Nursing Process: Implementation MSC: Client Needs: Safe and Effective Care Environment NOT: The National Quality Forum updated its publication Safe Practices for Better Healthcare in 2010, outlining 24 safe practices that should be used in all health care settings to reduce the risk of harm from the environment of care, processes, and systems. These are only a few of the recommended practices; however, nurses should be familiar with these guidelines. NURSINGTB.COM Chapter 02: The Family, Culture, Spirituality, and Home Care Perry: Maternal Child Nursing Care, 6th Edition MULTIPLE CHOICE 1. A married couple lives in a single-family house with their newborn son and the husband's daughter from a previous marriage. On the basis of the information given, what family form best describes this family? a. Married-blended family b. Extended family c. Nuclear family d. Same-sex family ANS: A Married-blended families are formed as the result of divorce and remarriage. Unrelated family members join together to create a new household. Members of an extended family are kin, or family members related by blood, such as grandparents, aunts, and uncles. A nuclear family is a traditional family with male and female partners and the children resulting from that union. A same-sex family is a family with homosexual partners who cohabit with or without children. PTS: 1 DIF: Cognitive Level: Knowledge OBJ: Nursing Process: Assessment MSC: Client Needs: Psychosocial Integrity 2. In what form do families tend to be most socially vulnerable? a. Married-blended family b. Extended family c. Nuclear family d. Single-parent family ANS: D NURSINGTB.COM The single-parent family tends to be vulnerable economically and socially, creating an unstable and deprived environment for the growth potential of children. The married-blended family, the extended family, and the nuclear family are not the most socially vulnerable. PTS: 1 DIF: Cognitive Level: Knowledge OBJ: Nursing Process: Planning MSC: Client Needs: Psychosocial Integrity 3. The nurse should be aware that the criteria used to make decisions and solve problems within families are based primarily on family: a. rituals and customs. b. values and beliefs. c. boundaries and channels. d. socialization processes. ANS: B Values and beliefs are the most prevalent factors in the decision-making and problem-solving techniques of families. Although culture may play a part in the decision-making process of a family, ultimately values and beliefs dictate the course of action taken by family members. Boundaries and channels affect the relationship between the family members and the health care team, not the decisions within the family. Socialization processes may help families with interactions with the community, but they are not the criteria used for decision making within the family. PTS: 1 DIF: Cognitive Level: Comprehension OBJ: Nursing Process: Planning MSC: Client Needs: Psychosocial Integrity 4. Using the family stress theory as an intervention approach for working with families experiencing parenting, the nurse can help the family change internal context factors. These include: a. Biologic and genetic makeup. b. Maturation of family members. c. The family's perception of the event. d. The prevailing cultural beliefs of society. ANS: C The family stress theory is concerned with the family's reaction to stressful events; internal context factors include elements that a family can control such as psychologic defenses. It is not concerned with biologic and genetic makeup, maturation of family members, or the prevailing cultural beliefs of society. PTS: 1 DIF: Cognitive Level: Comprehension OBJ: Nursing Process: Diagnosis MSC: Client Needs: Psychosocial Integrity NURSINGTB.COM 5. While working in the prenatal clinic, you care for a very diverse group of patients. When planning interventions for these families, you realize that acceptance of the interventions will be most influenced by: a. educational achievement. b. income level. c. subcultural group. d. individual beliefs. ANS: D The patient's beliefs are ultimately the key to acceptance of health care interventions. However, these beliefs may be influenced by factors such as educational level, income level, and ethnic background. Educational achievement, income level, and subcultural group all are important factors. However, the nurse must understand that a woman's concerns from her own point of view will have the most influence on her compliance. PTS: 1 DIF: Cognitive Level: Application OBJ: Nursing Process: Planning MSC: Client Needs: Psychosocial Integrity 6. The nurse's care of a Hispanic family includes teaching about infant care. When developing a plan of care, the nurse bases interventions on the knowledge that in traditional Hispanic families: a. breastfeeding is encouraged immediately after birth. b. male infants typically are circumcised. c. the maternal grandmother participates in the care of the mother and her infant. d. special herbs mixed in water are used to stimulate the passage of meconium. ANS: C In Hispanic families, the expectant mother is influenced strongly by her mother or mother-in-law. Breastfeeding often is delayed until the third postpartum day. Hispanic male infants usually are not circumcised. Olive or castor oil may be given to stimulate the passage of meconium. PTS: 1 DIF: Cognitive Level: Application OBJ: Nursing Process: Planning MSC: Client Needs: Psychosocial Integrity 7. The woman's family members are present when the home care maternal-child nurse arrives for a after birth and newborn visit. What should the nurse do? a. Observe the family members' interactions with the newborn and one another. b. Ask the woman to meet with her and the baby alone. c. Do a brief assessment on all family members present. d. Reschedule the visit for another time so that the mother and infant can be assessed privately. ANS: A The nurse should introduce herself to the patient and the other family members present. Family members in the home may be providing care and assistance to the mother and infant. However, this care may not be based on sound health practices. Nurses should take the opportunity to dispel myths while family members are present. The responsibility of the home care maternal-child nurse is to provide care to the new after birth mother and her infant, not to all family members. The nurse can politely ask about the other people in the home and their relationships with the woman. UnNleUssRSaInNiGnTdBic.CatOioMn is given that the woman would prefer privacy, the visit may continue. PTS: 1 DIF: Cognitive Level: Analysis OBJ: Nursing Process: Assessment MSC: Client Needs: Psychosocial Integrity 8. The nurse should be aware that during the childbearing experience an African-American woman is most likely to: a. seek prenatal care early in her pregnancy. b. avoid self-treatment of pregnancy-related discomfort. c. request liver in the after birth period to prevent anemia. d. arrive at the hospital in advanced labor. ANS: D African-American women often arrive at the hospital in far-advanced labor. These women may view pregnancy as a state of wellness, which is often the reason for delay in seeking prenatal care. African-American women practice many self-treatment options for various discomforts of pregnancy, and they may request liver in the after birth period, but this is based on a belief that the liver has a high blood content. PTS: 1 DIF: Cognitive Level: Comprehension OBJ: Nursing Process: Assessment MSC: Client Needs: Health Promotion and Maintenance 9. To provide competent care to an Asian-American family, the nurse should include which of the following questions during the assessment interview? a. “Do you prefer hot or cold beverages?” b. “Do you want milk to drink?” c. “Do you want music playing while you are in labor?” d. “Do you have a name selected for the baby?” ANS: A Asian-Americans often prefer warm beverages. Milk usually is excluded from the diet of this population. Asian-American women typically labor in a quiet atmosphere. Delaying naming the child is common for Asian-American families. PTS: 1 DIF: Cognitive Level: Application OBJ: Nursing Process: Assessment MSC: Client Needs: Physiologic Integrity 10. The patient's family is important to the maternity nurse because: a. they pay the bills. b. the nurse will know which family member to avoid. c. the nurse will know which mothers will really care for their children. d. the family culture and structure will influence nursing care decisions. ANS: D Family structure and culture influence the health decisions of mothers. PTS: 1 DIF: Cognitive Level: Comprehension OBJ: Nursing Process: Planning MSC: Client Needs: Psychosocial Integrity 11. A mother's household consists of her husband, his mother, and another child. She is living in a(n): a. extended family. b. single-parent family. c. married-blended family. d. nuclear family. ANS: A NURSINGTB.COM An extended family includes blood relatives living with the nuclear family. Both parents and a grandparent are living in this extended family. Single-parent families comprise an unmarried biologic or adoptive parent who may or may not be living with other adults. Married-blended refers to families reconstructed after divorce. A nuclear family is where male and female partners and their children live as an independent unit. PTS: 1 DIF: Cognitive Level: Application OBJ: Nursing Process: Assessment MSC: Client Needs: Psychosocial Integrity 12. A traditional family structure in which male and female partners and their children live as an independent unit is known as a(n): a. extended family. b. binuclear family. c. nuclear family. d. blended family. ANS: C About two thirds of U.S. households meet the definition of a nuclear family. Extended families include additional blood relatives other than the parents. A binuclear family involves two households. A blended family is reconstructed after divorce and involves the merger of two families. PTS: 1 DIF: Cognitive Level: Knowledge OBJ: Nursing Process: Assessment MSC: Client Needs: Psychosocial Integrity 13. Which statement about family systems theory is inaccurate? a. A family system is part of a larger suprasystem. b. A family as a whole is equal to the sum of the individual members. c. A change in one family member affects all family members. d. The family is able to create a balance between change and stability. ANS: B A family as a whole is greater than the sum of its parts. The other statements are characteristics of a system that states that a family is greater than the sum of its parts. PTS: 1 DIF: Cognitive Level: Comprehension OBJ: Nursing Process: Assessment MSC: Client Needs: Psychosocial Integrity 14. A pictorial tool that can assist the nurse in assessing aspects of family life related to health care is the: a. genogram. b. family values construct. c. life cycle model. d. human development wheel. ANS: A NURSINGTB.COM A genogram depicts the relationships of family members over generations. PTS: 1 DIF: Cognitive Level: Knowledge OBJ: Nursing Process: Assessment MSC: Client Needs: Psychosocial Integrity 15. The process by which people retain some of their own culture while adopting the practices of the dominant society is known as: a. acculturation. b. assimilation. c. ethnocentrism. d. cultural relativism. ANS: A Acculturation is the process by which people retain some of their own culture while adopting the practices of the dominant society. This process takes place over the course of generations. Assimilation is a loss of cultural identity. Ethnocentrism is the belief in the superiority of one's own culture over the cultures of others. Cultural relativism recognizes the roles of different cultures. PTS: 1 DIF: Cognitive Level: Knowledge OBJ: Nursing Process: Planning MSC: Client Needs: Psychosocial Integrity 16. When attempting to communicate with a patient who speaks a different language, the nurse should: a. respond promptly and positively to project authority. b. never use a family member as an interpreter. c. talk to the interpreter to avoid confusing the patient. d. provide as much privacy as possible. ANS: D Providing privacy creates an atmosphere of respect and puts the patient at ease. The nurse should not rush to judgment and should make sure that he or she understands the patient's message clearly. In crisis situations, the nurse may need to use a family member or neighbor as a translator. The nurse should talk directly to the patient to create an atmosphere of respect. PTS: 1 DIF: Cognitive Level: Application OBJ: Nursing Process: Implementation MSC: Client Needs: Psychosocial Integrity 17. In which culture is the father more likely to be expected to participate in the labor and delivery? a. Asian-American b. African-American c. European-American d. Hispanic ANS: C European-Americans expect the father to take a more active role in the labor and delivery than the other cultures. PTS: 1 DIF: CognitiNveURLSevINelG: TCBo.mCOprMehension OBJ: Nursing Process: Implementation MSC: Client Needs: Psychosocial Integrity 18. Which statement about cultural competence is not accurate? a. Local health care workers and community advocates can help extend health care to underserved populations. b. Nursing care is delivered in the context of the patient's culture but not in the context of the nurse's culture. c. Nurses must develop an awareness of and sensitivity to various cultures. d. A culture's economic, religious, and political structures influence practices that affect childbearing. ANS: B The cultural context of the nurse also affects nursing care. The work of local health care workers and community advocates is part of cultural competence; the nurse's cultural context is also important. Developing sensitivity to various cultures is part of cultural competence, but the nurse's cultural context is also important. The impact of economic, religious, and political structures is part of cultural competence; the nurse's cultural context is also important. PTS: 1 DIF: Cognitive Level: Comprehension OBJ: Nursing Process: Planning MSC: Client Needs: Psychosocial Integrity MATCHING You are getting ready to participate in discharge teaching with a non–English-speaking new mother. The interpreter has arrived in the patient care unit to assist you in providing culturally competent care. In the correct order, from 1 through 6, number the steps that you would take to work with the interpreter. a. Introduce yourself to the interpreter and converse informally. b. Outline your statements and questions, listing the key pieces of information you need to know. c. Make sure the interpreter is comfortable with technical terms. d. Learn something about the culture of the patient. e. Make notes on what you learned for future reference. f. Stop every now and then and ask the interpreter “How is it going?” 1. Step One 2. Step Two 3. Step Three 4. Step Four 5. Step Five 6. Step Six 1. ANS: B PTS: 1 DIF: Cognitive Level: Application OBJ: Nursing Process: Implementation MSC: Client Needs: Health Promotion and Maintenance NOT: To work successfully with an interpreter, the nurse must organize her teaching into four categories. These include actions that are necessary before the interview, meeting with the interpreter, during the interview, and after the interview. The nurse must be sensitive to cultural and situational differences (e.g., a woman from the Middle East may not wish to have a male interpreter present). 2. ANS: D PTS: 1 DIF: Cognitive Level: Application OBJ: Nursing Process: ImplementaNtioUnRSINMGSTCB.:CCOlMient Needs: Health Promotion and Maintenance NOT: To work successfully with an interpreter, the nurse must organize her teaching into four categories. These include actions that are necessary before the interview, meeting with the interpreter, during the interview, and after the interview. The nurse must be sensitive to cultural and situational differences (e.g., a woman from the Middle East may not wish to have a male interpreter present). 3. ANS: A PTS: 1 DIF: Cognitive Level: Application OBJ: Nursing Process: Implementation MSC: Client Needs: Health Promotion and Maintenance NOT: To work successfully with an interpreter, the nurse must organize her teaching into four categories. These include actions that are necessary before the interview, meeting with the interpreter, during the interview, and after the interview. The nurse must be sensitive to cultural and situational differences (e.g., a woman from the Middle East may not wish to have a male interpreter present). 4. ANS: C PTS: 1 DIF: Cognitive Level: Application OBJ: Nursing Process: Implementation MSC: Client Needs: Health Promotion and Maintenance NOT: To work successfully with an interpreter, the nurse must organize her teaching into four categories. These include actions that are necessary before the interview, meeting with the interpreter, during the interview, and after the interview. The nurse must be sensitive to cultural and situational differences (e.g., a woman from the Middle East may not wish to have a male interpreter present). 5. ANS: F PTS: 1 DIF: Cognitive Level: Application OBJ: Nursing Process: Implementation MSC: Client Needs: Health Promotion and Maintenance NOT: To work successfully with an interpreter, the nurse must organize her teaching into four categories. These include actions that are necessary before the interview, meeting with the interpreter, during the interview, and after the interview. The nurse must be sensitive to cultural and situational differences (e.g., a woman from the Middle East may not wish to have a male interpreter present). 6. ANS: E PTS: 1 DIF: Cognitive Level: Application OBJ: Nursing Process: Implementation MSC: Client Needs: Health Promotion and Maintenance NOT: To work successfully with an interpreter, the nurse must organize her teaching into four categories. These include actions that are necessary before the interview, meeting with the interpreter, during the interview, and after the interview. The nurse must be sensitive to cultural and situational differences (e.g., a woman from the Middle East may not wish to have a male interpreter present). NURSINGTB.COM Chapter 03: Assessment and Health Promotion Perry: Maternal Child Nursing Care, 6th Edition MULTIPLE CHOICE 1. The two primary functions of the ovaries are: a. normal female development and sex hormone release. b. ovulation and internal pelvic support. c. sexual response and ovulation. d. ovulation and hormone production. ANS: D The two functions of the ovaries are ovulation and hormone production. The presence of ovaries does not guarantee normal female development. The ovaries produce estrogen, progesterone, and androgen. Ovulation is the release of a mature ovum from the ovary; the ovaries are not responsible for internal pelvic support. Sexual response is a feedback mechanism involving the hypothalamus, anterior pituitary gland, and the ovaries. Ovulation does occur in the ovaries. PTS: 1 DIF: Cognitive Level: Knowledge OBJ: Nursing Process: Assessment MSC: Client Needs: Health Promotion and Maintenance 2. The uterus is a muscular, pear-shaped organ that is responsible for: a. cyclic menstruation. b. sex hormone production. c. fertilization. d. sexual arousal. ANS: A NURSINGTB.COM The uterus is an organ for reception, implantation, retention, and nutrition of the fertilized ovum; it also is responsible for cyclic menstruation. Hormone production and fertilization occur in the ovaries. Sexual arousal is a feedback mechanism involving the hypothalamus, the pituitary gland, and the ovaries. PTS: 1 DIF: Cognitive Level: Knowledge OBJ: Nursing Process: Assessment MSC: Client Needs: Health Promotion and Maintenance 3. Unique muscle fibers make the uterine myometrium ideally suited for: a. menstruation. b. the birth process. c. ovulation. d. fertilization. ANS: B The myometrium is made up of layers of smooth muscles that extend in three directions. These muscles assist in the birth process by expelling the fetus, ligating blood vessels after birth, and controlling the opening of the cervical os. PTS: 1 DIF: Cognitive Level: Application OBJ: Nursing Process: Assessment MSC: Client Needs: Health Promotion and Maintenance 4. The hormone responsible for maturation of mammary gland tissue is: a. estrogen. b. testosterone. c. prolactin. d. progesterone. ANS: D Progesterone causes maturation of the mammary gland tissue, specifically acinar structures of the lobules. Estrogen increases the vascularity of the breast tissue. Testosterone has no bearing on breast development. Prolactin is produced after birth and released from the pituitary gland. It is produced in response to infant suckling and emptying of the breasts. PTS: 1 DIF: Cognitive Level: Knowledge OBJ: Nursing Process: Assessment MSC: Client Needs: Health Promotion and Maintenance 5. Because of the effect of cyclic ovarian changes on the breast, the best time for breast self-examination (BSE) is: a. 5 to 7 days after menses ceases. b. Day 1 of the endometrial cycle. c. mid-menstrual cycle. d. any time during a shower or bath. ANS: A The physiologic alterations in breast size and activity reach their minimal level about 5 to 7 days after menstruation stops. All women should perform BSE during this phase of the menstrual cycle. PTS: 1 DIF: CognitiNveURLSevINelG: TKBn.oCwOlMedge OBJ: Nursing Process: Planning MSC: Client Needs: Health Promotion and Maintenance 6. Menstruation is periodic uterine bleeding: a. that occurs every 28 days. b. in which the entire uterine lining is shed. c. that is regulated by ovarian hormones. d. that leads to fertilization. ANS: C Menstruation is periodic uterine bleeding that is controlled by a feedback system involving three cycles: endometrial, hypothalamic-pituitary, and ovarian. The average length of a menstrual cycle is 28 days, but variations are normal. During the endometrial cycle, the functional two thirds of the endometrium are shed. Lack of fertilization leads to menstruation. PTS: 1 DIF: Cognitive Level: Knowledge OBJ: Nursing Process: Assessment MSC: Client Needs: Health Promotion and Maintenance 7. Individual irregularities in the ovarian (menstrual) cycle are most often caused by: a. variations in the follicular (preovulatory) phase. b. an intact hypothalamic-pituitary feedback mechanism. c. a functioning corpus luteum. d. a prolonged ischemic phase. ANS: A Almost all variations in the length of the ovarian cycle are the result of variations in the length of the follicular phase. An intact hypothalamic-pituitary feedback mechanism is regular, not irregular. The luteal phase begins after ovulation. The corpus luteum depends on the ovulatory phase and fertilization. During the ischemic phase, the blood supply to the functional endometrium is blocked and necrosis develops. The functional layer separates from the basal layer, and menstrual bleeding begins. PTS: 1 DIF: Cognitive Level: Comprehension OBJ: Nursing Process: Assessment MSC: Client Needs: Health Promotion and Maintenance 8. Prostaglandins are produced in most organs of the body, including the uterus. Other source(s) of prostaglandins is/are: a. ovaries. b. breast milk. c. menstrual blood. d. the vagina. ANS: C Menstrual blood is a potent source of prostaglandins. Prostaglandins are produced in most organs of the body and in menstrual blood. The ovaries, breast milk, and vagina are neither organs nor a source of prostaglandins. PTS: 1 DIF: Cognitive Level: Knowledge OBJ: Nursing Process: Assessment MSC: Client Needs: Health Promotion and Maintenance 9. Physiologically, sexual response can be characterized by: a. coitus, masturbation, and fantasy. b. myotonia and vasocongestion.NURSINGTB.COM c. erection and orgasm. d. excitement, plateau, and orgasm. ANS: B Physiologically, according to Masters (1992), sexual response can be analyzed in terms of two processes: vasocongestion and myotonia. Coitus, masturbation, and fantasy are forms of stimulation for the physical manifestation of the sexual response. Erection and orgasm occur in two of the four phases of the sexual response cycle. Excitement, plateau, and orgasm are three of the four phases of the sexual response cycle. PTS: 1 DIF: Cognitive Level: Knowledge OBJ: Nursing Process: Assessment MSC: Client Needs: Health Promotion and Maintenance 10. The long-term treatment plan for an adolescent with an eating disorder focuses on: a. managing the effects of malnutrition. b. establishing sufficient caloric intake. c. improving family dynamics. d. restructuring perception of body image. ANS: D The treatment of eating disorders is initially focused on reestablishing physiologic homeostasis. Once body systems are stabilized, the next goal of treatment for eating disorders is maintaining adequate caloric intake. Although family therapy is indicated when dysfunctional family relationships exist, the primary focus of therapy for eating disorders is to help the adolescent cope with complex issues. The focus of treatment in individual therapy for an eating disorder involves restructuring cognitive perceptions about the individual's body image. PTS: 1 DIF: Cognitive Level: Application OBJ: Nursing Process: Implementation MSC: Client Needs: Psychosocial Integrity 11. The nurse guides a woman to the examination room and asks her to remove her clothes and put on an examination gown with the front open. The woman states, “I have special undergarments that I do not remove for religious reasons.” The most appropriate response from the nurse would be: a. “You can't have an examination without removing all your clothes.” b. “I'll ask the doctor to modify the examination.” c. “Tell me about your undergarments. I'll explain the examination procedure, and then we can discuss how you can have your examination comfortably.” d. “What? I've never heard of such a thing! That sounds different and strange.” ANS: C This statement reflects cultural competence by the nurse and shows respect for the woman's religious practices. The nurse must respect the rich and unique qualities that cultural diversity brings to individuals. In recognizing the value of these differences, the nurse can modify the plan of care to meet the needs of each woman. NURSINGTB.COM PTS: 1 DIF: Cognitive Level: Application OBJ: Nursing Process: Planning MSC: Client Needs: Psychosocial Integrity 12. A 62-year-old woman has not been to the clinic for an annual examination for 5 years. The recent death of her husband reminded her that she should come for a visit. Her family doctor has retired, and she is going to see the women's health nurse practitioner for her visit. To facilitate a positive health care experience, the nurse should: a. remind the woman that she is long overdue for her examination and that she should come in annually. b. listen carefully and allow extra time for this woman's health history interview. c. reassure the woman that a nurse practitioner is just as good as her old doctor. d. encourage the woman to talk about the death of her husband and her fears about her own death. ANS: B The nurse has an opportunity to use reflection and empathy while listening and to ensure open and caring communication. Scheduling a longer appointment time may be necessary because older women may have longer histories or may need to talk. A respectful and reassuring approach to caring for women older than age 50 can help ensure that they continue to seek health care. Reminding the woman about her overdue examination, reassuring the woman that she has a good practitioner, and encouraging conversation about the death of her husband and her own death are not the best approaches with women in this age-group. PTS: 1 DIF: Cognitive Level: Application OBJ: Nursing Process: Planning MSC: Client Needs: Psychosocial Integrity 13. During a health history interview, a woman states that she thinks that she has “bumps” on her labia. She also states that she is not sure how to check herself. The correct response would be to: a. reassure the woman that the examination will not reveal any problems. b. explain the process of vulvar self-examination to the woman and reassure her that she should become familiar with normal and abnormal findings during the examination. c. reassure the woman that “bumps” can be treated. d. reassure her that most women have “bumps” on their labia. ANS: B During the assessment and evaluation, the responsibility for self-care, health promotion, and enhancement of wellness is emphasized. The pelvic examination provides a good opportunity for the practitioner to emphasize the need for regular vulvar self-examination. Providing reassurance to the woman concerning the “bumps” would not be an accurate response. PTS: 1 DIF: Cognitive Level: Application OBJ: Nursing Process: Assessment MSC: Client Needs: Physiologic Integrity 14. A woman arrives at the clinic for her annual examination. She tells the nurse that she thinks she has a vaginal infection and has been using an over-the-counter cream for the past 2 days to treat it. The nurse's initial response should be to: a. inform the woman that vaginal creams may interfere with the Papanicolaou (Pap) test for which she is scheduled. b. reassure the woman that using vaginal cream is not a problem for the examination. c. ask the woman to describe theNsUyRmSpINtoGmTsBt.ChaOtMindicate to her that she has a vaginal infection. d. ask the woman to reschedule the appointment for the examination. ANS: C An important element of the history and physical examination is the patient's description of any symptoms she may be experiencing. Although vaginal creams may interfere with the Pap test, the best response is for the nurse to inquire about the symptoms the patient is experiencing. Women should not douche, use vaginal medications, or have sexual intercourse for 24 to 48 hours before obtaining a Pap test. Although the woman may need to reschedule a visit for her Pap test, her current symptoms should still be addressed. PTS: 1 DIF: Cognitive Level: Application OBJ: Nursing Process: Assessment MSC: Client Needs: Physiologic Integrity 15. The transition phase during which ovarian function and hormone production decline is called: a. the climacteric. b. menarche. c. menopause. d. puberty. ANS: A The climacteric is a transitional phase during which ovarian function and hormone production decline. Menarche is the term that denotes the first menstruation. Menopause refers only to the last menstrual period. Puberty is a broad term that denotes the entire transitional stage between childhood and sexual maturity. PTS: 1 DIF: Cognitive Level: Knowledge OBJ: Nursing Process: Assessment MSC: Client Needs: Physiologic Integrity 16. Which statement would indicate that the patient requires additional instruction about breast self-examination? a. “Yellow discharge from my nipple is normal if I am having my period.” b. “I should check my breasts at the same time each month, like after my period.” c. “I should also feel in my armpit area while performing my breast examination.” d. “I should check each breast in a set way, such as in a circular motion.” ANS: A Discharge from the nipples requires further examination from a health care provider. “I should check my breasts at the same time each month, like after my period,” “I should also feel in my armpit area while performing my breast examination,” and “I should check each breast in a set way, such as in a circular motion” all indicate successful learning. PTS: 1 DIF: Cognitive Level: Analysis OBJ: Nursing Process: Assessment MSC: Client Needs: Health Promotion and Maintenance 17. The female reproductive organ(s) responsible for cyclic menstruation is/are the: a. uterus. b. ovaries. c. vaginal vestibule. d. urethra. ANS: A NURSINGTB.COM The uterus is responsible for cyclic menstruation. It also houses and nourishes the fertilized ovum and the fetus. The ovaries are responsible for ovulation and production of estrogen; the uterus is responsible for cyclic menstruation. The vaginal vestibule is an external organ that has openings to the urethra and vagina; the uterus is responsible for cyclic menstruation. The urethra is not a reproductive organ, although it is found in the area. PTS: 1 DIF: Cognitive Level: Knowledge OBJ: Nursing Process: Assessment MSC: Client Needs: Health Promotion and Maintenance 18. The body part that both protects the pelvic structures and accommodates the growing fetus during pregnancy is the: a. perineum. b. bony pelvis. c. vaginal vestibule. d. fourchette. ANS: B The bony pelvis protects and accommodates the growing fetus. The perineum covers the pelvic structures. The vaginal vestibule contains openings to the urethra and vagina. The fourchette is formed by the labia minor. PTS: 1 DIF: Cognitive Level: Knowledge OBJ: Nursing Process: Assessment MSC: Client Needs: Health Promotion and Maintenance 19. A fully matured endometrium that has reached the thickness of heavy, soft velvet describes the phase of the endometrial cycle. a. menstrual b. proliferative c. secretory d. ischemic ANS: C The secretory phase extends from the day of ovulation to approximately 3 days before the next menstrual cycle. During this phase, the endometrium becomes fully mature. During the menstrual phase, the endometrium is being shed; the endometrium is fully mature again during the secretory phase. The proliferative phase is a period of rapid growth, but the endometrium becomes fully mature again during the secretory phase. During the ischemic phase, the blood supply is blocked, and necrosis develops. The endometrium is fully mature during the secretory phase. PTS: 1 DIF: Cognitive Level: Comprehension OBJ: Nursing Process: Assessment MSC: Client Needs: Health Promotion and Maintenance 20. The stimulated release of gonadotropin-releasing hormone and follicle-stimulating hormone is part of the: a. menstrual cycle. b. endometrial cycle. c. ovarian cycle. d. hypothalamic-pituitary cycle. NURSINGTB.COM ANS: D The menstrual, endometrial, and ovarian cycles are interconnected. However, the cyclic release of hormones is the function of the hypothalamus and pituitary glands. PTS: 1 DIF: Cognitive Level: Knowledge OBJ: Nursing Process: Assessment MSC: Client Needs: Health Promotion and Maintenance 21. Certain fatty acids classified as hormones that are found in many body tissues and that have roles in many reproductive functions are known as: a. gonadotropin-releasing hormone (GnRH). b. prostaglandins (PGs). c. follicle-stimulating hormone (FSH). d. luteinizing hormone (LH). ANS: B PGs affect smooth muscle contraction and changes in the cervix. GnRH, FSH, and LH are part of the hypothalamic-pituitary cycle, which responds to the rise and fall of estrogen and progesterone. PTS: 1 DIF: Cognitive Level: Knowledge OBJ: Nursing Process: Assessment MSC: Client Needs: Health Promotion and Maintenance 22. Which statement regarding female sexual response is inaccurate? a. Women and men are more alike than different in their physiologic response to sexual arousal and orgasm. b. Vasocongestion is the congestion of blood vessels. c. The orgasmic phase is the final state of the sexual response cycle. d. Facial grimaces and spasms of hands and feet are often part of arousal. ANS: C The final state of the sexual response cycle is the resolution phase after orgasm. Men and women are surprisingly alike. Vasocongestion causes vaginal lubrication and engorgement of the genitals. Arousal is characterized by increased muscular tension (myotonia). PTS: 1 DIF: Cognitive Level: Knowledge OBJ: Nursing Process: Assessment MSC: Client Needs: Health Promotion and Maintenance 23. As part of their participation in the gynecologic portion of the physical examination, nurses should: a. take a firm approach that encourages the patient to facilitate the examination by following the physician's instructions exactly. b. explain the procedure as it unfolds and continue to question the patient to get information in a timely manner. c. take the opportunity to explain that the trendy vulvar self-examination is only for women at risk for cancer. d. Help the woman relax through proper placement of her hands and proper breathing during the examination. ANS: D Breathing techniques are important relaxation techniques that can help the patient during the examination. The nurse should enNcoUuRrSaIgNeGtThBe .pCaOtMient to participate in an active partnership with the care provider. Explanations during the procedure are fine, but many women are uncomfortable answering questions in the exposed and awkward position of the examination. Vulvar self-examination on a regular basis should be encouraged and taught during the examination. PTS: 1 DIF: Cognitive Level: Application OBJ: Nursing Process: Implementation MSC: Client Needs: Safe and Effective Care Environment 24. During which phase of the cycle of violence does the batterer become contrite and remorseful? a. Battering phase b. Honeymoon phase c. Tension-building phase d. Increased drug-taking phase ANS: B During the tension-building phase, the batterer becomes increasingly hostile, swears, threatens, and throws things. This is followed by the battering phase where violence actually occurs, and the victim feels powerless. During the honeymoon phase, the victim of IPV wants to believe that the battering will never happen again, and the batterer will promise anything to get back into the home. Often the batterer increases the use of drugs during the tension-building phase. PTS: 1 DIF: Cognitive Level: Knowledge OBJ: Nursing Process: Assessment MSC: Client Needs: Psychosocial Integrity 25. A patient at 24 weeks of gestation says she has a glass of wine with dinner every evening. The nurse will counsel her to eliminate all alcohol intake because: a. a daily consumption of alcohol indicates a risk for alcoholism. b. she will be at risk for abusing other substances as well. c. the fetus is placed at risk for altered brain growth. d. the fetus is at risk for multiple organ anomalies. ANS: C There is no period during pregnancy when it is safe to consume alcohol. The documented effects of alcohol consumption during pregnancy include intellectual disability, learning disabilities, high activity level, and short attention span. The brain grows most rapidly in the third trimester and is vulnerable to alcohol exposure during this time. Abuse of other substances has not been linked to alcohol use. PTS: 1 DIF: Cognitive Level: Comprehension OBJ: Nursing Process: Implementation MSC: Client Needs: Psychosocial Integrity 26. As a powerful central nervous system stimulant, which of these substances can lead to miscarriage, preterm labor, placental separation (abruption), and stillbirth? a. Heroin b. Alcohol c. PCP d. Cocaine ANS: D NURSINGTB.COM Cocaine is a powerful CNS stimulant. Effects on pregnancy associated with cocaine use include abruptio placentae, preterm labor, precipitous birth, and stillbirth. Heroin is an opiate. Its use in pregnancy is associated with preeclampsia, intrauterine growth restriction, miscarriage, premature rupture of membranes, infections, breech presentation, and preterm labor. The most serious effect of alcohol use in pregnancy is fetal alcohol syndrome. The major concerns regarding PCP use in pregnant women are its association with polydrug abuse and the neurobehavioral effects on the neonate. PTS: 1 DIF: Cognitive Level: Comprehension OBJ: Nursing Process: Assessment MSC: Client Needs: Psychosocial Integrity 27. The microscopic examination of scrapings from the cervix, endocervix, or other mucous membranes to detect premalignant or malignant cells is called: a. bimanual palpation. b. rectovaginal palpation. c. a Papanicolaou (Pap) test. d. a four As procedure. ANS: C The Pap test is a microscopic examination for cancer that should be performed regularly, depending on the patient's age. Bimanual palpation is a physical examination of the vagina. Rectovaginal palpation is a physical examination performed through the rectum. The four As is an intervention procedure to help a patient stop smoking. PTS: 1 DIF: Cognitive Level: Knowledge OBJ: Nursing Process: Implementation MSC: Client Needs: Physiologic Integrity 28. As a girl progresses through development, she may be at risk for a number of age-related conditions. While preparing a 21-year-old patient for her first adult physical examination and Papanicolaou (Pap) test, the nurse is aware of excessiveness shyness. The young woman states that she will not remove her bra because, “There is something wrong with my breasts; one is way bigger.” What is the best response by the nurse in this situation? a. “Please reschedule your appointment until you are more prepared.” b. “It is okay; the provider will not do a breast examination.” c. “I will explain normal growth and breast development to you.” d. “That is unfortunate; this must be very stressful for you.” ANS: C During adolescence, one breast may grow faster than the other. Discussion regarding this aspect of growth and development with the patient will reassure her that there may be nothing wrong with her breasts. Young women usually enter the health system for screening (Pap tests begin at age 21 or 3 years after first sexual activity). Situations such as these can produce great stress for the young woman, and the nurse and health care provider should treat her carefully. Asking her to reschedule would likely result in the patient's not returning for her appointment at all. A breast examination at her age is part of the complete physical examination. Young women should be taught about normal breast development and begin doing breast self-examinations. Although the last response shows empathy on the part of the nurse and acknowledges the patient's stress, it does not correct the patient's deficient knowledge related to normal growth and development. NURSINGTB.COM PTS: 1 DIF: Cognitive Level: Application OBJ: Nursing Process: Diagnosis MSC: Client Needs: Health Promotion and Maintenance 29. Which statement by the patient indicates that she understands breast self-examination? a. “I will examine both breasts in two different positions.” b. “I will perform breast self-examination 1 week after my menstrual period starts.” c. “I will examine the outer upper area of the breast only.” d. “I will use the palm of the hand to perform the examination.” ANS: B The woman should examine her breasts when hormonal influences are at their lowest level. The patient should be instructed to use four positions: standing with arms at her sides, standing with arms raised above her head, standing with hands pressed against hips, and lying down. The entire breast needs to be examined, including the outer upper area. The patient should use the sensitive pads of the middle three fingers. PTS: 1 DIF: Cognitive Level: Analysis OBJ: Nursing Process: Evaluation MSC: Client Needs: Health Promotion and Maintenance 30. A pregnant woman who abuses cocaine admits to exchanging sex for her drug habit. This behavior places her at a greater risk for: a. depression of the central nervous system. b. hypotension and vasodilation. c. sexually transmitted diseases. d. postmature birth. ANS: C Sex acts exchanged for drugs places the woman at increased risk for sexually transmitted diseases because of multiple partners and lack of protection. Cocaine is a central nervous system stimulant that causes hypertension and vasoconstriction. Premature delivery of the infant is one of the most common problems associated with cocaine use during pregnancy. PTS: 1 DIF: Cognitive Level: Comprehension OBJ: Nursing Process: Assessment MSC: Client Needs: Health Promotion and Maintenance 31. A woman who is older than 35 years may have difficulty achieving pregnancy primarily because: a. personal risk behaviors influence fertility. b. she has used contraceptives for an extended time. c. her ovaries may be affected by the aging process. d. prepregnancy medical attention is lacking. ANS: C Once the mature woman decides to conceive, a delay in becoming pregnant may occur because of the normal aging of the ovaries. Older adults participate in fewer risk behaviors than younger adults. The past use of contraceptives is not the problem. Prepregnancy medical care is both available and encouraged. PTS: 1 DIF: Cognitive Level: Knowledge OBJ: Nursing Process: Assessment MSC: Client Needs: Physiologic Integrity 32. The most dangerous effect on the NfeUtuRsSIoNfGaTmB.oCtOhMer who smokes cigarettes while pregnant is: a. genetic changes and anomalies. b. extensive central nervous system damage. c. fetal addiction to the substance inhaled. d. intrauterine growth restriction. ANS: D The major consequences of smoking tobacco during pregnancy are low-birth-weight infants, prematurity, and increased perinatal loss. Cigarettes normally will not cause genetic changes or extensive central nervous system damage. Addiction to tobacco is not a usual concern related to the neonate. PTS: 1 DIF: Cognitive Level: Comprehension OBJ: Nursing Process: Assessment MSC: Client Needs: Health Promotion and Maintenance 33. Despite warnings, prenatal exposure to alcohol continues to exceed by far exposure to illicit drugs. A diagnosis of fetal alcohol syndrome (FAS) is made when there are visible markers in each of three categories. Which category is not associated with the diagnosis of FAS? a. Respiratory conditions b. Impaired growth c. CNS abnormality d. Craniofacial dysmorphologies ANS: A Respiratory difficulties are not a category of conditions that are related to FAS. Abnormalities related to FAS include organ deformities, genital malformations, and kidney and urinary defects. Impaired growth is a visible marker for FAS. CNS abnormalities with neurologic and intellectual impairments are categories used to assist in the diagnosis of FAS. An infant with FAS manifests at least two craniofacial abnormalities, such as microcephaly, short palpebral fissures, poorly developed philtrum, thin upper lip, or flattening of the maxilla. PTS: 1 DIF: Cognitive Level: Knowledge OBJ: Nursing Process: Assessment MSC: Client Needs: Psychosocial Integrity MATCHING To promote wellness and prevent illness throughout the life span, it is important for the nurse to be cognizant of immunization recommendations for women older than 18 years. Match each immunization with the correct schedule. a. Tetanus-diphtheria-pertussis (Tdap) b. Measles, mumps, rubella c. Herpes Zoster d. Hepatitis B e. Influenza f. Human papillomavirus (HPV) 1. Three injections for girls between the ages 9 to 26. 2. Primary series of three injections. 3. Annually. 4. Once and then a booster every 10 years. 5. One dose after age 65. 6. Once if born after 1956. NURSINGTB.COM 1. ANS: F PTS: 1 DIF: Cognitive Level: Application OBJ: Nursing Process: Implementation MSC: Client Needs: Health Promotion and Maintenance NOT: These guidelines are applicable to most women; however, health care providers individualize the timing of tests and immunizations for each woman. 2. ANS: D PTS: 1 DIF: Cognitive Level: Application OBJ: Nursing Process: Implementation MSC: Client Needs: Health Promotion and Maintenance NOT: These guidelines are applicable to most women; however, health care providers individualize the timing of tests and immunizations for each woman. 3. ANS: E PTS: 1 DIF: Cognitive Level: Application OBJ: Nursing Process: Implementation MSC: Client Needs: Health Promotion and Maintenance NOT: These guidelines are applicable to most women; however, health care providers individualize the timing of tests and immunizations for each woman. 4. ANS: A PTS: 1 DIF: Cognitive Level: Application OBJ: Nursing Process: Implementation MSC: Client Needs: Health Promotion and Maintenance NOT: These guidelines are applicable to most women; however, health care providers individualize the timing of tests and immunizations for each woman. 5. ANS: C PTS: 1 DIF: Cognitive Level: Application OBJ: Nursing Process: Implementation MSC: Client Needs: Health Promotion and Maintenance NOT: These guidelines are applicable to most women; however, health care providers individualize the timing of tests and immunizations for each woman. 6. ANS: B PTS: 1 DIF: Cognitive Level: Application OBJ: Nursing Process: Implementation MSC: Client Needs: Health Promotion and Maintenance NOT: These guidelines are applicable to most women; however, health care providers individualize the timing of tests and immunizations for each woman. NURSINGTB.COM Chapter 04: Reproductive System Concerns Perry: Maternal Child Nursing Care, 6th Edition MULTIPLE CHOICE 1. When assessing a patient for amenorrhea, the nurse should be aware that this is unlikely to be caused by: a. anatomic abnormalities. b. type 1 diabetes mellitus. c. lack of exercise. d. hysterectomy. ANS: C Lack of exercise is not a cause of amenorrhea. Strenuous exercise may cause amenorrhea. Anatomic abnormalities, type 1 diabetes mellitus, and hysterectomy all are possible causes of amenorrhea. PTS: 1 DIF: Cognitive Level: Comprehension OBJ: Nursing Process: Assessment MSC: Client Needs: Health Promotion and Maintenance 2. When a nurse is counseling a woman for primary dysmenorrhea, which nonpharmacologic intervention might be recommended? a. Increasing the intake of red meat and simple carbohydrates b. Reducing the intake of diuretic foods such as peaches and asparagus c. Temporarily substituting physical activity for a sedentary lifestyle d. Using a heating pad on the abdNoUmReSnINtGoTrBel.iCeOvMe cramping ANS: D Heat minimizes cramping by increasing vasodilation and muscle relaxation and minimizing uterine ischemia. Dietary changes such as eating less red meat may be recommended for women experiencing dysmenorrhea. Increasing the intake of diuretics, including natural diuretics such as asparagus, cranberry juice, peaches, parsley, and watermelon, may help ease the symptoms associated with dysmenorrhea. Exercise has been found to help relieve menstrual discomfort through increased vasodilation and subsequent decreased ischemia. PTS: 1 DIF: Cognitive Level: Analysis OBJ: Nursing Process: Planning MSC: Client Needs: Physiologic Integrity 3. Which symptom described by a patient is characteristic of premenstrual syndrome (PMS)? a. “I feel irritable and moody a week before my period is supposed to start.” b. “I have lower abdominal pain beginning the third day of my menstrual period.” c. “I have nausea and headaches after my period starts, and they last 2 to 3 days.” d. “I have abdominal bloating and breast pain after a couple days of my period.” ANS: A PMS is a cluster of physical, psychologic, and behavioral symptoms that begin in the luteal phase of the menstrual cycle and resolve within a couple of days of the onset of menses. Complaints of lower abdominal pain, nausea and headaches, and abdominal bloating all are associated with PMS. However, the timing reflected is inaccurate. PTS: 1 DIF: Cognitive Level: Application OBJ: Nursing Process: Assessment MSC: Client Needs: Psychosocial Integrity 4. A woman complains of severe abdominal and pelvic pain around the time of menstruation that has gotten worse over the last 5 years. She also complains of pain during intercourse and has tried unsuccessfully to get pregnant for the past 18 months. These symptoms are most likely related to: a. endometriosis. b. PMS. c. primary dysmenorrhea. d. secondary dysmenorrhea. ANS: A Symptoms of endometriosis can change over time and may not reflect the extent of the disease. Major symptoms include dysmenorrhea and deep pelvic dyspareunia (painful intercourse). Impaired fertility may result from adhesions caused by endometriosis. Although endometriosis may be associated with secondary dysmenorrhea, it is not a cause of primary dysmenorrhea or PMS. In addition, this woman is complaining of dyspareunia and infertility, which are associated with endometriosis not with PMS or primary or secondary dysmenorrhea. PTS: 1 DIF: Cognitive Level: Comprehension OBJ: Nursing Process: Assessment MSC: Client Needs: Health Promotion and Maintenance 5. Nafarelin is currently used as a treatment for mild-to-severe endometriosis. The nurse should tell a woman taking this medication that the drug: a. stimulates the secretion of gonadotropin-releasing hormone (GnRH), thereby stimulating ovarian activity. NURSINGTB.COM b. should be sprayed into one nostril every other day. c. should be injected into subcutaneous tissue bid. d. can cause her to experience some hot flashes and bone loss. ANS: D Nafarelin is a GnRH agonist, and its side effects are similar to effects of menopause. The hypoestrogenism effect results in hot flashes and bone loss. Nafarelin is a GnRH agonist that suppresses the secretion of GnRH and is administered twice daily by nasal spray. PTS: 1 DIF: Cognitive Level: Comprehension OBJ: Nursing Process: Planning MSC: Client Needs: Health Promotion and Maintenance 6. While interviewing a 31-year-old woman before her routine gynecologic examination, the nurse collects data about the patient's recent menstrual cycles. The nurse should collect additional information associated with which patient statement? a. The woman says her menstrual flow lasts 5 to 6 days. b. She describes her flow as very heavy. c. She reports that she has had a small amount of spotting midway between her periods for the past 2 months. d. She says the length of her menstrual cycle varies from 26 to 29 days. ANS: B Menorrhagia is defined as excessive menstrual bleeding, in either duration or amount. Heavy bleeding can have many causes. The amount of bleeding and its effect on daily activities should be evaluated. A menstrual flow lasting 5 to 6 days is a normal finding. Mittlestaining, a small amount of bleeding or spotting that occurs at the time of ovulation (14 days before onset of the next menses), is considered normal. During her reproductive years, a woman may have physiologic variations in her menstrual cycle. Variations in the length of a menstrual cycle are considered normal. PTS: 1 DIF: Cognitive Level: Comprehension OBJ: Nursing Process: Assessment MSC: Client Needs: Health Promotion and Maintenance 7. When evaluating a patient whose primary complaint is amenorrhea, the nurse must be aware that lack of menstruation is most often the result of: a. stress. b. excessive exercise. c. pregnancy. d. eating disorders. ANS: C Amenorrhea, or the absence of menstrual flow, is most often a result of pregnancy. Although stress, excessive exercise, and eating disorders all may be contributing factors, none is the most common factor associated with amenorrhea. PTS: 1 DIF: Cognitive Level: Knowledge OBJ: Nursing Process: Assessment, Diagnosis MSC: Client Needs: Health Promotion and Maintenance 8. A 36-year-old woman has been giNvUenRSaINdiGaTgBno.CsOisMof uterine fibroids. When planning care for this patient, the nurse should know that: a. fibroids are malignant tumors of the uterus that require radiation or chemotherapy. b. fibroids increase in size during the perimenopausal period. c. menorrhagia is a common finding. d. the woman is unlikely to become pregnant as long as the fibroids are in her uterus. ANS: C The major symptoms associated with fibroids are menorrhagia and the physical effects produced by large myomas. Fibroids are benign tumors of the smooth muscle of the uterus, and their etiology is unknown. Fibroids are estrogen sensitive and shrink as levels of estrogen decline. Fibroids occur in 25% of women of reproductive age and are seen in 2% of pregnant women. PTS: 1 DIF: Cognitive Level: Comprehension OBJ: Nursing Process: Planning MSC: Client Needs: Health Promotion and Maintenance 9. During her gynecologic checkup, a 17-year-old girl states that recently she has been experiencing cramping and pain during her menstrual periods. The nurse would document this complaint as: a. amenorrhea. b. dysmenorrhea. c. dyspareunia. d. premenstrual syndrome (PMS). ANS: B Dysmenorrhea is pain during or shortly before menstruation. Amenorrhea is the absence of menstrual flow. Dyspareunia is pain during intercourse. PMS is a cluster of physical, psychologic, and behavioral symptoms that begin in the luteal phase of the menstrual cycle and resolve within a couple of days of the onset of menses. PTS: 1 DIF: Cognitive Level: Knowledge OBJ: Nursing Process: Diagnosis MSC: Client Needs: Health Promotion and Maintenance 10. With regard to dysmenorrhea, nurses should be aware that: a. it is more common in older women. b. it is more common in leaner women who exercise strenuously. c. symptoms can begin at any point in the ovulatory cycle. d. pain usually occurs in the suprapubic area or lower abdomen. ANS: D Pain is described as sharp and cramping or sometimes as a dull ache. It may radiate to the lower back or upper thighs. Dysmenorrhea is more common in women 17 to 24 years old, women who smoke, and women who are obese. Symptoms begin with menstruation or sometimes a few hours before the onset of flow. PTS: 1 DIF: Cognitive Level: Knowledge OBJ: Nursing Process: Diagnosis MSC: Client Needs: Physiologic Integrity 11. Which statement concerning cyclic perimenstrual pain and discomfort (CPPD) is accurate? a. Premenstrual dysphoric disorder (PMDD) is a milder form of premenstrual syndrome (PMS) and more common in younger women. b. Secondary dysmenorrhea is mNoUreRiSnItNeGnsTeBa.CnOdMmedically significant than primary dysmenorrhea. c. Premenstrual syndrome is a complex, poorly understood condition that may include any of a hundred symptoms. d. The causes of PMS have been well established. ANS: C PMS may manifest with one or more of a hundred or so physical and psychologic symptoms. PMDD is a more severe variant of PMS. Secondary dysmenorrhea is characterized by more muted pain than that seen in primary dysmenorrhea; the medical treatment is much the same. The cause of PMS is unknown. It may be a collection of different problems. PTS: 1 DIF: Cognitive Level: Comprehension OBJ: Nursing Process: Diagnosis MSC: Client Needs: Health Promotion and Maintenance 12. With regard to endometriosis, nurses should be aware that: a. it is characterized by the presence and growth of endometrial tissue inside the uterus. b. it is found more often in African-American women than in white or Asian women. c. it may worsen with repeated cycles or remain asymptomatic and disappear after menopause. d. it is unlikely to affect sexual intercourse or fertility. ANS: C Symptoms vary among women, ranging from nonexistent to incapacitating. With endometriosis, the endometrial tissue is outside the uterus. Symptoms vary among women, ranging from nonexistent to incapacitating. Endometriosis is found equally in white and African-American women and is slightly more prevalent in Asian women. Women can experience painful intercourse and impaired fertility. PTS: 1 DIF: Cognitive Level: Knowledge OBJ: Nursing Process: Assessment MSC: Client Needs: Health Promotion and Maintenance 13. One of the alterations in cyclic bleeding that occurs between periods is called: a. oligomenorrhea. b. menorrhagia. c. leiomyoma. d. metrorrhagia. ANS: D Metrorrhagia is bleeding between periods. It can be caused by progestin injections and implants. Oligomenorrhea is infrequent or scanty menstruation. Menorrhagia is excessive menstruation. Leiomyoma is a common cause of excessive bleeding. PTS: 1 DIF: Cognitive Level: Knowledge OBJ: Nursing Process: Diagnosis MSC: Client Needs: Physiologic Integrity 14. As relates to dysfunctional uterine bleeding (DUB), the nurse should be aware that: a. it is most commonly caused by anovulation. b. it most often occurs in middle age. c. the diagnosis of DUB should be first considered for abnormal menstrual bleeding. d. the most effective medical treaNtUmReSnItNiGs TstBe.rCoOidMs. ANS: A Anovulation may occur because of hypothalamic dysfunction or polycystic ovary syndrome. DUB most often occurs when the menstrual cycle is being established or when it draws to a close at menopause. A diagnosis of DUB is made only after all other causes of abnormal menstrual bleeding have been ruled out. The most effective medical treatment is oral or intravenous estrogen. PTS: 1 DIF: Cognitive Level: Knowledge OBJ: Nursing Process: Diagnosis MSC: Client Needs: Health Promotion and Maintenance 15. Management of primary dysmenorrhea often requires a multifaceted approach. The nurse who provides care for a patient with this condition should be aware that the optimal pharmacologic therapy for pain relief is: a. acetaminophen. b. oral contraceptives (OCPs). c. nonsteroidal anti-inflammatory drugs (NSAIDs). d. aspirin. ANS: C NSAIDs are prostaglandin inhibitors and show the strongest research results for pain relief. Often if one NSAID is not effective, another one can provide relief. Approximately 80% of women find relief from NSAIDs. Preparations containing acetaminophen are less effective for dysmenorrhea because they lack the antiprostaglandin properties of NSAIDs. OCPs are a reasonable choice for women who also want birth control. The benefit of OCPs is the reduction of menstrual flow and irregularities. OCPs may be contraindicated for some women and have numerous potential side effects. NSAIDs are the drug of choice. If a woman is taking a NSAID, she should avoid taking aspirin. PTS: 1 DIF: Cognitive Level: Application OBJ: Nursing Process: Planning MSC: Client Needs: Physiologic Integrity 16. The two primary areas of risk for sexually transmitted infections (STIs) are: a. sexual orientation and socioeconomic status. b. age and educational level. c. large number of sexual partners and race. d. risky sexual behaviors and inadequate preventive health behaviors. ANS: D Risky sexual behaviors and inadequate preventive health behaviors put a person at risk for acquiring or transmitting an STI. Although low socioeconomic status may be a factor in avoiding purchasing barrier protection, sexual orientation does not put one at higher risk. Younger individuals and individuals with less education may be unaware of proper prevention techniques; however, these are not the primary areas of risk for STIs. Having a large number of sexual partners is a risk-taking behavior, but race does not increase the risk for STIs. PTS: 1 DIF: Cognitive Level: Knowledge OBJ: Nursing Process: AssessmentNURSINMGSTCB.:CCOlMient Needs: Health Promotion and Maintenance 17. When evaluating a patient for sexually transmitted infections (STIs), the nurse should be aware that the most common bacterial STI is: a. gonorrhea. b. syphilis. c. chlamydia. d. candidiasis. ANS: C Chlamydia is the most common and fastest spreading STI among American women, with an estimated 3 million new cases each year. Gonorrhea and syphilis are bacterial STIs, but they are not the most common ones among American women. Candidiasis is caused by a fungus, not by bacteria. PTS: 1 DIF: Cognitive Level: Knowledge OBJ: Nursing Process: Assessment MSC: Client Needs: Health Promotion and Maintenance 18. The viral sexually transmitted infection (STI) that affects most people in the United States today is: a. herpes simplex virus type 2 (HSV-2). b. human papillomavirus (HPV). c. human immunodeficiency virus (HIV). d. cytomegalovirus (CMV). ANS: B HPV infection is the most prevalent viral STI seen in ambulatory health care settings. HSV-2, HIV, and CMV all are viral STIs but are not the most prevalent viral STIs. PTS: 1 DIF: Cognitive Level: Knowledge OBJ: Nursing Process: Assessment MSC: Client Needs: Health Promotion and Maintenance 19. The U.S. Centers for Disease Control and Prevention (CDC) recommends that HPV be treated with patient-applied: a. miconazole ointment. b. topical podofilox 0.5% solution or gel. c. penicillin given intramuscularly for two doses. d. metronidazole by mouth. ANS: B Available treatments are imiquimod, podophyllin, and podofilox. Miconazole ointment is used to treat athlete's foot. Intramuscular penicillin is used to treat syphilis. Metronidazole is used to treat bacterial vaginosis. PTS: 1 DIF: Cognitive Level: Knowledge OBJ: Nursing Process: Implementation MSC: Client Needs: Physiologic Integrity 20. A woman has a thick, white, lumpy, cottage cheese–like discharge, with patches on her labia and in her vagina. She complains of intense pruritus. The nurse practitioner would order which preparation for treatment? a. Miconazole b. Tetracycline c. Clindamycin d. Acyclovir ANS: A NURSINGTB.COM Miconazole and clotrimazole are the drugs of choice to treat candidiasis. Tetracycline is used to treat syphilis. Clindamycin is used to treat bacterial vaginosis. Acyclovir is used to treat genital herpes. PTS: 1 DIF: Cognitive Level: Comprehension OBJ: Nursing Process: Implementation MSC: Client Needs: Physiologic Integrity 21. To detect human immunodeficiency virus (HIV), most laboratory tests focus on the: a. virus. b. HIV antibodies. c. CD4 counts. d. CD8 counts. ANS: B The screening tool used to detect HIV is the enzyme immunoassay, which tests for the presence of antibodies to the virus. CD4 counts are associated with the incidence of acquired immunodeficiency syndrome (AIDS) in HIV-infected individuals. PTS: 1 DIF: Cognitive Level: Knowledge OBJ: Nursing Process: Planning MSC: Client Needs: Physiologic Integrity 22. Care management of a woman diagnosed with acute pelvic inflammatory disease (PID) most likely would include: a. oral antiviral therapy. b. bed rest in a semi-Fowler position. c. antibiotic regimen continued until symptoms subside. d. frequent pelvic examination to monitor the progress of healing. ANS: B A woman with acute PID should be on bed rest in a semi-Fowler position. Broad-spectrum antibiotics are used. Antibiotics must be taken as prescribed, even if symptoms subside. Few pelvic examinations should be conducted during the acute phase of the disease. PTS: 1 DIF: Cognitive Level: Comprehension OBJ: Nursing Process: Planning MSC: Client Needs: Physiologic Integrity 23. On vaginal examination of a 30-year-old woman, the nurse documents the following findings: profuse, thin, grayish white vaginal discharge with a “fishy” odor; complaint of pruritus. On the basis of these findings, the nurse suspects that this woman has: a. bacterial vaginosis (BV). b. candidiasis. c. trichomoniasis. d. gonorrhea. ANS: A Most women with BV complain of a characteristic “fishy” odor. The discharge usually is profuse; thin; and white, gray, or milky in color. Some women also may have mild irritation or pruritus. The discharge associaNteUdRwSiItNhGcTaBn.dCiOdMiasis is thick, white, and lumpy and resembles cottage cheese. Trichomoniasis may be asymptomatic, but women commonly have a characteristic yellowish-to-greenish, frothy, mucopurulent, copious, and malodorous discharge. Women with gonorrhea are often asymptomatic. They may have a purulent endocervical discharge, but discharge usually is minimal or absent. PTS: 1 DIF: Cognitive Level: Comprehension OBJ: Nursing Process: Assessment MSC: Client Needs: Health Promotion and Maintenance 24. The recommended treatment for the prevention of human immunodeficiency virus (HIV) transmission to the fetus during pregnancy is: a. acyclovir. b. ofloxacin. c. podophyllin. d. zidovudine. ANS: D Perinatal transmission of HIV has decreased significantly in the past decade as a result of prophylactic administration of the antiretroviral drug zidovudine to pregnant women in the prenatal and perinatal periods. Acyclovir is an antiviral treatment for HSV. Ofloxacin is an antibacterial treatment for gonorrhea. Podophyllin is a solution used in the treatment of human papillomavirus. PTS: 1 DIF: Cognitive Level: Knowledge OBJ: Nursing Process: Planning, Implementation MSC: Client Needs: Health Promotion and Maintenance 25. Which viral sexually transmitted infection is characterized by a primary infection followed by recurrent episodes? a. Herpes simplex virus (HSV)-2 b. Human papillomavirus (HPV) c. Human immunodeficiency virus (HIV) d. Cytomegalovirus (CMV) ANS: A The initial HSV genital infection is characterized by multiple painful lesions, fever, chills, malaise, and severe dysuria; it may last 2 to 3 weeks. Recurrent episodes of HSV infection commonly have only local symptoms that usually are less severe than the symptoms of the initial infection. With HPV infection, lesions are a chronic problem. HIV is a retrovirus. Seroconversion to HIV positivity usually occurs within 6 to 12 weeks after the virus has entered the body. Severe depression of the cellular immune system associated with HIV infection characterizes acquired immunodeficiency syndrome (AIDS). AIDS has no cure. In most adults, the onset of CMV infection is uncertain and asymptomatic. However, the disease may become a chronic, persistent infection. PTS: 1 DIF: Cognitive Level: Comprehension OBJ: Nursing Process: Assessment MSC: Client Needs: Health Promotion and Maintenance 26. The nurse should know that once human immunodeficiency virus (HIV) enters the body, seroconversion to HIV positivity usually occurs within: a. 6 to 10 days. b. 2 to 4 weeks. c. 6 to 12 weeks. d. 6 months. ANS: C NURSINGTB.COM Seroconversion to HIV positivity usually occurs within 6 to 12 weeks after the virus has entered the body. PTS: 1 DIF: Cognitive Level: Knowledge OBJ: Nursing Process: Assessment MSC: Client Needs: Physiologic Integrity 27. A 25-year-old single woman comes to the gynecologist's office for a follow-up visit related to her abnormal Papanicolaou (Pap) smear. The test revealed that the patient has human papillomavirus (HPV). The patient asks, “What is that? Can you get rid of it?” Your best response is: a. “It's just a little lump on your cervix. We can freeze it off.” b. “HPV stands for ‘human papillomavirus.' It is a sexually transmitted infection (STI) that may lead to cervical cancer. There is no known cure but symptoms are treated.” c. “HPV is a type of early human immunodeficiency virus (HIV). You will die from this.” d. “You probably caught this from your current boyfriend. He should get tested for this.” ANS: B It is important to inform the patient about STIs and the risks involved with HPV. The health care team has a duty to provide proper information to the patient, including information related to STIs. HPV and HIV are both viruses that can be transmitted sexually, but they are not the same virus. The onset of HPV can be insidious. Often STIs go unnoticed. Abnormal bleeding frequently is the initial symptom. The patient may have had HPV before her current boyfriend. You cannot make any deductions from this limited information. PTS: 1 DIF: Cognitive Level: Analysis OBJ: Nursing Process: Planning MSC: Client Needs: Health Promotion and Maintenance 28. Which of the following statements about the various forms of hepatitis is accurate? a. A vaccine exists for hepatitis C but not for hepatitis B. b. Hepatitis A is acquired by eating contaminated food or drinking polluted water. c. Hepatitis B is less contagious than human immunodeficiency virus (HIV). d. The incidence of hepatitis C is decreasing. ANS: B Contaminated milk and shellfish are common sources of infection with hepatitis A. A vaccine exists for hepatitis B but not for hepatitis C. Hepatitis B is more contagious than HIV. The incidence of hepatitis C is increasing. PTS: 1 DIF: Cognitive Level: Knowledge OBJ: Nursing Process: Assessment MSC: Client Needs: Physiologic Integrity 29. An essential component of counseling women regarding safe sex practices includes discussion regarding avoiding the exchange of body fluids. The physical barrier promoted for the prevention of sexually transmitted infections and human immunodeficiency virus is the condom. Nurses can help motivateNUpaRtSieINntGsTtBo.CuOseMcondoms by initiating a discussion related to a number of aspects of condom use. The most important of these is: a. strategies to enhance condom use. b. choice of colors and special features. c. leaving the decision up to the male partner. d. places to carry condoms safely. ANS: A When the nurse opens discussion on safe sex practices, it gives the woman permission to clear up any concerns or misapprehensions that she may have regarding condom use. The nurse can also suggest ways that the woman can enhance her condom negotiation and communications skills. These include role playing, rehearsal, cultural barriers, and situations that put the patient at risk. Although women can be taught the differences among condoms, such as size ranges, where to purchase, and price, this is not as important as negotiating the use of safe sex practices. Women must address the issue of condom use with every sexual contact. Some men need time to think about this. If they appear reluctant, the woman may want to reconsider the relationship. Although not ideal, women may safely choose to carry condoms in shoes, wallets, or inside their bra. They should be taught to keep the condom away from heat. This information is important; however, it is not germane if the woman cannot even discuss strategies on how to enhance condom use. PTS: 1 DIF: Cognitive Level: Analysis OBJ: Nursing Process: Planning MSC: Client Needs: Health Promotion and Maintenance 30. The nurse who is teaching a group of women about breast cancer would tell the women that: a. risk factors identify more than 50% of women who will develop breast cancer. b. nearly 90% of lumps found by women are malignant. c. 1 in 10 women in the United States will develop breast cancer in her lifetime. d. the exact cause of breast cancer is unknown. ANS: D The exact cause of breast cancer is unknown. Risk factors help to identify less than 30% of women in whom breast cancer eventually will develop. Women detect about 90% of all breast lumps. Of this 90%, only 20% to 25% are malignant. One in eight women in the United States will develop breast cancer in her lifetime. PTS: 1 DIF: Cognitive Level: Comprehension OBJ: Nursing Process: Assessment MSC: Client Needs: Health Promotion and Maintenance 31. Which diagnostic test is used to confirm a suspected diagnosis of breast cancer? a. Mammogram b. Ultrasound c. Needle aspiration d. CA 15.3 ANS: C When a suspicious mammogram is noted or a lump is detected, diagnosis is confirmed by needle aspiration, core needle biopsy, or needle localization biopsy. Mammography is a clinical screening tool that may aid early detection of breast cancers. Transillumination, thermography, and ultrasound breast imaging are being explored as methods of detecting early breast carcinoma. CA 15.3 is a serum tumor marker that is used to test for residual disease. NURSINGTB.COM PTS: 1 DIF: Cognitive Level: Knowledge OBJ: Nursing Process: Diagnosis MSC: Client Needs: Physiologic Integrity 32. After a mastectomy, a woman should be instructed to perform all of the following except: a. emptying surgical drains twice a day and as needed. b. avoiding lifting more than 4.5 kg (10 lbs) or reaching above her head until given permission by her surgeon. c. wearing clothing with snug sleeves to support the tissue of the arm on the operative side. d. reporting immediately if inflammation develops at the incision site or in the affected arm. ANS: C The woman should not be advised to wear snug clothing. Rather, she should be advised to avoid tight clothing, tight jewelry, and other causes of decreased circulation in the affected arm. As part of the teaching plan, the woman should be instructed to empty surgical drains, to avoid lifting more than 10 lbs or reaching above her head, and to report the development of incision site inflammation. PTS: 1 DIF: Cognitive Level: Application OBJ: Nursing Process: Planning MSC: Client Needs: Physiologic Integrity 33. A nurse practitioner performs a clinical breast examination on a woman diagnosed with fibroadenoma. The nurse knows that fibroadenoma is characterized by: a. inflammation of the milk ducts and glands behind the nipples. b. thick, sticky discharge from the nipple of the affected breast. c. lumpiness in both breasts that develops 1 week before menstruation. d. a single lump in one breast that can be expected to shrink as the woman ages. ANS: D Fibroadenomas are characterized by discrete, usually solitary lumps smaller than 3 cm in diameter. Fibroadenomas increase in size during pregnancy and shrink as the woman ages. Inflammation of the milk ducts is associated with mammary duct ectasia, not fibroadenoma. A thick, sticky discharge is associated with galactorrhea, not fibroadenoma. Lumpiness before menstruation is associated with fibrocystic changes of the breast. PTS: 1 DIF: Cognitive Level: Knowledge OBJ: Nursing Process: Assessment MSC: Client Needs: Physiologic Integrity 34. What important, immediate postoperative care practice should the nurse remember when caring for a woman who has had a mastectomy? a. The blood pressure (BP) cuff should not be applied to the affected arm. b. Venipuncture for blood work should be performed on the affected arm. c. The affected arm should be used for intravenous (IV) therapy. d. The affected arm should be held down close to the woman's side. ANS: A The affected arm should not be used for BP readings, IV therapy, or venipuncture. The affected arm should be elevated with pillows above the level of the right atrium. PTS: 1 DIF: Cognitive Level: Application OBJ: Nursing Process: ImplementaNtioUnRSINMGSTCB.:CCOlMient Needs: Physiologic Integrity 35. A woman has a breast mass that is not well delineated and is nonpalpable, immobile, and nontender. This is most likely: a. fibroadenoma. b. lipoma. c. intraductal papilloma. d. mammary duct ectasia. ANS: C Intraductal papilloma is the only benign breast mass that is nonpalpable. Fibroadenoma is well delineated, palpable, and movable. Lipoma is palpable and movable. Mammary duct ectasia is not well delineated and is immobile, but it is palpable and painful. PTS: 1 DIF: Cognitive Level: Application OBJ: Nursing Process: Diagnosis MSC: Client Needs: Physiologic Integrity 36. Fibrocystic changes in the breast most often appear in women in their 20s and 30s. The etiology is unknown but it may be an imbalance of estrogen and progesterone. The nurse who cares for this patient should be aware that treatment modalities are conservative. One proven modality that may provide relief is: a. diuretic administration. b. including caffeine daily in the diet. c. increased vitamin C supplementation. d. application of cold packs to the breast as necessary. ANS: A Diuretic administration plus a decrease in sodium and fluid intake are recommended. Although not supported by research, some advocate eliminating dimethylxanthines (caffeine) from the diet. Smoking should also be avoided, and alcohol consumption should be reduced. Vitamin E supplements are recommended; however, the patient should avoid megadoses because this is a fat-soluble vitamin. Pain-relief measures include applying heat to the breast, wearing a supportive bra, and taking nonsteroidal anti-inflammatory drugs. PTS: 1 DIF: Cognitive Level: Application OBJ: Nursing Process: Planning MSC: Client Needs: Physiologic Integrity 37. The nurse providing care in a women's health care setting must be aware regarding which sexually transmitted infection that can be successfully treated and cured? a. Herpes b. Acquired immunodeficiency syndrome (AIDS) c. Venereal warts d. Chlamydia ANS: D The usual treatment for infection by the bacterium Chlamydia is doxycycline or azithromycin. Concurrent treatment of all sexual partners is needed to prevent recurrence. There is no known cure for herpes, and treatment focuses on pain relief and preventing secondary infections. Because there is no known cure for AIDS, prevention and early detection are the primary focus of care management. Condylomata acuminata are caused by human papillomavirus. No treatment eradicates the virus. PTS: 1 DIF: CognitiNveURLSevINelG: TKBn.oCwOlMedge OBJ: Nursing Process: Assessment MSC: Client Needs: Physiologic Integrity 38. Which patient is most at risk for fibroadenoma of the breast? a. A 38-year-old woman b. A 50-year-old woman c. A 16-year-old girl d. A 27-year-old woman ANS: C Although it may occur at any age, fibroadenoma is most common in the teenage years. Ductal ectasia and intraductal papilloma become more common as a woman approaches menopause. Fibrocystic breast changes are more common during the reproductive years. PTS: 1 DIF: Cognitive Level: Comprehension OBJ: Nursing Process: Assessment MSC: Client Needs: Physiologic Integrity 39. The drug of choice for treatment of gonorrhea is: a. penicillin G. b. tetracycline. c. ceftriaxone. d. acyclovir. ANS: C Ceftriaxone is effective for treatment of all gonococcal infections. Penicillin is used to treat syphilis. Tetracycline is prescribed for chlamydial infections. Acyclovir is used to treat herpes genitalis. PTS: 1 DIF: Cognitive Level: Knowledge OBJ: Nursing Process: Planning MSC: Client Needs: Physiologic Integrity 40. The nurse providing education regarding breast care should explain to the woman that fibrocystic changes in breasts are: a. a disease of the milk ducts and glands in the breasts. b. a premalignant disorder characterized by lumps found in the breast tissue. c. lumpiness with or without tenderness found in varying degrees in the breast tissue of healthy women during menstrual cycles. d. lumpiness accompanied by tenderness after menses. ANS: C Fibrocystic changes are palpable thickenings in the breast usually associated with or without tenderness. The pain and tenderness fluctuate with the menstrual cycle. Fibrocystic changes are not premalignant changes. PTS: 1 DIF: Cognitive Level: Knowledge OBJ: Nursing Process: Assessment MSC: Client Needs: Health Promotion and Maintenance MULTIPLE RESPONSE 1. There is little consensus on the management of premenstrual dysphoric disorder (PMDD). However, nurses can advise womeNnUoRnSIsNeGvTerBa.lCsOeMlf-help modalities that often improve symptoms. The nurse knows that health teaching has been effective when the patient reports that she has adopted a number of lifestyle changes, including: (Select all that apply.) a. regular exercise. b. improved nutrition. c. a daily glass of wine. d. smoking cessation. e. oil of evening primrose. ANS: A, B, D, E These modalities may provide significant symptom relief in 1 to 2 months. If there is no improvement after these changes have been made, the patient may need to begin pharmacologic therapy. Women should decrease both their alcohol and caffeinated beverage consumption if they have PMDD. PTS: 1 DIF: Cognitive Level: Analysis OBJ: Nursing Process: Evaluation MSC: Client Needs: Physiologic Integrity 2. Examples of sexual risk behaviors associated with exposure to a sexually transmitted infection (STI) include: (Select all that apply.) a. oral sex. b. unprotected anal intercourse. c. multiple sex partners. d. dry kissing. e. abstinence. ANS: A, B, C Engaging in these sexual activities increases the exposure risk and the possibility of acquiring an STI. Dry kissing and abstinence are considered “safe” sexual practices. PTS: 1 DIF: Cognitive Level: Comprehension OBJ: Nursing Process: Assessment MSC: Client Needs: Health Promotion and Maintenance 3. The exact cause of breast cancer remains undetermined. Researchers have found that there are many common risk factors that increase a woman's chance of developing a malignancy. It is essential for the nurse who provides care to women of any age to be aware of which of the following risk factors? (Select all that apply.) a. Family history b. Late menarche c. Early menopause d. Race e. Nulliparity or first pregnancy after age 30 ANS: A, D, E Family history, race, and nulliparity are known risk factors for the development of breast cancer. Other risk factors include age, personal history of cancer, high socioeconomic status, sedentary lifestyle, hormone replacement therapy, recent use of oral contraceptives, never having breastfed a child, and drinking more than one alcoholic beverage per day. Early menarche and late menopause are risk factors for breast malignancy, not late menarche and early menopause. PTS: 1 DIF: Cognitive Level: Comprehension OBJ: Nursing Process: AssessmentNURSINMGSTCB.:CCOlMient Needs: Health Promotion and Maintenance MATCHING Herbal preparations have long been used for the management of menstrual problems, including dysmenorrhea, cramping and discomfort, and breast pain. For the nurse to counsel adequately the patient who elects to use this alternative modality, it is important to understand the action of these herbal preparations. Match the herbal medicine with the appropriate action. a. Uterine antispasmodic b. Uterotonic c. Anti-inflammatory d. Estrogen-like luteinizing hormone suppressant e. Decreases prolactin levels 1. Fennel, dong quai 2. Chaste tree fruit 3. Black cohosh 4. Valerian, wild yam 5. Ginger 1. ANS: B PTS: 1 DIF: Cognitive Level: Comprehension OBJ: Nursing Process: Implementation MSC: Client Needs: Health Promotion and Maintenance NOT: Herbal medicines may be valuable in treating dysmenorrhea; however, it is essential for women to understand that these therapies are not without potential toxicity and may cause drug interactions. 2. ANS: E PTS: 1 DIF: Cognitive Level: Comprehension OBJ: Nursing Process: Implementation MSC: Client Needs: Health Promotion and Maintenance NOT: Herbal medicines may be valuable in treating dysmenorrhea; however, it is essential for women to understand that these therapies are not without potential toxicity and may cause drug interactions. 3. ANS: D PTS: 1 DIF: Cognitive Level: Comprehension OBJ: Nursing Process: Implementation MSC: Client Needs: Health Promotion and Maintenance NOT: Herbal medicines may be valuable in treating dysmenorrhea; however, it is essential for women to understand that these therapies are not without potential toxicity and may cause drug interactions. 4. ANS: A PTS: 1 DIF: Cognitive Level: Comprehension OBJ: Nursing Process: Implementation MSC: Client Needs: Health Promotion and Maintenance NOT: Herbal medicines may be valuable in treating dysmenorrhea; however, it is essential for women to understand that these therapies are not without potential toxicity and may cause drug interactions. 5. ANS: C PTS: 1 DIF: Cognitive Level: Comprehension OBJ: Nursing Process: Implementation MSC: Client Needs: Health Promotion and Maintenance NOT: Herbal medicines may be valuable in treating dysmenorrhea; however, it is essential for women to understand that these therapies are not without potential toxicity and may cause drug interactions. NURSINGTB.COM Chapter 05: Infertility, Contraception, and Abortion Perry: Maternal Child Nursing Care, 6th Edition MULTIPLE CHOICE 1. A man smokes two packs of cigarettes a day. He wants to know if smoking is contributing to the difficulty he and his wife are having getting pregnant. The nurse's most appropriate response is: a. “Your sperm count seems to be okay in the first semen analysis.” b. “Only marijuana cigarettes affect sperm count.” c. “Smoking can give you lung cancer, even though it has no effect on sperm.” d. “Smoking can reduce the quality of your sperm.” ANS: D Use of tobacco, alcohol, and marijuana may affect sperm counts. “Your sperm count seems to be okay in the first semen analysis” is inaccurate. Sperm counts vary from day to day and depend on emotional and physical status and sexual activity. A single analysis may be inconclusive. A minimum of two analyses must be performed several weeks apart to assess male fertility. PTS: 1 DIF: Cognitive Level: Application OBJ: Nursing Process: Diagnosis MSC: Client Needs: Health Promotion and Maintenance 2. A couple comes in for an infertility workup, having attempted to get pregnant for 2 years. The woman, 37 years, has always had irregular menstrual cycles but is otherwise healthy. The man has fathered two children from a pNrUevRiSoIuNsGmTBar.CriOagMe and had a vasectomy reversal 2 years ago. The man has had two normal semen analyses, but the sperm seem to be clumped together. What additional test is needed? a. Testicular biopsy b. Antisperm antibodies c. Follicle-stimulating hormone (FSH) level d. Examination for testicular infection ANS: C The woman has irregular menstrual cycles. The scenario does not indicate that she has had any testing related to this irregularity. Hormone analysis is performed to assess endocrine function of the hypothalamic-pituitary-ovarian axis when menstrual cycles are absent or irregular. Determination of blood levels of prolactin, FSH, luteinizing hormone (LH), estradiol, progesterone, and thyroid hormones may be necessary to diagnose the cause of irregular menstrual cycles. A testicular biopsy would be indicated only in cases of azoospermia (no sperm cells) or severe oligospermia (low number of sperm cells). Antisperm antibodies are produced by a man against his own sperms. This is unlikely to be the case here because the man has already produced children. Examination for testicular infection would be done before semen analysis. Infection would affect spermatogenesis. PTS: 1 DIF: Cognitive Level: Analysis OBJ: Nursing Process: Diagnosis MSC: Client Needs: Health Promotion and Maintenance 3. A couple is trying to cope with an infertility problem. They want to know what they can do to preserve their emotional equilibrium. The nurse's most appropriate response is: a. “Tell your friends and family so they can help you.” b. “Talk only to other friends who are infertile because only they can help.” c. “Get involved with a support group. I'll give you some names.” d. “Start adoption proceedings immediately because it is very difficult to obtain an infant.” ANS: C Venting negative feelings may unburden the couple. A support group may provide a safe haven for the couple to share their experiences and gain insight from others' experiences. Although talking about their feelings may unburden them of negative feelings, infertility can be a major stressor that affects the couple's relationships with family and friends. Limiting their interactions to other infertile couples may be a beginning point for addressing psychosocial needs, but depending on where the other couple is in their own recovery process, this may or may not help them. The statement about adoption proceedings is not supportive of the psychosocial needs of this couple and may be detrimental to their well-being. PTS: 1 DIF: Cognitive Level: Application OBJ: Nursing Process: Assessment MSC: Client Needs: Psychosocial Integrity 4. A woman enquires about herbal alternative methods for improving fertility. Which statement by the nurse is the most appropriate when instructing the patient in which herbal preparations to avoid while trying to conceive? a. “You should avoid nettle leaf, dong quai, and vitamin E while you are trying to get pregnant.” b. “You may want to avoid licorice root, lavender, fennel, sage, and thyme while you are trying to conceive.” c. “You should not take anythingNwURitShINviGtaTmB.iCnOEM, calcium, or magnesium. They will make you infertile.” d. “Herbs have no bearing on fertility.” ANS: B Herbs that a woman should avoid while trying to conceive include licorice root, yarrow, wormwood, ephedra, fennel, golden seal, lavender, juniper, flaxseed, pennyroyal, passionflower, wild cherry, cascara, sage, thyme, and periwinkle. Nettle leaf, dong quai, and vitamin E all promote fertility. Vitamin E, calcium, and magnesium may promote fertility and conception. All supplements and herbs should be purchased from trusted sources. PTS: 1 DIF: Cognitive Level: Application OBJ: Nursing Process: Planning MSC: Client Needs: Health Promotion and Maintenance 5. In vitro fertilization-embryo transfer (IVF-ET) is a common approach for women with blocked fallopian tubes or unexplained infertility and for men with very low sperm counts. A husband and wife have arrived for their preprocedural interview. The husband asks the nurse to explain what the procedure entails. The nurse's most appropriate response is: a. “IVF-ET is a type of assisted reproductive therapy that involves collecting eggs from your wife's ovaries, fertilizing them in the laboratory with your sperm, and transferring the embryo to her uterus.” b. “A donor embryo will be transferred into your wife's uterus.” c. “Donor sperm will be used to inseminate your wife.” d. “Don't worry about the technical stuff; that's what we are here for.” ANS: A A woman's eggs are collected from her ovaries, fertilized in the laboratory with sperm, and transferred to her uterus after normal embryonic development has occurred. The statement, “A donor embryo will be transferred into your wife's uterus” describes therapeutic donor insemination. “Donor sperm will be used to inseminate your wife” describes the procedure for a donor embryo. “Don't worry about the technical stuff; that's what we are here for” discredits the patient's need for teaching and is an inappropriate response. PTS: 1 DIF: Cognitive Level: Application OBJ: Nursing Process: Planning MSC: Client Needs: Physiologic Integrity 6. With regard to the assessment of female, male, and couple infertility, nurses should be aware that: a. the couple's religious, cultural, and ethnic backgrounds provide emotional clutter that does not affect the clinical scientific diagnosis. b. the investigation takes 3 to 4 months and a significant financial investment. c. the woman is assessed first; if she is not the problem, the male partner is analyzed. d. semen analysis is for men; the postcoital test is for women. ANS: B Fertility assessment and diagnosis take time, money, and commitment from the couple. Religious, cultural, and ethnic-bred attitudes about fertility and related issues always have an impact on diagnosis and assessment. Both partners are assessed systematically and simultaneously, as individuals and as a couple. Semen analysis is for men, but the postcoital test is for the couple. PTS: 1 DIF: Cognitive Level: Application OBJ: Nursing Process: AssessmentNURSINMGSTCB.:CCOlMient Needs: Health Promotion and Maintenance 7. In their role of implementing a plan of care for infertile couples, nurses should: a. be comfortable with their sexuality and nonjudgmental about others to counsel their patients effectively. b. know about such nonmedical remedies as diet, exercise, and stress management. c. be able to direct patients to sources of information about what herbs to take that might help and which ones to avoid. d. do all of the above plus be knowledgeable about potential drug and surgical remedies. ANS: D Nurses should be open to and ready to help with a variety of medical and nonmedical approaches. PTS: 1 DIF: Cognitive Level: Comprehension OBJ: Nursing Process: Implementation MSC: Client Needs: Health Promotion and Maintenance 8. Although remarkable developments have occurred in reproductive medicine, assisted reproductive therapies are associated with numerous legal and ethical issues. Nurses can provide accurate information about the risks and benefits of treatment alternatives so couples can make informed decisions about their choice of treatment. Which issue would not need to be addressed by an infertile couple before treatment? a. Risks of multiple gestation. b. Whether or how to disclose the facts of conception to offspring. c. Freezing embryos for later use. d. Financial ability to cover the cost of treatment. ANS: D Although the method of payment is important, obtaining this information is not the responsibility of the nurse. Many states have mandated some form of insurance to assist couples with coverage for infertility. Risk of multiple gestation is a risk of treatment of which the couple needs to be aware. To minimize the chance of multiple gestation, generally only three or fewer embryos are transferred. The couple should be informed that there may be a need for multifetal reduction. Nurses can provide anticipatory guidance on this matter. Depending on the therapy chosen, there may be a need for donor oocytes, sperm, embryos, or a surrogate mother. Couples who have excess embryos frozen for later transfer must be fully informed before consenting to the procedure. A decision must be made regarding the disposal of embryos in the event of death or divorce or if the couple no longer wants the embryos at a future time. PTS: 1 DIF: Cognitive Level: Application OBJ: Nursing Process: Implementation MSC: Client Needs: Safe and Effective Care Environment 9. A woman has chosen the calendar method of conception control. During the assessment process, it is most important that the nurse: a. obtain a history of menstrual cycle lengths for the past 6 months. b. determine the patient's weight gain and loss pattern for the previous year. c. examine skin pigmentation and hair texture for hormonal changes. d. explore the patient's previous NexUpReSrIiNenGcTeBs.CwOitMh conception control. ANS: A The calendar method of conception control is based on the number of days in each cycle, counting from the first day of menses. The fertile period is determined after the lengths of menstrual cycles have been accurately recorded for 6 months. Weight gain or loss may be partly related to hormonal fluctuations, but it has no bearing on use of the calendar method. Integumentary changes may be related to hormonal changes, but they are not indicators for use of the calendar method. Exploring previous experiences with conception control may demonstrate patient understanding and compliancy, but it is not the most important aspect to assess for discussion of the calendar method. PTS: 1 DIF: Cognitive Level: Analysis OBJ: Nursing Process: Assessment MSC: Client Needs: Health Promotion and Maintenance 10. A woman is using the basal body temperature (BBT) method of contraception. She calls the clinic and tells the nurse, “My period is due in a few days, and my temperature has not gone up.” The nurse's most appropriate response is: a. “This probably means that you're pregnant.” b. “Don't worry; it's probably nothing.” c. “Have you been sick this month?” d. “You probably didn't ovulate during this cycle.” ANS: D The absence of a temperature decrease most likely is the result of lack of ovulation. Pregnancy cannot occur without ovulation (which is being measured using the BBT method). A comment such as “Don't worry; it's probably nothing” discredits the patient's concerns. Illness would most likely cause an increase in BBT. PTS: 1 DIF: Cognitive Level: Application OBJ: Nursing Process: Assessment MSC: Client Needs: Health Promotion and Maintenance 11. A married couple is discussing alternatives for pregnancy prevention and has asked about fertility awareness methods (FAMs). The nurse's most appropriate reply is: a. “They're not very effective, and it's very likely you'll get pregnant.” b. “They can be effective for many couples, but they require motivation.” c. “These methods have a few advantages and several health risks.” d. “You would be much safer going on the pill and not having to worry.” ANS: B FAMs are effective with proper vigilance about ovulatory changes in the body and adherence to coitus intervals. They are effective if used correctly by a woman with a regular menstrual cycle. The typical failure rate for all FAMs is 25% during the first year of use. FAMs have no associated health risks. The use of birth control has associated health risks. In addition, taking a pill daily requires compliance on the patient's part. PTS: 1 DIF: Cognitive Level: Application OBJ: Nursing Process: Planning MSC: Client Needs: Health Promotion and Maintenance 12. A male patient asks the nurse why it is better to purchase condoms that are not lubricated with nonoxynol-9 (a common spermicide). The nurse's most appropriate response is: a. “The lubricant prevents vaginaNlUiRrrSitIaNtGioTnB.”.COM b. “It has also been linked to an increase in the transmission of human immunodeficiency virus.” c. “The additional lubrication improves sex.” d. “Nonoxynol-9 improves penile sensitivity.” ANS: B The statement “Nonoxynol-9 does not provide protection against sexually transmitted infections, as originally thought; it has also been linked to an increase in the transmission of human immunodeficiency virus and can cause genital lesions” is true. Nonoxynol-9 may cause vaginal irritation, has no effect on the quality of sexual activity, and has no effect on penile sensitivity. PTS: 1 DIF: Cognitive Level: Application OBJ: Nursing Process: Planning MSC: Client Needs: Health Promotion and Maintenance 13. A woman who has a seizure disorder and takes barbiturates and phenytoin sodium daily asks the nurse about the pill as a contraceptive choice. The nurse's most appropriate response would be: a. “This is a highly effective method, but it has some side effects.” b. “Your current medications will reduce the effectiveness of the pill.” c. “The pill will reduce the effectiveness of your seizure medication.” d. “This is a good choice for a woman of your age and personal history.” ANS: B Because the liver metabolizes oral contraceptives, their effectiveness is reduced when they are taken simultaneously with anticonvulsants. The statement “Your current medications will reduce the effectiveness of the pill” is true, but it is not the most appropriate response. The anticonvulsant will reduce the effectiveness of the pill, not the other way around. The statement “This is a good choice for a woman of your age and personal history” does not teach the patient that the effectiveness of the pill may be reduced because of her anticonvulsant therapy. PTS: 1 DIF: Cognitive Level: Application OBJ: Nursing Process: Planning MSC: Client Needs: Health Promotion and Maintenance 14. Injectable progestins (DMPA, Depo-Provera) are a good contraceptive choice for women who: a. want menstrual regularity and predictability. b. have a history of thrombotic problems or breast cancer. c. have difficulty remembering to take oral contraceptives daily. d. are homeless or mobile and rarely receive health care. ANS: C Advantages of DMPA include a contraceptive effectiveness comparable to that of combined oral contraceptives with the requirement of only four injections a year. Disadvantages of injectable progestins are prolonged amenorrhea and uterine bleeding. Use of injectable progestin carries an increased risk of venous thrombosis and thromboembolism. To be effective, DMPA injections must be administered every 11 to 13 weeks. Access to health care is necessary to prevent pregnancy or potential complications. PTS: 1 DIF: Cognitive Level: Comprehension OBJ: Nursing Process: AssessmentNURSINMGSTCB.:CCOlMient Needs: Health Promotion and Maintenance 15. A woman currently uses a diaphragm and spermicide for contraception. She asks the nurse what the major differences are between the cervical cap and the diaphragm. The nurse's most appropriate response is: a. “No spermicide is used with the cervical cap, so it's less messy.” b. “The diaphragm can be left in place longer after intercourse.” c. “Repeated intercourse with the diaphragm is more convenient.” d. “The cervical cap can safely be used for repeated acts of intercourse without adding more spermicide later.” ANS: D The cervical cap can be inserted hours before sexual intercourse without the need for additional spermicide later. No additional spermicide is required for repeated acts of intercourse. Spermicide should be used inside the cap as an additional chemical barrier. The cervical cap should remain in place for 6 hours after the last act of intercourse. Repeated intercourse with the cervical cap is more convenient because no additional spermicide is needed. PTS: 1 DIF: Cognitive Level: Application OBJ: Nursing Process: Planning MSC: Client Needs: Health Promotion and Maintenance 16. A woman was treated recently for toxic shock syndrome (TSS). She has intercourse occasionally and uses over-the-counter protection. On the basis of her history, what contraceptive method should she and her partner avoid? a. Cervical cap b. Condom c. Vaginal film d. Vaginal sheath ANS: A Women with a history of TSS should not use a cervical cap. Condoms, vaginal films, and vaginal sheaths are not contraindicated for a woman with a history of TSS. PTS: 1 DIF: Cognitive Level: Comprehension OBJ: Nursing Process: Planning MSC: Client Needs: Health Promotion and Maintenance 17. An unmarried young woman describes her sex life as “active” and involving “many” partners. She wants a contraceptive method that is reliable and does not interfere with sex. She requests an intrauterine device (IUD). The nurse's most appropriate response is: a. “The IUD does not interfere with sex.” b. “The risk of pelvic inflammatory disease (PID) will be higher for you.” c. “The IUD will protect you from sexually transmitted infections (STIs).” d. “Pregnancy rates are high with IUDs.” ANS: B Disadvantages of IUDs include an increased risk of PID in the first 20 days after insertion and the risks of bacterial vaginosis and uterine perforation. The IUD offers no protection against STIs or human immunodeficiency virus. Because this woman has multiple sex partners, she is at higher risk of developing a STIN. TUhReSIINUGDTBd.oCeOsMnot protect against infection, as does a barrier method. Although the statement “The IUD does not interfere with sex” may be correct, it is not the most appropriate response. The IUD offers no protection from STIs. The typical failure rate of the IUD in the first year of use is 0.8%. PTS: 1 DIF: Cognitive Level: Application OBJ: Nursing Process: Planning MSC: Client Needs: Health Promotion and Maintenance 18. A woman is 16 weeks pregnant and has elected to terminate her pregnancy. The nurse knows that the most common technique used for medical termination of a pregnancy in the second trimester is: a. dilation and evacuation (D&E). b. instillation of hypertonic saline into the uterine cavity. c. intravenous administration of Pitocin. d. vacuum aspiration. ANS: A The most common technique for medical termination of a pregnancy in the second trimester is D&E. It is usually performed between 13 and 16 weeks. Hypertonic solutions injected directly into the uterus account for less than 1% of all abortions because other methods are safer and easier to use. Intravenous administration of Pitocin is used to induce labor in a woman with a third-trimester fetal demise. Vacuum aspiration is used for abortions in the first trimester. PTS: 1 DIF: Cognitive Level: Comprehension OBJ: Nursing Process: Implementation MSC: Client Needs: Physiologic Integrity 19. A woman will be taking oral contraceptives using a 28-day pack. The nurse should advise this woman to protect against pregnancy by: a. limiting sexual contact for one cycle after starting the pill. b. using condoms and foam instead of the pill for as long as she takes an antibiotic. c. taking one pill at the same time every day. d. throwing away the pack and using a backup method if she misses two pills during Week 1 of her cycle. ANS: C To maintain adequate hormone levels for contraception and to enhance compliance, patients should take oral contraceptives at the same time each day. If contraceptives are to be started at any time other than during normal menses or within 3 weeks after birth or abortion, another method of contraception should be used through the first week to prevent the risk of pregnancy. Taken exactly as directed, oral contraceptives prevent ovulation, and pregnancy cannot occur. No strong pharmacokinetic evidence indicates a link between the use of broad-spectrum antibiotics and altered hormone levels in oral contraceptive users. If the patient misses two pills during Week 1, she should take two pills a day for 2 days, finish the package, and use a backup method the next 7 consecutive days. PTS: 1 DIF: Cognitive Level: Application OBJ: Nursing Process: Planning MSC: Client Needs: Health Promotion and Maintenance 20. A woman had unprotected intercourse 36 hours ago and is concerned that she may become pregnant because it is her “fertile” time. She asks the nurse about emergency contraception. The nurse tells her that: a. it is too late; she needed to begNiUnRtrSeIaNtGmTeBn.tCwOMithin 24 hours after intercourse. b. Preven, an emergency contraceptive method, is 98% effective at preventing pregnancy. c. an over-the-counter antiemetic can be taken 1 hour before each contraceptive dose to prevent nausea and vomiting. d. the most effective approach is to use a progestin-only preparation. ANS: C To minimize the side effect of nausea that occurs with high doses of estrogen and progestin, the woman can take an over-the-counter antiemetic 1 hour before each dose. Emergency contraception is used within 72 hours of unprotected intercourse to prevent pregnancy. Postcoital contraceptive use is 74% to 90% effective at preventing pregnancy. Oral emergency contraceptive regimens may include progestin-only and estrogen-progestin pills. Women with contraindications to estrogen use should use progestin-only pills. PTS: 1 DIF: Cognitive Level: Analysis OBJ: Nursing Process: Planning MSC: Client Needs: Health Promotion and Maintenance 21. Which statement is true about the term contraceptive failure rate? a. It refers to the percentage of users expected to have an accidental pregnancy over a 5-year span. b. It refers to the minimum level that must be achieved to receive a government license. c. It increases over time as couples become more careless. d. It varies from couple to couple, depending on the method and the users. ANS: D Contraceptive effectiveness varies from couple to couple, depending on how well a contraceptive method is used and how well it suits the couple. The contraceptive failure rate measures the likelihood of accidental pregnancy in the first year only. Failure rates decline over time because users gain experience. PTS: 1 DIF: Cognitive Level: Knowledge OBJ: Nursing Process: Assessment MSC: Client Needs: Health Promotion and Maintenance 22. While instructing a couple regarding birth control, the nurse should be aware that the method called natural family planning: a. is the same as coitus interruptus, or “pulling out.” b. uses the calendar method to align the woman's cycle with the natural phases of the moon. c. is the only contraceptive practice acceptable to the Roman Catholic Church. d. relies on barrier methods during fertility phases. ANS: C Natural family planning is another name for periodic abstinence, which is the accepted way to pass safely through the fertility phases without relying on chemical or physical barriers. Natural family planning is the only contraceptive practice acceptable to the Roman Catholic Church. “Pulling out” is not the same as periodic abstinence, another name for natural family planning. The phases of the moon are not part of the calendar method or any method. PTS: 1 DIF: Cognitive Level: Comprehension OBJ: Nursing Process: Planning MSC: Client Needs: Health PromotiNonUaRnSdINMGaTiBnt.eCnOaMnce 23. Which contraceptive method best protects against sexually transmitted infections (STIs) and human immunodeficiency virus (HIV)? a. Periodic abstinence b. Barrier methods c. Hormonal methods d. They all offer about the same protection. ANS: B Barrier methods such as condoms best protect against STIs and HIV. Periodic abstinence and hormonal methods (the pill) offer no protection against STIs or HIV. PTS: 1 DIF: Cognitive Level: Application OBJ: Nursing Process: Planning MSC: Client Needs: Health Promotion and Maintenance 24. With regard to the noncontraceptive medical effects of combined oral contraceptive pills (COCs), nurses should be aware that: a. COCs can cause toxic shock syndrome if the prescription is wrong. b. hormonal withdrawal bleeding usually is a bit more profuse than in normal menstruation and lasts a week. c. COCs increase the risk of endometrial and ovarian cancer. d. the effectiveness of COCs can be altered by some over-the-counter medications and herbal supplements. ANS: D The effectiveness of COCs can be altered by some over-the-counter medications and herbal supplements. Toxic shock syndrome can occur in some diaphragm users, but it is not a consequence of taking oral contraceptive pills. Hormonal withdrawal bleeding usually is lighter than in normal menstruation and lasts a couple of days. Oral contraceptive pills offer protection against the risk of endometrial and ovarian cancers. PTS: 1 DIF: Cognitive Level: Comprehension OBJ: Nursing Process: Planning MSC: Client Needs: Health Promotion and Maintenance 25. With regard to the use of intrauterine devices (IUDs), nurses should be aware that: a. return to fertility can take several weeks after the device is removed. b. IUDs containing copper can provide an emergency contraception option if inserted within a few days of unprotected intercourse. c. IUDs offer the same protection against sexually transmitted infections (STIs) as the diaphragm. d. consent forms are not needed for IUD insertion. ANS: B The woman has up to 5 days to insert the IUD after unprotected sex. Return to fertility is immediate after removal of the IUD. IUDs offer no protection for STIs. A consent form is required for insertion, as is a negative pregnancy test. PTS: 1 DIF: Cognitive Level: Comprehension OBJ: Nursing Process: Assessment MSC: Client Needs: Health Promotion and Maintenance 26. Which of the following statements is the most complete and accurate description of medical abortions? NURSINGTB.COM a. They are performed only for maternal health. b. They can be achieved through surgical procedures or with drugs. c. They are mostly performed in the second trimester. d. They can be either elective or therapeutic. ANS: D Medical abortions are performed through the use of medications (rather than surgical procedures). They are mostly done in the first trimester, and they can be either elective (the woman's choice) or therapeutic (for reasons of maternal or fetal health). PTS: 1 DIF: Cognitive Level: Comprehension OBJ: Nursing Process: Assessment MSC: Client Needs: Health Promotion and Maintenance 27. Postcoital contraception with Ella: a. requires that the first dose be taken within 72 hours of unprotected intercourse. b. requires that the woman take second and third doses at 24 and 36 hours after the first dose. c. must be taken in conjunction with an IUD insertion. d. is commonly associated with the side effect of menorrhagia. ANS: A Emergency contraception is most effective when used within 72 hours of intercourse; however, it may be used with lessened effectiveness 120 hours later. Insertion of the copper IUD within 5 days of intercourse may also be used and is up to 99% effective. The most common side effect of postcoital contraception is nausea. PTS: 1 DIF: Cognitive Level: Comprehension OBJ: Nursing Process: Implementation MSC: Client Needs: Health Promotion and Maintenance 28. Informed consent concerning contraceptive use is important because some of the methods: a. are invasive procedures that require hospitalization. b. require a surgical procedure to insert. c. may not be reliable. d. have potentially dangerous side effects. ANS: D To make an informed decision about the use of contraceptives, it is important for couples to be aware of potential side effects. The only contraceptive method that is a surgical procedure and requires hospitalization is sterilization. Some methods have greater efficacy than others, and this should be included in the teaching. PTS: 1 DIF: Cognitive Level: Comprehension OBJ: Nursing Process: Assessment MSC: Client Needs: Safe and Effective Care Environment 29. A physician prescribes clomiphene citrate (Clomid, Serophene) for a woman experiencing infertility. She is very concerned about the risk of multiple births. The nurse's most appropriate response is: a. “This is a legitimate concern. NWUoRuSlIdNyGoTuBl.CikOeMto discuss this further before your treatment begins?” b. “No one has ever had more than triplets with Clomid.” c. “Ovulation will be monitored with ultrasound so that this will not happen.” d. “Ten percent is a very low risk, so you don't need to worry too much.” ANS: A The incidence of multiple pregnancies with the use of these medications is significantly increased. The patient's concern is legitimate and should be discussed so that she can make an informed decision. Stating that no one has ever had “more than triplets” is inaccurate and negates the patient's concerns. Ultrasound cannot ensure that a multiple pregnancy will not occur. The percentage quoted in this statement is inaccurate. The comment “don't worry” discredits the patient's concern. PTS: 1 DIF: Cognitive Level: Application OBJ: Nursing Process: Planning MSC: Client Needs: Health Promotion and Maintenance MULTIPLE RESPONSE 1. You (the nurse) are reviewing the educational packet provided to a patient about tubal ligation. What is an important fact you should point out? (Select all that apply.) a. “It is highly unlikely that you will become pregnant after the procedure.” b. “This is an effective form of 100% permanent sterilization. You won't be able to get pregnant.” c. “Sterilization offers some form of protection against sexually transmitted infections (STIs).” d. “Sterilization offers no protection against STIs.” e. “Your menstrual cycle will greatly increase after your sterilization.” ANS: A, D A woman is unlikely to become pregnant after tubal ligation, although it is not 100% effective. Sterilization offers no protection against STIs. The menstrual cycle typically remains the same after a tubal ligation. PTS: 1 DIF: Cognitive Level: Application OBJ: Nursing Process: Implementation MSC: Client Needs: Health Promotion and Maintenance COMPLETION 1. Practice of the calendar rhythm method is based on the number of days in each menstrual cycle. The fertile period is determined after monitoring each cycle for 6 months. The beginning of the fertile period is estimated by subtracting 18 days from the longest cycle and 11 days from the shortest. If the woman's cycles vary in length from 24 to 30 days, what would her fertile period be? to ANS: Day 10 to day 19 To avoid pregnancy, the couple must abstain from intercourse on days 10 through 19. Ovulation occurs on day 14 (plus NorUmRSiInNuGs T2Bd.CaOysMeither way). PTS: 1 DIF: Cognitive Level: Comprehension OBJ: Nursing Process: Assessment MSC: Client Needs: Health Promotion and Maintenance MATCHING Evaluation for infertility should be offered to couples who have failed to become pregnant after 1 year of regular intercourse or after 6 months if the woman is older than 35 years. Impaired fertility in women may be the result of numerous factors. Careful identification of the cause of infertility assists in determining the correct treatment plan. The nurse who chooses to work in the specialty of infertility must have an excellent understanding of these factors and causes. Match each factor affecting female infertility with the likely cause. a. Ovarian b. Tubal/peritoneal c. Uterine d. Vaginal/cervical e. Other factors 1. Endometrial or myometrial tumors 2. Anorexia 3. Isoimmunization 4. Thyroid dysfunction or obesity 5. Endometriosis 1. ANS: C PTS: 1 DIF: Cognitive Level: Comprehension OBJ: Nursing Process: Assessment MSC: Client Needs: Health Promotion and Maintenance NOT: Ovarian factors include congenital anomalies, primary or secondary anovulation (anorexia), and medications. Tubal or peritoneal factors include congenital anomalies of the tubes, reduced tubal motility, inflammation, adhesions, and disruption owing to tubal pregnancy and endometriosis. Uterine factors include developmental anomalies of the uterus, endometrial and fibroid tumors, and Asherman's syndrome. Vaginal-cervical factors include vaginal-cervical infections, inadequate cervical mucus, and isoimmunization (development of sperm antibodies). Other factors may include nutritional deficiencies, obesity, thyroid dysfunction, and idiopathic conditions. 2. ANS: A PTS: 1 DIF: Cognitive Level: Comprehension OBJ: Nursing Process: Assessment MSC: Client Needs: Health Promotion and Maintenance NOT: Ovarian factors include congenital anomalies, primary or secondary anovulation (anorexia), and medications. Tubal or peritoneal factors include congenital anomalies of the tubes, reduced tubal motility, inflammation, adhesions, and disruption owing to tubal pregnancy and endometriosis. Uterine factors include developmental anomalies of the uterus, endometrial and fibroid tumors, and Asherman's syndrome. Vaginal-cervical factors include vaginal-cervical infections, inadequate cervical mucus, and isoimmunization (development of sperm antibodies). Other factors may include nutritional deficiencies, obesity, thyroid dysfunction, and idiopathic conditions. 3. ANS: D PTS: 1 DIF: Cognitive Level: Comprehension OBJ: Nursing Process: Assessment MSC: Client Needs: Health Promotion and Maintenance NOT: Ovarian factors include congenital anomalies, primary or secondary anovulation (anorexia), and medications. Tubal or peritoneal factors include congenital anomalies of the tubes, reduced tubal motility, inflammation, adhesions, and disruption owing to tubal pregnancy and endometriosis. Uterine factors include developmental anomalies of the uterus, endometrial and fibroid tumors, and Asherman's syndrome. Vaginal-cervical factors include vaginal-cervical infections, inadequate cervical mucus, and isoimmunization (development of sperm antibodies). Other factors may include nutritional deficiencies, obesity, thyroid dysfunction, and idiopathic conditions. 4. ANS: E PTS: 1 NURSINDGITFB: .CCOoMgnitive Level: Comprehension OBJ: Nursing Process: Assessment MSC: Client Needs: Health Promotion and Maintenance NOT: Ovarian factors include congenital anomalies, primary or secondary anovulation (anorexia), and medications. Tubal or peritoneal factors include congenital anomalies of the tubes, reduced tubal motility, inflammation, adhesions, and disruption owing to tubal pregnancy and endometriosis. Uterine factors include developmental anomalies of the uterus, endometrial and fibroid tumors, and Asherman's syndrome. Vaginal-cervical factors include vaginal-cervical infections, inadequate cervical mucus, and isoimmunization (development of sperm antibodies). Other factors may include nutritional deficiencies, obesity, thyroid dysfunction, and idiopathic conditions. 5. ANS: B PTS: 1 DIF: Cognitive Level: Comprehension OBJ: Nursing Process: Assessment MSC: Client Needs: Health Promotion and Maintenance NOT: Ovarian factors include congenital anomalies, primary or secondary anovulation (anorexia), and medications. Tubal or peritoneal factors include congenital anomalies of the tubes, reduced tubal motility, inflammation, adhesions, and disruption owing to tubal pregnancy and endometriosis. Uterine factors include developmental anomalies of the uterus, endometrial and fibroid tumors, and Asherman's syndrome. Vaginal-cervical factors include vaginal-cervical infections, inadequate cervical mucus, and isoimmunization (development of sperm antibodies). Other factors may include nutritional deficiencies, obesity, thyroid dysfunction, and idiopathic conditions. Chapter 06: Genetics, Conception, and Fetal Development Perry: Maternal Child Nursing Care, 6th Edition MULTIPLE CHOICE 1. A father and mother are carriers of phenylketonuria (PKU). Their 2-year-old daughter has PKU. The couple tells the nurse that they are planning to have a second baby. Because their daughter has PKU, they are sure that their next baby won't be affected. What response by the nurse is most accurate? a. “Good planning; you need to take advantage of the odds in your favor.” b. “I think you'd better check with your doctor first.” c. “You are both carriers, so each baby has a 25% chance of being affected.” d. “The ultrasound indicates a boy, and boys are not affected by PKU.” ANS: C The chance is one in four that each child produced by this couple will be affected by PKU disorder. This couple still has an increased likelihood of having a child with PKU. Having one child already with PKU does not guarantee that they will not have another. These parents need to discuss their options with their physician. However, an opportune time has presented itself for the couple to receive correct teaching about inherited genetic risks. No correlation exists between gender and inheritance of the disorder because PKU is an autosomal recessive disorder. PTS: 1 DIF: Cognitive Level: Application OBJ: Nursing Process: Planning MSC: Client Needs: Health Promotion and Maintenance NURSINGTB.COM 2. The nurse is providing genetic counseling for an expectant couple who already have a child with trisomy 18. The nurse should: a. tell the couple they need to have an abortion within 2 to 3 weeks. b. explain that the fetus has a 50% chance of having the disorder. c. discuss options with the couple, including amniocentesis to determine whether the fetus is affected. d. refer the couple to a psychologist for emotional support. ANS: C Genetic testing, including amniocentesis, would need to be performed to determine whether the fetus is affected. The couple should be given information about the likelihood of having another baby with this disorder so that they can make an informed decision. A genetic counselor is the best source for determining genetic probability ratios. The couple eventually may need emotional support, but the status of the pregnancy must be determined first. PTS: 1 DIF: Cognitive Level: Comprehension OBJ: Nursing Process: Planning, Implementation MSC: Client Needs: Physiologic Integrity 3. The nurse is assessing the knowledge of new parents with a child born with maple syrup urine disease (MSUD). This is an autosomal recessive inherited disorder, which means that: a. both genes of a pair must be abnormal for the disorder to be expressed. b. only one copy of the abnormal gene is required for the disorder to be expressed. c. the disorder occurs in males and heterozygous females. d. the disorder is carried on the X chromosome. ANS: A MSUD is a type of autosomal recessive inheritance disorder in which both genes of a pair must be abnormal for the disorder to be expressed. MSUD is not an X-linked dominant or recessive disorder or an autosomal dominant inheritance disorder. PTS: 1 DIF: Cognitive Level: Comprehension OBJ: Nursing Process: Assessment MSC: Client Needs: Health Promotion and Maintenance 4. In presenting to obstetric nurses interested in genetics, the genetic nurse identifies the primary risk(s) associated with genetic testing as: a. anxiety and altered family relationships. b. denial of insurance benefits. c. high false-positive results associated with genetic testing. d. ethnic and socioeconomic disparity associated with genetic testing. ANS: B Decisions about genetic testing are shaped by socioeconomic status and the ability to pay for the testing. Some types of genetic testing are expensive and are not covered by insurance benefits. Anxiety and altered family relationships, high false-positive results, and ethnic and socioeconomic disparity are factors that may be difficulties associated with genetic testing, but they are not risks associated with testing. PTS: 1 DIF: Cognitive Level: Comprehension OBJ: Nursing Process: Implementation MSC: Client Needs: Health Promotion and Maintenance 5. A man's wife is pregnant for the tNhiUrdRStiImNGeT. OB.nCeOcMhild was born with cystic fibrosis, and the other child is healthy. The man wonders what the chance is that this child will have cystic fibrosis. This type of testing is known as: a. occurrence risk. b. recurrence risk. c. predictive testing. d. predisposition testing. ANS: B The couple already has a child with a genetic disease so they will be given a recurrence risk test. If a couple has not yet had children but are known to be at risk for having children with a genetic disease, they are given an occurrence risk test. Predictive testing is used to clarify the genetic status of an asymptomatic family member. Predisposition testing differs from presymptomatic testing in that a positive result does not indicate 100% risk of a condition developing. PTS: 1 DIF: Cognitive Level: Comprehension OBJ: Nursing Process: Planning MSC: Client Needs: Health Promotion and Maintenance 6. A key finding from the Human Genome Project is: a. approximately 20,500 genes make up the genome. b. all human beings are 80.99% identical at the DNA level. c. human genes produce only one protein per gene; other mammals produce three proteins per gene. d. single gene testing will become a standardized test for all pregnant patients in the future. ANS: A Approximately 20,500 genes make up the human genome; this is only twice as many as make up the genomes of roundworms and flies. Human beings are 99.9% identical at the DNA level. Most human genes produce at least three proteins. Single gene testing (e.g., alpha-fetoprotein) is already standardized for prenatal care. PTS: 1 DIF: Cognitive Level: Knowledge OBJ: Nursing Process: Assessment MSC: Client Needs: Health Promotion and Maintenance 7. In practical terms regarding genetic health care, nurses should be aware that: a. genetic disorders affect people of all socioeconomic backgrounds, races, and ethnic groups equally. b. genetic health care is more concerned with populations than individuals. c. the most important of all nursing functions is providing emotional support to the family during counseling. d. taking genetic histories is the province of large universities and medical centers. ANS: C Nurses should be prepared to help with various stress reactions from a couple facing the possibility of a genetic disorder. Although anyone may have a genetic disorder, certain disorders appear more often in certain ethnic and racial groups. Genetic health care is highly individualized because treatments are based on the phenotypic responses of the individual. Individual nurses at any facility can take a genetic history, although larger facilities may have better support services. PTS: 1 DIF: CognitiNveURLSevINelG: TCBo.mCOprMehension OBJ: Nursing Process: Planning MSC: Client Needs: Psychosocial Integrity 8. With regard to prenatal genetic testing, nurses should be aware that: a. maternal serum screening can determine whether a pregnant woman is at risk of carrying a fetus with Down syndrome. b. carrier screening tests look for gene mutations of people already showing symptoms of a disease. c. predisposition testing predicts with near certainty that symptoms will appear. d. presymptomatic testing is used to predict the likelihood of breast cancer. ANS: A Maternal serum screening identifies the risk for the neural tube defect and the specific chromosome abnormality involved in Down syndrome. Carriers of some diseases, such as sickle cell disease, do not display symptoms. Predisposition testing determines susceptibility, such as for breast cancer. presymptomatic testing indicates that symptoms are certain to appear if the gene is present. PTS: 1 DIF: Cognitive Level: Knowledge OBJ: Nursing Process: Planning MSC: Client Needs: Physiologic Integrity 9. With regard to the estimation and interpretation of the recurrence of risks for genetic disorders, nurses should be aware that: a. with a dominant disorder, the likelihood of the second child also having the condition is 100%. b. an autosomal recessive disease carries a one in eight risk of the second child also having the disorder. c. disorders involving maternal ingestion of drugs carry a one in four chance of being repeated in the second child. d. the risk factor remains the same no matter how many affected children are already in the family. ANS: D Each pregnancy is an independent event. The risk factor (e.g., one in two, one in four) remains the same for each child, no matter how many children are born to the family. In a dominant disorder, the likelihood of recurrence in subsequent children is 50% (one in two). An autosomal recessive disease carries a one in four chance of recurrence. In disorders involving maternal ingestion of drugs, subsequent children would be at risk only if the mother continued to take drugs; the rate of risk would be difficult to calculate. PTS: 1 DIF: Cognitive Level: Comprehension OBJ: Nursing Process: Diagnosis MSC: Client Needs: Health Promotion and Maintenance 10. The nurse must be cognizant that an individual's genetic makeup is known as his or her: a. genotype. b. phenotype. c. karyotype. d. chromotype. ANS: A The genotype comprises all the genes the individual can pass on to a future generation. The phenotype is the observable expression of an individual's genotype. The karyotype is a pictorial analysis of the number, fNorUmR,SaINndGTsBiz.eCOoMf an individual's chromosomes. Genotype refers to an individual's genetic makeup. PTS: 1 DIF: Cognitive Level: Knowledge OBJ: Nursing Process: Assessment MSC: Client Needs: Health Promotion and Maintenance 11. With regard to chromosome abnormalities, nurses should be aware that: a. they occur in approximately 10% of newborns. b. abnormalities of number are the leading cause of pregnancy loss. c. Down syndrome is a result of an abnormal chromosome structure. d. unbalanced translocation results in a mild abnormality that the child will outgrow. ANS: B Aneuploidy is an abnormality of number that also is the leading genetic cause of mental retardation. Chromosome abnormalities occur in less than 1% of newborns. Down syndrome is the most common form of trisomal abnormality, an abnormality of chromosome number (47 chromosomes). Unbalanced translocation is an abnormality of chromosome structure that often has serious clinical effects. PTS: 1 DIF: Cognitive Level: Comprehension OBJ: Nursing Process: Diagnosis MSC: Client Needs: Psychosocial Integrity 12. A woman's cousin gave birth to an infant with a congenital heart anomaly. The woman asks the nurse when such anomalies occur during development. Which response by the nurse is most accurate? a. “We don't really know when such defects occur.” b. “It depends on what caused the defect.” c. “They occur between the third and fifth weeks of development.” d. “They usually occur in the first 2 weeks of development.” ANS: C The cardiovascular system is the first organ system to function in the developing human. Blood vessel and blood formation begins in the third week, and the heart is developmentally complete in the fifth week. “We don't really know when such defects occur” is an inaccurate statement. Regardless of the cause, the heart is vulnerable during its period of development, the third to fifth weeks. “They usually occur in the first 2 weeks of development” is an inaccurate statement. PTS: 1 DIF: Cognitive Level: Application OBJ: Nursing Process: Evaluation MSC: Client Needs: Health Promotion and Maintenance 13. A pregnant woman at 25 weeks' gestation tells the nurse that she dropped a pan last week and her baby jumped at the noise. Which response by the nurse is most accurate? a. “That must have been a coincidence; babies can't respond like that.” b. “The fetus is demonstrating the aural reflex.” c. “Babies respond to sound starting at about 24 weeks of gestation.” d. “Let me know if it happens again; we need to report that to your midwife.” ANS: C “Babies respond to sound starting at about 24 weeks of gestation” is an accurate statement. “That must have been a coincidence; babies can't respond like that” is inaccurate. Acoustic stimulations can evoke a fetal heart rate response. There is no such thing as an aural reflex. The statement, “Let me know if itNhUaRpSpIeNnGs TaBg.aCinO;Mwe need to report that to your midwife” is not appropriate; it gives the impression that something is wrong. PTS: 1 DIF: Cognitive Level: Application OBJ: Nursing Process: Evaluation MSC: Client Needs: Health Promotion and Maintenance 14. At approximately weeks of gestation, lecithin is forming on the alveolar surfaces, the eyelids open, and the fetus measures approximately 27 cm crown to rump and weighs approximately 1110 g. a. 20 b. 24 c. 28 d. 30 ANS: C These milestones human development occur at approximately 28 weeks. PTS: 1 DIF: Cognitive Level: Knowledge OBJ: Nursing Process: Assessment MSC: Client Needs: Health Promotion and Maintenance 15. The nurse caring for the laboring woman should know that meconium is produced by: a. fetal intestines. b. fetal kidneys. c. amniotic fluid. d. the placenta. ANS: A As the fetus nears term, fetal waste products accumulate in the intestines as dark green-to-black, tarry meconium. PTS: 1 DIF: Cognitive Level: Knowledge OBJ: Nursing Process: Assessment MSC: Client Needs: Health Promotion and Maintenance 16. A woman asks the nurse, “What protects my baby's umbilical cord from being squashed while the baby's inside of me?” The nurse's best response is: a. “Your baby's umbilical cord is surrounded by connective tissue called Wharton jelly, which prevents compression of the blood vessels and ensures continued nourishment of your baby.” b. “Your baby's umbilical floats around in blood anyway.” c. “You don't need to worry about things like that.” d. “The umbilical cord is a group of blood vessels that are very well protected by the placenta.” ANS: A “Your baby's umbilical cord is surrounded by connective tissue called Wharton jelly, which prevents compression of the blood vessels and ensures continued nourishment of your baby” is the most appropriate response. “Your baby's umbilical floats around in blood anyway” is inaccurate. “You don't need to worry about things like that” is an inappropriate response. It negates the patient's need for teaching and discounts her feelings. The placenta does not protect the umbilical cord. The cord is protected by the surrounding Wharton jelly. PTS: 1 DIF: Cognitive Level: Application OBJ: Nursing Process: Planning MSC: Client Needs: Health PromotiNonUaRnSdINMGaTiBnt.eCnOaMnce 17. The is/are responsible for oxygen and carbon dioxide transport to and from the maternal bloodstream. a. decidua basalis b. blastocyst c. germ layer d. chorionic villi ANS: D Chorionic villi are finger-like projections that develop out of the trophoblast and extend into the blood-filled spaces of the endometrium. The villi obtain oxygen and nutrients from the maternal bloodstream and dispose of carbon dioxide and waste products into the maternal blood. The decidua basalis is the portion of the decidua (endometrium) under the blastocyst where the villi attach. The blastocyst is the embryonic development stage after the morula. Implantation occurs at this stage. The germ layer is a layer of the blastocyst. PTS: 1 DIF: Cognitive Level: Comprehension OBJ: Nursing Process: Assessment MSC: Client Needs: Health Promotion and Maintenance 18. A woman who is 8 months pregnant asks the nurse, “Does my baby have any antibodies to fight infection?” The most appropriate response by the nurse is: a. “Your baby has all the immune globulins necessary: IgG, IgM, and IgA.” b. “Your baby won't receive any antibodies until he/she is born and you breastfeed him.” c. “Your baby does not have any antibodies to fight infection.” d. “Your baby has IgG and IgM.” ANS: D During the third trimester, the only immune globulin that crosses the placenta, IgG, provides passive acquired immunity to specific bacterial toxins. The fetus produces IgM by the end of the first trimester. IgA is not produced by the baby. By the third trimester, the fetus has IgG and IgM. Breastfeeding supplies the baby with IgA. “Your baby does not have any antibodies to fight infection” is an inaccurate statement. PTS: 1 DIF: Cognitive Level: Application OBJ: Nursing Process: Planning MSC: Client Needs: Health Promotion and Maintenance 19. The measurement of lecithin in relation to sphingomyelin (L/S ratio) is used to determine fetal lung maturity. Which ratio reflects maturity of the lungs? a. 1.4:1 b. 1.8:1 c. 2:1 d. 1:1 ANS: C A ratio of 2:1 indicates a two-to-one ratio of L/S, an indicator of lung maturity. Ratios of 1.4:1, 1.8:1, and 1:1 indicate immaturity of the fetal lungs. PTS: 1 DIF: Cognitive Level: Knowledge OBJ: Nursing Process: Diagnosis MSC: Client Needs: Health Promotion and Maintenance 20. Sally comes in for her first prenataNlUeRxSaImNGinTaBti.oCnO.MThis is her first child. She asks you (the nurse), “How does my baby get air inside my uterus?” The correct response is: a. “The baby's lungs work in utero to exchange oxygen and carbon dioxide.” b. “The baby absorbs oxygen from your blood system.” c. “The placenta provides oxygen to the baby and excretes carbon dioxide into your bloodstream.” d. “The placenta delivers oxygen-rich blood through the umbilical artery to the baby's abdomen.” ANS: C The placenta functions by supplying oxygen and excreting carbon dioxide to the maternal bloodstream. The fetal lungs do not function for respiratory gas exchange in utero. The baby does not simply absorb oxygen from a woman's blood system. Blood and gas transport occur through the placenta. The placenta delivers oxygen-rich blood through the umbilical vein and not the artery. PTS: 1 DIF: Cognitive Level: Application OBJ: Nursing Process: Planning MSC: Client Needs: Health Promotion and Maintenance 21. The most basic information a maternity nurse should have concerning conception is that: a. ova are considered fertile 48 to 72 hours after ovulation. b. sperm remain viable in the woman's reproductive system for an average of 12 to 24 hours. c. conception is achieved when a sperm successfully penetrates the membrane surrounding the ovum. d. implantation in the endometrium occurs 6 to 10 days after conception. ANS: D After implantation, the endometrium is called the decidua. Ova are considered fertile for about 24 hours after ovulation. Sperm remain viable in the woman's reproductive system for an average of 2 to 3 days. Penetration of the ovum by the sperm is called fertilization. Conception occurs when the zygote, the first cell of the new individual, is formed. PTS: 1 DIF: Cognitive Level: Knowledge OBJ: Nursing Process: Assessment MSC: Client Needs: Health Promotion and Maintenance 22. A maternity nurse should be aware of which fact about the amniotic fluid? a. It serves as a source of oral fluid and a repository for waste from the fetus. b. The volume remains about the same throughout the term of a healthy pregnancy. c. A volume of less than 300 mL is associated with gastrointestinal malformations. d. A volume of more than 2 L is associated with fetal renal abnormalities. ANS: A Amniotic fluid serves as a source of oral fluid, serves as a repository for waste from the fetus, cushions the fetus, and helps maintain a constant body temperature. The volume of amniotic fluid changes constantly. Too little amniotic fluid (oligohydramnios) is associated with renal abnormalities. Too much amniotic fluid (hydramnios) is associated with gastrointestinal and other abnormalities. PTS: 1 DIF: Cognitive Level: Knowledge OBJ: Nursing Process: Assessment MSC: Client Needs: Health Promotion and Maintenance 23. As relates to the structure and funNctUioRnSIoNfGthTeB.pClOacMenta, the maternity nurse should be aware that: a. as the placenta widens, it gradually thins to allow easier passage of air and nutrients. b. as one of its early functions, the placenta acts as an endocrine gland. c. the placenta is able to keep out most potentially toxic substances such as cigarette smoke to which the mother is exposed. d. optimal blood circulation is achieved through the placenta when the woman is lying on her back or standing. ANS: B The placenta produces four hormones necessary to maintain the pregnancy. The placenta widens until week 20 and continues to grow thicker. Toxic substances such as nicotine and carbon monoxide readily cross the placenta into the fetus. Optimal circulation occurs when the woman is lying on her side. PTS: 1 DIF: Cognitive Level: Comprehension OBJ: Nursing Process: Assessment MSC: Client Needs: Health Promotion and Maintenance 24. With regard to the development of the respiratory system, maternity nurses should understand that: a. the respiratory system does not begin developing until after the embryonic stage. b. the infant's lungs are considered mature when the lecithin/sphingomyelin [L/S] ratio is 1:1, at about 32 weeks. c. maternal hypertension can reduce maternal-placental blood flow, accelerating lung maturity. d. fetal respiratory movements are not visible on ultrasound scans until at least 16 weeks. ANS: C A reduction in placental blood flow stresses the fetus, increases blood levels of corticosteroids, and accelerates lung maturity. Development of the respiratory system begins during the embryonic phase and continues into childhood. The infant's lungs are mature when the L/S ratio is 2:1, at about 35 weeks. Lung movements have been seen on ultrasound scans at 11 weeks. PTS: 1 DIF: Cognitive Level: Knowledge OBJ: Nursing Process: Diagnosis MSC: Client Needs: Health Promotion and Maintenance 25. Many parents-to-be have questions about multiple births. Maternity nurses should be able to tell them that: a. twinning and other multiple births are increasing because of the use of fertility drugs and delayed childbearing. b. dizygotic twins (two fertilized ova) have the potential to be conjoined twins. c. identical twins are more common in white families. d. fraternal twins are same gender, usually male. ANS: A If the parents-to-be are older and have taken fertility drugs, they would be very interested to know about twinning and other multiple births. Conjoined twins are monozygotic; they are from a single fertilized ovum in which division occurred very late. Identical twins show no racial or ethnic preference; fraternal twins are more common among African-American women. Fraternal twins can be difNfeUrRenSItNgGeTnBde.CrsOoMr the same gender. Identical twins are the same gender. PTS: 1 DIF: Cognitive Level: Knowledge OBJ: Nursing Process: Planning MSC: Client Needs: Health Promotion and Maintenance 26. The nurse caring for a pregnant patient knows that her health teaching regarding fetal circulation has been effective when the patient reports that she has been sleeping: a. in a side-lying position. b. on her back with a pillow under her knees. c. with the head of the bed elevated. d. on her abdomen. ANS: A Optimal circulation is achieved when the woman is lying at rest on her side. Decreased uterine circulation may lead to intrauterine growth restriction. Previously it was believed that the left lateral position promoted maternal cardiac output, enhancing blood flow to the fetus. However, it is now known that the side-lying position enhances uteroplacental blood flow. If a woman lies on her back with the pressure of the uterus compressing the vena cava, blood return to the right atrium is diminished. Although having the head of the bed elevated is recommended and ideal for later in pregnancy, the woman still must maintain a lateral tilt to the pelvis to avoid compression of the vena cava. Many women find lying on her abdomen uncomfortable as pregnancy advances. Side-lying is the ideal position to promote blood flow to the fetus. PTS: 1 DIF: Cognitive Level: Analysis OBJ: Nursing Process: Evaluation MSC: Client Needs: Health Promotion and Maintenance 27. A woman is 15 weeks pregnant with her first baby. She asks how long it will be before she feels the baby move. The best answer is: a. “You should have felt the baby move by now.” b. “Within the next month, you should start to feel fluttering sensations.” c. “The baby is moving; however, you can't feel it yet.” d. “Some babies are quiet, and you don't feel them move.” ANS: B Maternal perception of fetal movement usually begins 16 to 20 weeks after conception. Because this is her first pregnancy, movement is felt toward the later part of the 16- to 20-week time period. Stating that “you should have felt the baby move by now” is incorrect and may be alarming to the patient. Fetal movement should be felt by 16 to 20 weeks. If movement is not felt by the end of that time, further assessment will be necessary. PTS: 1 DIF: Cognitive Level: Knowledge OBJ: Nursing Process: Implementation MSC: Client Needs: Health Promotion and Maintenance 28. A new mother asks the nurse about the “white substance” covering her infant. The nurse explains that the purpose of vernix caseosa is to: a. protect the fetal skin from amniotic fluid. b. promote normal peripheral nervous system development. c. allow transport of oxygen and nutrients across the amnion. d. regulate fetal temperature. ANS: A NURSINGTB.COM Prolonged exposure to amniotic fluid during the fetal period could result in breakdown of the skin without the protection of the vernix caseosa. Normal development of the peripheral nervous system is dependent on nutritional intake of the mother. The amnion is the inner membrane that surrounds the fetus. It is not involved in the oxygen and nutrient exchange. The amniotic fluid aids in maintaining fetal temperature. PTS: 1 DIF: Cognitive Level: Knowledge OBJ: Nursing Process: Assessment MSC: Client Needs: Health Promotion and Maintenance 29. The placenta allows exchange of oxygen, nutrients, and waste products between the mother and fetus by: a. contact between maternal blood and fetal capillaries within the chorionic villi. b. interaction of maternal and fetal pH levels within the endometrial vessels. c. a mixture of maternal and fetal blood within the intervillous spaces. d. passive diffusion of maternal carbon dioxide and oxygen into the fetal capillaries. ANS: A Fetal capillaries within the chorionic villi are bathed with oxygen-rich and nutrient-rich maternal blood within the intervillous spaces. The endometrial vessels are part of the uterus. There is no interaction with the fetal blood at this point. Maternal and fetal bloods do not normally mix. Maternal carbon dioxide does not enter into the fetal circulation. PTS: 1 DIF: Cognitive Level: Knowledge OBJ: Nursing Process: Assessment MSC: Client Needs: Health Promotion and Maintenance MULTIPLE RESPONSE 1. Congenital disorders refer to conditions that are present at birth. These disorders may be inherited and caused by environmental factors or maternal malnutrition. Toxic exposures have the greatest effect on development between 15 and 60 days of gestation. For the nurse to be able to conduct a complete assessment of the newly pregnant patient, she should understand the significance of exposure to known human teratogens. These include: (Select all that apply.) a. infections. b. radiation. c. maternal conditions. d. drugs. e. chemicals. ANS: A, B, C, D, E Exposure to radiation and numerous infections may result in profound congenital deformities. These include but are not limited to varicella, rubella, syphilis, parvovirus, cytomegalovirus, and toxoplasmosis. Certain maternal conditions such as diabetes and phenylketonuria may also affect organs and other parts of the embryo during this developmental period. Drugs such as antiseizure medication and some antibiotics as well as chemicals, including lead, mercury, tobacco, and alcohol, also may result in structural and functional abnormalities. PTS: 1 DIF: Cognitive Level: Comprehension OBJ: Nursing Process: Assessment MSC: Client Needs: Health Promotion and Maintenance NURSINGTB.COM 2. Which congenital malformations result from multifactorial inheritance? (Select all that apply.) a. Cleft lip b. Congenital heart disease c. Cri du chat syndrome d. Anencephaly e. Pyloric stenosis ANS: A, B, D, E All these congenital malformations are associated with multifactorial inheritance. Cri du chat syndrome is related to a chromosome deletion. PTS: 1 DIF: Cognitive Level: Knowledge OBJ: Nursing Process: Diagnosis MSC: Client Needs: Psychosocial Integrity 3. Along with gas exchange and nutrient transfer, the placenta produces many hormones necessary for normal pregnancy. These include: (Select all that apply.) a. human chorionic gonadotropin (hCG). b. insulin. c. estrogen. d. progesterone. e. testosterone. ANS: A, C, D hCG causes the corpus luteum to persist and produce the necessary estrogens and progesterone for the first 6 to 8 weeks. Estrogens cause enlargement of the woman's uterus and breasts; cause growth of the ductal system in the breasts; and, as term approaches, play a role in the initiation of labor. Progesterone causes the endometrium to change, providing early nourishment. Progesterone also protects against spontaneous abortion by suppressing maternal reactions to fetal antigens and reduces unnecessary uterine contractions. Other hormones produced by the placenta include hCT, hCA, and numerous growth factors. Human placental lactogen promotes normal nutrition and growth of the fetus and maternal breast development for lactation. This hormone decreases maternal insulin sensitivity and utilization of glucose, making more glucose available for fetal growth. If a Y chromosome is present in the male fetus, hCG causes the fetal testes to secrete testosterone necessary for the normal development of male reproductive structures. PTS: 1 DIF: Cognitive Level: Comprehension OBJ: Nursing Process: Assessment MSC: Client Needs: Health Promotion and Maintenance NURSINGTB.COM Chapter 07: Anatomy and Physiology of Pregnancy Perry: Maternal Child Nursing Care, 6th Edition MULTIPLE CHOICE 1. A woman's obstetric history indicates that she is pregnant for the fourth time and all of her children from previous pregnancies are living. One was born at 39 weeks of gestation, twins were born at 34 weeks of gestation, and another child was born at 35 weeks of gestation. What is her gravidity and parity using the GTPAL system? a. 3-1-1-1-3 b. 4-1-2-0-4 c. 3-0-3-0-3 d. 4-2-1-0-3 ANS: B The correct calculation of this woman's gravidity and parity is 4-1-2-0-4. The numbers reflect the woman's gravidity and parity information. Using the GPTAL system, her information is calculated as: G: The first number reflects the total number of times the woman has been pregnant; she is pregnant for the fourth time. T: This number indicates the number of pregnancies carried to term, not the number of deliveries at term; only one of her pregnancies has resulted in a fetus at term. P: This is the number of pregnancies that resulted in a preterm birth; the woman has had two pregnancies in which she delivered preterm. A: This number signifies whether the woman has had any abortions or miscarriages before the period of viability; she has not. NURSINGTB.COM L: This number signifies the number of children born who are currently living; the woman has four children. PTS: 1 DIF: Cognitive Level: Comprehension OBJ: Nursing Process: Diagnosis MSC: Client Needs: Health Promotion and Maintenance 2. A woman at 10 weeks of gestation who is seen in the prenatal clinic with presumptive signs and symptoms of pregnancy likely will have: a. amenorrhea. b. positive pregnancy test. c. Chadwick's sign. d. Hegar's sign. ANS: A Amenorrhea is a presumptive sign of pregnancy. Presumptive signs of pregnancy are felt by the woman. A positive pregnancy test, the presence of Chadwick's sign, and the presence of Hegar's sign are all probable signs of pregnancy. PTS: 1 DIF: Cognitive Level: Comprehension OBJ: Nursing Process: Assessment MSC: Client Needs: Health Promotion and Maintenance 3. The nurse teaches a pregnant woman about the presumptive, probable, and positive signs of pregnancy. The woman demonstrates understanding of the nurse's instructions if she states that a positive sign of pregnancy is: a. a positive pregnancy test. b. fetal movement palpated by the nurse-midwife. c. Braxton Hicks contractions. d. quickening. ANS: B Positive signs of pregnancy are attributed to the presence of a fetus, such as hearing the fetal heartbeat or palpating fetal movement. A positive pregnancy test and Braxton Hicks contractions are probable signs of pregnancy. Quickening is a presumptive sign of pregnancy. PTS: 1 DIF: Cognitive Level: Comprehension OBJ: Nursing Process: Planning MSC: Client Needs: Health Promotion and Maintenance 4. A woman is at 14 weeks of gestation. The nurse would expect to palpate the fundus at which level? a. Not palpable above the symphysis at this time b. Slightly above the symphysis pubis c. At the level of the umbilicus d. Slightly above the umbilicus ANS: B In normal pregnancies, the uterus grows at a predictable rate. It may be palpated above the symphysis pubis sometime between the 12th and 14th weeks of pregnancy. As the uterus grows, it may be palpated above the symphysis pubis sometime between the 12th and 14th weeks of pregnancy. The uterus rises gradually to the level of the umbilicus at 22 to 24 weeks of gestation. PTS: 1 DIF: CognitiNveURLSevINelG: TCBo.mCOprMehension OBJ: Nursing Process: Assessment MSC: Client Needs: Health Promotion and Maintenance 5. During a patient's physical examination the nurse notes that the lower uterine segment is soft on palpation. The nurse would document this finding as: a. Hegar's sign. b. McDonald's sign. c. Chadwick's sign. d. Goodell's sign. ANS: A At approximately 6 weeks of gestation, softening and compressibility of the lower uterine segment occurs; this is called Hegar's sign. McDonald's sign indicates a fast food restaurant. Chadwick's sign is the blue-violet coloring of the cervix caused by increased vascularity; this occurs around the fourth week of gestation. Softening of the cervical tip is called Goodell's sign, which may be observed around the sixth week of pregnancy. PTS: 1 DIF: Cognitive Level: Comprehension OBJ: Nursing Process: Assessment, Implementation MSC: Client Needs: Health Promotion and Maintenance 6. Cardiovascular system changes occur during pregnancy. Which finding would be considered normal for a woman in her second trimester? a. Less audible heart sounds (S1, S2) b. Increased pulse rate c. Increased blood pressure d. Decreased red blood cell (RBC) production ANS: B Between 14 and 20 weeks of gestation, the pulse increases about 10 to 15 beats/min, which persists to term. Splitting of S1 and S2 is more audible. In the first trimester, blood pressure usually remains the same as at the prepregnancy level, but it gradually decreases up to about 20 weeks of gestation. During the second trimester, both the systolic and the diastolic pressures decrease by about 5 to 10 mm Hg. Production of RBCs accelerates during pregnancy. PTS: 1 DIF: Cognitive Level: Comprehension OBJ: Nursing Process: Assessment MSC: Client Needs: Physiologic Integrity 7. Numerous changes in the integumentary system occur during pregnancy. Which change persists after birth? a. Epulis b. Chloasma c. Telangiectasia d. Striae gravidarum ANS: D Striae gravidarum, or stretch marks, reflect separation within the underlying connective tissue of the skin. They usually fade after birth, although they never disappear completely. An epulis is a red, raised nodule on the gums that bleeds easily. Chloasma, or mask of pregnancy, is a blotchy, brown hyperpigmentation of the skin over the cheeks, nose, and forehead, especially in dark-complexioned pregnant women. Chloasma usually fades after the birth. Telangiectasia, or vascular spidersN,UaRreSItNinGyT, Bst.CarO-sMhaped or branch-like, slightly raised, pulsating end-arterioles usually found on the neck, thorax, face, and arms. They occur as a result of elevated levels of circulating estrogen. These usually disappear after birth. PTS: 1 DIF: Cognitive Level: Comprehension OBJ: Nursing Process: Planning MSC: Client Needs: Physiologic Integrity 8. The musculoskeletal system adapts to the changes that occur during pregnancy. A woman can expect to experience what change? a. Her center of gravity will shift backward. b. She will have increased lordosis. c. She will have increased abdominal muscle tone. d. She will notice decreased mobility of her pelvic joints. ANS: B An increase in the normal lumbosacral curve (lordosis) develops, and a compensatory curvature in the cervicodorsal region develops to help the woman maintain her balance. The center of gravity shifts forward. She will have decreased muscle tone. She will notice increased mobility of her pelvic joints. PTS: 1 DIF: Cognitive Level: Comprehension OBJ: Nursing Process: Planning MSC: Client Needs: Physiologic Integrity 9. A 31-year-old woman believes that she may be pregnant. She took an OTC pregnancy test 1 week ago after missing her period; the test was positive. During her assessment interview, the nurse enquires about the woman's last menstrual period and asks whether she is taking any medications. The woman states that she takes medicine for epilepsy. She has been under considerable stress lately at work and has not been sleeping well. She also has a history of irregular periods. Her physical examination does not indicate that she is pregnant. She has an ultrasound scan that reveals she is not pregnant. What is the most likely cause of the false-positive pregnancy test result? a. She took the pregnancy test too early. b. She takes anticonvulsants. c. She has a fibroid tumor. d. She has been under considerable stress and has a hormone imbalance. ANS: B Anticonvulsants may cause false-positive pregnancy test results. OTC pregnancy tests use enzyme-linked immunosorbent assay technology, which can yield positive results 4 days after implantation. Implantation occurs 6 to 10 days after conception. If the woman were pregnant, she would be into her third week at this point (having missed her period 1 week ago). Fibroid tumors do not produce hormones and have no bearing on hCG pregnancy tests. Although stress may interrupt normal hormone cycles (menstrual cycles), it does not affect human chorionic gonadotropin levels or produce positive pregnancy test results. PTS: 1 DIF: Cognitive Level: Application OBJ: Nursing Process: Assessment MSC: Client Needs: Physiologic Integrity 10. A woman is in her seventh month of pregnancy. She has been reporting nasal congestion and occasional epistaxis. The nurse suNspUeRcStsINthGaTtB: .COM a. this is a normal respiratory change in pregnancy caused by elevated levels of estrogen. b. this is an abnormal cardiovascular change, and the nosebleeds are an ominous sign. c. the woman is a victim of domestic violence and is being hit in the face by her partner. d. the woman has been using cocaine intranasally. ANS: A Elevated levels of estrogen cause capillaries to become engorged in the respiratory tract. This may result in edema in the nose, larynx, trachea, and bronchi. This congestion may cause nasal stuffiness and epistaxis. Cardiovascular changes in pregnancy may cause edema in lower extremities. Determining that the woman is a victim of domestic violence and was hit in the face cannot be made on the basis of the sparse facts provided. If the woman had been hit in the face, she most likely would have additional physical findings. Determination of the use of cocaine by the woman cannot be made on the basis of the sparse facts provided. PTS: 1 DIF: Cognitive Level: Application OBJ: Nursing Process: Assessment MSC: Client Needs: Health Promotion and Maintenance 11. The nurse caring for the pregnant patient must understand that the hormone essential for maintaining pregnancy is: a. estrogen. b. human chorionic gonadotropin (hCG). c. oxytocin. d. progesterone. ANS: D Progesterone is essential for maintaining pregnancy; it does so by relaxing smooth muscles. This reduces uterine activity and prevents miscarriage. Estrogen plays a vital role in pregnancy, but it is not the primary hormone for maintaining pregnancy. hCG levels increase at implantation but decline after 60 to 70 days. Oxytocin stimulates uterine contractions. PTS: 1 DIF: Cognitive Level: Comprehension OBJ: Nursing Process: Assessment MSC: Client Needs: Health Promotion and Maintenance 12. A patient at 24 weeks of gestation contacts the nurse at her obstetric provider's office to complain that she has cravings for dirt and gravel. The nurse is aware that this condition is known as and may indicate anemia. a. ptyalism b. pyrosis c. pica d. decreased peristalsis ANS: C Pica (a desire to eat nonfood substances) is an indication of iron deficiency and should be evaluated. Ptyalism (excessive salivation), pyrosis (heartburn), and decreased peristalsis are normal findings of gastrointestinal change during pregnancy. Food cravings during pregnancy are normal. PTS: 1 DIF: Cognitive Level: Analysis OBJ: Nursing Process: Assessment MSC: Client Needs: Health Promotion and Maintenance NURSINGTB.COM 13. Appendicitis may be difficult to diagnose in pregnancy because the appendix is: a. displaced upward and laterally, high and to the right. b. displaced upward and laterally, high and to the left. c. deep at McBurney point. d. displaced downward and laterally, low and to the right. ANS: A The appendix is displaced high and to the right, beyond McBurney point. PTS: 1 DIF: Cognitive Level: Comprehension OBJ: Nursing Process: Diagnosis MSC: Client Needs: Physiologic Integrity 14. A woman who has completed one pregnancy with a fetus (or fetuses) reaching the stage of fetal viability is called a: a. primipara. b. primigravida. c. multipara. d. nulligravida. ANS: A A primipara is a woman who has completed one pregnancy with a viable fetus. To remember terms, keep in mind: gravida is a pregnant woman; para comes from parity, meaning a viable fetus; primi means first; multi means many; and null means none. A primigravida is a woman pregnant for the first time. A multipara is a woman who has completed two or more pregnancies with a viable fetus. A nulligravida is a woman who has never been pregnant. PTS: 1 DIF: Cognitive Level: Comprehension OBJ: Nursing Process: Diagnosis MSC: Client Needs: Health Promotion and Maintenance 15. Which time-based description of a stage of development in pregnancy is accurate? a. Viability—22 to 37 weeks since the last menstrual period (LMP) (assuming a fetal weight >500 g). b. Full Term—Pregnancy from the beginning of week 39 of gestation to the end of week 40. c. Preterm—Pregnancy from 20 to 28 weeks. d. Postdate—Pregnancy that extends beyond 38 weeks. ANS: B Full Term is 39 to 40 weeks of gestation. Viability is the ability of the fetus to live outside the uterus before coming to term, or 22 to 24 weeks since LMP. Preterm is 20 to 37 weeks of gestation. Postdate or postterm is a pregnancy that extends beyond 42 weeks or what is considered the limit of full term. PTS: 1 DIF: Cognitive Level: Knowledge OBJ: Nursing Process: Diagnosis MSC: Client Needs: Health Promotion and Maintenance 16. Human chorionic gonadotropin (hCG) is an important biochemical marker for pregnancy and the basis for many tests. A maternNitUyRnSuIrNsGeTsBh.oCuOldMbe aware that: a. hCG can be detected 2.5 weeks after conception. b. the hCG level increases gradually and uniformly throughout pregnancy. c. much lower than normal increases in the level of hCG may indicate a postdate pregnancy. d. a higher than normal level of hCG may indicate an ectopic pregnancy or Down syndrome. ANS: D Higher levels also could be a sign of multiple gestation. hCG can be detected 7 to 8 days after conception. The hCG level fluctuates during pregnancy: peaking, declining, stabilizing, and increasing again. Abnormally slow increases may indicate impending miscarriage. PTS: 1 DIF: Cognitive Level: Knowledge OBJ: Nursing Process: Diagnosis MSC: Client Needs: Health Promotion and Maintenance 17. To reassure and educate pregnant patients about changes in the uterus, nurses should be aware that: a. lightening occurs near the end of the second trimester as the uterus rises into a different position. b. the woman's increased urinary frequency in the first trimester is the result of exaggerated uterine anteflexion caused by softening. c. Braxton Hicks contractions become more painful in the third trimester, particularly if the woman tries to exercise. d. the uterine souffle is the movement of the fetus. ANS: B The softening of the lower uterine segment is called Hegar's sign. Lightening occurs in the last 2 weeks of pregnancy, when the fetus descends. Braxton Hicks contractions become more defined in the final trimester but are not painful. Walking or exercise usually causes them to stop. The uterine souffle is the sound made by blood in the uterine arteries; it can be heard with a fetal stethoscope. PTS: 1 DIF: Cognitive Level: Comprehension OBJ: Nursing Process: Planning MSC: Client Needs: Health Promotion and Maintenance 18. To reassure and educate pregnant patients about changes in the cervix, vagina, and position of the fetus, nurses should be aware that: a. because of a number of changes in the cervix, abnormal Papanicolaou (Pap) tests are much easier to evaluate. b. Quickening is a technique of palpating the fetus to engage it in passive movement. c. the deepening color of the vaginal mucosa and cervix (Chadwick's sign) usually appears in the second trimester or later as the vagina prepares to stretch during labor. d. increased vascularity of the vagina increases sensitivity and may lead to a high degree of arousal, especially in the second trimester. ANS: D Increased sensitivity and an increased interest in sex sometimes go together. This frequently occurs during the second trimester. Cervical changes make evaluation of abnormal Pap tests more difficult. Quickening is the first recognition of fetal movements by the mother. Ballottement is a technique used tNoUpRalSpINatGeTtBhe.CfOeMtus. Chadwick's sign appears from the sixth to eighth weeks. PTS: 1 DIF: Cognitive Level: Comprehension OBJ: Nursing Process: Planning MSC: Client Needs: Health Promotion and Maintenance 19. The mucous plug that forms in the endocervical canal is called the: a. operculum. b. leukorrhea. c. funic souffle. d. ballottement. ANS: A The operculum protects against bacterial invasion. Leukorrhea is the mucus that forms the endocervical plug (the operculum). The funic souffle is the sound of blood flowing through the umbilical vessels. Ballottement is a technique for palpating the fetus. PTS: 1 DIF: Cognitive Level: Knowledge OBJ: Nursing Process: Assessment MSC: Client Needs: Physiologic Integrity 20. To reassure and educate pregnant patients about changes in their breasts, nurses should be aware that: a. the visibility of blood vessels that form an intertwining blue network indicates full function of Montgomery's tubercles and possibly infection of the tubercles. b. the mammary glands do not develop until 2 weeks before labor. c. lactation is inhibited until the estrogen level declines after birth. d. colostrum is the yellowish oily substance used to lubricate the nipples for breastfeeding. ANS: C Lactation is inhibited until after birth. The visible blue network of blood vessels is a normal outgrowth of a richer blood supply. The mammary glands are functionally complete by midpregnancy. Colostrum is a creamy, white-to-yellow premilk fluid that can be expressed from the nipples before birth. PTS: 1 DIF: Cognitive Level: Knowledge OBJ: Nursing Process: Planning MSC: Client Needs: Health Promotion and Maintenance 21. To reassure and educate pregnant patients about changes in their cardiovascular system, maternity nurses should be aware that: a. a pregnant woman experiencing disturbed cardiac rhythm, such as sinus arrhythmia requires close medical and obstetric observation, no matter how healthy she otherwise may appear. b. changes in heart size and position and increases in blood volume create auditory changes from 20 weeks to term. c. palpitations are twice as likely to occur in twin gestations. d. all of the above changes will likely occur. ANS: B Auscultatory changes should be discernible after 20 weeks of gestation. A healthy woman with no underlying heart disease does not need any therapy. The maternal heart rate increases in the third trimester, but palpitations may not occur. Auditory changes are discernible at 20 weeks. NURSINGTB.COM PTS: 1 DIF: Cognitive Level: Comprehension OBJ: Nursing Process: Planning MSC: Client Needs: Physiologic Integrity 22. To reassure and educate their pregnant patients about changes in their blood pressure, maternity nurses should be aware that: a. a blood pressure cuff that is too small produces a reading that is too low; a cuff that is too large produces a reading that is too high. b. shifting the patient's position and changing from arm to arm for different measurements produces the most accurate composite blood pressure reading at each visit. c. the systolic blood pressure increases slightly as pregnancy advances; the diastolic pressure remains constant. d. compression of the iliac veins and inferior vena cava by the uterus contributes to hemorrhoids in the later stage of term pregnancy. ANS: D Compression of the iliac veins and inferior vena cava also leads to varicose veins in the legs and vulva. The tightness of a cuff that is too small produces a reading that is too high; similarly the looseness of a cuff that is too large results in a reading that is too low. Because maternal positioning affects readings, blood pressure measurements should be obtained in the same arm and with the woman in the same position. The systolic blood pressure generally remains constant but may decline slightly as pregnancy advances. The diastolic blood pressure first decreases and then gradually increases. PTS: 1 DIF: Cognitive Level: Comprehension OBJ: Nursing Process: Planning, Implementation MSC: Client Needs: Physiologic Integrity 23. Some pregnant patients may complain of changes in their voice and impaired hearing. The nurse can tell these patients that these are common reactions to: a. a decreased estrogen level. b. displacement of the diaphragm, resulting in thoracic breathing. c. congestion and swelling, which occur because the upper respiratory tract has become more vascular. d. increased blood volume. ANS: C Estrogen levels increase, causing the upper respiratory tract to become more vascular producing swelling and congestion in the nose and ears leading to voice changes and impaired hearing. The diaphragm is displaced, and the volume of blood is increased. However, the main concern is increased estrogen levels. PTS: 1 DIF: Cognitive Level: Comprehension OBJ: Nursing Process: Planning MSC: Client Needs: Physiologic IntNegUriRtySINGTB.COM 24. To reassure and educate pregnant patients about the functioning of their kidneys in eliminating waste products, maternity nurses should be aware that: a. increased urinary output makes pregnant women less susceptible to urinary infection. b. increased bladder sensitivity and then compression of the bladder by the enlarging uterus results in the urge to urinate even if the bladder is almost empty. c. renal (kidney) function is more efficient when the woman assumes a supine position. d. using diuretics during pregnancy can help keep kidney function regular. ANS: B First bladder sensitivity and then compression of the bladder by the uterus result in the urge to urinate more often. Numerous anatomic changes make a pregnant woman more susceptible to urinary tract infection. Renal function is more efficient when the woman lies in the lateral recumbent position and less efficient when she is supine. Diuretic use during pregnancy can overstress the system and cause problems. PTS: 1 DIF: Cognitive Level: Comprehension OBJ: Nursing Process: Planning MSC: Client Needs: Physiologic Integrity 25. Which statement about a condition of pregnancy is accurate? a. Insufficient salivation (ptyalism) is caused by increases in estrogen. b. Acid indigestion (pyrosis) begins early but declines throughout pregnancy. c. Hyperthyroidism often develops (temporarily) because hormone production increases. d. Nausea and vomiting rarely have harmful effects on the fetus and may be beneficial. ANS: D Normal nausea and vomiting rarely produce harmful effects, and nausea and vomiting periods may be less likely to result in miscarriage or preterm labor. Ptyalism is excessive salivation, which may be caused by a decrease in unconscious swallowing or stimulation of the salivary glands. Pyrosis begins in the first trimester and intensifies through the third trimester. Increased hormone production does not lead to hyperthyroidism in pregnant women. PTS: 1 DIF: Cognitive Level: Application OBJ: Nursing Process: Assessment MSC: Client Needs: Health Promotion and Maintenance 26. A first-time mother at 18 weeks of gestation comes for her regularly scheduled prenatal visit. The patient tells the nurse that she is afraid that she is going into premature labor because she is beginning to have regular contractions. The nurse explains that this is the Braxton Hicks sign and teaches the patient that this type of contraction: a. is painless. b. increases with walking. c. causes cervical dilation. d. impedes oxygen flow to the fetus. ANS: A Uterine contractions can be felt through the abdominal wall soon after the fourth month of gestation. Braxton Hicks contractiNoUnsRSaIrNeGreTgBu.ClaOrMand painless and continue throughout the pregnancy. Although they are not painful, some women complain that they are annoying. Braxton Hicks contractions usually cease with walking or exercise. They can be mistaken for true labor; however, they do not increase in intensity or frequency or cause cervical dilation. In addition, they facilitate uterine blood flow through the intervillous spaces of the placenta and promote oxygen delivery to the fetus. PTS: 1 DIF: Cognitive Level: Comprehension OBJ: Nursing Process: Planning MSC: Client Needs: Health Promotion and Maintenance 27. Which finding in the urine analysis of a pregnant woman is considered a variation of normal? a. Proteinuria b. Glycosuria c. Bacteria in the urine d. Ketones in the urine ANS: B Small amounts of glucose may indicate “physiologic spilling.” The presence of protein could indicate kidney disease or preeclampsia. Urinary tract infections are associated with bacteria in the urine. An increase in ketones indicates that the patient is exercising too strenuously or has an inadequate fluid and food intake. PTS: 1 DIF: Cognitive Level: Analysis OBJ: Nursing Process: Assessment MSC: Client Needs: Physiologic Integrity 28. The maternity nurse understands that vascular volume increases 40% to 45% during pregnancy to: a. compensate for decreased renal plasma flow. b. provide adequate perfusion of the placenta. c. eliminate metabolic wastes of the mother. d. prevent maternal and fetal dehydration. ANS: B The primary function of increased vascular volume is to transport oxygen and nutrients to the fetus via the placenta. Renal plasma flow increases during pregnancy. Assisting with pulling metabolic wastes from the fetus for maternal excretion is one purpose of the increased vascular volume. PTS: 1 DIF: Cognitive Level: Comprehension OBJ: Nursing Process: Assessment MSC: Client Needs: Physiologic Integrity 29. Physiologic anemia often occurs during pregnancy as a result of: a. inadequate intake of iron. b. dilution of hemoglobin concentration. c. the fetus establishing iron stores. d. decreased production of erythrocytes. ANS: B When blood volume expansion is more pronounced and occurs earlier than the increase in red blood cells, the woman has physiologic anemia, which is the result of dilution of hemoglobin concentration rather than inadequate hemoglobin. Inadequate intake of iron may lead to true anemia. There is an increased production of erythrocytes during pregnancy. NURSINGTB.COM PTS: 1 DIF: Cognitive Level: Knowledge OBJ: Nursing Process: Assessment MSC: Client Needs: Physiologic Integrity 30. A patient in her first trimester complains of nausea and vomiting. She asks, “Why does this happen?” The nurse's best response is: a. “It is due to an increase in gastric motility.” b. “It may be due to changes in hormones.” c. “It is related to an increase in glucose levels.” d. “It is caused by a decrease in gastric secretions.” ANS: B Nausea and vomiting are believed to be caused by increased levels of hormones, decreased gastric motility, and hypoglycemia. Gastric motility decreases during pregnancy. Glucose levels decrease in the first trimester. Although gastric secretions decrease, this is not the main cause of nausea and vomiting. PTS: 1 DIF: Cognitive Level: Knowledge OBJ: Nursing Process: Implementation MSC: Client Needs: Physiologic Integrity MULTIPLE RESPONSE 1. The diagnosis of pregnancy is based on which positive signs of pregnancy? (Select all that apply.) a. Identification of fetal heartbeat b. Palpation of fetal outline c. Visualization of the fetus d. Verification of fetal movement e. Positive hCG test ANS: A, C, D Identification of fetal heartbeat, visualization of the fetus, and verification of fetal movement are all positive, objective signs of pregnancy. Palpation of fetal outline and a positive hCG test are probable signs of pregnancy. A tumor also can be palpated. Medication and tumors may lead to false-positive results on pregnancy tests. PTS: 1 DIF: Cognitive Level: Analysis OBJ: Nursing Process: Assessment, Diagnosis MSC: Client Needs: Health Promotion and Maintenance 2. During pregnancy, many changes occur as a direct result of the presence of the fetus. Which of these adaptations meet this criterion? (Select all that apply.) a. Leukorrhea b. Development of the operculum c. Quickening d. Ballottement e. Lightening ANS: C, D, E Leukorrhea is a white or slightly gray vaginal discharge that develops in response to cervical stimulation by estrogen and progesterone. Quickening is the first recognition of fetal movements or “feeling life.” QuicNkUenRiSnIgNGisToBf.tCeOnMdescribed as a flutter and is felt earlier in multiparous women than in primiparas. Lightening occurs when the fetus begins to descend into the pelvis. This occurs 2 weeks before labor in the nullipara and at the start of labor in the multipara. Mucus fills the cervical canal creating a plug otherwise known as the operculum. The operculum acts as a barrier against bacterial invasion during the pregnancy. Passive movement of the unengaged fetus is referred to as ballottement. PTS: 1 DIF: Cognitive Level: Comprehension OBJ: Nursing Process: Assessment MSC: Client Needs: Physiologic Integrity COMPLETION 1. A woman is 6 weeks pregnant. She had a previous spontaneous abortion at 14 weeks of gestation and a pregnancy that ended at 38 weeks with the birth of a stillborn girl. What is her gravidity and parity using the GTPAL system? ANS: 3-1-0-1-0 The correct calculation of this woman's gravidity and parity is 3-1-0-1-0. Using the GPTAL system, this patient's gravidity and parity information is calculated as follows: G: Total number of times the woman has been pregnant (she is pregnant for the third time). T: Number of pregnancies carried to term (she has had only one pregnancy that resulted in a fetus at term). P: Number of pregnancies that resulted in a preterm birth (none). A: Abortions or miscarriages before the period of viability (she has had one). L: Number of children born who are currently living (she has no living children). PTS: 1 DIF: Cognitive Level: Comprehension OBJ: Nursing Process: Diagnosis MSC: Client Needs: Health Promotion and Maintenance NURSINGTB.COM Chapter 08: Nursing Care of the Family During Pregnancy Perry: Maternal Child Nursing Care, 6th Edition MULTIPLE CHOICE 1. The nurse caring for a newly pregnant woman would advise her that ideally prenatal care should begin: a. before the first missed menstrual period. b. after the first missed menstrual period. c. after the second missed menstrual period. d. after the third missed menstrual period. ANS: B Prenatal care ideally should begin soon after the first missed menstrual period. Regular prenatal visits offer opportunities to ensure the health of the expectant mother and her infant. PTS: 1 DIF: Cognitive Level: Knowledge OBJ: Nursing Process: Planning MSC: Client Needs: Health Promotion and Maintenance 2. Prenatal testing for human immunodeficiency virus (HIV) is recommended for: a. all women, regardless of risk factors. b. a woman who has had more than one sexual partner. c. a woman who has had a sexually transmitted infection. d. a woman who is monogamous with her partner. ANS: A Testing for the antibody to HIV isNsUtrRoSnIgNlGyTrBec.CoOmMmended for all pregnant women. A HIV test is recommended for all women, regardless of risk factors. Women who test positive for HIV can be treated, reducing the risk of transmission to the fetus. PTS: 1 DIF: Cognitive Level: Comprehension OBJ: Nursing Process: Planning MSC: Client Needs: Health Promotion and Maintenance 3. Which symptom is considered a warning sign and should be reported immediately by the pregnant woman to her health care provider? a. Nausea with occasional vomiting b. Fatigue c. Urinary frequency d. Vaginal bleeding ANS: D Signs and symptoms that must be reported include severe vomiting, fever and chills, burning on urination, diarrhea, abdominal cramping, and vaginal bleeding. These symptoms may be signs of potential complications of the pregnancy. Nausea with occasional vomiting, fatigue, and urinary frequency are normal first-trimester complaints. Although they may be worrisome or annoying to the mother, they usually are not indications of pregnancy problems. PTS: 1 DIF: Cognitive Level: Comprehension OBJ: Nursing Process: Planning MSC: Client Needs: Physiologic Integrity 4. A pregnant woman at 10 weeks of gestation jogs three or four times per week. She is concerned about the effect of exercise on the fetus. The nurse should inform her: a. “You don't need to modify your exercising any time during your pregnancy.” b. “Stop exercising because it will harm the fetus.” c. “You may find that you need to modify your exercise to walking later in your pregnancy, around the seventh month.” d. “Jogging is too hard on your joints; switch to walking now.” ANS: C Typically running should be replaced with walking around the seventh month of pregnancy. The nurse should inform the woman that she may need to reduce her exercise level as the pregnancy progresses. Physical activity promotes a feeling of well-being in pregnant women. It improves circulation, promotes relaxation and rest, and counteracts boredom. Simple measures should be initiated to prevent injuries, such as warm-up and stretching exercises to prepare the joints for more strenuous exercise. PTS: 1 DIF: Cognitive Level: Application OBJ: Nursing Process: Planning MSC: Client Needs: Health Promotion and Maintenance 5. The multiple marker test is used to assess the fetus for which condition? a. Down syndrome b. Diaphragmatic hernia c. Congenital cardiac abnormality d. Anencephaly ANS: A The maternal serum level of alpha-fetoprotein is used to screen for Down syndrome, neural tube defects, and other chromosomNeURanSoINmGaTliBe.sC.OTMhe multiple marker test would not detect diaphragmatic hernia, congenital cardiac abnormality, or anencephaly. Additional testing, such as ultrasonography and amniocentesis, would be required to diagnose these conditions. PTS: 1 DIF: Cognitive Level: Knowledge OBJ: Nursing Process: Assessment MSC: Client Needs: Health Promotion and Maintenance 6. A woman who is 32 weeks' pregnant is informed by the nurse that a danger sign of pregnancy could be: a. constipation. b. alteration in the pattern of fetal movement. c. heart palpitations. d. edema in the ankles and feet at the end of the day. ANS: B An alteration in the pattern or amount of fetal movement may indicate fetal jeopardy. Constipation, heart palpitations, and ankle and foot edema are normal discomforts of pregnancy that occur in the second and third trimesters. PTS: 1 DIF: Cognitive Level: Comprehension OBJ: Nursing Process: Planning MSC: Client Needs: Health Promotion and Maintenance 7. A woman who is 14 weeks pregnant tells the nurse that she always had a glass of wine with dinner before she became pregnant. She has abstained during her first trimester and would like to know if it is safe for her to have a drink with dinner now. The nurse would tell her: a. “Since you're in your second trimester, there's no problem with having one drink with dinner.” b. “One drink every night is too much. One drink three times a week should be fine.” c. “Since you're in your second trimester, you can drink as much as you like.” d. “Because no one knows how much or how little alcohol it takes to cause fetal problems, the best course is to abstain throughout your pregnancy.” ANS: D The statement “Because no one knows how much or how little alcohol it takes to cause fetal problems, the best course is to abstain throughout your pregnancy” is accurate. A safe level of alcohol consumption during pregnancy has not yet been established. Although the consumption of occasional alcoholic beverages may not be harmful to the mother or her developing fetus, complete abstinence is strongly advised. PTS: 1 DIF: Cognitive Level: Application OBJ: Nursing Process: Planning MSC: Client Needs: Health Promotion and Maintenance 8. A pregnant woman at 18 weeks of gestation calls the clinic to report that she has been experiencing occasional backaches of mild-to-moderate intensity. The nurse would recommend that she: a. do Kegel exercises. b. do pelvic rock exercises. c. use a softer mattress. d. stay in bed for 24 hours. ANS: B Pelvic rock exercises may help stretch and strengthen the abdominal and lower back muscles and relieve low back pain. Kegel eNxUeRrcSiIsNeGs TinBc.CreOaMse the tone of the pelvic area, not the back. A softer mattress may not provide the support needed to maintain proper alignment of the spine and may contribute to back pain. Stretching and other exercises to relieve back pain should be performed several times a day. PTS: 1 DIF: Cognitive Level: Application OBJ: Nursing Process: Assessment MSC: Client Needs: Health Promotion and Maintenance 9. A woman is 3 months pregnant. At her prenatal visit, she tells the nurse that she does not know what is happening; one minute she's happy that she is pregnant, and the next minute she cries for no reason. Which response by the nurse is most appropriate? a. “Don't worry about it; you'll feel better in a month or so.” b. “Have you talked to your husband about how you feel?” c. “Perhaps you really don't want to be pregnant.” d. “Hormonal changes during pregnancy commonly result in mood swings.” ANS: D The statement “Hormonal changes during pregnancy commonly result in mood swings” is accurate and the most appropriate response by the nurse. The statement “Don't worry about it; you'll feel better in a month or so” dismisses the patient's concerns and is not the most appropriate response. Although women should be encouraged to share their feelings, “Have you talked to your husband about how you feel” is not the most appropriate response and does not provide the patient with a rationale for the psychosocial dynamics of her pregnancy. “Perhaps you really don't want to be pregnant” is completely inappropriate and deleterious to the psychologic well-being of the woman. Hormonal and metabolic adaptations often cause mood swings in pregnancy. The woman's responses are normal. She should be reassured about her feelings. PTS: 1 DIF: Cognitive Level: Application OBJ: Nursing Process: Diagnosis MSC: Client Needs: Psychosocial Integrity 10. The nurse should be aware that the partner's main role in pregnancy is to: a. provide financial support. b. protect the pregnant woman from “old wives' tales.” c. support and nurture the pregnant woman. d. make sure the pregnant woman keeps prenatal appointments. ANS: C The partner's main role in pregnancy is to nurture the pregnant woman and to respond her feelings of vulnerability. In older societies, the man enacted the ritual couvade. Changing cultural and professional attitudes have encouraged fathers' participation in the birth experience over the past 30 years. PTS: 1 DIF: Cognitive Level: Comprehension OBJ: Nursing Process: AssessmentNURSINMGSTCB.:CCOlMient Needs: Psychosocial Integrity 11. During the first trimester, a woman can expect which of the following changes in her sexual desire? a. An increase, because of enlarging breasts b. A decrease, because of nausea and fatigue c. No change d. An increase, because of increased levels of female hormones ANS: B Maternal physiologic changes such as breast enlargement, nausea, fatigue, abdominal changes, perineal enlargement, leukorrhea, pelvic vasocongestion, and orgasmic responses may affect sexuality and sexual expression. Libido may be depressed in the first trimester but often increases during the second and third trimesters. During pregnancy, the breasts may become enlarged and tender; this tends to interfere with coitus, decreasing the desire to engage in sexual activity. PTS: 1 DIF: Cognitive Level: Comprehension OBJ: Nursing Process: Diagnosis MSC: Client Needs: Health Promotion and Maintenance 12. Which behavior indicates that a woman is “seeking safe passage” for herself and her infant? a. She keeps all prenatal appointments. b. She “eats for two.” c. She drives her car slowly. d. She wears only low-heeled shoes. ANS: A The goal of prenatal care is to foster a safe birth for the infant and mother. Although eating properly, driving carefully, and using proper body mechanics all are healthy measures that a mother can take, obtaining prenatal care is the optimal method for providing safety for both herself and her baby. PTS: 1 DIF: Cognitive Level: Comprehension OBJ: Nursing Process: Evaluation MSC: Client Needs: Health Promotion and Maintenance 13. A 3-year-old girl's mother is 6 months pregnant. What concern is this child likely to verbalize? a. How the baby will “get out”? b. What the baby will eat? c. Whether her mother will die? d. What color eyes the baby has? ANS: B By age 3 or 4, children like to be told the story of their own beginning and accept its comparison with the present pregnancy. They like to listen to the fetal heartbeat and feel the baby move. Sometimes they worry about how the baby is being fed and what it wears. School-age children take a more clinical interest in their mother's pregnancy and may want to know, “How did the baby get in there?” and “How will it get out?” Whether her mother will die does not tend to be the focus of a child's questions about the impending birth of a sibling. The baby's eye color does not tend to be the focus of children's questions about the impending birth of a sibling. NURSINGTB.COM PTS: 1 DIF: Cognitive Level: Comprehension OBJ: Nursing Process: Assessment MSC: Client Needs: Health Promotion and Maintenance 14. In her work with pregnant women of various cultures, a nurse practitioner has observed various practices that seemed strange or unusual. She has learned that cultural rituals and practices during pregnancy seem to have one purpose in common. Which statement best describes that purpose? a. To promote family unity b. To ward off the “evil eye” c. To appease the gods of fertility d. To protect the mother and fetus during pregnancy ANS: D The purpose of all cultural practices is to protect the mother and fetus during pregnancy. Although many cultures consider pregnancy normal, certain practices are expected of women of all cultures to ensure a good outcome. Cultural prescriptions tell women what to do, and cultural proscriptions establish taboos. The purposes of these practices are to prevent maternal illness resulting from a pregnancy-induced imbalanced state and to protect the vulnerable fetus. PTS: 1 DIF: Cognitive Level: Comprehension OBJ: Nursing Process: Assessment MSC: Client Needs: Psychosocial Integrity 15. What type of cultural concern is the most likely deterrent to many women seeking prenatal care? a. Religion b. Modesty c. Ignorance d. Belief that physicians are evil ANS: B A concern for modesty is a deterrent to many women seeking prenatal care. For some women, exposing body parts, especially to a man, is considered a major violation of their modesty. Many cultural variations are found in prenatal care. Even if the prenatal care described is familiar to a woman, some practices may conflict with the beliefs and practices of a subculture group to which she belongs. PTS: 1 DIF: Cognitive Level: Analysis OBJ: Nursing Process: Evaluation MSC: Client Needs: Psychosocial Integrity 16. With regard to a woman's reordering of personal relationships during pregnancy, the maternity nurse should understand that: a. because of the special motherhood bond, a woman's relationship with her mother is even more important than with the father of the child. b. nurses need not get involved in any sexual issues the couple has during pregnancy, particularly if they have trouble communicating them to each other. c. women usually express two major relationship needs during pregnancy: feeling loved and valued and having the child accepted by the father. d. the woman's sexual desire is likely to be highest in the first trimester because of the excitement and because inNteUrcRoSuINrsGeTiBs .pChOyMsically easier. ANS: C Love and support help a woman feel better about her pregnancy. The most important person to the pregnant woman is usually the father. Nurses can facilitate communication between partners about sexual matters if, as is common, they are nervous about expressing their worries and feelings. The second trimester is the time when a woman's sense of well-being, along with certain physical changes, increases her desire for sex. Desire is decreased in the first and third trimesters. PTS: 1 DIF: Cognitive Level: Comprehension OBJ: Nursing Process: Planning MSC: Client Needs: Psychosocial Integrity 17. What represents a typical progression through the phases of a woman's establishing a relationship with the fetus? a. Accepts the fetus as distinct from herself—accepts the biologic fact of pregnancy—has a feeling of caring and responsibility. b. Fantasizes about the child's gender and personality—views the child as part of herself—becomes introspective. c. Views the child as part of herself—has feelings of well-being—accepts the biologic fact of pregnancy. d. “I am pregnant.”—“I am going to have a baby.”—“I am going to be a mother.” ANS: D The woman first centers on herself as pregnant, then on the baby as an entity separate from herself, and then on her responsibilities as a mother. The expressions, “I am pregnant,” “I am going to have a baby,” and “I am going to be a mother” sum up the progression through the three phases. PTS: 1 DIF: Cognitive Level: Application OBJ: Nursing Process: Diagnosis MSC: Client Needs: Psychosocial Integrity 18. As relates to the father's acceptance of the pregnancy and preparation for childbirth, the maternity nurse should know that: a. the father goes through three phases of acceptance of his own. b. the father's attachment to the fetus cannot be as strong as that of the mother because it does not start until after birth. c. in the last 2 months of pregnancy, most expectant fathers suddenly get very protective of their established lifestyle and resist making changes to the home. d. typically men remain ambivalent about fatherhood right up to the birth of their child. ANS: A A father typically goes through three phases of development to reach acceptance of fatherhood: the announcement phase, the moratorium phase, and the focusing phase. The father-child attachment can be as strong as the mother-child relationship and can also begin during pregnancy. In the last 2 months of pregnancy, many expectant fathers work hard to improve the environment of the home for the child. Typically, the expectant father's ambivalence ends by the first trimester, and he progresses to adjusting to the reality of the situation and then to focusing on his role. NURSINGTB.COM PTS: 1 DIF: Cognitive Level: Comprehension OBJ: Nursing Process: Diagnosis MSC: Client Needs: Psychosocial Integrity 19. With regard to the initial visit with a patient who is beginning prenatal care, nurses should be aware that: a. the first interview is a relaxed, get-acquainted affair in which nurses gather some general impressions. b. if nurses observe handicapping conditions, they should be sensitive and not enquire about them because the patient will do that in her own time. c. nurses should be alert to the appearance of potential parenting problems, such as depression or lack of family support. d. because of legal complications, nurses should not ask about illegal drug use; that is left to physicians. ANS: C Besides these potential problems, nurses need to be alert to the woman's attitude toward health care. The initial interview needs to be planned, purposeful, and focused on specific content. A lot of ground must be covered. Nurses must be sensitive to special problems, but they do need to inquire because discovering individual needs is important. People with chronic or handicapping conditions forget to mention them because they have adapted to them. Getting information on drug use is important and can be done confidentially. Actual testing for drug use requires the patient's consent. PTS: 1 DIF: Cognitive Level: Comprehension OBJ: Nursing Process: Assessment MSC: Client Needs: Psychosocial Integrity 20. With regard to follow-up visits for women receiving prenatal care, nurses should be aware that: a. the interview portions become more intensive as the visits become more frequent over the course of the pregnancy. b. monthly visits are scheduled for the first trimester, every 2 weeks for the second trimester, and weekly for the third trimester. c. during the abdominal examination, the nurse should be alert for supine hypotension. d. for pregnant women, a systolic blood pressure (BP) of 130 and a diastolic BP of 80 is sufficient to be considered hypertensive. ANS: C The woman lies on her back during the abdominal examination, possibly compressing the vena cava and aorta, which can cause a decrease in blood pressure and a feeling of faintness. The interview portion of follow-up examinations is less extensive than in the initial prenatal visits, during which so much new information must be gathered. Monthly visits are routinely scheduled for the first and second trimesters; visits increase to every 2 weeks at week 28 and to once a week at week 36. For pregnant women hypertension is defined as a systolic BP of 140 or greater and a diastolic BP of 90 or greater. PTS: 1 DIF: Cognitive Level: Comprehension OBJ: Nursing Process: Assessment MSC: Client Needs: Physiologic Integrity 21. While teaching the expectant mother about personal hygiene during pregnancy, maternity nurses should be aware that: a. tub bathing is permitted even iNnUlRatSeINpGreTgBn.aCnOcMy unless membranes have ruptured. b. the perineum should be wiped from back to front. c. bubble bath and bath oils are permissible because they add an extra soothing and cleansing action to the bath. d. expectant mothers should use specially treated soap to cleanse the nipples. ANS: A The main danger from taking baths is falling in the tub. The perineum should be wiped from front to back. Bubble baths and bath oils should be avoided because they may irritate the urethra. Soap, alcohol, ointments, and tinctures should not be used to cleanse the nipples because they remove protective oils. Warm water is sufficient. PTS: 1 DIF: Cognitive Level: Knowledge OBJ: Nursing Process: Planning MSC: Client Needs: Physiologic Integrity 22. To provide the patient with accurate information about dental care during pregnancy, maternity nurses should be aware that: a. dental care can be dropped from the priority list because the woman has enough to worry about and is getting a lot of calcium anyway. b. dental surgery, in particular, is contraindicated because of the psychologic stress it engenders. c. if dental treatment is necessary, the woman will be most comfortable with it in the second trimester. d. dental care interferes with the expectant mother's need to practice conscious relaxation. ANS: C The second trimester is best for dental treatment because that is when the woman will be able to sit most comfortably in the dental chair. Dental care such as brushing with fluoride toothpaste is especially important during pregnancy because nausea during pregnancy may lead to poor oral hygiene. Emergency dental surgery is permissible, but the mother must clearly understand the risks and benefits. Conscious relaxation is useful, and it may even help the woman get through any dental appointments; it is not a reason to avoid them. PTS: 1 DIF: Cognitive Level: Comprehension OBJ: Nursing Process: Planning MSC: Client Needs: Physiologic Integrity 23. When discussing work and travel during pregnancy with a pregnant patient, nurses should instruct them that: a. women should sit for as long as possible and cross their legs at the knees from time to time for exercise. b. women should avoid seat belts and shoulder restraints in the car because they press on the fetus. c. metal detectors at airport security checkpoints can harm the fetus if the woman passes through them a number of times. d. while working or traveling in a car or on a plane, women should arrange to walk around at least every 2 hours or so. ANS: D Periodic walking helps prevent thrombophlebitis. Pregnant women should avoid sitting or standing for long periods and crossing the legs at the knees. Pregnant women must wear lap belts and shoulder restraints. The NmUoRstSIcNoGmTmBo.CnOiMnjury to the fetus comes from injury to the mother. Metal detectors at airport security checkpoints do not harm fetuses. PTS: 1 DIF: Cognitive Level: Comprehension OBJ: Nursing Process: Planning MSC: Client Needs: Health Promotion and Maintenance 24. With regard to medications, herbs, shots, and other substances normally encountered by pregnant women, the maternity nurse should be aware that: a. both prescription and over-the-counter (OTC) drugs that otherwise are harmless can be made hazardous by metabolic deficiencies of the fetus. b. the greatest danger of drug-caused developmental deficits in the fetus is seen in the final trimester. c. killed-virus vaccines (e.g., tetanus) should not be given during pregnancy, but live-virus vaccines (e.g., measles) are permissible. d. no convincing evidence exists that secondhand smoke is potentially dangerous to the fetus. ANS: A Both prescription and OTC drugs that otherwise are harmless can be made hazardous by metabolic deficiencies of the fetus. This is especially true for new medications and combinations of drugs. The greatest danger of drug-caused developmental defects exists in the interval from fertilization through the first trimester, when a woman may not realize that she is pregnant. Live-virus vaccines should be part of after birth care; killed-virus vaccines may be administered during pregnancy. Secondhand smoke is associated with fetal growth restriction and increases in infant mortality. PTS: 1 DIF: Cognitive Level: Comprehension OBJ: Nursing Process: Assessment MSC: Client Needs: Health Promotion and Maintenance 25. Which statement about multifetal pregnancy is inaccurate? a. The expectant mother often develops anemia because the fetuses have a greater demand for iron. b. Twin pregnancies come to term with the same frequency as single pregnancies. c. The mother should be counseled to increase her nutritional intake and gain more weight. d. Backache and varicose veins often are more pronounced. ANS: B Twin pregnancies often end in prematurity. Serious efforts should be made to bring the pregnancy to term. A woman with a multifetal pregnancy often develops anemia, suffers more or worse backache, and needs to gain more weight. Counseling is needed to help her adjust to these conditions. PTS: 1 DIF: Cognitive Level: Comprehension OBJ: Nursing Process: Assessment MSC: Client Needs: Health Promotion and Maintenance NURSINGTB.COM 26. The phenomenon of someone other than the mother-to-be experiencing pregnancy-like symptoms such as nausea and weight gain applies to the: a. mother of the pregnant woman. b. couple's teenage daughter. c. sister of the pregnant woman. d. expectant father. ANS: D An expectant father's experiencing pregnancy-like symptoms is called the couvade syndrome. PTS: 1 DIF: Cognitive Level: Knowledge OBJ: Nursing Process: Diagnosis MSC: Client Needs: Psychosocial Integrity 27. In response to requests by the U.S. Public Health Service for new models of prenatal care, an innovative new approach to prenatal care known as centering pregnancy was developed. Which statement would accurately apply to the centering model of care? a. Group sessions begin with the first prenatal visit. b. At each visit, blood pressure, weight, and urine dipsticks are obtained by the nurse. c. Eight to twelve women are placed in gestational-age cohort groups. d. Outcomes are similar to those of traditional prenatal care. ANS: C Gestational-age cohorts comprise the groups with approximately 8 to 12 women in each group. This group remains intact throughout the pregnancy. Individual follow-up visits are scheduled as needed. Group sessions begin at 12 to 16 weeks of gestation and end with an early after birth visit. Before group sessions the patient has an individual assessment, physical examination, and history. At the beginning of each group meeting, patients measure their own blood pressure, weight, and urine dips and enter these in their record. Fetal heart rate assessment and fundal height are obtained by the nurse. Results evaluating this approach have been very promising. In a study of adolescent patients, there was a decrease in low-birth-weight infants and an increase in breastfeeding rates. PTS: 1 DIF: Cognitive Level: Application OBJ: Nursing Process: Implementation MSC: Client Needs: Health Promotion and Maintenance 28. While you are assessing the vital signs of a pregnant woman in her third trimester, the patient complains of feeling faint, dizzy, and agitated. Which nursing intervention is appropriate? a. Have the patient stand up and retake her blood pressure. b. Have the patient sit down and hold her arm in a dependent position. c. Have the patient lie supine for 5 minutes and recheck her blood pressure on both arms. d. Have the patient turn to her left side and recheck her blood pressure in 5 minutes. ANS: D Blood pressure is affected by maternal position during pregnancy. The supine position may cause occlusion of the vena cava and descending aorta. Turning the pregnant woman to a lateral recumbent position alleviates pressure on the blood vessels and quickly corrects supine hypotension. Pressures are significantly higher when the patient is standing. This option causes an increase in systolic and NdUiaRstSoINlicGTpBre.CssOuMres. The arm should be supported at the same level of the heart. The supine position may cause occlusion of the vena cava and descending aorta, creating hypotension. PTS: 1 DIF: Cognitive Level: Analysis OBJ: Nursing Process: Implementation MSC: Client Needs: Physiologic Integrity MULTIPLE RESPONSE 1. Signs and symptoms that a woman should report immediately to her health care provider include: (Select all that apply.) a. vaginal bleeding. b. rupture of membranes. c. heartburn accompanied by severe headache. d. decreased libido. e. Urinary frequency. ANS: A, B, C Vaginal bleeding, rupture of membranes, and severe headaches all are signs of potential complications in pregnancy. Patients should be advised to report these signs to the health care provider. Decreased libido and urinary frequency are common discomforts of pregnancy that do not require immediate health care interventions. PTS: 1 DIF: Cognitive Level: Analysis OBJ: Nursing Process: Planning, Implementation MSC: Client Needs: Physiologic Integrity 2. A woman has just moved to the United States from Mexico. She is 3 months pregnant and has arrived for her first prenatal visit. During her assessment interview, you discover that she has not had any immunizations. Which immunizations should she receive at this point in her pregnancy? (Select all that apply.) a. Tetanus b. Diphtheria c. Chickenpox d. Rubella e. Hepatitis B ANS: A, B, E Immunization with live or attenuated live viruses is contraindicated during pregnancy because of potential teratogenicity. Vaccines consisting of killed viruses may be used. Immunizations that may be administered during pregnancy include tetanus, diphtheria, recombinant hepatitis B, and rabies vaccines. Live-virus vaccines include those for measles (rubeola and rubella), chickenpox, and mumps. PTS: 1 DIF: Cognitive Level: Analysis OBJ: Nursing Process: Implementation MSC: Client Needs: Health Promotion and Maintenance COMPLETION 1. A woman arrives at the clinic for a pregnancy test. The first day of her last menstrual period (LMP) was September 10, 2013. Her expected date of birth (EDB) would be? NURSINGTB.COM ANS: June 17, 2014 Using Nägele's rule, June 17, 2014, is the correct EDB. The EDB is calculated by subtracting 3 months from the first day of the LMP and adding 7 days + 1 year to the day of the LMP. Therefore, with an LMP of September 10, 2013: September 10, 2013  3 months = June 10, 2013 + 7 days = June 17, 2013 + 1 year = June 17, 2014 PTS: 1 DIF: Cognitive Level: Knowledge OBJ: Nursing Process: Assessment MSC: Client Needs: Health Promotion and Maintenance MATCHING All pregnant women should be instructed to recognize and report potential complications for each trimester of pregnancy. Match the sign or symptom with a possible cause. a. Severe vomiting in early pregnancy b. Epigastric pain in late pregnancy c. Severe backache and flank pain d. Decreased fetal movement e. Glycosuria 1. Fetal jeopardy or intrauterine fetal death 2. Kidney infection or stones 3. Gestational diabetes 4. Hyperemesis gravidarum 5. Hypertension, preeclampsia 1. ANS: D PTS: 1 DIF: Cognitive Level: Comprehension OBJ: Nursing Process: Implementation MSC: Client Needs: Health Promotion and Maintenance NOT: It is essential for the nurse to plan education needed by the pregnant woman to recognize and report these potential complications a timely manner. A trusting relationship contributes to a positive outcome for the pregnancy. 2. ANS: C PTS: 1 DIF: Cognitive Level: Comprehension OBJ: Nursing Process: Implementation MSC: Client Needs: Health Promotion and Maintenance NOT: It is essential for the nurse to plan education needed by the pregnant woman to recognize and report these potential complications a timely manner. A trusting relationship contributes to a positive outcome for the pregnancy. 3. ANS: E PTS: 1 DIF: Cognitive Level: Comprehension OBJ: Nursing Process: Implementation MSC: Client Needs: Health Promotion and Maintenance NOT: It is essential for the nurse to plan education needed by the pregnant woman to recognize and report these potential complications a timely manner. A trusting relationship contributes to a positive outcome for the pregnancy. 4. ANS: A PTS: 1 DIF: Cognitive Level: Comprehension OBJ: Nursing Process: Implementation MSC: Client Needs: Health Promotion and Maintenance NOT: It is essential for the nurse to plan education needed by the pregnant woman to recognize and report these potential complications a timely manner. A trusting relationship contributes to a positive outcome for the pregnancy. 5. ANS: B PTS: 1 DIF: Cognitive Level: Comprehension OBJ: Nursing Process: ImplementaNtioUnRSINMGSTCB.:CCOlMient Needs: Health Promotion and Maintenance NOT: It is essential for the nurse to plan education needed by the pregnant woman to recognize and report these potential complications a timely manner. A trusting relationship contributes to a positive outcome for the pregnancy. Chapter 09: Maternal and Fetal Nutrition Perry: Maternal Child Nursing Care, 6th Edition MULTIPLE CHOICE 1. A 22-year-old woman pregnant with a single fetus has a preconception body mass index (BMI) of 24. When she was seen in the clinic at 14 weeks of gestation, she had gained 1.8 kg (4 lbs) since conception. How would the nurse interpret this? a. This weight gain indicates possible gestational hypertension. b. This weight gain indicates that the woman's infant is at risk for intrauterine growth restriction (IUGR). c. This weight gain cannot be evaluated until the woman has been observed for several more weeks. d. The woman's weight gain is appropriate for this stage of pregnancy. ANS: D The statement “The woman's weight gain is appropriate for this stage of pregnancy” is accurate. This woman's BMI is within the normal range. During the first trimester, the average total weight gain is only 1 to 2 kg. Although weight gain does indicate possible gestational hypertension, it does not apply to this patient. The desirable weight gain during pregnancy varies among women. The primary factor to consider in making a weight gain recommendation is the appropriateness of the prepregnancy weight for the woman's height. A commonly used method of evaluating the appropriateness of weight for height is the BMI. Although weight gain does indicate risk for IUGR, this does not apply to this patient. Weight gain should occur at a steady rate throughout the pregnancy. The optimal rate of weight gain also depends on the stage of the pNreUgRnSaInNcGy.TB.COM PTS: 1 DIF: Cognitive Level: Analysis OBJ: Nursing Process: Assessment MSC: Client Needs: Health Promotion and Maintenance 2. Which meal would provide the most absorbable iron? a. Toasted cheese sandwich, celery sticks, tomato slices, and a grape drink b. Oatmeal, whole wheat toast, jelly, and low-fat milk c. Black bean soup, wheat crackers, orange sections, and prunes d. Red beans and rice, cornbread, mixed greens, and decaffeinated tea ANS: C Food sources that are rich in iron include liver, meats, whole grain or enriched breads and cereals, deep green leafy vegetables, legumes, and dried fruits. In addition, the vitamin C in orange sections aids absorption. Dairy products and tea are not sources of iron. PTS: 1 DIF: Cognitive Level: Comprehension OBJ: Nursing Process: Planning MSC: Client Needs: Physiologic Integrity 3. A pregnant woman's diet consists almost entirely of whole grain breads and cereals, fruits, and vegetables. The nurse would be most concerned about this woman's intake of: a. calcium. b. protein. c. vitamin B12. d. folic acid. ANS: C This diet is consistent with that followed by a strict vegetarian (vegan). Vegans consume only plant products. Because vitamin B12 is found in foods of animal origin, this diet is deficient in vitamin B12. PTS: 1 DIF: Cognitive Level: Knowledge OBJ: Nursing Process: Assessment MSC: Client Needs: Physiologic Integrity 4. A pregnant woman experiencing nausea and vomiting should: a. drink a glass of water with a fat-free carbohydrate before getting out of bed in the morning. b. eat small, frequent meals (every 2 to 3 hours). c. increase her intake of high-fat foods to keep the stomach full and coated. d. limit fluid intake throughout the day. ANS: B Eating small, frequent meals is the correct suggestion for a woman experiencing nausea and vomiting. A pregnant woman experiencing nausea and vomiting should avoid consuming fluids early in the day or when nauseated, but should compensate by drinking fluids at other times. A pregnant woman experiencing nausea and vomiting should reduce her intake of fried and other fatty foods. PTS: 1 DIF: Cognitive Level: Comprehension OBJ: Nursing Process: Planning MSC: Client Needs: Physiologic Integrity 5. A pregnant woman reports that she is still playing tennis at 32 weeks of gestation. The nurse would be most concerned that durNinUgRaSnINdGaTftBe.rCtOeMnnis matches this woman consumes: a. several glasses of fluid. b. extra protein sources such as peanut butter. c. salty foods to replace lost sodium. d. easily digested sources of carbohydrate. ANS: A If no medical or obstetric problems contraindicate physical activity, pregnant women should get 30 minutes of moderate physical exercise daily. Liberal amounts of fluid should be consumed before, during, and after exercise because dehydration can trigger premature labor. The woman's calorie intake should be sufficient to meet the increased needs of pregnancy and the demands of exercise. PTS: 1 DIF: Cognitive Level: Comprehension OBJ: Nursing Process: Planning MSC: Client Needs: Physiologic Integrity 6. Which statement made by a lactating woman would lead the nurse to believe that the woman might have lactose intolerance? a. “I always have heartburn after I drink milk.” b. “If I drink more than a cup of milk, I usually have abdominal cramps and bloating.” c. “Drinking milk usually makes me break out in hives.” d. “Sometimes I notice that I have bad breath after I drink a cup of milk.” ANS: B Abdominal cramps and bloating are consistent with lactose intolerance. One problem that can interfere with milk consumption is lactose intolerance, which is the inability to digest milk sugar because of a lack of the enzyme lactase in the small intestine. Milk consumption may cause abdominal cramping, bloating, and diarrhea in people who are lactose intolerant, although many affected individuals can tolerate small amounts of milk without symptoms. PTS: 1 DIF: Cognitive Level: Application OBJ: Nursing Process: Assessment MSC: Client Needs: Physiologic Integrity 7. A pregnant woman's diet history indicates that she likes the following list of foods. The nurse would encourage this woman to consume more of which food to increase her calcium intake? a. Fresh apricots b. Canned clams c. Spaghetti with meat sauce d. Canned sardines ANS: D Sardines are rich in calcium. Fresh apricots, canned clams, and spaghetti with meat sauce are not high in calcium. PTS: 1 DIF: Cognitive Level: Comprehension OBJ: Nursing Process: Planning MSC: Client Needs: Physiologic Integrity 8. A 27-year-old pregnant woman had a preconceptual body mass index (BMI) of 18.0. The nurse knows that this woman's total recommended weight gain during pregnancy should be at least: a. 20 kg (44 lbs). b. 16 kg (35 lbs). c. 12.5 kg (27.5 lbs). d. 10 kg (22 lbs). ANS: C NURSINGTB.COM This woman has a normal BMI and should gain 11.5 to 16 kg during pregnancy. A weight gain of 20 kg would be unhealthy for most women. A weight gain 35 lbs is the high end of the range of weight this woman should gain in her pregnancy. A weight gain of 22 lbs would be appropriate for an obese woman. PTS: 1 DIF: Cognitive Level: Comprehension OBJ: Nursing Process: Planning MSC: Client Needs: Health Promotion and Maintenance 9. A woman in week 34 of pregnancy reports that she is very uncomfortable because of heartburn. The nurse would suggest that the woman: a. substitute other calcium sources for milk in her diet. b. lie down after each meal. c. reduce the amount of fiber she consumes. d. eat five small meals daily. ANS: D Eating small, frequent meals may help with heartburn, nausea, and vomiting. Substituting other calcium sources for milk, lying down after eating, and reducing fiber intake are inappropriate dietary suggestions for all pregnant women and do not alleviate heartburn. PTS: 1 DIF: Cognitive Level: Comprehension OBJ: Nursing Process: Planning MSC: Client Needs: Physiologic Integrity 10. A woman has come to the clinic for preconception counseling because she wants to start trying to get pregnant in 3 months. She can expect the following advice: a. “Discontinue all contraception now.” b. “Lose weight so that you can gain more during pregnancy.” c. “You may take any medications you have been taking regularly.” d. “Make sure that you include adequate folic acid in your diet.” ANS: D A healthy diet before conception is the best way to ensure that adequate nutrients are available for the developing fetus. A woman's folate or folic acid intake is of particular concern in the periconception period. Neural tube defects are more common in infants of women with a poor folic acid intake. Depending on the type of contraception used, discontinuing all contraception may not be appropriate advice. Losing weight is not appropriate advice. Depending on the type of medication the woman is taking, continuing its use may not be appropriate. PTS: 1 DIF: Cognitive Level: Application OBJ: Nursing Process: Planning MSC: Client Needs: Health Promotion and Maintenance 11. To prevent gastrointestinal upset, patients should be instructed to take iron supplements: a. on a full stomach. b. at bedtime. c. after eating a meal. d. with milk. ANS: B NURSINGTB.COM Patients should be instructed to take iron supplements at bedtime. Iron supplements are best absorbed if they are taken when the stomach is empty. Bran, tea, coffee, milk, and eggs may reduce absorption. Iron can be taken at bedtime if abdominal discomfort occurs when it is taken between meals. PTS: 1 DIF: Cognitive Level: Application OBJ: Nursing Process: Planning MSC: Client Needs: Physiologic Integrity 12. Women with an inadequate weight gain during pregnancy are at higher risk of giving birth to an infant with: a. spina bifida. b. intrauterine growth restriction. c. diabetes mellitus. d. Down syndrome. ANS: B Both normal-weight and underweight women with inadequate weight gain have an increased risk of giving birth to an infant with intrauterine growth restriction. Spina bifida, diabetes mellitus, and Down syndrome are not associated with inadequate maternal weight gain. PTS: 1 DIF: Cognitive Level: Comprehension OBJ: Nursing Process: Assessment MSC: Client Needs: Health Promotion and Maintenance 13. After you complete your nutritional counseling for a pregnant woman, you ask her to repeat your instructions so you can assess her understanding of the instructions given. Which statement indicates that she understands the role of protein in her pregnancy? a. “Protein will help my baby grow.” b. “Eating protein will prevent me from becoming anemic.” c. “Eating protein will make my baby have strong teeth after he/she is born.” d. “Eating protein will prevent me from being diabetic.” ANS: A Protein is the nutritional element basic to growth. An adequate protein intake is essential to meeting the increasing demands of pregnancy. These demands arise from the rapid growth of the fetus; the enlargement of the uterus, mammary glands, and placenta; the increase in the maternal blood volume; and the formation of amniotic fluid. Iron intake prevents anemia. Calcium intake is needed for fetal bone and tooth development. Glycemic control is needed in diabetics; protein is one nutritional factor to consider, but this is not the primary role of protein intake. PTS: 1 DIF: Cognitive Level: Application OBJ: Nursing Process: Evaluation MSC: Client Needs: Health Promotion and Maintenance 14. Maternal nutritional status is an especially significant factor of the many factors that influence the outcome of pregnancy because: a. it is very difficult to adjust because of people's ingrained eating habits. b. it is an important preventive measure for a variety of problems. c. women love obsessing about their weight and diets. d. a woman's preconception weight becomes irrelevant. ANS: B NURSINGTB.COM Nutritional status draws so much attention not only for its effect on a healthy pregnancy and birth but also because significant changes are within relatively easy reach. PTS: 1 DIF: Cognitive Level: Comprehension OBJ: Nursing Process: Evaluation MSC: Client Needs: Physiologic Integrity 15. Which statement regarding acronyms in nutrition is accurate? a. Dietary reference intakes (DRIs) consist of recommended dietary allowances (RDAs), adequate intakes (AIs), and upper limits (ULs). b. RDAs are the same as ULs except with better data. c. AIs offer guidelines for avoiding excessive amounts of nutrients. d. They all refer to green leafy vegetables, whole grains, and fruit. ANS: A DRIs consist of RDAs, AIs, and ULs. AIs are similar to RDAs except that they deal with nutrients about which data are insufficient for certainty (RDA status). ULs are guidelines for avoiding excesses of nutrients for which excess is toxic. Green leafy vegetables, whole grains, and fruit are important, but they are not the whole nutritional story. PTS: 1 DIF: Cognitive Level: Knowledge OBJ: Nursing Process: Assessment MSC: Client Needs: Physiologic Integrity 16. With regard to protein in the diet of pregnant women, nurses should be aware that: a. many protein-rich foods are also good sources of calcium, iron, and B vitamins. b. many women need to increase their protein intake during pregnancy. c. as with carbohydrates and fat, no specific recommendations exist for the amount of protein in the diet. d. high-protein supplements can be used without risk by women on macrobiotic diets. ANS: A Good protein sources such as meat, milk, eggs, and cheese have a lot of calcium and iron. Most women already eat a high-protein diet and do not need to increase their intake. Protein is sufficiently important that specific servings of meat and dairy are recommended. High-protein supplements are not recommended because they have been associated with an increased incidence of preterm births. PTS: 1 DIF: Cognitive Level: Knowledge OBJ: Nursing Process: Planning MSC: Client Needs: Physiologic Integrity 17. Which nutritional recommendation about fluids is accurate? a. A woman's daily intake should be 8 to 10 glasses (2.3 L) of water, milk, or juice. b. Coffee should be limited to no more than two cups, but tea and cocoa can be consumed without worry. c. Of the artificial sweeteners, only aspartame has been not associated with any maternity health concerns. d. Water with fluoride is especially encouraged because it reduces the child's risk of tooth decay. ANS: A Eight to ten glasses is the standard for fluids; however, they should be the right fluids. All beverages containing caffeine, including tea, cocoa, and some soft drinks should be avoided or drunk only in limited amounts.NAUrRtiSfiIcNiGalTsBw.CeOeMteners, including aspartame, have no ill effects on the normal mother or fetus; however, mothers with phenylketonuria should avoid aspartame. No evidence indicates that prenatal fluoride consumption reduces childhood tooth decay. PTS: 1 DIF: Cognitive Level: Comprehension OBJ: Nursing Process: Planning MSC: Client Needs: Physiologic Integrity 18. Which minerals and vitamins usually are recommended to supplement a pregnant woman's diet? a. Fat-soluble vitamins A and D b. Water-soluble vitamins C and B6 c. Iron and folate d. Calcium and zinc ANS: C Iron generally should be supplemented, and folic acid supplements often are needed because folate is so important. Fat-soluble vitamins should be supplemented as a medical prescription, as vitamin D might be for lactose-intolerant women. Water-soluble vitamin C sometimes is consumed in excess naturally; vitamin B6 is prescribed only if the woman has a very poor diet. Zinc sometimes is supplemented. Most women obtain enough calcium through their regular diet. PTS: 1 DIF: Cognitive Level: Application OBJ: Nursing Process: Planning MSC: Client Needs: Health Promotion and Maintenance 19. Which vitamins or minerals can lead to congenital malformations of the fetus if taken in excess by the mother? a. Zinc b. Vitamin D c. Folic acid d. Vitamin A ANS: D Zinc, vitamin D, and folic acid are vital to good maternal and fetal health and are highly unlikely to be consumed in excess. Vitamin A taken in excess causes a number of problems. An analog of vitamin A appears in prescribed acne medications, which must not be taken during pregnancy. PTS: 1 DIF: Cognitive Level: Comprehension OBJ: Nursing Process: Planning MSC: Client Needs: Health Promotion and Maintenance 20. With regard to nutritional needs during lactation, a maternity nurse should be aware that: a. the mother's intake of vitamin C, zinc, and protein now can be lower than during pregnancy. b. caffeine consumed by the mother accumulates in the infant, who may be unusually active and wakeful. c. critical iron and folic acid levels must be maintained. d. lactating women can go back to their prepregnant calorie intake. ANS: B A lactating woman needs to avoidNcUoRnSsIuNmGiTnBg.CtoOoMmuch caffeine. Vitamin C, zinc, and protein levels need to be moderately higher during lactation than during pregnancy. The recommendations for iron and folic acid are lower during lactation. Lactating women should consume about 500 kcal more than their prepregnancy intake, at least 1800 kcal daily overall. PTS: 1 DIF: Cognitive Level: Knowledge OBJ: Nursing Process: Planning MSC: Client Needs: Health Promotion and Maintenance 21. While taking a diet history, the nurse might be told that the expectant mother has cravings for ice chips, cornstarch, and baking soda. This represents a nutritional problem known as: a. preeclampsia. b. pyrosis. c. pica. d. purging. ANS: C The consumption of foods low in nutritional value or of nonfood substances (e.g., dirt, laundry starch) is called pica. PTS: 1 DIF: Cognitive Level: Knowledge OBJ: Nursing Process: Diagnosis MSC: Client Needs: Health Promotion and Maintenance 22. When counseling a patient about getting enough iron in her diet, the maternity nurse should tell her that: a. milk, coffee, and tea aid iron absorption if consumed at the same time as iron. b. iron absorption is inhibited by a diet rich in vitamin C. c. iron supplements are permissible for children in small doses. d. constipation is common with iron supplements. ANS: D Constipation can be a problem. Milk, coffee, and tea inhibit iron absorption when consumed at the same time as iron. Vitamin C promotes iron absorption. Children who ingest iron can get very sick and even die. PTS: 1 DIF: Cognitive Level: Knowledge OBJ: Nursing Process: Planning MSC: Client Needs: Physiologic Integrity 23. To help a woman reduce the severity of nausea caused by morning sickness, the nurse might suggest that she: a. try a tart food or drink such as lemonade or salty foods such as potato chips. b. drink plenty of fluids early in the day. c. brush her teeth immediately after eating. d. never snack before bedtime. ANS: A Some women can tolerate tart or salty foods when they are nauseous. The woman should avoid drinking too much when nausea is most likely, but she should make up the fluid levels later in the day when she feels better. The woman should avoid brushing her teeth immediately after eating. A small snack of cereal and milk or yogurt before bedtime may help the stomach in the morning. PTS: 1 DIF: Cognitive Level: Knowledge OBJ: Nursing Process: Planning MSC: Client Needs: Physiologic IntNegUriRtySINGTB.COM 24. Three servings of milk, yogurt, or cheese plus two servings of meat, poultry, or fish adequately supply the recommended amount of protein for a pregnant woman. Many patients are concerned about the increased levels of mercury in fish and may be afraid to include this source of nutrients in their diet. Sound advice by the nurse to assist the patient in determining which fish is safe to consume would include: a. canned white tuna is a preferred choice. b. avoid shark, swordfish, and mackerel. c. fish caught in local waterways are the safest. d. salmon and shrimp contain high levels of mercury. ANS: B As a precaution, the pregnant patient should avoid eating all of these and the less common tilefish. High levels of mercury can harm the developing nervous system of the fetus. It is essential for the nurse to assist the patient in understanding the differences between numerous sources of this product. A pregnant patient can take 12 ounces a week of canned light tuna; however, canned white, albacore, or tuna steaks contain higher levels of mercury and should be limited to no more than 6 ounces per week. It is a common misconception that fish caught in local waterways are the safest. Pregnant women and mothers of young children should check with local advisories about the safety of fish caught by families and friends in nearby bodies of water. If no information is available, these fish sources should be avoided, limited to less than 6 ounces, or the only fish consumed that week. Commercially caught fish that are low in mercury include salmon, shrimp, pollock, or catfish. PTS: 1 DIF: Cognitive Level: Comprehension OBJ: Nursing Process: Planning MSC: Client Needs: Health Promotion and Maintenance 25. Nutrition is one of the most significant factors influencing the outcome of a pregnancy. It is an alterable and important preventive measure for various potential problems, such as low birth weight and prematurity. While completing the physical assessment of the pregnant patient, the nurse can evaluate the patient's nutritional status by observing a number of physical signs. Which sign would indicate that the patient has unmet nutritional needs? a. Normal heart rate, rhythm, and blood pressure b. Bright, clear, shiny eyes c. Alert, responsive, and good endurance d. Edema, tender calves, and tingling ANS: D The physiologic changes of pregnancy may complicate the interpretation of physical findings. Lower-extremity edema often occurs when caloric and protein deficiencies are present; however, it may also be a common physical finding during the third trimester. It is essential that the nurse complete a thorough health history and physical assessment and request further laboratory testing if indicated. A malnourished pregnant patient may display rapid heart rate, abnormal rhythm, enlarged heart, and elevated blood pressure. A patient receiving adequate nutrition has bright, shiny eyes with no sores and moist, pink membranes. Pale or red membranes, dryness, infection, dull appearance of the cornea, or blue sclerae all are signs of poor nutrition. This patient is well nourished. Cachexia, listlessness, and tiring easily would be indications of poor nutritional status. PTS: 1 DIF: Cognitive Level: Analysis OBJ: Nursing Process: AssessmentNURSINMGSTCB.:CCOlMient Needs: Physiologic Integrity 26. Which pregnant woman should restrict her weight gain during pregnancy? a. Woman pregnant with twins b. Woman in early adolescence c. Woman shorter than 62 inches or 157 cm d. Woman who was 20 lbs overweight before pregnancy ANS: D A weight gain of 5 to 9 kg will provide sufficient nutrients for the fetus. Overweight and obese women should be advised to lose weight before conception to achieve the best pregnancy outcomes. A higher weight gain in twin gestations may help prevent low birth weights. Adolescents need to gain weight toward the higher acceptable range, which provides for their own growth as well as for fetal growth. In the past, women of short stature were advised to restrict their weight gain; however, evidence to support these guidelines has not been found. PTS: 1 DIF: Cognitive Level: Comprehension OBJ: Nursing Process: Assessment MSC: Client Needs: Health Promotion and Maintenance 27. The major source of nutrients in the diet of a pregnant woman should be composed of: a. simple sugars. b. fats. c. fiber. d. complex carbohydrates. ANS: D Complex carbohydrates supply the pregnant woman with vitamins, minerals, and fiber. The most common simple carbohydrate is table sugar, which is a source of energy but does not provide any nutrients. Fats provide 9 kcal in each gram, in contrast to carbohydrates and proteins, which provide only 4 kcal in each gram. Fiber is supplied primarily by complex carbohydrates. PTS: 1 DIF: Cognitive Level: Knowledge OBJ: Nursing Process: Assessment MSC: Client Needs: Physiologic Integrity 28. A pregnant woman's diet may not meet her need for folates. A good source of this nutrient is: a. chicken. b. cheese. c. potatoes. d. green leafy vegetables. ANS: D Sources of folates include green leafy vegetables, whole grains, fruits, liver, dried peas, and beans. Chicken and cheese are excellent sources of protein but are poor in folates. Potatoes contain carbohydrates and vitamins and minerals but are poor in folates. PTS: 1 DIF: Cognitive Level: Knowledge OBJ: Nursing Process: Assessment MSC: Client Needs: Physiologic Integrity 29. When providing care to the prenatal patient, the nurse understands that pica is defined as: a. intolerance of milk products. NURSINGTB.COM b. iron deficiency anemia. c. ingestion of nonfood substances. d. episodes of anorexia and vomiting. ANS: C The practice of eating substances not normally thought of as food is called pica. Clay or dirt and solid laundry starch are the substances most commonly ingested. Intolerance of milk products is referred to as lactose intolerance. Pica may produce iron deficiency anemia if proper nutrition is decreased. Pica is not related to anorexia and vomiting. PTS: 1 DIF: Cognitive Level: Knowledge OBJ: Nursing Process: Assessment MSC: Client Needs: Physiologic Integrity 30. The most important reason for evaluating the pattern of weight gain in pregnancy is to: a. prevent excessive adipose tissue deposits. b. identify potential nutritional problems or complications of pregnancy. c. assess the need to limit caloric intake in obese women. d. determine cultural influences on the woman's diet. ANS: B Maternal and fetal risks in pregnancy are increased when the mother is significantly overweight. Excessive adipose tissue may occur with excess weight gain; however, this is not the reason for monitoring the weight gain pattern. It is important to monitor the pattern of weight gain to identify complications. The pattern of weight gain is not influenced by cultural influences. PTS: 1 DIF: Cognitive Level: Comprehension OBJ: Nursing Process: Assessment MSC: Client Needs: Physiologic Integrity 31. A pregnant patient would like to know a good food source of calcium other than dairy products. Your best answer is: a. legumes. b. yellow vegetables. c. lean meat. d. whole grains. ANS: A Although dairy products contain the greatest amount of calcium, it is also found in legumes, nuts, dried fruits, and some dark green leafy vegetables. Yellow vegetables are rich in vitamin A. Lean meats are rich in protein and phosphorus. Whole grains are rich in zinc and magnesium. PTS: 1 DIF: Cognitive Level: Knowledge OBJ: Nursing Process: Implementation MSC: Client Needs: Physiologic Integrity 32. To determine the cultural influence on a patient's diet, the nurse should first: a. evaluate the patient's weight gain during pregnancy. b. assess the socioeconomic statuNsUoRfStIhNeGpTaBti.CenOtM. c. discuss the four food groups with the patient. d. identify the food preferences and methods of food preparation common to that culture. ANS: D Understanding the patient's food preferences and how she prepares food will assist the nurse in determining whether the patient's culture is adversely affecting her nutritional intake. Evaluation of a patient's weight gain during pregnancy should be included for all patients, not just for patients who are culturally different. The socioeconomic status of the patient may alter the nutritional intake but not the cultural influence. Teaching the food groups to the patient should come after assessing food preferences. PTS: 1 DIF: Cognitive Level: Application OBJ: Nursing Process: Assessment MSC: Client Needs: Psychosocial Integrity 33. Identify the goal of a patient with the following nursing diagnosis: Imbalanced Nutrition: Less Than Body Requirements related to diet choices inadequate to meet nutrient requirements of pregnancy. a. Gain a total of 30 lbs. b. Take daily supplements consistently. c. Decrease intake of snack foods. d. Increase intake of complex carbohydrates. ANS: A A weight gain of 30 lbs is one indication that the patient has gained a sufficient amount for the nutritional needs of pregnancy. A daily supplement is not the best goal for this patient. It does not meet the basic need of proper nutrition during pregnancy. Decreasing snack foods may be a problem and should be assessed; however, assessing weight gain is the best method of monitoring nutritional intake for this pregnant patient. Increasing the intake of complex carbohydrates is important for this patient, but monitoring the weight gain should be the end goal. PTS: 1 DIF: Cognitive Level: Application OBJ: Nursing Process: Planning MSC: Client Needs: Health Promotion and Maintenance MULTIPLE RESPONSE 1. Most women with uncomplicated pregnancies can use the nurse as their primary source for nutritional information. The nurse or midwife should refer a patient to a registered dietitian for in-depth nutritional counseling in the following situations: (Select all that apply.) a. preexisting or gestational illness such as diabetes. b. ethnic or cultural food patterns. c. obesity. d. vegetarian diet. e. allergy to tree nuts. ANS: A, B, C, D The nurse should be especially aware that conditions such as diabetes can require in-depth dietary planning and evaluation. To prevent issues with hypoglycemia and hyperglycemia and an increased risk for perinatal morbidity and mortality, this patient would benefit from a referral to a dietitian. ConsultationNwURitShINaGdTieBt.iCtiOanMmay ensure that cultural food beliefs are congruent with modern knowledge of fetal development and that adjustments can be made to ensure that all nutritional needs are met. The obese pregnant patient may be under the misapprehension that because of her excess weight little or no weight gain is necessary. According to the Institute of Medicine, a patient with a body mass index in the obese range should gain at least 7 kg to ensure a healthy outcome. This patient may require in-depth counseling on optimal food choices. The vegetarian patient needs to have her dietary intake carefully assessed to ensure that the optimal combination of amino acids and protein intake is achieved. Very strict vegetarians (vegans) who consume only plant products may also require vitamin B and mineral supplementation. A patient with a food allergy would not alter that component of her diet during pregnancy; therefore, no additional consultation is necessary. PTS: 1 DIF: Cognitive Level: Application OBJ: Nursing Process: Implementation MSC: Client Needs: Physiologic Integrity COMPLETION 1. A newly pregnant patient visits her provider's office for the first prenatal appointment. To estimate accurate weight gain throughout the pregnancy, the nurse will be evaluating the appropriateness of weight for height using the body mass index (BMI). The patient weighs 51 kg and is 1.57 m tall. The BMI is: ANS: 20.7 BMI = weight divided by height squared. BMI = 51 kg/(1.57 m)2, or 20.7. Prepregnant BMI can be classified into the following categories: <18.5, underweight or low; 18.5 to 24.9, normal; 25 to 29.9 overweight or high; and >30, obese. PTS: 1 DIF: Cognitive Level: Application OBJ: Nursing Process: Assessment MSC: Client Needs: Health Promotion and Maintenance NURSINGTB.COM Chapter 10: Assessment of High-Risk Pregnancy Perry: Maternal Child Nursing Care, 6th Edition MULTIPLE CHOICE 1. A woman arrives at the clinic seeking confirmation that she is pregnant. The following information is obtained: She is 24 years old with a body mass index (BMI) of 17.5. She admits to having used cocaine “several times” during the past year and drinks alcohol occasionally. Her blood pressure (BP) is 108/70 mm Hg, her pulse rate is 72 beats/min, and her respiratory rate is 16 breaths/min. The family history is positive for diabetes mellitus and cancer. Her sister recently gave birth to an infant with a neural tube defect (NTD). Which characteristics place the woman in a high risk category? a. Blood pressure, age, and BMI b. Drug/alcohol use, age, and family history c. Family history, blood pressure, and BMI d. Family history, BMI, and drug/alcohol abuse ANS: D Her family history of NTD, low BMI, and substance abuse all are high risk factors of pregnancy. The woman's BP is normal, and her age does not put her at risk. Her BMI is low and may indicate poor nutritional status, which would be a high risk. The woman's drug/alcohol use and family history put her in a high risk category, but her age does not. The woman's family history puts her in a high risk category. Her BMI is low and may indicate poor nutritional status, which would be high risk. Her BP is normal. PTS: 1 DIF: CognitiNveURLSevINelG: TCBo.mCOprMehension OBJ: Nursing Process: Assessment MSC: Client Needs: Physiologic Integrity 2. A 39-year-old primigravida thinks that she is about 8 weeks pregnant, although she has had irregular menstrual periods all her life. She has a history of smoking approximately one pack of cigarettes a day, but she tells you that she is trying to cut down. Her laboratory data are within normal limits. What diagnostic technique could be used with this pregnant woman at this time? a. Ultrasound examination b. Maternal serum alpha-fetoprotein (MSAFP) screening c. Amniocentesis d. Nonstress test (NST) ANS: A An ultrasound examination could be done to confirm the pregnancy and determine the gestational age of the fetus. It is too early in the pregnancy to perform MSAFP screening, amniocentesis, or NST. MSAFP screening is performed at 16 to 18 weeks of gestation, followed by amniocentesis if MSAFP levels are abnormal or if fetal/maternal anomalies are detected. NST is performed to assess fetal well-being in the third trimester. PTS: 1 DIF: Cognitive Level: Comprehension OBJ: Nursing Process: Assessment MSC: Client Needs: Health Promotion and Maintenance 3. The nurse sees a woman for the first time when she is 30 weeks pregnant. The woman has smoked throughout the pregnancy, and fundal height measurements now are suggestive of growth restriction in the fetus. In addition to ultrasound to measure fetal size, what other tool would be useful in confirming the diagnosis? a. Doppler blood flow analysis b. Contraction stress test (CST) c. Amniocentesis d. Daily fetal movement counts ANS: A Doppler blood flow analysis allows the examiner to study the blood flow noninvasively in the fetus and the placenta. It is a helpful tool in the management of high risk pregnancies because of intrauterine growth restriction (IUGR), diabetes mellitus, multiple fetuses, or preterm labor. Because of the potential risk of inducing labor and causing fetal distress, CST is not performed on a woman whose fetus is preterm. Indications for amniocentesis include diagnosis of genetic disorders or congenital anomalies, assessment of pulmonary maturity, and diagnosis of fetal hemolytic disease, not IUGR. Fetal kick count monitoring is performed to monitor the fetus in pregnancies complicated by conditions that may affect fetal oxygenation. Although this may be a useful tool at some point later in this woman's pregnancy, it is not used to diagnose IUGR. PTS: 1 DIF: Cognitive Level: Analysis OBJ: Nursing Process: Assessment, Diagnosis MSC: Client Needs: Health Promotion and Maintenance 4. A 41-week pregnant multigravida presents in the labor and delivery unit after a nonstress test indicated that her fetus could be eNxpUeRrSieINncGiTnBg.CsoOmMe difficulties in utero. Which diagnostic tool would yield more detailed information about the fetus? a. Ultrasound for fetal anomalies b. Biophysical profile (BPP) c. Maternal serum alpha-fetoprotein (MSAFP) screening d. Percutaneous umbilical blood sampling (PUBS) ANS: B Real-time ultrasound permits detailed assessment of the physical and physiologic characteristics of the developing fetus and cataloging of normal and abnormal biophysical responses to stimuli. BPP is a noninvasive, dynamic assessment of a fetus that is based on acute and chronic markers of fetal disease. An ultrasound for fetal anomalies would most likely have been performed earlier in the pregnancy. It is too late in the pregnancy to perform MSAFP screening. Also, MSAFP screening does not provide information related to fetal well-being. Indications for PUBS include prenatal diagnosis or inherited blood disorders, karyotyping of malformed fetuses, detection of fetal infection, determination of the acid-base status of a fetus with IUGR, and assessment and treatment of isoimmunization and thrombocytopenia in the fetus. PTS: 1 DIF: Cognitive Level: Comprehension OBJ: Nursing Process: Assessment, Diagnosis MSC: Client Needs: Health Promotion and Maintenance 5. A 40-year-old woman is 10 weeks pregnant. Which diagnostic tool would be appropriate to suggest to her at this time? a. Biophysical profile (BPP) b. Amniocentesis c. Maternal serum alpha-fetoprotein (MSAFP) screening d. Transvaginal ultrasound ANS: D Ultrasound would be performed at this gestational age for biophysical assessment of the infant. BPP would be a method of biophysical assessment of fetal well-being in the third trimester. Amniocentesis is performed after the 14th week of pregnancy. MSAFP screening is performed from week 15 to week 22 of gestation (weeks 16 to 18 are ideal). PTS: 1 DIF: Cognitive Level: Comprehension OBJ: Nursing Process: Planning MSC: Client Needs: Health Promotion and Maintenance 6. A patient asks her nurse, “My doctor told me that he is concerned with the grade of my placenta because I am overdue. What does that mean?” The best response by the nurse is: a. “Your placenta changes as your pregnancy progresses, and it is given a score that indicates the amount of calcium deposits it has. The more calcium deposits, the higher the grade, or number, that is assigned to the placenta. It also means that less blood and oxygen can be delivered to your baby.” b. “Your placenta isn't working properly, and your baby is in danger.” c. “This means that we will need to perform an amniocentesis to detect if you have any placental damage.” d. “Don't worry about it. Everything is fine.” ANS: A An accurate and appropriate response is, “Your placenta changes as your pregnancy progresses, and it is given a scoreNthUaRtSiInNdGicTaBte.CsOthMe amount of calcium deposits it has. The more calcium deposits, the higher the grade, or number, that is assigned to the placenta. It also means that less blood and oxygen can be delivered to your baby.” Although “Your placenta isn't working properly, and your baby is in danger” may be valid, it does not reflect therapeutic communication techniques and is likely to alarm the patient. An ultrasound, not an amniocentesis, is the method of assessment used to determine placental maturation. The response “Don't worry about it. Everything is fine” is not appropriate and discredits the patient's concerns. PTS: 1 DIF: Cognitive Level: Application OBJ: Nursing Process: Planning MSC: Client Needs: Health Promotion and Maintenance 7. A woman is undergoing a nipple-stimulated contraction stress test (CST). She is having contractions that occur every 3 minutes. The fetal heart rate (FHR) has a baseline of approximately 120 beats/min without any decelerations. The interpretation of this test is said to be: a. negative. b. positive. c. satisfactory. d. unsatisfactory. ANS: A Adequate uterine activity necessary for a CST consists of the presence of three contractions in a 10-minute time frame. If no decelerations are observed in the FHR pattern with the contractions, the findings are considered to be negative. A positive CST indicates the presence of repetitive later FHR decelerations. Satisfactory and unsatisfactory are not applicable terms. PTS: 1 DIF: Cognitive Level: Analysis OBJ: Nursing Process: Assessment, Diagnosis MSC: Client Needs: Health Promotion and Maintenance 8. When nurses help their expectant mothers assess the daily fetal movement counts, they should be aware that: a. alcohol or cigarette smoke can irritate the fetus into greater activity. b. “kick counts” should be taken every half hour and averaged every 6 hours, with every other 6-hour stretch off. c. the fetal alarm signal should go off when fetal movements stop entirely for 12 hours. d. obese mothers familiar with their bodies can assess fetal movement as well as average-size women. ANS: C No movement in a 12-hour period is cause for investigation and possibly intervention. Alcohol and cigarette smoke temporarily reduce fetal movement. The mother should count fetal activity (“kick counts”) two or three times daily for 60 minutes each time. Obese women have a harder time assessing fetal movement. PTS: 1 DIF: Cognitive Level: Comprehension OBJ: Nursing Process: Planning MSC: Client Needs: Health Promotion and Maintenance NURSINGTB.COM 9. In comparing the abdominal and transvaginal methods of ultrasound examination, nurses should explain to their patients that: a. both require the woman to have a full bladder. b. the abdominal examination is more useful in the first trimester. c. initially the transvaginal examination can be painful. d. the transvaginal examination allows pelvic anatomy to be evaluated in greater detail. ANS: D The transvaginal examination allows pelvic anatomy to be evaluated in greater detail and allows intrauterine pregnancies to be diagnosed earlier. The abdominal examination requires a full bladder; the transvaginal examination requires an empty bladder. The transvaginal examination is more useful in the first trimester; the abdominal examination works better after the first trimester. Neither method should be painful, although with the transvaginal examination the woman feels pressure as the probe is moved. PTS: 1 DIF: Cognitive Level: Knowledge OBJ: Nursing Process: Planning MSC: Client Needs: Health Promotion and Maintenance 10. In the first trimester, ultrasonography can be used to gain information on: a. amniotic fluid volume. b. location of gestational sacs. c. placental location and maturity. d. cervical length. ANS: B During the first trimester, ultrasound examination is performed to obtain information regarding the number, size, and location of gestational sacs; the presence or absence of fetal cardiac and body movements; the presences or absence of uterine abnormalities (e.g., bicornuate uterus or fibroids) or adnexal masses (e.g., ovarian cysts or an ectopic pregnancy); and pregnancy dating. PTS: 1 DIF: Cognitive Level: Knowledge OBJ: Nursing Process: Assessment MSC: Client Needs: Physiologic Integrity 11. Nurses should be aware that the biophysical profile (BPP): a. is an accurate indicator of impending fetal death. b. is a compilation of health risk factors of the mother during the later stages of pregnancy. c. consists of a Doppler blood flow analysis and an amniotic fluid index. d. involves an invasive form of ultrasound examination. ANS: A An abnormal BPP score is an indication that labor should be induced. The BPP evaluates the health of the fetus, requires many different measures, and is a noninvasive procedure. PTS: 1 DIF: Cognitive Level: Comprehension OBJ: Nursing Process: Assessment MSC: Client Needs: Health Promotion and Maintenance 12. Compared with contraction stress test (CST), nonstress test (NST) for antepartum fetal assessment: NURSINGTB.COM a. has no known contraindications. b. has fewer false-positive results. c. is more sensitive in detecting fetal compromise. d. is slightly more expensive. ANS: A CST has several contraindications. NST has a high rate of false-positive results, is less sensitive than the CST, and is relatively inexpensive. PTS: 1 DIF: Cognitive Level: Comprehension OBJ: Nursing Process: Assessment MSC: Client Needs: Health Promotion and Maintenance 13. The nurse providing care for the antepartum woman should understand that contraction stress test (CST): a. sometimes uses vibroacoustic stimulation. b. is an invasive test; however, contractions are stimulated. c. is considered negative if no late decelerations are observed with the contractions. d. is more effective than nonstress test (NST) if the membranes have already been ruptured. ANS: C No late decelerations are good news. Vibroacoustic stimulation is sometimes used with NST. CST is invasive if stimulation is by intravenous oxytocin but not if by nipple stimulation and is contraindicated if the membranes have ruptured. PTS: 1 DIF: Cognitive Level: Comprehension OBJ: Nursing Process: Assessment MSC: Client Needs: Health Promotion and Maintenance 14. Risk factors tend to be interrelated and cumulative in their effect. While planning the care for a laboring patient with diabetes mellitus, the nurse is aware that she is at a greater risk for: a. oligohydramnios. b. polyhydramnios. c. postterm pregnancy. d. Chromosomal abnormalities. ANS: B Polyhydramnios (amniotic fluid >2000 mL) is 10 times more likely to occur in diabetic compared with nondiabetic pregnancies. Polyhydramnios puts the mother at risk for premature rupture of membranes, premature labor, and after birth hemorrhage. Prolonged rupture of membranes, intrauterine growth restriction, intrauterine fetal death, and renal agenesis (Potter syndrome) all put the patient at risk for developing oligohydramnios. Anencephaly, placental insufficiency, and perinatal hypoxia all contribute to the risk for postterm pregnancy. Maternal age older than 35 years and balanced translocation (maternal and paternal) are risk factors for chromosome abnormalities. PTS: 1 DIF: Cognitive Level: Application OBJ: Nursing Process: Planning MSC: Client Needs: Physiologic Integrity 15. A pregnant woman's biophysical profile score is 8. She asks the nurse to explain the results. The nurse's best response is: a. “The test results are within normal limits.” b. “Immediate delivery by cesareNaUnRbSiIrNthGiTsBb.CeiOnMg considered.” c. “Further testing will be performed to determine the meaning of this score.” d. “An obstetric specialist will evaluate the results of this profile and, within the next week, will inform you of your options regarding delivery.” ANS: A The normal biophysical score ranges from 8 to 10 points if the amniotic fluid volume is adequate. A normal score allows conservative treatment of high risk patients. Delivery can be delayed if fetal well-being is indicated. Scores less than 4 should be investigated, and delivery could be initiated sooner than planned. This score is within normal range, and no further testing is required at this time. The results of the biophysical profile are usually available immediately after the procedure is performed. PTS: 1 DIF: Cognitive Level: Knowledge OBJ: Nursing Process: Implementation MSC: Client Needs: Health Promotion and Maintenance 16. Which analysis of maternal serum may predict chromosomal abnormalities in the fetus? a. Multiple-marker screening b. Lecithin/sphingomyelin [L/S] ratio c. Biophysical profile d. Type and crossmatch of maternal and fetal serum ANS: A Maternal serum can be analyzed for abnormal levels of alpha-fetoprotein, human chorionic gonadotropin, and estriol. The multiple-marker screening may predict chromosomal defects in the fetus. The L/S ratio is used to determine fetal lung maturity. A biophysical profile is used for evaluating fetal status during the antepartum period. Five variables are used, but none is concerned with chromosomal problems. The blood type and crossmatch would not predict chromosomal defects in the fetus. PTS: 1 DIF: Cognitive Level: Knowledge OBJ: Nursing Process: Assessment MSC: Client Needs: Physiologic Integrity 17. While working with the pregnant woman in her first trimester, the nurse is aware that chorionic villus sampling (CVS) can be performed during pregnancy at: a. 4 weeks. b. 8 weeks. c. 10 weeks. d. 14 weeks. ANS: C CVS can be performed in the first or second trimester, ideally between 10 and 13 weeks of gestation. During this procedure, a small piece of tissue is removed from the fetal portion of the placenta. If performed after 9 completed weeks of gestation, the risk of limb reduction is no greater than in the general population. PTS: 1 DIF: Cognitive Level: Knowledge OBJ: Nursing Process: Assessment MSC: Client Needs: Physiologic Integrity 18. Which nursing intervention is necessary before a second-trimester transabdominal ultrasound? a. Place the woman NPO for 12 NhoUuRrSsI.NGTB.COM b. Instruct the woman not void until after the test. c. Administer an enema. d. Perform an abdominal preparation. ANS: B When the uterus is still in the pelvis, visualization may be difficult. It is necessary to perform the test when the woman has a full bladder, which provides a “window” through which the uterus and its contents can be viewed. The woman needs a full bladder to elevate the uterus; therefore being NPO is not appropriate. Neither an enema nor an abdominal preparation is necessary for this procedure. PTS: 1 DIF: Cognitive Level: Knowledge OBJ: Nursing Process: Implementation MSC: Client Needs: Physiologic Integrity 19. The nurse recognizes that a nonstress test (NST) in which two or more fetal heart rate (FHR) accelerations of 15 beats/min or more occur with fetal movement in a 20-minute period is: a. nonreactive. b. positive. c. negative. d. reactive. ANS: D The NST is reactive (normal) when two or more FHR accelerations of at least 15 beats/min (each with a duration of at least 15 seconds) occur in a 20-minute period. A nonreactive result means that the heart rate did not accelerate during fetal movement. A positive result is not used with NST. Contraction stress test (CST) uses positive as a result term. A negative result is not used with NST. CST uses negative as a result term. PTS: 1 DIF: Cognitive Level: Comprehension OBJ: Nursing Process: Assessment MSC: Client Needs: Health Promotion and Maintenance MULTIPLE RESPONSE 1. Intrauterine growth restriction (IUGR) is associated with numerous pregnancy-related risk factors. (Select all that apply.) a. Poor material weight gain b. Chronic maternal infections c. Gestational hypertension d. Premature rupture of membranes e. Smoking ANS: A, B, C, E Poor material weight gain, chronic infections disease, gestational hypertension, and smoking are all risk factors associated with IUGR. Premature rupture of membranes is associated with preterm labor, not IUGR. PTS: 1 DIF: Cognitive Level: Analysis OBJ: Nursing Process: Diagnosis MSC: Client Needs: Health Promotion and Maintenance NURSINGTB.COM 2. Transvaginal ultrasonography is often performed during the first trimester. While preparing your 6-week gestation patient for this procedure, she expresses concerns over the necessity for this test. The nurse should explain that this diagnostic test may be indicated for a number of situations. (Select all that apply) a. Establish gestational age b. Obesity c. Fetal abnormalities d. Amniotic fluid volume e. Ectopic pregnancy ANS: A, B, C, E Transvaginal ultrasound is useful in obese women whose thick abdominal layers cannot be penetrated with traditional abdominal ultrasound. This procedure is also used for identifying ectopic pregnancy, estimating gestational age, confirming fetal viability, and identifying fetal abnormalities. Amniotic fluid volume is assessed during the second and third trimester. Conventional ultrasound would be used. PTS: 1 DIF: Cognitive Level: Application OBJ: Nursing Process: Planning MSC: Client Needs: Physiologic Integrity MATCHING Biophysical risks include factors that originate with either the mother or the fetus and affect the functioning of either one or both. The nurse who provides prenatal care should have an understanding of these risk factors. Match the specific pregnancy problem with the related risk factor. a. Polyhydramnios b. Intrauterine growth restriction (maternal cause) c. Oligohydramnios d. Chromosomal abnormalities e. Intrauterine growth restriction (fetoplacental cause) 1. Premature rupture of membranes 2. Advanced maternal age 3. Fetal congenital anomalies 4. Abnormal placenta development 5. Smoking, alcohol, and illicit drug use 1. ANS: C PTS: 1 DIF: Cognitive Level: Comprehension OBJ: Nursing Process: Implementation MSC: Client Needs: Health Promotion and Maintenance NOT: Each pregnancy problem can be attributed to a number of related risk factors. Polyhydramnios may also be the result of poorly controlled diabetes mellitus. Other maternal causes of IUGR include hypertensive disorders, diabetes, chronic renal disease, vascular disease, thrombophilia, poor weight gain, and cyanotic heart disease. Fetoplacental causes of IUGR may be related to chromosomal abnormalities, congenital malformations, intrauterine infection, or genetic syndromes. Other contributors to oligohydramnios are renal agenesis, prolonged pregnancy, uteroplacental insufficiency, and paternal hypertensive disorders. Although advanced maternal age is a well-known cause of chromosomal abnormalities, other causes include parental chromosome rearrangements and pregnancy with autosomal trisomy. 2. ANS: D PTS: 1 NURSINDGITFB: .CCOoMgnitive Level: Comprehension OBJ: Nursing Process: Implementation MSC: Client Needs: Health Promotion and Maintenance NOT: Each pregnancy problem can be attributed to a number of related risk factors. Polyhydramnios may also be the result of poorly controlled diabetes mellitus. Other maternal causes of IUGR include hypertensive disorders, diabetes, chronic renal disease, vascular disease, thrombophilia, poor weight gain, and cyanotic heart disease. Fetoplacental causes of IUGR may be related to chromosomal abnormalities, congenital malformations, intrauterine infection, or genetic syndromes. Other contributors to oligohydramnios are renal agenesis, prolonged pregnancy, uteroplacental insufficiency, and paternal hypertensive disorders. Although advanced maternal age is a well-known cause of chromosomal abnormalities, other causes include parental chromosome rearrangements and pregnancy with autosomal trisomy. 3. ANS: A PTS: 1 DIF: Cognitive Level: Comprehension OBJ: Nursing Process: Implementation MSC: Client Needs: Health Promotion and Maintenance NOT: Each pregnancy problem can be attributed to a number of related risk factors. Polyhydramnios may also be the result of poorly controlled diabetes mellitus. Other maternal causes of IUGR include hypertensive disorders, diabetes, chronic renal disease, vascular disease, thrombophilia, poor weight gain, and cyanotic heart disease. Fetoplacental causes of IUGR may be related to chromosomal abnormalities, congenital malformations, intrauterine infection, or genetic syndromes. Other contributors to oligohydramnios are renal agenesis, prolonged pregnancy, uteroplacental insufficiency, and paternal hypertensive disorders. Although advanced maternal age is a well-known cause of chromosomal abnormalities, other causes include parental chromosome rearrangements and pregnancy with autosomal trisomy. 4. ANS: E PTS: 1 DIF: Cognitive Level: Comprehension OBJ: Nursing Process: Implementation MSC: Client Needs: Health Promotion and Maintenance NOT: Each pregnancy problem can be attributed to a number of related risk factors. Polyhydramnios may also be the result of poorly controlled diabetes mellitus. Other maternal causes of IUGR include hypertensive disorders, diabetes, chronic renal disease, vascular disease, thrombophilia, poor weight gain, and cyanotic heart disease. Fetoplacental causes of IUGR may be related to chromosomal abnormalities, congenital malformations, intrauterine infection, or genetic syndromes. Other contributors to oligohydramnios are renal agenesis, prolonged pregnancy, uteroplacental insufficiency, and paternal hypertensive disorders. Although advanced maternal age is a well-known cause of chromosomal abnormalities, other causes include parental chromosome rearrangements and pregnancy with autosomal trisomy. 5. ANS: B PTS: 1 DIF: Cognitive Level: Comprehension OBJ: Nursing Process: Implementation MSC: Client Needs: Health Promotion and Maintenance NOT: Each pregnancy problem can be attributed to a number of related risk factors. Polyhydramnios may also be the result of poorly controlled diabetes mellitus. Other maternal causes of IUGR include hypertensive disorders, diabetes, chronic renal disease, vascular disease, thrombophilia, poor weight gain, and cyanotic heart disease. Fetoplacental causes of IUGR may be related to chromosomal abnormalities, congenital malformations, intrauterine infection, or genetic syndromes. Other contributors to oligohydramnios are renal agenesis, prolonged pregnancy, uteroplacental insufficiency, and paternal hypertensive disorders. Although advanced maternal age is a well-known cause of chromosomal abnormalities, other causes include parental chromosome rearrangements and pregnancy with autosomal trisomy. NURSINGTB.COM Chapter 11: High-Risk Perinatal Care: Preexisting Conditions Perry: Maternal Child Nursing Care, 6th Edition MULTIPLE CHOICE 1. In assessing the knowledge of a pregestational woman with type 1 diabetes concerning changing insulin needs during pregnancy, the nurse recognizes that further teaching is warranted when the patient states: a. “I will need to increase my insulin dosage during the first 3 months of pregnancy.” b. “Insulin dosage will likely need to be increased during the second and third trimesters.” c. “Episodes of hypoglycemia are more likely to occur during the first 3 months.” d. “Insulin needs should return to normal within 7 to 10 days after birth if I am bottle-feeding.” ANS: A Insulin needs are reduced in the first trimester because of increased insulin production by the pancreas and increased peripheral sensitivity to insulin. “Insulin dosage will likely need to be increased during the second and third trimesters,” “Episodes of hypoglycemia are more likely to occur during the first 3 months,” and “Insulin needs should return to normal within 7 to 10 days after birth if I am bottle-feeding” are accurate statements and signify that the woman has understood the teachings regarding control of her diabetes during pregnancy. PTS: 1 DIF: Cognitive Level: Application OBJ: Nursing Process: Evaluation MSC: Client Needs: Physiologic Integrity NURSINGTB.COM 2. Preconception counseling is critical to the outcome of diabetic pregnancies because poor glycemic control before and during early pregnancy is associated with: a. frequent episodes of maternal hypoglycemia. b. congenital anomalies in the fetus. c. polyhydramnios. d. hyperemesis gravidarum. ANS: B Preconception counseling is particularly important because strict metabolic control before conception and in the early weeks of gestation is instrumental in decreasing the risks of congenital anomalies. Frequent episodes of maternal hypoglycemia may occur during the first trimester (not before conception) as a result of hormone changes and the effects on insulin production and usage. Hydramnios occurs about 10 times more often in diabetic pregnancies than in nondiabetic pregnancies. Typically it is seen in the third trimester of pregnancy. Hyperemesis gravidarum may exacerbate hypoglycemic events because the decreased food intake by the mother and glucose transfer to the fetus contributes to hypoglycemia. PTS: 1 DIF: Cognitive Level: Comprehension OBJ: Nursing Process: Planning MSC: Client Needs: Physiologic Integrity 3. In planning for the care of a 30-year-old woman with pregestational diabetes, the nurse recognizes that the most important factor affecting pregnancy outcome is the: a. mother's age. b. number of years since diabetes was diagnosed. c. amount of insulin required prenatally. d. degree of glycemic control during pregnancy. ANS: D Women with excellent glucose control and no blood vessel disease should have good pregnancy outcomes. PTS: 1 DIF: Cognitive Level: Comprehension OBJ: Nursing Process: Planning MSC: Client Needs: Health Promotion and Maintenance 4. Screening at 24 weeks of gestation reveals that a pregnant woman has gestational diabetes mellitus (GDM). In planning her care, the nurse and the woman mutually agree that an expected outcome is to prevent injury to the fetus as a result of GDM. The nurse identifies that the fetus is at greatest risk for: a. macrosomia. b. congenital anomalies of the central nervous system. c. preterm birth. d. low birth weight. ANS: A Poor glycemic control later in pregnancy increases the rate of fetal macrosomia. Poor glycemic control during the preconception time frame and into the early weeks of the pregnancy is associated with congenital anomalies. Preterm labor or birth is more likely to occur with severe diabetes and is the greatest risk in women with pregestational diabetes. Increased weight, or macrosomia, is the greatest risk factor for this woman. PTS: 1 DIF: Cognitive Level: Comprehension OBJ: Nursing Process: Planning, ImNpUleRmSIeNntGaTtiBon.COM MSC: Client Needs: Physiologic Integrity 5. A 26-year-old primigravida has come to the clinic for her regular prenatal visit at 12 weeks. She appears thin and somewhat nervous. She reports that she eats a well-balanced diet, although her weight is 5 lbs less than it was at her last visit. The results of laboratory studies confirm that she has a hyperthyroid condition. Based on the available data, the nurse formulates a plan of care. What nursing diagnosis is most appropriate for the woman at this time? a. Deficient fluid volume b. Imbalanced nutrition: less than body requirements c. Imbalanced nutrition: more than body requirements d. Disturbed sleep pattern ANS: B This patient's clinical cues include weight loss, which would support the nursing diagnosis of Imbalanced nutrition: less than body requirements. No clinical signs or symptoms support the nursing diagnosis of Deficient fluid volume. This patient reports weight loss, not weight gain. Imbalanced nutrition: more than body requirements is not an appropriate nursing diagnosis. Although the patient reports nervousness based on the patient's other clinical symptoms the most appropriate nursing diagnosis would be Imbalanced nutrition: less than body requirements. PTS: 1 DIF: Cognitive Level: Analysis OBJ: Nursing Process: Diagnosis MSC: Client Needs: Physiologic Integrity 6. Maternal phenylketonuria (PKU) is an important health concern during pregnancy because: a. it is a recognized cause of preterm labor. b. the fetus may develop neurologic problems. c. a pregnant woman is more likely to die without dietary control. d. women with PKU are usually retarded and should not reproduce. ANS: B Children born to women with untreated PKU are more likely to be born with mental retardation, microcephaly, congenital heart disease, and low birth weight. Maternal PKU has no effect on labor. Women without dietary control of PKU are more likely to miscarry or bear a child with congenital anomalies. Screening for undiagnosed maternal PKU at the first prenatal visit may be warranted, especially in individuals with a family history of the disorder, with low intelligence of uncertain etiology, or who have given birth to microcephalic infants. PTS: 1 DIF: Cognitive Level: Comprehension OBJ: Nursing Process: Assessment MSC: Client Needs: Physiologic Integrity 7. In terms of the incidence and classification of diabetes, maternity nurses should know that: a. type 1 diabetes is most common. b. type 2 diabetes often goes undiagnosed. c. gestational diabetes mellitus (GDM) means that the woman will be receiving insulin treatment until 6 weeks after birth. d. type 1 diabetes may become type 2 during pregnancy. ANS: B Type 2 diabetes often goes undiagnosed because hyperglycemia develops gradually and often is not severe. Type 2 diabetes, somNeUtRimSIeNsGcTaBll.eCdOaMdult onset diabetes, is the most common. GDM refers to any degree of glucose intolerance first recognized during pregnancy. Insulin may or may not be needed. People do not go back and forth between type 1 and 2 diabetes. PTS: 1 DIF: Cognitive Level: Knowledge OBJ: Nursing Process: Assessment MSC: Client Needs: Physiologic Integrity 8. Metabolic changes throughout pregnancy that affect glucose and insulin in the mother and the fetus are complicated but important to understand. Nurses should understand that: a. insulin crosses the placenta to the fetus only in the first trimester, after which the fetus secretes its own. b. women with insulin-dependent diabetes are prone to hyperglycemia during the first trimester because they are consuming more sugar. c. during the second and third trimesters, pregnancy exerts a diabetogenic effect that ensures an abundant supply of glucose for the fetus. d. maternal insulin requirements steadily decline during pregnancy. ANS: C Pregnant women develop increased insulin resistance during the second and third trimesters. Insulin never crosses the placenta; the fetus starts making its own insulin around the 10th week. As a result of normal metabolic changes during pregnancy, insulin-dependent women are prone to hypoglycemia (low levels). Maternal insulin requirements may double or quadruple by the end of pregnancy. PTS: 1 DIF: Cognitive Level: Comprehension OBJ: Nursing Process: Assessment MSC: Client Needs: Physiologic Integrity 9. With regard to the association of maternal diabetes and other risk situations affecting mother and fetus, nurses should be aware that: a. Diabetic ketoacidosis (DKA) can lead to fetal death at any time during pregnancy. b. Hydramnios occurs approximately twice as often in diabetic pregnancies. c. Infections occur about as often and are considered about as serious in diabetic and nondiabetic pregnancies. d. Even mild to moderate hypoglycemic episodes can have significant effects on fetal well-being. ANS: A Prompt treatment of DKA is necessary to save the fetus and the mother. Hydramnios occurs 10 times more often in diabetic pregnancies. Infections are more common and more serious in pregnant women with diabetes. Mild-to-moderate hypoglycemic episodes do not appear to have significant effects on fetal well-being. PTS: 1 DIF: Cognitive Level: Comprehension OBJ: Nursing Process: Planning MSC: Client Needs: Physiologic Integrity 10. The nurse providing care for a woman with gestational diabetes understands that a laboratory test for glycosylated hemoglobin Alc: a. is now done for all pregnant women, not just those with or likely to have diabetes. b. is a snapshot of glucose control at the moment. c. would be considered evidence of good diabetes control with a result of 5% to 6%. d. is done on the patient's urine, not her blood. NURSINGTB.COM ANS: C A score of 5% to 6% indicates good control. This is an extra test for diabetic women, not one done for all pregnant women. This test defines glycemic control over the previous 4 to 6 weeks. Glycosylated hemoglobin level tests are done on the blood. PTS: 1 DIF: Cognitive Level: Comprehension OBJ: Nursing Process: Evaluation MSC: Client Needs: Health Promotion and Maintenance 11. A woman with gestational diabetes has had little or no experience reading and interpreting glucose levels. She shows the nurse her readings for the past few days. Which one should the nurse tell her indicates a need for adjustment (insulin or sugar)? a. 75 mg/dL before lunch. This is low; better eat now. b. 115 mg/dL 1 hour after lunch. This is a little high; maybe eat a little less next time. c. 115 mg/dL 2 hours after lunch; This is too high; it is time for insulin. d. 60 mg/dL just after waking up from a nap. This is too low; maybe eat a snack before going to sleep. ANS: D 60 mg/dL after waking from a nap is too low. During hours of sleep glucose levels should not be less than 70 mg/dL. Snacks before sleeping can be helpful. The premeal acceptable range is 65 to 95 mg/dL. The readings 1 hour after a meal should be less than 140 mg/dL. Two hours after eating, the readings should be less than 120 mg/dL. PTS: 1 DIF: Cognitive Level: Application OBJ: Nursing Process: Evaluation MSC: Client Needs: Health Promotion and Maintenance 12. A new mother with which of these thyroid disorders would be strongly discouraged from breastfeeding? a. Hyperthyroidism b. Phenylketonuria (PKU) c. Hypothyroidism d. Thyroid storm ANS: B PKU is a cause of mental retardation in infants; mothers with PKU pass on phenylalanine. A woman with hyperthyroidism or hypothyroidism would have no particular reason not to breastfeed. A thyroid storm is a complication of hyperthyroidism. PTS: 1 DIF: Cognitive Level: Comprehension OBJ: Nursing Process: Planning MSC: Client Needs: Physiologic Integrity 13. When caring for a pregnant woman with cardiac problems, the nurse must be alert for signs and symptoms of cardiac decompensation, which include: a. a regular heart rate and hypertension. b. an increased urinary output, tachycardia, and dry cough. c. shortness of breath, bradycardia, and hypertension. d. dyspnea; crackles; and an irregular, weak pulse. ANS: D Signs of cardiac decompensation include dyspnea; crackles; an irregular, weak, rapid pulse; rapid respirations; a moist, frequent cough; generalized edema; increasing fatigue; and cyanosis of the lips and nail beds.NAURreSgINulGaTr Bh.eCaOrtMrate and hypertension are not generally associated with cardiac decompensation. Tachycardia would indicate cardiac decompensation, but increased urinary output and a dry cough would not. Shortness of breath would indicate cardiac decompensation, but bradycardia and hypertension would not. PTS: 1 DIF: Cognitive Level: Comprehension OBJ: Nursing Process: Assessment MSC: Client Needs: Physiologic Integrity 14. While providing care in an obstetric setting, the nurse should understand that after birth care of the woman with cardiac disease: a. is the same as that for any pregnant woman. b. includes rest, stool softeners, and monitoring of the effect of activity. c. includes ambulating frequently, alternating with active range of motion. d. includes limiting visits with the infant to once per day. ANS: B Bed rest may be ordered, with or without bathroom privileges. Bowel movements without stress or strain for the woman are promoted with stool softeners, diet, and fluid. Care of the woman with cardiac disease in the after birth period is tailored to the woman's functional capacity. The woman will be on bed rest to conserve energy and reduce the strain on the heart. Although the woman may need help caring for the infant, breastfeeding and infant visits are not contraindicated. PTS: 1 DIF: Cognitive Level: Comprehension OBJ: Nursing Process: Planning MSC: Client Needs: Physiologic Integrity 15. A woman with asthma is experiencing a after birth hemorrhage. Which drug would not be used to treat her bleeding because it may exacerbate her asthma? a. Pitocin b. Nonsteroidal anti-inflammatory drugs (NSAIDs) c. Hemabate d. Fentanyl ANS: C Prostaglandin derivatives should not be used to treat women with asthma because they may exacerbate symptoms. Pitocin would be the drug of choice to treat this woman's bleeding because it would not exacerbate her asthma. NSAIDs are not used to treat bleeding. Fentanyl is used to treat pain, not bleeding. PTS: 1 DIF: Cognitive Level: Analysis OBJ: Nursing Process: Planning MSC: Client Needs: Physiologic Integrity 16. The use of methamphetamine (meth) has been described as a significant drug problem in the United States. In order to provide adequate nursing care to this patient population the nurse must be cognizant that methamphetamine: a. is similar to opiates. b. is a stimulant with vasoconstrictive characteristics. c. should not be discontinued during pregnancy. d. is associated with a low rate of relapse. ANS: B Methamphetamines are stimulants with vasoconstrictive characteristics similar to cocaine and are used similarly. As is the case wNiUthRScIoNcGaTinBe.CuOseMrs, methamphetamine users are urged to immediately stop all use during pregnancy. Unfortunately, because methamphetamine users are extremely psychologically addicted, the rate of relapse is very high. PTS: 1 DIF: Cognitive Level: Comprehension OBJ: Nursing Process: Assessment MSC: Client Needs: Psychosocial Integrity 17. Since the gene for cystic fibrosis was identified in 1989, data can be collected for the purposes of genetic counseling for couples regarding carrier status. According to statistics, how often does cystic fibrosis occur in Caucasian live births? a. 1 in 100 b. 1 in 1200 c. 1 in 2500 d. 1 in 3000 ANS: D Cystic fibrosis occurs in about 1 in 3000 Caucasian live births. PTS: 1 DIF: Cognitive Level: Comprehension OBJ: Nursing Process: Assessment MSC: Client Needs: Health Promotion and Maintenance 18. Which heart condition is not a contraindication for pregnancy? a. Peripartum cardiomyopathy b. Eisenmenger syndrome c. Heart transplant d. All of these contraindicate pregnancy. ANS: C Pregnancy is contraindicated for peripartum cardiomyopathy and Eisenmenger syndrome. Women who have had heart transplants are successfully having babies. However, conception should be postponed for at least 1 year after transplantation. PTS: 1 DIF: Cognitive Level: Comprehension OBJ: Nursing Process: Assessment MSC: Client Needs: Health Promotion and Maintenance 19. During a physical assessment of an at-risk patient, the nurse notes generalized edema, crackles at the base of the lungs, and some pulse irregularity. These are most likely signs of: a. euglycemia. b. rheumatic fever. c. pneumonia. d. cardiac decompensation. ANS: D Symptoms of cardiac decompensation may appear abruptly or gradually. Euglycemia is a condition of normal glucose levels. These symptoms indicate cardiac decompensation. Rheumatic fever can cause heart problems, but it does not manifest with these symptoms, which indicate cardiac decompensation. Pneumonia is an inflammation of the lungs and would not likely generate these symptoms, which indicate cardiac decompensation. PTS: 1 DIF: Cognitive Level: Analysis OBJ: Nursing Process: Assessment MSC: Client Needs: Physiologic Integrity 20. Nurses caring for antepartum womNeUnRwSIiNthGTcaBr.dCiOaMc conditions should be aware that: a. stress on the heart is greatest in the first trimester and the last 2 weeks before labor. b. women with Class II cardiac disease should avoid heavy exertion and any activity that causes even minor symptoms. c. women with Class III cardiac disease should have 8 to 10 hours of sleep every day and limit housework, shopping, and exercise. d. Women with Class I cardiac disease need bed rest through most of the pregnancy and face the possibility of hospitalization near term. ANS: B Class II cardiac disease is symptomatic with ordinary activity. Women in this category need to avoid heavy exertion and limit regular activities as symptoms dictate. Stress is greatest between weeks 28 and 32, when homodynamic changes reach their maximum. Class III cardiac disease is symptomatic with less than ordinary activity. These women need bed rest most of the day and face the possibility of hospitalization near term. Class I cardiac disease is asymptomatic at normal levels of activity. These women can carry on limited normal activities with discretion, although they still need a good amount of sleep. PTS: 1 DIF: Cognitive Level: Comprehension OBJ: Nursing Process: Planning MSC: Client Needs: Physiologic Integrity 21. As related to the care of the patient with anemia, the nurse should be aware that: a. it is the most common medical disorder of pregnancy. b. it can trigger reflex brachycardia. c. the most common form of anemia is caused by folate deficiency. d. thalassemia is a European version of sickle cell anemia. ANS: A Combined with any other complication, anemia can result in congestive heart failure. Reflex bradycardia is a slowing of the heart in response to the blood flow increases immediately after birth. The most common form of anemia is iron deficiency anemia. Both thalassemia and sickle cell hemoglobinopathy are hereditary but not directly related or confined to geographic areas. PTS: 1 DIF: Cognitive Level: Knowledge OBJ: Nursing Process: Planning MSC: Client Needs: Physiologic Integrity 22. The most common neurologic disorder accompanying pregnancy is: a. eclampsia. b. Bell's palsy. c. epilepsy. d. multiple sclerosis. ANS: C The effects of pregnancy on epilepsy are unpredictable. Eclampsia sometimes may be confused with epilepsy, which is the most common neurologic disorder accompanying pregnancy. Bell's palsy is a form of facial paralysis. Multiple sclerosis is a patchy demyelinization of the spinal cord that does not affect the normal course of pregnancy or birth. PTS: 1 DIF: Cognitive Level: Knowledge OBJ: Nursing Process: Planning MSC: Client Needs: Physiologic Integrity NURSINGTB.COM 23. With one exception, the safest pregnancy is one in which the woman is drug and alcohol free. For women addicted to opioids, treatment is the current standard of care during pregnancy. a. methadone maintenance b. detoxification c. smoking cessation d. 4 Ps Plus ANS: A Methadone maintenance treatment (MMT) is currently considered the standard of care for pregnant women who are dependent on heroin or other narcotics. Buprenorphine is another medication approved for opioid addiction treatment that is increasingly being used during pregnancy. Opioid replacement therapy has been shown to decrease opioid and other drug use, reduce criminal activity, improve individual functioning, and decrease rates of infections such as hepatitis B and C, HIV, and other sexually transmitted infections. Detoxification is the treatment used for alcohol addiction. Pregnant women requiring withdrawal from alcohol should be admitted for inpatient management. Women are more likely to stop smoking during pregnancy than at any other time in their lives. A smoking cessation program can assist in achieving this goal. The 4 Ps Plus is a screening tool designed specifically to identify pregnant women who need in-depth assessment related to substance abuse. PTS: 1 DIF: Cognitive Level: Application OBJ: Nursing Process: Planning MSC: Client Needs: Psychosocial Integrity 24. Which major neonatal complication is carefully monitored after the birth of the infant of a diabetic mother? a. Hypoglycemia b. Hypercalcemia c. Hypobilirubinemia d. Hypoinsulinemia ANS: A The neonate is at highest risk for hypoglycemia because fetal insulin production is accelerated during pregnancy to metabolize excessive glucose from the mother. At birth, the maternal glucose supply stops and the neonatal insulin exceeds the available glucose, thus leading to hypoglycemia. Hypocalcemia is associated with preterm birth, birth trauma, and asphyxia, all common problems of the infant of a diabetic mother. Excess erythrocytes are broken down after birth and release large amounts of bilirubin into the neonate's circulation, with resulting hyperbilirubinemia. Because fetal insulin production is accelerated during pregnancy, the neonate presents with hyperinsulinemia. PTS: 1 DIF: Cognitive Level: Comprehension OBJ: Nursing Process: Planning MSC: Client Needs: Health Promotion and Maintenance 25. Which factor is known to increase the risk of gestational diabetes mellitus? a. Underweight before pregnancy b. Maternal age younger than 25 years c. Previous birth of large infant d. Previous diagnosis of type 2 diabetes mellitus ANS: C Previous birth of a large infant sugNgUeRstSsINgGesTtBat.CioOnMal diabetes mellitus. Obesity (BMI of 30 or greater) creates a higher risk for gestational diabetes. A woman younger than 25 years generally is not at risk for gestational diabetes mellitus. The person with type 2 diabetes mellitus already has diabetes and will continue to have it after pregnancy. Insulin may be required during pregnancy because oral hypoglycemia drugs are contraindicated during pregnancy. PTS: 1 DIF: Cognitive Level: Comprehension OBJ: Nursing Process: Assessment MSC: Client Needs: Health Promotion and Maintenance 26. Glucose metabolism is profoundly affected during pregnancy because: a. pancreatic function in the islets of Langerhans is affected by pregnancy. b. the pregnant woman uses glucose at a more rapid rate than the nonpregnant woman. c. the pregnant woman increases her dietary intake significantly. d. placental hormones are antagonistic to insulin, thus resulting in insulin resistance. ANS: D Placental hormones, estrogen, progesterone, and human placental lactogen (HPL) create insulin resistance. Insulin is also broken down more quickly by the enzyme placental insulinase. Pancreatic functioning is not affected by pregnancy. The glucose requirements differ because of the growing fetus. The pregnant woman should increase her intake by 200 calories a day. PTS: 1 DIF: Cognitive Level: Comprehension OBJ: Nursing Process: Assessment MSC: Client Needs: Physiologic Integrity 27. To manage her diabetes appropriately and ensure a good fetal outcome, the pregnant woman with diabetes will need to alter her diet by: a. eating six small equal meals per day. b. reducing carbohydrates in her diet. c. eating her meals and snacks on a fixed schedule. d. increasing her consumption of protein. ANS: C Having a fixed meal schedule will provide the woman and the fetus with a steadier blood sugar level, provide better balance with insulin administration, and help prevent complications. It is more important to have a fixed meal schedule than equal division of food intake. Approximately 45% of the food eaten should be in the form of carbohydrates. PTS: 1 DIF: Cognitive Level: Comprehension OBJ: Nursing Process: Planning MSC: Client Needs: Health Promotion and Maintenance 28. When the pregnant diabetic woman experiences hypoglycemia while hospitalized, the nurse should intervene by having the patient: a. eat six saltine crackers. b. drink 8 ounces of orange juice with 2 tsp of sugar added. c. drink 4 ounces of orange juice followed by 8 ounces of milk. d. eat hard candy or commercial glucose wafers. ANS: A Crackers provide carbohydrates inNtUhReSfIoNrGmToBf.CpOoMlysaccharides. Orange juice and sugar will increase the blood sugar but not provide a slow-burning carbohydrate to sustain the blood sugar. Milk is a disaccharide and orange juice is a monosaccharide. They will provide an increase in blood sugar but will not sustain the level. Hard candy or commercial glucose wafers provide only monosaccharides. PTS: 1 DIF: Cognitive Level: Application OBJ: Nursing Process: Implementation MSC: Client Needs: Physiologic Integrity 29. Nursing intervention for the pregnant diabetic patient is based on the knowledge that the need for insulin: a. increases throughout pregnancy and the after birth period. b. decreases throughout pregnancy and the after birth period. c. varies depending on the stage of gestation. d. should not change because the fetus produces its own insulin. ANS: C Insulin needs decrease during the first trimester, when nausea, vomiting, and anorexia are a factor. They increase during the second and third trimesters, when the hormones of pregnancy create insulin resistance in maternal cells. Insulin needs increase during the second and third trimesters, when the hormones of pregnancy create insulin resistance in maternal cells. The insulin needs change throughout the different stages of pregnancy. PTS: 1 DIF: Cognitive Level: Comprehension OBJ: Nursing Process: Implementation MSC: Client Needs: Physiologic Integrity 30. In caring for a pregnant woman with sickle cell anemia, the nurse is aware that signs and symptoms of sickle cell crisis include: a. anemia. b. endometritis. c. fever and pain. d. urinary tract infection. ANS: C Women with sickle cell anemia have recurrent attacks (crisis) of fever and pain, most often in the abdomen, joints, and extremities. These attacks are attributed to vascular occlusion when RBCs assume the characteristic sickled shape. Crises are usually triggered by dehydration, hypoxia, or acidosis. Women with sickle cell anemia are not iron deficient. Therefore, routine iron supplementation, even that found in prenatal vitamins, should be avoided in order to prevent iron overload. Women with sickle cell trait usually are at greater risk for after birth endometritis (uterine wall infection); however, this is not likely to occur in pregnancy and is not a sign of crisis. These women are at an increased risk for UTIs; however, this is not an indication of sickle cell crisis. PTS: 1 DIF: Cognitive Level: Comprehension OBJ: Nursing Process: Assessment MSC: Client Needs: Physiologic Integrity MULTIPLE RESPONSE 1. Congenital anomalies can occur with the use of antiepileptic drugs (AEDs), including: (Select all that apply.) a. cleft lip. b. congenital heart disease. c. neural tube defects. d. gastroschisis. e. diaphragmatic hernia. ANS: A, B, C NURSINGTB.COM Congenital anomalies that can occur with AEDs include cleft lip or palate, congenital heart disease, urogenital defects, and neural tube defects. Gastroschisis and diaphragmatic hernia are not associated with the use of AEDs. PTS: 1 DIF: Cognitive Level: Comprehension OBJ: Nursing Process: Planning MSC: Client Needs: Physiologic Integrity 2. Diabetes refers to a group of metabolic diseases characterized by hyperglycemia resulting from defects in insulin action, insulin secretion, or both. Over time, diabetes causes significant changes in the microvascular and macrovascular circulations. These complications include: (Select all that apply.) a. atherosclerosis. b. retinopathy. c. IUFD. d. nephropathy. e. neuropathy. ANS: A, B, D, E These structural changes are most likely to affect a variety of systems, including the heart, eyes, kidneys, and nerves. Intrauterine fetal death (stillbirth) remains a major complication of diabetes in pregnancy; however, this is a fetal complication. PTS: 1 DIF: Cognitive Level: Comprehension OBJ: Nursing Process: Diagnosis MSC: Client Needs: Physiologic Integrity 3. Autoimmune disorders often occur during pregnancy because a large percentage of women with an autoimmune disorder are of childbearing age. Identify all disorders that fall into the category of collagen vascular disease. a. Multiple sclerosis b. Systemic lupus erythematosus c. Antiphospholipid syndrome d. Rheumatoid arthritis e. Myasthenia gravis ANS: B, C, D, E Multiple sclerosis is not an autoimmune disorder. This patchy demyelinization of the spinal cord may be a viral disorder. Autoimmune disorders (collagen vascular disease) make up a large group of conditions that disrupt the function of the immune system of the body. They include those listed, as well as systemic sclerosis. PTS: 1 DIF: Cognitive Level: Comprehension OBJ: Nursing Process: Assessment MSC: Client Needs: Health Promotion and Maintenance COMPLETION NURSINGTB.COM 1. Achieving and maintaining euglycemia comprise the primary goals of medical therapy for the pregnant woman with diabetes. These goals are achieved through a combination of diet, insulin, exercise, and blood glucose monitoring. The target blood glucose levels 1 hour after a meal should be . ANS: 130 to 140 mg/dL Target levels of blood glucose during pregnancy are lower than nonpregnant values. Accepted fasting levels are between 65 and 95 mg/dL, and 1-hour postmeal levels should be less than 130 to 140 mg/dL. Two-hour postmeal levels should be 120 mg/dL or less. PTS: 1 DIF: Cognitive Level: Application OBJ: Nursing Process: Implementation MSC: Client Needs: Health Promotion and Maintenance MATCHING You are preparing to teach an antepartum patient with gestational diabetes the correct method of administering an intermediate acting insulin (NPH) with a short-acting insulin (regular). In the correct order from 1 through 6, match the step number with the action that you would take to teach the patient self-administration of this combination of insulin. a. Without adding air, withdraw the correct dose of NPH insulin. b. Gently rotate the insulin to mix it, and wipe the stopper. c. Inject air equal to the dose of NPH insulin into the vial, and remove the syringe. d. Inject air equal to the dose of regular insulin into the vial, and withdraw the medication. e. Check the insulin bottles for the expiration date. f. Wash hands. 1. Step 1 2. Step 2 3. Step 3 4. Step 4 5. Step 5 6. Step 6 1. ANS: F PTS: 1 DIF: Cognitive Level: Application OBJ: Nursing Process: Implementation MSC: Client Needs: Safe and Effective Care Environment NOT: The regular insulin is always drawn up first when combining insulin. Other steps include ensuring that the insulin syringe corresponds to the concentration of insulin that you are using. The bottle should be checked before withdrawing the medication, to be certain that it is the appropriate type. 2. ANS: E PTS: 1 DIF: Cognitive Level: Application OBJ: Nursing Process: Implementation MSC: Client Needs: Safe and Effective Care Environment NOT: The regular insulin is always drawn up first when combining insulin. Other steps include ensuring that the insulin syringe corresponds to the concentration of insulin that you are using. The bottle should be checked before withdrawing the medication, to be certain that it is the appropriate type. 3. ANS: B PTS: 1 NURSINDGITFB: .CCOoMgnitive Level: Application OBJ: Nursing Process: Implementation MSC: Client Needs: Safe and Effective Care Environment NOT: The regular insulin is always drawn up first when combining insulin. Other steps include ensuring that the insulin syringe corresponds to the concentration of insulin that you are using. The bottle should be checked before withdrawing the medication, to be certain that it is the appropriate type. 4. ANS: C PTS: 1 DIF: Cognitive Level: Application OBJ: Nursing Process: Implementation MSC: Client Needs: Safe and Effective Care Environment NOT: The regular insulin is always drawn up first when combining insulin. Other steps include ensuring that the insulin syringe corresponds to the concentration of insulin that you are using. The bottle should be checked before withdrawing the medication, to be certain that it is the appropriate type. 5. ANS: D PTS: 1 DIF: Cognitive Level: Application OBJ: Nursing Process: Implementation MSC: Client Needs: Safe and Effective Care Environment NOT: The regular insulin is always drawn up first when combining insulin. Other steps include ensuring that the insulin syringe corresponds to the concentration of insulin that you are using. The bottle should be checked before withdrawing the medication, to be certain that it is the appropriate type. 6. ANS: A PTS: 1 DIF: Cognitive Level: Application OBJ: Nursing Process: Implementation MSC: Client Needs: Safe and Effective Care Environment NOT: The regular insulin is always drawn up first when combining insulin. Other steps include ensuring that the insulin syringe corresponds to the concentration of insulin that you are using. The bottle should be checked before withdrawing the medication, to be certain that it is the appropriate type. NURSINGTB.COM Chapter 12: High-Risk Perinatal Care: Gestational Conditions Perry: Maternal Child Nursing Care, 6th Edition MULTIPLE CHOICE 1. Women with hyperemesis gravidarum: a. are a majority because 80% of all pregnant women suffer from it at some time. b. have vomiting severe and persistent enough to cause weight loss, dehydration, and electrolyte imbalance. c. need intravenous (IV) fluid and nutrition for most of their pregnancy. d. often inspire similar, milder symptoms in their male partners and mothers. ANS: B Women with hyperemesis gravidarum have severe vomiting; however, treatment for several days sets things right in most cases. Although 80% of pregnant women experience nausea and vomiting, fewer than 1% (0.5%) proceed to this severe level. IV administration may be used at first to restore fluid levels, but it is seldom needed for very long. Women suffering from this condition want sympathy because some authorities believe that difficult relationships with mothers and/or partners may be the cause. PTS: 1 DIF: Cognitive Level: Comprehension OBJ: Nursing Process: Assessment MSC: Client Needs: Physiologic Integrity 2. Because pregnant women may need surgery during pregnancy, nurses should be aware that: a. the diagnosis of appendicitis may be difficult because the normal signs and symptoms mimic some normaNl UchRaSnINgeGsTiBn.CpOreMgnancy. b. rupture of the appendix is less likely in pregnant women because of the close monitoring. c. surgery for intestinal obstructions should be delayed as long as possible because it usually affects the pregnancy. d. when pregnancy takes over, a woman is less likely to have ovarian problems that require invasive responses. ANS: A Both appendicitis and pregnancy are linked with nausea, vomiting, and increased white blood cell count. Rupture of the appendix is two to three times more likely in pregnant women. Surgery to remove obstructions should be done right away. It usually does not affect the pregnancy. Pregnancy predisposes a woman to ovarian problems. PTS: 1 DIF: Cognitive Level: Comprehension OBJ: Nursing Process: Assessment MSC: Client Needs: Physiologic Integrity 3. What laboratory marker is indicative of disseminated intravascular coagulation (DIC)? a. Bleeding time of 10 minutes b. Presence of fibrin split products c. Thrombocytopenia d. Hyperfibrinogenemia ANS: B Degradation of fibrin leads to the accumulation of fibrin split products in the blood. Bleeding time in DIC is normal. Low platelets may occur with but are not indicative of DIC because they may result from other coagulopathies. Hypofibrinogenemia would occur with DIC. PTS: 1 DIF: Cognitive Level: Knowledge OBJ: Nursing Process: Assessment MSC: Client Needs: Physiologic Integrity 4. In caring for an immediate after birth patient, you note petechiae and oozing from her IV site. You would monitor her closely for the clotting disorder: a. disseminated intravascular coagulation (DIC). b. amniotic fluid embolism (AFE). c. hemorrhage. d. HELLP syndrome. ANS: A The diagnosis of DIC is made according to clinical findings and laboratory markers. Physical examination reveals unusual bleeding. Petechiae may appear around a blood pressure cuff on the woman's arm. Excessive bleeding may occur from the site of slight trauma such as venipuncture sites. These symptoms are not associated with AFE, nor is AFE a bleeding disorder. Hemorrhage occurs for a variety of reasons in the after birth patient. These symptoms are associated with DIC. Hemorrhage would be a finding associated with DIC and is not a clotting disorder in and of itself. HELLP is not a clotting disorder, but it may contribute to the clotting disorder DIC. PTS: 1 DIF: Cognitive Level: Comprehension OBJ: Nursing Process: Planning MSC: Client Needs: Physiologic Integrity 5. In caring for the woman with dissNemURinSaItNeGdTiBnt.CraOvMascular coagulation (DIC), what order should the nurse anticipate? a. Administration of blood b. Preparation of the patient for invasive hemodynamic monitoring c. Restriction of intravascular fluids d. Administration of steroids ANS: A Primary medical management in all cases of DIC involves correction of the underlying cause, volume replacement, blood component therapy, optimization of oxygenation and perfusion status, and continued reassessment of laboratory parameters. Central monitoring would not be ordered initially in a patient with DIC because this can contribute to more areas of bleeding. Management of DIC would include volume replacement, not volume restriction. Steroids are not indicated for the management of DIC. PTS: 1 DIF: Cognitive Level: Comprehension OBJ: Nursing Process: Planning MSC: Client Needs: Physiologic Integrity 6. A primigravida is being monitored in her prenatal clinic for preeclampsia. What finding should concern her nurse? a. Blood pressure (BP) increase to 138/86 mm Hg. b. Weight gain of 0.5 kg during the past 2 weeks. c. A dipstick value of 3+ for protein in her urine. d. Pitting pedal edema at the end of the day. ANS: C Proteinuria is defined as a concentration of 1+ or greater via dipstick measurement. A dipstick value of 3+ should alert the nurse that additional testing or assessment should be made. Generally, hypertension is defined as a BP of 140/90 or an increase in systolic pressure of 30 mm Hg or in diastolic pressure of 15 mm Hg. Preeclampsia may be manifested as a rapid weight gain of more than 2 kg in 1 week. Edema occurs in many normal pregnancies and in women with preeclampsia. Therefore, the presence of edema is no longer considered diagnostic of preeclampsia. PTS: 1 DIF: Cognitive Level: Analysis OBJ: Nursing Process: Diagnosis MSC: Client Needs: Physiologic Integrity 7. The labor of a pregnant woman with preeclampsia is going to be induced. Before initiating the Pitocin infusion, the nurse reviews the woman's latest laboratory test findings, which reveal a platelet count of 90,000, an elevated aspartate transaminase (AST) level, and a falling hematocrit. The nurse notifies the physician because the laboratory results are indicative of: a. eclampsia. b. disseminated intravascular coagulation (DIC). c. HELLP syndrome. d. idiopathic thrombocytopenia. ANS: C HELLP syndrome is a laboratory diagnosis for a variant of severe preeclampsia that involves hepatic dysfunction characterized by hemolysis (H), elevated liver enzymes (EL), and low platelets (LP). Eclampsia is determined by the presence of seizures. DIC is a potential complication associated with HELLP syndrome. Idiopathic thrombocytopenia is the presence of low platelets of unknown causeNaUnRdSiIsNnGoTtBa.CssOoMciated with preeclampsia. PTS: 1 DIF: Cognitive Level: Comprehension OBJ: Nursing Process: Diagnosis MSC: Client Needs: Physiologic Integrity 8. A woman with preeclampsia has a seizure. The nurse's primary duty during the seizure is to: a. insert an oral airway. b. suction the mouth to prevent aspiration. c. administer oxygen by mask. d. stay with the patient and call for help. ANS: D If a patient becomes eclamptic, the nurse should stay her and call for help. Insertion of an oral airway during seizure activity is no longer the standard of care. The nurse should attempt to keep the airway patent by turning the patient's head to the side to prevent aspiration. Once the seizure has ended, it may be necessary to suction the patient's mouth. Oxygen would be administered after the convulsion has ended. PTS: 1 DIF: Cognitive Level: Application OBJ: Nursing Process: Implementation MSC: Client Needs: Physiologic Integrity 9. A pregnant woman has been receiving a magnesium sulfate infusion for treatment of severe preeclampsia for 24 hours. On assessment the nurse finds the following vital signs: temperature of 37.3° C, pulse rate of 88 beats/min, respiratory rate of 10 breaths/min, blood pressure (BP) of 148/90 mm Hg, absent deep tendon reflexes, and no ankle clonus. The patient complains, “I'm so thirsty and warm.” The nurse: a. calls for a stat magnesium sulfate level. b. administers oxygen. c. discontinues the magnesium sulfate infusion. d. prepares to administer hydralazine. ANS: C The patient is displaying clinical signs and symptoms of magnesium toxicity. Magnesium should be discontinued immediately. In addition, calcium gluconate, the antidote for magnesium, may be administered. Hydralazine is an antihypertensive commonly used to treat hypertension in severe preeclampsia. Typically it is administered for a systolic BP greater than 160 mm Hg or a diastolic BP greater than 110 mm Hg. PTS: 1 DIF: Cognitive Level: Application OBJ: Nursing Process: Implementation MSC: Client Needs: Physiologic Integrity 10. A woman with severe preeclampsia has been receiving magnesium sulfate by intravenous infusion for 8 hours. The nurse assesses the woman and documents the following findings: temperature of 37.1° C, pulse rate of 96 beats/min, respiratory rate of 24 breaths/min, blood pressure (BP) of 155/112 mm Hg, 3+ deep tendon reflexes, and no ankle clonus. The nurse calls the physician, anticipating an order for: a. hydralazine. b. magnesium sulfate bolus. c. diazepam. d. calcium gluconate. ANS: A NURSINGTB.COM Hydralazine is an antihypertensive commonly used to treat hypertension in severe preeclampsia. Typically, it is administered for a systolic BP greater than 160 mm Hg or a diastolic BP greater than 110 mm Hg. An additional bolus of magnesium sulfate may be ordered for increasing signs of central nervous system irritability related to severe preeclampsia (e.g., clonus) or if eclampsia develops. Diazepam sometimes is used to stop or shorten eclamptic seizures. Calcium gluconate is used as the antidote for magnesium sulfate toxicity. The patient is not currently displaying any signs or symptoms of magnesium toxicity. PTS: 1 DIF: Cognitive Level: Analysis OBJ: Nursing Process: Planning MSC: Client Needs: Physiologic Integrity 11. A woman at 39 weeks of gestation with a history of preeclampsia is admitted to the labor and birth unit. She suddenly experiences increased contraction frequency of every 1 to 2 minutes; dark red vaginal bleeding; and a tense, painful abdomen. The nurse suspects the onset of: a. eclamptic seizure. b. rupture of the uterus. c. placenta previa. d. placental abruption. ANS: D Uterine tenderness in the presence of increasing tone may be the earliest finding of premature separation of the placenta (abruptio placentae or placental abruption). Women with hypertension are at increased risk for an abruption. Eclamptic seizures are evidenced by the presence of generalized tonic-clonic convulsions. Uterine rupture manifests as hypotonic uterine activity, signs of hypovolemia, and in many cases the absence of pain. Placenta previa manifests with bright red, painless vaginal bleeding. PTS: 1 DIF: Cognitive Level: Comprehension OBJ: Nursing Process: Diagnosis MSC: Client Needs: Physiologic Integrity 12. The patient that you are caring for has severe preeclampsia and is receiving a magnesium sulfate infusion. You become concerned after assessment when the woman exhibits: a. a sleepy, sedated affect. b. a respiratory rate of 10 breaths/min. c. deep tendon reflexes of 2. d. absent ankle clonus. ANS: B A respiratory rate of 10 breaths/min indicates that the patient is experiencing respiratory depression from magnesium toxicity. Because magnesium sulfate is a central nervous system depressant, the patient will most likely become sedated when the infusion is initiated. Deep tendon reflexes of two and absent ankle clonus are normal findings. PTS: 1 DIF: Cognitive Level: Comprehension OBJ: Nursing Process: Diagnosis MSC: Client Needs: Physiologic Integrity 13. The nurse caring for pregnant women must be aware that the most common medical complication of pregnancy is: a. hypertension. b. hyperemesis gravidarum. c. hemorrhagic complications. d. infections. ANS: A NURSINGTB.COM Preeclampsia and eclampsia are two noted deadly forms of hypertension. A large percentage of pregnant women will have nausea and vomiting, but a relatively few have the severe form called hyperemesis gravidarum. Hemorrhagic complications are the second most common medical complication of pregnancy; hypertension is the most common. PTS: 1 DIF: Cognitive Level: Comprehension OBJ: Nursing Process: Diagnosis, Planning MSC: Client Needs: Physiologic Integrity 14. Nurses should be aware that HELLP syndrome: a. is a mild form of preeclampsia. b. can be diagnosed by a nurse alert to its symptoms. c. is characterized by hemolysis, elevated liver enzymes, and low platelets. d. is associated with preterm labor but not perinatal mortality. ANS: C The acronym HELLP stands for hemolysis (H), elevated liver enzymes (EL), and low platelets (LP). HELLP syndrome is a variant of severe preeclampsia. HELLP syndrome is difficult to identify because the symptoms often are not obvious. It must be diagnosed in the laboratory. Preterm labor is greatly increased, and so is perinatal mortality. PTS: 1 DIF: Cognitive Level: Comprehension OBJ: Nursing Process: Diagnosis, Planning MSC: Client Needs: Physiologic Integrity 15. Nurses should be aware that chronic hypertension: a. is defined as hypertension that begins during pregnancy and lasts for the duration of pregnancy. b. is considered severe when the systolic blood pressure (BP) is greater than 140 mm Hg or the diastolic BP is greater than 90 mm Hg. c. is general hypertension plus proteinuria. d. can occur independently of or simultaneously with gestational hypertension. ANS: D Hypertension is present before pregnancy or diagnosed before 20 weeks of gestation and persists longer than 6 weeks after birth. The range for hypertension is systolic BP greater than 140 mm Hg or diastolic BP greater than 90 mm Hg. It becomes severe with a diastolic BP of 110 mm Hg or higher. Proteinuria is an excessive concentration of protein in the urine. It is a complication of hypertension, not a defining characteristic. PTS: 1 DIF: Cognitive Level: Comprehension OBJ: Nursing Process: Diagnosis, Planning MSC: Client Needs: Physiologic Integrity NURSINGTB.COM 16. In planning care for women with preeclampsia, nurses should be aware that: a. induction of labor is likely, as near term as possible. b. if at home, the woman should be confined to her bed, even with mild preeclampsia. c. a special diet low in protein and salt should be initiated. d. vaginal birth is still an option, even in severe cases. ANS: A Induction of labor is likely, as near term as possible; however, at less than 37 weeks of gestation, immediate delivery may not be in the best interest of the fetus. Strict bed rest is becoming controversial for mild cases; some women in the hospital are even allowed to move around. Diet and fluid recommendations are much the same as for healthy pregnant women, although some authorities have suggested a diet high in protein. Women with severe preeclampsia should expect a cesarean delivery. PTS: 1 DIF: Cognitive Level: Comprehension OBJ: Nursing Process: Planning MSC: Client Needs: Health Promotion and Maintenance 17. Magnesium sulfate is given to women with preeclampsia and eclampsia to: a. improve patellar reflexes and increase respiratory efficiency. b. shorten the duration of labor. c. prevent and treat convulsions. d. prevent a boggy uterus and lessen lochial flow. ANS: C Magnesium sulfate is the drug of choice to prevent convulsions, although it can generate other problems. Loss of patellar reflexes and respiratory depression are signs of magnesium toxicity. Magnesium sulfate can increase the duration of labor. Women are at risk for a boggy uterus and heavy lochial flow as a result of magnesium sulfate therapy. PTS: 1 DIF: Cognitive Level: Comprehension OBJ: Nursing Process: Implementation MSC: Client Needs: Physiologic Integrity 18. A woman presents to the emergency department with complaints of bleeding and cramping. The initial nursing history is significant for a last menstrual period 6 weeks ago. On sterile speculum examination, the primary care provider finds that the cervix is closed. The anticipated plan of care for this woman would be based on a probable diagnosis of which type of spontaneous abortion? a. Incomplete b. Inevitable c. Threatened d. Septic ANS: C A woman with a threatened abortion presents with spotting, mild cramps, and no cervical dilation. A woman with an incomplete abortion would present with heavy bleeding, mild to severe cramping, and cervical dilation. An inevitable abortion manifests with the same symptoms as an incomplete abortion: heavy bleeding, mild to severe cramping, and cervical dilation. A woman with a septic abortion presents with malodorous bleeding and typically a dilated cervix. PTS: 1 DIF: Cognitive Level: Comprehension OBJ: Nursing Process: Planning MSC: Client Needs: Physiologic IntNegUriRtySINGTB.COM 19. The perinatal nurse is giving discharge instructions to a woman after suction curettage secondary to a hydatidiform mole. The woman asks why she must take oral contraceptives for the next 12 months. The best response from the nurse would be: a. “If you get pregnant within 1 year, the chance of a successful pregnancy is very small. Therefore, if you desire a future pregnancy, it would be better for you to use the most reliable method of contraception available.” b. “The major risk to you after a molar pregnancy is a type of cancer that can be diagnosed only by measuring the same hormone that your body produces during pregnancy. If you were to get pregnant, it would make the diagnosis of this cancer more difficult.” c. “If you can avoid a pregnancy for the next year, the chance of developing a second molar pregnancy is rare. Therefore, to improve your chance of a successful pregnancy, it is better not to get pregnant at this time.” d. “Oral contraceptives are the only form of birth control that will prevent a recurrence of a molar pregnancy.” ANS: B This is an accurate statement. Beta-human chorionic gonadotropin (hCG) levels will be drawn for 1 year to ensure that the mole is completely gone. There is an increased chance of developing choriocarcinoma after the development of a hydatidiform mole. The goal is to achieve a “zero” hCG level. If the woman were to become pregnant, it could obscure the presence of the potentially carcinogenic cells. Women should be instructed to use birth control for 1 year after treatment for a hydatidiform mole. The rationale for avoiding pregnancy for 1 year is to ensure that carcinogenic cells are not present. Any contraceptive method except an intrauterine device is acceptable. PTS: 1 DIF: Cognitive Level: Application OBJ: Nursing Process: Planning, Implementation MSC: Client Needs: Physiologic Integrity 20. The most prevalent clinical manifestation of abruptio placentae (as opposed to placenta previa) is: a. bleeding. b. intense abdominal pain. c. uterine activity. d. cramping. ANS: B Pain is absent with placenta previa and may be agonizing with abruptio placentae. Bleeding may be present in varying degrees for both placental conditions. Uterine activity and cramping may be present with both placental conditions. PTS: 1 DIF: Cognitive Level: Knowledge OBJ: Nursing Process: Diagnosis MSC: Client Needs: Physiologic Integrity NURSINGTB.COM 21. Methotrexate is recommended as part of the treatment plan for which obstetric complication? a. Complete hydatidiform mole b. Missed abortion c. Unruptured ectopic pregnancy d. Abruptio placentae ANS: C Methotrexate is an effective, nonsurgical treatment option for a hemodynamically stable woman whose ectopic pregnancy is unruptured and less than 4 cm in diameter. Methotrexate is not indicated or recommended as a treatment option for complete hydatidiform mole, missed abortion, and abruptio placentae. PTS: 1 DIF: Cognitive Level: Knowledge OBJ: Nursing Process: Planning MSC: Client Needs: Physiologic Integrity 22. A 26-year-old pregnant woman, gravida 2, para 1-0-0-1 is 28 weeks pregnant when she experiences bright red, painless vaginal bleeding. On her arrival at the hospital, what would be an expected diagnostic procedure? a. Amniocentesis for fetal lung maturity b. Ultrasound for placental location c. Contraction stress test (CST) d. Internal fetal monitoring ANS: B The presence of painless bleeding should always alert the health care team to the possibility of placenta previa. This can be confirmed through ultrasonography. Amniocentesis would not be performed on a woman who is experiencing bleeding. In the event of an imminent delivery, the fetus would be presumed to have immature lungs at this gestational age, and the mother would be given corticosteroids to aid in fetal lung maturity. A CST would not be performed at a preterm gestational age. Furthermore, bleeding would be a contraindication to this test. Internal fetal monitoring would be contraindicated in the presence of bleeding. PTS: 1 DIF: Cognitive Level: Application OBJ: Nursing Process: Assessment MSC: Client Needs: Health Promotion and Maintenance 23. A laboring woman with no known risk factors suddenly experiences spontaneous rupture of membranes (ROM). The fluid consists of bright red blood. Her contractions are consistent with her current stage of labor. There is no change in uterine resting tone. The fetal heart rate begins to decline rapidly after the ROM. The nurse should suspect the possibility of: a. placenta previa. b. vasa previa. c. severe abruptio placentae. d. disseminated intravascular coagulation (DIC). ANS: B Vasa previa is the result of a velamentous insertion of the umbilical cord. The umbilical vessels are not surrounded by Wharton jelly and have no supportive tissue. They are at risk for laceration at any time, but laceration occurs most frequently during ROM. The sudden appearance of bright red blood at the time of ROM and a sudden change in the fetal heart rate without other known risk factors should immediately alert the nurse to the possibility of vasa previa. The presence of placenta pNreUvRiSaImNGoTstBl.iCkOeMly would be ascertained before labor and would be considered a risk factor for this pregnancy. In addition, if the woman had a placenta previa, it is unlikely that she would be allowed to pursue labor and a vaginal birth. With the presence of severe abruptio placentae, the uterine tonicity would typically be tetanus (i.e., a board-like uterus). DIC is a pathologic form of diffuse clotting that consumes large amounts of clotting factors and causes widespread external bleeding, internal bleeding, or both. DIC is always a secondary diagnosis, often associated with obstetric risk factors such as HELLP syndrome. This woman did not have any prior risk factors. PTS: 1 DIF: Cognitive Level: Analysis OBJ: Nursing Process: Diagnosis MSC: Client Needs: Physiologic Integrity 24. A woman arrives for evaluation of her symptoms, which include a missed period, adnexal fullness, tenderness, and dark red vaginal bleeding. On examination the nurse notices an ecchymotic blueness around the woman's umbilicus and recognizes this assessment finding as: a. normal integumentary changes associated with pregnancy. b. Turner's sign associated with appendicitis. c. Cullen's sign associated with a ruptured ectopic pregnancy. d. Chadwick's sign associated with early pregnancy. ANS: C Cullen's sign, the blue ecchymosis seen in the umbilical area, indicates hematoperitoneum associated with an undiagnosed ruptured intraabdominal ectopic pregnancy. Linea nigra on the abdomen is the normal integumentary change associated with pregnancy. It manifests as a brown, pigmented, vertical line on the lower abdomen. Turner's sign is ecchymosis in the flank area, often associated with pancreatitis. Chadwick's sign is the blue-purple color of the cervix that may be seen during or around the eighth week of pregnancy. PTS: 1 DIF: Cognitive Level: Analysis OBJ: Nursing Process: Assessment MSC: Client Needs: Physiologic Integrity 25. As related to the care of the patient with miscarriage, nurses should be aware that: a. it is a natural pregnancy loss before labor begins. b. it occurs in fewer than 5% of all clinically recognized pregnancies. c. it often can be attributed to careless maternal behavior such as poor nutrition or excessive exercise. d. if it occurs before the 12th week of pregnancy, it may manifest only as moderate discomfort and blood loss. ANS: D Before the sixth week the only evidence may be a heavy menstrual flow. After the 12th week more severe pain, similar to that of labor, is likely. Miscarriage is a natural pregnancy loss, but by definition it occurs before 20 weeks of gestation, before the fetus is viable. Miscarriages occur in approximately 10% to 15% of all clinically recognized pregnancies. Miscarriage can be caused by a number of disorders or illnesses outside of the mother's control or knowledge. PTS: 1 DIF: Cognitive Level: Comprehension OBJ: Nursing Process: AssessmentNURSINMGSTCB.:CCOlMient Needs: Physiologic Integrity 26. Which condition would not be classified as a bleeding disorder in late pregnancy? a. Placenta previa b. Abruptio placentae c. Spontaneous abortion d. Cord insertion ANS: C Spontaneous abortion is another name for miscarriage; by definition it occurs early in pregnancy. Placenta previa is a cause of bleeding disorders in later pregnancy. Abruptio placentae is a cause of bleeding disorders in later pregnancy. Cord insertion is a cause of bleeding disorders in later pregnancy. PTS: 1 DIF: Cognitive Level: Knowledge OBJ: Nursing Process: Assessment MSC: Client Needs: Physiologic Integrity 27. In providing nutritional counseling for the pregnant woman experiencing cholecystitis, the nurse would: a. assess the woman's dietary history for adequate calories and proteins. b. instruct the woman that the bulk of calories should come from proteins. c. instruct the woman to eat a low-fat diet and avoid fried foods. d. instruct the woman to eat a low-cholesterol, low-salt diet. ANS: C Instructing the woman to eat a low-fat diet and avoid fried foods is appropriate nutritional counseling for this patient. Caloric and protein intake do not predispose a woman to the development of cholecystitis. The woman should be instructed to limit protein intake and choose foods that are high in carbohydrates. A low-cholesterol diet may be the result of limiting fats. However, a low-salt diet is not indicated. PTS: 1 DIF: Cognitive Level: Application OBJ: Nursing Process: Implementation MSC: Client Needs: Physiologic Integrity 28. Which maternal condition always necessitates delivery by cesarean section? a. Partial abruptio placentae b. Total placenta previa c. Ectopic pregnancy d. Eclampsia ANS: B In total placenta previa, the placenta completely covers the cervical os. The fetus would die if a vaginal delivery occurred. If the mother has stable vital signs and the fetus is alive, a vaginal delivery can be attempted in cases of partial abruptio placentae. If the fetus has died, a vaginal delivery is preferred. The most common ectopic pregnancy is a tubal pregnancy, which is usually detected and treated in the first trimester. Labor can be safely induced if the eclampsia is under control. PTS: 1 DIF: Cognitive Level: Comprehension OBJ: Nursing Process: Assessment MSC: Client Needs: Physiologic Integrity 29. Spontaneous termination of a pregnancy is considered to be an abortion if: a. the pregnancy is less than 20 wNeUeRkSsI.NGTB.COM b. the fetus weighs less than 1000 g. c. the products of conception are passed intact. d. no evidence exists of intrauterine infection. ANS: A An abortion is the termination of pregnancy before the age of viability (20 weeks). The weight of the fetus is not considered because some older fetuses may have a low birth weight. A spontaneous abortion may be complete or incomplete. A spontaneous abortion may be caused by many problems, one being intrauterine infection. PTS: 1 DIF: Cognitive Level: Knowledge OBJ: Nursing Process: Assessment MSC: Client Needs: Health Promotion and Maintenance 30. An abortion in which the fetus dies but is retained within the uterus is called a(n): a. inevitable abortion. b. missed abortion. c. incomplete abortion. d. threatened abortion. ANS: B Missed abortion refers to retention of a dead fetus in the uterus. An inevitable abortion means that the cervix is dilating with the contractions. An incomplete abortion means that not all of the products of conception were expelled. With a threatened abortion the woman has cramping and bleeding but not cervical dilation. PTS: 1 DIF: Cognitive Level: Knowledge OBJ: Nursing Process: Assessment MSC: Client Needs: Physiologic Integrity 31. A placenta previa in which the placental edge just reaches the internal os is more commonly known as: a. total. b. partial. c. complete. d. marginal. ANS: D A placenta previa that does not cover any part of the cervix is termed marginal. With a total placenta previa, the placenta completely covers the os. When the patient experiences a partial placenta previa, the lower border of the placenta is within 3 cm of the internal cervical os but does not completely cover the os. A complete placenta previa is termed total. The placenta completely covers the internal cervical os. PTS: 1 DIF: Cognitive Level: Knowledge OBJ: Nursing Process: Assessment MSC: Client Needs: Physiologic Integrity 32. Which condition indicates concealed hemorrhage when the patient experiences an abruptio placentae? a. Decrease in abdominal pain b. Bradycardia c. Hard, board-like abdomen d. Decrease in fundal height ANS: C NURSINGTB.COM Concealed hemorrhage occurs when the edges of the placenta do not separate. The formation of a hematoma behind the placenta and subsequent infiltration of the blood into the uterine muscle results in a very firm, board-like abdomen. Abdominal pain may increase. The patient will have shock symptoms that include tachycardia. As bleeding occurs, the fundal height will increase. PTS: 1 DIF: Cognitive Level: Analysis OBJ: Nursing Process: Assessment MSC: Client Needs: Physiologic Integrity 33. The priority nursing intervention when admitting a pregnant woman who has experienced a bleeding episode in late pregnancy is to: a. assess fetal heart rate (FHR) and maternal vital signs. b. perform a venipuncture for hemoglobin and hematocrit levels. c. place clean disposable pads to collect any drainage. d. monitor uterine contractions. ANS: A Assessment of the FHR and maternal vital signs will assist the nurse in determining the degree of the blood loss and its effect on the mother and fetus. The most important assessment is to check mother/fetal well-being. The blood levels can be obtained later. It is important to assess future bleeding; however, the top priority remains mother/fetal well-being. Monitoring uterine contractions is important but not the top priority. PTS: 1 DIF: Cognitive Level: Application OBJ: Nursing Process: Implementation MSC: Client Needs: Health Promotion and Maintenance 34. A patient with pregnancy-induced hypertension is admitted complaining of pounding headache, visual changes, and epigastric pain. Nursing care is based on the knowledge that these signs are an indication of: a. anxiety due to hospitalization. b. worsening disease and impending convulsion. c. effects of magnesium sulfate. d. gastrointestinal upset. ANS: B Headache and visual disturbances are caused by increased cerebral edema. Epigastric pain indicates distention of the hepatic capsules and often warns that a convulsion is imminent. These are danger signs showing increased cerebral edema and impending convulsion and should be treated immediately. The patient has not been started on magnesium sulfate treatment yet. Also, these are not anticipated effects of the medication. PTS: 1 DIF: Cognitive Level: Analysis OBJ: Nursing Process: Assessment MSC: Client Needs: Physiologic Integrity 35. Which order should the nurse expect for a patient admitted with a threatened abortion? a. Bed rest b. Ritodrine IV c. NPO d. Narcotic analgesia every 3 hours, prn ANS: A NURSINGTB.COM Decreasing the woman's activity level may alleviate the bleeding and allow the pregnancy to continue. Ritodrine IV is not the first drug of choice for tocolytic medications. There is no reason for having the woman placed NPO. At times dehydration may produce contractions, so hydration is important. Narcotic analgesia will not decrease the contractions. It may mask the severity of the contractions. PTS: 1 DIF: Cognitive Level: Comprehension OBJ: Nursing Process: Planning MSC: Client Needs: Health Promotion and Maintenance 36. A 32-year-old primigravida is admitted with a diagnosis of ectopic pregnancy. Nursing care is based on the knowledge that: a. bed rest and analgesics are the recommended treatment. b. she will be unable to conceive in the future. c. a D&C will be performed to remove the products of conception. d. hemorrhage is the major concern. ANS: D Severe bleeding occurs if the fallopian tube ruptures. The recommended treatment is to remove the pregnancy before rupture in order to prevent hemorrhaging. If the tube must be removed, the woman's fertility will decrease; however, she will not be infertile. D&C is performed on the inside of the uterine cavity. The ectopic pregnancy is located within the tubes. PTS: 1 DIF: Cognitive Level: Comprehension OBJ: Nursing Process: Planning MSC: Client Needs: Physiologic Integrity 37. Approximately 10% to 15% of all clinically recognized pregnancies end in miscarriage. Which is the most common cause of spontaneous abortion? a. Chromosomal abnormalities b. Infections c. Endocrine imbalance d. Immunologic factors ANS: A At least 50% of pregnancy losses result from chromosomal abnormalities that are incompatible with life. Maternal infection may be a cause of early miscarriage. Endocrine imbalances such as hypothyroidism or diabetes are possible causes for early pregnancy loss. Women who have repeated early pregnancy losses appear to have immunologic factors that play a role in spontaneous abortion incidents. PTS: 1 DIF: Cognitive Level: Knowledge OBJ: Nursing Process: Assessment MSC: Client Needs: Health Promotion and Maintenance 38. The nurse caring for a woman hospitalized for hyperemesis gravidarum should expect that initial treatment to involve: a. corticosteroids to reduce inflammation. b. IV therapy to correct fluid and electrolyte imbalances. c. an antiemetic, such as pyridoxine, to control nausea and vomiting. d. enteral nutrition to correct nutritional deficits. ANS: B Initially, the woman who is unableNUtoRkSeINepGTdBo.wCOn Mclear liquids by mouth requires IV therapy for correction of fluid and electrolyte imbalances. Corticosteroids have been used successfully to treat refractory hyperemesis gravidarum; however, they are not the expected initial treatment for this disorder. Pyridoxine is vitamin B6, not an antiemetic. Promethazine, a common antiemetic, may be prescribed. In severe cases of hyperemesis gravidarum, enteral nutrition via a feeding tube may be necessary to correct maternal nutritional deprivation. This is not an initial treatment for this patient. PTS: 1 DIF: Cognitive Level: Comprehension OBJ: Nursing Process: Implementation MSC: Client Needs: Physiologic Integrity MULTIPLE RESPONSE 1. A patient who has undergone a dilation and curettage for early pregnancy loss is likely to be discharged the same day. The nurse must ensure that vital signs are stable, bleeding has been controlled, and the woman has adequately recovered from the administration of anesthesia. To promote an optimal recovery, discharge teaching should include: (Select all that apply.) a. iron supplementation. b. resumption of intercourse at 6 weeks following the procedure. c. referral to a support group if necessary. d. expectation of heavy bleeding for at least 2 weeks. e. emphasizing the need for rest. ANS: A, C, E The woman should be advised to consume a diet high in iron and protein. For many women iron supplementation is also necessary. Acknowledge that the patient has experienced a loss, albeit early. She can be taught to expect mood swings and possibly depression. Referral to a support group, clergy, or professional counseling may be necessary. Discharge teaching should emphasize the need for rest. Nothing should be placed in the vagina for 2 weeks after the procedure. This includes tampons and vaginal intercourse. The purpose of this recommendation is to prevent infection. Should infection occur, antibiotics may be prescribed. The patient should expect a scant, dark discharge for 1 to 2 weeks. Should heavy, profuse, or bright bleeding occur, she should be instructed to contact her provider. PTS: 1 DIF: Cognitive Level: Application OBJ: Nursing Process: Implementation MSC: Client Needs: Physiologic Integrity 2. The reported incidence of ectopic pregnancy in the United States has risen steadily over the past two decades. Causes include the increase in STDs accompanied by tubal infection and damage. The popularity of contraceptive devices such as the IUD has also increased the risk for ectopic pregnancy. The nurse who suspects that a patient has early signs of ectopic pregnancy should be observing her for symptoms such as: (Select all that apply.) a. pelvic pain. b. abdominal pain. c. unanticipated heavy bleeding. d. vaginal spotting or light bleeding. e. missed period. ANS: A, B, D, E A missed period or spotting can easily be mistaken by the patient as early signs of pregnancy. More subtle signs depend on exacNtlUyRwShINerGeTtBh.eCOimMplantation occurs. The nurse must be thorough in her assessment because pain is not a normal symptom of early pregnancy. As the fallopian tube tears open and the embryo is expelled, the patient often exhibits severe pain accompanied by intra-abdominal hemorrhage. This may progress to hypovolemic shock with minimal or even no external bleeding. In about half of women, shoulder and neck pain results from irritation of the diaphragm from the hemorrhage. PTS: 1 DIF: Cognitive Level: Application OBJ: Nursing Process: Assessment MSC: Client Needs: Physiologic Integrity MATCHING Because most pregnant women continue their usual activities, trauma remains a common complication during pregnancy. Approximately 30,000 women in the United States experience treatable injuries related to trauma each year. As a result of the physiologic alterations that accompany pregnancy, special considerations for mother and fetus are necessary when trauma occurs. Match the maternal system adaptation in pregnancy with the clinical response to trauma. a. Increased oxygen consumption b. Increased heart rate c. Decreased gastric motility d. Displacement of abdominal viscera e. Increase in clotting factors 1. Decreased placental perfusion in supine position 2. Increased risk of thrombus formation 3. Altered pain referral 4. Increased risk of acidosis 5. Increased risk of aspiration 1. ANS: B PTS: 1 DIF: Cognitive Level: Application OBJ: Nursing Process: Assessment MSC: Client Needs: Physiologic Integrity NOT: Immediate priorities for the stabilization of the pregnant woman after trauma should be identical to that of the nonpregnant trauma patient. Fetal survival depends on maternal survival, and stabilization of the mother improves the chance of fetal well-being. Trauma may affect a number of systems within the body, and it is important for the nurse caring for this patient to be aware of normal system alterations in the pregnant woman. Care should be adapted according to the body system that has been injured. The effects of trauma on pregnancy are also influenced by the length of gestation, type and severity, and the degree of disruption of uterine and fetal physiologic features. 2. ANS: E PTS: 1 DIF: Cognitive Level: Application OBJ: Nursing Process: Assessment MSC: Client Needs: Physiologic Integrity NOT: Immediate priorities for the stabilization of the pregnant woman after trauma should be identical to that of the nonpregnant trauma patient. Fetal survival depends on maternal survival, and stabilization of the mother improves the chance of fetal wellbeing. Trauma may affect a number of systems within the body, and it is important for the nurse caring for this patient to be aware of normal system alterations in the pregnant woman. Care should be adapted according to the body system that has been injured. The effects of trauma on pregnancy are also influenced by the length of gestation, type and severity, and the degree of disruption of uterine and fetal physiologic features. 3. ANS: D PTS: 1 DIF: Cognitive Level: Application OBJ: Nursing Process: Assessment MSC: Client Needs: Physiologic Integrity NOT: Immediate priorities for the stabilization of the pregnant woman after trauma should be identical to that of the nonpregnant trNauUmRSaIpNaGtiTenBt..CFOeMtal survival depends on maternal survival, and stabilization of the mother improves the chance of fetal wellbeing. Trauma may affect a number of systems within the body, and it is important for the nurse caring for this patient to be aware of normal system alterations in the pregnant woman. Care should be adapted according to the body system that has been injured. The effects of trauma on pregnancy are also influenced by the length of gestation, type and severity, and the degree of disruption of uterine and fetal physiologic features. 4. ANS: A PTS: 1 DIF: Cognitive Level: Application OBJ: Nursing Process: Assessment MSC: Client Needs: Physiologic Integrity NOT: Immediate priorities for the stabilization of the pregnant woman after trauma should be identical to that of the nonpregnant trauma patient. Fetal survival depends on maternal survival, and stabilization of the mother improves the chance of fetal wellbeing. Trauma may affect a number of systems within the body, and it is important for the nurse caring for this patient to be aware of normal system alterations in the pregnant woman. Care should be adapted according to the body system that has been injured. The effects of trauma on pregnancy are also influenced by the length of gestation, type and severity, and the degree of disruption of uterine and fetal physiologic features. 5. ANS: C PTS: 1 DIF: Cognitive Level: Application OBJ: Nursing Process: Assessment MSC: Client Needs: Physiologic Integrity NOT: Immediate priorities for the stabilization of the pregnant woman after trauma should be identical to that of the nonpregnant trauma patient. Fetal survival depends on maternal survival, and stabilization of the mother improves the chance of fetal wellbeing. Trauma may affect a number of systems within the body, and it is important for the nurse caring for this patient to be aware of normal system alterations in the pregnant woman. Care should be adapted according to the body system that has been injured. The effects of trauma on pregnancy are also influenced by the length of gestation, type and severity, and the degree of disruption of uterine and fetal physiologic features. Chapter 13: Labor and Birth Processes Perry: Maternal Child Nursing Care, 6th Edition MULTIPLE CHOICE 1. A new mother asks the nurse when the “soft spot” on her son's head will go away. The nurse's answer is based on the knowledge that the anterior fontanel closes after birth by months. a. 2 b. 8 c. 12 d. 18 ANS: D The larger of the two fontanels, the anterior fontanel, closes by 18 months after birth. PTS: 1 DIF: Cognitive Level: Knowledge OBJ: Nursing Process: Planning MSC: Client Needs: Health Promotion and Maintenance 2. When assessing a woman in labor, the nurse is aware that the relationship of the fetal body parts to one another is called fetal: a. lie. b. presentation. c. attitude. d. position. ANS: C NURSINGTB.COM Attitude is the relation of the fetal body parts to one another. Lie is the relation of the long axis (spine) of the fetus to the long axis (spine) of the mother. Presentation refers to the part of the fetus that enters the pelvic inlet first and leads through the birth canal during labor at term. Position is the relation of the presenting part to the four quadrants of the mother's pelvis. PTS: 1 DIF: Cognitive Level: Knowledge OBJ: Nursing Process: Assessment MSC: Client Needs: Health Promotion and Maintenance 3. When assessing the fetus using Leopold maneuvers, the nurse feels a round, firm, movable fetal part in the fundal portion of the uterus and a long, smooth surface in the mother's right side close to midline. What is the likely position of the fetus? a. ROA b. LSP c. RSA d. LOA ANS: C The fetus is positioned anteriorly in the right side of the maternal pelvis with the sacrum as the presenting part. RSA is the correct three-letter abbreviation to indicate this fetal position. The first letter indicates the presenting part in either the right or left side of the maternal pelvis. The second letter indicates the anatomic presenting part of the fetus. The third letter stands for the location of the presenting part in relation to the anterior, posterior, or transverse portion of the maternal pelvis. Palpation of a round, firm fetal part in the fundal portion of the uterus would be the fetal head, indicating that the fetus is in a breech position with the sacrum as the presenting part in the maternal pelvis. Palpation of the fetal spine along the mother's right side denotes the location of the presenting part in the mother's pelvis. The ability to palpate the fetal spine indicates that the fetus is anteriorly positioned in the maternal pelvis. PTS: 1 DIF: Cognitive Level: Comprehension OBJ: Nursing Process: Assessment MSC: Client Needs: Health Promotion and Maintenance 4. The nurse has received report regarding her patient in labor. The woman's last vaginal examination was recorded as 3 cm, 30%, and –2. The nurse's interpretation of this assessment is that: a. the cervix is effaced 3 cm, it is dilated 30%, and the presenting part is 2 cm above the ischial spines. b. the cervix is 3 cm dilated, it is effaced 30%, and the presenting part is 2 cm above the ischial spines. c. the cervix is effaced 3 cm, it is dilated 30%, and the presenting part is 2 cm below the ischial spines. d. the cervix is dilated 3 cm, it is effaced 30%, and the presenting part is 2 cm below the ischial spines. ANS: B NURSINGTB.COM The correct description of the vaginal examination for this woman in labor is the cervix is 3 cm dilated, it is effaced 30%, and the presenting part is 2 cm above the ischial spines. The sterile vaginal examination is recorded as centimeters of cervical dilation, percentage of cervical dilation, and the relationship of the presenting part to the ischial spines (either above or below). PTS: 1 DIF: Cognitive Level: Comprehension OBJ: Nursing Process: Assessment, Planning MSC: Client Needs: Health Promotion and Maintenance 5. To care for a laboring woman adequately, the nurse understands that the stage of labor varies the most in length? a. first b. second c. third d. fourth ANS: A The first stage of labor is considered to last from the onset of regular uterine contractions to full dilation of the cervix. The first stage is much longer than the second and third stages combined. In a first-time pregnancy the first stage of labor can take up to 20 hours. The second stage of labor lasts from the time the cervix is fully dilated to the birth of the fetus. The average length is 20 minutes for a multiparous woman and 50 minutes for a nulliparous woman. The third stage of labor lasts from the birth of the fetus until the placenta is delivered. This stage may be as short as 3 minutes or as long as 1 hour. The fourth stage of labor, recovery, lasts about 2 hours after delivery of the placenta. PTS: 1 DIF: Cognitive Level: Knowledge OBJ: Nursing Process: Planning MSC: Client Needs: Health Promotion and Maintenance 6. The nurse would expect which maternal cardiovascular finding during labor? a. Increased cardiac output b. Decreased pulse rate c. Decreased white blood cell (WBC) count d. Decreased blood pressure ANS: A During each contraction, 400 mL of blood is emptied from the uterus into the maternal vascular system. This increases cardiac output by about 51% above baseline pregnancy values at term. The heart rate increases slightly during labor. The WBC count can increase during labor. During the first stage of labor, uterine contractions cause systolic readings to increase by about 10 mm Hg. During the second stage, contractions may cause systolic pressures to increase by 30 mm Hg and diastolic readings to increase by 25 mm Hg. PTS: 1 DIF: Cognitive Level: Comprehension OBJ: Nursing Process: Diagnosis MSC: Client Needs: Health PromotiNonUaRnSdINMGaTiBnt.eCnOaMnce 7. The factors that affect the process of labor and birth, known commonly as the five Ps, include all except: a. passenger. b. passageway. c. powers. d. pressure. ANS: D The five Ps are passenger (fetus and placenta), passageway (birth canal), powers (contractions), position of the mother, and psychologic response. PTS: 1 DIF: Cognitive Level: Knowledge OBJ: Nursing Process: Assessment MSC: Client Needs: Health Promotion and Maintenance 8. The slight overlapping of cranial bones or shaping of the fetal head during labor is called: a. lightening. b. molding. c. Ferguson reflex. d. Valsalva maneuver. Fetal head formation is called molding. Molding also permits adaptation to various diameters of the maternal pelvis. Lightening is the mother's sensation of decreased abdominal distention, which usually occurs the week before labor. The Ferguson reflex is the contraction urge of the uterus after stimulation of the cervix. The Valsalva maneuver describes conscious pushing during the second stage of labor. PTS: 1 DIF: Cognitive Level: Comprehension OBJ: Nursing Process: Assessment MSC: Client Needs: Health Promotion and Maintenance 9. Which presentation is described accurately in terms of both presenting part and frequency of occurrence? a. Cephalic: occiput; at least 95% b. Breech: sacrum; 10% to 15% c. Shoulder: scapula; 10% to 15% d. Cephalic: cranial; 80% to 85% ANS: A In cephalic presentations (head first), the presenting part is the occiput; this occurs in 96% of births. In a breech birth, the sacrum emerges first; this occurs in about 3% of births. In shoulder presentations, the scapula emerges first; this occurs in only 1% of births. PTS: 1 DIF: Cognitive Level: Comprehension OBJ: Nursing Process: Diagnosis MSC: Client Needs: Health Promotion and Maintenance 10. With regard to factors that affect how the fetus moves through the birth canal, nurses should be aware that: a. the fetal attitude describes the angle at which the fetus exits the uterus. b. of the two primary fetal lies, thNeUhRoSrIiNzGoTntBa.lClOieMis that in which the long axis of the fetus is parallel to the long axis of the mother. c. the normal attitude of the fetus is called general flexion. d. the transverse lie is preferred for vaginal birth. ANS: C The normal attitude of the fetus is general flexion. The fetal attitude is the relation of fetal body parts to one another. The horizontal lie is perpendicular to the mother; in the longitudinal (or vertical) lie the long axes of the fetus and the mother are parallel. Vaginal birth cannot occur if the fetus stays in a transverse lie. PTS: 1 DIF: Cognitive Level: Comprehension OBJ: Nursing Process: Planning MSC: Client Needs: Health Promotion and Maintenance 11. As relates to fetal positioning during labor, nurses should be aware that: a. position is a measure of the degree of descent of the presenting part of the fetus through the birth canal. b. birth is imminent when the presenting part is at +4 to +5 cm below the spine. c. the largest transverse diameter of the presenting part is the suboccipitobregmatic diameter. d. engagement is the term used to describe the beginning of labor. The station of the presenting part should be noted at the beginning of labor so that the rate of descent can be determined. Position is the relation of the presenting part of the fetus to the four quadrants of the mother's pelvis; station is the measure of degree of descent. The largest diameter usually is the biparietal diameter. The suboccipitobregmatic diameter is the smallest, although one of the most critical. Engagement often occurs in the weeks just before labor in nulliparas and before or during labor in multiparas. PTS: 1 DIF: Cognitive Level: Knowledge OBJ: Nursing Process: Assessment MSC: Client Needs: Health Promotion and Maintenance 12. Which basic type of pelvis includes the correct description and percentage of occurrence in women? a. Gynecoid: classic female; heart shaped; 75% b. Android: resembling the male; wider oval; 15% c. Anthropoid: resembling the ape; narrower; 10% d. Platypelloid: flattened, wide, shallow; 3% ANS: D A platypelloid pelvis is flattened, wide, and shallow; about 3% of women have this shape. The gynecoid shape is the classical female shape, slightly ovoid and rounded; about 50% of women have this shape. An android, or male-like, pelvis is heart shaped; about 23% of women have this shape. An anthropoid, or ape-like, pelvis is oval and wider; about 24% of women have this shape. PTS: 1 DIF: Cognitive Level: Comprehension OBJ: Nursing Process: Assessment MSC: Client Needs: Health Promotion and Maintenance 13. In relation to primary and secondaNrUy RpSoIwNeGrTsB, t.ChOe Mmaternity nurse comprehends that: a. primary powers are responsible for effacement and dilation of the cervix. b. effacement generally is well ahead of dilation in women giving birth for the first time; they are closer together in subsequent pregnancies. c. scarring of the cervix caused by a previous infection or surgery may make the delivery a bit more painful, but it should not slow or inhibit dilation. d. pushing in the second stage of labor is more effective if the woman can breathe deeply and control some of her involuntary needs to push, as the nurse directs. ANS: A The primary powers are responsible for dilation and effacement; secondary powers are concerned with expulsion of the fetus. Effacement generally is well ahead of dilation in first-timers; they are closer together in subsequent pregnancies. Scarring of the cervix may slow dilation. Pushing is more effective and less fatiguing when the woman begins to push only after she has the urge to do so. PTS: 1 DIF: Cognitive Level: Comprehension OBJ: Nursing Process: Planning MSC: Client Needs: Health Promotion and Maintenance 14. While providing care to a patient in active labor, the nurse should instruct the woman that: a. the supine position commonly used in the United States increases blood flow. b. the “all fours” position, on her hands and knees, is hard on her back. c. frequent changes in position will help relieve her fatigue and increase her comfort. d. in a sitting or squatting position, her abdominal muscles will have to work harder. ANS: C Frequent position changes relieve fatigue, increase comfort, and improve circulation. Blood flow can be compromised in the supine position; any upright position benefits cardiac output. The “all fours” position is used to relieve backache in certain situations. In a sitting or squatting position, the abdominal muscles work in greater harmony with uterine contractions. PTS: 1 DIF: Cognitive Level: Application OBJ: Nursing Process: Planning, Implementation MSC: Client Needs: Health Promotion and Maintenance 15. Which description of the four stages of labor is correct for both definition and duration? a. First stage: onset of regular uterine contractions to full dilation; less than 1 hour to 20 hours b. Second stage: full effacement to 4 to 5 cm; visible presenting part; 1 to 2 hours c. Third state: active pushing to birth; 20 minutes (multiparous women), 50 minutes (first-timer) d. Fourth stage: delivery of the placenta to recovery; 30 minutes to 1 hour ANS: A Full dilation may occur in less than 1 hour, but in first-time pregnancy it can take up to 20 hours. The second stage extends from full dilation to birth and takes an average of 20 to 50 minutes, although 2 hours is still considered normal. The third stage extends from birth to expulsion of the placenta and usually takes a few minutes. The fourth stage begins after expulsion of the placenta and lasts until homeostasis is reestablished (about 2 hours). PTS: 1 DIF: Cognitive Level: Comprehension OBJ: Nursing Process: Diagnosis MSC: Client Needs: Health PromotiNonUaRnSdINMGaTiBnt.eCnOaMnce 16. With regard to the turns and other adjustments of the fetus during the birth process, known as the mechanism of labor, nurses should be aware that: a. the seven critical movements must progress in a more or less orderly sequence. b. asynclitism sometimes is achieved by means of the Leopold maneuver. c. the effects of the forces determining descent are modified by the shape of the woman's pelvis and the size of the fetal head. d. at birth the baby is said to achieve “restitution” (i.e., a return to the C-shape of the womb). ANS: C The size of the maternal pelvis and the ability of the fetal head to mold also affect the process. The seven identifiable movements of the mechanism of labor occur in combinations simultaneously, not in precise sequences. Asynclitism is the deflection of the baby's head; the Leopold maneuver is a means of judging descent by palpating the mother's abdomen. Restitution is the rotation of the baby's head after the infant is born. PTS: 1 DIF: Cognitive Level: Comprehension OBJ: Nursing Process: Planning, Implementation MSC: Client Needs: Health Promotion and Maintenance 17. In order to evaluate the condition of the patient accurately during labor, the nurse should be aware that: a. the woman's blood pressure will increase during contractions and fall back to prelabor normal between contractions. b. use of the Valsalva maneuver is encouraged during the second stage of labor to relieve fetal hypoxia. c. having the woman point her toes will reduce leg cramps. d. the endogenous endorphins released during labor will raise the woman's pain threshold and produce sedation. ANS: D The endogenous endorphins released during labor will raise the woman's pain threshold and produce sedation. In addition, physiologic anesthesia of the perineal tissues, caused by the pressure of the presenting part, decreases the mother's perception of pain. Blood pressure increases during contractions but remains somewhat elevated between them. Use of the Valsalva maneuver is discouraged during second-stage labor because of a number of unhealthy outcomes, including fetal hypoxia. Pointing the toes can cause leg cramps, as can the process of labor itself. PTS: 1 DIF: Cognitive Level: Comprehension OBJ: Nursing Process: Planning MSC: Client Needs: Health Promotion and Maintenance 18. The maternity nurse understands that as the uterus contracts during labor, maternal-fetal exchange of oxygen and waste products: a. continues except when placental functions are reduced. b. increases as blood pressure decreases. c. diminishes as the spiral arteries are compressed. d. is not significantly affected. ANS: C Uterine contractions during labor NteUnRdStIoNdGeTcBr.eCaOseMcirculation through the spiral electrodes and subsequent perfusion through the intervillous space. The maternal blood supply to the placenta gradually stops with contractions. The exchange of oxygen and waste products decreases. The exchange of oxygen and waste products is affected by contractions. PTS: 1 DIF: Cognitive Level: Comprehension OBJ: Nursing Process: Assessment MSC: Client Needs: Physiologic Integrity 19. Which statement is the best rationale for assessing maternal vital signs between contractions? a. During a contraction, assessing fetal heart rates is the priority. b. Maternal circulating blood volume increases temporarily during contractions. c. Maternal blood flow to the heart is reduced during contractions. d. Vital signs taken during contractions are not accurate. ANS: B During uterine contractions, blood flow to the placenta temporarily stops, causing a relative increase in the mother's blood volume, which in turn temporarily increases blood pressure and slows pulse. It is important to monitor fetal response to contractions; however, this question is concerned with the maternal vital signs. Maternal blood flow is increased during a contraction. Vital signs are altered by contractions but are considered accurate for that period of time. PTS: 1 DIF: Cognitive Level: Knowledge OBJ: Nursing Process: Implementation MSC: Client Needs: Health Promotion and Maintenance 20. In order to care for obstetric patients adequately, the nurse understands that labor contractions facilitate cervical dilation by: a. contracting the lower uterine segment. b. enlarging the internal size of the uterus. c. promoting blood flow to the cervix. d. pulling the cervix over the fetus and amniotic sac. ANS: D Effective uterine contractions pull the cervix upward at the same time that the fetus and amniotic sac are pushed downward. The contractions are stronger at the fundus. The internal size becomes smaller with the contractions; this helps to push the fetus down. Blood flow decreases to the uterus during a contraction. PTS: 1 DIF: Cognitive Level: Comprehension OBJ: Nursing Process: Assessment MSC: Client Needs: Health Promotion and Maintenance 21. To teach patients about the process of labor adequately, the nurse knows that which event is the best indicator of true labor? a. Bloody show b. Cervical dilation and effacement c. Fetal descent into the pelvic inlet d. Uterine contractions every 7 minutes ANS: B The conclusive distinction between true and false labor is that contractions of true labor cause progressive change in the cervix. Bloody show can occur before true labor. Fetal descent can occur before true labor. False labor may have contractions that occur this frequently; however, this is usually inconsistent. NURSINGTB.COM PTS: 1 DIF: Cognitive Level: Knowledge OBJ: Nursing Process: Assessment MSC: Client Needs: Health Promotion and Maintenance 22. Which occurrence is associated with cervical dilation and effacement? a. Bloody show b. False labor c. Lightening d. Bladder distention ANS: A As the cervix begins to soften, dilate, and efface, expulsion of the mucous plug that sealed the cervix during pregnancy occurs. This causes rupture of small cervical capillaries. Cervical dilation and effacement do not occur with false labor. Lightening is the descent of the fetus toward the pelvic inlet before labor. Bladder distention occurs when the bladder is not emptied frequently. It may slow down the descent of the fetus during labor. PTS: 1 DIF: Cognitive Level: Comprehension OBJ: Nursing Process: Assessment MSC: Client Needs: Health Promotion and Maintenance 23. A primigravida at 39 weeks of gestation is observed for 2 hours in the intrapartum unit. The fetal heart rate has been normal. Contractions are 5 to 9 minutes apart, 20 to 30 seconds in duration, and of mild intensity. Cervical dilation is 1 to 2 cm and uneffaced (unchanged from admission). Membranes are intact. The nurse should expect the woman to be: a. admitted and prepared for a cesarean birth. b. admitted for extended observation. c. discharged home with a sedative. d. discharged home to await the onset of true labor. ANS: D This situation describes a woman with normal assessments who is probably in false labor and will probably not deliver rapidly once true labor begins. These are all indications of false labor without fetal distress. There is no indication that further assessment or cesarean birth is indicated. The patient will likely be discharged; however, there is no indication that a sedative is needed. PTS: 1 DIF: Cognitive Level: Analysis OBJ: Nursing Process: Assessment MSC: Client Needs: Safe and Effective Care Environment 24. Which nursing assessment indicates that a woman who is in second-stage labor is almost ready to give birth? a. The fetal head is felt at 0 station during vaginal examination. b. Bloody mucus discharge increases. c. The vulva bulges and encircles the fetal head. d. The membranes rupture during a contraction. ANS: C During the active pushing (descent) phase, the woman has strong urges to bear down as the presenting part of the fetus descends and presses on the stretch receptors of the pelvic floor. The vulva stretches and begins to bulge encircling the fetal head. Birth of the head occurs when the station is +4. A 0 stationNiUnRdiScIaNtGesTBen.CgOaMgement. Bloody show occurs throughout the labor process and is not an indication of an imminent birth. Rupture of membranes can occur at any time during the labor process and does not indicate an imminent birth. PTS: 1 DIF: Cognitive Level: Analysis OBJ: Nursing Process: Assessment MSC: Client Needs: Health Promotion and Maintenance MULTIPLE RESPONSE 1. Signs that precede labor include: (Select all that apply.) a. lightening. b. exhaustion. c. bloody show. d. rupture of membranes. e. decreased fetal movement. ANS: A, C, D Signs that precede labor may include lightening, urinary frequency, backache, weight loss, surge of energy, bloody show, and rupture of membranes. Many women experience a burst of energy before labor. A decrease in fetal movement is an ominous sign that does not always correlate with labor. PTS: 1 DIF: Cognitive Level: Comprehension OBJ: Nursing Process: Planning, Implementation MSC: Client Needs: Health Promotion and Maintenance 2. Which factors influence cervical dilation? (Select all that apply.) a. Strong uterine contractions. b. The force of the presenting fetal part against the cervix. c. The size of the female pelvis. d. The pressure applied by the amniotic sac. e. Scarring of the cervix. ANS: A, B, D, E Dilation of the cervix occurs by the drawing upward of the musculofibrous components of the cervix, which is caused by strong uterine contractions. Pressure exerted by the amniotic fluid while the membranes are intact or by the force applied by the presenting part also can promote cervical dilation. Scarring of the cervix as a result of a previous infection or surgery may slow cervical dilation. Pelvic size does not affect cervical dilation. PTS: 1 DIF: Cognitive Level: Comprehension OBJ: Nursing Process: Planning MSC: Client Needs: Health Promotion and Maintenance MATCHING For vaginal birth to be successful, the fetus must adapt to the birth canal during the descent. The turns and other adjustments necessary in the human birth process are termed the “mechanism of labor.” Please list the seven cardinal movements in the mechanism of labor in the correct order. a. Flexion b. Internal rotation c. External rotation d. Expulsion e. Engagement f. Descent g. Extension 1. One 2. Two 3. Three 4. Four 5. Five 6. Six 7. Seven NURSINGTB.COM 1. ANS: E PTS: 1 DIF: Cognitive Level: Comprehension OBJ: Nursing Process: Assessment MSC: Client Needs: Physiologic Integrity NOT: In a vertex presentation the cardinal movements, in order, are: engagement, descent, flexion, internal rotation, extension, external rotation (restitution), and finally birth by expulsion. Although these movements are discussed separately, in actuality a combination of movements occurs simultaneously (i.e., engagement involves both descent and flexion). 2. ANS: F PTS: 1 DIF: Cognitive Level: Comprehension OBJ: Nursing Process: Assessment MSC: Client Needs: Physiologic Integrity NOT: In a vertex presentation the cardinal movements, in order, are: engagement, descent, flexion, internal rotation, extension, external rotation (restitution), and finally birth by expulsion. Although these movements are discussed separately, in actuality a combination of movements occurs simultaneously (i.e., engagement involves both descent and flexion). 3. ANS: A PTS: 1 DIF: Cognitive Level: Comprehension OBJ: Nursing Process: Assessment MSC: Client Needs: Physiologic Integrity NOT: In a vertex presentation the cardinal movements, in order, are: engagement, descent, flexion, internal rotation, extension, external rotation (restitution), and finally birth by expulsion. Although these movements are discussed separately, in actuality a combination of movements occurs simultaneously (i.e., engagement involves both descent and flexion). 4. ANS: B PTS: 1 DIF: Cognitive Level: Comprehension OBJ: Nursing Process: Assessment MSC: Client Needs: Physiologic Integrity NOT: In a vertex presentation the cardinal movements, in order, are: engagement, descent, flexion, internal rotation, extension, external rotation (restitution), and finally birth by expulsion. Although these movements are discussed separately, in actuality a combination of movements occurs simultaneously (i.e., engagement involves both descent and flexion). 5. ANS: G PTS: 1 DIF: Cognitive Level: Comprehension OBJ: Nursing Process: Assessment MSC: Client Needs: Physiologic Integrity NOT: In a vertex presentation the cardinal movements, in order, are: engagement, descent, flexion, internal rotation, extension, external rotation (restitution), and finally birth by expulsion. Although these movements are discussed separately, in actuality a combination of movements occurs simultaneously (i.e., engagement involves both descent and flexion). 6. ANS: C PTS: 1 DIF: Cognitive Level: Comprehension OBJ: Nursing Process: Assessment MSC: Client Needs: Physiologic Integrity NOT: In a vertex presentation the cardinal movements, in order, are: engagement, descent, flexion, internal rotation, extension, external rotation (restitution), and finally birth by expulsion. Although these movements are discussed separately, in actuality a combination of movements occurs simultaneously (i.e., engagement involves both descent and flexion). 7. ANS: D PTS: 1 NURSINDGITFB: .CCOoMgnitive Level: Comprehension OBJ: Nursing Process: Assessment MSC: Client Needs: Physiologic Integrity NOT: In a vertex presentation the cardinal movements, in order, are: engagement, descent, flexion, internal rotation, extension, external rotation (restitution), and finally birth by expulsion. Although these movements are discussed separately, in actuality a combination of movements occurs simultaneously (i.e., engagement involves both descent and flexion). Chapter 14: Maximizing Comfort for the Laboring Woman Perry: Maternal Child Nursing Care, 6th Edition MULTIPLE CHOICE 1. An 18-year-old pregnant woman, gravida 1, is admitted to the labor and birth unit with moderate contractions every 5 minutes that last 40 seconds. The woman states, “My contractions are so strong that I don't know what to do with myself.” The nurse should: a. assess for fetal well-being. b. encourage the woman to lie on her side. c. disturb the woman as little as possible. d. recognize that pain is personalized for each individual. ANS: D Each woman's pain during childbirth is unique and is influenced by a variety of physiologic, psychosocial, and environmental factors. A critical issue for the nurse is how support can make a difference in the pain of the woman during labor and birth. Assessing for fetal well-being includes no information that would indicate fetal distress or a logical reason to be overly concerned about the well-being of the fetus. The left lateral position is used to alleviate fetal distress, not maternal stress. The nurse has an obligation to provide physical, emotional, and psychosocial care and support to the laboring woman. This patient clearly needs support. PTS: 1 DIF: Cognitive Level: Application OBJ: Nursing Process: Diagnosis MSC: Client Needs: Physiologic Integrity 2. Nursing care measures are commoNnUlyRSoIfNfeGrTeBd.CtoOwMomen in labor. Which nursing measure reflects application of the gate-control theory? a. Massaging the woman's back b. Changing the woman's position c. Giving the prescribed medication d. Encouraging the woman to rest between contractions ANS: A According to the gate-control theory, pain sensations travel along sensory nerve pathways to the brain, but only a limited number of sensations, or messages, can travel through these nerve pathways at one time. Distraction techniques such as massage or stroking, music, focal points, and imagery reduce or completely block the capacity of nerve pathways to transmit pain. These distractions are thought to work by closing down a hypothetic gate in the spinal cord and thus preventing pain signals from reaching the brain. The perception of pain is thereby diminished. Changing the woman's position, giving prescribed medication, and encouraging rest do not reduce or block the capacity of nerve pathways to transmit pain using the gate-control theory. PTS: 1 DIF: Cognitive Level: Comprehension OBJ: Nursing Process: Implementation MSC: Client Needs: Physiologic Integrity 3. A laboring woman received an opioid agonist (meperidine) intravenously 90 minutes before she gave birth. Which medication should be available to reduce the postnatal effects of Demerol on the neonate? a. Fentanyl (Sublimaze) b. Promethazine (Phenergan) c. Naloxone (Narcan) d. Nalbuphine (Nubain) ANS: C An opioid antagonist can be given to the newborn as one part of the treatment for neonatal narcosis, which is a state of central nervous system (CNS) depression in the newborn produced by an opioid. Opioid antagonists such as naloxone (Narcan) can promptly reverse the CNS depressant effects, especially respiratory depression. Fentanyl, promethazine, and nalbuphine do not act as opioid antagonists to reduce the postnatal effects of Demerol on the neonate. Although meperidine (Demerol) is a low-cost medication and readily available, the use of Demerol in labor has been controversial because of its effects on the neonate. PTS: 1 DIF: Cognitive Level: Knowledge OBJ: Nursing Process: Planning, Implementation MSC: Client Needs: Physiologic Integrity 4. A woman in labor has just received an epidural block. The most important nursing intervention is to: a. limit parenteral fluids. b. monitor the fetus for possible tachycardia. c. monitor the maternal blood pressure for possible hypotension. d. monitor the maternal pulse for possible bradycardia. ANS: C The most important nursing intervention for a woman who has received an epidural block is to monitor the maternal blood pressure frequently for signs of hypotension. Intravenous fluids are increased for a woman receiviNngURanSINepGiTdBur.CalO, Mto prevent hypotension. The nurse observes for signs of fetal bradycardia. The nurse monitors for signs of maternal tachycardia secondary to hypotension. PTS: 1 DIF: Cognitive Level: Application OBJ: Nursing Process: Implementation MSC: Client Needs: Physiologic Integrity 5. The nurse should be aware that an effective plan to achieve adequate pain relief without maternal risk is most effective if: a. the mother gives birth without any analgesic or anesthetic. b. the mother and family's priorities and preferences are incorporated into the plan. c. the primary health care provider decides the best pain relief for the mother and family. d. the nurse informs the family of all alternative methods of pain relief available in the hospital setting. ANS: B The assessment of the woman, her fetus, and her labor is a joint effort of the nurse and the primary health care providers, who consult with the woman about their findings and recommendations. The needs of each woman are different and many factors must be considered before a decision is made whether pharmacologic methods, nonpharmacologic methods, or a combination of the two will be used to manage labor pain. PTS: 1 DIF: Cognitive Level: Comprehension OBJ: Nursing Process: Planning MSC: Client Needs: Psychosocial Integrity 6. A woman in the active phase of the first stage of labor is using a shallow pattern of breathing, which is about twice the normal adult breathing rate. She starts to complain about feeling light-headed and dizzy and states that her fingers are tingling. The nurse should: a. notify the woman's physician. b. tell the woman to slow the pace of her breathing. c. administer oxygen via a mask or nasal cannula. d. help her breathe into a paper bag. ANS: D This woman is experiencing the side effects of hyperventilation, which include the symptoms of lightheadedness, dizziness, tingling of the fingers, or circumoral numbness. Having the woman breathe into a paper bag held tightly around her mouth and nose may eliminate respiratory alkalosis. This enables her to rebreathe carbon dioxide and replace the bicarbonate ion. PTS: 1 DIF: Cognitive Level: Application OBJ: Nursing Process: Implementation MSC: Client Needs: Physiologic Integrity 7. A woman is experiencing back labor and complains of intense pain in her lower back. An effective relief measure would be to use: a. counterpressure against the sacrum. b. pant-blow (breaths and puffs) breathing techniques. c. effleurage. d. conscious relaxation or guided imagery. ANS: A Counterpressure is a steady pressuNrUe RaSpIpNliGeTdBb.CyOaMsupport person to the sacral area with the fist or heel of the hand. This technique helps the woman cope with the sensations of internal pressure and pain in the lower back. The pain-management techniques of pant-blow, effleurage, and conscious relaxation or guided imagery are usually helpful for contractions per the gate-control theory. PTS: 1 DIF: Cognitive Level: Comprehension OBJ: Nursing Process: Implementation MSC: Client Needs: Physiologic Integrity 8. If an opioid antagonist is administered to a laboring woman, she should be told that: a. her pain will decrease. b. her pain will return. c. she will feel less anxious. d. she will no longer feel the urge to push. ANS: B The woman should be told that the pain that was relieved by the opioid analgesic will return with administration of the opioid antagonist. Opioid antagonists, such as Narcan, promptly reverse the central nervous system (CNS) depressant effects of opioids. In addition, the antagonist counters the effect of the stress-induced levels of endorphins. An opioid antagonist is especially valuable if labor is more rapid than expected and birth is anticipated when the opioid is at its peak effect. PTS: 1 DIF: Cognitive Level: Comprehension OBJ: Nursing Process: Planning MSC: Client Needs: Physiologic Integrity 9. The role of the nurse with regard to informed consent is to: a. inform the patient about the procedure and have her sign the consent form. b. act as a patient advocate and help clarify the procedure and the options. c. call the physician to see the patient. d. witness the signing of the consent form. ANS: B Nurses play a part in the informed consent process by clarifying and describing procedures or by acting as the woman's advocate and asking the primary health care provider for further explanations. The physician is responsible for informing the woman of her options, explaining the procedure, and advising the patient about potential risk factors. The physician must be present to explain the procedure to the patient. However, the nurse's responsibilities go further than simply asking the physician to see the patient. The nurse may witness the signing of the consent form. However, depending on the state's guidelines, the woman's husband or another hospital health care employee may sign as witness. PTS: 1 DIF: Cognitive Level: Comprehension OBJ: Nursing Process: Implementation MSC: Client Needs: Safe and Effective Care Environment 10. A first-time mother is concerned about the type of medications she will receive during labor. She is in a fair amount of pain and is nauseous. In addition, she appears to be very anxious. You explain that opioid analgesics are often used with sedatives because: a. “The two together work the best for you and your baby.” b. “Sedatives help the opioid work better, and they also will assist you to relax and relieve your nausea.” c. “They work better together soNyUoRuScIaNnGsTlBe.eCpOuMntil you have the baby.” d. “This is what the doctor has ordered for you.” ANS: B Sedatives can be used to reduce the nausea and vomiting that often accompany opioid use. In addition, some ataractics reduce anxiety and apprehension and potentiate the opioid analgesic affects. A potentiator may cause the two drugs to work together more effectively, but it does not ensure maternal or fetal complications will not occur. Sedation may be a related effect of some ataractics, but it is not the goal. Furthermore, a woman is unlikely to be able to sleep through transitional labor and birth. “This is what the doctor has ordered for you” may be true, but it is not an acceptable comment for the nurse to make. PTS: 1 DIF: Cognitive Level: Application OBJ: Nursing Process: Planning, Implementation MSC: Client Needs: Physiologic Integrity 11. To help patients manage discomfort and pain during labor, nurses should be aware that: a. the predominant pain of the first stage of labor is the visceral pain located in the lower portion of the abdomen. b. referred pain is the extreme discomfort between contractions. c. the somatic pain of the second stage of labor is more generalized and related to fatigue. d. pain during the third stage is a somewhat milder version of the second stage. ANS: A This pain comes from cervical changes, distention of the lower uterine segment, and uterine ischemia. Referred pain occurs when the pain that originates in the uterus radiates to the abdominal wall, lumbosacral area of the back, iliac crests, and gluteal area. Second-stage labor pain is intense, sharp, burning, and localized. Third-stage labor pain is similar to that of the first stage. PTS: 1 DIF: Cognitive Level: Knowledge OBJ: Nursing Process: Assessment MSC: Client Needs: Health Promotion and Maintenance 12. Which statement correctly describes the effects of various pain factors? a. Higher prostaglandin levels arising from dysmenorrhea can blunt the pain of childbirth. b. Upright positions in labor increase the pain factor because they cause greater fatigue. c. Women who move around trying different positions are experiencing more pain. d. Levels of pain-mitigating -endorphins are higher during a spontaneous, natural childbirth. ANS: D Higher endorphin levels help women tolerate pain and reduce anxiety and irritability. Higher prostaglandin levels correspond to more severe labor pains. Upright positions in labor usually result in improved comfort and less pain. Moving freely to find more comfortable positions is important for reducing pain and muscle tension. PTS: 1 DIF: Cognitive Level: Comprehension OBJ: Nursing Process: Diagnosis MSC: Client Needs: Physiologic Integrity NURSINGTB.COM 13. Nurses with an understanding of cultural differences regarding likely reactions to pain may be better able to help patients. Nurses should know that women may be stoic until late in labor, when they may become vocal and request pain relief. a. Chinese b. Arab or Middle Eastern c. Hispanic d. African-American ANS: C Hispanic women may be stoic early and more vocal and ready for medications later. Chinese women may not show reactions to pain. Medical interventions must be offered more than once. Arab or Middle Eastern women may be vocal in response to labor pain from the start. They may prefer pain medications. African-American women may express pain openly; use of medications for pain is more likely to vary with the individual. PTS: 1 DIF: Cognitive Level: Knowledge OBJ: Nursing Process: Assessment MSC: Client Needs: Psychosocial Integrity 14. With regard to a pregnant woman's anxiety and pain experience, nurses should be aware that: a. even mild anxiety must be treated. b. severe anxiety increases tension, which increases pain, which in turn increases fear and anxiety, and so on. c. anxiety may increase the perception of pain, but it does not affect the mechanism of labor. d. women who have had a painful labor will have learned from the experience and have less anxiety the second time because of increased familiarity. ANS: B Anxiety and pain reinforce each other in a negative cycle. Mild anxiety is normal for a woman in labor and likely needs no special treatment other than the standard reassurances. Anxiety increases muscle tension and ultimately can build sufficiently to slow the progress of labor. Unfortunately, an anxious, painful first labor is likely to carry over, through expectations and memories, into an anxious and painful experience in the second pregnancy. PTS: 1 DIF: Cognitive Level: Comprehension OBJ: Nursing Process: Assessment MSC: Client Needs: Psychosocial Integrity 15. Nurses should be aware of the differences experience can make in labor pain such as: a. sensory pain for nulliparous women often is greater than for multiparous women during early labor. b. affective pain for nulliparous women usually is less than for multiparous women throughout the first stage of labor. c. women with a history of substance abuse experience more pain during labor. d. multiparous women have more fatigue from labor and therefore experience more pain. ANS: A Sensory pain is greater for nulliparous women because their reproductive tract structures are less supple. Affective pain is higher for nulliparous women during the first stage but decreases for both nulliparous and multiparous during the second stage. Women with a history of substance abuse experience the saNmUeRaSmINoGuTnBt .oCfOpMain as those without such a history. Nulliparous women have longer labors and therefore experience more fatigue. PTS: 1 DIF: Cognitive Level: Knowledge OBJ: Nursing Process: Diagnosis MSC: Client Needs: Physiologic Integrity 16. In the current practice of childbirth preparation, emphasis is placed on: a. the Dick-Read (natural) childbirth method. b. the Lamaze (psychoprophylactic) method. c. the Bradley (husband-coached) method. d. having expectant parents attend childbirth preparation in any or no specific method. ANS: D Encouraging expectant parents to attend childbirth preparation class is most important because preparation increases a woman's confidence and thus her ability to cope with labor and birth. Although still popular, the “method” format of classes is being replaced with other offerings such as Hypnobirthing and Birthing from Within. PTS: 1 DIF: Cognitive Level: Comprehension OBJ: Nursing Process: Planning MSC: Client Needs: Health Promotion and Maintenance 17. With regard to breathing techniques during labor, maternity nurses should understand that: a. breathing techniques in the first stage of labor are designed to increase the size of the abdominal cavity to reduce friction. b. by the time labor has begun, it is too late for instruction in breathing and relaxation. c. controlled breathing techniques are most difficult near the end of the second stage of labor. d. the patterned-paced breathing technique can help prevent hyperventilation. ANS: A First-stage techniques promote relaxation of abdominal muscles, thereby increasing the size of the abdominal cavity. Instruction in simple breathing and relaxation techniques early in labor is possible and effective. Controlled breathing techniques are most difficult in the transition phase at the end of the first stage of labor when the cervix is dilated 8 to 10 cm. Patterned-paced breathing sometimes can lead to hyperventilation. PTS: 1 DIF: Cognitive Level: Comprehension OBJ: Nursing Process: Implementation MSC: Client Needs: Physiologic Integrity 18. Maternity nurses often have to answer questions about the many, sometimes unusual ways people have tried to make the birthing experience more comfortable. For instance, nurses should be aware that: a. music supplied by the support person has to be discouraged because it could disturb others or upset the hospital routine. b. women in labor can benefit from sitting in a bathtub, but they must limit immersion to no longer than 15 minutes at a time. c. effleurage is permissible, but counterpressure is almost always counterproductive. d. electrodes attached to either side of the spine to provide high-intensity electrical impulses facilitate the release of endorphins. ANS: D NURSINGTB.COM Transcutaneous electrical nerve stimulation does help. Music may be very helpful for reducing tension and certainly can be accommodated by the hospital. Women can stay in a bath as long as they want, although repeated baths with breaks may be more effective than a long soak. Counterpressure can help the woman cope with lower back pain. PTS: 1 DIF: Cognitive Level: Application OBJ: Nursing Process: Planning MSC: Client Needs: Physiologic Integrity 19. With regard to systemic analgesics administered during labor, nurses should be aware that: a. systemic analgesics cross the maternal blood-brain barrier as easily as they do the fetal blood-brain barrier. b. effects on the fetus and newborn can include decreased alertness and delayed sucking. c. intramuscular (IM) administration is preferred over intravenous (IV) administration. d. IV patient-controlled analgesia (PCA) results in increased use of an analgesic. ANS: B Effects depend on the specific drug given, the dosage, and the timing. Systemic analgesics cross the fetal blood-brain barrier more readily than the maternal blood-brain barrier. IV administration is preferred over IM administration because the drug acts faster and more predictably. PCA results in decreased use of an analgesic. PTS: 1 DIF: Cognitive Level: Knowledge OBJ: Nursing Process: Planning MSC: Client Needs: Health Promotion and Maintenance 20. With regard to nerve block analgesia and anesthesia, nurses should be aware that: a. most local agents are related chemically to cocaine and end in the suffix -caine. b. local perineal infiltration anesthesia is effective when epinephrine is added, but it can be injected only once. c. a pudendal nerve block is designed to relieve the pain from uterine contractions. d. a pudendal nerve block, if done correctly, does not significantly lessen the bearing-down reflex. ANS: A Common agents include lidocaine and chloroprocaine. Injections can be repeated to prolong the anesthesia. A pudendal nerve block relieves pain in the vagina, vulva, and perineum but not the pain from uterine contractions, and it lessens or shuts down the bearing-down reflex. PTS: 1 DIF: Cognitive Level: Comprehension OBJ: Nursing Process: Planning MSC: Client Needs: Physiologic Integrity 21. With regard to spinal and epidural (block) anesthesia, nurses should know that: a. this type of anesthesia is commonly used for cesarean births but is not suitable for vaginal births. b. a high incidence of after-birth headache is seen with spinal blocks. c. epidural blocks allow the woman to move freely. d. spinal and epidural blocks are never used together. ANS: B Headaches may be prevented or mNiUtigRaStIeNdGtToBs.ComOMe degree by a number of methods. Spinal blocks may be used for vaginal births, but the woman must be assisted through labor. Epidural blocks limit the woman's ability to move freely. Combined use of spinal and epidural blocks is becoming increasingly popular. PTS: 1 DIF: Cognitive Level: Comprehension OBJ: Nursing Process: Evaluation MSC: Client Needs: Physiologic Integrity 22. A woman in labor is breathing into a mouthpiece just before the start of her regular contractions. As she inhales, a valve opens, and gas is released. She continues to inhale the gas slowly and deeply until the contraction starts to subside. When the inhalation stops, the valve closes. This procedure is: a. not used much anymore. b. likely to be used in the second stage of labor but not in the first stage. c. an application of nitrous oxide. d. a prelude to cesarean birth. ANS: C This is an application of nitrous oxide, which could be used in either the first or second stage of labor (or both) as part of the preparation for a vaginal birth. Nitrous oxide is self-administered and found to be very helpful. PTS: 1 DIF: Cognitive Level: Comprehension OBJ: Nursing Process: Implementation MSC: Client Needs: Physiologic Integrity 23. After change-of-shift report the nurse assumes care of a multiparous patient in labor. The woman is complaining of pain that radiates to her abdominal wall, lower back, and buttocks and down her thighs. Before implementing a plan of care, the nurse should understand that this type of pain is: a. visceral. b. referred. c. somatic. d. afterpain. ANS: B As labor progresses the woman often experiences referred pain. This occurs when pain that originates in the uterus radiates to the abdominal wall, the lumbosacral area of the back, the gluteal area, and thighs. The woman usually has pain only during a contraction and is free from pain between contractions. Visceral pain is that which predominates in the first stage of labor. This pain originates from cervical changes, distention of the lower uterine segment, and uterine ischemia. Visceral pain is located over the lower portion of the abdomen. Somatic pain is described as intense, sharp, burning, and well localized. This results from stretching of the perineal tissues and the pelvic floor. This occurs during the second stage of labor. Pain experienced during the third stage of labor or afterward during the early after birth period is uterine. This pain is very similar to that experienced in the first stage of labor. PTS: 1 DIF: Cognitive Level: Comprehension OBJ: Nursing Process: Planning MSC: Client Needs: Health Promotion and Maintenance 24. It is important for the nurse to develop a realistic birth plan with the pregnant woman in her care. The nurse can explain that a major advantage of nonpharmacologic pain management is: a. greater and more complete paiNnUrRelSiIeNfGisTBpo.CsOsiMble. b. no side effects or risks to the fetus are involved. c. the woman remains fully alert at all times. d. a more rapid labor is likely. ANS: B Because nonpharmacologic pain management does not include analgesics, adjunct drugs, or anesthesia, it is harmless to the mother and the fetus. There is less pain relief with nonpharmacologic pain management during childbirth. The woman's alertness is not altered by medication; however, the increase in pain will decrease alertness. Pain management may or may not alter the length of labor. At times when pain is decreased, the mother relaxes and labor progresses at a quicker pace. PTS: 1 DIF: Cognitive Level: Knowledge OBJ: Nursing Process: Assessment MSC: Client Needs: Physiologic Integrity 25. The nurse providing newborn stabilization must be aware that the primary side effect of maternal narcotic analgesia in the newborn is: a. respiratory depression. b. bradycardia. c. acrocyanosis. d. tachypnea. ANS: A An infant delivered within 1 to 4 hours of maternal analgesic administration is at risk for respiratory depression from the sedative effects of the narcotic. Bradycardia is not the anticipated side effect of maternal analgesics. Acrocyanosis is an expected finding in a newborn and is not related to maternal analgesics. The infant who is having a side effect to maternal analgesics normally would have a decrease in respirations, not an increase. PTS: 1 DIF: Cognitive Level: Knowledge OBJ: Nursing Process: Assessment MSC: Client Needs: Physiologic Integrity 26. The nerve block used in labor that provides anesthesia to the lower vagina and perineum is called: a. an epidural. b. a pudendal. c. a local. d. a spinal block. ANS: B A pudendal block anesthetizes the lower vagina and perineum to provide anesthesia for an episiotomy and use of low forceps if needed. An epidural provides anesthesia for the uterus, perineum, and legs. A local provides anesthesia for the perineum at the site of the episiotomy. A spinal block provides anesthesia for the uterus, perineum, and down the legs. PTS: 1 DIF: Cognitive Level: Knowledge OBJ: Nursing Process: Assessment MSC: Client Needs: Physiologic Integrity 27. Which method of pain management is safest for a gravida 3 para 2 admitted at 8 cm cervical dilation? a. Epidural anesthesia b. Narcotics c. Spinal block NURSINGTB.COM d. Breathing and relaxation techniques ANS: D Nonpharmacologic methods of pain management may be the best option for a woman in advanced labor. It is unlikely that enough time remains to administer epidural or spinal anesthesia. A narcotic given at this time may reach its peak about the time of birth and result in respiratory depression in the newborn. PTS: 1 DIF: Cognitive Level: Application OBJ: Nursing Process: Assessment MSC: Client Needs: Physiologic Integrity 28. The laboring woman who imagines her body opening to let the baby out is using a mental technique called: a. dissociation. b. effleurage. c. imagery. d. distraction. ANS: C Imagery is a technique of visualizing images that will assist the woman in coping with labor. Dissociation helps the woman learn to relax all muscles except those that are working. Effleurage is self-massage. Distraction can be used in the early latent phase by having the woman engage in another activity. PTS: 1 DIF: Cognitive Level: Comprehension OBJ: Nursing Process: Assessment MSC: Client Needs: Psychosocial Integrity 29. The obstetric nurse is preparing the patient for an emergency cesarean birth, with no time to administer spinal anesthesia. The nurse is aware and prepared for the greatest risk of administering general anesthesia to the patient. This risk is: a. respiratory depression. b. uterine relaxation. c. inadequate muscle relaxation. d. aspiration of stomach contents. ANS: D Aspiration of acidic gastric contents with possible airway obstruction is a potentially fatal complication of general anesthesia. Respirations can be altered during general anesthesia, and the anesthesiologist will take precautions to maintain proper oxygenation. Uterine relaxation can occur with some anesthesia; however, this can be monitored and prevented. Inadequate muscle relaxation can be improved with medication. PTS: 1 DIF: Cognitive Level: Comprehension OBJ: Nursing Process: Assessment MSC: Client Needs: Physiologic Integrity 30. To assist the woman after delivery of the infant, the nurse knows that the blood patch is used after spinal anesthesia to relieve: NURSINGTB.COM a. hypotension. b. headache. c. neonatal respiratory depression. d. loss of movement. ANS: B The subarachnoid block may cause a postspinal headache resulting from loss of cerebrospinal fluid from the puncture in the dura. When blood is injected into the epidural space in the area of the dural puncture, it forms a seal over the hole to stop leaking of cerebrospinal fluid. Hypotension is prevented by increasing fluid volume before the procedure. Neonatal respiratory depression is not an expected outcome with spinal anesthesia. Loss of movement is an expected outcome of spinal anesthesia. PTS: 1 DIF: Cognitive Level: Knowledge OBJ: Nursing Process: Assessment MSC: Client Needs: Physiologic Integrity MULTIPLE RESPONSE 1. Maternal hypotension is a potential side effect of regional anesthesia and analgesia. What nursing interventions could you use to raise the patient's blood pressure? (Select all that apply.) a. Place the woman in a supine position. b. Place the woman in a lateral position. c. Increase intravenous (IV) fluids. d. Administer oxygen. e. Perform a vaginal examination. ANS: B, C, D Nursing interventions for maternal hypotension arising from analgesia or anesthesia include turning the woman to a lateral position, increasing IV fluids, administering oxygen via face mask, elevating the woman's legs, notifying the physician, administering an IV vasopressor, and monitoring the maternal and fetal status at least every 5 minutes until these are stable. Placing the patient in a supine position would cause venous compression, thereby limiting blood flow to and oxygenation of the placenta and fetus. A sterile vaginal examination has no bearing on maternal blood pressure. PTS: 1 DIF: Cognitive Level: Application OBJ: Nursing Process: Implementation MSC: Client Needs: Physiologic Integrity 2. The class of drugs known as opioid analgesics (butorphanol, nalbuphine) is not suitable for administration to women with known opioid dependence. The antagonistic activity could precipitate withdrawal symptoms (abstinence syndrome) in both mothers and newborns. Signs of opioid/narcotic withdrawal in the mother would include: (Select all that apply.) a. yawning, runny nose. b. increase in appetite. c. chills and hot flashes. d. constipation. e. irritability, restlessness. ANS: A, C, E The woman experiencing maternaNl UopRiSoIiNdGwTBit.hCdOraMwal syndrome will exhibit yawning, runny nose, sneezing, anorexia, chills or hot flashes, vomiting, diarrhea, abdominal pain, irritability, restlessness, muscle spasms, weakness, and drowsiness. It is important for the nurse to assess both mother and baby and to plan care accordingly. PTS: 1 DIF: Cognitive Level: Application OBJ: Nursing Process: Assessment MSC: Client Needs: Physiologic Integrity 3. While developing an intrapartum care plan for the patient in early labor, it is important that the nurse recognize that psychosocial factors may influence a woman's experience of pain. These include: (Select all that apply.) a. culture. b. anxiety and fear. c. previous experiences with pain. d. intervention of caregivers. e. support systems. ANS: A, B, C, E Culture: A woman's sociocultural roots influence how she perceives, interprets, and responds to pain during childbirth. Some cultures encourage loud and vigorous expressions of pain, whereas others value self-control. The nurse should avoid praising some behaviors (stoicism) while belittling others (noisy expression). Anxiety and fear: Extreme anxiety and fear magnify sensitivity to pain and impair a woman's ability to tolerate it. Anxiety and fear increase muscle tension in the pelvic area, which counters the expulsive forces of uterine contractions and pushing efforts. Previous experiences with pain: Fear and withdrawal are a natural response to pain during labor. Learning about these normal sensations ahead of time helps a woman suppress her natural reactions of fear regarding the impending birth. If a woman previously had a long and difficult labor, she is likely to be anxious. She may also have learned ways to cope and may use these skills to adapt to the present labor experience. Support systems: An anxious partner is less able to provide help and support to a woman during labor. A woman's family and friends can be an important source of support if they convey realistic and positive information about labor and delivery. Although the intervention of caregivers may be necessary for the well-being of the woman and her fetus, some interventions add discomfort to the natural pain of labor (i.e., fetal monitor straps, intravenous lines). PTS: 1 DIF: Cognitive Level: Application OBJ: Nursing Process: Planning MSC: Client Needs: Psychosocial Integrity MATCHING Many women seek alternative or complementary pain relief during labor to delay or avoid pharmacologic or invasive therapies. Evidence regarding the use of these modalities has been sparse, and so they remain underuNtUilRizSeIdN.GPTuBb.lCisOhMed reviews of the best-known therapies identified the benefits of each modality for the woman in labor. Please match the alternative modality with the correct research finding. a. Walking b. Massage c. Acupuncture d. Water immersion e. Aromatherapy 1. Less pain intensity, decreased use of analgesia, fewer instrumental births. 2. Significantly decreased use of analgesia, shorter labor. 3. No difference when compared with placebo. 4. Less pain and anxiety during the first stage of labor. 5. Reduced length of labor, increased satisfaction of pain relief. 1. ANS: C PTS: 1 DIF: Cognitive Level: Application OBJ: Nursing Process: Assessment MSC: Client Needs: Health Promotion and Maintenance NOT: Evidence remains insufficient regarding the use of various alternative or complimentary therapies; however, it suggests that they do not cause harm. Nonpharmacologic methods of pain relief may be most effective in latent or early active labor. Women's needs must be reevaluated frequently, and women should be offered various pain relief modalities throughout labor. 2. ANS: D PTS: 1 DIF: Cognitive Level: Application OBJ: Nursing Process: Assessment MSC: Client Needs: Health Promotion and Maintenance NOT: Evidence remains insufficient regarding the use of various alternative or complimentary therapies; however, it suggests that they do not cause harm. Nonpharmacologic methods of pain relief may be most effective in latent or early active labor. Women's needs must be reevaluated frequently, and women should be offered various pain relief modalities throughout labor. 3. ANS: E PTS: 1 DIF: Cognitive Level: Application OBJ: Nursing Process: Assessment MSC: Client Needs: Health Promotion and Maintenance NOT: Evidence remains insufficient regarding the use of various alternative or complimentary therapies; however, it suggests that they do not cause harm. Nonpharmacologic methods of pain relief may be most effective in latent or early active labor. Women's needs must be reevaluated frequently, and women should be offered various pain relief modalities throughout labor. 4. ANS: B PTS: 1 DIF: Cognitive Level: Application OBJ: Nursing Process: Assessment MSC: Client Needs: Health Promotion and Maintenance NOT: Evidence remains insufficient regarding the use of various alternative or complimentary therapies; however, it suggests that they do not cause harm. Nonpharmacologic methods of pain relief may be most effective in latent or early active labor. Women's needs must be reevaluated frequently, and women should be offered various pain relief modalities throughout labor. 5. ANS: A PTS: 1 DIF: Cognitive Level: Application OBJ: Nursing Process: Assessment MSC: Client Needs: Health Promotion and Maintenance NOT: Evidence remains insufficient regarding the use of various alternative or complimentary therapies; however, it suggests that they do not cause harm. Nonpharmacologic methods of pain relief may be most effective in latent or early active labor. Women's needs must be reevaluated frequently, and women should be offered various pain relief modalities throughout labor. NURSINGTB.COM Chapter 15: Fetal Assessment During Labor Perry: Maternal Child Nursing Care, 6th Edition MULTIPLE CHOICE 1. While evaluating an external monitor tracing of a woman in active labor, the nurse notes that the fetal heart rate (FHR) for five sequential contractions begins to decelerate late in the contraction, with the nadir of the decelerations occurring after the peak of the contraction. The nurse's first priority is to: a. change the woman's position. b. notify the care provider. c. assist with amnioinfusion. d. insert a scalp electrode. ANS: A Late decelerations may be caused by maternal supine hypotension syndrome. They usually are corrected when the woman turns on her side to displace the weight of the gravid uterus from the vena cava. If the fetus does not respond to primary nursing interventions for late decelerations, the nurse would continue with subsequent intrauterine resuscitation measures, including notifying the care provider. An amnioinfusion may be used to relieve pressure on an umbilical cord that has not prolapsed. The FHR pattern associated with this situation most likely reveals variable deceleration. A fetal scalp electrode would provide accurate data for evaluating the well-being of the fetus; however, this is not a nursing intervention that would alleviate late decelerations, nor is it the nurse's first priority. PTS: 1 DIF: CognitiNveURLSevINelG: TABp.pClOicMation OBJ: Nursing Process: Implementation MSC: Client Needs: Physiologic Integrity 2. The nurse caring for the laboring woman should understand that early decelerations are caused by: a. altered fetal cerebral blood flow. b. umbilical cord compression. c. uteroplacental insufficiency. d. spontaneous rupture of membranes. ANS: A Early decelerations are the fetus's response to fetal head compression. Variable decelerations are associated with umbilical cord compression. Late decelerations are associated with uteroplacental insufficiency. Spontaneous rupture of membranes has no bearing on the fetal heart rate unless the umbilical cord prolapses, which would result in variable or prolonged bradycardia. PTS: 1 DIF: Cognitive Level: Comprehension OBJ: Nursing Process: Assessment MSC: Client Needs: Physiologic Integrity 3. The nurse providing care for the laboring woman comprehends that accelerations with fetal movement: a. are reassuring. b. are caused by umbilical cord compression. c. warrant close observation. d. are caused by uteroplacental insufficiency. ANS: A Episodic accelerations in the fetal heart rate (FHR) occur during fetal movement and are indications of fetal well-being. Umbilical cord compression results in variable decelerations in the FHR. Accelerations in the FHR are an indication of fetal well-being and do not warrant close observation. Uteroplacental insufficiency would result in late decelerations in the FHR. PTS: 1 DIF: Cognitive Level: Knowledge OBJ: Nursing Process: Assessment MSC: Client Needs: Physiologic Integrity 4. The nurse providing care for the laboring woman realizes that variable fetal heart rate (FHR) decelerations are caused by: a. altered fetal cerebral blood flow. b. umbilical cord compression. c. uteroplacental insufficiency. d. fetal hypoxemia. ANS: B Variable decelerations can occur any time during the uterine contracting phase and are caused by compression of the umbilical cord. Altered fetal cerebral blood flow would result in early decelerations in the FHR. Uteroplacental insufficiency would result in late decelerations in the FHR. Fetal hypoxemia would result in tachycardia initially and then bradycardia if hypoxia continues. PTS: 1 DIF: Cognitive Level: Knowledge OBJ: Nursing Process: Assessment MSC: Client Needs: Physiologic Integrity NURSINGTB.COM 5. The nurse providing care for the laboring woman should understand that late fetal heart rate (FHR) decelerations are the result of: a. altered cerebral blood flow. b. umbilical cord compression. c. uteroplacental insufficiency. d. meconium fluid. ANS: C Uteroplacental insufficiency would result in late decelerations in the FHR. Altered fetal cerebral blood flow would result in early decelerations in the FHR. Umbilical cord compression would result in variable decelerations in the FHR. Meconium-stained fluid may or may not produce changes in the fetal heart rate, depending on the gestational age of the fetus and whether other causative factors associated with fetal distress are present. PTS: 1 DIF: Cognitive Level: Knowledge OBJ: Nursing Process: Assessment MSC: Client Needs: Physiologic Integrity 6. The nurse providing care for the laboring woman should understand that amnioinfusion is used to treat: a. variable decelerations. b. late decelerations. c. fetal bradycardia. d. fetal tachycardia. ANS: A Amnioinfusion is used during labor either to dilute meconium-stained amniotic fluid or to supplement the amount of amniotic fluid to reduce the severity of variable decelerations caused by cord compression. Amnioinfusion has no bearing on late decelerations, fetal bradycardia, or fetal tachycardia alterations in fetal heart rate (FHR) tracings. PTS: 1 DIF: Cognitive Level: Knowledge OBJ: Nursing Process: Implementation MSC: Client Needs: Health Promotion and Maintenance 7. The nurse caring for the woman in labor should understand that maternal hypotension can result in: a. early decelerations. b. fetal dysrhythmias. c. uteroplacental insufficiency. d. spontaneous rupture of membranes. ANS: C Low maternal blood pressure reduces placental blood flow during uterine contractions and results in fetal hypoxemia. Maternal hypotension is not associated with early decelerations, fetal dysrhythmias, or spontaneous rupture of membranes. PTS: 1 DIF: Cognitive Level: Comprehension OBJ: Nursing Process: Assessment MSC: Client Needs: Health Promotion and Maintenance 8. The nurse caring for a laboring woman is aware that maternal cardiac output can be increased by: a. change in position. b. oxytocin administration. c. regional anesthesia. d. intravenous analgesic. ANS: A NURSINGTB.COM Maternal supine hypotension syndrome is caused by the weight and pressure of the gravid uterus on the ascending vena cava when the woman is in a supine position. This reduces venous return to the woman's heart, as well as cardiac output, and subsequently reduces her blood pressure. The nurse can encourage the woman to change positions and avoid the supine position. Oxytocin administration, regional anesthesia, and intravenous analgesic may reduce maternal cardiac output. PTS: 1 DIF: Cognitive Level: Comprehension OBJ: Nursing Process: Implementation MSC: Client Needs: Physiologic Integrity 9. While evaluating an external monitor tracing of a woman in active labor whose labor is being induced, the nurse notes that the fetal heart rate (FHR) begins to decelerate at the onset of several contractions and returns to baseline before each contraction ends. The nurse should: a. change the woman's position. b. discontinue the oxytocin infusion. c. insert an internal monitor. d. document the finding in the patient's record. ANS: D The FHR indicates early decelerations, which are not an ominous sign and do not require any intervention. The nurse should simply document these findings. PTS: 1 DIF: Cognitive Level: Application OBJ: Nursing Process: Implementation MSC: Client Needs: Health Promotion and Maintenance 10. Which fetal heart rate (FHR) finding would concern the nurse during labor? a. Accelerations with fetal movement b. Early decelerations c. An average FHR of 126 beats/min d. Late decelerations ANS: D Late decelerations are caused by uteroplacental insufficiency and are associated with fetal hypoxemia. They are considered ominous if persistent and uncorrected. Accelerations in the FHR are an indication of fetal well-being. Early decelerations in the FHR are associated with head compression as the fetus descends into the maternal pelvic outlet; they generally are not a concern during normal labor. PTS: 1 DIF: Cognitive Level: Analysis OBJ: Nursing Process: Assessment MSC: Client Needs: Health Promotion and Maintenance 11. The most common cause of decreased variability in the fetal heart rate (FHR) that lasts 30 minutes or less is: a. altered cerebral blood flow. b. fetal hypoxemia. c. umbilical cord compression. d. fetal sleep cycles. ANS: D NURSINGTB.COM A temporary decrease in variability can occur when the fetus is in a sleep state. These sleep states do not usually last longer than 30 minutes. Altered fetal cerebral blood flow would result in early decelerations in the FHR. Fetal hypoxemia would be evidenced by tachycardia initially and then bradycardia. A persistent decrease or loss of FHR variability may be seen. Umbilical cord compression would result in variable decelerations in the FHR. PTS: 1 DIF: Cognitive Level: Comprehension OBJ: Nursing Process: Assessment MSC: Client Needs: Health Promotion and Maintenance 12. Fetal well-being during labor is assessed by: a. the response of the fetal heart rate (FHR) to uterine contractions (UCs). b. maternal pain control. c. accelerations in the FHR. d. an FHR above 110 beats/min. ANS: A Fetal well-being during labor can be measured by the response of the FHR to UCs. In general, reassuring FHR patterns are characterized by an FHR baseline in the range of 110 to 160 beats/min with no periodic changes, a moderate baseline variability, and accelerations with fetal movement. Maternal pain control is not the measure used to determine fetal well-being in labor. Although FHR accelerations are a reassuring pattern, they are only one component of the criteria by which fetal well-being is assessed. Although an FHR above 110 beats/min may be reassuring, it is only one component of the criteria by which fetal well-being is assessed. More information would be needed to determine fetal well-being. PTS: 1 DIF: Cognitive Level: Comprehension OBJ: Nursing Process: Assessment MSC: Client Needs: Physiologic Integrity 13. You are evaluating the fetal monitor tracing of your patient, who is in active labor. Suddenly you see the fetal heart rate (FHR) drop from its baseline of 125 beats/min down to 80 beats/min. You reposition the mother, provide oxygen, increase intravenous (IV) fluid, and perform a vaginal examination. The cervix has not changed. Five minutes have passed, and the fetal heart rate remains in the 80s. What additional nursing measures should you take? a. Call for staff assistance. b. Insert a Foley catheter. c. Start Pitocin. d. Notify the care provider immediately. ANS: D To relieve an FHR deceleration, the nurse can reposition the mother, increase IV fluid, and provide oxygen. If oxytocin is infusing, it should be discontinued. If the FHR does not resolve, the primary care provider should be notified immediately. Inserting a Foley catheter is an inappropriate nursing action.NIUf RthSeINFGHTRB.CwOeMre to continue in a nonreassuring pattern, a cesarean section could be warranted, which would require a Foley catheter. However, the physician must make that determination. Pitocin may place additional stress on the fetus. PTS: 1 DIF: Cognitive Level: Evaluation OBJ: Nursing Process: Implementation MSC: Client Needs: Health Promotion and Maintenance 14. What three measures should the nurse implement to provide intrauterine resuscitation? Select the response that best indicates the priority of actions that should be taken. a. Call the provider, reposition the mother, and perform a vaginal examination. b. Reposition the mother, increase intravenous (IV) fluid, and provide oxygen via face mask. c. Administer oxygen to the mother, increase IV fluid, and notify the care provider. d. Perform a vaginal examination, reposition the mother, and provide oxygen via face mask. ANS: B Repositioning the mother, increasing intravenous (IV) fluid, and providing oxygen via face mask are correct nursing actions for intrauterine resuscitation. The nurse should initiate intrauterine resuscitation in an ABC manner, similar to basic life support. The first priority is to open the maternal and fetal vascular systems by repositioning the mother for improved perfusion. The second priority is to increase blood volume by increasing the IV fluid. The third priority is to optimize oxygenation of the circulatory volume by providing oxygen via face mask. If these interventions do not resolve the fetal heart rate issue quickly, the primary provider should be notified immediately. PTS: 1 DIF: Cognitive Level: Evaluation OBJ: Nursing Process: Implementation MSC: Client Needs: Health Promotion and Maintenance 15. Perinatal nurses are legally responsible for: a. correctly interpreting fetal heart rate (FHR) patterns, initiating appropriate nursing interventions, and documenting the outcomes. b. greeting the patient on arrival, assessing her, and starting an intravenous line. c. applying the external fetal monitor and notifying the care provider. d. making sure that the woman is comfortable. ANS: A Nurses who care for women during childbirth are legally responsible for correctly interpreting FHR patterns, initiating appropriate nursing interventions based on those patterns, and documenting the outcomes of those interventions. Greeting the patient, assessing her, and starting an IV; applying the external fetal monitor and notifying the care provider; and making sure the woman is comfortable may be activities that a nurse performs, but they are not activities for which the nurse is legally responsible. PTS: 1 DIF: Cognitive Level: Comprehension OBJ: Nursing Process: Assessment, Planning, Implementation MSC: Client Needs: Safe and Effective Care Environment 16. As a perinatal nurse you realize that a fetal heart rate that is tachycardic, is bradycardic, or has late decelerations or loss of variability is nonreassuring and is associated with: a. hypotension. b. cord compression. c. maternal drug use. d. hypoxemia. ANS: D NURSINGTB.COM Nonreassuring heart rate patterns are associated with fetal hypoxemia. Fetal bradycardia may be associated with maternal hypotension. Fetal variable decelerations are associated with cord compression. Maternal drug use is associated with fetal tachycardia. PTS: 1 DIF: Cognitive Level: Analysis OBJ: Nursing Process: Assessment MSC: Client Needs: Health Promotion and Maintenance 17. A new patient and her partner arrive in the labor, delivery, recovery, and after birth unit for the birth of their first child. You apply the electronic fetal monitor (EFM) to the woman. Her partner asks you to explain what is printing on the graph, referring to the EFM strip. He wants to know what the baby's heart rate should be. Your best response is: a. “Don't worry about that machine; that's my job.” b. “The top line graphs the baby's heart rate. Generally the heart rate is between 110 and 160. The heart rate will fluctuate in response to what is happening during labor.” c. “The top line graphs the baby's heart rate, and the bottom line lets me know how strong the contractions are.” d. “Your doctor will explain all of that later.” ANS: B “The top line graphs the baby's heart rate. Generally the heart rate is between 110 and 160. The heart rate will fluctuate in response to what is happening during labor” educates the partner about fetal monitoring and provides support and information to alleviate his fears. “Don't worry about that machine; that's my job” discredits the partner's feelings and does not provide the teaching he is requesting. “The top line graphs the baby's heart rate, and the bottom line lets me know how strong the contractions are” provides inaccurate information and does not address the partner's concerns about the fetal heart rate. The EFM graphs the frequency and duration of the contractions, not the intensity. Nurses should take every opportunity to provide patient and family teaching, especially when information is requested. PTS: 1 DIF: Cognitive Level: Application OBJ: Nursing Process: Planning MSC: Client Needs: Psychosocial Integrity 18. A normal uterine activity pattern in labor is characterized by: a. contractions every 2 to 5 minutes. b. contractions lasting about 2 minutes. c. contractions about 1 minute apart. d. a contraction intensity of about 1000 mm Hg with relaxation at 50 mm Hg. ANS: A Contractions normally occur every 2 to 5 minutes and last less than 90 seconds (intensity 800 mm Hg) with about 30 seconds in between (20 mm Hg or less). PTS: 1 DIF: Cognitive Level: Knowledge OBJ: Nursing Process: Assessment MSC: Client Needs: Health Promotion and Maintenance 19. According to standard professional thinking, nurses should auscultate the fetal heart rate (FHR): NURSINGTB.COM a. every 15 minutes in the active phase of the first stage of labor in the absence of risk factors. b. every 20 minutes in the second stage, regardless of whether risk factors are present. c. before and after ambulation and rupture of membranes. d. more often in a woman's first pregnancy. ANS: C The FHR should be auscultated before and after administration of medications and induction of anesthesia. In the active phase of the first stage of labor, the FHR should be auscultated every 30 minutes if no risk factors are involved; with risk factors it should be auscultated every 15 minutes. In the second stage of labor the FHR should be auscultated every 15 minutes if no risk factors are involved; with risk factors it should be auscultated every 5 minutes. The fetus of a first-time mother is automatically at greater risk. PTS: 1 DIF: Cognitive Level: Comprehension OBJ: Nursing Process: Implementation MSC: Client Needs: Health Promotion and Maintenance 20. When using intermittent auscultation (IA) for fetal heart rate, nurses should be aware that: a. they can be expected to cover only two or three patients when IA is the primary method of fetal assessment. b. the best course is to use the descriptive terms associated with electronic fetal monitoring (EFM) when documenting results. c. if the heartbeat cannot be found immediately, a shift must be made to EFM. d. ultrasound can be used to find the fetal heartbeat and reassure the mother if initial difficulty was a factor. ANS: D Locating fetal heartbeats often takes time. Mothers can be reassured verbally and by the ultrasound pictures if ultrasound is used to help locate the heartbeat. When used as the primary method of fetal assessment, auscultation requires a nurse-to-patient ratio of one to one. Documentation should use only terms that can be numerically defined; the usual visual descriptions of EFM are inappropriate. PTS: 1 DIF: Cognitive Level: Comprehension OBJ: Nursing Process: Assessment, Planning MSC: Client Needs: Health Promotion and Maintenance 21. When using intermittent auscultation (IA) to assess uterine activity, the nurse should be cognizant that: a. the examiner's hand should be placed over the fundus before, during, and after contractions. b. the frequency and duration of contractions is measured in seconds for consistency. c. contraction intensity is given a judgment number of 1 to 7 by the nurse and patient together. d. the resting tone between contractions is described as either placid or turbulent. ANS: A The assessment is done by palpation; duration, frequency, intensity, and resting tone must be assessed. The duration of contractions is measured in seconds; the frequency is measured in minutes. The intensity of contractNioUnRs SuIsNuGaTllBy.CisOdMescribed as mild, moderate, or strong. The resting tone usually is characterized as soft or relaxed. PTS: 1 DIF: Cognitive Level: Knowledge OBJ: Nursing Process: Assessment MSC: Client Needs: Health Promotion and Maintenance 22. What is an advantage of external electronic fetal monitoring? a. The ultrasound transducer can accurately measure short-term variability and beat-to-beat changes in the fetal heart rate. b. The tocotransducer can measure and record the frequency, regularity, intensity, and approximate duration of uterine contractions (UCs). c. The tocotransducer is especially valuable for measuring uterine activity during the first stage of labor. d. Once correctly applied by the nurse, the transducer need not be repositioned even when the woman changes positions. ANS: C The tocotransducer is especially valuable for measuring uterine activity during the first stage of labor, particularly when the membranes are intact. Short-term changes cannot be measured with this technology. The tocotransducer cannot measure and record the intensity of UCs. The transducer must be repositioned when the woman or fetus changes position. PTS: 1 DIF: Cognitive Level: Comprehension OBJ: Nursing Process: Implementation MSC: Client Needs: Health Promotion and Maintenance 23. When assessing the relative advantages and disadvantages of internal and external electronic fetal monitoring, nurses comprehend that both: a. can be used when membranes are intact. b. measure the frequency, duration, and intensity of uterine contractions. c. may need to rely on the woman to indicate when uterine activity (UA) is occurring. d. can be used during the antepartum and intrapartum periods. ANS: D External monitoring can be used in both periods; internal monitoring can be used only in the intrapartum period. For internal monitoring the membranes must have ruptured, and the cervix must be sufficiently dilated. Internal monitoring measures the intensity of contractions; external monitoring cannot do this. With external monitoring, the woman may need to alert the nurse that UA is occurring; internal monitoring does not require this. PTS: 1 DIF: Cognitive Level: Comprehension OBJ: Nursing Process: Planning MSC: Client Needs: Health Promotion and Maintenance 24. During labor a fetus with an average heart rate of 135 beats/min over a 10-minute period would be considered to have: a. bradycardia. b. a normal baseline heart rate. c. tachycardia. d. hypoxia. ANS: B The baseline heart rate is measured over 10 minutes; a normal range is 110 to 160 beats/min. Bradycardia is a fetal heart rate (FHR) below 110 beats/min for 10 minutes or longer. Tachycardia is an FHR over 160 bNeUatRsS/mINiGnTfBo.rC1O0Mminutes or longer. Hypoxia is an inadequate supply of oxygen; no indication of this condition exists with a baseline heart rate in the normal range. PTS: 1 DIF: Cognitive Level: Knowledge OBJ: Nursing Process: Assessment MSC: Client Needs: Physiologic Integrity 25. The nurse caring for the woman in labor should understand that increased variability of the fetal heart rate may be caused by: a. narcotics. b. barbiturates. c. methamphetamines. d. tranquilizers. ANS: C Narcotics, barbiturates, and tranquilizers may be causes of decreased variability; methamphetamines may cause increased variability. PTS: 1 DIF: Cognitive Level: Comprehension OBJ: Nursing Process: Assessment MSC: Client Needs: Health Promotion and Maintenance 26. Which deceleration of the fetal heart rate would not require the nurse to change the maternal position? a. Early decelerations b. Late decelerations c. Variable decelerations d. It is always a good idea to change the woman's position. ANS: A Early decelerations (and accelerations) generally do not need any nursing intervention. Late decelerations suggest that the nurse should change the maternal position (lateral); variable decelerations also require a maternal position change (side to side). Although changing positions throughout labor is recommended, it is not required in response to early decelerations. PTS: 1 DIF: Cognitive Level: Comprehension OBJ: Nursing Process: Assessment, Planning MSC: Client Needs: Physiologic Integrity 27. What correctly matches the type of deceleration with its likely cause? a. Early deceleration—umbilical cord compression b. Late deceleration—uteroplacental inefficiency c. Variable deceleration—head compression d. Prolonged deceleration—cause unknown ANS: B Late deceleration is caused by uteroplacental inefficiency. Early deceleration is caused by head compression. Variable deceleration is caused by umbilical cord compression. Prolonged deceleration has a variety of either benign or critical causes. PTS: 1 DIF: Cognitive Level: Knowledge OBJ: Nursing Process: Assessment MSC: Client Needs: Physiologic Integrity NURSINGTB.COM 28. A nurse may be called on to stimulate the fetal scalp: a. as part of fetal scalp blood sampling. b. in response to tocolysis. c. in preparation for fetal oxygen saturation monitoring. d. to elicit an acceleration in the fetal heart rate (FHR). ANS: D The scalp can be stimulated using digital pressure during a vaginal examination. Fetal scalp blood sampling involves swabbing the scalp with disinfectant before a sample is collected. The nurse would stimulate the fetal scalp to elicit an acceleration of the FHR. Tocolysis is relaxation of the uterus. Fetal oxygen saturation monitoring involves the insertion of a sensor. PTS: 1 DIF: Cognitive Level: Application OBJ: Nursing Process: Implementation MSC: Client Needs: Health Promotion and Maintenance 29. In assisting with the two factors that have an effect on fetal status (i.e., pushing and positioning), nurses should: a. encourage the woman's cooperation in avoiding the supine position. b. advise the woman to avoid the semi-Fowler position. c. encourage the woman to hold her breath and tighten her abdominal muscles to produce a vaginal response. d. instruct the woman to open her mouth and close her glottis, letting air escape after the push. ANS: A The woman should maintain a side-lying position. The semi-Fowler position is the recommended side-lying position with a lateral tilt to the uterus. The Valsalva maneuver, which encourages the woman to hold her breath and tighten her abdominal muscles, should be avoided. Both the mouth and glottis should be open, letting air escape during the push. PTS: 1 DIF: Cognitive Level: Comprehension OBJ: Nursing Process: Implementation MSC: Client Needs: Health Promotion and Maintenance 30. Which maternal condition is considered a contraindication for the application of internal monitoring devices? a. Unruptured membranes b. Cervix dilated to 4 cm c. External monitors in current use d. Fetus with a known heart defect ANS: A In order to apply internal monitoring devices, the membranes must be ruptured. Cervical dilation of 4 cm permits the insertion of fetal scalp electrodes and intrauterine catheter. The external monitor can be discontinued after the internal ones are applied. A compromised fetus should be monitored with the most accurate monitoring devices. PTS: 1 DIF: Cognitive Level: Comprehension OBJ: Nursing Process: Planning MSC: Client Needs: Physiologic Integrity 31. The nurse knows that proper placement of the tocotransducer for electronic fetal monitoring is located: a. over the uterine fundus. b. on the fetal scalp. c. inside the uterus. NURSINGTB.COM d. over the mother's lower abdomen. ANS: A The tocotransducer monitors uterine activity and should be placed over the fundus, where the most intensive uterine contractions occur. The tocotransducer is for external use. PTS: 1 DIF: Cognitive Level: Comprehension OBJ: Nursing Process: Implementation MSC: Client Needs: Health Promotion and Maintenance MULTIPLE RESPONSE 1. A tiered system of categorizing FHR has been recommended by regulatory agencies. Nurses, midwives, and physicians who care for women in labor must have a working knowledge of fetal monitoring standards and understand the significance of each category. These categories include: (Select all that apply.) a. reassuring. b. Category I. c. Category II. d. nonreassuring. e. Category III. ANS: B, C, E The three tiered system of FHR tracings include Category I, II, and III. Category I is a normal tracing requiring no action. Category II FHR tracings are indeterminate. This category includes tracings that do not meet Category I or III criteria. Category III tracings are abnormal and require immediate intervention. PTS: 1 DIF: Cognitive Level: Comprehension OBJ: Nursing Process: Assessment, Planning MSC: Client Needs: Physiologic Integrity 2. The baseline fetal heart rate (FHR) is the average rate during a 10-minute segment. Changes in FHR are categorized as periodic or episodic. These patterns include both accelerations and decelerations. The labor nurse is evaluating the patient's most recent 10-minute segment on the monitor strip and notes a late deceleration. This is likely to be caused by which physiologic alteration? (Select all that apply.) a. Spontaneous fetal movement b. Compression of the fetal head c. Placental abruption d. Cord around the baby's neck e. Maternal supine hypotension ANS: C, E Late decelerations are almost always caused by uteroplacental insufficiency. Insufficiency is caused by uterine tachysystole, maternal hypotension, epidural or spinal anesthesia, IUGR, intraamniotic infection, or placental abruption. Spontaneous fetal movement, vaginal examination, fetal scalp stimulation, fetal reaction to external sounds, uterine contractions, fundal pressure and abdominal palpation are all likely to cause accelerations of the FHR. Early decelerations are most oftenNUthReSrIeNsGuTltBo.Cf OfeMtal head compression and may be caused by uterine contractions, fundal pressure, vaginal examination, and placement of an internal electrode. A variable deceleration is likely caused by umbilical cord compression. This may happen when the umbilical cord is around the baby's neck, arm, leg, or other body part or when there is a short cord, a knot in the cord, or a prolapsed cord. PTS: 1 DIF: Cognitive Level: Analysis OBJ: Nursing Process: Assessment, Diagnosis MSC: Client Needs: Physiologic Integrity MATCHING Fetal well-being in labor can be measured by the response of the FHR to uterine contractions. Please match the characteristic of normal uterine activity during labor with the correct description. a. Frequency b. Duration c. Strength d. Resting tone e. Relaxation time 1. Commonly 45 seconds or more in the second stage of labor. 2. Generally ranging from two to five contractions per 10 minutes of labor. 3. Average of 10 mm Hg. 4. Peaking at 40 to 70 mm Hg in the first stage of labor. 5. Remaining fairly stable throughout the first and second stages. 1. ANS: E PTS: 1 DIF: Cognitive Level: Evaluation OBJ: Nursing Process: Assessment MSC: Client Needs: Physiologic Integrity NOT: Should the FHR respond outside of these evidence-based parameters for uterine activity during labor, intervention may be required. Contraction frequency generally ranges from 2 to 5 contractions per 10 minute during labor, with lower frequencies seen in the first stage of labor and higher frequencies during the second stage. Duration of contractions remains fairly stable throughout the first and second stages and rarely exceeds 90 seconds. The strength of uterine contractions ranges from 40 to 70 mm Hg in the first stage and may rise to more than 80 mm Hg in the second stage. Resting tone averages 10 mm Hg, and if using palpation, the uterus should feel soft. In the first stage of labor, relaxation time is commonly 60 seconds or more, and it is 45 seconds or more in the second stage. 2. ANS: A PTS: 1 DIF: Cognitive Level: Evaluation OBJ: Nursing Process: Assessment MSC: Client Needs: Physiologic Integrity NOT: Should the FHR respond outside of these evidence-based parameters for uterine activity during labor, intervention may be required. Contraction frequency generally ranges from 2 to 5 contractions per 10 minute during labor, with lower frequencies seen in the first stage of labor and higher frequencies during the second stage. Duration of contractions remains fairly stable throughout the first and second stages and rarely exceeds 90 seconds. The strength of uterine contractions ranges from 40 to 70 mm Hg in the first stage and may rise to more than 80 mm Hg in the second stage. Resting tone averages 10 mm Hg, and if using palpation, the uterus should feel soft. In the first stage of labor, relaxation time is commonly 60 seconds or more, and it is 45 seconds or more in the second stage. 3. ANS: D PTS: 1 DIF: Cognitive Level: Evaluation OBJ: Nursing Process: Assessment MSC: Client Needs: Physiologic Integrity NOT: Should the FHR respond outside of these evidence-based parameters for uterine activity during labor, intervention may be required. CNoUnRtrSaIcNtGioTnBf.rCeOquMency generally ranges from 2 to 5 contractions per 10 minute during labor, with lower frequencies seen in the first stage of labor and higher frequencies during the second stage. Duration of contractions remains fairly stable throughout the first and second stages and rarely exceeds 90 seconds. The strength of uterine contractions ranges from 40 to 70 mm Hg in the first stage and may rise to more than 80 mm Hg in the second stage. Resting tone averages 10 mm Hg, and if using palpation, the uterus should feel soft. In the first stage of labor, relaxation time is commonly 60 seconds or more, and it is 45 seconds or more in the second stage. 4. ANS: C PTS: 1 DIF: Cognitive Level: Evaluation OBJ: Nursing Process: Assessment MSC: Client Needs: Physiologic Integrity NOT: Should the FHR respond outside of these evidence-based parameters for uterine activity during labor, intervention may be required. Contraction frequency generally ranges from 2 to 5 contractions per 10 minute during labor, with lower frequencies seen in the first stage of labor and higher frequencies during the second stage. Duration of contractions remains fairly stable throughout the first and second stages and rarely exceeds 90 seconds. The strength of uterine contractions ranges from 40 to 70 mm Hg in the first stage and may rise to more than 80 mm Hg in the second stage. Resting tone averages 10 mm Hg, and if using palpation, the uterus should feel soft. In the first stage of labor, relaxation time is commonly 60 seconds or more, and it is 45 seconds or more in the second stage. 5. ANS: B PTS: 1 DIF: Cognitive Level: Evaluation OBJ: Nursing Process: Assessment MSC: Client Needs: Physiologic Integrity NOT: Should the FHR respond outside of these evidence-based parameters for uterine activity during labor, intervention may be required. Contraction frequency generally ranges from 2 to 5 contractions per 10 minute during labor, with lower frequencies seen in the first stage of labor and higher frequencies during the second stage. Duration of contractions remains fairly stable throughout the first and second stages and rarely exceeds 90 seconds. The strength of uterine contractions ranges from 40 to 70 mm Hg in the first stage and may rise to more than 80 mm Hg in the second stage. Resting tone averages 10 mm Hg, and if using palpation, the uterus should feel soft. In the first stage of labor, relaxation time is commonly 60 seconds or more, and it is 45 seconds or more in the second stage. NURSINGTB.COM Chapter 16: Nursing Care of the Family During Labor and Birth Perry: Maternal Child Nursing Care, 6th Edition MULTIPLE CHOICE 1. The nurse recognizes that a woman is in true labor when she states: a. “I passed some thick, pink mucus when I urinated this morning.” b. “My bag of waters just broke.” c. “The contractions in my uterus are getting stronger and closer together.” d. “My baby dropped, and I have to urinate more frequently now.” ANS: C Regular, strong contractions with the presence of cervical change indicate that the woman is experiencing true labor. Loss of the mucous plug (operculum) often occurs during the first stage of labor or before the onset of labor, but it is not the indicator of true labor. Spontaneous rupture of membranes often occurs during the first stage of labor, but it is not the indicator of true labor. The presenting part of the fetus typically becomes engaged in the pelvis at the onset of labor, but this is not the indicator of true labor. PTS: 1 DIF: Cognitive Level: Application OBJ: Nursing Process: Assessment MSC: Client Needs: Health Promotion and Maintenance 2. The nurse teaches a pregnant woman about the characteristics of true labor contractions. The nurse evaluates the woman's understanding of the instructions when she states, “True labor contractions will: a. subside when I walk around.”NURSINGTB.COM b. cause discomfort over the top of my uterus.” c. continue and get stronger even if I relax and take a shower.” d. remain irregular but become stronger.” ANS: C True labor contractions occur regularly, becoming stronger, lasting longer, and occurring closer together. They may become intense during walking and continue despite comfort measures. Typically true labor contractions are felt in the lower back, radiating to the lower portion of the abdomen. During false labor, contractions tend to be irregular and felt in the abdomen above the navel. Typically the contractions often stop with walking or a change of position. PTS: 1 DIF: Cognitive Level: Application OBJ: Nursing Process: Evaluation MSC: Client Needs: Health Promotion and Maintenance 3. When a nulliparous woman telephones the hospital to report that she is in labor, the nurse initially should: a. tell the woman to stay home until her membranes rupture. b. emphasize that food and fluid intake should stop. c. arrange for the woman to come to the hospital for labor evaluation. d. ask the woman to describe why she believes she is in labor. ANS: D Assessment begins at the first contact with the woman, whether by telephone or in person. By asking the woman to describe her signs and symptoms, the nurse can begin the assessment and gather data. The amniotic membranes may or may not spontaneously rupture during labor. The patient may be instructed to stay home until the uterine contractions become strong and regular. The nurse may want to discuss the appropriate oral intake for early labor such as light foods or clear liquids, depending on the preference of the patient or her primary health care provider. Before instructing the woman to come to the hospital, the nurse should initiate the assessment during the telephone interview. PTS: 1 DIF: Cognitive Level: Application OBJ: Nursing Process: Assessment MSC: Client Needs: Health Promotion and Maintenance 4. When planning care for a laboring woman whose membranes have ruptured, the nurse recognizes that the woman's risk for has increased. a. intrauterine infection b. hemorrhage c. precipitous labor d. supine hypotension ANS: A When the membranes rupture, microorganisms from the vagina can ascend into the amniotic sac and cause chorioamnionitis and placentitis. Rupture of membranes (ROM) is not associated with fetal or maternal bleeding. Although ROM may increase the intensity of contractions and facilitate active labor, it does not result in precipitous labor. ROM has no correlation with supine hypotension. PTS: 1 DIF: Cognitive Level: Comprehension OBJ: Nursing Process: Diagnosis, PNlaUnRnSinINgGTB.COM MSC: Client Needs: Physiologic Integrity 5. Which action is correct when palpation is used to assess the characteristics and pattern of uterine contractions? a. Place the hand on the abdomen below the umbilicus and palpate uterine tone with the fingertips. b. Determine the frequency by timing from the end of one contraction to the end of the next contraction. c. Evaluate the intensity by pressing the fingertips into the uterine fundus. d. Assess uterine contractions every 30 minutes throughout the first stage of labor. ANS: C The nurse or primary care provider may assess uterine activity by palpating the fundal section of the uterus using the fingertips. Many women may experience labor pain in the lower segment of the uterus that may be unrelated to the firmness of the contraction detectable in the uterine fundus. The frequency of uterine contractions is determined by palpating from the beginning of one contraction to the beginning of the next contraction. Assessment of uterine activity is performed in intervals based on the stage of labor. As labor progresses this assessment is performed more frequently. PTS: 1 DIF: Cognitive Level: Application OBJ: Nursing Process: Assessment MSC: Client Needs: Health Promotion and Maintenance 6. When assessing a woman in the first stage of labor, the nurse recognizes that the most conclusive sign that uterine contractions are effective would be: a. dilation of the cervix. b. descent of the fetus. c. rupture of the amniotic membranes. d. increase in bloody show. ANS: A The vaginal examination reveals whether the woman is in true labor. Cervical change, especially dilation, in the presence of adequate labor indicates that the woman is in true labor. Descent of the fetus, or engagement, may occur before labor. Rupture of membranes may occur with or without the presence of labor. Bloody show may indicate slow, progressive cervical change (e.g., effacement) in both true and false labor. PTS: 1 DIF: Cognitive Level: Comprehension OBJ: Nursing Process: Assessment, Diagnosis MSC: Client Needs: Health Promotion and Maintenance 7. The nurse who performs vaginal examinations to assess a woman's progress in labor should: a. perform an examination at least once every hour during the active phase of labor. b. perform the examination with the woman in the supine position. c. wear two clean gloves for each examination. d. discuss the findings with the woman and her partner. ANS: D The nurse should discuss the findings of the vaginal examination with the woman and her partner and report them to the primary care provider. A vaginal examination should be performed only when indicated byNtUhReSsItNaGtuTsBo.Cf OthMe woman and her fetus. The woman should be positioned to avoid supine hypotension. The examiner should wear a sterile glove while performing a vaginal examination for a laboring woman. PTS: 1 DIF: Cognitive Level: Application OBJ: Nursing Process: Assessment MSC: Client Needs: Psychosocial Integrity 8. A multiparous woman has been in labor for 8 hours. Her membranes have just ruptured. The nurse's initial response would be to: a. prepare the woman for imminent birth. b. notify the woman's primary health care provider. c. document the characteristics of the fluid. d. assess the fetal heart rate and pattern. ANS: D The umbilical cord may prolapse when the membranes rupture. The fetal heart rate and pattern should be monitored closely for several minutes immediately after ROM to ascertain fetal well-being, and the findings should be documented. Rupture of membranes (ROM) may increase the intensity and frequency of the uterine contractions, but it does not indicate that birth is imminent. The nurse may notify the primary care provider after ROM occurs and fetal well-being and the response to ROM have been assessed. The nurse's priority is to assess fetal well-being. The nurse should document the characteristics of the amniotic fluid, but the initial response is to assess fetal well-being and the response to ROM. PTS: 1 DIF: Cognitive Level: Application OBJ: Nursing Process: Assessment MSC: Client Needs: Health Promotion and Maintenance 9. A nulliparous woman who has just begun the second stage of her labor would most likely: a. experience a strong urge to bear down. b. show perineal bulging. c. feel tired yet relieved that the worst is over. d. show an increase in bright red bloody show. ANS: C Common maternal behaviors during the latent phase of the second stage of labor include feeling a sense of accomplishment and optimism because “the worst is over.” During the latent phase of the second stage of labor, the urge to bear down often is absent or only slight during the acme of contractions. Perineal bulging occurs during the transition phase of the second stage of labor, not at the beginning of the second stage. An increase in bright red bloody show occurs during the descent phase of the second stage of labor. PTS: 1 DIF: Cognitive Level: Comprehension OBJ: Nursing Process: Evaluation MSC: Client Needs: Psychosocial Integrity 10. The nurse knows that the second stage of labor, the descent phase, has begun when: a. the amniotic membranes rupture. b. the cervix cannot be felt during a vaginal examination. c. the woman experiences a strong urge to bear down. d. the presenting part is below the ischial spines. ANS: C During the descent phase of the seNcUonRdSIsNtGagTeBo.CfOlaMbor, the woman may experience an increase in the urge to bear down. Rupture of membranes has no significance in determining the stage of labor. The second stage of labor begins with full cervical dilation. Many women may have an urge to bear down when the presenting part is below the level of the ischial spines. This can occur during the first stage of labor, as early as 5-cm dilation. PTS: 1 DIF: Cognitive Level: Comprehension OBJ: Nursing Process: Planning MSC: Client Needs: Health Promotion and Maintenance 11. When managing the care of a woman in the second stage of labor, the nurse uses various measures to enhance the progress of fetal descent. These measures include: a. encouraging the woman to try various upright positions, including squatting and standing. b. telling the woman to start pushing as soon as her cervix is fully dilated. c. continuing an epidural anesthetic so pain is reduced and the woman can relax. d. coaching the woman to use sustained, 10- to 15-second, closed-glottis bearing-down efforts with each contraction. ANS: A Upright positions and squatting both may enhance the progress of fetal descent. Many factors dictate when a woman will begin pushing. Complete cervical dilation is necessary, but it is only one factor. If the fetal head is still in a higher pelvic station, the physician or midwife may allow the woman to “labor down” (allowing more time for fetal descent, thereby reducing the amount of pushing needed) if the woman is able. The epidural may mask the sensations and muscle control needed for the woman to push effectively. Closed glottic breathing may trigger the Valsalva maneuver, which increases intrathoracic and cardiovascular pressures, reducing cardiac output and inhibiting perfusion of the uterus and placenta. In addition, holding the breath for longer than 5 to 7 seconds diminishes the perfusion of oxygen across the placenta and results in fetal hypoxia. PTS: 1 DIF: Cognitive Level: Comprehension OBJ: Nursing Process: Implementation MSC: Client Needs: Health Promotion and Maintenance 12. Through vaginal examination the nurse determines that a woman is 4 cm dilated, and the external fetal monitor shows uterine contractions every 3.5 to 4 minutes. The nurse would report this as: a. first stage, latent phase. b. first stage, active phase. c. first stage, transition phase. d. second stage, latent phase. ANS: B The first stage, active phase of maternal progress indicates that the woman is in the active phase of the first stage of labor. During the latent phase of the first stage of labor, the expected maternal progress would be 0 to 3 cm dilation with contractions every 5 to 30 minutes. During the transition phase of the first staNgeURoSf IlNabGoTrB, .tChOeMexpected maternal progress is 8 to 10 cm dilation with contractions every 2 to 3 minutes. During the latent phase of the second stage of labor, the woman is completely dilated and experiences a restful period of “laboring down.” PTS: 1 DIF: Cognitive Level: Comprehension OBJ: Nursing Process: Assessment MSC: Client Needs: Health Promotion and Maintenance 13. The most critical nursing action in caring for the newborn immediately after birth is: a. keeping the newborn's airway clear. b. fostering parent-newborn attachment. c. drying the newborn and wrapping the infant in a blanket. d. administering eyedrops and vitamin K. ANS: A The care given immediately after the birth focuses on assessing and stabilizing the newborn. Although fostering parent-infant attachment is an important task for the nurse, it is not the most critical nursing action in caring for the newborn immediately after birth. The nursing activities would be (in order of importance) to maintain a patent airway, support respiratory effort, and prevent cold stress by drying the newborn and covering the infant with a warmed blanket or placing the newborn under a radiant warmer. After the newborn has been stabilized, the nurse assesses the newborn's physical condition, weighs and measures the newborn, administers prophylactic eye ointment and a vitamin K injection, affixes an identification bracelet, wraps the newborn in warm blankets, and then gives the infant to the partner or mother when he or she is ready. PTS: 1 DIF: Cognitive Level: Comprehension OBJ: Nursing Process: Implementation MSC: Client Needs: Health Promotion and Maintenance 14. When assessing a multiparous woman who has just given birth to an 8-lb boy, the nurse notes that the woman's fundus is firm and has become globular in shape. A gush of dark red blood comes from her vagina. The nurse concludes that: a. the placenta has separated. b. a cervical tear occurred during the birth. c. the woman is beginning to hemorrhage. d. clots have formed in the upper uterine segment. ANS: A Placental separation is indicated by a firmly contracting uterus, a change in the uterus from a discoid to a globular ovoid shape, a sudden gush of dark red blood from the introitus, an apparent lengthening of the umbilical cord, and a finding of vaginal fullness. Cervical tears that do not extend to the vagina result in minimal blood loss. Signs of hemorrhage are a boggy uterus, bright red vaginal bleeding, alterations in vital signs, pallor, lightheadedness, restlessness, decreased urinary output, and alteration in the level of consciousness. If clots have formed in the upper uterine segment, the nurse would expect to find the uterus boggy and displaced to the side. PTS: 1 DIF: Cognitive Level: Comprehension OBJ: Nursing Process: Assessment, Diagnosis MSC: Client Needs: Health Promotion and Maintenance 15. The nurse expects to administer an oxytocic (e.g., Pitocin, Methergine) to a woman after expulsion of her placenta to: a. relieve pain. b. stimulate uterine contraction. c. prevent infection. d. facilitate rest and relaxation. ANS: B NURSINGTB.COM Oxytocics stimulate uterine contractions, which reduce blood loss after the third stage of labor. Oxytocics are not used to treat pain or prevent infection. They cause the uterus to contract, which reduces blood loss. Oxytocics do not facilitate rest and relaxation. PTS: 1 DIF: Cognitive Level: Knowledge OBJ: Nursing Process: Planning, Implementation MSC: Client Needs: Health Promotion and Maintenance 16. After an emergency birth, the nurse encourages the woman to breastfeed her newborn. The primary purpose of this activity is to: a. facilitate maternal-newborn interaction. b. stimulate the uterus to contract. c. prevent neonatal hypoglycemia. d. initiate the lactation cycle. ANS: B Stimulation of the nipples through breastfeeding or manual stimulation causes the release of oxytocin and prevents maternal hemorrhage. Breastfeeding facilitates maternal-newborn interaction, but it is not the primary reason a woman is encouraged to breastfeed after an emergency birth. The primary intervention for preventing neonatal hypoglycemia is thermoregulation. Cold stress can result in hypoglycemia. The woman is encouraged to breastfeed after an emergency birth to stimulate the release of oxytocin, which prevents hemorrhage. Breastfeeding is encouraged to initiate the lactation cycle, but it is not the primary reason for this activity after an emergency birth. PTS: 1 DIF: Cognitive Level: Comprehension OBJ: Nursing Process: Implementation MSC: Client Needs: Physiologic Integrity 17. A pregnant woman is in her third trimester. She asks the nurse to explain how she can tell true labor from false labor. The nurse would explain that “true” labor contractions: a. increase with activity such as ambulation. b. decrease with activity. c. are always accompanied by the rupture of the bag of waters. d. alternate between a regular and an irregular pattern. ANS: A True labor contractions become more intense with walking. False labor contractions often stop with walking or position changes. Rupture of membranes may occur before or during labor. True labor contractions are regular. PTS: 1 DIF: Cognitive Level: Comprehension OBJ: Nursing Process: Planning MSC: Client Needs: Health Promotion and Maintenance 18. A woman who is 39 weeks pregnaNnUt ReSxIpNrGesTsBe.sCfOeMar about her impending labor and how she will manage. The nurse's best response is: a. “Don't worry about it. You'll do fine.” b. “It's normal to be anxious about labor. Let's discuss what makes you afraid.” c. “Labor is scary to think about, but the actual experience isn't.” d. “You can have an epidural. You won't feel anything.” ANS: B “It's normal to be anxious about labor. Let's discuss what makes you afraid” allows the woman to share her concerns with the nurse and is a therapeutic communication tool. “Don't worry about it. You'll do fine” negates the woman's fears and is not therapeutic. “Labor is scary to think about, but the actual experience isn't” negates the woman's fears and offers a false sense of security. It is not true that every woman may have an epidural. A number of criteria must be met for use of an epidural. Furthermore, many women still experience the feeling of pressure with an epidural. PTS: 1 DIF: Cognitive Level: Application OBJ: Nursing Process: Planning MSC: Client Needs: Psychosocial Integrity 19. For the labor nurse, care of the expectant mother begins with any or all of these situations, with the exception of: a. the onset of progressive, regular contractions. b. the bloody, or pink, show. c. the spontaneous rupture of membranes. d. formulation of the woman's plan of care for labor. ANS: D Labor care begins when progressive, regular contractions begin; the blood-tinged mucoid vaginal discharge appears; or fluid is discharged from the vagina. The woman and nurse can formulate their plan of care before labor or during treatment. PTS: 1 DIF: Cognitive Level: Knowledge OBJ: Nursing Process: Implementation MSC: Client Needs: Health Promotion and Maintenance 20. Nurses can help their patients by keeping them informed about the distinctive stages of labor. Which description of the phases of the first stage of labor is accurate? a. Latent: Mild, regular contractions; no dilation; bloody show; duration of 2 to 4 hours b. Active: Moderate, regular contractions; 4- to 7-cm dilation; duration of 3 to 6 hours c. Lull: No contractions; dilation stable; duration of 20 to 60 minutes d. Transition: Very strong but irregular contractions; 8- to 10-cm dilation; duration of 1 to 2 hours ANS: B The active phase is characterized by moderate, regular contractions; 4- to 7-cm dilation; and a duration of 3 to 6 hours. The latent phase is characterized by mild-to-moderate, irregular contractions; dilation up to 3 cm; brownish-to-pale pink mucus, and a duration of 6 to 8 hours. No official “lull” phase exists in the first stage. The transition phase is characterized by strong to very strong, regular contractions; 8- to 10-cm dilation; and a duration of 20 to 40 minutes. PTS: 1 DIF: CognitiNveURLSevINelG: TCBo.mCOprMehension OBJ: Nursing Process: Planning MSC: Client Needs: Health Promotion and Maintenance 21. It is paramount for the obstetric nurse to understand the regulatory procedures and criteria for admitting a woman to the hospital labor unit. Which guideline is an important legal requirement of maternity care? a. The patient is not considered to be in true labor (according to the Emergency Medical Treatment and Active Labor Act [EMTALA]) until a qualified health care provider says she is. b. The woman can have only her male partner or predesignated “doula” with her at assessment. c. The patient's weight gain is calculated to determine whether she is at greater risk for cephalopelvic disproportion (CPD) and cesarean birth. d. The nurse may exchange information about the patient with family members. ANS: C According to EMTALA, a woman is entitled to active labor care and is presumed to be in “true” labor until a qualified health care provider certifies otherwise. A woman can have anyone she wishes present for her support. The risk for CPD is especially great for petite women or those who have gained 16 kg or more. All patients should have their weight and BMI calculated on admission. This is part of standard nursing care on a maternity unit and not a regulatory concern. According to the Health Insurance Portability and Accountability Act (HIPAA), the patient must give consent for others to receive any information related to her condition. PTS: 1 DIF: Cognitive Level: Comprehension OBJ: Nursing Process: Planning MSC: Client Needs: Health Promotion and Maintenance 22. Leopold maneuvers would be an inappropriate method of assessment to determine: a. gender of the fetus. b. number of fetuses. c. fetal lie and attitude. d. degree of the presenting part's descent into the pelvis. ANS: A Leopold maneuvers help identify the number of fetuses, the fetal lie and attitude, and the degree of descent of the presenting part into the pelvis. The gender of the fetus is not a goal of the examination at this time. PTS: 1 DIF: Cognitive Level: Knowledge OBJ: Nursing Process: Assessment MSC: Client Needs: Health Promotion and Maintenance 23. In documenting labor experiences, nurses should know that a uterine contraction is described according to all these characteristics except: a. frequency (how often contractions occur). b. intensity (the strength of the contraction at its peak). c. resting tone (the tension in the uterine muscle). d. appearance (shape and height). ANS: D Uterine contractions are described in terms of frequency, intensity, duration, and resting tone. NURSINGTB.COM PTS: 1 DIF: Cognitive Level: Knowledge OBJ: Nursing Process: Implementation MSC: Client Needs: Health Promotion and Maintenance 24. Because the risk for childbirth complications may be revealed, nurses should know that the point of maximal intensity (PMI) of the fetal heart tone (FHT) is: a. usually directly over the fetal abdomen. b. in a vertex position heard above the mother's umbilicus. c. heard lower and closer to the midline of the mother's abdomen as the fetus descends and rotates internally. d. in a breech position heard below the mother's umbilicus. ANS: C Nurses should be prepared for the shift. The PMI of the FHT usually is directly over the fetal back. In a vertex position it is heard below the mother's umbilicus. In a breech position it is heard above the mother's umbilicus. PTS: 1 DIF: Cognitive Level: Comprehension OBJ: Nursing Process: Assessment MSC: Client Needs: Health Promotion and Maintenance 25. With regard to a woman's intake and output during labor, nurses should be aware that: a. the tradition of restricting the laboring woman to clear liquids and ice chips is being challenged because regional anesthesia is used more often than general anesthesia. b. intravenous (IV) fluids usually are necessary to ensure that the laboring woman stays hydrated. c. routine use of an enema empties the rectum and is very helpful for producing a clean, clear delivery. d. when a nulliparous woman experiences the urge to defecate, it often means birth will follow quickly. ANS: A Women are awake with regional anesthesia and are able to protect their own airway, which reduces the worry over aspiration. Routine IV fluids during labor are unlikely to be beneficial and may be harmful. Routine use of an enema is at best ineffective and may be harmful. A multiparous woman may feel the urge to defecate and it may mean birth will follow quickly, but not for a first timer. PTS: 1 DIF: Cognitive Level: Comprehension OBJ: Nursing Process: Planning MSC: Client Needs: Health Promotion and Maintenance 26. If a woman complains of back labor pain, the nurse could best suggest that she: a. lie on her back for a while with her knees bent. b. do less walking around. c. take some deep, cleansing breaths. d. lean over a birth ball with her knees on the floor. ANS: D The hands-and-knees position, with or without the aid of a birth ball, should help with the back pain. The supine position should be discouraged. Walking generally is encouraged. PTS: 1 DIF: Cognitive Level: Application OBJ: Nursing Process: ImplementaNtioUnRSINMGSTCB.:CCOlMient Needs: Health Promotion and Maintenance 27. Which description of the phases of the second stage of labor is accurate? a. Latent phase: Feeling sleepy, fetal station 2+ to 4+, duration 30 to 45 minutes b. Active phase: Overwhelmingly strong contractions, Ferguson reflux activated, duration 5 to 15 minutes c. Descent phase: Significant increase in contractions, Ferguson reflux activated, average duration varied d. Transitional phase: Woman “laboring down,” fetal station 0, duration 15 minutes ANS: C The descent phase begins with a significant increase in contractions; the Ferguson reflex is activated, and the duration varies, depending on a number of factors. The latent phase is the lull, or “laboring down,” period at the beginning of the second stage. It lasts 10 to 30 minutes on average. The second stage of labor has no active phase. The transition phase is the final phase in the second stage of labor; contractions are strong and painful. PTS: 1 DIF: Cognitive Level: Comprehension OBJ: Nursing Process: Assessment MSC: Client Needs: Health Promotion and Maintenance 28. Nurses alert to signs of the onset of the second stage of labor can be certain that this stage has begun when: a. the woman has a sudden episode of vomiting. b. the nurse is unable to feel the cervix during a vaginal examination. c. bloody show increases. d. the woman involuntarily bears down. ANS: B The only certain objective sign that the second stage has begun is the inability to feel the cervix because it is fully dilated and effaced. Vomiting, an increase in bloody show, and involuntary bearing down are only suggestions of second-stage labor. PTS: 1 DIF: Cognitive Level: Knowledge OBJ: Nursing Process: Assessment MSC: Client Needs: Health Promotion and Maintenance 29. A means of controlling the birth of the fetal head with a vertex presentation is: a. the Ritgen maneuver. b. fundal pressure. c. the lithotomy position. d. the De Lee apparatus. ANS: A The Ritgen maneuver extends the head during the actual birth and protects the perineum. Gentle, steady pressure against the fundus of the uterus facilitates vaginal birth. The lithotomy position has been commonly used in Western cultures, partly because it is convenient for the health care provider. The De Lee apparatus is used to suction fluid from the infant's mouth. PTS: 1 DIF: Cognitive Level: Knowledge OBJ: Nursing Process: Implementation MSC: Client Needs: Health Promotion and Maintenance 30. Which collection of risk factors most likely would result in damaging lacerations (including episiotomies)? a. A dark-skinned woman who hNasURhaSdINmGTorBe.CthOaMn one pregnancy, who is going through prolonged second-stage labor, and who is attended by a midwife. b. A reddish-haired mother of two who is going through a breech birth. c. A dark-skinned, first-time mother who is going through a long labor. d. A first-time mother with reddish hair whose rapid labor was overseen by an obstetrician. ANS: D Reddish-haired women have tissue that is less distensible than that of darker-skinned women and therefore may have less efficient healing. First time mothers are also more at risk, especially with breech births, long second-stage labors, or rapid labors in which there is insufficient time for the perineum to stretch. The rate of episiotomies is higher when obstetricians rather than midwives attend births. PTS: 1 DIF: Cognitive Level: Application OBJ: Nursing Process: Diagnosis MSC: Client Needs: Physiologic Integrity 31. Concerning the third stage of labor, nurses should be aware that: a. the placenta eventually detaches itself from a flaccid uterus. b. an expectant or active approach to managing this stage of labor reduces the risk of complications. c. it is important that the dark, roughened maternal surface of the placenta appear before the shiny fetal surface. d. the major risk for women during the third stage is a rapid heart rate. ANS: B Active management facilitates placental separation and expulsion, thus reducing the risk of complications. The placenta cannot detach itself from a flaccid (relaxed) uterus. Which surface of the placenta comes out first is not clinically important. The major risk for women during the third stage of labor is after birth hemorrhage. PTS: 1 DIF: Cognitive Level: Comprehension OBJ: Nursing Process: Planning, Implementation MSC: Client Needs: Physiologic Integrity 32. For women who have a history of sexual abuse, a number of traumatic memories may be triggered during labor. The woman may fight the labor process and react with pain or anger. Alternately, she may become a passive player and emotionally absent herself from the process. The nurse is in a unique position of being able to assist the patient to associate the sensations of labor with the process of childbirth and not the past abuse. The nurse can implement a number of care measures to help the patient view the childbirth experience in a positive manner. Which intervention would be key for the nurse to use while providing care? a. Telling the patient to relax and that it won't hurt much. b. Limiting the number of procedures that invade her body. c. Reassuring the patient that as the nurse you know what is best. d. Allowing unlimited care providers to be with the patient. ANS: B The number of invasive procedures such as vaginal examinations, internal monitoring, and intravenous therapy should be limited as much as possible. The nurse should always avoid words and phrases that may result in the patient's recalling the phrases of her abuser (e.g., “Relax, this won't hurt” or “Just open your legs.”) The woman's sense of control should be maintained at all times. The nurseNsUhRouSIlNd GexTpBl.aCiOnMprocedures at the patient's pace and wait for permission to proceed. Protecting the patient's environment by providing privacy and limiting the number of staff who observe the patient will help to make her feel safe. PTS: 1 DIF: Cognitive Level: Comprehension OBJ: Nursing Process: Implementation MSC: Client Needs: Psychosocial Integrity 33. As the United States and Canada continue to become more culturally diverse, it is increasingly important for the nursing staff to recognize a wide range of varying cultural beliefs and practices. Nurses need to develop respect for these culturally diverse practices and learn to incorporate these into a mutually agreed on plan of care. Although it is common practice in the United States for the father of the baby to be present at the birth, in many societies this is not the case. When implementing care, the nurse would anticipate that a woman from which country would have the father of the baby in attendance? a. Mexico b. China c. Iran d. India ANS: A A woman from Mexico may be stoic about discomfort until the second stage, at which time she will request pain relief. Fathers and female relatives are usually in attendance during the second stage of labor. The father of the baby is expected to provide encouragement, support, and reassurance that all will be well. Fathers are usually not present in China. The Iranian father will not be present. Female support persons and female care providers are preferred. For many, a male caregiver is unacceptable. The father is usually not present in India, but female relatives are usually present. Natural childbirth methods are preferred. PTS: 1 DIF: Cognitive Level: Application OBJ: Nursing Process: Planning MSC: Client Needs: Psychosocial Integrity 34. A patient whose cervix is dilated to 5 cm is considered to be in which phase of labor? a. Latent phase b. Active phase c. Second stage d. Third stage ANS: B The latent phase is from the beginning of true labor until 3 cm of cervical dilation. The active phase of labor is characterized by cervical dilation of 4 to 7 cm. The second stage of labor begins when the cervix is completely dilated until the birth of the baby. The third stage of labor is from the birth of the baby until the expulsion of the placenta. This patient is in the active phase of labor. PTS: 1 DIF: Cognitive Level: Knowledge OBJ: Nursing Process: Assessment MSC: Client Needs: Health Promotion and Maintenance 35. The primary difference between thNeUlRaSbIoNrGoTfBa.CnOuMllipara and that of a multipara is the: a. amount of cervical dilation. b. total duration of labor. c. level of pain experienced. d. sequence of labor mechanisms. ANS: B Multiparas usually labor more quickly than nulliparas, thus making the total duration of their labor shorter. Cervical dilation is the same for all labors. The level of pain is individual to the woman, not to the number of labors she has experienced. The sequence of labor mechanisms remains the same with all labors. PTS: 1 DIF: Cognitive Level: Comprehension OBJ: Nursing Process: Assessment MSC: Client Needs: Health Promotion and Maintenance 36. A woman who is gravida 3 para 2 enters the intrapartum unit. The most important nursing assessments are: a. contraction pattern, amount of discomfort, and pregnancy history. b. fetal heart rate, maternal vital signs, and the woman's nearness to birth. c. identification of ruptured membranes, the woman's gravida and para, and her support person. d. last food intake, when labor began, and cultural practices the couple desires. ANS: B All options describe relevant intrapartum nursing assessments; however, this focused assessment has priority. If the maternal and fetal conditions are normal and birth is not imminent, other assessments can be performed in an unhurried manner. This includes: gravida, para, support person, pregnancy history, pain assessment, last food intake, and cultural practices. PTS: 1 DIF: Cognitive Level: Application OBJ: Nursing Process: Assessment MSC: Client Needs: Health Promotion and Maintenance 37. A primigravida at 39 weeks of gestation is observed for 2 hours in the intrapartum unit. The fetal heart rate has been normal. Contractions are 5 to 9 minutes apart, 20 to 30 seconds in duration, and of mild intensity. Cervical dilation is 1 to 2 cm and uneffaced (unchanged from admission). Membranes are intact. The nurse should expect the woman to be: a. admitted and prepared for a cesarean birth. b. admitted for extended observation. c. discharged home with a sedative. d. discharged home to await the onset of true labor. ANS: D This situation describes a woman with normal assessments who is probably in false labor and will likely not deliver rapidly once true labor begins. There is no indication that further assessments or observations are indicated; therefore, the patient will be discharged along with instructions to return when contractions increase in intensity and frequency. Neither a cesarean birth nor a sedative is required at this time. PTS: 1 DIF: Cognitive Level: Analysis OBJ: Nursing Process: Assessment MSC: Client Needs: Safe and EffectNivUeRCSaIrNeGETnBv.iCroOnMment 38. A laboring woman is lying in the supine position. The most appropriate nursing action at this time is to: a. ask her to turn to one side. b. elevate her feet and legs. c. take her blood pressure. d. determine whether fetal tachycardia is present. ANS: A The woman's supine position may cause the heavy uterus to compress her inferior vena cava, thus reducing blood return to her heart and reducing placental blood flow. Elevating her legs will not relieve the pressure from the inferior vena cava. If the woman is allowed to stay in the supine position and blood flow to the placental is reduced significantly, fetal tachycardia may occur. The most appropriate nursing action is to prevent this from occurring by turning the woman to her side. Blood pressure readings may be obtained when the patient is in the appropriate and safest position. PTS: 1 DIF: Cognitive Level: Application OBJ: Nursing Process: Implementation MSC: Client Needs: Physiologic Integrity 39. Which nursing assessment indicates that a woman who is in second-stage labor is almost ready to give birth? a. The fetal head is felt at 0 station during vaginal examination. b. Bloody mucus discharge increases. c. The vulva bulges and encircles the fetal head. d. The membranes rupture during a contraction. ANS: C A bulging vulva that encircles the fetal head describes crowning, which occurs shortly before birth. Birth of the head occurs when the station is +4. A 0 station indicates engagement. Bloody show occurs throughout the labor process and is not an indication of an imminent birth. Rupture of membranes can occur at any time during the labor process and does not indicate an imminent birth. PTS: 1 DIF: Cognitive Level: Analysis OBJ: Nursing Process: Assessment MSC: Client Needs: Health Promotion and Maintenance 40. At 1 minute after birth, the nurse assesses the newborn to assign an Apgar score. The apical heart rate is 110 bpm, and the infant is crying vigorously with the limbs flexed. The infant's trunk is pink, but the hands and feet are blue. What is the correct Apgar score for this infant? a. 7 b. 8 c. 9 d. 10 ANS: C The Apgar score is 9 because 1 point is deducted from the total score of 10 for the infant's blue hands and feet. The baby received 2 points for each of the categories except color. Because the infant's hands and feet were blue, this category is given a grade of 1. PTS: 1 DIF: CognitiNveURLSevINelG: TABp.pClOicMation OBJ: Nursing Process: Assessment MSC: Client Needs: Health Promotion and Maintenance 41. The nurse thoroughly dries the infant immediately after birth primarily to: a. stimulate crying and lung expansion. b. remove maternal blood from the skin surface. c. reduce heat loss from evaporation. d. increase blood supply to the hands and feet. ANS: C Infants are wet with amniotic fluid and blood at birth, and this accelerates evaporative heat loss. The primary purpose of drying the infant is to prevent heat loss. Rubbing the infant does stimulate crying; however, it is not the main reason for drying the infant. This process does not remove all the maternal blood. PTS: 1 DIF: Cognitive Level: Comprehension OBJ: Nursing Process: Implementation MSC: Client Needs: Physiologic Integrity MATCHING The vaginal examination is an essential component of labor assessment. It reveals whether the patient is in true labor and enables the examiner to determine whether membranes have ruptured. This examination is often stressful and uncomfortable for the patient and should be performed only when indicated. Please match the correct step number, from 1 to 7, with each component of a vaginal examination of the laboring woman. a. After obtaining permission, gently insert the index and middle fingers into the vagina. b. Explain findings to the patient. c. Position the woman to prevent supine hypotension. d. Use sterile gloves and soluble gel for lubrication. e. Document findings and report to the provider. f. Cleanse the perineum and vulva if necessary. g. Determine dilation, presenting part, status of membranes, and characteristics of amniotic fluid. 1. Step 1 2. Step 2 3. Step 3 4. Step 4 5. Step 5 6. Step 6 7. Step 7 1. ANS: D PTS: 1 DIF: Cognitive Level: Application OBJ: Nursing Process: Assessment MSC: Client Needs: Physiologic Integrity NOT: The vaginal examination should be performed on admission, before administering analgesics, when a significant change in uterine NacUtiRvSitIyNhGaTsBo.cCcOuMrred, on maternal perception of perineal pressure, when membranes rupture, or when you note variable decelerations of the fetal heart rate. A full explanation of the examination and support of the woman are important in reducing the level of stress and discomfort. 2. ANS: C PTS: 1 DIF: Cognitive Level: Application OBJ: Nursing Process: Assessment MSC: Client Needs: Physiologic Integrity NOT: The vaginal examination should be performed on admission, before administering analgesics, when a significant change in uterine activity has occurred, on maternal perception of perineal pressure, when membranes rupture, or when you note variable decelerations of the fetal heart rate. A full explanation of the examination and support of the woman are important in reducing the level of stress and discomfort. 3. ANS: F PTS: 1 DIF: Cognitive Level: Application OBJ: Nursing Process: Assessment MSC: Client Needs: Physiologic Integrity NOT: The vaginal examination should be performed on admission, before administering analgesics, when a significant change in uterine activity has occurred, on maternal perception of perineal pressure, when membranes rupture, or when you note variable decelerations of the fetal heart rate. A full explanation of the examination and support of the woman are important in reducing the level of stress and discomfort. 4. ANS: A PTS: 1 DIF: Cognitive Level: Application OBJ: Nursing Process: Assessment MSC: Client Needs: Physiologic Integrity NOT: The vaginal examination should be performed on admission, before administering analgesics, when a significant change in uterine activity has occurred, on maternal perception of perineal pressure, when membranes rupture, or when you note variable decelerations of the fetal heart rate. A full explanation of the examination and support of the woman are important in reducing the level of stress and discomfort. 5. ANS: G PTS: 1 DIF: Cognitive Level: Application OBJ: Nursing Process: Assessment MSC: Client Needs: Physiologic Integrity NOT: The vaginal examination should be performed on admission, before administering analgesics, when a significant change in uterine activity has occurred, on maternal perception of perineal pressure, when membranes rupture, or when you note variable decelerations of the fetal heart rate. A full explanation of the examination and support of the woman are important in reducing the level of stress and discomfort. 6. ANS: B PTS: 1 DIF: Cognitive Level: Application OBJ: Nursing Process: Assessment MSC: Client Needs: Physiologic Integrity NOT: The vaginal examination should be performed on admission, before administering analgesics, when a significant change in uterine activity has occurred, on maternal perception of perineal pressure, when membranes rupture, or when you note variable decelerations of the fetal heart rate. A full explanation of the examination and support of the woman are important in reducing the level of stress and discomfort. 7. ANS: E PTS: 1 DIF: Cognitive Level: Application OBJ: Nursing Process: Assessment MSC: Client Needs: Physiologic Integrity NOT: The vaginal examination should be performed on admission, before administering analgesics, when a significant change in uterine activity has occurred, on maternal perception of perineal pressure, when membranes rupture, or when you note variable decelerations of the fetal heart rate. A full explanation of the examination and support of the woman are important in reducing the level of stress and discomfort. NURSINGTB.COM Chapter 17: Labor and Birth Complications Perry: Maternal Child Nursing Care, 6th Edition MULTIPLE CHOICE 1. In planning for home care of a woman with preterm labor, which concern must the nurse address? a. Nursing assessments will be different from those done in the hospital setting. b. Restricted activity and medications will be necessary to prevent recurrence of preterm labor. c. Prolonged bed rest may cause negative physiologic effects. d. Home health care providers will be necessary. ANS: C Prolonged bed rest may cause adverse effects such as weight loss, loss of appetite, muscle wasting, weakness, bone demineralization, decreased cardiac output, risk for thrombophlebitis, alteration in bowel functions, sleep disturbance, and prolonged after birth recovery. Nursing assessments will differ somewhat from those performed in the acute care setting, but this is not the concern that needs to be addressed. Restricted activity and medication may prevent preterm labor, but not in all women. In addition, the plan of care is individualized to meet the needs of each woman. Many women will receive home health nurse visits, but care is individualized for each woman. PTS: 1 DIF: Cognitive Level: Analysis OBJ: Nursing Process: Planning MSC: Client Needs: Health Promotion and Maintenance NURSINGTB.COM 2. The nurse providing care for a woman with preterm labor who is receiving terbutaline would include which intervention to identify side effects of the drug? a. Assessing deep tendon reflexes (DTRs) b. Assessing for chest discomfort and palpitations c. Assessing for bradycardia d. Assessing for hypoglycemia ANS: B Terbutaline is a 2-adrenergic agonist that affects the cardiopulmonary and metabolic systems of the mother. Signs of cardiopulmonary decompensation would include chest pain and palpitations. Assessing DTRs would not address these concerns. 2-Adrenergic agonist drugs cause tachycardia, not bradycardia. The metabolic effect leads to hyperglycemia, not hypoglycemia. PTS: 1 DIF: Cognitive Level: Analysis OBJ: Nursing Process: Assessment MSC: Client Needs: Physiologic Integrity 3. In evaluating the effectiveness of magnesium sulfate for the treatment of preterm labor, what finding would alert the nurse to possible side effects? a. Urine output of 160 mL in 4 hours b. Deep tendon reflexes 2+ and no clonus c. Respiratory rate of 16 breaths/min d. Serum magnesium level of 10 mg/dL ANS: D The therapeutic range for magnesium sulfate management is 5 to 8 mg/dL. A serum magnesium level of 10 mg/dL could lead to signs and symptoms of magnesium toxicity, including oliguria and respiratory distress. Urine output of 160 mL in 4 hours, deep tendon reflexes 2+ with no clonus, and respiratory rate of 16 breaths/min are normal findings. PTS: 1 DIF: Cognitive Level: Comprehension OBJ: Nursing Process: Evaluation MSC: Client Needs: Physiologic Integrity 4. A woman in preterm labor at 30 weeks of gestation receives two 12-mg doses of betamethasone intramuscularly. The purpose of this pharmacologic treatment is to: a. stimulate fetal surfactant production. b. reduce maternal and fetal tachycardia associated with ritodrine administration. c. suppress uterine contractions. d. maintain adequate maternal respiratory effort and ventilation during magnesium sulfate therapy. ANS: A Antenatal glucocorticoids given as intramuscular injections to the mother accelerate fetal lung maturity. Inderal would be given to reduce the effects of ritodrine administration. Betamethasone has no effect on uterine contractions. Calcium gluconate would be given to reverse the respiratory depressive effects of magnesium sulfate therapy. PTS: 1 DIF: Cognitive Level: Comprehension OBJ: Nursing Process: Planning MSC: Client Needs: Physiologic Integrity 5. A woman at 26 weeks of gestation is being assessed to determine whether she is experiencing preterm labor. What finding indicNatUesRSthINatGpTrBe.tCeOrmMlabor is occurring? a. Estriol is not found in maternal saliva. b. Irregular, mild uterine contractions are occurring every 12 to 15 minutes. c. Fetal fibronectin is present in vaginal secretions. d. The cervix is effacing and dilated to 2 cm. ANS: D Cervical changes such as shortened endocervical length, effacement, and dilation are predictors of imminent preterm labor. Changes in the cervix accompanied by regular contractions indicate labor at any gestation. Estriol is a form of estrogen produced by the fetus that is present in plasma at 9 weeks of gestation. Levels of salivary estriol have been shown to increase before preterm birth. Irregular, mild contractions that do not cause cervical change are not considered a threat. The presence of fetal fibronectin in vaginal secretions between 24 and 36 weeks of gestation could predict preterm labor, but it has only a 20% to 40% positive predictive value. Of more importance are other physiologic clues of preterm labor such as cervical changes. PTS: 1 DIF: Cognitive Level: Application OBJ: Nursing Process: Assessment, Planning MSC: Client Needs: Health Promotion and Maintenance 6. A primigravida at 40 weeks of gestation is having uterine contractions every 1.5 to 2 minutes and says that they are very painful. Her cervix is dilated 2 cm and has not changed in 3 hours. The woman is crying and wants an epidural. What is the likely status of this woman's labor? a. She is exhibiting hypotonic uterine dysfunction. b. She is experiencing a normal latent stage. c. She is exhibiting hypertonic uterine dysfunction. d. She is experiencing pelvic dystocia. ANS: C Women who experience hypertonic uterine dysfunction, or primary dysfunctional labor, often are anxious first-time mothers who are having painful and frequent contractions that are ineffective at causing cervical dilation or effacement to progress. With hypotonic uterine dysfunction, the woman initially makes normal progress into the active stage of labor; then the contractions become weak and inefficient or stop altogether. The contraction pattern seen in this woman signifies hypertonic uterine activity. Typically uterine activity in this phase occurs at 4- to 5-minute intervals lasting 30 to 45 seconds. Pelvic dystocia can occur whenever contractures of the pelvic diameters reduce the capacity of the bony pelvis, including the inlet, midpelvis, outlet, or any combination of these planes. PTS: 1 DIF: Cognitive Level: Application OBJ: Nursing Process: Diagnosis MSC: Client Needs: Health Promotion and Maintenance 7. Which assessment is least likely to be associated with a breech presentation? a. Meconium-stained amniotic fluid b. Fetal heart tones heard at or above the maternal umbilicus c. Preterm labor and birth d. Postterm gestation ANS: D Postterm gestation is not likely to be seen with a breech presentation. The presence of meconium in a breech presentatioNn UmRaSyINreGsTuBlt.CfOroMm pressure on the fetal wall as it traverses the birth canal. Fetal heart tones heard at the level of the umbilical level of the mother are a typical finding in a breech presentation because the fetal back would be located in the upper abdominal area. Breech presentations often occur in preterm births. PTS: 1 DIF: Cognitive Level: Analysis OBJ: Nursing Process: Assessment MSC: Client Needs: Health Promotion and Maintenance 8. A woman is having her first child. She has been in labor for 15 hours. Two hours ago her vaginal examination revealed the cervix to be dilated to 5 cm and 100% effaced, and the presenting part was at station 0. Five minutes ago her vaginal examination indicated that there had been no change. What abnormal labor pattern is associated with this description? a. Prolonged latent phase b. Protracted active phase c. Arrest of active phase d. Protracted descent ANS: C With an arrest of the active phase, the progress of labor has stopped. This patient has not had any anticipated cervical change, thus indicating an arrest of labor. In the nulliparous woman a prolonged latent phase typically would last more than 20 hours. A protracted active phase, the first or second stage of labor, would be prolonged (slow dilation). With protracted descent, the fetus would fail to descend at an anticipated rate during the deceleration phase and second stage of labor. PTS: 1 DIF: Cognitive Level: Analysis OBJ: Nursing Process: Assessment, Diagnosis MSC: Client Needs: Health Promotion and Maintenance 9. In evaluating the effectiveness of oxytocin induction, the nurse would expect: a. contractions lasting 80 to 90 seconds, 2 to 3 minutes apart. b. the intensity of contractions to be at least 110 to 130 mm Hg. c. labor to progress at least 2 cm/hr dilation. d. At least 30 mU/min of oxytocin will be needed to achieve cervical dilation. ANS: A The goal of induction of labor would be to produce contractions that occur every 2 to 3 minutes and last 60 to 90 seconds. The intensity of the contractions should be 80 to 90 mm Hg by intrauterine pressure catheter. Cervical dilation of 1 cm/hr in the active phase of labor would be the goal in an oxytocin induction. The dose is increased by 1 to 2 mU/min at intervals of 30 to 60 minutes until the desired contraction pattern is achieved. Doses are increased up to a maximum of 20 to 40 mU/min. PTS: 1 DIF: Cognitive Level: Analysis OBJ: Nursing Process: Planning MSC: Client Needs: Health Promotion and Maintenance 10. A pregnant woman's amniotic membranes rupture. Prolapsed umbilical cord is suspected. What intervention would be the top priority? a. Placing the woman in the knee-chest position. b. Covering the cord in sterile gauze soaked in saline. c. Preparing the woman for a cesarean birth. d. Starting oxygen by face mask.NURSINGTB.COM ANS: A The woman is assisted into a position (e.g., modified Sims position, Trendelenburg position, or the knee-chest position) in which gravity keeps the pressure of the presenting part off the cord. Although covering the cord in sterile gauze soaked saline, preparing the woman for a cesarean, and starting oxygen by face mark are appropriate nursing interventions in the event of a prolapsed cord, the intervention of top priority would be positioning the mother to relieve cord compression. PTS: 1 DIF: Cognitive Level: Application OBJ: Nursing Process: Implementation MSC: Client Needs: Physiologic Integrity 11. Prepidil (prostaglandin gel) has been ordered for a pregnant woman at 43 weeks of gestation. The nurse recognizes that this medication will be administered to: a. enhance uteroplacental perfusion in an aging placenta. b. increase amniotic fluid volume. c. ripen the cervix in preparation for labor induction. d. stimulate the amniotic membranes to rupture. ANS: C It is accurate to state that Prepidil will be administered to ripen the cervix in preparation for labor induction. It is not administered to enhance uteroplacental perfusion in an aging placenta, increase amniotic fluid volume, or stimulate the amniotic membranes to rupture. PTS: 1 DIF: Cognitive Level: Application OBJ: Nursing Process: Planning MSC: Client Needs: Physiologic Integrity 12. The nurse, caring for a patient whose labor is being augmented with oxytocin, recognizes that the oxytocin should be discontinued immediately if there is evidence of: a. uterine contractions occurring every 8 to 10 minutes. b. a fetal heart rate (FHR) of 180 with absence of variability. c. the patient's needing to void. d. rupture of the patient's amniotic membranes. ANS: B This FHR is nonreassuring. The oxytocin should be discontinued immediately, and the physician should be notified. The oxytocin should be discontinued if uterine hyperstimulation occurs. Uterine contractions that are occurring every 8 to 10 minutes do not qualify as hyperstimulation. The patient's needing to void is not an indication to discontinue the oxytocin induction immediately or to call the physician. Unless a change occurs in the FHR pattern that is nonreassuring or the patient experiences uterine hyperstimulation, the oxytocin does not need to be discontinued. The physician should be notified that the patient's membranes have ruptured. PTS: 1 DIF: Cognitive Level: Evaluation OBJ: Nursing Process: Planning MSC: Client Needs: Physiologic Integrity 13. Nurses should know some basic definitions concerning preterm birth, preterm labor, and low birth weight. For instance: a. the terms preterm birth and low birth weight can be used interchangeably. b. preterm labor is defined as cervical changes and uterine contractions occurring between 20 and 37 weeks of pNreUgRnSaInNcGyT. B.COM c. low birth weight is anything below 3.7 lbs. d. in the United States early in this century, preterm birth accounted for 18% to 20% of all births. ANS: B Before 20 weeks, it is not viable (miscarriage); after 37 weeks, it can be considered term. Although these terms are used interchangeably, they have different meanings: preterm birth describes the length of gestation (37 weeks) regardless of weight; low birth weight describes weight only (2500 g or less) at the time of birth, whenever it occurs. Low birth weight is anything less than 2500 g, or about 5.5 lbs. In 2003 the preterm birth rate in the United States was 12.3%, but it is increasing in frequency. PTS: 1 DIF: Cognitive Level: Knowledge OBJ: Nursing Process: Assessment MSC: Client Needs: Health Promotion and Maintenance 14. With regard to the care management of preterm labor, nurses should be aware that: a. all women must be considered at risk for preterm labor and prediction is so hit-and-miss, teaching pregnant women the symptoms probably causes more harm through false alarms. b. Braxton Hicks contractions often signal the onset of preterm labor. c. preterm labor is likely to be the start of an extended labor, a woman with symptoms can wait several hours before contacting the primary caregiver. d. the diagnosis of preterm labor is based on gestational age, uterine activity, and progressive cervical change. ANS: D Gestational age of 20 to 37 weeks, uterine contractions, and a cervix that is 80% effaced or dilated 2 cm indicates preterm labor. It is essential that nurses teach women how to detect the early symptoms of preterm labor. Braxton Hicks contractions resemble preterm labor contractions, but they are not true labor. Waiting too long to see a health care provider could result in not administering essential medications. Preterm labor is not necessarily long-term labor. PTS: 1 DIF: Cognitive Level: Comprehension OBJ: Nursing Process: Planning MSC: Client Needs: Safe and Effective Care Environment 15. As relates to the use of tocolytic therapy to suppress uterine activity, nurses should be aware that: a. the drugs can be given efficaciously up to the designated beginning of term at 37 weeks. b. there are no important maternal (as opposed to fetal) contraindications. c. its most important function is to afford the opportunity to administer antenatal glucocorticoids. d. if the patient develops pulmonary edema while receiving tocolytics, intravenous (IV) fluids should be given. ANS: C Buying time for antenatal glucocorticoids to accelerate fetal lung development may be the best reason to use tocolytics. Once the pregnancy has reached 34 weeks, the risks of tocolytic therapy outweigh the benefits. There are important maternal contraindications to tocolytic therapy. Tocolytic-induced edemaNcUaRnSbINeGcTauBs.CeOd Mby IV fluids. PTS: 1 DIF: Cognitive Level: Comprehension OBJ: Nursing Process: Planning MSC: Client Needs: Physiologic Integrity 16. With regard to dysfunctional labor, nurses should be aware that: a. women who are underweight are more at risk. b. women experiencing precipitous labor are about the only “dysfunctionals” not to be exhausted. c. hypertonic uterine dysfunction is more common than hypotonic dysfunction. d. abnormal labor patterns are most common in older women. ANS: B Precipitous labor lasts less than 3 hours. Short women more than 30 lbs overweight are more at risk for dysfunctional labor. Hypotonic uterine dysfunction, in which the contractions become weaker, is more common. Abnormal labor patterns are more common in women less than 20 years of age. PTS: 1 DIF: Cognitive Level: Comprehension OBJ: Nursing Process: Planning MSC: Client Needs: Health Promotion and Maintenance 17. The least common cause of long, difficult, or abnormal labor (dystocia) is: a. midplane contracture of the pelvis. b. compromised bearing-down efforts as a result of pain medication. c. disproportion of the pelvis. d. low-lying placenta. ANS: C The least common cause of dystocia is disproportion of the pelvis. PTS: 1 DIF: Cognitive Level: Knowledge OBJ: Nursing Process: Planning MSC: Client Needs: Health Promotion and Maintenance 18. Nurses should be aware that the induction of labor: a. can be achieved by external and internal version techniques. b. is also known as a trial of labor (TOL). c. is almost always done for medical reasons. d. is rated for viability by a Bishop score. ANS: D Induction of labor is likely to be more successful with a Bishop score of 9 or higher for first-time mothers and 5 or higher for veterans. Version is turning of the fetus to a better position by a physician for an easier or safer birth. A trial of labor is the observance of a woman and her fetus for several hours of active labor to assess the safety of vaginal birth. Two thirds of cases of induced labor are elective and are not done for medical reasons. PTS: 1 DIF: Cognitive Level: Comprehension OBJ: Nursing Process: Diagnosis MSC: Client Needs: Safe and Effective Care Environment 19. While caring for the patient who requires an induction of labor, the nurse should be cognizant that: a. ripening the cervix usually results in a decreased success rate for induction. b. labor sometimes can be induceNdUwRSitIhNGbaTlBlo.CoOnMcatheters or laminaria tents. c. oxytocin is less expensive than prostaglandins and more effective but creates greater health risks. d. amniotomy can be used to make the cervix more favorable for labor. ANS: B Balloon catheters or laminaria tents are mechanical means of ripening the cervix. Ripening the cervix, making it softer and thinner, increases the success rate of induced labor. Prostaglandin E1 is less expensive and more effective than oxytocin but carries a greater risk. Amniotomy is the artificial rupture of membranes, which is used to induce labor only when the cervix is already ripe. PTS: 1 DIF: Cognitive Level: Comprehension OBJ: Nursing Process: Planning MSC: Client Needs: Health Promotion and Maintenance 20. With regard to the process of augmentation of labor, the nurse should be aware that it: a. is part of the active management of labor that is instituted when the labor process is unsatisfactory. b. relies on more invasive methods when oxytocin and amniotomy have failed. c. is a modern management term to cover up the negative connotations of forceps-assisted birth. d. uses vacuum cups. ANS: A Augmentation is part of the active management of labor that stimulates uterine contractions after labor has started but is not progressing satisfactorily. Augmentation uses amniotomy and oxytocin infusion, as well as some gentler, noninvasive methods. Forceps-assisted births and vacuum-assisted births are appropriately used at the end of labor and are not part of augmentation. PTS: 1 DIF: Cognitive Level: Comprehension OBJ: Nursing Process: Planning MSC: Client Needs: Health Promotion and Maintenance 21. The exact cause of preterm labor is unknown and believed to be multifactorial. Infection is thought to be a major factor in many preterm labors. Select the type of infection that has not been linked to preterm births. a. Viral b. Periodontal c. Cervical d. Urinary tract ANS: A The infections that increase the risk of preterm labor and birth are all bacterial. They include cervical, urinary tract, periodontal, and other bacterial infections. Therefore, it is important for the patient to participate in early, continual, and comprehensive prenatal care. Evidence has shown a link between periodontal infections and preterm labor. Researchers recommend regular dental care before and during pregnancy, oral assessment as a routine part of prenatal care, and scrupulous oral hygiene to prevent infection. Cervical infections of a bacterial nature have been linked to preterm labor and birth. The presence of urinary tract infections increases the risk of preterm labor and birth. NURSINGTB.COM PTS: 1 DIF: Cognitive Level: Knowledge OBJ: Nursing Process: Assessment MSC: Client Needs: Physiologic Integrity 22. The standard of care for obstetrics dictates that an internal version may be used to manipulate the: a. fetus from a breech to a cephalic presentation before labor begins. b. fetus from a transverse lie to a longitudinal lie before cesarean birth. c. second twin from an oblique lie to a transverse lie before labor begins. d. second twin from a transverse lie to a breech presentation during vaginal birth. ANS: D Internal version is used only during vaginal birth to manipulate the second twin into a presentation that allows it to be born vaginally. For internal version to occur, the cervix needs to be completely dilated. PTS: 1 DIF: Cognitive Level: Knowledge OBJ: Nursing Process: Assessment MSC: Client Needs: Physiologic Integrity 23. The nurse practicing in a labor setting knows that the woman most at risk for uterine rupture is: a. a gravida 3 who has had two low-segment transverse cesarean births. b. a gravida 2 who had a low-segment vertical incision for delivery of a 10-lb infant. c. a gravida 5 who had two vaginal births and two cesarean births. d. a gravida 4 who has had all cesarean births. ANS: D The risk of uterine rupture increases for the patient who has had multiple prior births with no vaginal births. As the number of prior uterine incisions increases, so does the risk for uterine rupture. Low-segment transverse cesarean scars do not predispose the patient to uterine rupture. PTS: 1 DIF: Cognitive Level: Comprehension OBJ: Nursing Process: Assessment MSC: Client Needs: Physiologic Integrity 24. Before the physician performs an external version, the nurse should expect an order for a: a. tocolytic drug. b. contraction stress test (CST). c. local anesthetic. d. Foley catheter. ANS: A A tocolytic drug will relax the uterus before and during version, thus making manipulation easier. CST is used to determine the fetal response to stress. A local anesthetic is not used with external version. The bladder should be emptied; however, catheterization is not necessary. PTS: 1 DIF: Cognitive Level: Comprehension OBJ: Nursing Process: Planning MSC: Client Needs: Physiologic Integrity 25. A maternal indication for the use of forceps is: a. a wide pelvic outlet. b. maternal exhaustion. c. a history of rapid deliveries. NURSINGTB.COM d. failure to progress past 0 station. ANS: B A mother who is exhausted may be unable to assist with the expulsion of the fetus. The patient with a wide pelvic outlet will likely not require vacuum extraction. With a rapid delivery, vacuum extraction is not necessary. A station of 0 is too high for a vacuum extraction. PTS: 1 DIF: Cognitive Level: Knowledge OBJ: Nursing Process: Assessment MSC: Client Needs: Physiologic Integrity 26. The priority nursing intervention after an amniotomy should be to: a. assess the color of the amniotic fluid. b. change the patient's gown. c. estimate the amount of amniotic fluid. d. assess the fetal heart rate. ANS: D The fetal heart rate must be assessed immediately after the rupture of the membranes to determine whether cord prolapse or compression has occurred. Secondary to FHR assessment, amniotic fluid amount, color, odor, and consistency is assessed. Dry clothing is important for patient comfort; however, it is not the top priority. PTS: 1 DIF: Cognitive Level: Application OBJ: Nursing Process: Implementation MSC: Client Needs: Physiologic Integrity 27. The priority nursing care associated with an oxytocin (Pitocin) infusion is: a. measuring urinary output. b. increasing infusion rate every 30 minutes. c. monitoring uterine response. d. evaluating cervical dilation. ANS: C Because of the risk of hyperstimulation, which could result in decreased placental perfusion and uterine rupture, the nurse's priority intervention is monitoring uterine response. Monitoring urinary output is also important; however, it is not the top priority during the administration of Pitocin. The infusion rate may be increased after proper assessment that it is an appropriate interval to do so. Monitoring labor progression is the standard of care for all labor patients. PTS: 1 DIF: Cognitive Level: Comprehension OBJ: Nursing Process: Implementation MSC: Client Needs: Physiologic Integrity 28. Immediately after the forceps-assisted birth of an infant, the nurse should: a. assess the infant for signs of trauma. b. give the infant prophylactic antibiotics. c. apply a cold pack to the infant's scalp. d. measure the circumference of the infant's head. ANS: A The infant should be assessed for NbrUuRisSiInNgGoTrBa.CbOraMsions at the site of application, facial palsy, and subdural hematoma. Prophylactic antibiotics are not necessary with a forceps delivery. A cold pack would put the infant at risk for cold stress and is contraindicated. Measuring the circumference of the head is part of the initial nursing assessment. PTS: 1 DIF: Cognitive Level: Application OBJ: Nursing Process: Implementation MSC: Client Needs: Physiologic Integrity 29. Surgical, medical, or mechanical methods may be used for labor induction. Which technique is considered a mechanical method of induction? a. Amniotomy b. Intravenous Pitocin c. Transcervical catheter d. Vaginal insertion of prostaglandins ANS: C Placement of a balloon-tipped Foley catheter into the cervix is a mechanical method of induction. Other methods to expand and gradually dilate the cervix include hydroscopic dilators such as laminaria tents (made from desiccated seaweed), or Lamicel (contains magnesium sulfate). Amniotomy is a surgical method of augmentation and induction. Intravenous Pitocin and insertion of prostaglandins are medical methods of induction. PTS: 1 DIF: Cognitive Level: Application OBJ: Nursing Process: Implementation MSC: Client Needs: Physiologic Integrity MULTIPLE RESPONSE 1. Complications and risks associated with cesarean births include: (Select all that apply.) a. placental abruption. b. wound dehiscence. c. hemorrhage. d. urinary tract infections. e. fetal injuries. ANS: B, C, D, E Placental abruption and placenta previa are both indications for cesarean birth and are not complications thereof. Wound dehiscence, hemorrhage, urinary tract infection, and fetal injuries are all possible complications and risks associated with delivery by cesarean section. PTS: 1 DIF: Cognitive Level: Comprehension OBJ: Nursing Process: Evaluation MSC: Client Needs: Physiologic Integrity 2. Induction of labor is considered an acceptable obstetric procedure if it is in the best interest to deliver the fetus. The charge nurse in the labor and delivery unit is often asked to schedule patients for this procedure and therefore must be cognizant of the specific conditions appropriate for labor induction. These include: (Select all that apply.) a. rupture of membranes at or near term. b. convenience of the woman or her physician. c. chorioamnionitis (inflammation of the amniotic sac). d. postterm pregnancy. e. fetal death. ANS: A, C, D, E NURSINGTB.COM These are all acceptable indications for induction. Other conditions include intrauterine growth retardation (IUGR), maternal-fetal blood incompatibility, hypertension, and placental abruption. Elective inductions for the convenience of the woman or her provider are not recommended; however, they have become commonplace. Factors such as rapid labors and living a long distance from a health care facility may be valid reasons in such a circumstance. Elective delivery should not occur before 39 weeks' completed gestation. PTS: 1 DIF: Cognitive Level: Application OBJ: Nursing Process: Planning MSC: Client Needs: Physiologic Integrity Chapter 18: Postpartum Physiologic Changes Perry: Maternal Child Nursing Care, 6th Edition MULTIPLE CHOICE 1. A woman gave birth to an infant boy 10 hours ago. Where would the nurse expect to locate this woman's fundus? a. One centimeter above the umbilicus b. Two centimeters below the umbilicus c. Midway between the umbilicus and the symphysis pubis d. Nonpalpable abdominally ANS: A Within 12 hours after delivery the fundus may be approximately 1 cm above the umbilicus. The fundus descends about 1 to 2 cm every 24 hours. Within 12 hours after delivery the fundus may be approximately 1 cm above the umbilicus. By the sixth after birth week the fundus normally is halfway between the symphysis pubis and the umbilicus. The fundus should be easily palpated using the maternal umbilicus as a reference point. PTS: 1 DIF: Cognitive Level: Comprehension OBJ: Nursing Process: Assessment MSC: Client Needs: Health Promotion and Maintenance 2. Which woman is most likely to experience strong afterpains? a. A woman who experienced oligohydramnios b. A woman who is a gravida 4, para 4-0-0-4 c. A woman who is bottle-feedinNgUhReSrIiNnGfaTnBt.COM d. A woman whose infant weighed 5 lbs, 3 ounces ANS: B Afterpains are more common in multiparous women. Afterpains are more noticeable with births in which the uterus was greatly distended, as in a woman who experienced polyhydramnios or a woman who delivered a large infant. Breastfeeding may cause afterpains to intensify. PTS: 1 DIF: Cognitive Level: Comprehension OBJ: Nursing Process: Assessment MSC: Client Needs: Health Promotion and Maintenance 3. A woman gave birth to a healthy infant boy 5 days ago. What type of lochia would the nurse expect to find when assessing this woman? a. Lochia rubra b. Lochia sangra c. Lochia alba d. Lochia serosa ANS: D Lochia serosa, which consists of blood, serum, leukocytes, and tissue debris, generally occurs around day 3 or 4 after childbirth. Lochia rubra consists of blood and decidual and trophoblastic debris. The flow generally lasts 3 to 4 days and pales, becoming pink or brown. There is no such term as lochia sangra. Lochia alba occurs in most women after day 10 and can continue up to 6 weeks after childbirth. PTS: 1 DIF: Cognitive Level: Comprehension OBJ: Nursing Process: Assessment MSC: Client Needs: Health Promotion and Maintenance 4. Which hormone remains elevated in the immediate after birth period of the breastfeeding woman? a. Estrogen b. Progesterone c. Prolactin d. Human placental lactogen ANS: C Prolactin levels in the blood increase progressively throughout pregnancy. In women who breastfeed, prolactin levels remain elevated into the sixth week after birth. Estrogen and progesterone levels decrease markedly after expulsion of the placenta and reach their lowest levels 1 week into the after birth period. Human placental lactogen levels decrease dramatically after expulsion of the placenta. PTS: 1 DIF: Cognitive Level: Comprehension OBJ: Nursing Process: Assessment MSC: Client Needs: Health Promotion and Maintenance 5. Two days ago a woman gave birth to a full-term infant. Last night she awakened several times to urinate and noted that her gown and bedding were wet from profuse diaphoresis. One mechanism for the diaphoresis and diuresis that this woman is experiencing during the early after birth period is: a. elevated temperature caused by after birth infection. b. increased basal metabolic rate after giving birth. c. loss of increased blood volumNe UasRsSoIcNiGatTeBd.CwOitMh pregnancy. d. increased venous pressure in the lower extremities. ANS: C Within 12 hours of birth women begin to lose the excess tissue fluid that has accumulated during pregnancy. One mechanism for reducing these retained fluids is the profuse diaphoresis that often occurs, especially at night, for the first 2 or 3 days after childbirth. Postpartal diuresis is another mechanism by which the body rids itself of excess fluid. An elevated temperature would cause chills and may cause dehydration, not diaphoresis and diuresis. Diaphoresis and diuresis sometimes are referred to as reversal of the water metabolism of pregnancy, not as the basal metabolic rate. Postpartal diuresis may be caused by the removal of increased venous pressure in the lower extremities. PTS: 1 DIF: Cognitive Level: Comprehension OBJ: Nursing Process: Diagnosis MSC: Client Needs: Physiologic Integrity 6. A woman gave birth to a 7-lb, 3-ounce infant boy 2 hours ago. The nurse determines that the woman's bladder is distended because her fundus is now 3 cm above the umbilicus and to the right of the midline. In the immediate after birth period, the most serious consequence likely to occur from bladder distention is: a. urinary tract infection. b. excessive uterine bleeding. c. a ruptured bladder. d. bladder wall atony. ANS: B Excessive bleeding can occur immediately after birth if the bladder becomes distended because it pushes the uterus up and to the side and prevents it from contracting firmly. A urinary tract infection may result from overdistention of the bladder, but it is not the most serious consequence. A ruptured bladder may result from a severely overdistended bladder. However, vaginal bleeding most likely would occur before the bladder reaches this level of overdistention. Bladder distention may result from bladder wall atony. The most serious concern associated with bladder distention is excessive uterine bleeding. PTS: 1 DIF: Cognitive Level: Comprehension OBJ: Nursing Process: Diagnosis, Planning MSC: Client Needs: Health Promotion and Maintenance 7. The nurse caring for the after birth woman understands that breast engorgement is caused by: a. overproduction of colostrum. b. accumulation of milk in the lactiferous ducts. c. hyperplasia of mammary tissue. d. congestion of veins and lymphatics. ANS: D Breast engorgement is caused by the temporary congestion of veins and lymphatics, not by overproduction of colostrum, overproduction of milk, or hyperplasia of mammary tissue. PTS: 1 DIF: Cognitive Level: Knowledge OBJ: Nursing Process: Assessment MSC: Client Needs: Health Promotion and Maintenance 8. A woman gave birth to a 7-lb, 6-oNuUncReSIiNnfGaTnBt .gCiOrlM1 hour ago. The birth was vaginal, and the estimated blood loss (EBL) was approximately 1500 mL. When assessing the woman's vital signs, the nurse would be concerned to see: a. temperature 37.9° C, heart rate 120, respirations 20, blood pressure (BP) 90/50. b. temperature 37.4° C, heart rate 88, respirations 36, BP 126/68. c. temperature 38° C, heart rate 80, respirations 16, BP 110/80. d. temperature 36.8° C, heart rate 60, respirations 18, BP 140/90. ANS: A An EBL of 1500 mL with tachycardia and hypotension suggests hypovolemia caused by excessive blood loss. An increased respiratory rate of 36 may be secondary to pain from the birth. Temperature may increase to 38° C during the first 24 hours as a result of the dehydrating effects of labor. A BP of 140/90 is slightly elevated, which may be caused by the use of oxytocic medications. PTS: 1 DIF: Cognitive Level: Comprehension OBJ: Nursing Process: Assessment, Diagnosis MSC: Client Needs: Physiologic Integrity 9. Which statement by a newly delivered woman indicates that she knows what to expect about her menstrual activity after childbirth? a. “My first menstrual cycle will be lighter than normal and then will get heavier every month thereafter.” b. “My first menstrual cycle will be heavier than normal and will return to my prepregnant volume within three or four cycles.” c. “I will not have a menstrual cycle for 6 months after childbirth.” d. “My first menstrual cycle will be heavier than normal and then will be light for several months after.” ANS: B “My first menstrual cycle will be heavier than normal and will return to my prepregnant volume within three or four cycles” is an accurate statement and indicates her understanding of her expected menstrual activity. She can expect her first menstrual cycle to be heavier than normal (which occurs by 3 months after childbirth), and the volume of her subsequent cycles will return to prepregnant levels within three or four cycles. PTS: 1 DIF: Cognitive Level: Application OBJ: Nursing Process: Evaluation MSC: Client Needs: Health Promotion and Maintenance 10. The interval between the birth of the newborn and the return of the reproductive organs to their normal nonpregnant state is called the: a. involutionary period because of what happens to the uterus. b. lochia period because of the nature of the vaginal discharge. c. mini-tri period because it lasts only 3 to 6 weeks. d. puerperium, or fourth trimester of pregnancy. ANS: D The puerperium, also called the fourth trimester or the after birth period of pregnancy, lasts about 3 to 6 weeks. Involution marks the end of the puerperium, or the fourth trimester of pregnancy. Lochia refers to the various vaginal discharges during the puerperium, or fourth trimester of pregnancy. PTS: 1 DIF: CognitiNveURLSevINelG: TKBn.oCwOlMedge OBJ: Nursing Process: Assessment MSC: Client Needs: Health Promotion and Maintenance 11. The self-destruction of excess hypertrophied tissue in the uterus is called: a. autolysis. b. subinvolution. c. afterpain. d. diastasis. ANS: A Autolysis is caused by a decrease in hormone levels. Subinvolution is failure of the uterus to return to a nonpregnant state. Afterpain is caused by uterine cramps 2 to 3 days after birth. Diastasis refers to the separation of muscles. PTS: 1 DIF: Cognitive Level: Knowledge OBJ: Nursing Process: Assessment MSC: Client Needs: Physiologic Integrity 12. With regard to the after birth uterus, nurses should be aware that: a. at the end of the third stage of labor it weighs approximately 500 g. b. after 2 weeks after birth it should not be palpable abdominally. c. after 2 weeks after birth it weighs 100 g. d. it returns to its original (prepregnancy) size by 6 weeks after birth. ANS: B After 2 weeks after birth, the uterus should not be palpable abdominally; however, it has not yet returned to its original size. At the end of the third stage of labor, the uterus weighs approximately 1000 g. It takes 6 full weeks for the uterus to return to its original size. After 2 weeks after birth the uterus weighs about 350 g, not its original size. The normal self-destruction of excess hypertrophied tissue accounts for the slight increase in uterine size after each pregnancy. PTS: 1 DIF: Cognitive Level: Comprehension OBJ: Nursing Process: Assessment MSC: Client Needs: Health Promotion and Maintenance 13. With regard to after birth pains, nurses should be aware that these pains are: a. caused by mild, continuous contractions for the duration of the after birth period. b. more common in first-time mothers. c. more noticeable in births in which the uterus was overdistended. d. alleviated somewhat when the mother breastfeeds. ANS: C A large baby or multiple babies overdistend the uterus. The cramping that causes after birth pains arises from periodic, vigorous contractions and relaxations, which persist through the first part of the after birth period. After birth pains are more common in multiparous women because first-time mothers have better uterine tone. Breastfeeding intensifies after birth pain because it stimulates contractions. PTS: 1 DIF: Cognitive Level: Comprehension OBJ: Nursing Process: Assessment MSC: Client Needs: Health Promotion and Maintenance 14. Post birth uterine/vaginal discharge, called lochia: a. is similar to a light menstrual pNeUrRioSdINfGorTtBh.Ce OfiMrst 6 to 12 hours. b. is usually greater after cesarean births. c. will usually decrease with ambulation and breastfeeding. d. should smell like normal menstrual flow unless an infection is present. ANS: D An offensive odor usually indicates an infection. Lochia flow should approximate a heavy menstrual period for the first 2 hours and then steadily decrease. Less lochia usually is seen after cesarean births and usually increases with ambulation and breastfeeding. PTS: 1 DIF: Cognitive Level: Comprehension OBJ: Nursing Process: Assessment MSC: Client Needs: Health Promotion and Maintenance 15. With regard to after birth ovarian function, nurses should be aware that: a. almost 75% of women who do not breastfeed resume menstruating within a month after birth. b. ovulation occurs slightly earlier for breastfeeding women. c. because of menstruation/ovulation schedules, contraception considerations can be postponed until after the puerperium. d. the first menstrual flow after childbirth usually is heavier than normal. ANS: D The first flow is heavier, but within three or four cycles, it is back to normal. Ovulation can occur within the first month, but for 70% of nonlactating women, it returns within 12 weeks after birth. Breastfeeding women take longer to resume ovulation. Because many women ovulate before their first after birth menstrual period, contraceptive options need to be discussed early in the puerperium. PTS: 1 DIF: Cognitive Level: Comprehension OBJ: Nursing Process: Planning MSC: Client Needs: Physiologic Integrity 16. As relates to the condition and reconditioning of the urinary system after childbirth, nurses should be aware that: a. kidney function returns to normal a few days after birth. b. diastasis recti abdominis is a common condition that alters the voiding reflex. c. fluid loss through perspiration and increased urinary output accounts for a weight loss of more than 2 kg during the puerperium. d. with adequate emptying of the bladder, bladder tone usually is restored 2 to 3 weeks after childbirth. ANS: C Excess fluid loss through other means occurs as well. Kidney function usually returns to normal in about a month. Diastasis recti abdominis is the separation of muscles in the abdominal wall; it has no effect on the voiding reflex. Bladder tone usually is restored 5 to 7 days after childbirth. PTS: 1 DIF: Cognitive Level: Comprehension OBJ: Nursing Process: Planning MSC: Client Needs: Physiologic Integrity 17. Knowing that the condition of theNnUeRwSImNoGtThBer.C'sObMreasts will be affected by whether she is breastfeeding, nurses should be able to tell their patients all the following statements except: a. breast tenderness is likely to persist for about a week after the start of lactation. b. as lactation is established, a mass may form that can be distinguished from cancer by its position shift from day to day. c. in nonlactating mothers colostrum is present for the first few days after childbirth. d. if suckling is never begun (or is discontinued), lactation ceases within a few days to a week. ANS: A Breast tenderness should persist for 24 to 48 hours after lactation begins. That movable, noncancerous mass is a filled milk sac. Colostrum is present for a few days whether the mother breastfeeds or not. A mother who does not want to breastfeed should also avoid stimulating her nipples. PTS: 1 DIF: Cognitive Level: Comprehension OBJ: Nursing Process: Planning, Implementation MSC: Client Needs: Health Promotion and Maintenance 18. With regard to the after birth changes and developments in a woman's cardiovascular system, nurses should be aware that: a. cardiac output, the pulse rate, and stroke volume all return to prepregnancy normal values within a few hours of childbirth. b. respiratory function returns to nonpregnant levels by 6 to 8 weeks after birth. c. the lowered white blood cell count after pregnancy can lead to false-positive results on tests for infections. d. a hypercoagulable state protects the new mother from thromboembolism, especially after a cesarean birth. ANS: B Respirations should decrease to within the woman's normal prepregnancy range by 6 to 8 weeks after birth. Stroke volume increases, and cardiac output remains high for a couple of days. However, the heart rate and blood pressure return to normal quickly. Leukocytosis increases 10 to 12 days after childbirth and can obscure the diagnosis of acute infections (false-negative results). The hypercoagulable state increases the risk of thromboembolism, especially after a cesarean birth. PTS: 1 DIF: Cognitive Level: Comprehension OBJ: Nursing Process: Planning MSC: Client Needs: Physiologic Integrity 19. Which condition, not uncommon in pregnancy, is likely to require careful medical assessment during the puerperium? a. Varicosities of the legs b. Carpal tunnel syndrome c. Periodic numbness and tingling of the fingers d. Headaches ANS: D Headaches in the after birth period can have a number of causes, some of which deserve medical attention. Total or nearly total regression of varicosities is expected after childbirth. Carpal tunnel syndrome is relieved in childbirth when the compression on the median nerve is lessened. Periodic numbness of thNeUfiRnSgIeNrGs TuBsu.CaOllMy disappears after birth unless carrying the baby aggravates the condition. PTS: 1 DIF: Cognitive Level: Comprehension OBJ: Nursing Process: Evaluation MSC: Client Needs: Physiologic Integrity 20. Several changes in the integumentary system that appear during pregnancy disappear after birth, although not always completely. What change is almost certain to be completely reversed? a. Nail brittleness b. Darker pigmentation of the areolae and linea nigra c. Striae gravidarum on the breasts, abdomen, and thighs d. Spider nevi ANS: A The nails return to their prepregnancy consistency and strength. Some women have permanent darker pigmentation of the areolae and linea nigra. Striae gravidarum (stretch marks) usually do not completely disappear. For some women spider nevi persist indefinitely. PTS: 1 DIF: Cognitive Level: Comprehension OBJ: Nursing Process: Planning MSC: Client Needs: Physiologic Integrity 21. Childbirth may result in injuries to the vagina and uterus. Pelvic floor exercises also known as Kegel exercises will help to strengthen the perineal muscles and encourage healing. The nurse knows that the patient understands the correct process for completing these conditioning exercises when she reports: a. “I contract my thighs, buttocks, and abdomen.” b. “I do 10 of these exercises every day.” c. “I stand while practicing this new exercise routine.” d. “I pretend that I am trying to stop the flow of urine midstream.” ANS: D The woman can pretend that she is attempting to stop the passing of gas or the flow of urine midstream. This will replicate the sensation of the muscles drawing upward and inward. Each contraction should be as intense as possible without contracting the abdomen, buttocks, or thighs. Guidelines suggest that these exercises should be done 24 to 100 times per day. Positive results are shown with a minimum of 24 to 45 repetitions per day. The best position to learn Kegel exercises is to lie supine with knees bent. A secondary position is on the hands and knees. PTS: 1 DIF: Cognitive Level: Analysis OBJ: Nursing Process: Evaluation MSC: Client Needs: Health Promotion and Maintenance 22. Which maternal event is abnormal in the early after birth period? a. Diuresis and diaphoresis b. Flatulence and constipation c. Extreme hunger and thirst d. Lochial color changes from rubra to alba ANS: D NURSINGTB.COM For the first 3 days after childbirth, lochia is termed rubra. Lochia serosa follows, and then at about 11 days, the discharge becomes clear, colorless, or white. Diuresis and diaphoresis are the methods by which the body rids itself of increased plasma volume. Urine output of 3000 mL/day is common for the first few days after delivery and is facilitated by hormonal changes in the mother. Bowel tone remains sluggish for days. Many women anticipate pain during defecation and are unwilling to exert pressure on the perineum. The new mother is hungry because of energy used in labor and thirsty because of fluid restrictions during labor. PTS: 1 DIF: Cognitive Level: Knowledge OBJ: Nursing Process: Assessment MSC: Client Needs: Health Promotion and Maintenance 23. Which finding 12 hours after birth requires further assessment? a. The fundus is palpable two fingerbreadths above the umbilicus. b. The fundus is palpable at the level of the umbilicus. c. The fundus is palpable one fingerbreadth below the umbilicus. d. The fundus is palpable two fingerbreadths below the umbilicus. ANS: A The fundus rises to the umbilicus after delivery and remains there for about 24 hours. A fundus that is above the umbilicus may indicate uterine atony or urinary retention. A fundus that is palpable at or below the level of the umbilicus is a normal finding for a patient who is 12 hours after birth. Palpation of the fundus 2 fingerbreadths below the umbilicus is an unusual finding for 12 hours after birth; however, it is still appropriate. PTS: 1 DIF: Cognitive Level: Application OBJ: Nursing Process: Assessment MSC: Client Needs: Physiologic Integrity 24. If the patient's white blood cell (WBC) count is 25,000/mm on her second after birth day, the nurse should: a. tell the physician immediately. b. have the laboratory draw blood for reanalysis. c. recognize that this is an acceptable range at this point after birth. d. begin antibiotic therapy immediately. ANS: C During the first 10 to 12 days after childbirth, values between 20,000 and 25,000/mm are common. Because this is a normal finding there is no reason to alert the physician. There is no need for reassessment or antibiotics because it is expected for the WBCs to be elevated. PTS: 1 DIF: Cognitive Level: Knowledge OBJ: Nursing Process: Implementation MSC: Client Needs: Health Promotion and Maintenance 25. Which documentation on a woman's chart on after birth day 14 indicates a normal involution process? a. Moderate bright red lochial flow b. Breasts firm and tender c. Fundus below the symphysis and not palpable d. Episiotomy slightly red and puffy ANS: C The fundus descends 1 cm/day, soNbUyRSafINteGr TbBir.tChOdMay 14 it is no longer palpable. The lochia should be changed by this day to serosa. Breasts are not part of the involution process. The episiotomy should not be red or puffy at this stage. PTS: 1 DIF: Cognitive Level: Knowledge OBJ: Nursing Process: Assessment MSC: Client Needs: Physiologic Integrity MULTIPLE RESPONSE 1. Changes in blood volume after childbirth depend on several factors such as blood loss during childbirth and the amount of extravascular water (physiologic edema) mobilized and excreted. A after birth nurse anticipates blood loss of: (Select all that apply.) a. 100 mL. b. 250 mL or less. c. 300 to 500 mL. d. 500 to 1000 mL. e. 1500 mL or greater. ANS: C, D The average blood loss for a vaginal birth of a single fetus ranges from 300 to 500 mL (10% of blood volume). The typical blood loss for women who gave birth by cesarean is 500 to 1000 mL (15% to 30% of blood volume). During the first few days after birth the plasma volume decreases further as a result diuresis. Pregnancy-induced hypervolemia (an increase in blood volume of at least 35%) allows most women to tolerate considerable blood loss during childbirth. PTS: 1 DIF: Cognitive Level: Comprehension OBJ: Nursing Process: Assessment, Planning MSC: Client Needs: Physiologic Integrity MATCHING The physiologic changes that occur during the reversal of the processes of pregnancy are distinctive; however, they are normal. To provide care during this recovery period the nurse must synthesize knowledge regarding anticipated maternal changes and deviations from normal. Please match the vital signs finding that the after birth nurse may encounter with the probable cause: a. Elevated temperature within the first 24 hours b. Rapid pulse c. Elevated temperature at 36 hours after birth d. Hypertension e. Hypoventilation 1. Puerperal sepsis 2. Unusually high epidural or spinal block 3. Dehydrating effects of labor NURSINGTB.COM 4. Hypovolemia resulting from hemorrhage 5. Excessive use of oxytocin 1. ANS: C PTS: 1 DIF: Cognitive Level: Comprehension OBJ: Nursing Process: Assessment MSC: Client Needs: Physiologic Integrity NOT: During the first 24 hours after birth, temperature may increase to 38° C as a result of the dehydrating effects of labor. After 24 hours the woman should be afebrile. Other causes of fever include mastitis, endometritis, urinary tract infection, and other systemic infections. Pulse, along with stroke volume and cardiac output, remains elevated for the first hour or so after childbirth. A rapid pulse, or one that is increasing, may indicate hypovolemia as a result of hemorrhage. Hypoventilation may occur after an unusually high subarachnoid block or epidural narcotic after a cesarean birth. An increased reading in blood pressure may result from the excessive use of the vasopressor or oxytocic medication. Because gestational hypertension can persist into or occur first in the after birth period, routine evaluation of blood pressure is necessary. 2. ANS: E PTS: 1 DIF: Cognitive Level: Comprehension OBJ: Nursing Process: Assessment MSC: Client Needs: Physiologic Integrity NOT: During the first 24 hours after birth, temperature may increase to 38° C as a result of the dehydrating effects of labor. After 24 hours the woman should be afebrile. Other causes of fever include mastitis, endometritis, urinary tract infection, and other systemic infections. Pulse, along with stroke volume and cardiac output, remains elevated for the first hour or so after childbirth. A rapid pulse, or one that is increasing, may indicate hypovolemia as a result of hemorrhage. Hypoventilation may occur after an unusually high subarachnoid block or epidural narcotic after a cesarean birth. An increased reading in blood pressure may result from the excessive use of the vasopressor or oxytocic medication. Because gestational hypertension can persist into or occur first in the after birth period, routine evaluation of blood pressure is necessary. 3. ANS: A PTS: 1 DIF: Cognitive Level: Comprehension OBJ: Nursing Process: Assessment MSC: Client Needs: Physiologic Integrity NOT: During the first 24 hours after birth, temperature may increase to 38° C as a result of the dehydrating effects of labor. After 24 hours the woman should be afebrile. Other causes of fever include mastitis, endometritis, urinary tract infection, and other systemic infections. Pulse, along with stroke volume and cardiac output, remains elevated for the first hour or so after childbirth. A rapid pulse, or one that is increasing, may indicate hypovolemia as a result of hemorrhage. Hypoventilation may occur after an unusually high subarachnoid block or epidural narcotic after a cesarean birth. An increased reading in blood pressure may result from the excessive use of the vasopressor or oxytocic medication. Because gestational hypertension can persist into or occur first in the after birth period, routine evaluation of blood pressure is necessary. 4. ANS: B PTS: 1 DIF: Cognitive Level: Comprehension OBJ: Nursing Process: Assessment MSC: Client Needs: Physiologic Integrity NOT: During the first 24 hours after birth, temperature may increase to 38° C as a result of the dehydrating effects of labor. After 24 hours the woman should be afebrile. Other causes of fever include mastitis, endometritis, urinary tract infection, and other systemic infections. Pulse, along with stroke volume and cardiac output, remains elevated for the first hour or so after childbirth. A rapid pulse, or one that is increasing, may indicate hypovolemia as a result of hemorrhage. Hypoventilation may occur after an unusually high subarachnoid block or epidural narcotic after a cesarean birth. An increased reading in blood pressure may result from the excessive use of the vasopressor or oxytocic medication. Because gestational hypertension can persist into or occur first in the after birth period, routine evaluation of blood pressure is necessary. 5. ANS: D PTS: 1 DIF: Cognitive Level: Comprehension OBJ: Nursing Process: Assessment MSC: Client Needs: Physiologic Integrity NOT: During the first 24 hours after birth, temperature may increase to 38° C as a result of the dehydrating effects of labor. After 24 hours the woman should be afebrile. Other causes of fever include mastitis, endometritis, urinary tract infection, and other systemic infections. Pulse, along with stroke volume and cardiac output, remNUaiRnSs IeNleGvTaBte.dCOfoMr the first hour or so after childbirth. A rapid pulse, or one that is increasing, may indicate hypovolemia as a result of hemorrhage. Hypoventilation may occur after an unusually high subarachnoid block or epidural narcotic after a cesarean birth. An increased reading in blood pressure may result from the excessive use of the vasopressor or oxytocic medication. Because gestational hypertension can persist into or occur first in the after birth period, routine evaluation of blood pressure is necessary. Chapter 19: Nursing Care of the Family During the Postpartum Period Perry: Maternal Child Nursing Care, 6th Edition MULTIPLE CHOICE 1. A 25-year-old gravida 2, para 2-0-0-2 gave birth 4 hours ago to a 9-lb, 7-ounce boy after augmentation of labor with Pitocin. She puts on her call light and asks for her nurse right away, stating, “I'm bleeding a lot.” The most likely cause of after birth hemorrhage in this woman is: a. retained placental fragments. b. unrepaired vaginal lacerations. c. uterine atony. d. puerperal infection. ANS: C This woman gave birth to a macrosomic boy after Pitocin augmentation. The most likely cause of bleeding 4 hours after delivery, combined with these risk factors, is uterine atony. Although retained placental fragments may cause after birth hemorrhage, this typically would be detected in the first hour after delivery of the placenta and is not the most likely cause of hemorrhage in this woman. Although unrepaired vaginal lacerations may cause bleeding, they typically would occur in the period immediately after birth. Puerperal infection can cause subinvolution and subsequent bleeding; however, this typically would be detected 24 hours after delivery. PTS: 1 DIF: Cognitive Level: Analysis OBJ: Nursing Process: AssessmentNURSINMGSTCB.:CCOlMient Needs: Health Promotion and Maintenance 2. On examining a woman who gave birth 5 hours ago, the nurse finds that the woman has completely saturated a perineal pad within 15 minutes. The nurse's first action is to: a. begin an intravenous (IV) infusion of Ringer's lactate solution. b. assess the woman's vital signs. c. call the woman's primary health care provider. d. massage the woman's fundus. ANS: D The nurse should assess the uterus for atony. Uterine tone must be established to prevent excessive blood loss. The nurse may begin an IV infusion to restore circulatory volume, but this would not be the first action. Blood pressure is not a reliable indicator of impending shock from impending hemorrhage; assessing vital signs should not be the nurse's first action. The physician would be notified after the nurse completes the assessment of the woman. PTS: 1 DIF: Cognitive Level: Application OBJ: Nursing Process: Implementation MSC: Client Needs: Health Promotion and Maintenance 3. A woman gave birth vaginally to a 9-lb, 12-ounce girl yesterday. Her primary health care provider has written orders for perineal ice packs, use of a sitz bath tid, and a stool softener. What information is most closely correlated with these orders? a. The woman is a gravida 2, para 2. b. The woman had a vacuum-assisted birth. c. The woman received epidural anesthesia. d. The woman has an episiotomy. ANS: D These orders are typical interventions for a woman who has had an episiotomy, lacerations, and hemorrhoids. A multiparous classification is not an indication for these orders. A vacuum-assisted birth may be used in conjunction with an episiotomy, which would indicate these interventions. Use of epidural anesthesia has no correlation with these orders. PTS: 1 DIF: Cognitive Level: Comprehension OBJ: Nursing Process: Planning MSC: Client Needs: Health Promotion and Maintenance 4. The laboratory results for a after birth woman are as follows: blood type, A; Rh status, positive; rubella titer, 1:8 (EIA 0.8); hematocrit, 30%. How would the nurse best interpret these data? a. Rubella vaccine should be given. b. A blood transfusion is necessary. c. Rh immune globulin is necessary within 72 hours of birth. d. A Kleihauer-Betke test should be performed. ANS: A This patient's rubella titer indicates that she is not immune and that she needs to receive a vaccine. These data do not indicate that the patient needs a blood transfusion. Rh immune globulin is indicated only if the patient has a negative Rh status and the infant has a positive Rh status. A Kleihauer-Betke test should be performed if a large fetomaternal transfusion is suspected, especially if the mother is Rh negative. The data do not provide any indication for performing this test. PTS: 1 DIF: CognitiNveURLSevINelG: TCBo.mCOprMehension OBJ: Nursing Process: Planning MSC: Client Needs: Health Promotion and Maintenance 5. A woman gave birth 48 hours ago to a healthy infant girl. She has decided to bottle-feed. During your assessment you notice that both of her breasts are swollen, warm, and tender on palpation. The woman should be advised that this condition can best be treated by: a. running warm water on her breasts during a shower. b. applying ice to the breasts for comfort. c. expressing small amounts of milk from the breasts to relieve pressure. d. wearing a loose-fitting bra to prevent nipple irritation. ANS: B Applying ice to the breasts for comfort is appropriate for treating engorgement in a mother who is bottle-feeding. This woman is experiencing engorgement, which can be treated by using ice packs (because she is not breastfeeding) and cabbage leaves. A bottle-feeding mother should avoid any breast stimulation, including pumping or expressing milk. A bottle-feeding mother should wear a well-fitted support bra or breast binder continuously for at least the first 72 hours after giving birth. A loose-fitting bra will not aid lactation suppression. Furthermore, the shifting of the bra against the breasts may stimulate the nipples and thereby stimulate lactation. PTS: 1 DIF: Cognitive Level: Application OBJ: Nursing Process: Implementation MSC: Client Needs: Health Promotion and Maintenance 6. A 25-year-old multiparous woman gave birth to an infant boy 1 day ago. Today her husband brings a large container of brown seaweed soup to the hospital. When the nurse enters the room, the husband asks for help with warming the soup so that his wife can eat it. The nurse's most appropriate response is to ask the woman: a. “Didn't you like your lunch?” b. “Does your doctor know that you are planning to eat that?” c. “What is that anyway?” d. “I'll warm the soup in the microwave for you.” ANS: D “I'll warm the soup in the microwave for you” shows cultural sensitivity to the dietary preferences of the woman and is the most appropriate response. Cultural dietary preferences must be respected. Women may request that family members bring favorite or culturally appropriate foods to the hospital. “What is that anyway?” does not show cultural sensitivity. PTS: 1 DIF: Cognitive Level: Application OBJ: Nursing Process: Implementation MSC: Client Needs: Psychosocial Integrity 7. In many hospitals new mothers are routinely presented with gift bags containing samples of infant formula. This practice: a. is inconsistent with the Baby-Friendly Hospital Initiative. b. promotes longer periods of breastfeeding. c. is perceived as supportive to both bottle-feeding and breastfeeding mothers. d. is associated with earlier cessation of breastfeeding. ANS: A Infant formula should not be given to mothers who are breastfeeding. Such gifts are associated with earlier cessation of breastfeedNiUnRg.SIBNaGbTyB-F.CrOieMndly USA prohibits the distribution of any gift bags or formula to new mothers. PTS: 1 DIF: Cognitive Level: Comprehension OBJ: Nursing Process: Assessment MSC: Client Needs: Safe and Effective Care Environment 8. A after birth woman overhears the nurse tell the obstetrics clinician that she has a positive Homans' sign and asks what it means. The nurse's best response is: a. “You have pitting edema in your ankles.” b. “You have deep tendon reflexes rated 2+.” c. “You have calf pain when the nurse flexes your foot.” d. “You have a ‘fleshy' odor to your vaginal drainage.” ANS: C Discomfort in the calf with sharp dorsiflexion of the foot may indicate deep vein thrombosis. Edema is within normal limits for the first few days until the excess interstitial fluid is remobilized and excreted. Deep tendon reflexes should be 1+ to 2+. A “fleshy” odor, not a foul odor, is within normal limits. PTS: 1 DIF: Cognitive Level: Knowledge OBJ: Nursing Process: Implementation MSC: Client Needs: Physiologic Integrity 9. In the recovery room, if a woman is asked either to raise her legs (knees extended) off the bed or to flex her knees, place her feet flat on the bed, and raise her buttocks well off the bed, most likely she is being tested to see whether she: a. has recovered from epidural or spinal anesthesia. b. has hidden bleeding underneath her. c. has regained some flexibility. d. is a candidate to go home after 6 hours. ANS: A If the numb or prickly sensations are gone from her legs after these movements, she has likely recovered from the epidural or spinal anesthesia. PTS: 1 DIF: Cognitive Level: Comprehension OBJ: Nursing Process: Evaluation MSC: Client Needs: Physiologic Integrity 10. Under the Newborns' and Mothers' Health Protection Act, all health plans are required to allow new mothers and newborns to remain in the hospital for a minimum of hours after a normal vaginal birth and for hours after a cesarean birth. a. 24, 73 b. 24, 96 c. 48, 96 d. 48, 120 ANS: C The specified stays are 48 hours (2 days) for a vaginal birth and 96 hours (4 days) for a cesarean birth. The attending provider and the mother together can decide on an earlier discharge. NURSINGTB.COM PTS: 1 DIF: Cognitive Level: Knowledge OBJ: Nursing Process: Planning MSC: Client Needs: Safe and Effective Care Environment 11. In a variation of rooming-in, called couplet care, the mother and infant share a room, and the mother shares the care of the infant with: a. the father of the infant. b. her mother (the infant's grandmother). c. her eldest daughter (the infant's sister). d. the nurse. ANS: D In couplet care the mother shares a room with the newborn and shares infant care with a nurse educated in maternity and infant care. PTS: 1 DIF: Cognitive Level: Knowledge OBJ: Nursing Process: Implementation MSC: Client Needs: Health Promotion and Maintenance 12. Nursing care in the fourth trimester includes an important intervention sometimes referred to as taking the time to mother the mother. Specifically this expression refers to: a. formally initializing individualized care by confirming the woman's and infant's identification (ID) numbers on their respective wrist bands. (“This is your baby.”) b. teaching the mother to check the identity of any person who comes to remove the baby from the room. (“It's a dangerous world out there.”) c. including other family members in the teaching of self-care and child care. (“We're all in this together.”) d. nurturing the woman by providing encouragement and support as she takes on the many tasks of motherhood. ANS: D Many professionals believe that the nurse's nurturing and support function is more important than providing physical care and teaching. Matching ID wrist bands is more of a formality, but it is also a get-acquainted procedure. “Mothering the mother” is more a process of encouraging and supporting the woman in her new role. Having the mother check IDs is a security measure for protecting the baby from abduction. Teaching the whole family is just good nursing practice. PTS: 1 DIF: Cognitive Level: Comprehension OBJ: Nursing Process: Implementation MSC: Client Needs: Psychosocial Integrity 13. Excessive blood loss after childbirth can have several causes; the most common is: a. vaginal or vulvar hematomas. b. unrepaired lacerations of the vagina or cervix. c. failure of the uterine muscle to contract firmly. d. retained placental fragments. ANS: C Uterine atony can best be thwarted by maintaining good uterine tone and preventing bladder distention. Although vaginal or vulvar hematomas, unpaired lacerations of the vagina or cervix, and retained placental fragments are possible causes of excessive blood loss, uterine muscle failure (uterine atony) is the most common cause. PTS: 1 DIF: CognitiNveURLSevINelG: TKBn.oCwOlMedge OBJ: Nursing Process: Evaluation MSC: Client Needs: Health Promotion and Maintenance 14. A hospital has a number of different perineal pads available for use. A nurse is observed soaking several of them and writing down what she sees. This activity indicates that the nurse is trying to: a. improve the accuracy of blood loss estimation, which usually is a subjective assessment. b. determine which pad is best. c. demonstrate that other nurses usually underestimate blood loss. d. reveal to the nurse supervisor that one of them needs some time off. ANS: A Saturation of perineal pads is a critical indicator of excessive blood loss, and anything done to aid in assessment is valuable. The nurse is noting the saturation volumes and soaking appearances. It is possible that the nurse is trying to determine which pad is best, but it is more likely that the nurse is noting saturation volumes and soaking appearances to improve the accuracy of blood loss estimation. Nurses usually overestimate blood loss, if anything. PTS: 1 DIF: Cognitive Level: Application OBJ: Nursing Process: Assessment MSC: Client Needs: Physiologic Integrity 15. Because a full bladder prevents the uterus from contracting normally, nurses intervene to help the woman empty her bladder spontaneously as soon as possible. If all else fails, the last thing the nurse could try is: a. pouring water from a squeeze bottle over the woman's perineum. b. placing oil of peppermint in a bedpan under the woman. c. asking the physician to prescribe analgesics. d. inserting a sterile catheter. ANS: D Invasive procedures usually are the last to be tried, especially with so many other simple and easy methods available (e.g., water, peppermint vapors, pain medication). Pouring water over the perineum may stimulate voiding. It is easy, noninvasive, and should be tried early. The oil of peppermint releases vapors that may relax the necessary muscles. If the woman is anticipating pain from voiding, pain medications may be helpful. Other nonmedical means and pain medication should be tried before insertion of a catheter. PTS: 1 DIF: Cognitive Level: Comprehension OBJ: Nursing Process: Implementation MSC: Client Needs: Health Promotion and Maintenance 16. If a woman is at risk for thrombus and is not ready to ambulate, nurses may intervene by performing a number of interventions. Which intervention should the nurse avoid? a. Putting the patient in antiembolic stockings (TED hose) and/or sequential compression device (SCD) boots. b. Having the patient flex, extend, and rotate her feet, ankles, and legs. c. Having the patient sit in a chair. d. Notifying the physician immediately if a positive Homans' sign occurs. ANS: C Sitting immobile in a chair will not help. Bed exercise and prophylactic footwear may. TED hose and SCD boots are recommended. Bed exercises, such as flexing, extending, and rotating her feet, ankles, and legs, are usefNulU. RASIpNoGsTitBiv.CeOHMomans' sign (calf muscle pain or warmth, redness, or tenderness) requires the physician's immediate attention. PTS: 1 DIF: Cognitive Level: Comprehension OBJ: Nursing Process: Implementation MSC: Client Needs: Physiologic Integrity 17. As relates to rubella and Rh issues, nurses should be aware that: a. breastfeeding mothers cannot be vaccinated with the live attenuated rubella virus. b. women should be warned that the rubella vaccination is teratogenic, and that they must avoid pregnancy for 1 month after vaccination. c. Rh immune globulin is safely administered intravenously because it cannot harm a nursing infant. d. Rh immune globulin boosts the immune system and thereby enhances the effectiveness of vaccinations. ANS: B Women should understand they must practice contraception for 1 month after being vaccinated. Because the live attenuated rubella virus is not communicable in breast milk, breastfeeding mothers can be vaccinated. Rh immune globulin is administered intramuscularly; it should never be given to an infant. Rh immune globulin suppresses the immune system and therefore could thwart the rubella vaccination. PTS: 1 DIF: Cognitive Level: Comprehension OBJ: Nursing Process: Planning MSC: Client Needs: Health Promotion and Maintenance 18. Discharge instruction, or teaching the woman what she needs to know to care for herself and her newborn, officially begins: a. at the time of admission to the nurse's unit. b. when the infant is presented to the mother at birth. c. during the first visit with the physician in the unit. d. when the take-home information packet is given to the couple. ANS: A Discharge planning, the teaching of maternal and newborn care, begins on the woman's admission to the unit, continues throughout her stay, and actually never ends as long as she has contact with medical personnel. PTS: 1 DIF: Cognitive Level: Comprehension OBJ: Nursing Process: Planning MSC: Client Needs: Health Promotion and Maintenance 19. A recently delivered mother and her baby are at the clinic for a 6-week after birth checkup. The nurse should be concerned that psychosocial outcomes are not being met if the woman: a. discusses her labor and birth experience excessively. b. believes that her baby is more attractive and clever than any others. c. has not given the baby a name. d. has a partner or family members who react very positively about the baby. ANS: C If the mother is having difficulty naming her new infant, it may be a signal that she is not adapting well to parenthood. Other red flags include refusal to hold or feed the baby, lack of interaction with the infant, and becoming upset when the baby vomits or needs a diaper change. A new mother who is having difficulty would be unwilling to discuss her labor and birth experience. An appropriate nNuUrsRiSnIgNdGiTaBgn.CoOsMis could be Impaired parenting related to a long, difficult labor, or unmet expectations of birth. A mother who is willing to discuss her birth experience is making a healthy personal adjustment. The mother who is not coping well would find her baby unattractive and messy. She may also be overly disappointed in the baby's sex. The patient may voice concern that the baby reminds her of a family member whom she does not like. Having a partner and/or other family members react positively is an indication that this new mother has a good support system in place. This support system will help reduce anxiety related to her new role as a mother. PTS: 1 DIF: Cognitive Level: Synthesis OBJ: Nursing Process: Evaluation MSC: Client Needs: Psychosocial Integrity 20. Postpartal overdistention of the bladder and urinary retention can lead to which complications? a. After birth hemorrhage and eclampsia b. Fever and increased blood pressure c. After birth hemorrhage and urinary tract infection d. Urinary tract infection and uterine rupture ANS: C Incomplete emptying and overdistention of the bladder can lead to urinary tract infection. Overdistention of the bladder displaces the uterus and prevents contraction of the uterine muscle, thus leading to after birth hemorrhage. There is no correlation between bladder distention and high blood pressure or eclampsia. The risk of uterine rupture decreases after the birth of the infant. PTS: 1 DIF: Cognitive Level: Comprehension OBJ: Nursing Process: Assessment MSC: Client Needs: Physiologic Integrity 21. Rho immune globulin will be ordered after birth if which situation occurs? a. Mother Rh–, baby Rh+ b. Mother Rh–, baby Rh– c. Mother Rh+, baby Rh+ d. Mother Rh+, baby Rh– ANS: A An Rh– mother delivering an Rh+ baby may develop antibodies to fetal cells that entered her bloodstream when the placenta separated. The Rho immune globulin works to destroy the fetal cells in the maternal circulation before sensitization occurs. If mother and baby are both Rh+ or Rh– the blood types are alike, so no antibody formation would be anticipated. If the Rh+ blood of the mother comes in contact with the Rh– blood of the infant, no antibodies would develop because the antigens are in the mother's blood, not the infant's. PTS: 1 DIF: Cognitive Level: Comprehension OBJ: Nursing Process: Assessment MSC: Client Needs: Physiologic Integrity 22. Which nursing action is most appropriate to correct a boggy uterus that is displaced above and to the right of the umbilicus? NURSINGTB.COM a. Notify the physician of an impending hemorrhage. b. Assess the blood pressure and pulse. c. Evaluate the lochia. d. Assist the patient in emptying her bladder. ANS: D Urinary retention may cause overdistention of the urinary bladder, which lifts and displaces the uterus. Nursing actions need to be implemented before notifying the physician. It is important to evaluate blood pressure, pulse, and lochia if the bleeding continues; however, the focus at this point in time is to assist the patient in emptying her bladder. PTS: 1 DIF: Cognitive Level: Application OBJ: Nursing Process: Implementation MSC: Client Needs: Health Promotion and Maintenance 23. When caring for a newly delivered woman, the nurse is aware that the best measure to prevent abdominal distention after a cesarean birth is: a. rectal suppositories. b. early and frequent ambulation. c. tightening and relaxing abdominal muscles. d. carbonated beverages. ANS: B Activity will aid the movement of accumulated gas in the gastrointestinal tract. Rectal suppositories can be helpful after distention occurs; however, they do not prevent it. Ambulation is the best prevention. Carbonated beverages may increase distention. PTS: 1 DIF: Cognitive Level: Knowledge OBJ: Nursing Process: Planning MSC: Client Needs: Physiologic Integrity 24. The nurse caring for the after birth woman understands that breast engorgement is caused by: a. overproduction of colostrum. b. accumulation of milk in the lactiferous ducts and glands. c. hyperplasia of mammary tissue. d. congestion of veins and lymphatics. ANS: D Breast engorgement is caused by the temporary congestion of veins and lymphatics. Breast engorgement is not the result of overproduction of colostrum. Accumulation of milk in the lactiferous ducts and glands does not cause breast engorgement. Hyperplasia of mammary tissue does not cause breast engorgement. PTS: 1 DIF: Cognitive Level: Knowledge OBJ: Nursing Process: Assessment MSC: Client Needs: Health Promotion and Maintenance MATCHING Much of a woman's behavior during the after birth period is strongly influenced by her cultural background. Nurses are likely to come into contact with women from many different countries and cultures. All cultureNsUhRaSvIeNdGeTvBe.lCoOpeMd safe and satisfying methods of caring for new mothers and babies. Please match the cultural norm with the nationality of origin. a. Prefer not to give babies colostrum b. Eat only warm foods and hot drinks c. Take the placenta home to bury d. Will not eat pork or pork products e. Have an IUD inserted after the first child 1. Muslim countries 2. Korean or other South East Asian countries. 3. Chinese 4. Haitian 5. Mexican 1. ANS: D PTS: 1 DIF: Cognitive Level: Comprehension OBJ: Nursing Process: Planning MSC: Client Needs: Health Promotion and Maintenance NOT: Chinese, Mexican, Korean, and South Asian women may wish to eat only warm foods and drinks hot drinks to replace blood loss and restore the balance of hot and cold in their bodies. These women may also wish to stay warm and avoid bathing, exercising, and hair washing for a number of days after childbirth. Self-management may not be a priority; care by family members is preferred. They may also prefer not to give the baby colostrum. Haitian women may request to take the placenta home to bury or burn. Muslim women follow very strict laws on modesty and diet. Birth control is government mandated in mainland China. Most Chinese women have an intrauterine device inserted after the birth of their first child. 2. ANS: A PTS: 1 DIF: Cognitive Level: Comprehension OBJ: Nursing Process: Planning MSC: Client Needs: Health Promotion and Maintenance NOT: Chinese, Mexican, Korean, and South Asian women may wish to eat only warm foods and drinks hot drinks to replace blood loss and restore the balance of hot and cold in their bodies. These women may also wish to stay warm and avoid bathing, exercising, and hair washing for a number of days after childbirth. Self-management may not be a priority; care by family members is preferred. They may also prefer not to give the baby colostrum. Haitian women may request to take the placenta home to bury or burn. Muslim women follow very strict laws on modesty and diet. Birth control is government mandated in mainland China. Most Chinese women have an intrauterine device inserted after the birth of their first child. 3. ANS: E PTS: 1 DIF: Cognitive Level: Comprehension OBJ: Nursing Process: Planning MSC: Client Needs: Health Promotion and Maintenance NOT: Chinese, Mexican, Korean, and South Asian women may wish to eat only warm foods and drinks hot drinks to replace blood loss and restore the balance of hot and cold in their bodies. These women may also wish to stay warm and avoid bathing, exercising, and hair washing for a number of days after childbirth. Self-management may not be a priority; care by family members is preferred. They may also prefer not to give the baby colostrum. Haitian women may request to take the placenta home to bury or burn. Muslim women follow very strict laws on modesty and diet. Birth control is government mandated in mainland China. Most Chinese women have an intrauterine device inserted after the birth of their first child. 4. ANS: C PTS: 1 DIF: Cognitive Level: Comprehension OBJ: Nursing Process: Planning MSC: Client Needs: Health Promotion and Maintenance NOT: Chinese, Mexican, Korean, and South Asian women may wish to eat only warm foods and drinks hot drinks to replace blood loss and restore the balance of hot and cold in their bodies. These women may also wish to stay warm and avoid bathing, exercising, and hair washing for a number of days after childbirth. Self-management may not be a priority; care by family members is preferred. They may also prefer not to give the baby colostrum. Haitian women may request to take the placenta home to bury or burn. Muslim women follow very strict laws on modesty and diet. Birth control is government mandated in mainland China. Most Chinese women have an intrauterine device inserted after the birth of their first child. NURSINGTB.COM 5. ANS: B PTS: 1 DIF: Cognitive Level: Comprehension OBJ: Nursing Process: Planning MSC: Client Needs: Health Promotion and Maintenance NOT: Chinese, Mexican, Korean, and South Asian women may wish to eat only warm foods and drinks hot drinks to replace blood loss and restore the balance of hot and cold in their bodies. These women may also wish to stay warm and avoid bathing, exercising, and hair washing for a number of days after childbirth. Self-management may not be a priority; care by family members is preferred. They may also prefer not to give the baby colostrum. Haitian women may request to take the placenta home to bury or burn. Muslim women follow very strict laws on modesty and diet. Birth control is government mandated in mainland China. Most Chinese women have an intrauterine device inserted after the birth of their first child. Chapter 20: Transition to Parenthood Perry: Maternal Child Nursing Care, 6th Edition MULTIPLE CHOICE 1. After giving birth to a healthy infant boy, a primiparous woman, 16 years old, is admitted to the after birth unit. An appropriate nursing diagnosis for her at this time is risk for impaired parenting related to deficient knowledge of newborn care. In planning for the woman's discharge, what should the nurse be certain to include in the plan of care? a. Instruct the patient how to feed and bathe her infant. b. Give the patient written information on bathing her infant. c. Advise the patient that all mothers instinctively know how to care for their infants. d. Provide time for the patient to bathe her infant after she views an infant bath demonstration. ANS: D Having the mother demonstrate infant care is a valuable method of assessing the patient's understanding of her newly acquired knowledge, especially in this age group, because she may inadvertently neglect her child. Although verbalizing how to care for the infant is a form of patient education, it is not the most developmentally appropriate teaching for a teenage mother. Advising the patient that all mothers instinctively know how to care for their infants is an inappropriate statement; it is belittling and false. PTS: 1 DIF: Cognitive Level: Application OBJ: Nursing Process: Diagnosis, Planning MSC: Client Needs: Health PromotiNonUaRnSdINMGaTiBnt.eCnOaMnce 2. The nurse observes several interactions between a after birth woman and her new son. What behavior, if exhibited by this woman, would the nurse identify as a possible maladaptive behavior regarding parent-infant attachment? a. Talks and coos to her son b. Seldom makes eye contact with her son c. Cuddles her son close to her d. Tells visitors how well her son is feeding ANS: B The woman should be encouraged to hold her infant in the en face position and make eye contact with the infant. Normal infant-parent interactions include talking and cooing to her son, cuddling her son close to her, and telling visitors how well her son is feeding. PTS: 1 DIF: Cognitive Level: Application OBJ: Nursing Process: Assessment, Diagnosis MSC: Client Needs: Psychosocial Integrity 3. The nurse observes that a 15-year-old mother seems to ignore her newborn. A strategy that the nurse can use to facilitate mother-infant attachment in this mother is to: a. tell the mother she must pay attention to her infant. b. show the mother how the infant initiates interaction and pays attention to her. c. demonstrate for the mother different positions for holding her infant while feeding. d. arrange for the mother to watch a video on parent-infant interaction. ANS: B Pointing out the responsiveness of the infant is a positive strategy for facilitating parent-infant attachment. Telling the mother that she must pay attention to her infant may be perceived as derogatory and is not appropriate. Educating the young mother in infant care is important; however, pointing out the responsiveness of her baby is a better tool for facilitating mother-infant attachment. Videos are an educational tool that can demonstrate parent-infant attachment, but encouraging the mother to recognize the infant's responsiveness is more appropriate. PTS: 1 DIF: Cognitive Level: Application OBJ: Nursing Process: Implementation MSC: Client Needs: Health Promotion and Maintenance 4. The nurse hears a primiparous woman talking to her son and telling him that his chin is just like his dad's chin. This woman's statement reflects: a. mutuality. b. synchrony. c. claiming. d. reciprocity. ANS: C Claiming refers to the process by which the child is identified in terms of likeness to other family members. Mutuality occurs when the infant's behaviors and characteristics call forth a corresponding set of maternal behaviors and characteristics. Synchrony refers to the “fit” between the infant's cues and the parent's responses. Reciprocity is a type of body movement or behavior that provides the observer with cues. PTS: 1 DIF: CognitiNveURLSevINelG: TCBo.mCOprMehension OBJ: Nursing Process: Evaluation MSC: Client Needs: Psychosocial Integrity 5. New parents express concern that, because of the mother's emergency cesarean birth under general anesthesia, they did not have the opportunity to hold and bond with their daughter immediately after her birth. The nurse's response should convey to the parents that: a. attachment, or bonding, is a process that occurs over time and does not require early contact. b. the time immediately after birth is a critical period for people. c. early contact is essential for optimum parent-infant relationships. d. they should just be happy that the infant is healthy. ANS: A Attachment, or bonding, is a process that occurs over time and does not require early contact. The formerly accepted definition of bonding held that the period immediately after birth was a critical time for bonding to occur. Research since has indicated that parent-infant attachment occurs over time. A delay does not inhibit the process. Parent-infant attachment involves activities such as touching, holding, and gazing; it is not exclusively eye contact. A response that conveys that the parents should just be happy that the infant is healthy is inappropriate because it is derogatory and belittling. PTS: 1 DIF: Cognitive Level: Application OBJ: Nursing Process: Implementation MSC: Client Needs: Health Promotion and Maintenance 6. During a phone follow-up conversation with a woman who is 4 days' after birth, the woman tells the nurse, “I don't know what's wrong. I love my son, but I feel so let down. I seem to cry for no reason!” The nurse would recognize that the woman is experiencing: a. taking-in. b. postpartum depression (PPD). c. postpartum (PP) blues. d. attachment difficulty. ANS: C During the PP blues women are emotionally labile, often crying easily and for no apparent reason. This lability seems to peak around the fifth PP day. The taking-in phase is the period after birth when the mother focuses on her own psychologic needs. Typically this period lasts 24 hours. PPD is an intense, pervasive sadness marked by severe, labile mood swings; it is more serious and persistent than the PP blues. Crying is not a maladaptive attachment response; it indicates PP blues. PTS: 1 DIF: Cognitive Level: Comprehension OBJ: Nursing Process: Assessment, Diagnosis MSC: Client Needs: Psychosocial Integrity 7. The nurse can help a father in his transition to parenthood by: a. pointing out that the infant turned at the sound of his voice. b. encouraging him to go home to get some sleep. c. telling him to tape the infant's diaper a different way. d. suggesting that he let the infant sleep in the bassinet. ANS: A Infants respond to the sound of voNicUeRs.SIBNeGcTaBus.CeOaMttachment involves a reciprocal interchange, observing the interaction between parent and infant is very important. Separation of the parent and infant does not encourage parent-infant attachment. Educating the parent in infant care techniques is important; however, the manner in which a diaper is taped is not relevant and does not enhance parent-infant interactions. Parent-infant attachment involves touching, holding, and cuddling. It is appropriate for a father to want to hold the infant as the baby sleeps. PTS: 1 DIF: Cognitive Level: Application OBJ: Nursing Process: Implementation MSC: Client Needs: Health Promotion and Maintenance 8. The nurse notes that a Vietnamese woman does not cuddle or interact with her newborn other than to feed him, change his diapers or soiled clothes, and put him to bed. In evaluating the woman's behavior with her infant, the nurse realizes that: a. what appears to be a lack of interest in the newborn is in fact the Vietnamese way of demonstrating intense love by attempting to ward off evil spirits. b. the woman is inexperienced in caring for newborns. c. the woman needs a referral to a social worker for further evaluation of her parenting behaviors once she goes home with the newborn. d. extra time needs to be planned for assisting the woman in bonding with her newborn. ANS: A The nurse may observe a Vietnamese woman who gives minimal care to her infant and refuses to cuddle or interact with her infant. The apparent lack of interest in the newborn is this cultural group's attempt to ward off evil spirits and actually reflects an intense love and concern for the infant. It is important to educate the woman in infant care, but it is equally important to acknowledge her cultural beliefs and practices. PTS: 1 DIF: Cognitive Level: Comprehension OBJ: Nursing Process: Evaluation MSC: Client Needs: Psychosocial Integrity 9. Many first-time parents do not plan on their parents' help immediately after the newborn arrives. What statement by the nurse is the most appropriate when counseling new parents about the involvement of grandparents? a. “You should tell your parents to leave you alone.” b. “Grandparents can help you with parenting skills and also help preserve family traditions.” c. “Grandparent involvement can be very disruptive to the family.” d. “They are getting old. You should let them be involved while they can.” ANS: B “Grandparents can help you with parenting skills and also help preserve family traditions” is the most appropriate response. Intergenerational help may be perceived as interference; however, a statement of this sort is not therapeutic to the adaptation of the family. Not only is “Grandparent involvement can be very disruptive to the family” invalid, it also is not an appropriate nursing response. Regardless of age, grandparents can help with parenting skills and preserve family traditions. Talking about the age of the grandparents is not the most appropriate statement, and it does not demonstrate sensitivity on the part of the nurse. NURSINGTB.COM PTS: 1 DIF: Cognitive Level: Analysis OBJ: Nursing Process: Planning MSC: Client Needs: Psychosocial Integrity 10. When the infant's behaviors and characteristics call forth a corresponding set of maternal behaviors and characteristics, this is called: a. mutuality. b. bonding. c. claiming. d. acquaintance. ANS: A Mutuality extends the concept of attachment to include this shared set of behaviors. Bonding is the process over time of parents forming an emotional attachment to their infant. Mutuality refers to a shared set of behaviors that is a part of the bonding process. Claiming is the process by which parents identify their new baby in terms of likeness to other family members and their differences and uniqueness. Like mutuality, acquaintance is part of attachment. It describes how parents get to know their baby during the immediate after birth period through eye contact, touching, and talking. PTS: 1 DIF: Cognitive Level: Knowledge OBJ: Nursing Process: Evaluation MSC: Client Needs: Psychosocial Integrity 11. In follow-up appointments or visits with parents and their new baby, it may be useful if the nurse can identify parental behaviors that can either facilitate or inhibit attachment. Which one is a facilitating behavior? a. The parents have difficulty naming the infant. b. The parents hover around the infant, directing attention to and pointing at the infant. c. The parents make no effort to interpret the actions or needs of the infant. d. The parents do not move from fingertip touch to palmar contact and holding. ANS: B Hovering over the infant and obviously paying attention to the baby are facilitating behaviors. Inhibiting behaviors include difficulty naming the infant, making no effort to interpret the actions or needs of the infant, and not moving from fingertip touch to palmar contact and holding. PTS: 1 DIF: Cognitive Level: Comprehension OBJ: Nursing Process: Assessment MSC: Client Needs: Health Promotion and Maintenance 12. With regard to parents' early and extended contact with their infant and the relationships built, nurses should be aware that: a. immediate contact is essential for the parent-child relationship. b. skin-to-skin contact is preferable to contact with the body totally wrapped in a blanket. c. extended contact is especially important for adolescents and low-income women because they are at risk for parenting inadequacies. d. mothers need to take precedence over their partners and other family matters. ANS: C NURSINGTB.COM Nurses should encourage any activity that optimizes family extended contact. Immediate contact facilitates the attachment process but is not essential; otherwise, adopted infants would not establish the affectionate ties they do. The mode of infant-mother contact does not appear to have any important effect. Mothers and their partners are considered equally important. PTS: 1 DIF: Cognitive Level: Comprehension OBJ: Nursing Process: Planning MSC: Client Needs: Psychosocial Integrity 13. In the United States the en face position is preferred immediately after birth. Nurses can facilitate this process by all of these actions except: a. washing both the infant's face and the mother's face. b. placing the infant on the mother's abdomen or breast with their heads on the same plane. c. dimming the lights. d. delaying the instillation of prophylactic antibiotic ointment in the infant's eyes. ANS: A To facilitate the position in which the parent's and infant's faces are approximately 8 inches apart on the same plane, allowing them to make eye contact, the nurse can place the infant at the proper height on the mother's body, dim the light so that the infant's eyes open, and delay putting ointment in the infant's eyes. PTS: 1 DIF: Cognitive Level: Comprehension OBJ: Nursing Process: Implementation MSC: Client Needs: Health Promotion and Maintenance 14. Other early sensual contacts between infant and mother involve sound and smell. Nurses should be aware that, despite what folk wisdom may say: a. high-pitched voices irritate newborns. b. infants can learn to distinguish their mother's voice from others soon after birth. c. all babies in the hospital smell alike. d. a mother's breast milk has no distinctive odor. ANS: B Infants know the sound of their mother's voice early. Infants respond positively to high-pitched voices. Each infant has a unique odor. Infants quickly learn to distinguish the odor of their mother's breast milk. PTS: 1 DIF: Cognitive Level: Knowledge OBJ: Nursing Process: Planning MSC: Client Needs: Health Promotion and Maintenance 15. After they are born, a crying infant may be soothed by being held in a position in which the newborn can hear the mother's heartbeat. This phenomenon is known as: a. entrainment. b. reciprocity. c. synchrony. d. biorhythmicity. ANS: D The newborn is in rhythm with the mother. The infant develops a personal biorhythm with the parents' help over time. Entrainment is the movement of newborns in time to the structure of adult speech. Reciprocity is body NmUoRvSeImNeGnTtBo.Cr ObeMhavior that gives cues to the person's desires. These take several weeks to develop with a new baby. Synchrony is the fit between the infant's behavioral cues and the parent's responses. PTS: 1 DIF: Cognitive Level: Knowledge OBJ: Nursing Process: Implementation MSC: Client Needs: Health Promotion and Maintenance 16. Of the many factors that influence parental responses, nurses should be conscious of negative stereotypes that apply to specific patient populations. Which response could be an inappropriate stereotype of adolescent mothers? a. An adolescent mother's egocentricity and unmet developmental needs interfere with her ability to parent effectively. b. An adolescent mother is likely to use less verbal instruction, be less responsive, and interact less positively than other mothers. c. Adolescent mothers have a higher documented incidence of child abuse. d. Mothers older than 35 often deal with more stress related to work and career issues and decreasing libido. ANS: C Adolescent mothers are more inclined to have a number of parenting difficulties that benefit from counseling; however, a higher incidence of child abuse is not one of them. Midlife mothers have many competencies, but they are more likely to have to deal with career issues and the accompanying stress. PTS: 1 DIF: Cognitive Level: Comprehension OBJ: Nursing Process: Evaluation MSC: Client Needs: Psychosocial Integrity 17. When working with parents who have some form of sensory impairment, nurses should understand that _ is an inaccurate statement. a. “One of the major difficulties visually impaired parents experience is the skepticism of health care professionals.” b. “Visually impaired mothers cannot overcome the infant's need for eye-to-eye contact.” c. “The best approach for the nurse is to assess the parents' capabilities rather than focusing on their disabilities.” d. “Technologic advances, including the Internet, can provide deaf parents with a full range of parenting activities and information.” ANS: B Other sensory output can be provided by the parent, other people can participate, and other coping devices can be used. The skepticism, open or hidden, of health care professionals places an additional and unneeded hurdle for the parents. After the parents' capabilities have been assessed (including some the nurse may not have expected), the nurse can help find ways to assist the parents that play to their strengths. The Internet affords an extra teaching tool for the deaf, as do videos with subtitles or nurses signing. A number of electronic devices can turn sound into light flashes to help pick up a child's cry. Sign language is readily acquired by young children. PTS: 1 DIF: Cognitive Level: Comprehension OBJ: Nursing Process: Planning MSC: Client Needs: Health Promotion and Maintenance 18. With regard to the adaptation of other family members, mainly siblings and grandparents, to the newborn, nurses should be awNarUeRtShIaNt:GTB.COM a. sibling rivalry cannot be dismissed as overblown psychobabble; negative feelings and behaviors can take a long time to blow over. b. participation in preparation classes helps both siblings and grandparents. c. in the United States paternal and maternal grandparents consider themselves of equal importance and status. d. in the past few decades the number of grandparents providing permanent care to their grandchildren has been declining. ANS: B Preparing older siblings and grandparents helps everyone to adapt. Sibling rivalry should be expected initially, but the negative behaviors associated with it have been overemphasized and stop in a comparatively short time. In the United States, in contrast to other cultures, paternal grandparents frequently consider themselves secondary to maternal grandparents. The number of grandparents providing permanent child care has been on the increase. PTS: 1 DIF: Cognitive Level: Comprehension OBJ: Nursing Process: Planning MSC: Client Needs: Psychosocial Integrity 19. Nursing activities that promote parent-infant attachment are many and varied. One activity that should not be overlooked is management of the environment. While providing routine mother-baby care, the nurse should ensure that: a. the baby is able to return to the nursery at night so that the new mother can sleep. b. routine times for care are established to reassure the parents. c. the father should be encouraged to go home at night to prepare for mother-baby discharge. d. an environment that fosters as much privacy as possible should be created. ANS: D Care providers need to knock before gaining entry. Nursing care activities should be grouped. Once the baby has demonstrated adjustment to extrauterine life (either in the mother's room or the transitional nursery), all care should be provided in one location. This important principle of family-centered maternity care fosters attachment by giving parents the opportunity to learn about their infant 24 hours a day. One nurse should provide care to both mother and baby in this couplet care or rooming-in model. It is not necessary for the baby to return to the nursery at night. In fact, the mother will sleep better with the infant close by. Care should be individualized to meet the parents' needs, not the routines of the staff. Teaching goals should be developed in collaboration with the parents. The father, or other significant other, should be permitted to sleep in the room with the mother. The maternity unit should develop policies that allow for the presence of significant others as much as the new mother desires. PTS: 1 DIF: Cognitive Level: Application OBJ: Nursing Process: Implementation MSC: Client Needs: Health Promotion and Maintenance 20. The early after birth period is a time of emotional and physical vulnerability. Many mothers can easily become psychologically overwhelmed by the reality of their new parental responsibilities. Fatigue compounds these issues. Although the baby blues are a common occurrence in the after birth period, about one-half million women in America experience a more severe syndrome known as postpartum depression (PPD). Which statement regarding PPD is essential for the nurse to bNe UawRSaIrNeGoTfBw.ChOenMattempting to formulate a nursing diagnosis? a. PPD symptoms are consistently severe. b. This syndrome affects only new mothers. c. PPD can easily go undetected. d. Only mental health professionals should teach new parents about this condition. ANS: C PPD can go undetected because parents do not voluntarily admit to this type of emotional distress out of embarrassment, fear, or guilt. PPD symptoms range from mild to severe, with women having both good and bad days. Both mothers and fathers should be screened. PPD may also affect new fathers. The nurse should include information on PPD and how to differentiate this from the baby blues for all patients on discharge. Nurses also can urge new parents to report symptoms and seek follow-up care promptly if symptoms occur. PTS: 1 DIF: Cognitive Level: Synthesis OBJ: Nursing Process: Diagnosis MSC: Client Needs: Psychosocial Integrity 21. The mother-baby nurse is able to recognize reciprocal attachment behavior. This refers to: a. the positive feedback an infant exhibits toward parents during the attachment process. b. behavior during the sensitive period when the infant is in the quiet alert stage. c. unidirectional behavior exhibited by the infant, initiated and enhanced by eye contact. d. behavior by the infant during the sensitive period to elicit feelings of “falling in love” from the parents. ANS: A In this definition, “reciprocal” refers to the feedback from the infant during the attachment process. This is a good time for bonding; however, it does not define reciprocal attachment. Reciprocal attachment applies to feedback behavior and is not unidirectional. PTS: 1 DIF: Cognitive Level: Application OBJ: Nursing Process: Assessment MSC: Client Needs: Health Promotion and Maintenance 22. The after birth woman who continually repeats the story of her labor, delivery, and recovery experience is: a. providing others with her knowledge of events. b. making the birth experience “real.” c. taking hold of the events leading to her labor and delivery. d. accepting her response to labor and delivery. ANS: B Reliving the birth experience makes the event real and helps the mother realize that the pregnancy is over and that the infant is born and is now a separate individual. The retelling of the story is to satisfy her needs, not the needs of others. This new mother is in the taking-in phase, trying to make the birth experience seem real and separate the infant from herself. PTS: 1 DIF: Cognitive Level: Knowledge OBJ: Nursing Process: Assessment MSC: Client Needs: Psychosocial Integrity 23. On observing a woman on her first after birth day sitting in bed while her newborn lies awake in the bassinet, the nurse should: NURSINGTB.COM a. realize that this situation is perfectly acceptable. b. offer to hand the baby to the woman. c. hand the baby to the woman. d. explain “taking in” to the woman. ANS: C During the “taking-in” phase of maternal adaptation (the mother may be passive and dependent), the nurse should encourage bonding when the infant is in the quiet alert stage. This is done best by simply giving the baby to the mother. The patient is exhibiting expected behavior during the taking-in phase; however, interventions by the nurse can facilitate infant bonding. The patient will learn best during the taking-hold phase. PTS: 1 DIF: Cognitive Level: Application OBJ: Nursing Process: Implementation MSC: Client Needs: Psychosocial Integrity 24. A nurse is observing a family. The mother is holding the baby she delivered less than 24 hours ago. Her husband is watching his wife and asking questions about newborn care. The 4-year-old brother is punching his mother on the back. The nurse should: a. report the incident to the social services department. b. advise the parents that the toddler needs to be reprimanded. c. report to oncoming staff that the mother is probably not a good disciplinarian. d. realize that this is a normal family adjusting to family change. ANS: D The observed behaviors are normal variations of families adjusting to change. There is no need to report this one incident. Giving advice at this point would make the parents feel inadequate. PTS: 1 DIF: Cognitive Level: Analysis OBJ: Nursing Process: Assessment MSC: Client Needs: Psychosocial Integrity 25. The best way for the nurse to promote and support the maternal-infant bonding process is to: a. help the mother identify her positive feelings toward the newborn. b. encourage the mother to provide all newborn care. c. assist the family with rooming-in. d. return the newborn to the nursery during sleep periods. ANS: C Close and frequent interaction between mother and infant, which is facilitated by rooming-in, is important in the bonding process. This is often referred to as the mother-baby care or couplet care. Having the mother express her feelings is important; however, it is not the best way to promote bonding. The mother needs time to rest and recuperate; she should not be expected to do all of the care. The patient needs to observe the infant during all stages so she will be aware of what to anticipate when they go home. PTS: 1 DIF: Cognitive Level: Comprehension OBJ: Nursing Process: Planning MSC: Client Needs: Psychosocial Integrity 26. During which phase of maternal adjustment will the mother relinquish the baby of her fantasies and accept the real baby? a. Letting go b. Taking hold c. Taking in d. Taking on ANS: A NURSINGTB.COM Accepting the real infant and relinquishing the fantasy infant occurs during the letting-go phase of maternal adjustment. During the taking-hold phase the mother assumes responsibility for her own care and shifts her attention to the infant. In the taking-in phase the mother is primarily focused on her own needs. There is no taking-on phase of maternal adjustment. PTS: 1 DIF: Cognitive Level: Knowledge OBJ: Nursing Process: Assessment MSC: Client Needs: Psychosocial Integrity 27. A 25-year-old gravida 1 para 1 who had an emergency cesarean birth 3 days ago is scheduled for discharge. As you prepare her for discharge, she begins to cry. Your initial action should be to: a. assess her for pain. b. point out how lucky she is to have a healthy baby. c. explain that she is experiencing after birth blues. d. allow her time to express her feelings. ANS: D Although many women experience transient after birth blues, they need assistance in expressing their feelings. This condition affects 50% to 80% of new mothers. There should be no assumption that the patient is in pain, when in fact she may have no pain whatsoever. This is “blocking” communication and inappropriate in this situation. The patient needs the opportunity to express her feelings first; patient teaching can occur later. PTS: 1 DIF: Cognitive Level: Application OBJ: Nursing Process: Implementation MSC: Client Needs: Psychosocial Integrity 28. A man calls the nurse's station and states that his wife, who delivered 2 days ago, is happy one minute and crying the next. The man says, “She was never like this before the baby was born.” The nurse's initial response could be to: a. tell him to ignore the mood swings, as they will go away. b. reassure him that this behavior is normal. c. advise him to get immediate psychological help for her. d. instruct him in the signs, symptoms, and duration of after birth blues. ANS: B Before providing further instructions, inform family members of the fact that after birth blues are a normal process. Telling her partner to “ignore the mood swings” does not encourage further communication and may belittle the husband's concerns. After birth blues are usually short-lived; no medical intervention is needed. Patient teaching is important; however, the new father's anxieties need to be allayed before he will be receptive to teaching. PTS: 1 DIF: Cognitive Level: Application OBJ: Nursing Process: Implementation MSC: Client Needs: Psychosocial Integrity 29. To promote bonding and attachmeNnUt RimSImNGedTiBa.tCeOlyMafter delivery, the most important nursing intervention is to: a. allow the mother quiet time with her infant. b. assist the mother in assuming an en face position with her newborn. c. teach the mother about the concepts of bonding and attachment. d. assist the mother in feeding her baby. ANS: B Assisting the mother in assuming an en face position with her newborn will support the bonding process. The mother should be given as much privacy as possible; however, nursing assessments must still be continued during this critical time. The mother has just delivered and is more focused on the infant; she will not be receptive to teaching at this time. This is a good time to initiate breastfeeding; however, the mother first needs time to explore the new infant and begin the bonding process. PTS: 1 DIF: Cognitive Level: Application OBJ: Nursing Process: Implementation MSC: Client Needs: Health Promotion and Maintenance 30. A new father states, “I know nothing about babies,” but he seems to be interested in learning. This is an ideal opportunity for the nurse to: a. continue to observe his interaction with the newborn. b. tell him when he does something wrong. c. show no concern, as he will learn on his own. d. include him in teaching sessions. ANS: D The nurse must be sensitive to the father's needs and include him whenever possible. As fathers take on their new role, the nurse should praise every attempt, even if his early care is awkward. It is important to note the bonding process of the mother and the father; however, that does not satisfy the expressed needs of the father. The new father should be encouraged in caring for his baby by pointing out the things that he does right. Criticizing him will discourage him. PTS: 1 DIF: Cognitive Level: Application OBJ: Nursing Process: Planning MSC: Client Needs: Health Promotion and Maintenance MULTIPLE RESPONSE 1. Which concerns about parenthood are often expressed by visually impaired mothers? (Select all that apply.) a. Infant safety b. Transportation c. The ability to care for the infant d. Missing out visually e. Needing extra time for parenting activities to accommodate the visual limitations ANS: A, B, D, E Concerns expressed by visually impaired mothers include infant safety, extra time needed for parenting activities, transportation, handling other people's reactions, providing proper discipline, and missing out visually. Blind people sense reluctance on the part of others to acknowledge that they have a right to be parents; however, blind parents are fully capable of caring for their infants. NURSINGTB.COM PTS: 1 DIF: Cognitive Level: Comprehension OBJ: Nursing Process: Evaluation MSC: Client Needs: Health Promotion and Maintenance 2. A parent who has a hearing impairment is presented with a number of challenges in parenting. Which nursing approaches are appropriate for working with hearing-impaired new parents? (Select all that apply.) a. Use devices that transform sound into light. b. Assume that the patient knows sign language. c. Speak quickly and loudly. d. Ascertain whether the patient can read lips before teaching. e. Written messages aid in communication. ANS: A, D, E Section 504 of the Rehabilitation Act of 1973 requires that hospitals use various communication techniques and resources with the deaf and hard of hearing patient. This includes devices such as door alarms, cry alarms, and amplifiers. Before initiating communication, the nurse needs to be aware of the parents' preferences for communication. Not all hearing-impaired patients know sign language. Do they wear a hearing aid? Do they read lips? Do they wish to have a sign language interpreter? If the parent relies on lipreading, the nurse should sit close enough so that the parent can visualize lip movements. The nurse should speak clearly in a regular voice volume, in short, simple sentences. Written messages such as on a black or white erasable board can be useful. Written materials should be reviewed with the parents before discharge. PTS: 1 DIF: Cognitive Level: Application OBJ: Nursing Process: Planning MSC: Client Needs: Safe and Effective Care Environment NURSINGTB.COM Chapter 21: Postpartum Complications Perry: Maternal Child Nursing Care, 6th Edition MULTIPLE CHOICE 1. The perinatal nurse is caring for a woman in the immediate postbirth period. Assessment reveals that the woman is experiencing profuse bleeding. The most likely etiology for the bleeding is: a. uterine atony. b. uterine inversion. c. vaginal hematoma. d. vaginal laceration. ANS: A Uterine atony is marked hypotonia of the uterus. It is the leading cause of after birth hemorrhage. Uterine inversion may lead to hemorrhage, but it is not the most likely source of this patient's bleeding. Furthermore, if the woman is experiencing a uterine inversion, it would be evidenced by the presence of a large, red, rounded mass protruding from the introitus. A vaginal hematoma may be associated with hemorrhage. However, the most likely clinical finding would be pain, not the presence of profuse bleeding. A vaginal laceration may cause hemorrhage, but it is more likely that profuse bleeding would result from uterine atony. A vaginal laceration should be suspected if vaginal bleeding continues in the presence of a firm, contracted uterine fundus. PTS: 1 DIF: Cognitive Level: Comprehension OBJ: Nursing Process: Diagnosis MSC: Client Needs: Physiologic IntNegUriRtySINGTB.COM 2. A primary nursing responsibility when caring for a woman experiencing an obstetric hemorrhage associated with uterine atony is to: a. establish venous access. b. perform fundal massage. c. prepare the woman for surgical intervention. d. catheterize the bladder. ANS: B The initial management of excessive after birth bleeding is firm massage of the uterine fundus. Although establishing venous access may be a necessary intervention, the initial intervention would be fundal massage. The woman may need surgical intervention to treat her after birth hemorrhage, but the initial nursing intervention would be to assess the uterus. After uterine massage the nurse may want to catheterize the patient to eliminate any bladder distention that may be preventing the uterus from contracting properly. PTS: 1 DIF: Cognitive Level: Application OBJ: Nursing Process: Implementation MSC: Client Needs: Physiologic Integrity 3. The perinatal nurse caring for the after birth woman understands that late postpartum hemorrhage (PPH) is most likely caused by: a. subinvolution of the placental site. b. defective vascularity of the decidua. c. cervical lacerations. d. coagulation disorders. ANS: A Late PPH may be the result of subinvolution of the uterus, pelvic infection, or retained placental fragments. Late PPH is not typically a result of defective vascularity of the decidua, cervical lacerations, or coagulation disorders. PTS: 1 DIF: Cognitive Level: Comprehension OBJ: Nursing Process: Planning MSC: Client Needs: Physiologic Integrity 4. Which woman is at greatest risk for early postpartum hemorrhage (PPH)? a. A primiparous woman (G 2 P 1 0 0 1) being prepared for an emergency cesarean birth for fetal distress. b. A woman with severe preeclampsia who is receiving magnesium sulfate and whose labor is being induced. c. A multiparous woman (G 3 P 2 0 0 2) with an 8-hour labor. d. A primigravida in spontaneous labor with preterm twins. ANS: B Magnesium sulfate administration during labor poses a risk for PPH. Magnesium acts as a smooth muscle relaxant, thereby contributing to uterine relaxation and atony. Although many causes and risk factors are associated with PPH, the primiparous woman being prepared for an emergency C-section, the multiparous woman with 8-hour labor, and the primigravida in spontaneous labor do not pose risk factors or causes of early PPH. PTS: 1 DIF: Cognitive Level: Analysis OBJ: Nursing Process: Planning MSC: Client Needs: Physiologic Integrity NURSINGTB.COM 5. The first and most important nursing intervention when a nurse observes profuse after birth bleeding is to: a. call the woman's primary health care provider. b. administer the standing order for an oxytocic. c. palpate the uterus and massage it if it is boggy. d. assess maternal blood pressure and pulse for signs of hypovolemic shock. ANS: C The initial management of excessive after birth bleeding is firm massage of the uterine fundus. Although calling the health care provider, administering an oxytocic, and assessing maternal BP are appropriate interventions, the primary intervention should be to assess the uterus. Uterine atony is the leading cause of postpartum hemorrhage (PPH). PTS: 1 DIF: Cognitive Level: Application OBJ: Nursing Process: Implementation MSC: Client Needs: Physiologic Integrity 6. When caring for a after birth woman experiencing hemorrhagic shock, the nurse recognizes that the most objective and least invasive assessment of adequate organ perfusion and oxygenation is: a. absence of cyanosis in the buccal mucosa. b. cool, dry skin. c. diminished restlessness. d. urinary output of at least 30 mL/hr. ANS: D Hemorrhage may result in hemorrhagic shock. Shock is an emergency situation in which the perfusion of body organs may become severely compromised and death may occur. The presence of adequate urinary output indicates adequate tissue perfusion. The assessment of the buccal mucosa for cyanosis can be subjective. The presence of cool, pale, clammy skin would be an indicative finding associated with hemorrhagic shock. Hemorrhagic shock is associated with lethargy, not restlessness. PTS: 1 DIF: Cognitive Level: Analysis OBJ: Nursing Process: Assessment MSC: Client Needs: Physiologic Integrity 7. One of the first symptoms of puerperal infection to assess for in the after birth woman is: a. fatigue continuing for longer than 1 week. b. pain with voiding. c. profuse vaginal bleeding with ambulation. d. temperature of 38° C (100.4° F) or higher on two successive days starting 24 hours after birth. ANS: D After birth or puerperal infection is any clinical infection of the genital canal that occurs within 28 days after miscarriage, induced abortion, or childbirth. The definition used in the United States continues to be the presence of a fever of 38° C (100.4° F) or higher on two successive days of the first 10 after birth days, starting 24 hours after birth. Fatigue would be a late finding associated with infection. Pain with voiding may indicate a urinary tract infection, but it is not typically one of the earlier symptoms of infection. Profuse lochia may be associated with endometritis, but it is not the first symptom associated with infection. PTS: 1 DIF: CognitiNveURLSevINelG: TCBo.mCOprMehension OBJ: Nursing Process: Assessment MSC: Client Needs: Health Promotion and Maintenance 8. Nurses need to know the basic definitions and incidence data about postpartum hemorrhage (PPH). For instance: a. PPH is easy to recognize early; after all, the woman is bleeding. b. traditionally it takes more than 1000 mL of blood after vaginal birth and 2500 mL after cesarean birth to define the condition as PPH. c. if anything, nurses and doctors tend to overestimate the amount of blood loss. d. traditionally PPH has been classified as early or late with respect to birth. ANS: D Early PPH is also known as primary, or acute, PPH; late PPH is known as secondary PPH. Unfortunately PPH can occur with little warning and often is recognized only after the mother has profound symptoms. Traditionally a 500-mL blood loss after a vaginal birth and a 1000-mL blood loss after a cesarean birth constitute PPH. Medical personnel tend to underestimate blood loss by as much as 50% in their subjective observations. PTS: 1 DIF: Cognitive Level: Knowledge OBJ: Nursing Process: Diagnosis MSC: Client Needs: Physiologic Integrity 9. A woman who has recently given birth complains of pain and tenderness in her leg. On physical examination the nurse notices warmth and redness over an enlarged, hardened area. The nurse should suspect and should confirm the diagnosis by . a. disseminated intravascular coagulation; asking for laboratory tests b. von Willebrand disease; noting whether bleeding times have been extended c. thrombophlebitis; using real-time and color Doppler ultrasound d. coagulopathies; drawing blood for laboratory analysis ANS: C Pain and tenderness in the extremities, which show warmth, redness, and hardness, likely indicate thrombophlebitis. Doppler ultrasound is a common noninvasive way to confirm diagnosis. PTS: 1 DIF: Cognitive Level: Analysis OBJ: Nursing Process: Assessment MSC: Client Needs: Physiologic Integrity 10. What PPH conditions are considered medical emergencies that require immediate treatment? a. Inversion of the uterus and hypovolemic shock b. Hypotonic uterus and coagulopathies c. Subinvolution of the uterus and idiopathic thrombocytopenic purpura d. Uterine atony and disseminated intravascular coagulation ANS: A Inversion of the uterus and hypovolemic shock are considered medical emergencies. Although hypotonic uterus and coagulopathies, subinvolution of the uterus and idiopathic thrombocytopenic purpura, and uterine atony and disseminated intravascular coagulation are serious conditions, they are not necessarily medical emergencies that require immediate treatment. PTS: 1 DIF: Cognitive Level: Comprehension OBJ: Nursing Process: ImplementaNtioUnRSINMGSTCB.:CCOlMient Needs: Physiologic Integrity 11. What infection is contracted mostly by first-time mothers who are breastfeeding? a. Endometritis b. Wound infections c. Mastitis d. Urinary tract infections ANS: C Mastitis is infection in a breast, usually confined to a milk duct. Most women who suffer this are primiparas who are breastfeeding. PTS: 1 DIF: Cognitive Level: Knowledge OBJ: Nursing Process: Diagnosis MSC: Client Needs: Health Promotion and Maintenance 12. Despite popular belief, there is a rare type of hemophilia that affects women of childbearing age. von Willebrand disease is the most common of the hereditary bleeding disorders and can affect males and females alike. It results from a factor VIII deficiency and platelet dysfunction. Although factor VIII levels increase naturally during pregnancy, there is an increased risk for after birth hemorrhage from birth until 4 weeks after delivery as levels of von Willebrand factor (vWf) and factor VIII decrease. The treatment that should be considered first for the patient with von Willebrand disease who experiences a after birth hemorrhage is: a. cryoprecipitate. b. factor VIII and vWf. c. desmopressin. d. hemabate. ANS: C Desmopressin is the primary treatment of choice. This hormone can be administered orally, nasally, and intravenously. This medication promotes the release of factor VIII and vWf from storage. Cryoprecipitate may be used; however, because of the risk of possible donor viruses, other modalities are considered safer. Treatment with plasma products such as factor VIII and vWf is an acceptable option for this patient. Because of the repeated exposure to donor blood products and possible viruses, this is not the initial treatment of choice. Although the administration of this prostaglandin is known to promote contraction of the uterus during after birth hemorrhage, it is not effective for the patient who presents with a bleeding disorder. PTS: 1 DIF: Cognitive Level: Application OBJ: Nursing Process: Implementation MSC: Client Needs: Physiologic Integrity 13. When a woman is diagnosed with postpartum depression (PPD) with psychotic features, one of the main concerns is that she may: a. have outbursts of anger. b. neglect her hygiene. c. harm her infant. d. lose interest in her husband. ANS: C Thoughts of harm to oneself' or the infant are among the most serious symptoms of PPD and require immediate assessment and intervention. Although outbursts of anger, hygiene neglect, and loss of interest in her husband are attributable to PPD, the major concern would be the potential to harm herself or her infNaUnRt.SINGTB.COM PTS: 1 DIF: Cognitive Level: Comprehension OBJ: Nursing Process: Diagnosis MSC: Client Needs: Psychosocial Integrity 14. To provide adequate after birth care, the nurse should be aware that postpartum depression (PPD) without psychotic features: a. means that the woman is experiencing the baby blues. In addition she has a visit with a counselor or psychologist. b. is more common among older, Caucasian women because they have higher expectations. c. is distinguished by irritability, severe anxiety, and panic attacks. d. will disappear on its own without outside help. ANS: C PPD is also characterized by spontaneous crying long after the usual duration of the baby blues. PPD, even without psychotic features, is more serious and persistent than after birth baby blues. It is more common among younger mothers and African-American mothers. Most women need professional help to get through PPD, including pharmacologic intervention. PTS: 1 DIF: Cognitive Level: Comprehension OBJ: Nursing Process: Assessment MSC: Client Needs: Psychosocial Integrity 15. To provide adequate after birth care, the nurse should be aware that postpartum depression (PPD) with psychotic features: a. is more likely to occur in women with more than two children. b. is rarely delusional and then is usually about someone trying to harm her (the mother). c. although serious, is not likely to need psychiatric hospitalization. d. may include bipolar disorder (formerly called “manic depression”). ANS: D Manic mood swings are possible. PPD is more likely to occur in first-time mothers. Delusions may be present in 50% of women with PPD, usually about something being wrong with the infant. PPD with psychosis is a psychiatric emergency that requires hospitalization. PTS: 1 DIF: Cognitive Level: Comprehension OBJ: Nursing Process: Assessment MSC: Client Needs: Psychosocial Integrity 16. With shortened hospital stays, new mothers are often discharged before they begin to experience symptoms of the baby blues or after birth depression. As part of the discharge teaching, the nurse can prepare the mother for this adjustment to her new role by instructing her regarding self-care activities to help prevent after birth depression. The most accurate statement as related to these activities is to: a. stay home and avoid outside activities to ensure adequate rest. b. be certain that you are the only caregiver for your baby to facilitate infant attachment. c. keep feelings of sadness and adjustment to your new role to yourself. d. realize that this is a common occurrence that affects many women. ANS: D Should the new mother experience symptoms of the baby blues, it is important that she be aware that this is nothing to be ashNaUmReSdINoGf.TUB.pCOtoM80% of women experience this type of mild depression after the birth of their infant. Although it is important for the mother to obtain enough rest, she should not distance herself from family and friends. Her spouse or partner can communicate the best visiting times so the new mother can obtain adequate rest. It is also important that she does not isolate herself at home during this time of role adjustment. Even if breastfeeding, other family members can participate in the infant's care. If depression occurs, the symptoms can often interfere with mothering functions, and this support will be essential. The new mother should share her feelings with someone else. It is also important that she not overcommit herself or think she has to be “superwoman.” A telephone call to the hospital warm line may provide reassurance with lactation issues and other infant care questions. Should symptoms continue, a referral to a professional therapist may be necessary. PTS: 1 DIF: Cognitive Level: Application OBJ: Nursing Process: Implementation MSC: Client Needs: Psychosocial Integrity 17. Complicated bereavement: a. occurs when, in multiple births, one child dies, and the other or others live. b. is a state in which the parents are ambivalent, as with an abortion. c. is an extremely intense grief reaction that persists for a long time. d. is felt by the family of adolescent mothers who lose their babies. ANS: C Parents showing signs of complicated grief should be referred for counseling. Multiple births in which not all the babies survive creates a complicated parenting situation, but this is not complicated bereavement. Abortion can generate complicated emotional responses, but they do not constitute complicated bereavement. Families of lost adolescent pregnancies may have to deal with complicated issues, but this is not complicated bereavement. PTS: 1 DIF: Cognitive Level: Comprehension OBJ: Nursing Process: Diagnosis MSC: Client Needs: Psychosocial Integrity 18. Early after birth hemorrhage is defined as a blood loss greater than: a. 500 mL in the first 24 hours after vaginal delivery. b. 750 mL in the first 24 hours after vaginal delivery. c. 1000 mL in the first 48 hours after cesarean delivery. d. 1500 mL in the first 48 hours after cesarean delivery. ANS: A The average amount of bleeding after a vaginal birth is 500 mL. Blood loss after a cesarean birth averages 1000 mL. Early after birth hemorrhage occurs in the first 24 hours, not 48 hours. Late after birth hemorrhage is 48 hours and later. PTS: 1 DIF: Cognitive Level: Knowledge OBJ: Nursing Process: Assessment MSC: Client Needs: Physiologic Integrity 19. A woman delivered a 9-lb, 10-ounce baby 1 hour ago. When you arrive to perform her 15-minute assessment, she tells you that she “feels all wet underneath.” You discover that both pads are completely saturated and that she is lying in a 6-inch-diameter puddle of blood. What is your first action? a. Call for help. NURSINGTB.COM b. Assess the fundus for firmness. c. Take her blood pressure. d. Check the perineum for lacerations. ANS: B Firmness of the uterus is necessary to control bleeding from the placental site. The nurse should first assess for firmness and massage the fundus as indicated. Assessing blood pressure is an important assessment with a bleeding patient; however, the top priority is to control the bleeding. If bleeding continues in the presence of a firm fundus, lacerations may be the cause. PTS: 1 DIF: Cognitive Level: Application OBJ: Nursing Process: Implementation MSC: Client Needs: Physiologic Integrity 20. A steady trickle of bright red blood from the vagina in the presence of a firm fundus suggests: a. uterine atony. b. lacerations of the genital tract. c. perineal hematoma. d. infection of the uterus. ANS: B Undetected lacerations will bleed slowly and continuously. Bleeding from lacerations is uncontrolled by uterine contraction. The fundus is not firm in the presence of uterine atony. A hematoma would develop internally. Swelling and discoloration would be noticeable; however, bright bleeding would not be. With an infection of the uterus there would be an odor to the lochia and systemic symptoms such as fever and malaise. PTS: 1 DIF: Cognitive Level: Comprehension OBJ: Nursing Process: Assessment MSC: Client Needs: Physiologic Integrity 21. Which instruction should be included in the discharge teaching plan to assist the patient in recognizing early signs of complications? a. Palpate the fundus daily to ensure that it is soft. b. Notify the physician of any increase in the amount of lochia or a return to bright red bleeding. c. Report any decrease in the amount of brownish red lochia. d. The passage of clots as large as an orange can be expected. ANS: B An increase in lochia or a return to bright red bleeding after the lochia has become pink indicates a complication. The fundus should stay firm. The lochia should decrease in amount over time. Large clots after discharge are a sign of complications and should be reported. PTS: 1 DIF: Cognitive Level: Application OBJ: Nursing Process: Implementation MSC: Client Needs: Health Promotion and Maintenance 22. If nonsurgical treatment for late after birth hemorrhage is ineffective, which surgical procedure is appropriate to correct the cause of this condition? a. Hysterectomy b. Laparoscopy c. Laparotomy d. D&C ANS: D NURSINGTB.COM D&C allows examination of the uterine contents and removal of any retained placental fragments or blood clots. Hysterectomy is the removal of the uterus and is not indicated for this condition. A laparoscopy is the insertion of an endoscope through the abdominal wall to examine the peritoneal cavity. It is not the appropriate treatment for this condition. A laparotomy is also not indicated for this condition. A laparotomy is a surgical incision into the peritoneal cavity to explore it. PTS: 1 DIF: Cognitive Level: Comprehension OBJ: Nursing Process: Assessment MSC: Client Needs: Physiologic Integrity 23. Which condition is a transient, self-limiting mood disorder that affects new mothers after childbirth? a. After birth depression b. After birth psychosis c. After birth bipolar disorder d. After birth blues ANS: D After birth blues or “baby blues” is a transient self-limiting disease that is believed to be related to hormonal fluctuations after childbirth. After birth depression is not the normal worries (blues) that many new mothers experience. Many caregivers believe that after birth depression is underdiagnosed and underreported. After birth psychosis is a rare condition that usually surfaces within 3 weeks of delivery. Hospitalization of the woman is usually necessary for treatment of this disorder. Bipolar disorder is one of the two categories of after birth psychosis, characterized by both manic and depressive episodes. PTS: 1 DIF: Cognitive Level: Knowledge OBJ: Nursing Process: Assessment MSC: Client Needs: Psychosocial Integrity 24. Anxiety disorders are the most common mental disorders that affect women. While providing care to the maternity patient, the nurse should be aware that one of these disorders is likely to be triggered by the process of labor and birth. This disorder is: a. phobias. b. panic disorder. c. posttraumatic stress disorder (PTSD). d. obsessive-compulsive disorder (OCD). ANS: C In PTSD, women perceive childbirth as a traumatic event. They have nightmares and flashbacks about the event, anxiety, and avoidance of reminders of the traumatic event. Phobias are irrational fears that may lead a person to avoid certain objects, events, or situations. Panic disorders include episodes of intense apprehension, fear, and terror. Symptoms may manifest themselves as palpitations, chest pain, choking, or smothering. OCD symptoms include recurrent, persistent, and intrusive thoughts. The mother may repeatedly check and recheck her infant onceNhUeRoSrINshGeTBis.CbOoMrn, even though she realizes that this is irrational. OCD is best treated with medications. PTS: 1 DIF: Cognitive Level: Knowledge OBJ: Nursing Process: Assessment MSC: Client Needs: Psychosocial Integrity MULTIPLE RESPONSE 1. Medications used to manage postpartum hemorrhage (PPH) include: (Select all that apply.) a. Pitocin. b. Methergine. c. Terbutaline. d. Hemabate. e. magnesium sulfate. ANS: A, B, D Pitocin, Methergine, and Hemabate are all used to manage PPH. Terbutaline and magnesium sulfate are tocolytics; relaxation of the uterus causes or worsens PPH. PTS: 1 DIF: Cognitive Level: Comprehension OBJ: Nursing Process: Implementation MSC: Client Needs: Physiologic Integrity MATCHING PPH may be sudden and result in rapid blood loss. The nurse must be alert to the symptoms of hemorrhage and hypovolemic shock and be prepared to act quickly to minimize blood loss. Astute assessment of circulatory status can be done with noninvasive monitoring. Please match the type of noninvasive assessment that the RN would perform with the appropriate clinical manifestation or body system. a. Palpation b. Auscultation c. Inspection d. Observation e. Measurement 1. Pulse oximetry 2. Heart sounds 3. Arterial pulses 4. Skin color, temperature, turgor 5. Presence or absence of anxiety 1. ANS: E PTS: 1 DIF: Cognitive Level: Application OBJ: Nursing Process: Assessment MSC: Client Needs: Physiologic Integrity NOT: To perform a complete noninvasive assessment of circulatory status in after birth patients who are bleeding, the nurse must perform the following: palpation (rate, quality, equality) of arterial pulses; auscultation of heart sounds/murmurs and breath sounds; inspection of skin color, temperature, and turgor, level of consciousness, capillary refill, neck veins, and mucous membranes; observation of either the presence or absence of anxiety, apprehension, restlessness, and disorientation; and measurement of blood pressure, pulse oximetry, and urinary output. 2. ANS: B PTS: 1 DIF: Cognitive Level: Application OBJ: Nursing Process: AssessmentNURSINMGSTCB.:CCOlMient Needs: Physiologic Integrity NOT: To perform a complete noninvasive assessment of circulatory status in after birth patients who are bleeding, the nurse must perform the following: palpation (rate, quality, equality) of arterial pulses; auscultation of heart sounds/murmurs and breath sounds; inspection of skin color, temperature, and turgor, level of consciousness, capillary refill, neck veins, and mucous membranes; observation of either the presence or absence of anxiety, apprehension, restlessness, and disorientation; and measurement of blood pressure, pulse oximetry, and urinary output. 3. ANS: A PTS: 1 DIF: Cognitive Level: Application OBJ: Nursing Process: Assessment MSC: Client Needs: Physiologic Integrity NOT: To perform a complete noninvasive assessment of circulatory status in after birth patients who are bleeding, the nurse must perform the following: palpation (rate, quality, equality) of arterial pulses; auscultation of heart sounds/murmurs and breath sounds; inspection of skin color, temperature, and turgor, level of consciousness, capillary refill, neck veins, and mucous membranes; observation of either the presence or absence of anxiety, apprehension, restlessness, and disorientation; and measurement of blood pressure, pulse oximetry, and urinary output. 4. ANS: C PTS: 1 DIF: Cognitive Level: Application OBJ: Nursing Process: Assessment MSC: Client Needs: Physiologic Integrity NOT: To perform a complete noninvasive assessment of circulatory status in after birth patients who are bleeding, the nurse must perform the following: palpation (rate, quality, equality) of arterial pulses; auscultation of heart sounds/murmurs and breath sounds; inspection of skin color, temperature, and turgor, level of consciousness, capillary refill, neck veins, and mucous membranes; observation of either the presence or absence of anxiety, apprehension, restlessness, and disorientation; and measurement of blood pressure, pulse oximetry, and urinary output. 5. ANS: D PTS: 1 DIF: Cognitive Level: Application OBJ: Nursing Process: Assessment MSC: Client Needs: Physiologic Integrity NOT: To perform a complete noninvasive assessment of circulatory status in after birth patients who are bleeding, the nurse must perform the following: palpation (rate, quality, equality) of arterial pulses; auscultation of heart sounds/murmurs and breath sounds; inspection of skin color, temperature, and turgor, level of consciousness, capillary refill, neck veins, and mucous membranes; observation of either the presence or absence of anxiety, apprehension, restlessness, and disorientation; and measurement of blood pressure, pulse oximetry, and urinary output. NURSINGTB.COM Chapter 22: Physiologic and Behavioral Adaptations of the Newborn Perry: Maternal Child Nursing Care, 6th Edition MULTIPLE CHOICE 1. A woman gave birth to a healthy 7-lb, 13-ounce infant girl. The nurse suggests that the woman place the infant to her breast within 15 minutes after birth. The nurse knows that breastfeeding is effective during the first 30 minutes after birth because this is the: a. transition period. b. first period of reactivity. c. organizational stage. d. second period of reactivity. ANS: B The first period of reactivity is the first phase of transition and lasts up to 30 minutes after birth. The infant is highly alert during this phase. The transition period is the phase between intrauterine and extrauterine existence. There is no such phase as the organizational stage. The second period of reactivity occurs roughly between 4 and 8 hours after birth, after a period of prolonged sleep. PTS: 1 DIF: Cognitive Level: Comprehension OBJ: Nursing Process: Planning MSC: Client Needs: Health Promotion and Maintenance 2. Part of the health assessment of a newborn is observing the infant's breathing pattern. A full-term newborn's breathing pattern is predominantly: a. abdominal with synchronous cNhUeRstSmINoGvTeBm.CeOntMs. b. chest breathing with nasal flaring. c. diaphragmatic with chest retraction. d. deep with a regular rhythm. ANS: A In normal infant respiration the chest and abdomen rise synchronously, and breaths are shallow and irregular. Breathing with nasal flaring is a sign of respiratory distress. Diaphragmatic breathing with chest retraction is a sign of respiratory distress. Infant breaths are not deep with a regular rhythm. PTS: 1 DIF: Cognitive Level: Comprehension OBJ: Nursing Process: Assessment MSC: Client Needs: Physiologic Integrity 3. While assessing the newborn, the nurse should be aware that the average expected apical pulse range of a full-term, quiet, alert newborn is: a. 80 to 100 beats/min. b. 100 to 120 beats/min. c. 120 to 160 beats/min. d. 150 to 180 beats/min. ANS: C The average infant heart rate while awake is 120 to 160 beats/min. The newborn's heart rate may be about 85 to 100 beats/min while sleeping. The infant's heart rate typically is a bit higher when alert but quiet. A heart rate of 150 to 180 beats/min is typical when the infant cries. PTS: 1 DIF: Cognitive Level: Comprehension OBJ: Nursing Process: Assessment MSC: Client Needs: Health Promotion and Maintenance 4. A newborn is placed under a radiant heat warmer, and the nurse evaluates the infant's body temperature every hour. Maintaining the newborn's body temperature is important for preventing: a. respiratory depression. b. cold stress. c. tachycardia. d. vasoconstriction. ANS: B Loss of heat must be controlled to protect the infant from the metabolic and physiologic effects of cold stress, and that is the primary reason for placing a newborn under a radiant heat warmer. Cold stress results in an increased respiratory rate and vasoconstriction. PTS: 1 DIF: Cognitive Level: Comprehension OBJ: Nursing Process: Assessment MSC: Client Needs: Physiologic Integrity 5. An African-American woman noticed some bruises on her newborn girl's buttocks. She asks the nurse who spanked her daughter. The nurse explains that these marks are called: a. lanugo. b. vascular nevi. c. nevus flammeus. d. Mongolian spots. ANS: D NURSINGTB.COM A Mongolian spot is a bluish black area of pigmentation that may appear over any part of the exterior surface of the body. It is more commonly noted on the back and buttocks and most frequently is seen on infants whose ethnic origins are Mediterranean, Latin American, Asian, or African. Lanugo is the fine, downy hair seen on a term newborn. A vascular nevus, commonly called a strawberry mark, is a type of capillary hemangioma. A nevus flammeus, commonly called a port-wine stain, is most frequently found on the face. PTS: 1 DIF: Cognitive Level: Comprehension OBJ: Nursing Process: Diagnosis MSC: Client Needs: Health Promotion and Maintenance 6. While examining a newborn, the nurse notes uneven skinfolds on the buttocks and a click when performing the Ortolani maneuver. The nurse recognizes these findings as a sign that the newborn probably has: a. polydactyly. b. clubfoot. c. hip dysplasia. d. webbing. ANS: C The Ortolani maneuver is used to detect the presence of hip dysplasia. Polydactyly is the presence of extra digits. Clubfoot (talipes equinovarus) is a deformity in which the foot turns inward and is fixed in a plantar-flexion position. Webbing, or syndactyly, is a fusing of the fingers or toes. PTS: 1 DIF: Cognitive Level: Knowledge OBJ: Nursing Process: Diagnosis MSC: Client Needs: Health Promotion and Maintenance 7. A new mother states that her infant must be cold because the baby's hands and feet are blue. The nurse explains that this is a common and temporary condition called: a. acrocyanosis. b. erythema neonatorum. c. harlequin color. d. vernix caseosa. ANS: A Acrocyanosis, or the appearance of slightly cyanotic hands and feet, is caused by vasomotor instability, capillary stasis, and a high hemoglobin level. Acrocyanosis is normal and appears intermittently over the first 7 to 10 days. Erythema toxicum (also called erythema neonatorum) is a transient newborn rash that resembles flea bites. The harlequin sign is a benign, transient color change in newborns. Half of the body is pale, and the other half is ruddy or bluish red with a line of demarcation. Vernix caseosa is a cheese-like, whitish substance that serves as a protective covering. PTS: 1 DIF: Cognitive Level: Knowledge OBJ: Nursing Process: Diagnosis MSC: Client Needs: Health Promotion and Maintenance 8. The nurse assessing a newborn knNoUwRsStIhNaGt TthBe.CmOoMst critical physiologic change required of the newborn is: a. closure of fetal shunts in the circulatory system. b. full function of the immune defense system at birth. c. maintenance of a stable temperature. d. initiation and maintenance of respirations. ANS: D The most critical adjustment of a newborn at birth is the establishment of respirations. The cardiovascular system changes markedly after birth as a result of fetal respiration, which reduces pulmonary vascular resistance to the pulmonary blood flow and initiates a chain of cardiac changes that support the cardiovascular system. The infant relies on passive immunity received from the mother for the first 3 months of life. After the establishment of respirations, heat regulation is critical to newborn survival. PTS: 1 DIF: Cognitive Level: Comprehension OBJ: Nursing Process: Assessment MSC: Client Needs: Physiologic Integrity 9. The parents of a newborn ask the nurse how much the newborn can see. The parents specifically want to know what type of visual stimuli they should provide for their newborn. The nurse responds to the parents by telling them: a. “Infants can see very little until about 3 months of age.” b. “Infants can track their parent's eyes and distinguish patterns; they prefer complex patterns.” c. “The infant's eyes must be protected. Infants enjoy looking at brightly colored stripes.” d. “It's important to shield the newborn's eyes. Overhead lights help them see better.” ANS: B “Infants can track their parent's eyes and distinguish patterns; they prefer complex patterns” is an accurate statement. Development of the visual system continues for the first 6 months of life. Visual acuity is difficult to determine, but the clearest visual distance for the newborn appears to be 19 cm. Infants prefer to look at complex patterns, regardless of the color. Infants prefer low illumination and withdraw from bright light. PTS: 1 DIF: Cognitive Level: Application OBJ: Nursing Process: Planning MSC: Client Needs: Health Promotion and Maintenance 10. While evaluating the reflexes of a newborn, the nurse notes that with a loud noise the newborn symmetrically abducts and extends his arms, his fingers fan out and form a “C” with the thumb and forefinger, and he has a slight tremor. The nurse would document this finding as a positive: a. tonic neck reflex. b. glabellar (Myerson) reflex. c. Babinski reflex. d. Moro reflex. ANS: D The characteristics displayed by the infant are associated with a positive Moro reflex. The tonic neck reflex occurs when the infant extends the leg on the side to which the infant's head simultaneously turns. The glabellaNrUrRefSlIeNxGiTs Be.lCicOitMed by tapping on the infant's head while the eyes are open. A characteristic response is blinking for the first few taps. The Babinski reflex occurs when the sole of the foot is stroked upward along the lateral aspect of the sole and then across the ball of the foot. A positive response occurs when all the toes hyperextend, with dorsiflexion of the big toe. PTS: 1 DIF: Cognitive Level: Comprehension OBJ: Nursing Process: Assessment MSC: Client Needs: Health Promotion and Maintenance 11. While assessing the integument of a 24-hour-old newborn, the nurse notes a pink, papular rash with vesicles superimposed on the thorax, back, and abdomen. The nurse should: a. notify the physician immediately. b. move the newborn to an isolation nursery. c. document the finding as erythema toxicum. d. take the newborn's temperature and obtain a culture of one of the vesicles. ANS: C Erythema toxicum (or erythema neonatorum) is a newborn rash that resembles flea bites. This is a normal finding that does not require notification of the physician, isolation of the newborn, or any additional interventions. PTS: 1 DIF: Cognitive Level: Application OBJ: Nursing Process: Assessment MSC: Client Needs: Health Promotion and Maintenance 12. A patient is warm and asks for a fan in her room for her comfort. The nurse enters the room to assess the mother and her infant and finds the infant unwrapped in his crib with the fan blowing over him on “high.” The nurse instructs the mother that the fan should not be directed toward the newborn and the newborn should be wrapped in a blanket. The mother asks why. The nurse's best response is: a. “Your baby may lose heat by convection, which means that he will lose heat from his body to the cooler ambient air. You should keep him wrapped and prevent cool air from blowing on him.” b. “Your baby may lose heat by conduction, which means that he will lose heat from his body to the cooler ambient air. You should keep him wrapped and prevent cool air from blowing on him.” c. “Your baby may lose heat by evaporation, which means that he will lose heat from his body to the cooler ambient air. You should keep him wrapped and prevent cool air from blowing on him.” d. “Your baby will get cold stressed easily and needs to be bundled up at all times.” ANS: A “Your baby may lose heat by convection, which means that he will lose heat from his body to the cooler ambient air. You should keep him wrapped and prevent cool air from blowing on him” is an accurate statement. Conduction is the loss of heat from the body surface to cooler surfaces, not air, in direct contact with the newborn. Evaporation is loss of heat that occurs when a liquid is converted into a vapor. In the newborn heat loss by evaporation occurs as a result of vaporization of moisture from the skin. Cold stress may occur from excessive heat loss, but this does not imply that the infant will become stressed if not bundled at all times. Furthermore, excessive bundling may result in a rise in the infant's temperature. PTS: 1 DIF: CognitiNveURLSevINelG: TABp.pClOicMation OBJ: Nursing Process: Implementation MSC: Client Needs: Health Promotion and Maintenance 13. A first-time father is changing the diaper of his 1-day-old daughter. He asks the nurse, “What is this black, sticky stuff in her diaper?” The nurse's best response is: a. “That's meconium, which is your baby's first stool. It's normal.” b. “That's transitional stool.” c. “That means your baby is bleeding internally.” d. “Oh, don't worry about that. It's okay.” ANS: A “That's meconium, which is your baby's first stool. It's normal” is an accurate statement and the most appropriate response. Transitional stool is greenish brown to yellowish brown and usually appears by the third day after initiation of feeding. “That means your baby is bleeding internally” is not accurate. “Oh, don't worry about that. It's okay” is not an appropriate statement. It is belittling to the father and does not educate him about the normal stool patterns of his daughter. PTS: 1 DIF: Cognitive Level: Application OBJ: Nursing Process: Implementation MSC: Client Needs: Health Promotion and Maintenance 14. The transition period between intrauterine and extrauterine existence for the newborn: a. consists of four phases, two reactive and two of decreased responses. b. is referred to as the neonatal period and lasts from birth to day 28 of life. c. applies to full-term births only. d. varies by socioeconomic status and the mother's age. ANS: B Changes begin right after birth; the cutoff time when the transition is considered over (although the baby keeps changing) is 28 days. The transition period has three phases: first reactivity, decreased response, and second reactivity. All newborns experience this transition regardless of age or type of birth. Although stress can cause variation in the phases, the mother's age and wealth do not disturb the pattern. PTS: 1 DIF: Cognitive Level: Comprehension OBJ: Nursing Process: Evaluation MSC: Client Needs: Health Promotion and Maintenance 15. Which statement describing the first phase of the transition period is inaccurate? a. It lasts no longer than 30 minutes. b. It is marked by spontaneous tremors, crying, and head movements. c. It includes the passage of meconium. d. It may involve the infant's suddenly sleeping briefly. ANS: D The first phase is an active phase in which the baby is alert. Decreased activity and sleep mark the second phase. The first phase is the shortest, lasting less than 30 minutes. Such exploratory behaviors include spontaneous startle reactions. In the first phase the newborn also produces saliva. PTS: 1 DIF: Cognitive Level: Comprehension OBJ: Nursing Process: Assessment MSC: Client Needs: Health Promotion and Maintenance 16. With regard to the respiratory devNelUoRpSmINenGtToBf.CthOeMnewborn, nurses should be aware that: a. the first gasping breath is an exaggerated respiratory reaction within 1 minute of birth. b. Newborns must expel the fluid from the respiratory system within a few minutes of birth. c. Newborns are instinctive mouth breathers. d. Seesaw respirations are no cause for concern in the first hour after birth. ANS: A The first breath produces a cry. Newborns continue to expel fluid for the first hour of life. Newborns are natural nose breathers; they may not have the mouth-breathing response to nasal blockage for 3 weeks. Seesaw respirations instead of normal abdominal respirations are not normal and should be reported. PTS: 1 DIF: Cognitive Level: Comprehension OBJ: Nursing Process: Planning MSC: Client Needs: Physiologic Integrity 17. With regard to the newborn's developing cardiovascular system, nurses should be aware that: a. the heart rate of a crying infant may rise to 120 beats/min. b. heart murmurs heard after the first few hours are cause for concern. c. the point of maximal impulse (PMI) often is visible on the chest wall. d. persistent bradycardia may indicate respiratory distress syndrome (RDS). ANS: C The newborn's thin chest wall often allows the PMI to be seen. The normal heart rate for infants who are not sleeping is 120 to 160 beats/min. However, a crying infant temporarily could have a heart rate of 180 beats/min. Heart murmurs during the first few days of life have no pathologic significance; an irregular heart rate past the first few hours should be evaluated further. Persistent tachycardia may indicate RDS; bradycardia may be a sign of congenital heart blockage. PTS: 1 DIF: Cognitive Level: Comprehension OBJ: Nursing Process: Assessment MSC: Client Needs: Health Promotion and Maintenance 18. By knowing about variations in infants' blood count, nurses can explain to their patients that: a. a somewhat lower than expected red blood cell count could be the result of delay in clamping the umbilical cord. b. the early high white blood cell (WBC) count is normal at birth and should decrease rapidly. c. platelet counts are higher than in adults for a few months. d. even a modest vitamin K deficiency means a problem with the ability of the blood to clot properly. ANS: B The WBC count is high on the first day of birth and then declines rapidly. Delayed clamping of the cord results in an increase in hemoglobin and the red blood cell count. The platelet count essentially is the same for newborns and adults. Clotting is sufficient to prevent hemorrhage unless the vitamin K deficiency is significant. PTS: 1 DIF: Cognitive Level: Comprehension OBJ: Nursing Process: Planning MSC: Client Needs: Health Promotion and Maintenance NURSINGTB.COM 19. What infant response to cool environmental conditions is either not effective or not available to them? a. Constriction of peripheral blood vessels b. Metabolism of brown fat c. Increased respiratory rates d. Unflexing from the normal position ANS: D The newborn's flexed position guards against heat loss because it reduces the amount of body surface exposed to the environment. The newborn's body is able to constrict the peripheral blood vessels to reduce heat loss. Burning brown fat generates heat. The respiratory rate may rise to stimulate muscular activity, which generates heat. PTS: 1 DIF: Cognitive Level: Comprehension OBJ: Nursing Process: Planning MSC: Client Needs: Physiologic Integrity 20. As related to the normal functioning of the renal system in newborns, nurses should be aware that: a. the pediatrician should be notified if the newborn has not voided in 24 hours. b. breastfed infants likely will void more often during the first days after birth. c. “Brick dust” or blood on a diaper is always a cause to notify the physician. d. weight loss from fluid loss and other normal factors should be made up in 4 to 7 days. ANS: A A newborn who has not voided in 24 hours may have any of a number of problems, some of which deserve the attention of the pediatrician. Formula-fed infants tend to void more frequently in the first 3 days; breastfed infants void less during this time because the mother's breast milk has not come in yet. Brick dust may be uric acid crystals; blood spotting could be caused by withdrawal of maternal hormones (pseudomenstruation) or a circumcision. The physician must be notified only if there is no apparent cause of bleeding. Weight loss from fluid loss may take 14 days to regain. PTS: 1 DIF: Cognitive Level: Comprehension OBJ: Nursing Process: Planning, Implementation MSC: Client Needs: Physiologic Integrity 21. With regard to the gastrointestinal (GI) system of the newborn, nurses should be aware that: a. the newborn's cheeks are full because of normal fluid retention. b. the nipple of the bottle or breast must be placed well inside the baby's mouth because teeth have been developing in utero, and one or more may even be through. c. regurgitation during the first day or two can be reduced by burping the infant and slightly elevating the baby's head. d. bacteria are already present in the infant's GI tract at birth because they traveled through the placenta. ANS: C Avoiding overfeeding can also reduce regurgitation. The newborn's cheeks are full because of well-developed sucking pads. Teeth do develop in utero, but the nipple is placed deep because the baby cannot move food from tNhUe RliSpIsNtGoTtBh.eCOphMarynx. Bacteria are not present at birth, but they soon enter through various orifices. PTS: 1 DIF: Cognitive Level: Comprehension OBJ: Nursing Process: Planning MSC: Client Needs: Health Promotion and Maintenance 22. Which statement describing physiologic jaundice is incorrect? a. Neonatal jaundice is common, but kernicterus is rare. b. The appearance of jaundice during the first 24 hours or beyond day 7 indicates a pathologic process. c. Because jaundice may not appear before discharge, parents need instruction on how to assess it and when to call for medical help. d. Breastfed babies have a lower incidence of jaundice. ANS: D Breastfeeding is associated with an increased incidence of jaundice. Neonatal jaundice occurs in 60% of newborns; the complication called kernicterus is rare. Jaundice in the first 24 hours or that persists past day 7 is cause for medical concern. Parents need to know how to assess for jaundice in their newborn. PTS: 1 DIF: Cognitive Level: Comprehension OBJ: Nursing Process: Diagnosis MSC: Client Needs: Health Promotion and Maintenance 23. The cheese-like, whitish substance that fuses with the epidermis and serves as a protective coating is called: a. vernix caseosa. b. surfactant. c. caput succedaneum. d. acrocyanosis. ANS: A This protection, vernix caseosa, is needed because the infant's skin is so thin. Surfactant is a protein that lines the alveoli of the infant's lungs. Caput succedaneum is the swelling of the tissue over the presenting part of the fetal head. Acrocyanosis is cyanosis of the hands and feet that results in a blue coloring. PTS: 1 DIF: Cognitive Level: Knowledge OBJ: Nursing Process: Assessment MSC: Client Needs: Health Promotion and Maintenance 24. What marks on a baby's skin may indicate an underlying problem that requires notification of a physician? a. Mongolian spots on the back b. Telangiectatic nevi on the nose or nape of the neck c. Petechiae scattered over the infant's body d. Erythema toxicum anywhere on the body ANS: C Petechiae (bruises) scattered over the infant's body should be reported to the pediatrician because they may indicate underlying problems. Mongolian spots are bluish-black spots that resemble bruises but fade gradually over months and have no clinical significance. Telangiectatic nevi (stork bites, angel kisses) fade by the second year and have no clinical significance. Erythema toxicum is an appalling-looking rash, but it has no clinical significance and requires no treatment. NURSINGTB.COM PTS: 1 DIF: Cognitive Level: Comprehension OBJ: Nursing Process: Assessment MSC: Client Needs: Physiologic Integrity 25. An examiner who discovers unequal movement or uneven gluteal skinfolds during the Ortolani maneuver would then: a. tell the parents that one leg may be longer than the other, but they will equal out by the time the infant is walking. b. alert the physician that the infant has a dislocated hip. c. inform the parents and physician that molding has not taken place. d. suggest that, if the condition does not change, surgery to correct vision problems may be needed. ANS: B The Ortolani maneuver is a technique for checking hip integrity. Unequal movement suggests that the hip is dislocated. The physician should be notified. PTS: 1 DIF: Cognitive Level: Application OBJ: Nursing Process: Implementation MSC: Client Needs: Health Promotion and Maintenance 26. One reason the brain is vulnerable to nutritional deficiencies and trauma in early infancy is the: a. incompletely developed neuromuscular system. b. primitive reflex system. c. presence of various sleep-wake states. d. cerebellum growth spurt. ANS: D The vulnerability of the brain likely is to the result of the cerebellum growth spurt. The neuromuscular system is almost completely developed at birth. The reflex system is not relevant. The various sleep-wake states are not relevant. PTS: 1 DIF: Cognitive Level: Analysis OBJ: Nursing Process: Diagnosis MSC: Client Needs: Health Promotion and Maintenance 27. The nurse caring for the newborn should be aware that the sensory system least mature at the time of birth is: a. vision. b. hearing. c. smell. d. taste. ANS: A The visual system continues to develop for the first 6 months. As soon as the amniotic fluid drains from the ear (minutes), the infant's hearing is similar to that of an adult. Newborns have a highly developed sense of smell. The newborn can distinguish and react to various tastes. PTS: 1 DIF: Cognitive Level: Knowledge OBJ: Nursing Process: Planning MSC: Client Needs: Health Promotion and Maintenance 28. During life in utero, oxygenation oNfUtRhSeINfeGtuTsBo.CcOcuMrs through transplacental gas exchange. When birth occurs, four factors combine to stimulate the respiratory center in the medulla. The initiation of respiration then follows. Which is not one of these essential factors? a. Chemical b. Mechanical c. Thermal d. Psychologic ANS: D Psychologic factor is not one of the essential factors in the initiation of breathing; the fourth factor is sensory. The sensory factors include handling by the provider, drying by the nurse, lights, smells, and sounds. Chemical factors are essential for the initiation of breathing. During labor, decreased levels of oxygen and increased levels of carbon dioxide seem to have a cumulative effect that is involved in the initiation of breathing. Clamping of the cord may also contribute to the start of respirations. Prostaglandins are known to inhibit breathing, and clamping of the cord results in a drop in the level of prostaglandins. Mechanical factors are also necessary to initiate respirations. As the infant passes through the birth canal, the chest is compressed. With birth the chest is relaxed, which allows for negative intrathoracic pressure that encourages air to flow into the lungs. The profound change in temperature between intrauterine and extrauterine life stimulates receptors in the skin to communicate with the receptors in the medulla. This also contributes to the initiation of breathing. PTS: 1 DIF: Cognitive Level: Comprehension OBJ: Nursing Process: Implementation MSC: Client Needs: Physiologic Integrity 29. A collection of blood between the skull bone and its periosteum is known as a cephalhematoma. To reassure the new parents whose infant develops such a soft bulge, it is important that the nurse be aware that this condition: a. may occur with spontaneous vaginal birth. b. happens only as the result of a forceps or vacuum delivery. c. is present immediately after birth. d. will gradually absorb over the first few months of life. ANS: A Bleeding may occur during a spontaneous vaginal delivery as a result of the pressure against the maternal bony pelvis. The soft, irreducible fullness does not pulsate or bulge when the infant cries. Low forceps and other difficult extractions may result in bleeding. However, cephalhematomas can also occur spontaneously. The swelling may appear unilaterally or bilaterally and is usually minimal or absent at birth. It increases over the first 2 to 3 days of life. Cephalhematomas disappear gradually over 2 to 3 weeks. A less common condition results in calcification of the hematoma, which may persist for months. PTS: 1 DIF: Cognitive Level: Knowledge OBJ: Nursing Process: Planning MSC: Client Needs: Health Promotion and Maintenance 30. A nursing student is helping the nursery nurses with morning vital signs. A baby born 10 hours ago by cesarean section is found to have moist lung sounds. What is the best interpretation of these data? a. The nurse should notify the pediatrician stat for this emergency situation. b. The neonate must have aspirated surfactant. c. If this baby was born vaginally, it could indicate a pneumothorax. d. The lungs of a baby deliveredNbUyRcSeIsNaGreTaBn.CsOecMtion may sound moist for 24 hours after birth. ANS: D The condition will resolve itself within a few hours. For this common condition of newborns, surfactant acts to keep the expanded alveoli partially open between respirations. In vaginal births, absorption of remaining lung fluid is accelerated by the process of labor and delivery. Remaining lung fluid will move into interstitial spaces and be absorbed by the circulatory and lymphatic systems. This is a particularly common condition for infants delivered by cesarean section. Surfactant is produced by the lungs, so aspiration is not a concern. PTS: 1 DIF: Cognitive Level: Comprehension OBJ: Nursing Process: Assessment MSC: Client Needs: Health Promotion and Maintenance 31. Nurses can prevent evaporative heat loss in the newborn by: a. drying the baby after birth and wrapping the baby in a dry blanket. b. keeping the baby out of drafts and away from air conditioners. c. placing the baby away from the outside wall and the windows. d. warming the stethoscope and the nurse's hands before touching the baby. ANS: A Because the infant is wet with amniotic fluid and blood, heat loss by evaporation occurs quickly. Heat loss by convection occurs when drafts come from open doors and air currents created by people moving around. If the heat loss is caused by placing the baby near cold surfaces or equipment, it is referred to as a radiation heat loss. Conduction heat loss occurs when the baby comes in contact with cold objects or surfaces. PTS: 1 DIF: Cognitive Level: Comprehension OBJ: Nursing Process: Assessment MSC: Client Needs: Physiologic Integrity 32. A first-time dad is concerned that his 3-day-old daughter's skin looks “yellow.” In the nurse's explanation of physiologic jaundice, what fact should be included? a. Physiologic jaundice occurs during the first 24 hours of life. b. Physiologic jaundice is caused by blood incompatibilities between the mother and infant blood types. c. The bilirubin levels of physiologic jaundice peak between 72 to 96 hours of life. d. This condition is also known as “breast milk jaundice.” ANS: C Physiologic jaundice becomes visible when the serum bilirubin reaches a level of 5 mg/dL or greater, which occurs when the baby is approximately 3 days old. This finding is within normal limits for the newborn. Pathologic jaundice occurs during the first 24 hours of life. Pathologic jaundice is caused by blood incompatibilities, causing excessive destruction of erythrocytes, and must be investigated. Breast milk jaundice occurs in one third of breastfed infants at 2 weeks and is caused by an insufficient intake of fluids. PTS: 1 DIF: Cognitive Level: Knowledge OBJ: Nursing Process: Implementation MSC: Client Needs: Health Promotion and Maintenance NURSINGTB.COM 33. Cardiovascular changes that cause the foramen ovale to close at birth are a direct result of: a. increased pressure in the right atrium. b. increased pressure in the left atrium. c. decreased blood flow to the left ventricle. d. changes in the hepatic blood flow. ANS: B With the increase in the blood flow to the left atrium from the lungs, the pressure is increased, and the foramen ovale is functionally closed. The pressure in the right atrium decreases at birth. It is higher during fetal life. Blood flow increases to the left ventricle after birth. The hepatic blood flow changes, but that is not the reason for the closure of the foramen ovale. PTS: 1 DIF: Cognitive Level: Knowledge OBJ: Nursing Process: Assessment MSC: Client Needs: Physiologic Integrity 34. The nurse should immediately alert the physician when: a. the infant is dusky and turns cyanotic when crying. b. acrocyanosis is present at age 1 hour. c. the infant's blood glucose level is 45 mg/dL. d. the infant goes into a deep sleep at age 1 hour. ANS: A An infant who is dusky and becomes cyanotic when crying is showing poor adaptation to extrauterine life. Acrocyanosis is an expected finding during the early neonatal life. This is within normal range for a newborn. Infants enter the period of deep sleep when they are about 1 hour old. PTS: 1 DIF: Cognitive Level: Application OBJ: Nursing Process: Assessment MSC: Client Needs: Physiologic Integrity 35. While assessing the newborn, the nurse should be aware that the average expected apical pulse range of a full-term, quiet, alert newborn is: a. 80 to 100 beats/min. b. 100 to 120 beats/min. c. 120 to 160 beats/min. d. 150 to 180 beats/min. ANS: C The average infant heart rate while awake is 120 to 160 beats/min. The newborn's heart rate may be about 85 to 100 beats/min while sleeping. The infant's heart rate typically is a bit higher when alert but quiet. A heart rate of 150 to 180 beats/min is typical when the infant cries. PTS: 1 DIF: Cognitive Level: Comprehension OBJ: Nursing Process: Assessment MSC: Client Needs: Health Promotion and Maintenance 36. In administering vitamin K to the infant shortly after birth, the nurse understands that vitamin K is: a. important in the production of red blood cells. b. necessary in the production ofNpUlaRtSeIlNetGs.TB.COM c. not initially synthesized because of a sterile bowel at birth. d. responsible for the breakdown of bilirubin and prevention of jaundice. ANS: C The bowel is initially sterile in the newborn, and vitamin K cannot be synthesized until food is introduced into the bowel. Vitamin K is necessary to activate blood clotting factors. The platelet count in term newborns is near adult levels. Vitamin K is necessary to activate prothrombin and other clotting factors. PTS: 1 DIF: Cognitive Level: Comprehension OBJ: Nursing Process: Implementation MSC: Client Needs: Physiologic Integrity 37. A meconium stool can be differentiated from a transitional stool in the newborn because the meconium stool is: a. seen at age 3 days. b. the residue of a milk curd. c. passed in the first 12 hours of life. d. lighter in color and looser in consistency. ANS: C Meconium stool is usually passed in the first 12 hours of life, and 99% of newborns have their first stool within 48 hours. If meconium is not passed by 48 hours, obstruction is suspected. Meconium stool is the first stool of the newborn and is made up of matter remaining in the intestines during intrauterine life. Meconium is dark and sticky. PTS: 1 DIF: Cognitive Level: Knowledge OBJ: Nursing Process: Assessment MSC: Client Needs: Health Promotion and Maintenance 38. The process in which bilirubin is changed from a fat-soluble product to a water-soluble product is known as: a. enterohepatic circuit. b. conjugation of bilirubin. c. unconjugation of bilirubin. d. albumin binding. ANS: B Conjugation of bilirubin is the process of changing the bilirubin from a fat-soluble to a water-soluble product. This is the route by which part of the bile produced by the liver enters the intestine, is reabsorbed by the liver, and then is recycled into the intestine. Unconjugated bilirubin is fat soluble. Albumin binding is to attach something to a protein molecule. PTS: 1 DIF: Cognitive Level: Knowledge OBJ: Nursing Process: Assessment MSC: Client Needs: Physiologic Integrity 39. Which newborn reflex is elicited by stroking the lateral sole of the infant's foot from the heel to the ball of the foot? a. Babinski b. Tonic neck c. Stepping d. Plantar grasp ANS: A NURSINGTB.COM The Babinski reflex causes the toes to flare outward and the big toe to dorsiflex. The tonic neck reflex (also called the fencing reflex) refers to the posture assumed by newborns when in a supine position. The stepping reflex occurs when infants are held upright with their heel touching a solid surface and the infant appears to be walking. Plantar grasp reflex is similar to the palmar grasp reflex: when the area below the toes is touched, the infant's toes curl over the nurse's finger. PTS: 1 DIF: Cognitive Level: Knowledge OBJ: Nursing Process: Assessment MSC: Client Needs: Health Promotion and Maintenance 40. Infants in whom cephalhematomas develop are at increased risk for: a. infection. b. jaundice. c. caput succedaneum. d. erythema toxicum. ANS: B Cephalhematomas are characterized by bleeding between the bone and its covering, the periosteum. Because of the breakdown of the red blood cells within a hematoma, the infants are at greater risk for jaundice. Cephalhematomas do not increase the risk for infections. Caput is an edematous area on the head from pressure against the cervix. Erythema toxicum is a benign rash of unknown cause that consists of blotchy red areas. PTS: 1 DIF: Cognitive Level: Application OBJ: Nursing Process: Assessment MSC: Client Needs: Physiologic Integrity MULTIPLE RESPONSE 1. What are modes of heat loss in the newborn? (Select all that apply.) a. Perspiration b. Convection c. Radiation d. Conduction e. Urination ANS: B, C, D Convection, radiation, evaporation, and conduction are the four modes of heat loss in the newborn. Perspiration and urination are not modes of heat loss in newborns. PTS: 1 DIF: Cognitive Level: Analysis OBJ: Nursing Process: Diagnosis MSC: Client Needs: Health Promotion and Maintenance MATCHING The healthy infant must accomplish both behavioral and biologic tasks to develop normally. Behavioral characteristics form the basis of the social capabilities of the infant. Newborns pass through a hierarchy of developmental challenges as they adapt to their environment and caregivers. This progression in behavior is the basis for the Brazelton Neonatal Behavioral Assessment (NBAS). Please match the cluster of neonatal behavior with the correct level on the NBAS scale. a. Habituation b. Orientation c. Range of state d. Autonomic stability e. Regulation of state NURSINGTB.COM 1. Signs of stress related to homeostatic adjustment 2. Ability to respond to discrete stimuli while asleep 3. Measure of general arousability 4. How the infant responds when aroused 5. Ability to attend to visual and auditory stimuli while alert 1. ANS: D PTS: 1 DIF: Cognitive Level: Application OBJ: Nursing Process: Assessment MSC: Client Needs: Physiologic Integrity NOT: Other clusters of neonatal behavior include motor performance, quality of movement and tone and reflexes, and assessment of neonatal reflexes. 2. ANS: A PTS: 1 DIF: Cognitive Level: Application OBJ: Nursing Process: Assessment MSC: Client Needs: Physiologic Integrity NOT: Other clusters of neonatal behavior include motor performance, quality of movement and tone and reflexes, and assessment of neonatal reflexes. 3. ANS: C PTS: 1 DIF: Cognitive Level: Application OBJ: Nursing Process: Assessment MSC: Client Needs: Physiologic Integrity NOT: Other clusters of neonatal behavior include motor performance, quality of movement and tone and reflexes, and assessment of neonatal reflexes. 4. ANS: E PTS: 1 DIF: Cognitive Level: Application OBJ: Nursing Process: Assessment MSC: Client Needs: Physiologic Integrity NOT: Other clusters of neonatal behavior include motor performance, quality of movement and tone and reflexes, and assessment of neonatal reflexes. 5. ANS: B PTS: 1 DIF: Cognitive Level: Application OBJ: Nursing Process: Assessment MSC: Client Needs: Physiologic Integrity NOT: Other clusters of neonatal behavior include motor performance, quality of movement and tone and reflexes, and assessment of neonatal reflexes. NURSINGTB.COM Chapter 23: Nursing Care of the Newborn and Family Perry: Maternal Child Nursing Care, 6th Edition MULTIPLE CHOICE 1. An infant boy was born just a few minutes ago. The nurse is conducting the initial assessment. Part of the assessment includes the Apgar score. The Apgar assessment is performed: a. only if the newborn is in obvious distress. b. once by the obstetrician, just after the birth. c. at least twice, 1 minute and 5 minutes after birth. d. every 15 minutes during the newborn's first hour after birth. ANS: C Apgar scoring is performed at 1 minute and 5 minutes after birth. Scoring may continue at 5-minute intervals if the infant is in distress and requires resuscitation efforts. PTS: 1 DIF: Cognitive Level: Comprehension OBJ: Nursing Process: Assessment MSC: Client Needs: Health Promotion and Maintenance 2. A new father wants to know what medication was put into his infant's eyes and why it is needed. The nurse explains to the father that the purpose of the erythromycin ophthalmic ointment is to: a. destroy an infectious exudate caused by Staphylococcus that could make the infant blind. b. prevent gonorrheal and chlamydial infection of the infant's eyes potentially acquired from the birth canal. NURSINGTB.COM c. prevent potentially harmful exudate from invading the tear ducts of the infant's eyes, leading to dry eyes. d. prevent the infant's eyelids from sticking together and help the infant see. ANS: B The purpose of the erythromycin ophthalmic ointment is to prevent gonorrheal and chlamydial infection of the infant's eyes potentially acquired from the birth canal. Prophylactic ophthalmic ointment is instilled in the eyes of all neonates to prevent gonorrheal or chlamydial infection. Prophylactic ophthalmic ointment is not instilled to prevent dry eyes. Prophylactic ophthalmic ointment has no bearing on vision other than to protect against infection that may lead to vision problems. PTS: 1 DIF: Cognitive Level: Comprehension OBJ: Nursing Process: Planning MSC: Client Needs: Health Promotion and Maintenance 3. A 3.8-kg infant was delivered vaginally at 39 weeks after a 30-minute second stage. There was a nuchal cord. After birth the infant is noted to have petechiae over the face and upper back. Information given to the infant's parents should be based on the knowledge that petechiae: a. are benign if they disappear within 48 hours of birth. b. result from increased blood volume. c. should always be further investigated. d. usually occur with forceps delivery. ANS: A Petechiae, or pinpoint hemorrhagic areas, acquired during birth may extend over the upper portion of the trunk and face. These lesions are benign if they disappear within 2 days of birth and no new lesions appear. Petechiae may result from decreased platelet formation. In this situation the presence of petechiae is most likely a soft-tissue injury resulting from the nuchal cord at birth. Unless they do not dissipate in 2 days, there is no reason to alarm the family. Petechiae usually occur with a breech presentation vaginal birth. PTS: 1 DIF: Cognitive Level: Application OBJ: Nursing Process: Assessment MSC: Client Needs: Health Promotion and Maintenance 4. A newborn is jaundiced and receiving phototherapy via ultraviolet bank lights. An appropriate nursing intervention when caring for an infant with hyperbilirubinemia and receiving phototherapy by this method would be to: a. apply an oil-based lotion to the newborn's skin to prevent dying and cracking. b. limit the newborn's intake of milk to prevent nausea, vomiting, and diarrhea. c. place eye shields over the newborn's closed eyes. d. change the newborn's position every 4 hours. ANS: C The infant's eyes must be protected by an opaque mask to prevent overexposure to the light. Eye shields should cover the eyes completely but not occlude the nares. Lotions and ointments should not be applied to the infant because they absorb heat, and this can cause burns. The lights increase insensible water loss, placing the infant at risk for fluid loss and dehydration. Therefore, it is important that the infant be adequately hydrated. The infant should be turned every 2 hours to expose all body surfaces to the light. PTS: 1 DIF: CognitiNveURLSevINelG: TABp.pClOicMation OBJ: Nursing Process: Planning MSC: Client Needs: Safe and Effective Care Environment 5. Early this morning, an infant boy was circumcised using the PlastiBell method. The nurse tells the mother that she and the infant can be discharged after: a. the bleeding stops completely. b. yellow exudate forms over the glans. c. the PlastiBell rim falls off. d. the infant voids. ANS: D The infant should be observed for urination after the circumcision. Bleeding is a common complication after circumcision. The nurse will check the penis for 12 hours after a circumcision to assess and provide appropriate interventions for prevention and treatment of bleeding. Yellow exudates cover the glans penis in 24 hours after the circumcision. This is part of normal healing and not an infective process. The PlastiBell remains in place for about a week and falls off when healing has taken place. PTS: 1 DIF: Cognitive Level: Comprehension OBJ: Nursing Process: Planning MSC: Client Needs: Health Promotion and Maintenance 6. A mother expresses fear about changing her infant's diaper after he is circumcised. What does the woman need to be taught to take care of the infant when she gets home? a. Cleanse the penis with prepackaged diaper wipes every 3 to 4 hours. b. Apply constant, firm pressure by squeezing the penis with the fingers for at least 5 minutes if bleeding occurs. c. Cleanse the penis gently with water and put petroleum jelly around the glans after each diaper change. d. Wash off the yellow exudate that forms on the glans at least once every day to prevent infection. ANS: C Cleansing the penis gently with water and putting petroleum jelly around the glans after each diaper change are appropriate when caring for an infant who has had a circumcision. With each diaper change, the penis should be washed off with warm water to remove any urine or feces. If bleeding occurs, the nurse should apply gentle pressure to the site of the bleeding with a sterile gauze square. Yellow exudates cover the glans penis in 24 hours after the circumcision. This is part of normal healing and not an infective process. The exudates should not be removed. PTS: 1 DIF: Cognitive Level: Application OBJ: Nursing Process: Planning MSC: Client Needs: Health Promotion and Maintenance 7. When preparing to administer a hepatitis B vaccine to a newborn, the nurse should: a. obtain a syringe with a 25-gauge, 5/8-inch needle. b. confirm that the newborn's mother has been infected with the hepatitis B virus. c. assess the dorsogluteal muscle as the preferred site for injection. d. confirm that the newborn is at least 24 hours old. ANS: A The hepatitis B vaccine should be administered with a 25-gauge, 5/8-inch needle. Hepatitis B vaccination is recommended for aNllUinRfSaInNtGs.TIBf.CthOeMinfant is born to an infected mother who is a chronic carrier, hepatitis vaccine and hepatitis B immune globulin should be administered within 12 hours of birth. Hepatitis B vaccine should be given in the vastus lateralis muscle. Hepatitis B vaccine can be given at birth. PTS: 1 DIF: Cognitive Level: Application OBJ: Nursing Process: Implementation MSC: Client Needs: Health Promotion and Maintenance 8. The nurse is performing a gestational age and physical assessment on the newborn. The infant appears to have an excessive amount of saliva. The nurse recognizes that this finding: a. is normal. b. indicates that the infant is hungry. c. may indicate that the infant has a tracheoesophageal fistula or esophageal atresia. d. may indicate that the infant has a diaphragmatic hernia. ANS: C The presence of excessive saliva in a neonate should alert the nurse to the possibility of tracheoesophageal fistula or esophageal atresia. PTS: 1 DIF: Cognitive Level: Analysis OBJ: Nursing Process: Assessment MSC: Client Needs: Physiologic Integrity 9. As part of Standard Precautions, nurses wear gloves when handling the newborn. The chief reason is: a. to protect the baby from infection. b. that it is part of the Apgar protocol. c. to protect the nurse from contamination by the newborn. d. the nurse has primary responsibility for the baby during the first 2 hours. ANS: C Gloves are worn to protect the nurse from infection until the blood and amniotic fluid are cleaned off the newborn. PTS: 1 DIF: Cognitive Level: Comprehension OBJ: Nursing Process: Implementation MSC: Client Needs: Safe and Effective Care Environment 10. The nurse's initial action when caring for an infant with a slightly decreased temperature is to: a. notify the physician immediately. b. place a cap on the infant's head. c. tell the mother that the infant must be kept in the nursery and observed for the next 4 hours. d. change the formula because this is a sign of formula intolerance. ANS: B Keeping the head well covered with a cap will prevent further heat loss from the head, and having the mother place the infant skin to skin should increase the infant's temperature. Nursing actions are needed first to correct the problem. If the problem persists after interventions, notification may then be necessary. A slightly decreased temperature can be treated in the mother's room. This would be an excellent time for parent teaching on prevention of cold stress. Mild temperature instability is an expected deviation from normal during the first days as the infant adapts to external life. NURSINGTB.COM PTS: 1 DIF: Cognitive Level: Application OBJ: Nursing Process: Implementation MSC: Client Needs: Health Promotion and Maintenance 11. An Apgar score of 10 at 1 minute after birth would indicate a(n): a. infant having no difficulty adjusting to extrauterine life and needing no further testing. b. infant in severe distress who needs resuscitation. c. prediction of a future free of neurologic problems. d. infant having no difficulty adjusting to extrauterine life but who should be assessed again at 5 minutes after birth. ANS: D An initial Apgar score of 10 is a good sign of healthy adaptation; however, it must be repeated at the 5-minute mark. PTS: 1 DIF: Cognitive Level: Comprehension OBJ: Nursing Process: Planning MSC: Client Needs: Physiologic Integrity 12. With regard to umbilical cord care, nurses should be aware that: a. the stump can easily become infected. b. a nurse noting bleeding from the vessels of the cord should immediately call for assistance. c. the cord clamp is removed at cord separation. d. the average cord separation time is 5 to 7 days. ANS: A The cord stump is an excellent medium for bacterial growth. The nurse should first check the clamp (or tie) and apply a second one. If the bleeding does not stop, the nurse calls for assistance. The cord clamp is removed after 24 hours when it is dry. The average cord separation time is 10 to 14 days. PTS: 1 DIF: Cognitive Level: Comprehension OBJ: Nursing Process: Planning MSC: Client Needs: Safe and Effective Care Environment 13. In the classification of newborns by gestational age and birth weight, the appropriate for gestational age (AGA) weight would: a. fall between the 25th and 75th percentiles for the infant's age. b. depend on the infant's length and the size of the head. c. fall between the 10th and 90th percentiles for the infant's age. d. be modified to consider intrauterine growth restriction (IUGR). ANS: C The AGA range is large: between the 10th and the 90th percentiles for the infant's age. The infant's length and size of the head are measured, but they do not affect the normal weight designation. IUGR applies to the fetus, not the newborn's weight. PTS: 1 DIF: Cognitive Level: Comprehension OBJ: Nursing Process: Diagnosis MSC: Client Needs: Health Promotion and Maintenance 14. During the complete physical examination 24 hours after birth: a. the parents are excused to reduce their normal anxiety. b. the nurse can gauge the neonaNteU'sRmSINatGuTriBty.ClOevMel by assessing the infant's general appearance. c. once often neglected, blood pressure is now routinely checked. d. when the nurse listens to the heart, the S1 and S2 sounds can be heard; the first sound is somewhat higher in pitch and sharper than the second. ANS: B The nurse will be looking at skin color, alertness, cry, head size, and other features. The parents' presence actively involves them in child care and gives the nurse a chance to observe interactions. Blood pressure is not usually taken unless cardiac problems are suspected. The second sound is higher and sharper than the first. PTS: 1 DIF: Cognitive Level: Comprehension OBJ: Nursing Process: Assessment MSC: Client Needs: Health Promotion and Maintenance 15. As related to laboratory tests and diagnostic tests in the hospital after birth, nurses should be aware that: a. all states test for phenylketonuria (PKU), hypothyroidism, cystic fibrosis, and sickle cell diseases. b. federal law prohibits newborn genetic testing without parental consent. c. if genetic screening is done before the infant is 24 hours old, it should be repeated at age 1 to 2 weeks. d. hearing screening is now mandated by federal law. ANS: C If done very early, genetic screening should be repeated. States all test for PKU and hypothyroidism, but other genetic defects are not universally covered. Federal law mandates newborn genetic screening, but not screening for hearing problems (although more than half the states do mandate hearing screening). PTS: 1 DIF: Cognitive Level: Knowledge OBJ: Nursing Process: Planning MSC: Client Needs: Physiologic Integrity 16. As part of their teaching function at discharge, nurses should educate parents regarding safe sleep. Which statement is incorrect? a. Prevent exposure to people with upper respiratory tract infections. b. Keep the infant away from secondhand smoke. c. Avoid loose bedding, water beds, and beanbag chairs. d. Place the infant on his or her abdomen to sleep. ANS: D The infant should be laid down to sleep on his or her back for better breathing and to prevent sudden infant death syndrome. Infants are vulnerable to respiratory infections; infected people must be kept away. Secondhand smoke can damage lungs. Infants can suffocate in loose bedding, and furniture that can trap them. Per AAP guidelines, infants should always be placed “back to sleep” and allowed tummy time to play, to prevent plagiocephaly. PTS: 1 DIF: Cognitive Level: Comprehension OBJ: Nursing Process: Planning MSC: Client Needs: Safe and Effective Care Environment 17. The normal term infant has little difficulty clearing the airway after birth. Most secretions are brought up to the oropharynx by the cough reflex. However, if the infant has excess secretions, the mouth and nasal paNsUsaRgSeIsNcGaTnBb.CeOcMleared easily with a bulb syringe. When instructing parents on the correct use of this piece of equipment, it is important that the nurse teach them to: a. avoid suctioning the nares. b. insert the compressed bulb into the center of the mouth. c. suction the mouth first. d. remove the bulb syringe from the crib when finished. ANS: C The mouth should be suctioned first to prevent the infant from inhaling pharyngeal secretions by gasping as the nares are suctioned. The nasal passages should be suctioned one nostril at a time. After compression of the bulb it should be inserted into one side of the mouth. If the bulb is inserted into the center of the mouth, the gag reflex is likely to be initiated. When the infant's cry no longer sounds as though it is through mucus or a bubble, suctioning can be stopped. The bulb syringe should remain in the crib so that it is easily accessible if needed again. PTS: 1 DIF: Cognitive Level: Application OBJ: Nursing Process: Implementation MSC: Client Needs: Health Promotion and Maintenance 18. When teaching parents about mandatory newborn screening, it is important for the nurse to explain that the main purpose is to: a. keep the state records updated. b. allow accurate statistical information. c. document the number of births. d. recognize and treat newborn disorders early. ANS: D Early treatment of disorders will prevent morbidity associated with inborn errors of metabolism or other genetic conditions. Keeping records and reporting for statistical purposes are not the primary reason for the screening test. The number of births recorded is not reported from the newborn screening test. PTS: 1 DIF: Cognitive Level: Application OBJ: Nursing Process: Assessment MSC: Client Needs: Physiologic Integrity 19. To prevent the abduction of newborns from the hospital, the nurse should: a. instruct the mother not to give her infant to anyone except the one nurse assigned to her that day. b. apply an electronic and identification bracelet to mother and infant. c. carry the infant when transporting him or her in the halls. d. restrict the amount of time infants are out of the nursery. ANS: B A measure taken by many facilities is to band both the mother and the baby with matching identification bracelets and band the infant with an electronic device that will alarm if the infant is removed from the maternity unit. It is impossible for one nurse to be on call for one mother and baby for the entire shift, so parents need to be able to identify the nurses who are working on the unit. Infants should always be transported in their bassinette, for both safety and security reasons. All maternity unit nursing staff should have unique identification bracelets in comparison with the rest of the hospital. Infants should remain with their parents and spend as little time in the nursNeUryRSasINpGoTsBsi.bClOe.M PTS: 1 DIF: Cognitive Level: Comprehension OBJ: Nursing Process: Implementation MSC: Client Needs: Safe and Effective Care Environment 20. The nurse administers vitamin K to the newborn for which reason? a. Most mothers have a diet deficient in vitamin K, which results in the infant's being deficient. b. Vitamin K prevents the synthesis of prothrombin in the liver and must be given by injection. c. Bacteria that synthesize vitamin K are not present in the newborn's intestinal tract. d. The supply of vitamin K is inadequate for at least 3 to 4 months, and the newborn must be supplemented. ANS: C Bacteria that synthesize vitamin K are not present in the newborn's intestinal tract. Vitamin K is provided because the newborn does not have the intestinal flora to produce this vitamin for the first week. The maternal diet has no bearing on the amount of vitamin K found in the newborn. Vitamin K promotes the formation of clotting factors in the liver and is used for the prevention and treatment of hemorrhagic disease in the newborn. Vitamin K is not produced in the intestinal tract of the newborn until after microorganisms are introduced. By day 8, normal newborns are able to produce their own vitamin K. PTS: 1 DIF: Cognitive Level: Comprehension OBJ: Nursing Process: Implementation MSC: Client Needs: Health Promotion and Maintenance 21. Nursing follow-up care often includes home visits for the new mother and her infant. Which information related to home visits is correct? a. Ideally, the visit is scheduled within 72 hours after discharge. b. Home visits are available in all areas. c. Visits are completed within a 30-minute time frame. d. Blood draws are not a part of the home visit. ANS: A The home visit is ideally scheduled within 72 hours after discharge. This timing allows early assessment and intervention for problems with feedings, jaundice, newborn adaptation, and maternal-infant interaction. Because home visits are expensive, they are not available in all geographic areas. Visits are usually 60 to 90 minutes in length to allow enough time for assessment and teaching. When jaundice is found, the nurse can discuss the implications and check the transcutaneous bilirubin level or draw blood for testing. PTS: 1 DIF: Cognitive Level: Application OBJ: Nursing Process: Planning MSC: Client Needs: Health Promotion and Maintenance MULTIPLE RESPONSE 1. Pain should be assessed regularly in all newborn infants. If the infant is displaying physiologic or behavioral cues indicating pain, measures should be taken to manage the pain. Examples of nonpharmacologic pain management techniques include: (Select all that apply.) a. swaddling. b. nonnutritive sucking. NURSINGTB.COM c. skin-to-skin contact with the mother. d. sucrose. e. acetaminophen. ANS: A, B, C, D Swaddling, nonnutritive sucking, skin-to-skin contact with the mother, and sucrose are all appropriate nonpharmacologic techniques used to manage pain in neonates. Acetaminophen is a pharmacologic method of treating pain. PTS: 1 DIF: Cognitive Level: Comprehension OBJ: Nursing Process: Implementation MSC: Client Needs: Physiologic Integrity 2. Hearing loss is one of the genetic disorders included in the universal screening program. Auditory screening of all newborns within the first month of life is recommended by the American Academy of Pediatrics. Reasons for having this testing performed include: (Select all that apply.) a. prevention or reduction of developmental delay. b. reassurance for concerned new parents. c. early identification and treatment. d. helping the child communicate better. e. recommendation by the Joint Committee on Infant Hearing. ANS: A, C, D, E New parents are often anxious regarding this test and the impending results; however, it is not the reason for the screening to be performed. Auditory screening is usually done before hospital discharge. It is important for the nurse to ensure that the infant receives the appropriate testing and that the test is fully explained to the parents. For infants who are referred for further testing and follow-up, it is important for the nurse to provide further explanation and emotional support. All other responses are appropriate reasons for auditory screening of the newborn. Infants who do not pass the screening test should have it repeated. If they still do not pass the test, they should have a full audiologic and medical evaluation by 3 months of age. If necessary, the infant should be enrolled in early intervention by 6 months of age. PTS: 1 DIF: Cognitive Level: Application OBJ: Nursing Process: Diagnosis MSC: Client Needs: Physiologic Integrity COMPLETION 1. At 1 minute after birth, the nurse assesses the infant and notes a heart rate of 80 beats/min, some flexion of the extremities, a weak cry, grimacing, and a pink body with blue extremities. The nurse would calculate an Apgar score of . ANS: 5 Each of the five signs the nurse noted would score an Apgar of 1 for a total of 5. Signs include heart rate, respiratory effort, muscle tone, reflex irritability, and color. The highest possible Apgar score is 10. NURSINGTB.COM PTS: 1 DIF: Cognitive Level: Application OBJ: Nursing Process: Assessment MSC: Client Needs: Physiologic Integrity Chapter 24: Newborn Nutrition and Feeding Perry: Maternal Child Nursing Care, 6th Edition MULTIPLE CHOICE 1. A new mother recalls from prenatal class that she should try to feed her newborn daughter when she exhibits feeding readiness cues rather than waiting until her infant is crying frantically. On the basis of this information, this woman should feed her infant about every 2.5 to 3 hours when she: a. waves her arms in the air. b. makes sucking motions. c. has hiccups. d. stretches her legs out straight. ANS: B Sucking motions, rooting, mouthing, and hand-to-mouth motions are examples of feeding-readiness cues. Waving the arms in the air, hiccupping, and stretching the legs out straight are not typical feeding-readiness cues. PTS: 1 DIF: Cognitive Level: Comprehension OBJ: Nursing Process: Planning MSC: Client Needs: Health Promotion and Maintenance 2. A new father is ready to take his wife and newborn son home. He proudly tells the nurse who is discharging them that within the next week he plans to start feeding the infant cereal between breastfeeding sessions. The nurse can explain to him that beginning solid foods before 4 to 6 months may: NURSINGTB.COM a. decrease the infant's intake of sufficient calories. b. lead to early cessation of breastfeeding. c. help the infant sleep through the night. d. limit the infant's growth. ANS: B Introduction of solid foods before the infant is 4 to 6 months of age may result in overfeeding and decreased intake of breast milk. It is not true that feeding of solids helps infants sleep through the night. The proper balance of carbohydrate, protein, and fat for an infant to grow properly is in the breast milk or formula. PTS: 1 DIF: Cognitive Level: Application OBJ: Nursing Process: Evaluation MSC: Client Needs: Health Promotion and Maintenance 3. A pregnant woman wants to breastfeed her infant; however, her husband is not convinced that there are any scientific reasons to do so. The nurse can give the couple printed information comparing breastfeeding and bottle-feeding. Which statement is most accurate? Bottle-feeding using commercially prepared infant formulas: a. increases the risk that the infant will develop allergies. b. helps the infant sleep through the night. c. ensures that the infant is getting iron in a form that is easily absorbed. d. requires that multivitamin supplements be given to the infant. ANS: A Exposure to cow's milk poses a risk of developing allergies, eczema, and asthma. “Bottle-feeding using commercially prepared infant formulas helps the infant sleep through the night” is a false statement. Iron is better absorbed from breast milk than from formula. Commercial formulas are designed to meet the nutritional needs of the infant and resemble breast milk. PTS: 1 DIF: Cognitive Level: Application OBJ: Nursing Process: Diagnosis MSC: Client Needs: Physiologic Integrity 4. A after birth woman telephones about her 4-day-old infant. She is not scheduled for a weight check until the infant is 10 days old, and she is worried about whether breastfeeding is going well. Effective breastfeeding is indicated by the newborn who: a. sleeps for 6 hours at a time between feedings. b. has at least one breast milk stool every 24 hours. c. gains 1 to 2 ounces per week. d. has at least 6 to 8 wet diapers per day. ANS: D After day 4, when the mother's milk comes in, the infant should have 6 to 8 wet diapers every 24 hours. Sleeping for 6 hours between feedings is not an indication of whether the infant is breastfeeding well. Typically infants sleep 2 to 4 hours between feedings, depending on whether they are being fed on a 2- to 3-hour schedule or cluster fed. The infant should have a minimum of three bowel movements in a 24-hour period. Breastfed infants typically gain 15 to 30 g/day. PTS: 1 DIF: Cognitive Level: Comprehension OBJ: Nursing Process: Evaluation MSC: Client Needs: Health Promotion and Maintenance NURSINGTB.COM 5. A primiparous woman is delighted with her newborn son and wants to begin breastfeeding as soon as possible. The nurse can facilitate the infant's correct latch-on by helping the woman hold the infant: a. with his arms folded together over his chest. b. curled up in a fetal position. c. with his head cupped in her hand. d. with his head and body in alignment. ANS: D The infant's head and body should be in correct alignment with the mother and the breast during latch-on and feeding. Holding the infant with his arms folded together over his chest, curled up in a fetal position, or with his head cupped in her hand are not ideal positions to facilitate latch-on. PTS: 1 DIF: Cognitive Level: Comprehension OBJ: Nursing Process: Implementation MSC: Client Needs: Health Promotion and Maintenance 6. A breastfeeding woman develops engorged breasts at 3 days' after birth. What action would help this woman achieve her goal of reducing the engorgement? The woman: a. skips feedings to let her sore breasts rest. b. avoids using a breast pump. c. breastfeeds her infant every 2 hours. d. reduces her fluid intake for 24 hours. ANS: C The mother should be instructed to attempt feeding her infant every 2 hours while massaging the breasts as the infant is feeding. Skipping feedings may cause further swelling and discomfort. If the infant does not feed adequately and empty the breast, the mother may pump to extract the milk and relieve some of the discomfort. Dehydration further irritates swollen breast tissue. PTS: 1 DIF: Cognitive Level: Comprehension OBJ: Nursing Process: Implementation MSC: Client Needs: Physiologic Integrity 7. At a 2-month well-baby examination, it was discovered that a breastfed infant had only gained 10 ounces in the past 4 weeks. The mother and the nurse agree that, to gain weight faster, the infant needs to: a. begin solid foods. b. have a bottle of formula after every feeding. c. add at least one extra breastfeeding session every 24 hours. d. start iron supplements. ANS: C Usually the solution to slow weight gain is to improve the feeding technique. Position and latch-on are evaluated, and adjustments are made. It may help to add a feeding or two in a 24-hour period. Solid foods should not be introduced to an infant for at least 4 to 6 months. Bottle-feeding may cause nipple confusion and limit the supply of milk. Iron supplements have no bearing on weight gain. PTS: 1 DIF: Cognitive Level: Application OBJ: Nursing Process: Planning, ImNpUleRmSIeNntGaTtiBon.COM MSC: Client Needs: Physiologic Integrity 8. A new mother wants to be sure that she is meeting her daughter's needs while feeding her commercially prepared infant formula. The nurse should evaluate the mother's knowledge about appropriate infant care. The mother meets her child's needs when she: a. adds rice cereal to her formula at 2 weeks of age to ensure adequate nutrition. b. warms the bottles using a microwave oven. c. burps her infant during and after the feeding as needed. d. refrigerates any leftover formula for the next feeding. ANS: C Most infants swallow air when fed from a bottle and should be given a chance to burp several times during a feeding and after the feeding. Solid food should not be introduced to the infant for at least 4 to 6 months after birth. A microwave should never be used to warm any food to be given to an infant. The heat is not distributed evenly, and this may pose a risk of burning the infant. Any formula left in the bottle after the feeding should be discarded because the infant's saliva has mixed with it. PTS: 1 DIF: Cognitive Level: Comprehension OBJ: Nursing Process: Evaluation MSC: Client Needs: Health Promotion and Maintenance 9. The nurse is discussing storage of breast milk with a mother whose infant is preterm and in the special care unit. What statement would indicate that the mother needs additional teaching? a. “I can store my breast milk in the refrigerator for 3 months.” b. “I can store my breast milk in the freezer for 3 months.” c. “I can store my breast milk at room temperature for 8 hours.” d. “I can store my breast milk in the refrigerator for 3 to 5 days.” ANS: A If the mother states that she can store her breast milk in the refrigerator for 3 months, she needs additional teaching about safe storage. Breast milk can be stored at room temperature for 8 hours, in the refrigerator for 3 to 5 days, in the freezer for 3 months, or in a deep freezer for 6 to 12 months. It is accurate and does not require additional teaching if the mother states that she can store her breast milk in the freezer for 3 months, at room temperature for 8 hours, and in the refrigerator for 3 to 5 days. PTS: 1 DIF: Cognitive Level: Analysis OBJ: Nursing Process: Evaluation MSC: Client Needs: Health Promotion and Maintenance 10. According to the recommendations of the American Academy of Pediatrics on infant nutrition: a. infants should be given only human milk for the first 6 months of life. b. infants fed on formula should be started on solid food sooner than breastfed infants. c. if infants are weaned from breast milk before 12 months, they should receive cow's milk, not formula. d. after 6 months mothers should shift from breast milk to cow's milk. ANS: A Breastfeeding/human milk should also be the sole source of milk for the second 6 months. Infants start on solids when they aNreURreSaIdNyG,TuBs.uCaOllMy at 6 months, whether they start on formula or breast milk. If infants are weaned from breast milk before 12 months, they should receive iron-fortified formula, not cow's milk. PTS: 1 DIF: Cognitive Level: Knowledge OBJ: Nursing Process: Planning MSC: Client Needs: Physiologic Integrity 11. Which statement concerning the benefits or limitations of breastfeeding is inaccurate? a. Breast milk changes over time to meet changing needs as infants grow. b. Long-term studies have shown that the benefits of breast milk continue after the infant is weaned. c. Breast milk/breastfeeding may enhance cognitive development. d. Breastfeeding increases the risk of childhood obesity. ANS: D Breastfeeding actually decreases the risk of childhood obesity. There are multiple benefits of breastfeeding. Breast milk changes over time to meet changing needs as infants grow. Long-term studies have shown that the benefits of breast milk continue after the infant is weaned. Breast milk/breastfeeding may enhance cognitive development. PTS: 1 DIF: Cognitive Level: Comprehension OBJ: Nursing Process: Planning MSC: Client Needs: Health Promotion and Maintenance 12. The nurse is explaining the benefits associated with breastfeeding to a new mother. Which statement by the nurse would be inaccurate and provide conflicting information to the patient? a. Women who breastfeed have a decreased risk of breast cancer. b. Breastfeeding is an effective method of birth control. c. Breastfeeding increases bone density. d. Breastfeeding may enhance after birth weight loss. ANS: B Women who breastfeed have a decreased risk of breast cancer, an increase in bone density, and a possibility of quicker after birth weight loss. Breastfeeding delays the return of fertility; however, it is not an effective birth control method. PTS: 1 DIF: Cognitive Level: Comprehension OBJ: Nursing Process: Planning MSC: Client Needs: Health Promotion and Maintenance 13. While discussing the societal impacts of breastfeeding, the nurse should be cognizant of the benefits and educate the patient accordingly. Which statement as part of this discussion would be incorrect? a. Breastfeeding requires fewer supplies and less cumbersome equipment. b. Breastfeeding saves families money. c. Breastfeeding costs employers in terms of time lost from work. d. Breastfeeding benefits the environment. ANS: C Actually less time is lost to work by breastfeeding mothers, in part because infants are healthier. Breastfeeding is convenient because it does not require cleaning or transporting bottles and other equipment. It saves families money because the cost of formula far exceeds the cost of extra food for the lactating mother. Breastfeeding uses a renewable resource; it does not need fossil fuels, advertising, shipping, or disposal. NURSINGTB.COM PTS: 1 DIF: Cognitive Level: Comprehension OBJ: Nursing Process: Evaluation MSC: Client Needs: Health Promotion and Maintenance 14. The best reason for recommending formula over breastfeeding is that: a. the mother has a medical condition or is taking drugs that could be passed along to the infant via breast milk. b. the mother lacks confidence in her ability to breastfeed. c. other family members or care providers also need to feed the baby. d. the mother sees bottle-feeding as more convenient. ANS: A Breastfeeding is contraindicated when mothers have certain viruses, are undergoing chemotherapy, or are using/abusing illicit drugs. A lack of confidence, the need for others to feed the baby, and the convenience of bottle-feeding are all honest reasons for not breastfeeding, although further education concerning the ease of breastfeeding and its convenience, benefits, and adaptability (expressing milk into bottles) could change some minds. In any case the nurse must provide information in a nonjudgmental manner and respect the mother's decision. Nonetheless, breastfeeding is definitely contraindicated when the mother has medical or drug issues of her own. PTS: 1 DIF: Cognitive Level: Comprehension OBJ: Nursing Process: Planning MSC: Client Needs: Physiologic Integrity 15. With regard to the nutrient needs of breastfed and formula-fed infants, nurses should understand that: a. breastfed infants need extra water in hot climates. b. during the first 3 months breastfed infants consume more energy than do formula-fed infants. c. breastfeeding infants should receive oral vitamin D drops daily at least during the first 2 months. d. vitamin K injections at birth are not needed for infants fed on specially enriched formula. ANS: C Human milk contains only small amounts of vitamin D. Neither breastfed nor formula-fed infants need to be given water, even in very hot climates. During the first 3 months formula-fed infants consume more energy than do breastfed infants and therefore tend to grow more rapidly. Vitamin K shots are required for all infants because the bacteria that produce it are absent from the baby's stomach at birth. PTS: 1 DIF: Cognitive Level: Comprehension OBJ: Nursing Process: Planning MSC: Client Needs: Physiologic Integrity 16. Nurses providing nutritional instruction should be cognizant of the uniqueness of human milk. Which statement is correct? a. Frequent feedings during predictable growth spurts stimulate increased milk production. b. The milk of preterm mothers is the same as the milk of mothers who gave birth at term. c. The milk at the beginning of thNeUfReSeIdNiGngTBis.CtOheMsame as the milk at the end of the feeding. d. Colostrum is an early, less concentrated, less rich version of mature milk. ANS: A These growth spurts (10 days, 3 weeks, 6 weeks, 3 months) usually last 24 to 48 hours, after which infants resume normal feeding. The milk of mothers of preterm infants is different from that of mothers of full-term infants to meet the needs of these newborns. Milk changes composition during feeding. The fat content of the milk increases as the infant feeds. Colostrum precedes mature milk and is more concentrated and richer in proteins and minerals (but not fat). PTS: 1 DIF: Cognitive Level: Comprehension OBJ: Nursing Process: Planning MSC: Client Needs: Health Promotion and Maintenance 17. In assisting the breastfeeding mother position the baby, nurses should keep in mind that: a. the cradle position usually is preferred by mothers who had a cesarean birth. b. women with perineal pain and swelling prefer the modified cradle position. c. whatever the position used, the infant is “belly to belly” with the mother. d. while supporting the head, the mother should push gently on the occiput. ANS: C The infant inevitably faces the mother, belly to belly. The football position usually is preferred after cesarean birth. Women with perineal pain and swelling prefer the side-lying position because they can rest while breastfeeding. The mother should never push on the back of the head. It may cause the baby to bite, hyperextend the neck, or develop an aversion to being brought near the breast. PTS: 1 DIF: Cognitive Level: Application OBJ: Nursing Process: Implementation MSC: Client Needs: Physiologic Integrity 18. The process whereby parents awaken the infant to feed every 3 hours during the day and at least every 4 hours at night is: a. known as demand feeding. b. necessary during the first 24 to 48 hours after birth. c. used to set up the supply-meets-demand system. d. a way to control cluster feeding. ANS: B The parents do this to make sure that the infant has at least eight feedings in 24 hours. Demand feeding is when the infant determines the frequency of feedings; this is appropriate once the infant is feeding well and gaining weight. The supply-meets-demand system is a milk production system that occurs naturally. Cluster feeding is not a problem if the baby has eight feedings in 24 hours. PTS: 1 DIF: Cognitive Level: Comprehension OBJ: Nursing Process: Diagnosis MSC: Client Needs: Health Promotion and Maintenance 19. With regard to basic care of the breastfeeding mother, nurses should be able to advise her that she: NURSINGTB.COM a. will need an extra 1000 calories a day to maintain energy and produce milk. b. can go back to prepregnancy consumption patterns of any drinks, as long as she ingests enough calcium. c. should avoid trying to lose large amounts of weight. d. must avoid exercising because it is too fatiguing. ANS: C Large weight loss would release fat-stored contaminants into her breast milk. It would also likely involve eating too little and/or exercising too much. A breastfeeding mother need add only 200 to 500 extra calories to her diet to provide extra nutrients for the infant. The mother can go back to her consumption patterns of any drinks as long as she ingests enough calcium, only if she does not drink alcohol, limits coffee to no more than two cups (caffeine in chocolate, tea, and some sodas), and reads the herbal tea ingredients carefully. The mother needs her rest, but moderate exercise is healthy. PTS: 1 DIF: Cognitive Level: Comprehension OBJ: Nursing Process: Planning MSC: Client Needs: Physiologic Integrity 20. The breastfeeding mother should be taught a safe method to remove the breast from the baby's mouth. Which suggestion by the nurse is most appropriate? a. Slowly remove the breast from the baby's mouth when the infant has fallen asleep and the jaws are relaxed. b. Break the suction by inserting your finger into the corner of the infant's mouth. c. A popping sound occurs when the breast is correctly removed from the infant's mouth. d. Elicit the Moro reflex to wake the baby and remove the breast when the baby cries. ANS: B Inserting a finger into the corner of the baby's mouth between the gums to break the suction avoids trauma to the breast. The infant who is sleeping may lose grasp on the nipple and areola, resulting in “chewing” on the nipple that makes it sore. A popping sound indicates improper removal of the breast from the baby's mouth and may cause cracks or fissures in the breast. Most mothers prefer the infant to continue to sleep after the feeding. Gentle wake-up techniques are recommended. PTS: 1 DIF: Cognitive Level: Application OBJ: Nursing Process: Implementation MSC: Client Needs: Health Promotion and Maintenance 21. A newly delivered mother who intends to breastfeed tells her nurse, “I am so relieved that this pregnancy is over so I can start smoking again.” The nurse encourages the patient to refrain from smoking. However, this new mother insists that she will resume smoking. The nurse will need to adapt her health teaching to ensure that the patient is aware that: a. smoking has little or no effect on milk production. b. there is no relation between smoking and the time of feedings. c. the effects of secondhand smoke on infants are less significant than for adults. d. the mother should always smoke in another room. ANS: D The new mother should be encouraged not to smoke. If she continues to smoke, she should be encouraged to always smoke in another room removed from the baby. Smoking may impair milk production. When the producNtUs RofSItNoGbaTcBc.oCOaMre broken down, they cross over into the breast milk. Tobacco also results in a reduction of the immunologic properties of breast milk. Research supports that mothers should not smoke within 2 hours before a feeding. The effects of secondhand smoke on infants include sudden infant death syndrome. PTS: 1 DIF: Cognitive Level: Application OBJ: Nursing Process: Planning MSC: Client Needs: Physiologic Integrity 22. Which type of formula is not diluted before being administered to an infant? a. Powdered b. Concentrated c. Ready-to-use d. Modified cow's milk ANS: C Ready-to-use formula can be poured directly from the can into baby's bottle and is good (but expensive) when a proper water supply is not available. Formula should be well mixed to dissolve the powder and make it uniform in consistency. Improper dilution of concentrated formula may cause malnutrition or sodium imbalances. Cow's milk is more difficult for the infant to digest and is not recommended, even if it is diluted. PTS: 1 DIF: Cognitive Level: Comprehension OBJ: Nursing Process: Assessment MSC: Client Needs: Physiologic Integrity 23. How many kilocalories per kilogram (kcal/kg) of body weight does a breastfed term infant require each day? a. 50 to 65 b. 75 to 90 c. 95 to 110 d. 150 to 200 ANS: C For the first 3 months the infant needs 110 kcal/kg/day. At ages 3 to 6 months the requirement is 100 kcal/kg/day. This level decreases slightly to 95 kcal/kg/day from 6 to 9 months and increases again to 100 kcal/kg/day until the baby reaches 12 months. PTS: 1 DIF: Cognitive Level: Knowledge OBJ: Nursing Process: Assessment MSC: Client Needs: Physiologic Integrity 24. The hormone necessary for milk production is: a. estrogen. b. prolactin. c. progesterone. d. lactogen. ANS: B Prolactin, secreted by the anterior pituitary, is a hormone that causes the breasts to produce milk. Estrogen decreases the effectiveness of prolactin and prevents mature breast milk from being produced. Progesterone decreases the effectiveness of prolactin and prevents mature breast milk from being produced. Human placental lactogen decreases the effectiveness of prolactin and prevents mature breast milk from being produced. NURSINGTB.COM PTS: 1 DIF: Cognitive Level: Knowledge OBJ: Nursing Process: Assessment MSC: Client Needs: Physiologic Integrity 25. To initiate the milk ejection reflex (MER), the mother should be advised to: a. wear a firm-fitting bra. b. drink plenty of fluids. c. place the infant to the breast. d. apply cool packs to her breast. ANS: C Oxytocin, which causes the MER reflex, increases in response to nipple stimulation. A firm bra is important to support the breast; however, will not initiate the MER reflex. Drinking plenty of fluids is necessary for adequate milk production, but this alone will not initiate the MER reflex. Cool packs to the breast will decrease the MER reflex. PTS: 1 DIF: Cognitive Level: Comprehension OBJ: Nursing Process: Implementation MSC: Client Needs: Health Promotion and Maintenance 26. As the nurse assists a new mother with breastfeeding, the patient asks, “If formula is prepared to meet the nutritional needs of the newborn, what is in breast milk that makes it better?” The nurse's best response is that it contains: a. more calories. b. essential amino acids. c. important immunoglobulins. d. more calcium. ANS: C Breast milk contains immunoglobulins that protect the newborn against infection. The calorie count of formula and breast milk is about the same. All the essential amino acids are in both formula and breast milk; however, the concentrations may differ. Calcium levels are higher in formula than in breast milk. This higher level can cause an excessively high renal solute load if the formula is not diluted properly. PTS: 1 DIF: Cognitive Level: Knowledge OBJ: Nursing Process: Implementation MSC: Client Needs: Physiologic Integrity 27. When responding to the question “Will I produce enough milk for my baby as she grows and needs more milk at each feeding?” the nurse should explain that: a. the breast milk will gradually become richer to supply additional calories. b. as the infant requires more milk, feedings can be supplemented with cow's milk. c. early addition of baby food will meet the infant's needs. d. the mother's milk supply will increase as the infant demands more at each feeding. ANS: D The amount of milk produced depends on the amount of stimulation of the breast. Increased demand with more frequent and longer breastfeeding sessions results in more milk available for the infant. Mature breast milk will stay the same. The amounts will increase as the infant feeds for longer times. Supplementation will decrease the amount of stimulation of the breast and decrease the milk production. Solids should not be added until about 4 to 6 months, when the infant's immune system is more mature. This will decrease the chance of allergy formations. NURSINGTB.COM PTS: 1 DIF: Cognitive Level: Knowledge OBJ: Nursing Process: Implementation MSC: Client Needs: Physiologic Integrity 28. To prevent nipple trauma, the nurse should instruct the new mother to: a. limit the feeding time to less than 5 minutes. b. position the infant so the nipple is far back in the mouth. c. assess the nipples before each feeding. d. wash the nipples daily with mild soap and water. ANS: B If the infant's mouth does not cover as much of the areola as possible, the pressure during sucking will be applied to the nipple, thus causing trauma to the area. Stimulating the breast for less than 5 minutes will not produce the extra milk the infant may need. This will also limit access to the higher-fat “hindmilk.” Assessing the nipples for trauma is important; however, this action alone will not prevent sore nipples. Soap can be drying to the nipples and should be avoided during breastfeeding. PTS: 1 DIF: Cognitive Level: Application OBJ: Nursing Process: Implementation MSC: Client Needs: Physiologic Integrity 29. Parents have been asked by the neonatologist to provide breast milk for their newborn son, who was born prematurely at 32 weeks of gestation. The nurse who instructs them about pumping, storing, and transporting the milk needs to assess their knowledge of lactation. Which statement is valid? a. A premature infant more easily digests breast milk than formula. b. A glass of wine just before pumping will help reduce stress and anxiety. c. The mother should pump only as much as the infant can drink. d. The mother should pump every 2 to 3 hours, including during the night. ANS: A Human milk is the ideal food for preterm infants, with benefits that are unique in addition to those received by term, healthy infants. Greater physiologic stability occurs with breastfeeding compared with formula feeding. Consumption of alcohol during lactation is approached with caution. Excessive amounts can have serious effects on the infant and can adversely affect the mother's milk ejection reflex. To establish an optimal milk supply, the mother should be instructed to pump 8 to 10 times a day for 10 to 15 minutes on each breast. PTS: 1 DIF: Cognitive Level: Analysis OBJ: Nursing Process: Evaluation MSC: Client Needs: Health Promotion and Maintenance 30. A new mother asks whether she should feed her newborn colostrum, because it is not “real milk.” The nurse's most appropriate answer is: a. colostrum is high in antibodies, protein, vitamins, and minerals. b. colostrum is lower in calories than milk and should be supplemented by formula. c. giving colostrum is important in helping the mother learn how to breastfeed before she goes home. d. colostrum is unnecessary for newborns. ANS: A Colostrum is important because it has high levels of the nutrients needed by the neonate and helps protect against infection. Supplementation is not necessary; it will decrease stimulation to the breast and decrease the prodNuUcRtiSoInNGofTmB.iClkO.MIt is important for the mother to feel comfortable in this role before discharge; however, the importance of the colostrum to the infant is the top priority. Colostrum provides immunities and enzymes necessary to cleanse the gastrointestinal system, among other things. PTS: 1 DIF: Cognitive Level: Knowledge OBJ: Nursing Process: Implementation MSC: Client Needs: Physiologic Integrity MULTIPLE RESPONSE 1. Examples of appropriate techniques to wake a sleepy infant for breastfeeding include: (Select all that apply.) a. unwrapping the infant. b. changing the diaper. c. talking to the infant. d. slapping the infant's hands and feet. e. applying a cold towel to the infant's abdomen. ANS: A, B, C Unwrapping the infant, changing the diaper, and talking to the infant are appropriate techniques to use when trying to wake a sleepy infant. Slapping the infant's hand and feet and applying a cold towel to the infant's abdomen are not appropriate. The parent can rub the infant's hands or feet to wake the infant. Applying a cold towel to the infant's abdomen may lead to cold stress in the infant. The parent may want to apply a cool cloth to the infant's face to wake the infant. PTS: 1 DIF: Cognitive Level: Application OBJ: Nursing Process: Implementation MSC: Client Needs: Health Promotion and Maintenance 2. A nurse is discussing the signs and symptoms of mastitis with a mother who is breastfeeding. What signs and symptoms should the nurse include in her discussion? (Select all that apply.) a. Breast tenderness b. Warmth in the breast c. An area of redness on the breast often resembling the shape of a pie wedge d. A small white blister on the tip of the nipple e. Fever and flu-like symptoms ANS: A, B, C, E Breast tenderness, breast warmth, breast redness, and fever and flu-like symptoms are commonly associated with mastitis and should be included in the nurse's discussion of mastitis. A small white blister on the tip of the nipple generally is not associated with mastitis. It is commonly seen in women who have a plugged milk duct. PTS: 1 DIF: Cognitive Level: Analysis OBJ: Nursing Process: Planning MSC: Client Needs: Physiologic Integrity 3. Late in pregnancy, the woman's bNreUaRstSsINshGoTuBl.dCObeMassessed by the nurse to identify any potential concerns related to breastfeeding. Some nipple conditions make it necessary to provide intervention before birth. These include: (Select all that apply.) a. everted nipples. b. flat nipples. c. inverted nipples. d. nipples that contract when compressed. e. cracked nipples. ANS: B, C, D Flat nipples appear soft, like the areola, and do not stand erect unless stimulated by rolling them between the fingers. Inverted nipples are retracted into the breast tissue. These nipples appear normal; however, they will draw inward when the areola is compressed by the infant's mouth. Dome-shaped devices known as breast shells can be worn during the last weeks of pregnancy and between feedings after birth. The shells are placed inside the bra with the opening over the nipple. The shells exert slight pressure against the areola to help the nipples protrude. The helpfulness of breast shells is debated. A breast pump can be used to draw the nipples out before feedings after delivery. Everted nipples protrude and are normal. No intervention will be required. Cracked, blistered, and bleeding nipples occur after breastfeeding has been initiated and are the result of improper latch. The infant should be repositioned during feeding. Application of colostrum and breast milk after feedings will aid in healing. PTS: 1 DIF: Cognitive Level: Application OBJ: Nursing Process: Assessment MSC: Client Needs: Health Promotion and Maintenance NURSINGTB.COM Chapter 25: The High-Risk Newborn Perry: Maternal Child Nursing Care, 6th Edition MULTIPLE CHOICE 1. A macrosomic infant is born after a difficult forceps-assisted delivery. After stabilization the infant is weighed, and the birth weight is 4550 g (9 lbs, 6 ounces). The nurse's most appropriate action is to: a. leave the infant in the room with the mother. b. take the infant immediately to the nursery. c. perform a gestational age assessment to determine whether the infant is large for gestational age. d. monitor blood glucose levels frequently and observe closely for signs of hypoglycemia. ANS: D This infant is macrosomic (more than 4000 g) and is at high risk for hypoglycemia. Blood glucose levels should be monitored frequently, and the infant should be observed closely for signs of hypoglycemia. Observation may occur in the nursery or in the mother's room, depending on the condition of the fetus. Regardless of gestational age, this infant is macrosomic. PTS: 1 DIF: Cognitive Level: Application OBJ: Nursing Process: Implementation MSC: Client Needs: Physiologic Integrity 2. Infants of mothers with diabetes (NIDUMRSsI)NaGreTBa.tChOiMgher risk for developing: a. anemia. b. hyponatremia. c. respiratory distress syndrome. d. sepsis. ANS: C IDMs are at risk for macrosomia, birth injury, perinatal asphyxia, respiratory distress syndrome, hypoglycemia, hypocalcemia, hypomagnesemia, cardiomyopathy, hyperbilirubinemia, and polycythemia. They are not at risk for anemia, hyponatremia, or sepsis. PTS: 1 DIF: Cognitive Level: Comprehension OBJ: Nursing Process: Planning MSC: Client Needs: Physiologic Integrity 3. An infant was born 2 hours ago at 37 weeks of gestation and weighing 4.1 kg. The infant appears chubby with a flushed complexion and is very tremulous. The tremors are most likely the result of: a. birth injury. b. hypocalcemia. c. hypoglycemia. d. seizures. ANS: C Hypoglycemia is common in the macrosomic infant. Signs of hypoglycemia include jitteriness, apnea, tachypnea, and cyanosis. PTS: 1 DIF: Cognitive Level: Comprehension OBJ: Nursing Process: Assessment MSC: Client Needs: Physiologic Integrity 4. When assessing the preterm infant the nurse understands that compared with the term infant, the preterm infant has: a. few blood vessels visible through the skin. b. more subcutaneous fat. c. well-developed flexor muscles. d. greater surface area in proportion to weight. ANS: D Preterm infants have greater surface area in proportion to their weight. More subcutaneous fat and well-developed muscles are indications of a more mature infant. PTS: 1 DIF: Cognitive Level: Analysis OBJ: Nursing Process: Assessment MSC: Client Needs: Physiologic Integrity 5. On day 3 of life, a newborn continues to require 100% oxygen by nasal cannula. The parents ask whether they can hold their infant during his next gavage feeding. Given that this newborn is physiologically stable, what response would the nurse give? a. “Parents are not allowed to hold infants who depend on oxygen.” b. “You may hold only your baby's hand during the feeding.” c. “Feedings cause more physiologic stress, so the baby must be closely monitored. Therefore, I don't think you should hold the baby.” d. “You may hold your baby durNinUgRtShIeNfGeTeBd.iCnOg.M” ANS: D “You may hold your baby during the feeding” is an accurate statement. Parental interaction via holding is encouraged during gavage feedings so that the infant will associate the feeding with positive interactions. Nasal cannula oxygen therapy allows for easier feedings and psychosocial interactions. The parent can swaddle the infant during gavage feedings to help the infant associate the feeding with positive interactions. Some parents like to do kangaroo care while gavage feeding their infant. Swaddling or kangaroo care during feedings provides positive interactions for the infant. PTS: 1 DIF: Cognitive Level: Application OBJ: Nursing Process: Planning MSC: Client Needs: Health Promotion and Maintenance 6. A premature infant with respiratory distress syndrome receives artificial surfactant. How would the nurse explain surfactant therapy to the parents? a. “Surfactant improves the ability of your baby's lungs to exchange oxygen and carbon dioxide.” b. “The drug keeps your baby from requiring too much sedation.” c. “Surfactant is used to reduce episodes of periodic apnea.” d. “Your baby needs this medication to fight a possible respiratory tract infection.” ANS: A Surfactant can be administered as an adjunct to oxygen and ventilation therapy. With administration of artificial surfactant, respiratory compliance is improved until the infant can generate enough surfactant on his or her own. Surfactant has no bearing on the sedation needs of the infant. Surfactant is used to improve respiratory compliance, including the exchange of oxygen and carbon dioxide. The goal of surfactant therapy in an infant with respiratory distress syndrome (RDS) is to stimulate production of surfactant in the type 2 cells of the alveoli. The clinical presentation of RDS and neonatal pneumonia may be similar. The infant may be started on broad-spectrum antibiotics to treat infection. PTS: 1 DIF: Cognitive Level: Application OBJ: Nursing Process: Planning MSC: Client Needs: Physiologic Integrity 7. When providing an infant with a gavage feeding, which of the following should be documented each time? a. The infant's abdominal circumference after the feeding. b. The infant's heart rate and respirations. c. The infant's suck and swallow coordination. d. The infant's response to the feeding. ANS: D Documentation of a gavage feeding should include the size of the feeding tube, the amount and quality of the residual from the previous feeding, the type and quantity of the fluid instilled, and the infant's response to the procedure. Abdominal circumference is not measured after a gavage feeding. Vital signs may be obtained before feeding. However, the infant's response to the feeding is more important. Some older infants may be learning to suck, but the important factor to document would be the infant's response to the feeding (including attempts to suck). NURSINGTB.COM PTS: 1 DIF: Cognitive Level: Application OBJ: Nursing Process: Evaluation MSC: Client Needs: Physiologic Integrity 8. An infant is to receive gastrostomy feedings. What intervention should the nurse institute to prevent bloating, gastrointestinal reflux into the esophagus, vomiting, and respiratory compromise? a. Rapid bolusing of the entire amount in 15 minutes. b. Warm cloths to the abdomen for the first 10 minutes. c. Slow, small, warm bolus feedings over 30 minutes. d. Cold, medium bolus feedings over 20 minutes. ANS: C Feedings by gravity are done slowly over 20- to 30-minute periods to prevent adverse reactions. Rapid bolusing of the entire amount in 15 minutes would most likely lead to the adverse reactions listed. Temperature stability in the newborn is critical. Warm cloths to the abdomen for the first 10 minutes would not be appropriate because it is not a thermoregulated environment. Additionally, abdominal warming is not indicated with feedings of any kind. Small feedings at room temperature are recommended to prevent adverse reactions. PTS: 1 DIF: Cognitive Level: Application OBJ: Nursing Process: Implementation MSC: Client Needs: Physiologic Integrity 9. A newborn was admitted to the neonatal intensive care unit after being delivered at 29 weeks of gestation to a 28-year-old multiparous, married, Caucasian woman whose pregnancy was uncomplicated until premature rupture of membranes and preterm birth. The newborn's parents arrive for their first visit after the birth. The parents walk toward the bedside but remain approximately 5 feet away from the bed. The nurse's most appropriate action would be to: a. wait quietly at the newborn's bedside until the parents come closer. b. go to the parents, introduce himself or herself, and gently encourage the parents to come meet their infant; explain the equipment first, and then focus on the newborn. c. leave the parents at the bedside while they are visiting so they can have some privacy. d. tell the parents only about the newborn's physical condition, and caution them to avoid touching their baby. ANS: B The nurse is instrumental in the initial interactions with the infant. The nurse can help the parents “see” the infant, rather than focus on the equipment. The importance and purpose of the apparatus that surrounds their infant also should be explained to them. Parents often need encouragement and recognition from the nurse to acknowledge the reality of the infant's condition. Parents need to see and touch their infant as soon as possible to acknowledge the reality of the birth and the infant's appearance and condition. Encouragement from the nurse is instrumental in this process. Telling the parents only about the newborn's physical condition and cautioning them to avoid touching their baby is an inappropriate action. PTS: 1 DIF: Cognitive Level: Application OBJ: Nursing Process: Implementation MSC: Client Needs: Psychosocial Integrity NURSINGTB.COM 10. Necrotizing enterocolitis (NEC) is an inflammatory disease of the gastrointestinal mucosa. The signs of NEC are nonspecific. Some generalized signs include: a. hypertonia, tachycardia, and metabolic alkalosis. b. abdominal distention, temperature instability, and grossly bloody stools. c. hypertension, absence of apnea, and ruddy skin color. d. scaphoid abdomen, no residual with feedings, and increased urinary output. ANS: B Some generalized signs of NEC include decreased activity, hypotonia, pallor, recurrent apnea and bradycardia, decreased oxygen saturation values, respiratory distress, metabolic acidosis, oliguria, hypotension, decreased perfusion, temperature instability, cyanosis, abdominal distention, residual gastric aspirates, vomiting, grossly bloody stools, abdominal tenderness, and erythema of the abdominal wall. The infant may display hypotonia, bradycardia, and metabolic acidosis. PTS: 1 DIF: Cognitive Level: Comprehension OBJ: Nursing Process: Assessment MSC: Client Needs: Physiologic Integrity 11. An infant is being discharged from the neonatal intensive care unit after 70 days of hospitalization. The infant was born at 30 weeks of gestation with several conditions associated with prematurity, including respiratory distress syndrome, mild bronchopulmonary dysplasia, and retinopathy of prematurity requiring surgical treatment. During discharge teaching the infant's mother asks the nurse whether her baby will meet developmental milestones on time, as did her son who was born at term. The nurse's most appropriate response is: a. “Your baby will develop exactly like your first child did.” b. “Your baby does not appear to have any problems at the present time.” c. “Your baby will need to be corrected for prematurity. Your baby is currently 40 weeks of postconceptional age and can be expected to be doing what a 40-week-old infant would be doing.” d. “Your baby will need to be followed very closely.” ANS: C The age of a preterm newborn is corrected by adding the gestational age and the postnatal age. The infant's responses are evaluated accordingly against the norm expected for the corrected age of the infant. Although it is impossible to predict with complete accuracy the growth and development potential of each preterm infant, certain measurable factors predict normal growth and development. The preterm infant experiences catch-up body growth during the first 2 to 3 years of life. The growth and developmental milestones are corrected for gestational age until the child is approximately 2.5 years old. Stating that the baby does not appear to have any problems at the present time is inaccurate. Development will need to be evaluated over time. PTS: 1 DIF: Cognitive Level: Application OBJ: Nursing Process: Planning MSC: Client Needs: Health PromotiNonUaRnSdINMGaTiBnt.eCnOaMnce 12. A pregnant woman was admitted for induction of labor at 43 weeks of gestation with sure dates. A nonstress test (NST) in the obstetrician's office revealed a nonreactive tracing. On artificial rupture of membranes, thick, meconium-stained fluid was noted. The nurse caring for the infant after birth should anticipate: a. meconium aspiration, hypoglycemia, and dry, cracked skin. b. excessive vernix caseosa covering the skin, lethargy, and respiratory distress syndrome. c. golden yellow- to green stained–skin and nails, absence of scalp hair, and an increased amount of subcutaneous fat. d. hyperglycemia, hyperthermia, and an alert, wide-eyed appearance. ANS: A Meconium aspiration, hypoglycemia, and dry, cracked skin are consistent with a postmature infant. Excessive vernix caseosa covering the skin, lethargy, and respiratory distress syndrome would be consistent with a very premature infant. The skin may be meconium stained, but the infant would most likely have longer hair and decreased amounts of subcutaneous fat. Postmaturity with a nonreactive NST would indicate hypoxia. Signs and symptoms associated with fetal hypoxia are hypoglycemia, temperature instability, and lethargy. PTS: 1 DIF: Cognitive Level: Analysis OBJ: Nursing Process: Planning MSC: Client Needs: Physiologic Integrity 13. In caring for the preterm infant, what complication is thought to be a result of high arterial blood oxygen level? a. Necrotizing enterocolitis (NEC) b. Retinopathy of prematurity (ROP) c. Bronchopulmonary dysplasia (BPD) d. Intraventricular hemorrhage (IVH) ANS: B ROP is thought to occur as a result of high levels of oxygen in the blood. NEC is caused by the interference of blood supply to the intestinal mucosa. Necrotic lesions occur at that site. BPD is caused by the use of positive pressure ventilation against the immature lung tissue. IVH results from rupture of the fragile blood vessels in the ventricles of the brain. It is most often associated with hypoxic injury, increased blood pressure, and fluctuating cerebral blood flow. PTS: 1 DIF: Cognitive Level: Comprehension OBJ: Nursing Process: Assessment MSC: Client Needs: Physiologic Integrity 14. In the assessment of a preterm infant, the nurse notices continued respiratory distress even though oxygen and ventilation have been provided. The nurse should suspect: a. hypovolemia and/or shock. b. a nonneutral thermal environment. c. central nervous system injury. d. pending renal failure. ANS: A The nurse should suspect hypovolemia and/or shock. Other symptoms could include hypotension, prolonged capillary rNeUfiRllS,IaNnGdTtBa.cChOyMcardia followed by bradycardia. Intervention is necessary. PTS: 1 DIF: Cognitive Level: Application OBJ: Nursing Process: Diagnosis MSC: Client Needs: Physiologic Integrity 15. Premature infants who exhibit 5 to 10 seconds of respiratory pauses followed by 10 to 15 seconds of compensatory rapid respiration are: a. suffering from sleep or wakeful apnea. b. experiencing severe swings in blood pressure. c. trying to maintain a neutral thermal environment. d. breathing in a respiratory pattern common to premature infants. ANS: D This pattern is called periodic breathing and is common to premature infants. It may still require nursing intervention of oxygen and/or ventilation. Apnea is a cessation of respirations for 20 seconds or longer. It should not be confused with periodic breathing. PTS: 1 DIF: Cognitive Level: Comprehension OBJ: Nursing Process: Assessment MSC: Client Needs: Health Promotion and Maintenance 16. The nurse practicing in the perinatal setting should promote kangaroo care regardless of an infant's gestational age. This intervention: a. is adopted from classical British nursing traditions. b. helps infants with motor and central nervous system impairment. c. helps infants to interact directly with their parents and enhances their temperature regulation. d. gets infants ready for breastfeeding. ANS: C Kangaroo care is skin-to-skin holding in which the infant, dressed only in a diaper, is placed directly on the parent's bare chest and then covered. The procedure helps infants interact with their parents and regulates their temperature, among other developmental benefits. PTS: 1 DIF: Cognitive Level: Knowledge OBJ: Nursing Process: Implementation MSC: Client Needs: Health Promotion and Maintenance 17. For clinical purposes, preterm and postterm infants are defined as: a. preterm before 34 weeks if appropriate for gestational age (AGA) and before 37 weeks if small for gestational age (SGA). b. Postterm after 40 weeks if large for gestational age (LGA) and beyond 42 weeks if AGA. c. Preterm before 37 weeks, and postterm beyond 42 weeks, no matter the size for gestational age at birth. d. Preterm, SGA before 38 to 40 weeks, and postterm, LGA beyond 40 to 42 weeks. ANS: C Preterm and postterm are strictly measures of time—before 37 weeks and beyond 42 weeks, respectively—regardless of size for gestational age. PTS: 1 DIF: Cognitive Level: Comprehension OBJ: Nursing Process: Diagnosis MSC: Client Needs: Health Promotion and Maintenance NURSINGTB.COM 18. As related to the eventual discharge of the high risk newborn or transfer to a different facility, nurses and families should be aware that: a. infants will stay in the neonatal intensive care unit (NICU) until they are ready to go home. b. once discharged to home, the high risk infant should be treated like any healthy term newborn. c. parents of high risk infants need special support and detailed contact information. d. if a high risk infant and mother need transfer to a specialized regional center, it is better to wait until after birth and the infant is stabilized. ANS: C High risk infants can cause profound parental stress and emotional turmoil. Parents need support, special teaching, and quick access to various resources available to help them care for their baby. Parents and their high risk infant should spend a night or two in a predischarge room, where care for the infant is provided away from the NICU. Just because high risk infants are discharged does not mean that they are normal, healthy babies. Follow-up by specialized practitioners is essential. Ideally, the mother and baby are transported with the fetus in utero; this reduces neonatal morbidity and mortality. PTS: 1 DIF: Cognitive Level: Comprehension OBJ: Nursing Process: Planning MSC: Client Needs: Psychosocial Integrity 19. Necrotizing enterocolitis (NEC) is an acute inflammatory disease of the gastrointestinal mucosa that can progress to perforation of the bowel. Approximately 2% to 5% of premature infants succumb to this fatal disease. Care is supportive; however, known interventions may decrease the risk of NEC. To develop an optimal plan of care for this infant, the nurse must understand which intervention has the greatest effect on lowering the risk of NEC: a. early enteral feedings. b. breastfeeding. c. exchange transfusion. d. prophylactic probiotics. ANS: B A decrease in the incidence of NEC is directly correlated with exclusive breastfeeding. Breast milk enhances maturation of the gastrointestinal tract and contains immune factors that contribute to a lower incidence or severity of NEC, Crohn's disease, and celiac illness. The neonatal intensive care unit nurse can be very supportive of the mother in terms of providing her with equipment to pump breast milk, ensuring privacy, and encouraging skin-to-skin contact with the infant. Early enteral feedings of formula or hyperosmolar feedings are a risk factor known to contribute to the development of NEC. The mother should be encouraged to pump or feed breast milk exclusively. Exchange transfusion may be necessary; however, it is a known risk factor for the development of NEC. Although still early, a study in 2005 found that the introduction of prophylactic probiotics appeared to enhance the normal flora of the bowel and therefore decrease the severity of NEC when it did occur. This treatment modality is not as widespread as encouraging breastfeeding; however, it is another strategy that the care providers of these extremely fragile infants may have at their disposal. PTS: 1 DIF: Cognitive Level: Application OBJ: Nursing Process: Planning MSC: Client Needs: Physiologic IntNegUriRtySINGTB.COM 20. Because of the premature infant's decreased immune functioning, what nursing diagnosis should the nurse include in a plan of care for a premature infant? a. Delayed growth and development b. Ineffective thermoregulation c. Ineffective infant feeding pattern d. Risk for infection ANS: D The nurse needs to understand that decreased immune functioning increases the risk for infection. Growth and development, thermoregulation, and feeding may be affected, although only indirectly. PTS: 1 DIF: Cognitive Level: Application OBJ: Nursing Process: Planning MSC: Client Needs: Physiologic Integrity 21. A pregnant woman at 37 weeks of gestation has had ruptured membranes for 26 hours. A cesarean section is performed for failure to progress. The fetal heart rate (FHR) before birth is 180 beats/min with limited variability. At birth the newborn has Apgar scores of 6 and 7 at 1 and 5 minutes and is noted to be pale and tachypneic. On the basis of the maternal history, the cause of this newborn's distress is most likely to be: a. hypoglycemia. b. phrenic nerve injury. c. respiratory distress syndrome. d. sepsis. ANS: D The prolonged rupture of membranes and the tachypnea (before and after birth) both suggest sepsis. An FHR of 180 beats/min is also indicative. This infant is at high risk for sepsis. PTS: 1 DIF: Cognitive Level: Comprehension OBJ: Nursing Process: Assessment MSC: Client Needs: Physiologic Integrity 22. The most important nursing action in preventing neonatal infection is: a. good hand washing. b. isolation of infected infants. c. separate gown technique. d. Standard Precautions. ANS: A Virtually all controlled clinical trials have demonstrated that effective hand washing is responsible for the prevention of nosocomial infection in nursery units. Measures to be taken include Standard Precautions, careful and thorough cleaning, frequent replacement of used equipment, and disposal of excrement and linens in an appropriate manner. Overcrowding must be avoided in nurseries. However, the most important nursing action for preventing neonatal infection is effective hand washing. PTS: 1 DIF: Cognitive Level: Comprehension OBJ: Nursing Process: Implementation MSC: Client Needs: Safe and Effective Care Environment 23. A pregnant woman presents in labNoUr RaSt ItNerGmT,Bh.CaOviMng had no prenatal care. After birth her infant is noted to be small for gestational age with small eyes and a thin upper lip. The infant also is microcephalic. On the basis of her infant's physical findings, this woman should be questioned about her use of which substance during pregnancy? a. Alcohol b. Cocaine c. Heroin d. Marijuana ANS: A The description of the infant suggests fetal alcohol syndrome, which is consistent with maternal alcohol consumption during pregnancy. Fetal brain, kidney, and urogenital system malformations have been associated with maternal cocaine ingestions. Heroin use in pregnancy frequently results in intrauterine growth restriction. The infant may have a shrill cry and sleep cycle disturbances and present with poor feeding, tachypnea, vomiting, diarrhea, hypothermia or hyperthermia, and sweating. Studies have found a higher incidence of meconium staining in infants born of mothers who used marijuana during pregnancy. PTS: 1 DIF: Cognitive Level: Comprehension OBJ: Nursing Process: Assessment MSC: Client Needs: Physiologic Integrity 24. A plan of care for an infant experiencing symptoms of drug withdrawal should include: a. administering chloral hydrate for sedation. b. feeding every 4 to 6 hours to allow extra rest. c. swaddling the infant snugly and holding the baby tightly. d. playing soft music during feeding. ANS: C The infant should be wrapped snugly to reduce self-stimulation behaviors and protect the skin from abrasions. Phenobarbital or diazepam may be administered to decrease central nervous system (CNS) irritability. The infant should be fed in small, frequent amounts, and burped well to diminish aspiration and maintain hydration. The infant should not be stimulated (such as with music) because this will increase activity and potentially increase CNS irritability. PTS: 1 DIF: Cognitive Level: Application OBJ: Nursing Process: Implementation MSC: Client Needs: Physiologic Integrity 25. Human immunodeficiency virus (HIV) may be perinatally transmitted: a. only in the third trimester from the maternal circulation. b. by a needlestick injury at birth from unsterile instruments. c. only through the ingestion of amniotic fluid. d. through the ingestion of breast milk from an infected mother. ANS: D Postnatal transmission of HIV through breastfeeding may occur. Transmission of HIV from the mother to the infant may occur transplacentally at various gestational ages. Transmission close to or at the time of birth is thought to account for 50% to 80% of cases. PTS: 1 DIF: Cognitive Level: Comprehension OBJ: Nursing Process: Planning MSC: Client Needs: Physiologic Integrity 26. The abuse of which of the following substances during pregnancy is the leading cause of cognitive impairment in the UniteNdUSRtaStIeNsG?TB.COM a. Alcohol b. Tobacco c. Marijuana d. Heroin ANS: A Alcohol abuse during pregnancy is recognized as one of the leading causes of cognitive impairment in the United States. PTS: 1 DIF: Cognitive Level: Knowledge OBJ: Nursing Process: Assessment MSC: Client Needs: Psychosocial Integrity 27. During a prenatal examination, the woman reports having two cats at home. The nurse informs her that she should not be cleaning the litter box while she is pregnant. When the woman asks why, the nurse's best response would be: a. “Your cats could be carrying toxoplasmosis. This is a zoonotic parasite that can infect you and have severe effects on your unborn child.” b. “You and your baby can be exposed to the human immunodeficiency virus (HIV) in your cats' feces.” c. “It's just gross. You should make your husband clean the litter boxes.” d. “Cat feces are known to carry Escherichia coli, which can cause a severe infection in both you and your baby.” ANS: A Toxoplasmosis is a multisystem disease caused by the protozoal Toxoplasma gondii parasite, commonly found in cats, dogs, pigs, sheep, and cattle. About 30% of women who contract toxoplasmosis during gestation transmit the disease to their children. Clinical features ascribed to toxoplasmosis include hydrocephalus or microcephaly, chorioretinitis, seizures, or cerebral calcifications. HIV is not transmitted by cats. Although suggesting that the woman's husband clean the litter boxes may be a valid statement, it is not appropriate, does not answer the client's question, and is not the nurse's best response. E. coli is found in normal human fecal flora. It is not transmitted by cats. PTS: 1 DIF: Cognitive Level: Application OBJ: Nursing Process: Planning MSC: Client Needs: Safe and Effective Care Environment 28. A primigravida has just delivered a healthy infant girl. The nurse is about to administer erythromycin ointment in the infant's eyes when the mother asks, “What is that medicine for?” The nurse responds: a. “It is an eye ointment to help your baby see you better.” b. “It is to protect your baby from contracting herpes from your vaginal tract.” c. “Erythromycin is given prophylactically to prevent a gonorrheal infection.” d. “This medicine will protect your baby's eyes from drying out over the next few days.” ANS: C With the prophylactic use of erythromycin, the incidence of gonococcal conjunctivitis has declined to less than 0.5%. Eye prophylaxis is administered at or shortly after birth to prevent ophthalmia neonatorum. Erythromycin has no bearing on enhancing vision, is used to prevent an infection caused by gonorrhea, not herpes, and is not used for eye lubrication. NURSINGTB.COM PTS: 1 DIF: Cognitive Level: Application OBJ: Nursing Process: Planning MSC: Client Needs: Health Promotion and Maintenance 29. With regard to injuries to the infant's plexus during labor and birth, nurses should be aware that: a. if the nerves are stretched with no avulsion, they should recover completely in 3 to 6 months. b. Erb palsy is damage to the lower plexus. c. parents of children with brachial palsy are taught to pick up the child from under the axillae. d. breastfeeding is not recommended for infants with facial nerve paralysis until the condition resolves. ANS: A If the nerves are stretched with no avulsion, they should recover completely in 3 to 6 months. However, if the ganglia are disconnected completely from the spinal cord, the damage is permanent. Erb palsy is damage to the upper plexus and is less serious than brachial palsy. Parents of children with brachial palsy are taught to avoid picking up the child under the axillae or by pulling on the arms. Breastfeeding is not contraindicated, but both the mother and infant will need help from the nurse at the start. PTS: 1 DIF: Cognitive Level: Comprehension OBJ: Nursing Process: Planning MSC: Client Needs: Physiologic Integrity 30. To care adequately for infants at risk for neonatal bacterial infection, nurses should be aware that: a. congenital infection progresses more slowly than does nosocomial infection. b. nosocomial infection can be prevented by effective hand washing; early-onset infections cannot. c. infections occur with about the same frequency in boy and girl infants, although female mortality is higher. d. the clinical sign of a rapid, high fever makes infection easier to diagnose. ANS: B Hand washing is an effective preventive measure for late-onset (nosocomial) infections because these infections come from the environment around the infant. Early-onset, or congenital, infections are caused by the normal flora at the maternal vaginal tract and progress more rapidly than do nosocomial (late-onset) infections. Infection occurs about twice as often in boys and results in higher mortality. Clinical signs of neonatal infection are nonspecific and are similar to those of noninfectious problems, thus making diagnosis difficult. PTS: 1 DIF: Cognitive Level: Comprehension OBJ: Nursing Process: Implementation MSC: Client Needs: Safe and Effective Care Environment 31. Near the end of the first week of life, an infant who has not been treated for any infection develops a copper-colored, maculopapular rash on the palms and around the mouth and anus. The newborn is showing signs of: a. gonorrhea. b. herpes simplex virus infection. c. congenital syphilis. NURSINGTB.COM d. human immunodeficiency virus. ANS: C The rash is indicative of congenital syphilis. The lesions may extend over the trunk and extremities. PTS: 1 DIF: Cognitive Level: Analysis OBJ: Nursing Process: Diagnosis MSC: Client Needs: Physiologic Integrity 32. What bacterial infection is definitely decreasing because of effective drug treatment? a. Escherichia coli infection b. Tuberculosis c. Candidiasis d. Group B streptococcal infection ANS: D Penicillin has significantly decreased the incidence of group B streptococcal infection. E. coli may be increasing, perhaps because of the increasing use of ampicillin (resulting in a more virulent E. coli resistant to the drug). Tuberculosis is increasing in the United States and Canada. Candidiasis is a fairly benign fungal infection. PTS: 1 DIF: Cognitive Level: Comprehension OBJ: Nursing Process: Evaluation MSC: Client Needs: Health Promotion and Maintenance 33. In caring for the mother who has abused (or is abusing) alcohol and for her infant, nurses should be aware that: a. the pattern of growth restriction of the fetus begun in prenatal life is halted after birth, and normal growth takes over. b. two-thirds of newborns with fetal alcohol syndrome (FAS) are boys. c. alcohol-related neurodevelopmental disorders not sufficient to meet FAS criteria (learning disabilities, speech, and language problems) are often not detected until the child goes to school. d. both the distinctive facial features of the FAS infant and the diminished mental capacities tend toward normal over time. ANS: C Some learning problems do not become evident until the child is at school. The pattern of growth restriction persists after birth. Two-thirds of newborns with FAS are girls. Although the distinctive facial features of the FAS infant tend to become less evident, the mental capacities never become normal. PTS: 1 DIF: Cognitive Level: Comprehension OBJ: Nursing Process: Planning MSC: Client Needs: Health Promotion and Maintenance 34. A careful review of the literature on the various recreational and illicit drugs reveals that: a. more longer-term studies are needed to assess the lasting effects on infants when mothers have taken or are taking illegal drugs. b. heroin and methadone cross the placenta; marijuana, cocaine, and phencyclidine (PCP) do not. c. mothers should discontinue heroin use (detox) any time they can during pregnancy. NURSINGTB.COM d. methadone withdrawal for infants is less severe and shorter than heroin withdrawal. ANS: A Studies on the effects of marijuana and cocaine use by mothers are somewhat contradictory. More long-range studies are needed. Just about all these drugs cross the placenta, including marijuana, cocaine, and PCP. Drug withdrawal is accompanied by fetal withdrawal, which can lead to fetal death. Therefore, detoxification from heroin is not recommended, particularly later in pregnancy. Methadone withdrawal is more severe and more prolonged than heroin withdrawal. PTS: 1 DIF: Cognitive Level: Analysis OBJ: Nursing Process: Evaluation MSC: Client Needs: Physiologic Integrity 35. Providing care for the neonate born to a mother who abuses substances can present a challenge for the health care team. Nursing care for this infant requires a multisystem approach. The first step in the provision of this care is: a. pharmacologic treatment. b. reduction of environmental stimuli. c. neonatal abstinence syndrome scoring. d. adequate nutrition and maintenance of fluid and electrolyte balance. ANS: C Neonatal abstinence syndrome (NAS) is the term used to describe the cohort of symptoms associated with drug withdrawal in the neonate. The Neonatal Abstinence Scoring System evaluates central nervous system (CNS), metabolic, vasomotor, respiratory, and gastrointestinal disturbances. This evaluation tool enables the care team to develop an appropriate plan of care. The infant is scored throughout the length of stay, and the treatment plan is adjusted accordingly. Pharmacologic treatment is based on the severity of withdrawal symptoms. Symptoms are determined by using a standard assessment tool. Medications of choice are morphine, phenobarbital, diazepam, or diluted tincture of opium. Swaddling, holding, and reducing environmental stimuli are essential in providing care to the infant who is experiencing withdrawal. These nursing interventions are appropriate for the infant who displays CNS disturbances. Poor feeding is one of the gastrointestinal symptoms common to this client population. Fluid and electrolyte balance must be maintained and adequate nutrition provided. These infants often have a poor suck reflex and may need to be fed via gavage. PTS: 1 DIF: Cognitive Level: Application OBJ: Nursing Process: Assessment MSC: Client Needs: Physiologic Integrity 36. While completing a newborn assessment, the nurse should be aware that the most common birth injury is: a. to the soft tissues. b. caused by forceps gripping the head on delivery. c. fracture of the humerus and femur. d. fracture of the clavicle. ANS: D The most common birth injury is fracture of the clavicle (collarbone). It usually heals without treatment, although the arm and shNoUuRldSeINr GmTaBy.CbOeMimmobilized for comfort. PTS: 1 DIF: Cognitive Level: Knowledge OBJ: Nursing Process: Assessment MSC: Client Needs: Physiologic Integrity 37. The most common cause of pathologic hyperbilirubinemia is: a. hepatic disease. b. hemolytic disorders in the newborn. c. postmaturity. d. congenital heart defect. ANS: B Hemolytic disorders in the newborn are the most common cause of pathologic jaundice. Hepatic damage may be a cause of pathologic hyperbilirubinemia, but it is not the most common cause. Prematurity would be a potential cause of pathologic hyperbilirubinemia in neonates, but it is not the most common cause. Congenital heart defect is not a common cause of pathologic hyperbilirubinemia in neonates. PTS: 1 DIF: Cognitive Level: Knowledge OBJ: Nursing Process: Diagnosis MSC: Client Needs: Physiologic Integrity 38. Which infant would be more likely to have Rh incompatibility? a. Infant of an Rh-negative mother and a father who is Rh positive and homozygous for the Rh factor. b. Infant who is Rh negative and whose mother is Rh negative. c. Infant of an Rh-negative mother and a father who is Rh positive and heterozygous for the Rh factor. d. Infant who is Rh positive and whose mother is Rh positive. ANS: A If the mother is Rh negative and the father is Rh positive and homozygous for the Rh factor, all the children will be Rh positive. Only Rh-positive children of an Rh-negative mother are at risk for Rh incompatibility. If the mother is Rh negative and the father is Rh positive and heterozygous for the factor, there is a 50% chance that each infant born of the union will be Rh positive and a 50% chance that each will be born Rh negative. PTS: 1 DIF: Cognitive Level: Comprehension OBJ: Nursing Process: Planning MSC: Client Needs: Health Promotion and Maintenance 39. With regard to hemolytic diseases of the newborn, nurses should be aware that: a. Rh incompatibility matters only when an Rh-negative child is born to an Rh-positive mother. b. ABO incompatibility is more likely than Rh incompatibility to precipitate significant anemia. c. exchange transfusions frequently are required in the treatment of hemolytic disorders. d. the indirect Coombs' test is performed on the mother before birth; the direct Coombs' test is performed on the cord blood after birth. ANS: D An indirect Coombs' test may be performed on the mother a few times during pregnancy. Only the Rh-positive child of an Rh-negative mother is at risk. ABO incompatibility is more common than Rh incompatibility NbuUtRcSaIuNsGeTsBle.CsOs Msevere problems; significant anemia, for instance, is rare with ABO. Exchange transfers are needed infrequently because of the decrease in the incidence of severe hemolytic disease in newborns from Rh incompatibility. PTS: 1 DIF: Cognitive Level: Comprehension OBJ: Nursing Process: Planning MSC: Client Needs: Health Promotion and Maintenance 40. An infant with severe meconium aspiration syndrome (MAS) is not responding to conventional treatment. Which highly technical method of treatment may be necessary for an infant who does not respond to conventional treatment? a. Extracorporeal membrane oxygenation b. Respiratory support with a ventilator c. Insertion of a laryngoscope and suctioning of the trachea d. Insertion of an endotracheal tube ANS: A Extracorporeal membrane oxygenation is a highly technical method that oxygenates the blood while bypassing the lungs, thus allowing the infant's lungs to rest and recover. The infant is likely to have been first connected to a ventilator. Laryngoscope insertion and tracheal suctioning are performed after birth before the infant takes the first breath. An endotracheal tube will be in place to facilitate deep tracheal suctioning and ventilation. PTS: 1 DIF: Cognitive Level: Knowledge OBJ: Nursing Process: Evaluation MSC: Client Needs: Physiologic Integrity 41. The goal of treatment of the infant with phenylketonuria (PKU) is to: a. cure mental retardation. b. prevent central nervous system (CNS) damage, which leads to mental retardation. c. prevent gastrointestinal symptoms. d. cure the urinary tract infection. ANS: B CNS damage can occur as a result of toxic levels of phenylalanine. No known cure exists for mental retardation. Digestive problems are a clinical manifestation of PKU. PKU does not involve any urinary problems. PTS: 1 DIF: Cognitive Level: Comprehension OBJ: Nursing Process: Planning MSC: Client Needs: Physiologic Integrity 42. HIV may be perinatally transmitted: a. only in the third trimester from the maternal circulation. b. from the use of unsterile instruments. c. only through the ingestion of amniotic fluid. d. through the ingestion of breast milk from an infected mother. ANS: D Postnatal transmission of HIV through breastfeeding may occur. Transmission of HIV from the mother to the infant may occur transplacentally at various gestational ages. This is highly unlikely because most health care facilities must meet sterility standards for all instrumentation. Transmission of HIV may occur during birth from blood or secretions. PTS: 1 DIF: Cognitive Level: Comprehension OBJ: Nursing Process: Planning MSC: Client Needs: Physiologic IntNegUriRtyS:INRGedTuBc.tCioOnMof Risk Potential MULTIPLE RESPONSE 1. Infants born between 34 0/7 and 36 6/7 weeks of gestation are called late-preterm infants because they have many needs similar to those of preterm infants. Because they are more stable than early-preterm infants, they may receive care that is much like that of a full-term baby. The mother-baby or nursery nurse knows that these babies are at increased risk for: (Select all that apply.) a. problems with thermoregulation. b. cardiac distress. c. hyperbilirubinemia. d. sepsis. e. hyperglycemia. ANS: A, C, D Thermoregulation problems, hyperbilirubinemia, and sepsis are all conditions related to immaturity and warrant close observation. After discharge the infant is at risk for rehospitalization related to these problems. AWHONN launched the Near-Term Infant Initiative to study the problem and ways to ensure that these infants receive adequate care. The nurse should ensure that this infant is feeding adequately before discharge and that parents are taught the signs and symptoms of these complications. Late-preterm infants are also at increased risk for respiratory distress and hypoglycemia. PTS: 1 DIF: Cognitive Level: Comprehension OBJ: Nursing Process: Assessment MSC: Client Needs: Health Promotion and Maintenance MATCHING Car seat safety is an essential part of discharge planning, and infants younger than 37 weeks of gestation should have a period of observation in an appropriate car seat to monitor for possible apnea, bradycardia, and decreased SaO2. The nurse who is about to perform a car seat evaluation on a late-preterm infant will perform the sequence of the test in which order? a. Secure the infant in the car seat per guidelines using blanket rolls on the side. b. Leave the infant undisturbed for 90 to 120 minutes. c. Set the heart rate alarm at 80 beats/min and the apnea alarm at 20 seconds. d. Document the infant's tolerance to the test. e. Perform the evaluation 1 to 7 days before discharge. f. Use the parent's car seat. g. Set the pulse oximeter low alarm at 88%. OBJ: Nursing Process: Assessment MSC: Client Needs: Physiologic Integrity NOT: An episode of desaturation, bradycardia or apnea (20 seconds or more) constitutes a failure, and evaluation by the practitioner must occur before discharge. A car bed with the infant supine may be considered, and similar testing should occur in the car bed. 2. ANS: E PTS: 1 DIF: Cognitive Level: Application OBJ: Nursing Process: Assessment MSC: Client Needs: Physiologic Integrity NOT: An episode of desaturation, bradycardia or apnea (20 seconds or more) constitutes a failure, and evaluation by the practitioner must occur before discharge. A car bed with the infant supine may be considered, and similar testing should occur in the car bed. 3. ANS: A PTS: 1 DIF: Cognitive Level: Application OBJ: Nursing Process: Assessment MSC: Client Needs: Physiologic Integrity NOT: An episode of desaturation, bradycardia or apnea (20 seconds or more) constitutes a failure, and evaluation by the practitioner must occur before discharge. A car bed with the infant supine may be considered, and similar testing should occur in the car bed. 4. ANS: G PTS: 1 DIF: Cognitive Level: Application OBJ: Nursing Process: Assessment MSC: Client Needs: Physiologic Integrity NOT: An episode of desaturation, bradycardia or apnea (20 seconds or more) constitutes a failure, and evaluation by the practitioner must occur before discharge. A car bed with the infant supine may be considered, and similar testing should occur in the car bed. 5. ANS: C PTS: 1 DIF: Cognitive Level: Application OBJ: Nursing Process: Assessment MSC: Client Needs: Physiologic Integrity NOT: An episode of desaturation, bradycardia or apnea (20 seconds or more) constitutes a failure, and evaluation by the practitioner must occur before discharge. A car bed with the infant supine may be considered, and similar testing should occur in the car bed. 6. ANS: B PTS: 1 DIF: Cognitive Level: Application OBJ: Nursing Process: Assessment MSC: Client Needs: Physiologic Integrity NOT: An episode of desaturation, bradycardia or apnea (20 seconds or more) constitutes a failure, and evaluation by the practitioner must occur before discharge. A car bed with the infant supine may be considered, and similar testing should occur in the car bed. 7. ANS: D PTS: 1 DIF: Cognitive Level: Application OBJ: Nursing Process: Assessment MSC: Client Needs: Physiologic Integrity NOT: An episode of desaturation, bradycardia or apnea (20 seconds or more) constitutes a failure, and evaluation by the practitioner must occur before discharge. A car bed with the infant supine may be considered, and similar testing should occur in the car bed. NURSINGTB.COM Chapter 26: 21st Century Pediatric Nursing Perry: Maternal Child Nursing Care, 6th Edition MULTIPLE CHOICE 1. From a worldwide perspective, infant mortality in the United States: a. is the highest of the other developed nations. b. lags behind five other developed nations. c. is the lowest infant death rate of developed nations. d. lags behind most other developed nations. ANS: A Although the death rate has decreased, the United States still ranks last among nations with the lowest infant death rates. The United States has the highest infant death rate of developed nations. PTS: 1 DIF: Cognitive Level: Knowledge OBJ: Nursing Process: Assessment MSC: Client Needs: Health Promotion and Maintenance 2. The major cause of death for children older than 1 year is: a. cancer. b. infection. c. unintentional injuries. d. congenital abnormalities. ANS: C Unintentional injuries (accidents) NaUreRtShIeNGleTaBd.iCnOg Mcause of death after age 1 year through adolescence. Congenital anomalies are the leading cause of death in those younger than 1 year and are less significant in this age-group. There have been major declines in deaths attributed infection as a result of improved therapies. Cancer is the second leading cause of death in this age-group. PTS: 1 DIF: Cognitive Level: Comprehension OBJ: Nursing Process: Planning MSC: Client Needs: Health Promotion and Maintenance 3. In addition to injuries, the leading causes of death in adolescents ages 15 to 19 years are: a. suicide, cancer. b. homicide, suicide c. homicide, heart disease. d. drowning, cancer. ANS: B In this age-group the leading cause of death is accidents, followed by homicide and suicide. Other causes of death include cancer and heart disease. PTS: 1 DIF: Cognitive Level: Knowledge OBJ: Nursing Process: Planning MSC: Client Needs: Health Promotion and Maintenance 4. The leading cause of death from unintentional injuries in children is: a. poisoning. b. drowning. c. motor vehicle related fatalities. d. fire- and burn-related fatalities. ANS: C Motor vehicle related fatalities comprise the leading cause of death in children, as either passengers or pedestrians. Poisoning is the ninth leading cause of death. Drowning is the second leading cause of death. Fire- and burn-related fatalities are the third leading cause of death. PTS: 1 DIF: Cognitive Level: Knowledge OBJ: Nursing Process: Planning MSC: Client Needs: Health Promotion and Maintenance 5. Which of the following is descriptive of deaths caused by unintentional injuries? a. More deaths occur in males. b. More deaths occur in females. c. The pattern of deaths varies widely in Western societies. d. The pattern of deaths does not vary according to age and sex. ANS: A Most deaths from unintentional injuries occur in males. The pattern of death caused by unintentional injuries is consistent in Western societies. Causes of unintentional deaths vary with age and gender. PTS: 1 DIF: Cognitive Level: Comprehension OBJ: Nursing Process: Planning MSC: Client Needs: Health Promotion and Maintenance 6. The type of injury a child is especially susceptible to at a specific age is most closely related to: a. physical health of the child. NURSINGTB.COM b. developmental level of the child. c. educational level of the child. d. number of responsible adults in the home. ANS: B The child's developmental stage determines the type of injury that is likely to occur. The child's physical health may facilitate his or her recovery from an injury. Educational level is related to developmental level, but it is not as important as the child's developmental level in determining the type of injury. The number of responsible adults in the home may affect the number of unintentional injuries, but the type of injury is related to the child's developmental stage. PTS: 1 DIF: Cognitive Level: Comprehension OBJ: Nursing Process: Planning MSC: Client Needs: Health Promotion and Maintenance 7. Which statement regarding childhood morbidity is the most accurate? a. Morbidity does not vary with age. b. Morbidity is not distributed randomly. c. Little can be done to improve morbidity. d. Unintentional injuries do not have an effect on morbidity. ANS: B Morbidity is not distributed randomly in children. Increased morbidity is associated with certain groups of children, including children living in poverty and those who were low birth weight. Morbidity does vary with age. The types of illnesses in children are different for each age-group. Morbidity can be decreased with interventions focused on groups with high morbidity and on decreasing unintentional injuries, which also affect morbidity. PTS: 1 DIF: Cognitive Level: Comprehension OBJ: Nursing Process: Planning MSC: Client Needs: Health Promotion and Maintenance 8. Which statement is most descriptive of pediatric family-centered care? a. It reduces the effect of cultural diversity on the family. b. It encourages family dependence on the health care system. c. It recognizes that the family is the constant in a child's life. d. It avoids expecting families to be part of the decision-making process. ANS: C The key components of family-centered care are for the nurse to support, respect, encourage, and embrace the family's strength by developing a partnership with the child's parents. Family-centered care recognizes the family as the constant in the child's life. The nurse should support the cultural diversity of the family, not reduce its effect. The family should be enabled and empowered to work with the health care system and to be part of the decision-making process. PTS: 1 DIF: Cognitive Level: Comprehension OBJ: Nursing Process: Implementation MSC: Client Needs: Health Promotion and Maintenance 9. The nurse is preparing staff in-service education about atraumatic care for pediatric patients. Which intervention should the nurNsUe RinScINluGdTeB?.COM a. Prepare the child for separation from parents during hospitalization by reviewing a video. b. Prepare the child before any unfamiliar treatment or procedure by demonstrating on a stuffed animal. c. Help the child accept the loss of control associated with hospitalization. d. Help the child accept pain that is connected with a treatment or procedure. ANS: B Preparing the child for any unfamiliar treatments, controlling pain, allowing privacy, providing play activities for expression of fear and aggression, providing choices, and respecting cultural differences are components of atraumatic care. In providing atraumatic care, the separation of child from parents during hospitalization is minimized. The nurse should promote a sense of control for the child. Preventing and minimizing bodily injury and pain are major components of atraumatic care. PTS: 1 DIF: Cognitive Level: Application OBJ: Nursing Process: Implementation MSC: Client Needs: Health Promotion and Maintenance 10. Which statement best describes the process of critical thinking? a. It is a simple developmental process. b. It is purposeful and goal directed. c. It is based on deliberate and irrational thought. d. It assists individuals in guessing what is most appropriate. ANS: B Critical thinking is a complex, developmental process based on rational and deliberate thought. When thinking is clear, precise, accurate, relevant, consistent, and fair, a logical connection develops between the elements of thought and the problem at hand. PTS: 1 DIF: Cognitive Level: Comprehension OBJ: Nursing Process: Planning MSC: Client Needs: Safe and Effective Care Environment 11. A clinic nurse is planning a teaching session about childhood obesity prevention for parents of school-age children. The nurse should include which associated risk of obesity in the teaching plan? a. Type I diabetes b. Respiratory disease c. Celiac disease d. Type II diabetes ANS: D Childhood obesity has been associated with the rise of type II diabetes in children. Type I diabetes is not associated with obesity and has a genetic component. Respiratory disease is not associated with obesity, and celiac disease is the inability to metabolize gluten in foods and is not associated with obesity. PTS: 1 DIF: Cognitive Level: Application OBJ: Nursing Process: Planning MSC: Client Needs: Health Promotion and Maintenance 12. Which is now referred to as the “new morbidity”? a. Limitations in the major activiNtiUeRs SoIfNdGaTilBy.CliOvMing b. Unintentional injuries that cause chronic health problems c. Discoveries of new therapies to treat health problems d. Behavioral, social, and educational problems that alter health ANS: D The new morbidity reflects the behavioral, social, and educational problems that interfere with the child's social and academic development. It is also referred to a “‘pediatric social illness'.” Limitations in major activities of daily living and unintentional injuries that result in chronic health problems are included in morbidity data. Discovery of new therapies would be reflected in changes in morbidity data over time. PTS: 1 DIF: Cognitive Level: Knowledge OBJ: Nursing Process: Assessment MSC: Client Needs: Health Promotion and Maintenance 13. Which action by the nurse demonstrates use of evidence-based practice (EBP)? a. Gathering equipment for a procedure b. Documenting changes in a patient's status c. Questioning the use of daily central line dressing changes d. Clarifying a physician's prescription for morphine ANS: C The nurse who questions the daily central line dressing change is ascertaining whether clinical interventions result in positive outcomes for patients. This demonstrates evidence-based practice (EBP), which implies questioning why something is effective and whether a better approach exists. Gathering equipment for a procedure and documenting changes in a patient's status are practices that follow established guidelines. Clarifying a physician's prescription for morphine constitutes safe nursing care. PTS: 1 DIF: Cognitive Level: Application OBJ: Nursing Process: Evaluation MSC: Client Needs: Safe and Effective Care Environment: Management of Care MULTIPLE RESPONSE 1. The nursing process is a method of problem identification and problem solving that describes what the nurse actually does. The five steps include: (Select all that apply.) a. assessment. b. diagnosis. c. planning. d. documentation e. implementation. f. evaluation ANS: A, B, C, E, F The accepted model is assessment, diagnosis, planning, implementation, and evaluation. The diagnosis phase is separated into two steps: nursing diagnosis and outcome identification. Although documentation is not one of the five steps of the nursing process, it is essential for evaluation. The nurse can assess, diagnose and identify problems, plan, and implement without documentation; however,NeUvaRlSuIaNtGioTnBi.sCObeMst performed with written evidence of progress toward outcomes. PTS: 1 DIF: Cognitive Level: Knowledge OBJ: Nursing Process: Assessment, Diagnosis, Planning, Implementation, and Evaluation MSC: Client Needs: Health Promotion and Maintenance 2. Which behaviors by the nurse indicate a therapeutic relationship with children and families? (Select all that apply.) a. Spending off-duty time with children and families b. Asking questions if families are not participating in the care c. Clarifying information for families d. Buying toys for a hospitalized child e. Learning about the family's religious preferences ANS: B, C, E Asking questions if families are not participating in the care, clarifying information for families, and learning about the family's religious preferences are positive actions and foster therapeutic relationships with children and families. Spending off-duty time with children and families and buying toys for a hospitalized child are negative actions and indicate over involvement with children and families that is nontherapeutic. PTS: 1 DIF: Cognitive Level: Comprehension OBJ: Nursing Process: Evaluation MSC: Client Needs: Psychosocial Integrity Chapter 27: Family, Social, Cultural, and Religious Influences on Child Health Promotion Perry: Maternal Child Nursing Care, 6th Edition MULTIPLE CHOICE 1. What type of family is one in which all members are related by blood? a. Consanguineous b. Affinal c. Family of origin d. Household ANS: A A consanguineous family is one of the most common types and consists of members who have a blood relationship. The affinal family is one made up of marital relationships. Although the parents are married, they may each bring children from a previous relationship. The family of origin is the family unit that a person is born into. Considerable controversy has been generated about the newer concepts of families (i.e., communal, single-parent, or homosexual families). To accommodate these other varieties of family styles, the descriptive term household is frequently used. PTS: 1 DIF: Cognitive Level: Knowledge OBJ: Nursing Process: Assessment MSC: Client Needs: Health Promotion and Maintenance 2. A 3-year-old girl was adopted immediately after birth. The parents have just asked the nurse how they should tell the child thatNsUhReSiIsNaGdToBp.tCeOd.MWhich guideline concerning adoption should the nurse use in planning her response? a. Telling the child is an important aspect of their parental responsibilities. b. The best time to tell the child is between ages 7 and 10 years. c. It is not necessary to tell the child who was adopted so young. d. It is best to wait until the child asks about it. ANS: A It is important for the parents not to withhold information about the adoption from the child. It is an essential component of the child's identity. There is no recommended best time to tell children. It is believed that children should be told young enough so they do not remember a time when they did not know. It should be done before the children enter school to keep third parties from telling the children before the parents have had the opportunity. PTS: 1 DIF: Cognitive Level: Analysis OBJ: Nursing Process: Implementation MSC: Client Needs: Health Promotion and Maintenance 3. The mother of a school-age child tells the school nurse that she and her spouse are going through a divorce. The child has not been doing well in school and sometimes has trouble sleeping. The nurse should recognize this as: a. indicative of maladjustment. b. common reaction to divorce. c. suggestive of lack of adequate parenting. d. unusual response that indicates need for referral. ANS: B Parental divorce affects school-age children in many ways. In addition to difficulties in school, they often have profound sadness, depression, fear, insecurity, frequent crying, loss of appetite, and sleep disorders. Uncommon responses to parental divorce include indications of maladjustment, the suggestion of lack of adequate parenting, and the need for referral. PTS: 1 DIF: Cognitive Level: Application OBJ: Nursing Process: Assessment MSC: Client Needs: Psychosocial Integrity 4. A mother brings 6-month-old Eric to the clinic for a well-baby checkup. She comments, “I want to go back to work, but I don't want Eric to suffer because I'll have less time with him.” The nurse's most appropriate answer is: a. “I'm sure he'll be fine if you get a good baby-sitter.” b. “You will need to stay home until Eric starts school.” c. “You should go back to work so Eric will get used to being with others.” d. “Let's talk about the child care options that will be best for Eric.” ANS: D “Let's talk about the child care options that will be best for Eric” is an open-ended statement that will assist the mother in exploring her concerns about what is best for both her and Eric. “I'm sure he'll be fine if you get a good baby-sitter,” “You will need to stay home until Eric starts school,” and “You should go back to work so Eric will get used to being with others” are directive statements and do not address the effect of her working on Eric. PTS: 1 DIF: Cognitive Level: Application OBJ: Nursing Process: Implementation MSC: Client Needs: Psychosocial Integrity 5. Which term best describes a groupNUofRpSIeNoGplTeBw.ChOoMshare a set of values, beliefs, practices, social relationships, law, politics, economics, and norms of behavior? a. Race b. Culture c. Ethnicity d. Social group ANS: B Culture is a pattern of assumptions, beliefs, and practices that unconsciously frames or guides the outlook and decisions of a group of people. A culture is composed of individuals who share a set of values, beliefs, and practices that serve as a frame of reference for individual perceptions and judgments. Race is defined as a division of humankind who possesses traits transmissible by descent and sufficient to characterize it as a distinct human type. Ethnicity is an affiliation of a set of persons who share a unique cultural, social, and linguistic heritage. A social group consists of systems of roles carried out in groups. Examples of primary social groups include the family and peer groups. PTS: 1 DIF: Cognitive Level: Knowledge OBJ: Nursing Process: Assessment MSC: Client Needs: Psychosocial Integrity 6. Which term best describes the emotional attitude that one's own ethnic group is superior to others? a. Culture b. Ethnicity c. Superiority d. Ethnocentrism ANS: D Ethnocentrism is the belief that one's way of living and behaving is the best way. This includes the emotional attitude that the values, beliefs, and perceptions of one's ethnic group are superior to those of others. Culture is a pattern of assumptions, beliefs, and practices that unconsciously frames or guides the outlook and decisions of a group of people. A culture is composed of individuals who share a set of values, beliefs, and practices that serve as a frame of reference for individual perception and judgments. Ethnicity is an affiliation of a set of persons who share a unique cultural, social, and linguistic heritage. Superiority is the state or quality of being superior; it does not include ethnicity. PTS: 1 DIF: Cognitive Level: Comprehension OBJ: Nursing Process: Assessment MSC: Client Needs: Psychosocial Integrity 7. A Chinese toddler has pneumonia. The nurse notices that the parent consistently feeds the child only the broth that comes on the clear liquid tray. Food items such as Jell-O, Popsicles, and juices are left. What would best explain this? a. The parent is trying to feed child only what child likes most. b. The parent is trying to restore normal balance through appropriate “hot” remedies. c. Hispanics believe that the “evil eye” enters when a person gets cold. d. Hispanics believe that an innate energy called chi is strengthened by eating soup. ANS: B In several groups, including Filipino, Chinese, Arabic, and Hispanic cultures, hot and cold describe certain properties completely unrelated to temperature. Respiratory conditions such as pneumonia are “cold” conditionNsUaRnSdINaGreTBtr.eCaOteMd with “hot” foods. This may be true, but it is unlikely that a toddler would consistently prefer the broth to Jell-O, Popsicles, and juice. The evil eye applies to a state of imbalance of health, not curative actions. Chinese individuals believe in chi as an innate energy. PTS: 1 DIF: Cognitive Level: Application OBJ: Nursing Process: Implementation MSC: Client Needs: Psychosocial Integrity 8. The nurse discovers welts on the back of a Vietnamese child during a home health visit. The child's mother says that she has rubbed the edge of a coin on her child's oiled skin. The nurse should recognize that this is: a. child abuse. b. a cultural practice to rid the body of disease. c. a cultural practice to treat enuresis or temper tantrums. d. a child discipline measure common in the Vietnamese culture. ANS: B A cultural practice to rid the body of disease is descriptive of coining. The welts are created by repeatedly rubbing a coin on the child's oiled skin. The mother is attempting to rid the child's body of disease. The mother was engaged in an attempt to heal the child. This behavior is not child abuse, a cultural practice to treat enuresis or temper tantrums, or a disciplinary measure. PTS: 1 DIF: Cognitive Level: Comprehension OBJ: Nursing Process: Assessment MSC: Client Needs: Psychosocial Integrity 9. The father of a hospitalized child tells the nurse, “He can't have meat. We are Buddhist and vegetarians.” The nurse's best intervention is to: a. order the child a meatless tray. b. ask a Buddhist priest to visit. c. explain that hospital patients are exempt from dietary rules. d. help the parent understand that meat provides protein needed for healing. ANS: A It is essential for the nurse to respect the religious practices of the child and family. The nurse should arrange a dietary consultation to ensure that nutritionally complete vegetarian meals are prepared by the hospital kitchen. The nurse should be able to arrange for a vegetarian tray. The nurse should not encourage the child and parent to go against their religious beliefs. Nutritionally complete, acceptable vegetarian meals should be provided. PTS: 1 DIF: Cognitive Level: Application OBJ: Nursing Process: Implementation MSC: Client Needs: Psychosocial Integrity 10. In which cultural group is good health considered to be a balance between yin and yang? a. Asians b. Australian aborigines c. Native Americans d. African-Americans ANS: A In Chinese health beliefs, the forces termed yin and yang must be kept in balance to maintain health. This belief is not consistenNt UwRitShINAGuTsBtr.CalOiaMn aborigines, Native Americans, or African-Americans. PTS: 1 DIF: Cognitive Level: Comprehension OBJ: Nursing Process: Assessment MSC: Client Needs: Psychosocial Integrity Chapter 28: Developmental and Genetic Influences on Child Health Promotion Perry: Maternal Child Nursing Care, 6th Edition MULTIPLE CHOICE 1. The head-to-tail direction of growth is referred to as: a. cephalocaudal. b. proximodistal. c. mass to specific. d. sequential. ANS: A The first pattern of development is the head-to-tail, or cephalocaudal, direction. The head end of the organism develops first and is large and complex, whereas the lower end is smaller and simpler, and development takes place at a later time. Proximodistal, or near-to-far, is the second pattern of development. Limb buds develop before fingers and toes. Postnatally the child has control of the shoulder before achieving mastery of the hands. Mass to specific is not a specific pattern of development. In all dimensions of growth, a definite, sequential pattern is followed. PTS: 1 DIF: Cognitive Level: Comprehension OBJ: Nursing Process: Assessment MSC: Client Needs: Health Promotion and Maintenance 2. Which term refers to those times in an individual's life when he or she is more susceptible to positive or negative influences? a. Sensitive period b. Sequential period c. Terminal points d. Differentiation points ANS: A NURSINGTB.COM Sensitive periods are limited times during the process of growth when the organism will interact with a particular environment in a specific manner. These times make the organism more susceptible to positive or negative influences. The sequential period, terminal points, and differentiation points are developmental times that do not make the organism more susceptible to environmental interaction. PTS: 1 DIF: Cognitive Level: Comprehension OBJ: Nursing Process: Planning MSC: Client Needs: Health Promotion and Maintenance 3. An infant who weighs 7 lbs at birth would be expected to weigh how many pounds at age 1 year? a. 14 lbs b. 16 lbs c. 18 lbs d. 21 lbs ANS: D In general birth, weight triples by the end of the first year of life. For an infant who was 7 lbs at birth, 21 lbs would be the anticipated weight at the first birthday. Weights of 14, 16, and 18 lbs are less what would be expected for an infant with a birth weight of 7 lbs. PTS: 1 DIF: Cognitive Level: Comprehension OBJ: Nursing Process: Assessment MSC: Client Needs: Health Promotion and Maintenance 4. By what age does birth length usually double? a. 1 year b. 2 years c. 4 years d. 6 years ANS: C Linear growth or height occurs almost entirely as a result of skeletal growth and is considered a stable measurement of general growth. On average most children have doubled their birth length at age 4 years. One year and 2 years are too young for doubling of length. PTS: 1 DIF: Cognitive Level: Comprehension OBJ: Nursing Process: Assessment MSC: Client Needs: Health Promotion and Maintenance 5. How does the onset of the pubertal growth spurt compare in girls and boys? a. It occurs earlier in boys. b. It occurs earlier in girls. c. It is about the same in both boys and girls. d. In both boys and girls it depends on their growth in infancy. ANS: B NURSINGTB.COM Usually, the pubertal growth spurt begins earlier in girls. It typically occurs between the ages of 10 and 14 years for girls and 11 and 16 years for boys. The average earliest age at onset is 1 year earlier for girls. There does not appear to be a relation to growth during infancy. PTS: 1 DIF: Cognitive Level: Comprehension OBJ: Nursing Process: Assessment MSC: Client Needs: Health Promotion and Maintenance 6. A 13-year-old girl asks the nurse how much taller she will become. She has been growing about 2 inches per year but grew 4 inches this past year. Menarche recently occurred. The nurse should base her response on knowing that: a. growth cannot be predicted. b. the pubertal growth spurt lasts about 1 year. c. mature height is achieved when menarche occurs. d. approximately 95% of mature height is achieved when menarche occurs. ANS: D Although growth cannot be definitely predicted, at the time of the beginning of menstruation or the skeletal age of 13 years, most girls have grown to about 95% of their adult height. They may have some additional growth (5%) until the epiphyseal plates are closed. Responding that the pubertal growth spurt last about 1 year does not address the girl's question. Young women usually will grow approximately 5% more after the onset of menstruation. PTS: 1 DIF: Cognitive Level: Comprehension OBJ: Nursing Process: Assessment MSC: Client Needs: Health Promotion and Maintenance 7. A child's skeletal age is best determined by: a. assessment of dentition. b. assessment of height over time. c. facial bone development. d. radiographs of the hand and wrist. ANS: D The most accurate measure of skeletal age is radiologic examination of the growth plates. These are the epiphyseal cartilage plates. Radiographs of the hand and wrist provide the most useful screening to determine skeletal age. Age of tooth eruption varies considerably in children. It would not be a good determinant of skeletal age. Assessment of height over time will provide a record of the child's height, not skeletal age. Facial bone development does not reflect the child's skeletal age, which is determined by radiographic assessment. PTS: 1 DIF: Cognitive Level: Knowledge OBJ: Nursing Process: Assessment MSC: Client Needs: Health Promotion and Maintenance 8. Trauma to which site can result in a growth problem for children's long bones? a. Matrix b. Connective tissue c. Calcified cartilage d. Epiphyseal cartilage plate ANS: D The epiphyseal cartilage plate is the area of active growth. Bone injury at the epiphyseal plate can significantly affect subsequent growth and development. Trauma or infection can result in deformity. The matrix, connectiveNtUisRsSuIeN, GanTdB.cCaOlcMified cartilage are not areas of active growth. Trauma in these sites will not result in growth problems for the long bones. PTS: 1 DIF: Cognitive Level: Knowledge OBJ: Nursing Process: Diagnosis MSC: Client Needs: Health Promotion and Maintenance 9. Lymphoid tissues in children such as lymph nodes are: a. adult size by age 1 year. b. adult size by age 13 years. c. half their adult size by age 5 years. d. twice their adult size by age 10 to 12 years. ANS: D Lymph nodes increase rapidly and reach adult size at approximately age 6 years. They continue growing until they reach maximal development at age 10 to 12 years, which is twice their adult size. A rapid decline in size occurs until they reach adult size by the end of adolescence. PTS: 1 DIF: Cognitive Level: Knowledge OBJ: Nursing Process: Assessment MSC: Client Needs: Health Promotion and Maintenance 10. Which statement is true about the basal metabolic rate (BMR) in children? a. It is reduced by fever. b. It is slightly higher in boys than in girls at all ages. c. It increases with the age of child. d. It decreases as proportion of surface area to body mass increases. ANS: B The BMR is the rate of metabolism when the body is at rest. At all ages the rate is slightly higher in boys than in girls. The rate is increased by fever. The BMR is highest in infancy and then closely relates to the proportion of surface area to body mass. As the child grows, the proportion decreases progressively to maturity. PTS: 1 DIF: Cognitive Level: Comprehension OBJ: Nursing Process: Assessment MSC: Client Needs: Health Promotion and Maintenance 11. A mother reports that her 6-year-old child is highly active and irritable and that she has irregular habits and adapts slowly to new routines, people, or situations. According to Chess and Thomas, which category of temperament best describes this child? a. Easy child b. Difficult child c. Slow-to-warm-up child d. Fast-to-warm-up child ANS: B This is a description of difficult children, who compose about 10% of the population. Negative withdrawal responses are typical of this type of child, who requires a more structured environment. Mood expressions are usually intense and primarily negative. These children exhibit frequent periods of crying and often violent tantrums. Easy children are even tempered, regular, and predictable in their habits. They are open and adaptable to change. Approximately 40% of children fit this description. Slow-to-warm-up children typically react negatively and with mild intensity to new stimuli and adapt slowly with repeated contact. Approximately 10% of children fiNt UthRisSIdNeGsTcrBip.CtOioMn. Fast-to-warm-up children are not one of the categories identified by Chess and Thomas. PTS: 1 DIF: Cognitive Level: Comprehension OBJ: Nursing Process: Diagnosis MSC: Client Needs: Health Promotion and Maintenance 12. By the time children reach their 12 birthday, they should have learned to trust others and should have developed a sense of: a. identity. b. industry. c. integrity. d. intimacy. ANS: B Industry is the developmental task of school-age children. By age 12 years, children engage in tasks that they can carry through to completion. They learn to compete and cooperate with others, and they learn rules. Identity versus role confusion is the developmental task of adolescence. Integrity and intimacy are not developmental tasks of childhood. PTS: 1 DIF: Cognitive Level: Knowledge OBJ: Nursing Process: Assessment MSC: Client Needs: Health Promotion and Maintenance 13. The predominant characteristic of the intellectual development of the child ages 2 to 7 years is egocentricity. What best describes this concept? a. Selfishness b. Self-centeredness c. Preferring to play alone d. Inability to put self in another's place ANS: D According to Piaget, this age child is in the preoperational stage of development. Children interpret objects and events not in terms of their general properties but in terms of their relationships or their use to them. This egocentrism does not allow children of this age to put themselves in another's place. Selfishness, self-centeredness, and preferring to play alone do not describe the concept of egocentricity. PTS: 1 DIF: Cognitive Level: Knowledge OBJ: Nursing Process: Assessment MSC: Client Needs: Health Promotion and Maintenance 14. The nurse is observing parents playing with their 10-month-old daughter. What should the nurse recognize as evidence that the child is developing object permanence? a. She looks for the toy the parents hide under the blanket. b. She returns the blocks to the same spot on the table. c. She recognizes that a ball of clay is the same when flattened out. d. She bangs two cubes held in her hands. ANS: A Object permanence is the realization that items that leave the visual field still exist. When the infant searches for the toy under the blanket, it is an indication that object permanence has developed. Returning blocks to the same spot on a table is not an example of object permanence. Recognizing a ball of clay is the same when flat is an example of conservation, which occurs during the concrete operations stage from 7 to 11 years. Banging cubes together is a simple repetitive activity charNacUtRerSiIsNtiGcToBf.CdeOvMeloping a sense of cause and effect. PTS: 1 DIF: Cognitive Level: Knowledge OBJ: Nursing Process: Assessment MSC: Client Needs: Health Promotion and Maintenance 15. What is the characteristic of the preoperational stage of cognitive development? a. Thinking is logical. b. Thinking is concrete. c. Reasoning is inductive. d. Generalizations can be made. ANS: B Preoperational thinking is concrete and tangible. Children in this age-group cannot reason beyond the observable, and they lack the ability to make deductions or generalizations. Increasingly logical thought, inductive reasoning, and the ability to make generalizations are characteristic of the concrete operations stage of development, ages 7 to 11 years. PTS: 1 DIF: Cognitive Level: Comprehension OBJ: Nursing Process: Assessment MSC: Client Needs: Health Promotion and Maintenance 16. Which behavior is most characteristic of the concrete operations stage of cognitive development? a. Progression from reflex activity to imitative behavior. b. Inability to put oneself in another's place. c. Increasingly logical and coherent thought processes. d. Ability to think in abstract terms and draw logical conclusions. ANS: C During the concrete operations stage of development, which occurs approximately between ages 7 and 11 years, increasingly logical and coherent thought processes occur. This is characterized by the child's ability to classify, sort, order, and organize facts to use in problem solving. The progression from reflex activity to imitative behavior is characteristic of the sensorimotor stage of development. The inability to put oneself in another's place is characteristic of the preoperational stage of development. The ability to think in abstract terms and draw logical conclusions is characteristic of the formal operations stage of development. PTS: 1 DIF: Cognitive Level: Comprehension OBJ: Nursing Process: Assessment MSC: Client Needs: Health Promotion and Maintenance 17. According to Kohlberg, children develop moral reasoning as they mature. What is the most characteristic of a preschooler's stage of moral development? a. Obeying the rules of correct behavior is important. b. Showing respect for authority is important behavior. c. Behavior that pleases others is considered good. d. Actions are determined as good or bad in terms of their consequences. ANS: D Preschoolers are most likely to exhibit characteristics of Kohlberg's preconventional level of moral development. During this stage they are culturally oriented to labels of good or bad, right or wrong. Children integrate these concepts based on the physical or pleasurable consequences of their actions. Obeying rules of correct behavior, showing respect for authority, and knowing that behavior that pleases others is considered good are characteristic of Kohlberg's conventional level oNfUmRSoIrNaGl dTeBv.CeOloMpment. PTS: 1 DIF: Cognitive Level: Comprehension OBJ: Nursing Process: Planning MSC: Client Needs: Health Promotion and Maintenance 18. At what age do children tend to imitate the religious gestures and behaviors of others without understanding their significance? a. Toddlerhood b. Young school-age period c. Older school-age period d. Adolescence ANS: A Toddlerhood is a time of imitative behavior. Children will copy the behavior of others without comprehending any significance or meaning to the activities. During the school-age period most children develop a strong interest in religion. The existence of a deity is accepted, and petitions to an omnipotent being are important. Although adolescents become more skeptical and uncertain about religious beliefs, they do understand the significance of religious rituals. PTS: 1 DIF: Cognitive Level: Comprehension OBJ: Nursing Process: Planning MSC: Client Needs: Health Promotion and Maintenance 19. A toddler playing with sand and water would be participating in play. a. skill b. dramatic c. social-affective d. sense-pleasure ANS: D The toddler playing with sand and water is engaging in sense-pleasure play. This is characterized by nonsocial situations in which the child is stimulated by objects in the environment. Infants engage in skill play when they persistently demonstrate and exercise newly acquired abilities. Dramatic play is the predominant form of play in the preschool period. Children pretend and fantasize. Social-affective play is one of the first types of play in which infants engage. The infant responds to interactions with people. PTS: 1 DIF: Cognitive Level: Comprehension OBJ: Nursing Process: Assessment MSC: Client Needs: Health Promotion and Maintenance 20. In what type of play are children engaged in similar or identical activity without organization, division of labor, or mutual goal? a. Solitary b. Parallel c. Associative d. Cooperative ANS: C In associative play no group goal is present. Each child acts according to his or her own wishes. Although the children may be involved in similar activities, no organization, division of labor, leadership assignment, or mutual goal exists. Solitary play describes children playing alone with toys different from those used by other children in the same area. Parallel play describes children playing independently but being among other children. Cooperative play is organized. Children play in a grouNpUwRiStIhNoGtThBer.CcOhMildren who play activities for a common goal. PTS: 1 DIF: Cognitive Level: Comprehension OBJ: Nursing Process: Implementation MSC: Client Needs: Health Promotion and Maintenance 21. The nurse observes some children in the playroom. Which play situation exhibits the characteristics of parallel play? a. Kimberly and Amanda sharing clay to each make things. b. Brian playing with his truck next to Kristina playing with her truck. c. Adam playing a board game with Kyle, Steven, and Erich. d. Danielle playing with a music box on her mother's lap. ANS: B An example of parallel play is when both children are engaged in similar activities in proximity to each other; however, they are each engaged in their own play, such as Brian and Kristina playing with their own trucks side by side. Sharing clay is characteristic of associative play. A group of children playing a board game is characteristic of cooperative play. Playing alone on the mother's lap is an example of solitary play. PTS: 1 DIF: Cognitive Level: Analysis OBJ: Nursing Process: Assessment MSC: Client Needs: Health Promotion and Maintenance 22. Three children playing a board game would be an example of: a. solitary play. b. parallel play. c. associative play. d. cooperative play. ANS: D Using a board game requires cooperative play. The children must be able to play in a group and carry out the formal game. In solitary, parallel, and associative play, children do not play in a group with a common goal. PTS: 1 DIF: Cognitive Level: Comprehension OBJ: Nursing Process: Planning MSC: Client Needs: Health Promotion and Maintenance 23. Which function of play is a major component of play at all ages? a. Creativity b. Socialization c. Intellectual development d. Sensorimotor activity ANS: D Sensorimotor activity is a major component of play at all ages. Active play is essential for muscle development and allows the release of surplus energy. Through sensorimotor play, children explore their physical world by using tactile, auditory, visual, and kinesthetic stimulation. Creativity, socialization, and intellectual development are each functions of play that are major components at different ages. PTS: 1 DIF: Cognitive Level: Comprehension OBJ: Nursing Process: Assessment MSC: Client Needs: Health Promotion and Maintenance NURSINGTB.COM 24. What is probably the single most important influence on growth at all stages of development? a. Nutrition b. Heredity c. Culture d. Environment ANS: A Nutrition is the single most important influence on growth. Dietary factors regulate growth at all stages of development, and their effects are exerted in numerous and complex ways. Adequate nutrition is closely related to good health throughout life. Heredity, culture, and environment all contribute to the child's growth and development; however, good nutrition is essential throughout the life span for optimal health. PTS: 1 DIF: Cognitive Level: Application OBJ: Nursing Process: Planning MSC: Client Needs: Health Promotion and Maintenance 25. Which strategy would be the least appropriate for a child to use to cope? a. Learning problem solving b. Listening to music c. Having parents solve problems d. Using relaxation techniques Children respond to everyday stress by trying to change the circumstances or adjust to the circumstances the way they are. Strategies that provide relaxation and other stress-reduction techniques should be used. An inappropriate response would be for the parents to solve the problems. Some children develop socially unacceptable strategies such as lying, stealing, or cheating. Learning problem solving, listening to music, and using relaxation techniques are positive approaches for coping in children. PTS: 1 DIF: Cognitive Level: Application OBJ: Nursing Process: Implementation MSC: Client Needs: Psychosocial Integrity 26. The intrauterine environment can have a profound and permanent effect on the developing fetus with or without chromosome or gene abnormalities. Most adverse intrauterine effects are the result of teratogens. The nurse is cognizant that this group of agents does not include: a. accutane b. rubella c. amniotic bands d. alcohol ANS: C Amniotic bands are a congenital anomaly known as a “disruption” that occurs with the breakdown of previously normal tissue. Congenital amputations caused by amniotic bands are not the result of a teratogen. Other agents include Dilantin, warfarin, cytomegalovirus, radiation, and maternal PKU. PTS: 1 DIF: Cognitive Level: Comprehension OBJ: Nursing Process: Diagnosis MSC: Client Needs: Physiologic Integrity 27. The karyotype of a person is 47, XNYU,R+S2IN1G. TTBh.iCsOpMerson is a: a. normal male. b. male with Down syndrome. c. normal female. d. female with Turner syndrome. ANS: B This person is male because his sex chromosomes are XY. He has one extra copy of chromosome 21 (for a total of 47 instead of 46), resulting in Down syndrome. A normal male would have 46 chromosomes. A normal female would have 46 chromosomes and XX for the sex chromosomes. A female with Turner syndrome would have 45 chromosomes; the sex chromosomes would have just one X. PTS: 1 DIF: Cognitive Level: Comprehension OBJ: Nursing Process: Assessment MSC: Client Needs: Physiologic Integrity 28. Frequent developmental assessments are important for which reason? a. Stable developmental periods during infancy provide an opportunity to identify any delays or deficits. b. Infants need stimulation specific to the stage of development. c. Critical periods of development occur during childhood. d. Child development is unpredictable and needs monitoring. Critical periods are blocks of time during which children are ready to master specific developmental tasks. The earlier that delays in development are discovered and intervention initiated, the less dramatic their effect will be. Infancy is a dynamic time of development that requires frequent evaluations to assess appropriate developmental progress. Infants in a nurturing environment will develop appropriately and will not necessarily need stimulation specific to their developmental stage. Normal growth and development are orderly and proceed in a predictable pattern on the basis of each individual's abilities and potentials. PTS: 1 DIF: Cognitive Level: Comprehension OBJ: Nursing Process: Assessment MSC: Client Needs: Health Promotion and Maintenance 29. The theorist who viewed developmental progression as a lifelong series of conflicts that need resolution is: a. Erikson. b. Freud. c. Kohlberg. d. Piaget. ANS: A Erik Erikson viewed development as a series of conflicts affected by social and cultural factors. Each conflict must be resolved for the child to progress emotionally, with unsuccessful resolution leaving the child emotionally disabled. Sigmund Freud proposed a psychosexual theory of development. He proposed that certain parts of the body assume psychological significance as foci of sexual energy. The foci shift as the individual moves through the different stages (oral, anal, phallic, latency, and genital) of development. Lawrence Kohlberg described moral development as having three levels (preconventional, conventional, and postconventionNalU).RHSIiNs GthTeBo.rCyOcMlosely parallels Piaget's. Jean Piaget's cognitive theory interprets how children learn and think and how this thinking progresses and differs from adult thinking. Stages of his theory include sensorimotor, preoperations, concrete operations, and formal operations. PTS: 1 DIF: Cognitive Level: Comprehension OBJ: Nursing Process: Assessment MSC: Client Needs: Health Promotion and Maintenance 30. Which “expected outcome” would be developmentally appropriate for a hospitalized 4- year-old child? a. The child will be dressed and fed by the parents. b. The child will independently ask for play materials or other personal needs. c. The child will be able to verbalize an understanding of the reason for the hospitalization. d. The child will have a parent stay in the room at all times. ANS: B Erikson identifies initiative as a developmental task for the preschool child. Initiating play activities and asking for play materials or assistance with personal needs demonstrate developmental appropriateness. Parents need to foster appropriate developmental behavior in the 4-year-old child. Dressing and feeding the child do not encourage independent behavior. A 4-year-old child cannot be expected to cognitively understand the reason for hospitalization. Expecting the child to verbalize an understanding for hospitalization is an inappropriate outcome. Parents staying with the child throughout a hospitalization are an inappropriate outcome. Although children benefit from parental involvement, parents may not have the support structure to stay in the room with the child at all times. PTS: 1 DIF: Cognitive Level: Application OBJ: Nursing Process: Planning MSC: Client Needs: Health Promotion and Maintenance MULTIPLE RESPONSE 1. Play serves many purposes. In teaching parents about appropriate activities, the nurse should inform them that play serves the following function: (Select all that apply.) a. intellectual development. b. physical development. c. self-awareness. d. creativity. e. temperament development. ANS: A, C, D A common statement is that play is the work of childhood. Intellectual development is enhanced through the manipulation and exploration of objects. Self-awareness is the process of developing a self-identity. ThisNpUrRocSeINssGiTsBf.aCcOilMitated through play. In addition, creativity is developed through the experimentation characteristic of imaginative play. Physical development depends on many factors; play is not one of them. Temperament refers to behavioral tendencies that are observable from the time of birth. The actual behaviors but not the child's temperament attributes may be modified through play. PTS: 1 DIF: Cognitive Level: Comprehension OBJ: Nursing Process: Assessment MSC: Client Needs: Health Promotion and Maintenance MATCHING Match the sequence of cephalocaudal development that the nurse expects to find in the normal infant with the appropriate step numbers. Begin with the first development expected, sequencing to the final. a. Crawl b. Creep c. Stand d. Walk 1. Step 1 2. Step 2 3. Step 3 4. Step 4 1. ANS: A PTS: 1 DIF: Cognitive Level: Application OBJ: Nursing Process: Implementation MSC: Client Needs: Health Promotion and Maintenance NOT: Cephalocaudal development is from head to tail. Infants achieve structural control of the head before they have control of their trunks and extremities, they lift their head while prone, obtain complete head control, sit unsupported, crawl, and walk sequentially. 2. ANS: B PTS: 1 DIF: Cognitive Level: Application OBJ: Nursing Process: Implementation MSC: Client Needs: Health Promotion and Maintenance NOT: Cephalocaudal development is from head to tail. Infants achieve structural control of the head before they have control of their trunks and extremities, they lift their head while prone, obtain complete head control, sit unsupported, crawl, and walk sequentially. 3. ANS: C PTS: 1 DIF: Cognitive Level: Application OBJ: Nursing Process: Implementation MSC: Client Needs: Health Promotion and Maintenance NOT: Cephalocaudal development is from head to tail. Infants achieve structural control of the head before they have control of their trunks and extremities, they lift their head while prone, obtain complete head control, sit unsupported, crawl, and walk sequentially. 4. ANS: D PTS: 1 DIF: Cognitive Level: Application OBJ: Nursing Process: Implementation MSC: Client Needs: Health Promotion and Maintenance NOT: Cephalocaudal development is from head to tail. Infants achieve structural control of the head before they have control of their trunks and extremities, they lift their head while prone, obtain complete head control, sit unsupported, crawl, and walk sequentially. NURSINGTB.COM Chapter 29: Communication and Physical Assessment of the Child and Family Perry: Maternal Child Nursing Care, 6th Edition MULTIPLE CHOICE 1. The nurse is seeing an adolescent boy and his parents in the clinic for the first time. What should the nurse do first? a. Introduce himself or herself. b. Make the family comfortable. c. Explain the purpose of the interview. d. Give an assurance of privacy. ANS: A The first thing that nurses must do is to introduce themselves to the patient and family. Parents and other adults should be addressed with appropriate titles unless they specify a preferred name. During the initial part of the interview the nurse should include general conversation to help make the family feel at ease. Next, the purpose of the interview and the nurse's role should be clarified. The interview should take place in an environment as free of distraction as possible. In addition, the nurse should clarify which information will be shared with other members of the health care team and any limits to the confidentiality. PTS: 1 DIF: Cognitive Level: Application OBJ: Nursing Process: Implementation MSC: Client Needs: Psychosocial Integrity 2. Which action is most likely to encourage parents to talk about their feelings related to their child's illness? a. Be sympathetic. b. Use direct questions. c. Use open-ended questions. d. Avoid periods of silence. ANS: C NURSINGTB.COM Closed-ended questions should be avoided when attempting to elicit parents' feelings. Open-ended questions require the parent to respond with more than a brief answer. Sympathy is having feelings or emotions in common with another person rather than understanding those feelings (empathy). Sympathy is not therapeutic in the helping relationship. Direct questions may obtain limited information. In addition, the parent may consider them threatening. Silence can be an effective interviewing tool. It allows sharing of feelings in which two or more people absorb the emotion in depth. Silence permits the interviewee to sort out thoughts and feelings and search for responses to questions. PTS: 1 DIF: Cognitive Level: Application OBJ: Nursing Process: Implementation MSC: Client Needs: Psychosocial Integrity 3. What is the single most important factor to consider when communicating with children? a. The child's physical condition b. The presence or absence of the child's parent c. The child's developmental level d. The child's nonverbal behaviors ANS: C The nurse must be aware of the child's developmental stage to engage in effective communication. The use of both verbal and nonverbal communication should be appropriate to the developmental level. Although the child's physical condition is a consideration, developmental level is much more important. The parents' presence is important when communicating with young children, but it may be detrimental when speaking with adolescents. Nonverbal behaviors vary in importance based on the child's developmental level. PTS: 1 DIF: Cognitive Level: Comprehension OBJ: Nursing Process: Implementation MSC: Client Needs: Psychosocial Integrity 4. What is an important consideration for the nurse who is communicating with a very young child? a. Speak loudly, clearly, and directly. b. Use transition objects such as a doll. c. Disguise own feelings, attitudes, and anxiety. d. Initiate contact with the child when the parent is not present. ANS: B Using a transition object allows the young child an opportunity to evaluate an unfamiliar person (the nurse). This facilitates communication with this age child. Speaking loudly, clearly, and directly tends to increase anxiety in very young children. The nurse must be honest with the child. Attempts at deception lead to a lack of trust. Whenever possible, the parent should be present for interactions with young children. PTS: 1 DIF: Cognitive Level: Comprehension OBJ: Nursing Process: ImplementaNtioUnRSINMGSTCB.:CCOlMient Needs: Psychosocial Integrity 5. Which age-group is most concerned with body integrity? a. Toddler b. Preschooler c. School-age child d. Adolescent ANS: C School-age children have a heightened concern about body integrity. They place importance and value on their bodies and are overly sensitive to anything that constitutes a threat or suggestion of injury. Body integrity is not as important a concern to children in the toddler, preschooler, and adolescent age-groups. PTS: 1 DIF: Cognitive Level: Comprehension OBJ: Nursing Process: Planning MSC: Client Needs: Health Promotion and Maintenance 6. An 8-year-old girl asks the nurse how the blood pressure apparatus works. The most appropriate nursing action is to: a. ask her why she wants to know. b. determine why she is so anxious. c. explain in simple terms how it works. d. tell her she will see how it works as it is used. ANS: C School-age children require explanations and reasons for everything. They are interested in the functional aspect of all procedures, objects, and activities. It is appropriate for the nurse to explain how equipment works and what will happen to the child. A nurse should respond positively to requests for information about procedures and health information. By not responding, the nurse may be limiting communication with the child. The child is not exhibiting anxiety, just requesting clarification of what will be occurring. The nurse must explain how the blood pressure cuff works so the child can then observe during the procedure. PTS: 1 DIF: Cognitive Level: Comprehension OBJ: Nursing Process: Implementation MSC: Client Needs: Health Promotion and Maintenance 7. When the nurse interviews an adolescent, it is especially important to: a. focus the discussion on the peer group. b. allow an opportunity to express feelings. c. emphasize that confidentiality will always be maintained. d. use the same type of language as the adolescent. ANS: B Adolescents, like all children, need an opportunity to express their feelings. Often they will interject feelings into their words. The nurse must be alert to the words and feelings expressed. Although the peer group is important to this age-group, the focus of the interview should be on the adolescent. The nurse should clarify which information will be shared with other members of the health care team and any limits to confidentiality. The nurse should maintain a professional relationship with adolescents. To avoid misinterpretation of words and phrases that the adolescent may use, the nurse should clarify terms frequently. PTS: 1 DIF: Cognitive Level: Comprehension OBJ: Nursing Process: Planning MSC: Client Needs: Psychosocial InNteUgRriStyINGTB.COM 8. The nurse is taking a health history on an adolescent. Which best describes how the chief complaint should be determined? a. Ask for a detailed listing of symptoms. b. Ask the adolescent, “Why did you come here today?” c. Use what the adolescent says to determine, in correct medical terminology, what the problem is. d. Interview the parent away from the adolescent to determine the chief complaint. ANS: B The chief complaint is the specific reason for the child's visit to the clinic, office, or hospital. Because the adolescent is the focus of the history, this is an appropriate way to determine the chief complaint. A listing of symptoms will make it difficult to determine the chief complaint. The adolescent should be prompted to tell which symptom caused him or her to seek help at this time. The chief complaint is usually written in the words that the parent or adolescent uses to describe the reason for seeking help. The parent and adolescent may be interviewed separately, but the nurse should determine the reason the adolescent is seeking attention at this time. PTS: 1 DIF: Cognitive Level: Application OBJ: Nursing Process: Assessment MSC: Client Needs: Health Promotion and Maintenance 9. Where in the health history should the nurse describe all details related to the chief complaint? a. Past history b. Chief complaint c. Present illness d. Review of systems ANS: C The history of the present illness is a narrative of the chief complaint from its earliest onset through its progression to the present. The focus of the present illness is on all factors relevant to the main problem, even if they have disappeared or changed during the onset, interval, and present. Past history refers to information that relates to previous aspects of the child's health, not to the current problem. The chief complaint is the specific reason for the child's visit to the clinic, office, or hospital. It does not contain the narrative portion describing the onset and progression. The review of systems is a specific review of each body system. PTS: 1 DIF: Cognitive Level: Comprehension OBJ: Nursing Process: Implementation MSC: Client Needs: Health Promotion and Maintenance 10. The nurse is interviewing the mother of an infant. She reports, “I had a difficult delivery, and my baby was born prematurely.” This information should be recorded under which heading? a. Birth history b. Present illness c. Chief complaint d. Review of systems ANS: A The birth history refers to information that relates to previous aspects of the child's health, not to the current problem. The mother's difficult delivery and prematurity are important parts of the past history of an infant. The hNisUtRoSryINoGfTthBe.CpOrMesent illness is a narrative of the chief complaint from its earliest onset through its progression to the present. Unless the chief complaint is directly related to the prematurity, this information is not included in the history of present illness. The chief complaint is the specific reason for the child's visit to the clinic, office, or hospital. It would not include the birth information. The review of systems is a specific review of each body system. It does not include the premature birth. Sequelae such as pulmonary dysfunction would be included. PTS: 1 DIF: Cognitive Level: Comprehension OBJ: Nursing Process: Assessment MSC: Client Needs: Health Promotion and Maintenance 11. When interviewing the mother of a 3-year-old child, the nurse asks about developmental milestones such as the age of walking without assistance. This should be considered because these milestones are: a. unnecessary information because the child is age 3 years. b. an important part of the family history. c. an important part of the child's past growth and development. d. an important part of the child's review of systems. ANS: C Information about the attainment of developmental milestones is important to obtain. It provides data about the child's growth and development that should be included in the history. Developmental milestones provide important information about the child's physical, social, and neurologic health. The developmental milestones are specific to this child. If pertinent, attainment of milestones by siblings would be included in the family history. The review of systems does not include the developmental milestones. PTS: 1 DIF: Cognitive Level: Comprehension OBJ: Nursing Process: Assessment MSC: Client Needs: Health Promotion and Maintenance 12. The nurse is taking a sexual history on an adolescent girl. The best way to determine whether she is sexually active is to: a. ask her, “Are you sexually active?” b. ask her, “Are you having sex with anyone?” c. ask her, “Are you having sex with a boyfriend?” d. ask both the girl and her parent if she is sexually active. ANS: B Asking the adolescent girl if she is having sex with anyone is a direct question that is well understood. The phrase sexually active is broadly defined and may not provide specific information to the nurse to provide necessary care. The word anyone is preferred to using gender-specific terms such as boyfriend or girlfriend. Because homosexual experimentation may occur, it is preferable to use gender-neutral terms. Questioning about sexual activity should occur when the adolescent is alone. PTS: 1 DIF: Cognitive Level: Application OBJ: Nursing Process: Assessment MSC: Client Needs: Health Promotion and Maintenance NURSINGTB.COM 13. When doing a nutritional assessment on an Hispanic family, the nurse learns that their diet consists mainly of vegetables, legumes, and starches. The nurse should recognize that this diet: a. indicates that they live in poverty. b. is lacking in protein. c. may provide sufficient amino acids. d. should be enriched with meat and milk. ANS: C The diet that contains vegetable, legumes, and starches may provide sufficient essential amino acids, even though the actual amount of meat or dairy protein is low. Many cultures use diets that contain this combination of foods. It does not indicate poverty. Combinations of foods contain the essential amino acids necessary for growth. A dietary assessment should be done, but many vegetarian diets are sufficient for growth. PTS: 1 DIF: Cognitive Level: Application OBJ: Nursing Process: Assessment MSC: Client Needs: Health Promotion and Maintenance 14. Which parameter correlates best with measurements of the body's total protein stores? a. Height b. Weight c. Skin-fold thickness d. Upper arm circumference ANS: D Upper arm circumference is correlated with measurements of total muscle mass. Muscle serves as the body's major protein reserve and is considered an index of the body's protein stores. Height is reflective of past nutritional status. Weight is indicative of current nutritional status. Skin-fold thickness is a measurement of the body's fat content. PTS: 1 DIF: Cognitive Level: Comprehension OBJ: Nursing Process: Assessment MSC: Client Needs: Health Promotion and Maintenance 15. An appropriate approach to performing a physical assessment on a toddler is to: a. always proceed in a head-to-toe direction. b. perform traumatic procedures first. c. use minimal physical contact initially. d. demonstrate use of equipment. ANS: C Parents can remove the child's clothing, and the child can remain on the parent's lap. The nurse should use minimal physical contact initially to gain the child's cooperation. The head-to-toe assessment can be done in older children but usually must be adapted in younger children. Traumatic procedures should always be performed last. These will most likely upset the child and inhibit cooperation. The nurse should introduce the equipment slowly. The child can inspect the equipment, but demonstrations are usually too complex for this age-group. PTS: 1 DIF: Cognitive Level: Comprehension OBJ: Nursing Process: Assessment MSC: Client Needs: Health Promotion and Maintenance 16. With the National Center for HealNthUSRStaItNisGtTicBs.C(NOCMHS) criteria, which body mass index (BMI)–for-age percentile indicates a risk for being overweight? a. 10th percentile b. 9th percentile c. 85th percentile d. 95th percentile ANS: C Children who have BMI-for-age greater than or equal to the 85th percentile and less than the 95th percentile are at risk for being overweight. Children in the 9th and 10th percentiles are within normal limits. Children who are greater than or equal to the 95th percentile are considered overweight. PTS: 1 DIF: Cognitive Level: Comprehension OBJ: Nursing Process: Diagnosis MSC: Client Needs: Health Promotion and Maintenance 17. Which tool measures body fat most accurately? a. Stadiometer b. Calipers c. Cloth tape measure d. Paper or metal tape measure ANS: B Calipers are used to measure skin-fold thickness, which is an indicator of body fat content. Stadiometers are used to measure height. Cloth tape measures should not be used because they can stretch. Paper or metal tape measures can be used for recumbent lengths and other body measurements that must be made. PTS: 1 DIF: Cognitive Level: Comprehension OBJ: Nursing Process: Assessment MSC: Client Needs: Health Promotion and Maintenance 18. By what age do the head and chest circumferences generally become equal? a. 1 month b. 6 to 9 months c. 1 to 2 years d. 2.5 to 3 years ANS: C Head circumference begins larger than chest circumference. Between ages 1 and 2 years, they become approximately equal. Head circumference is larger than chest circumference at ages 1 month and 6 to 9 months. Chest circumference is larger than head circumference at age 2.5 to 3 years. PTS: 1 DIF: Cognitive Level: Comprehension OBJ: Nursing Process: Assessment MSC: Client Needs: Health Promotion and Maintenance 19. The earliest age at which a satisfactory radial pulse can be taken in children is: a. 1 year. b. 2 years. c. 3 years. d. 6 years. ANS: B NURSINGTB.COM Satisfactory radial pulses can be used in children older than 2 years. In infants and young children the apical pulse is more reliable. The radial pulse can be used for assessment at ages 3 and 6 years. PTS: 1 DIF: Cognitive Level: Comprehension OBJ: Nursing Process: Assessment MSC: Client Needs: Health Promotion and Maintenance 20. Where is the best place to observe for the presence of petechiae in dark-skinned individuals? a. Face b. Buttocks c. Oral mucosa d. Palms and soles ANS: C Petechiae, small distinct pinpoint hemorrhages, are difficult to see in dark skin unless they are in the mouth or conjunctiva. PTS: 1 DIF: Cognitive Level: Comprehension OBJ: Nursing Process: Assessment MSC: Client Needs: Health Promotion and Maintenance 21. When palpating the child's cervical lymph nodes, the nurse notes that they are tender, enlarged, and warm. The best explanation for this is: a. some form of cancer. b. local scalp infection common in children. c. infection or inflammation distal to the site. d. infection or inflammation close to the site. ANS: D Small nontender nodes are normal. Tender, enlarged, and warm lymph nodes may indicate infection or inflammation close to their location. Tender lymph nodes do not usually indicate cancer. A scalp infection usually does not cause inflamed lymph nodes. The lymph nodes close to the site of inflammation or infection would be inflamed. PTS: 1 DIF: Cognitive Level: Analysis OBJ: Nursing Process: Assessment MSC: Client Needs: Health Promotion and Maintenance 22. The nurse has just started assessing a young child who is febrile and appears very ill. There is hyperextension of the child's head (opisthotonos) with pain on flexion. The most appropriate action is to: a. refer for immediate medical evaluation. b. continue the assessment to determine the cause of neck pain. c. ask the parent when the child's neck was injured. d. record “head lag” on the assessment record and continue the assessment of the child. ANS: A These symptoms indicate meningeal irritation and need immediate evaluation. Continuing the assessment is not necessary. No indication of injury is present. This is not descriptive of head lag. NURSINGTB.COM PTS: 1 DIF: Cognitive Level: Analysis OBJ: Nursing Process: Assessment MSC: Client Needs: Health Promotion and Maintenance 23. The nurse should expect the anterior fontanel to close at age: a. 2 months. b. 2 to 4 months. c. 6 to 8 months. d. 12 to 18 months. ANS: D Ages 2 through 8 months are too early. The expected closure of the anterior fontanel occurs between ages 12 and 18 months; if it closes at these earlier ages, the child should be referred for further evaluation. PTS: 1 DIF: Cognitive Level: Comprehension OBJ: Nursing Process: Assessment MSC: Client Needs: Health Promotion and Maintenance 24. During a funduscopic examination of a school-age child, the nurse notes a brilliant, uniform red reflex in both eyes. The nurse should recognize that this is: a. a normal finding. b. an abnormal finding; the child needs referral to an ophthalmologist. c. a sign of a possible visual defect; the child needs vision screening. d. a sign of small hemorrhages, which usually resolve spontaneously. ANS: A A brilliant, uniform red reflex is an important normal and expected finding. It rules out many serious defects of the cornea, aqueous chamber, lens, and vitreous chamber. PTS: 1 DIF: Cognitive Level: Knowledge OBJ: Nursing Process: Assessment MSC: Client Needs: Health Promotion and Maintenance 25. Binocularity, the ability to fixate on one visual field with both eyes simultaneously, is normally present by what age? a. 1 month b. 3 to 4 months c. 6 to 8 months d. 12 months ANS: B Binocularity is usually achieved by ages 3 to 4 months. Age 1 month is too young for binocularity. If binocularity is not achieved by 6 months, the child must be observed for strabismus. PTS: 1 DIF: Cognitive Level: Comprehension OBJ: Nursing Process: Assessment MSC: Client Needs: Health Promotion and Maintenance 26. The most frequently used test for measuring visual acuity is the: a. Denver Eye Screening test. b. Allen picture card test. c. Ishihara vision test. d. Snellen letter chart. ANS: D NURSINGTB.COM The Snellen letter chart, which consists of lines of letters of decreasing size, is the most frequently used test for visual acuity. Single cards (Denver—letter E; Allen—pictures) are used for children age 2 years and older who are unable to use the Snellen letter chart. The Ishihara vision test is used for color vision. PTS: 1 DIF: Cognitive Level: Comprehension OBJ: Nursing Process: Assessment MSC: Client Needs: Health Promotion and Maintenance 27. The nurse is testing an infant's visual acuity. By what age should the infant be able to fix on and follow a target? a. 1 month b. 1 to 2 months c. 3 to 4 months d. 6 months ANS: C Visual fixation and following a target should be present by ages 3 to 4 months. Ages 1 to 2 months are too young for this developmental milestone. If the infant is not able to fix and follow by 6 months of age, further ophthalmologic evaluation is needed. PTS: 1 DIF: Cognitive Level: Comprehension OBJ: Nursing Process: Assessment MSC: Client Needs: Health Promotion and Maintenance 28. The appropriate placement of a tongue blade for assessment of the mouth and throat is the: a. the center back area of the tongue. b. the side of the tongue. c. against the soft palate. d. on the lower jaw. ANS: B The side of the tongue is the correct position. It avoids the gag reflex yet allows visualization. Placement on the center back area of the tongue elicits the gag reflex. Against the soft palate and on the lower jaw are not appropriate places for the tongue blade. PTS: 1 DIF: Cognitive Level: Comprehension OBJ: Nursing Process: Assessment MSC: Client Needs: Health Promotion and Maintenance 29. What type of breath sound is normally heard over the entire surface of the lungs, except for the upper intrascapular area and the area beneath the manubrium? a. Vesicular b. Bronchial c. Adventitious d. Bronchovesicular ANS: A Vesicular breath sounds are heard over the entire surface of lungs, with the exception of the upper intrascapular area and the area beneath the manubrium. Bronchial breath sounds are heard only over the trachea near the suprasternal notch. Adventitious breath sounds are not usually heard over the chest. These sounds occur in addition to normal or abnormal breath sounds. Bronchovesicular breath sounds are heard over the manubrium and in the upper intrascapular regions where tracheNaUaRnSdINbGroTnBc.hCiObMifurcate. PTS: 1 DIF: Cognitive Level: Comprehension OBJ: Nursing Process: Assessment MSC: Client Needs: Health Promotion and Maintenance 30. Which term is used to describe breath sounds that are produced as air passes through narrowed passageways? a. Rubs b. Rattles c. Wheezes d. Crackles ANS: C Wheezes are produced as air passes through narrowed passageways. The sound is similar when the narrowing is caused by exudates, inflammation, spasm, or tumor. Rubs are the sound created by the friction of one surface rubbing over another. Pleural friction rub is caused by inflammation of the pleural space. Rattles is the term formerly used for crackles. Crackles are the sounds made when air passes through fluid or moisture. PTS: 1 DIF: Cognitive Level: Comprehension OBJ: Nursing Process: Assessment MSC: Client Needs: Health Promotion and Maintenance 31. The nurse must assess a child's capillary refilling time. This can be accomplished by: a. inspecting the chest. b. auscultating the heart. c. palpating the apical pulse. d. palpating the skin to produce a slight blanching. ANS: D Capillary refilling time is assessed by pressing lightly on the skin to produce blanching and then noting the amount of time it takes for the blanched area to refill. Inspecting the chest, auscultating the heart, and palpating the apical pulse will not provide an assessment of capillary filling time. PTS: 1 DIF: Cognitive Level: Comprehension OBJ: Nursing Process: Assessment MSC: Client Needs: Health Promotion and Maintenance 32. What heart sound is produced by vibrations within the heart chambers or in the major arteries from the back-and-forth flow of blood? a. S1, S2 b. S3, S4 c. Murmur d. Physiologic splitting ANS: C Murmurs are the sounds that are produced in the heart chambers or major arteries from the back-and-forth flow of blood. S1 is the closure of the tricuspid and mitral valves, and S2 is the closure of the pulmonic and aortic valves, and both are considered normal heart sounds. S3 is a normal heart sound sometimes heard in children. S4 is rarely heard as a normal heart sound. If heard, medical evaluation is required. Physiologic splitting is the distinction of the two sounds in S2, which widens on inspiration. It is a significant normal finding. PTS: 1 DIF: CognitiNveURLSevINelG: TCBo.mCOprMehension OBJ: Nursing Process: Assessment MSC: Client Needs: Health Promotion and Maintenance 33. The nurse has a 2-year-old boy sit in “tailor” position during palpation for the testes. The rationale for this position is that: a. it prevents cremasteric reflex. b. undescended testes can be palpated. c. this tests the child for an inguinal hernia. d. the child does not yet have a need for privacy. ANS: A The tailor position stretches the muscle responsible for the cremasteric reflex. This prevents its contraction, which pulls the testes into the pelvic cavity. Undescended testes cannot be predictably palpated. Inguinal hernias are not detected by this method. This position is used for inhibiting the cremasteric reflex. Privacy should always be provided for children. PTS: 1 DIF: Cognitive Level: Comprehension OBJ: Nursing Process: Assessment MSC: Client Needs: Health Promotion and Maintenance 34. During examination of a toddler's extremities, the nurse notes that the child is bowlegged. The nurse should recognize that this finding is: a. abnormal and requires further investigation. b. abnormal unless it occurs in conjunction with knock-knee. c. normal if the condition is unilateral or asymmetric. d. normal because the lower back and leg muscles are not yet well developed. ANS: D Lateral bowing of the tibia (bowlegged) is common in toddlers when they begin to walk, not an abnormal finding. It usually persists until all of their lower back and leg muscles are well developed. Further evaluation is needed if it persists beyond ages 2 to 3 years, especially in African-American children. PTS: 1 DIF: Cognitive Level: Comprehension OBJ: Nursing Process: Diagnosis MSC: Client Needs: Health Promotion and Maintenance 35. Kimberly is having a checkup before starting kindergarten. The nurse asks her to do the “finger-to-nose” test. The nurse is testing for: a. deep tendon reflexes. b. cerebellar function. c. sensory discrimination. d. ability to follow directions. ANS: B The finger-to-nose-test is an indication of cerebellar function. This test checks balance and coordination. Each deep tendon reflex is tested separately. Each sense is tested separately. Although this test enables the nurse to evaluate the child's ability to follow directions, it is used primarily for cerebellar function. PTS: 1 DIF: Cognitive Level: Comprehension OBJ: Nursing Process: Assessment MSC: Client Needs: Health Promotion and Maintenance 36. The nurse is meeting a 5-year-old child for the first time and would like the child to cooperate during a dressing change. The nurNseURdeScINidGeTsBto.CdOoMa simple magic trick using gauze. This should be interpreted as: a. inappropriate, because of child's age. b. a way to establish rapport. c. too distracting, when cooperation is important. d. acceptable, if there is adequate time. ANS: B A magic trick or other simple game may help alleviate anxiety for a 5-year-old. It is an excellent method to build rapport and facilitate cooperation during a procedure. Magic tricks appeal to the natural curiosity of young children. The nurse should establish rapport with the child. Failure to do so may cause the procedure to take longer and be more traumatic. PTS: 1 DIF: Cognitive Level: Analysis OBJ: Nursing Process: Communication, Documentation MSC: Client Needs: Psychosocial Integrity 37. During a routine health assessment, the nurse notes that an 8-month-old infant has significant head lag. Which is the nurse's most appropriate action? a. Teach the parents appropriate exercises. b. Recheck head control at the next visit. c. Refer the child for further evaluation. d. Refer the child for further evaluation if the anterior fontanel is still open. ANS: C Significant head lag after age 6 months strongly indicates cerebral injury and is referred for further evaluation. Reduction of head lag is part of normal development. Exercises will not be effective. The lack of achievement of this developmental milestone must be evaluated. PTS: 1 DIF: Cognitive Level: Application OBJ: Nursing Process: Assessment MSC: Client Needs: Health Promotion and Maintenance MULTIPLE RESPONSE 1. The nurse must check vital signs on a 2-year-old boy who is brought to the clinic for his 24-month checkup. Which criteria should the nurse use in determining the appropriate-size blood pressure cuff? (Select all that apply) a. The cuff is labeled “toddler.” b. The cuff bladder width is approximately 40% of the circumference of the upper arm. c. The cuff bladder length covers 80% to 100% of the circumference of the upper arm. d. The cuff bladder covers 50% to 66% of the length of the upper arm. ANS: B, C Research has demonstrated that cuff selection with a bladder width that is 40% of the arm circumference will usually have a bladder length that is 80% to 100% of the upper arm circumference. This size cuff will most accurately reflect measured radial artery pressure. The name of the cuff is a representative size that may not be suitable for any individual child. Choosing a cuff by limb circumference more accurately reflects arterial pressure than choosing a cuff by length. NURSINGTB.COM PTS: 1 DIF: Cognitive Level: Comprehension OBJ: Nursing Process: Assessment MSC: Client Needs: Health Promotion and Maintenance 2. Which data would be included in a health history? (Select all that apply.) a. Review of systems b. Physical assessment c. Sexual history d. Growth measurements e. Nutritional assessment f. Family medical history ANS: A, C, E, F The review of systems, sexual history, nutritional assessment, and family medical history are part of the health history. Physical assessment and growth measurements are components of the physical examination. PTS: 1 DIF: Cognitive Level: Application OBJ: Nursing Process: Assessment MSC: Client Needs: Health Promotion and Maintenance 3. A school nurse is screening children for scoliosis. Which assessment findings should the nurse expect to observe for scoliosis? (Select all that apply.) a. Complaints of a sore back b. Asymmetry of the shoulders c. An uneven hemline d. Inability to bend at the waist e. Unequal waist angles ANS: B, C, E The assessment findings associated with scoliosis include asymmetry of the shoulder and hips, trouser pant leg length appearing shorter on one side, or an uneven hemline on a skirt, indicating unequal leg length. The child may also complain of a sore back. The child is able to bend at the waist adequately. PTS: 1 DIF: Cognitive Level: Application OBJ: Nursing Process: Evaluation MSC: Client Needs: Health Promotion and Maintenance 4. A nurse is performing an assessment on a school-age child. Which findings suggest the child is receiving an excess of vitamin A? (Select all that apply.) a. Delayed sexual development b. Edema c. Pruritus d. Jaundice e. Paresthesia ANS: A, C, D Excess vitamin A can cause delayed sexual development, pruritus, and jaundice. Edema is seen with excess sodium. Paresthesia occurs with excess riboflavin. PTS: 1 DIF: Cognitive Level: Application OBJ: Nursing Process: Assessment MSC: Client Needs: Health Promotion and Maintenance 5. A nurse is planning to use an interNpUreRtSeIrNdGuTriBn.gCOaMhealth history interview of a non-English speaking patient and family. Which nursing care guidelines should the nurse include when using an interpreter? (Select all that apply.) a. Elicit one answer at a time. b. Interrupt the interpreter if the response from the family is lengthy. c. Comments to the interpreter about the family should be made in English. d. Arrange for the family to speak with the same interpreter, if possible. e. Introduce the interpreter to the family. ANS: A, D, E When using an interpreter, the nurse should pose questions to elicit only one answer at a time, such as: “Do you have pain?” rather than “Do you have any pain, tiredness, or loss of appetite?” Refrain from interrupting family members and the interpreter while they are conversing. Introduce the interpreter to family and allow some time before the interview for them to become acquainted. Refrain from interrupting family members and the interpreter while they are conversing. Avoid commenting to the interpreter about family members because they may understand some English. PTS: 1 DIF: Cognitive Level: Application OBJ: Nursing Process: Assessment MSC: Client Needs: Health Promotion and Maintenance MATCHING Match the assessment examination techniques used when performing an abdominal assessment with the sequential step numbers. Begin with the first technique and end with the last. a. Auscultation b. Palpation c. Inspection d. Percussion 1. Step 1 2. Step 2 3. Step 3 4. Step 4 1. ANS: C PTS: 1 DIF: Cognitive Level: Application OBJ: Nursing Process: Assessment MSC: Client Needs: Health Promotion and Maintenance NOT: The correct order of abdominal examination is inspection, auscultation, percussion, and palpation. Palpation is always performed last because it may distort the normal abdominal sounds. 2. ANS: A PTS: 1 DIF: Cognitive Level: Application OBJ: Nursing Process: Assessment MSC: Client Needs: Health Promotion and Maintenance NOT: The correct order of abdominal examination is inspection, auscultation, percussion, and palpation. Palpation is always performed last because it may distort the normal abdominal sounds. 3. ANS: D PTS: 1 DIF: Cognitive Level: Application OBJ: Nursing Process: Assessment MSC: Client Needs: Health Promotion and Maintenance NOT: The correct order of abdominal examination is inspection, auscultation, percussion, and palpation. Palpation is always performed last because it may distort the normal abdominal sounds. 4. ANS: B PTS: 1 DIF: Cognitive Level: Application OBJ: Nursing Process: Assessment MSC: Client Needs: Health Promotion and Maintenance NOT: The correct order of abdominaNlUeRxaSmINiGnaTtBio.nCOisMinspection, auscultation, percussion, and palpation. Palpation is always performed last because it may distort the normal abdominal sounds. Chapter 30: Pain Assessment and Management in Children Perry: Maternal Child Nursing Care, 6th Edition MULTIPLE CHOICE 1. Kyle, age 6 months, is brought to the clinic. His parent says, “I think he hurts. He cries and rolls his head from side to side a lot.” This most likely suggests which feature of pain? a. Type b. Severity c. Duration d. Location ANS: D The child is displaying a local sign of pain. Rolling the head from side to side and pulling at ears indicate pain in the ear. The child's behavior indicates the location of the pain. The behavior does not provide information about the type, severity, or duration. PTS: 1 DIF: Cognitive Level: Comprehension OBJ: Nursing Process: Assessment MSC: Client Needs: Physiologic Integrity 2. Physiologic measurements in children's pain assessment are: a. the best indicator of pain in children of all ages. b. essential to determine whether a child is telling the truth about pain. c. of most value when children also report having pain. d. of limited value as sole indicator of pain. ANS: D NURSINGTB.COM Physiologic manifestations of pain may vary considerably and may not provide a consistent measure of pain. Heart rate may increase or decrease. The same signs that may suggest fear, anxiety, or anger also indicate pain. In chronic pain the body adapts, and these signs decrease or stabilize. These signs are of limited value and must be viewed in the context of a pain-rating scale, behavioral assessment, and parental report. When the child states that pain exists, it does. That is the truth. PTS: 1 DIF: Cognitive Level: Application OBJ: Nursing Process: Assessment MSC: Client Needs: Physiologic Integrity 3. The pediatric nurse understands that nonpharmacologic strategies for pain management: a. may reduce pain perception. b. make pharmacologic strategies unnecessary. c. usually take too long to implement. d. trick children into believing they do not have pain. ANS: A Nonpharmacologic techniques provide coping strategies that may help reduce pain perception, make the pain more tolerable, decrease anxiety, and enhance the effectiveness of analgesics. Nonpharmacologic techniques should be learned before the pain occurs. With severe pain it is best to use both pharmacologic and nonpharmacologic measures for pain control. The nonpharmacologic strategy should be matched with the child's pain severity and taught to the child before the onset of the painful experience. Some of the techniques may facilitate the child's experience with mild pain, but the child will still know that discomfort is present. PTS: 1 DIF: Cognitive Level: Analysis OBJ: Nursing Process: Planning MSC: Client Needs: Physiologic Integrity 4. Which drug is usually the best choice for patient-controlled analgesia (PCA) for a child in the immediate postoperative period? a. Codeine b. Morphine c. Methadone d. Meperidine ANS: B The most commonly prescribed medications for PCA are morphine, hydromorphone, and fentanyl. Parenteral use of codeine is not recommended. Methadone is not available in parenteral form in the United States. Meperidine is not used for continuous and extended pain relief. PTS: 1 DIF: Cognitive Level: Analysis OBJ: Nursing Process: Implementation MSC: Client Needs: Physiologic Integrity 5. A lumbar puncture is needed on aNsUchRoSoINl-GagTeB.cChOilMd. The most appropriate action to provide analgesia during this procedure is to apply: a. 4% Liposomal Lidocaine (LMX) 15 minutes before the procedure. b. a transdermal fentanyl (Duragesic) patch immediately before the procedure. c. eutectic mixture of local anesthetics (EMLA) 1 hour before the procedure. d. EMLA 30 minutes before the procedure. ANS: C EMLA is an effective analgesic agent when applied to the skin 60 minutes before a procedure. It eliminates or reduces the pain from most procedures involving skin puncture. LMX must be applied 30 minutes before the procedure. Transdermal fentanyl patches are useful for continuous pain control, not rapid pain control. For maximal effectiveness, EMLA must be applied approximately 60 minutes in advance. PTS: 1 DIF: Cognitive Level: Analysis OBJ: Nursing Process: Implementation MSC: Client Needs: Physiologic Integrity 6. The nurse is caring for a child receiving intravenous (IV) morphine for severe postoperative pain. The nurse observes a slower respiratory rate, and the child cannot be aroused. The most appropriate management of this child is for the nurse to: a. administer naloxone (Narcan). b. discontinue the IV infusion. c. discontinue morphine until the child is fully awake. d. stimulate the child by calling his or her name, shaking gently, and asking the child to breathe deeply. ANS: A The management of opioid-induced respiratory depression includes lowering the rate of infusion and stimulating the child. If the respiratory rate is depressed and the child cannot be aroused, IV naloxone should be administered. The child will be in pain because of the reversal of the morphine. The morphine should be discontinued, but naloxone is indicated if the child is unresponsive. PTS: 1 DIF: Cognitive Level: Analysis OBJ: Nursing Process: Implementation MSC: Client Needs: Physiologic Integrity 7. When pain is assessed in an infant, it is inappropriate for the nurse to assess for: a. facial expressions of pain. b. localization of pain. c. crying. d. thrashing of extremities. ANS: B Infants are unable to localize pain. Frowning, grimacing, and facial flinching in an infant may indicate pain. Infants often exhibit high-pitched, tense, harsh crying to express pain. Infants may exhibit thrashing of extremities in response to a painful stimulus. PTS: 1 DIF: Cognitive Level: Comprehension OBJ: Nursing Process: Assessment MSC: Client Needs: Physiologic Integrity 8. The nurse caring for the child in pain understands that distraction: a. can give total pain relief to theNcUhRilSdIN. GTB.COM b. is effective when the child is in severe pain. c. is the best method for pain relief. d. must be developmentally appropriate to refocus attention. ANS: D Distraction can be very effective in helping to control pain; however, it must be appropriate to the child's developmental level. Distraction can help control pain, but it is rarely able to provide total pain relief. Children in severe pain are not distractible. Children may use distraction to help control pain, although it is not the best method for pain relief. PTS: 1 DIF: Cognitive Level: Comprehension OBJ: Nursing Process: Planning MSC: Client Needs: Physiologic Integrity 9. Which medication is the most effective choice for treating pain associated with sickle cell crisis in a newly admitted 5-year-old child? a. Morphine b. Acetaminophen c. Ibuprofen d. Midazolam ANS: A Opioids, such as morphine, are the preferred drugs for the management of acute, severe pain, including postoperative pain, posttraumatic pain, pain from vaso-occlusive crisis, and chronic cancer pain. Acetaminophen provides only mild analgesic relief and is not appropriate for a newly admitted child with sickle cell crisis. Ibuprofen is a type of nonsteroidal anti-inflammatory drug (NSAID) that is used primarily for pain associated with inflammation. It is appropriate for mild to moderate pain, but it is not adequate for this patient. Midazolam (Versed) is a short-acting drug used for conscious sedation, for preoperative sedation, and as an induction agent for general anesthesia. PTS: 1 DIF: Cognitive Level: Application OBJ: Nursing Process: Implementation MSC: Client Needs: Physiologic Integrity 10. Which assessment indicates to a nurse that a school-aged child is in need of pain medication? a. The child is lying rigidly in bed and not moving. b. The child's current vital signs are consistent with vital signs over the past 4 hours. c. The child becomes quiet when held and cuddled. d. The child has just returned from the recovery room. ANS: A Behaviors such as crying, distressed facial expressions, certain motor responses such as lying rigidly in bed and not moving, and interrupted sleep patterns are indicative of pain in children. Current vital signs that are consistent with earlier vital signs do not indicate that the child is feeling pain. Response to comforting behaviors does not suggest that the child is feeling pain. A child who is returning from the recovery room may or may not be in pain. Most times the child's pain is under adequate control at this time. The child may be fearful or having anxiety because of the strange surroundings and having just completed surgery. NURSINGTB.COM PTS: 1 DIF: Cognitive Level: Application OBJ: Nursing Process: Assessment MSC: Client Needs: Physiologic Integrity 11. A 2-year-old child has been returned to the nursing unit after an inguinal hernia repair. Which pain assessment tool should the nurse use to assess this child for the presence of pain? a. FACES pain rating tool b. Numeric scale c. Oucher scale d. FLACC tool ANS: D A behavioral pain tool should be used when the child is preverbal or does not have the language skills to express pain. The FLACC (face, legs, activity, cry, consolability) tool should be used with a 2-year-old child. The FACES, numeric, and Oucher scales are all self-report pain rating tools. Self-report measures are not sufficiently valid for children younger than 3 years of age because many children are not able to self-report their pain accurately. PTS: 1 DIF: Cognitive Level: Application OBJ: Nursing Process: Assessment MSC: Client Needs: Physiologic Integrity 12. A nurse is gathering a history on a school-age child admitted for a migraine headache. The child states, “I have been getting a migraine every 2 or 3 months for the last year.” The nurse documents this as which type of pain? a. Acute b. Chronic c. Recurrent d. Subacute ANS: C Pain that is episodic and that recurs is defined as recurrent pain. The time frame within which episodes of pain recur is at least 3 months. Recurrent pain in children includes migraine headache, episodic sickle cell pain, recurrent abdominal pain (RAP), and recurrent limb pain. Acute pain is pain that lasts for less than 3 months. Chronic pain is pain that lasts, on a daily basis, for more than 3 months. Subacute is not a term for documenting type of pain. PTS: 1 DIF: Cognitive Level: Comprehension OBJ: Nursing Process: Communication, Documentation MSC: Client Needs: Physiologic Integrity MULTIPLE RESPONSE 1. An appropriate tool to assess pain in a 3-year-old child is the: (Select all that apply.) a. Visual Analog Scale (VAS) b. Adolescent and pediatric pain tool c. Oucher tool d. FACES pain-rating scale ANS: C, D The Oucher tool can be used to assess pain in children 3 to 12 years of age. The FACES pain-rating scale can be used to assess pain for children 3 years of age and older. The VAS is indicated for use with older schooNl-UagReSIcNhGilTdBre.CnOaMnd adolescents. It can be used with younger school-age children, although less abstract tools are more appropriate. The adolescent and pediatric pain tool is indicated for use with children 8 to 17 years of age. PTS: 1 DIF: Cognitive Level: Comprehension OBJ: Nursing Process: Assessment MSC: Client Needs: Physiologic Integrity 2. The nurse is monitoring a patient for side effects associated with opioid analgesics. Which side effects should the nurse expect to monitor for? (Select all that apply.) a. Diarrhea b. Respiratory depression c. Hypertension d. Pruritus e. Sweating ANS: B, D, E Side effects of opioids include respiratory depression, pruritus, and sweating. Constipation may occur, not diarrhea, and orthostatic hypotension may occur but not hypertension. PTS: 1 DIF: Cognitive Level: Comprehension OBJ: Nursing Process: Implementation MSC: Client Needs: Physiologic Integrity 3. Which dietary recommendations should a nurse make to an adolescent patient to manage constipation related to opioid analgesic administration? (Select all that apply.) a. Bran cereal b. Decrease fluid intake c. Prune juice d. Cheese e. Vegetables ANS: A, C, E To manage the side effect of constipation caused by opioids, fluids should be increased, and bran cereal and vegetables are recommended to increase fiber. Prune juice can act as a nonpharmacologic laxative. Fluids should be increased, not decreased, and cheese can cause constipation so it should not be recommended. PTS: 1 DIF: Cognitive Level: Application OBJ: Nursing Process: Implementation MSC: Client Needs: Physiologic Integrity COMPLETION 1. A dose of oxycodone (OxyContin) 2 mg/kg has been ordered for a child weighing 33 lbs. The nurse should administer mg of OxyContin. (Record your answer as a whole number.) ANS: 30 The child's weight is divided by 2.2 to obtain the weight in kilograms. Kilograms in weight are then multiplied by the prescribed 2 mg. 33 lbs/2.2 = 15 kg. 15 kg  2 mg = 30 mg. NURSINGTB.COM PTS: 1 DIF: Cognitive Level: Application OBJ: Nursing Process: Implementation MSC: Client Needs: Physiologic Integrity 2. The nurse is using the FLACC scale to evaluate pain in a preverbal child. The nurse makes the following assessment: Face: occasional grimace; Leg: relaxed; Activity: squirming, tense; Cry: no cry; Consolability: content, relaxed. The nurse records the FLACC assessment as . (Record your answer as a whole number.) ANS: 2 The FLACC scale is recorded per the following table: 0 1 2 Face No particular expression or smile Occasional grimace or frown, withdrawn, disinterested Frequent to constant frown, clenched jaw, quivering chin Legs Normal position or relaxed Uneasy, restless, tense Kicking or legs drawn up Activity Lying quietly, normal position, moves easily Squirming, shifting back and forth, tense Arched, rigid, or jerking Cry No cry (awake or Moans or whimpers, Crying steadily, asleep) occasional complaint screams or sobs, frequent complaints Consolability Content, relaxed Reassured by occasional touching, hugging, or talking to; distractible Difficult to console or comfort PTS: 1 DIF: Cognitive Level: Application OBJ: Nursing Process: Implementation MSC: Client Needs: Physiologic Integrity MATCHING A patient receiving an intravenous opioid analgesic has become apneic. Match the nursing interventions with the step numbers in order from the highest priority (first intervention) to the lowest priority (last intervention). a. Initiate resuscitation efforts as appropriate b. Administer the prescribed naloxone (Narcan) dose by slow IV push c. Prepare to administer naloxone (Narcan) as needed every 2 minutes until desired effect 1. Step 1 2. Step 2 3. Step 3 1. ANS: A PTS: 1 DIF: Cognitive Level: Application OBJ: Nursing Process: Implementation MSC: Client Needs: Physiologic Integrity NOT: Initiate resuscitation efforts as appropriate. Administer naloxone (Narcan). Administer bolus by slow intravenous (IV) push every 2 mNiUnuRtSeIsNuGnTtiBl .eCffOeMct is obtained. 2. ANS: B PTS: 1 DIF: Cognitive Level: Application OBJ: Nursing Process: Implementation MSC: Client Needs: Physiologic Integrity NOT: Initiate resuscitation efforts as appropriate. Administer naloxone (Narcan). Administer bolus by slow intravenous (IV) push every 2 minutes until effect is obtained. 3. ANS: C PTS: 1 DIF: Cognitive Level: Application OBJ: Nursing Process: Implementation MSC: Client Needs: Physiologic Integrity NOT: Initiate resuscitation efforts as appropriate. Administer naloxone (Narcan). Administer bolus by slow intravenous (IV) push every 2 minutes until effect is obtained. Chapter 31: The Infant and Family Perry: Maternal Child Nursing Care, 6th Edition MULTIPLE CHOICE 1. Which statement accurately describes an event associated with an infant's physical development? a. Anterior fontanel closes by age 6 to 10 months. b. Binocularity is well established by age 8 months. c. Birth weight doubles by age 5 months and triples by age 1 year. d. Maternal iron stores persist during the first 12 months of life. ANS: C Growth is very rapid during the first year of life. The birth weight approximately doubles by age 5 to 6 months and triples by age 1 year. The anterior fontanel closes at age 12 to 18 months. Binocularity is not established until age 15 months. Maternal iron stores are usually depleted by age 6 months. PTS: 1 DIF: Cognitive Level: Comprehension OBJ: Nursing Process: Assessment MSC: Client Needs: Health Promotion and Maintenance 2. The nurse assessing a 6-month-old healthy infant who weighed 7 lbs at birth, shares with the parents that the infant should weigh approximately how many pounds? a. 10 lbs. b. 15 lbs. c. 20 lbs. d. 25 lbs. ANS: B NURSINGTB.COM Birth weight doubles at about age 5 to 6 months. At 6 months, an infant who weighed 7 lbs at birth would weigh approximately 15 lbs. Ten pounds is too little; the infant would have gone from the 50th percentile at birth to below the 5th percentile. Twenty pounds or more is too much; the infant would have tripled the birth weight at 6 months. PTS: 1 DIF: Cognitive Level: Analysis OBJ: Nursing Process: Assessment MSC: Client Needs: Health Promotion and Maintenance 3. The nurse is doing a routine assessment on a 14-month-old infant and notes that the anterior fontanel is closed. This should be interpreted as: a. a normal finding. b. a questionable finding—the infant should be rechecked in 1 month. c. an abnormal finding—indicates the need for immediate referral to a practitioner. d. an abnormal finding—indicates the need for developmental assessment. ANS: A Because the anterior fontanel normally closes between ages 12 and 18 months, this is a normal finding, and no further intervention is required. PTS: 1 DIF: Cognitive Level: Analysis OBJ: Nursing Process: Assessment MSC: Client Needs: Health Promotion and Maintenance 4. By what age does the posterior fontanel usually close? a. 6 to 8 weeks b. 10 to 12 weeks c. 4 to 6 months d. 8 to 10 months ANS: A The bones surrounding the posterior fontanel fuse and close by age 6 to 8 weeks. Ten weeks or longer is too late and indicates a problem. PTS: 1 DIF: Cognitive Level: Knowledge OBJ: Nursing Process: Knowledge MSC: Client Needs: Health Promotion and Maintenance 5. The parents of a 9-month-old infant tell the nurse that they have noticed foods such as peas and corn are not completely digested and can be seen in their infant's stool. The nurse bases her explanation on what fact? a. Children should not be given fibrous foods until the digestive tract matures at age 4 years. b. The infant should not be given any solid foods until this digestive problem is resolved. c. This is abnormal and requires further investigation. d. This is normal because of the immaturity of digestive processes at this age. ANS: D The immaturity of the digestive tract is evident in the appearance of the stools. Solid foods are passed incompletely broken down in the feces but it is not necessity to eliminate solid foods. An excess quantity of fiber predisposes the child to large, bulky stools. This is a normal part of the maturational process, and nNoUfuRrStIhNeGr TinBv.CeOstMigation is necessary. PTS: 1 DIF: Cognitive Level: Comprehension OBJ: Nursing Process: Implementation MSC: Client Needs: Health Promotion and Maintenance 6. A 3-month-old infant, born at 38 weeks of gestation, will hold a rattle if it is put in her hands; however, she will not voluntarily grasp it. How should the nurse interpret this behavior? a. Normal development b. Significant developmental lag c. Slightly delayed development caused by prematurity d. Suggestive of a neurologic disorder such as cerebral palsy ANS: A This indicates normal development. Reflexive grasping occurs during the first 2 to 3 months and then gradually becomes voluntary. No evidence of developmental lag, delayed development, or neurologic dysfunction is present by this behavior. PTS: 1 DIF: Cognitive Level: Comprehension OBJ: Nursing Process: Assessment MSC: Client Needs: Health Promotion and Maintenance 7. The nurse determines an infant of 7 months is demonstrating appropriate fine motor development when performing which action? a. Transferring a rattler from one hand to the other. b. Using thumb and index finger to grasp a piece of food. c. Holding a crayon and make a mark on paper. d. Releasing cubes into a cup. ANS: A By age 7 months, infants can transfer objects from one hand to the other, crossing the midline. The crude pincer grasp is apparent at about age 9 months. The infant can scribble spontaneously at age 15 months. At age 12 months, the infant can release cubes into a cup. PTS: 1 DIF: Cognitive Level: Analysis OBJ: Nursing Process: Assessment MSC: Client Needs: Health Promotion and Maintenance 8. In terms of gross motor development, what hallmark action should the nurse identify for the parents of a 5-month-old infant to anticipate? a. Roll from abdomen to back. b. Roll from back to abdomen. c. Sit erect without support. d. Move from prone to sitting position. ANS: A Rolling from abdomen to back is developmentally appropriate for a 5-month-old infant. The ability to roll from back to abdomen usually occurs at 6 months old. Sitting erect without support is a developmental milestone usually achieved by 8 months. The 10-month-old infant can usually move from a prone to a sitting position. PTS: 1 DIF: Cognitive Level: Application OBJ: Nursing Process: Assessment MSC: Client Needs: Health Promotion and Maintenance 9. At which age can most infants sit steadily unsupported? a. 4 months b. 6 months c. 8 months d. 10 months ANS: C NURSINGTB.COM Sitting erect without support is a developmental milestone usually achieved by 8 months. At age 4 months, an infant can sit with support. At age 6 months, the infant will maintain a sitting position if propped. By 10 months, the infant can maneuver from a prone to a sitting position. PTS: 1 DIF: Cognitive Level: Comprehension OBJ: Nursing Process: Assessment MSC: Client Needs: Health Promotion and Maintenance 10. By what age should the nurse expect that an infant will be able to pull to a standing position? a. 6 months b. 8 months c. 9 months d. 11 to 12 months ANS: C Most infants can pull themselves to a standing position at age 9 months. Any infant who cannot pull to a standing position by age 11 to 12 months should be referred for further evaluation for developmental dysplasia of the hips (DDH). At 6 months, the infant has just obtained coordination of arms and legs. By age 8 months, infants can bear full weight on their legs. PTS: 1 DIF: Cognitive Level: Comprehension OBJ: Nursing Process: Assessment MSC: Client Needs: Health Promotion and Maintenance 11. According to Piaget, the 6-month-old infant would be in what stage of the sensorimotor phase? a. Use of reflexes b. Primary circular reactions c. Secondary circular reactions d. Coordination of secondary schemata ANS: C Infants are usually in the secondary circular reaction stage from age 4 months to 8 months. This stage is characterized by a continuation of the primary circular reaction for the response that results. For example, shaking of a rattle is performed to hear the noise of the rattle, not just for shaking. The use of reflexes is primarily during the first month of life. The primary circular reaction stage marks the replacement of reflexes with voluntary acts. The infant is in this stage from age 1 month to 4 months. The fourth sensorimotor stage is coordination of secondary schemata. This is a transitional stage in which increasing motor skills enable greater exploration of the environment. PTS: 1 DIF: Cognitive Level: Comprehension OBJ: Nursing Process: Assessment MSC: Client Needs: Health Promotion and Maintenance 12. Which behavior indicates that an infant has developed object permanence? a. Recognizes familiar face such as the mother b. Recognizes familiar object such as a bottle c. Actively searches for a hiddenNoUbRjeScINt GTB.COM d. Secures objects by pulling on a string ANS: C During the first 6 months of life, infants believe that objects exist only as long as they can see them. When infants search for an object that is out of sight, this signals the attainment of object permanence, whereby an infant knows that an object exists even when it is not visible. Between ages 8 and 12 weeks, infants begin to respond differentially to their mothers. They cry, smile, vocalize, and show distinct preference for their mothers. This preference is one of the stages that influence the attachment process, but it is too early for object permanence. Recognizing familiar objects is an important transition for the infant, but it does not signal object permanence. The ability to understand cause and effect, such as pulling on a string to secure an object, is part of secondary schema development. PTS: 1 DIF: Cognitive Level: Analysis OBJ: Nursing Process: Assessment MSC: Client Needs: Health Promotion and Maintenance 13. A parent asks the nurse “At what age do most babies begin to fear strangers?” The nurse responds that most infants begin to fear strangers at what age? a. 2 months b. 4 months c. 6 months d. 12 months ANS: C Between ages 6 and 8 months, fear of strangers and stranger anxiety become prominent and are related to the infant's ability to discriminate between familiar and nonfamiliar people. At age 2 months, the infant is just beginning to respond differentially to the mother. At age 4 months, the infant is beginning the process of separation individuation when the infant begins to recognize self and mother as separate beings. Twelve months is too late and requires referral for evaluation if the infant does not fear strangers at this age. PTS: 1 DIF: Cognitive Level: Comprehension OBJ: Nursing Process: Diagnosis MSC: Client Needs: Health Promotion and Maintenance 14. The nurse is interviewing the father of 10-month-old. When the child, playing on the floor and notices an electrical outlet and reaches up to touch it, the father says “No” firmly and removes the child away from the outlet. The nurse should use this opportunity to teach the father that the child is capable of understanding what association? a. Understand the word “No.” b. Father always means “No.” c. Electrical outlets are dangerous. d. Spanking as a deterrent. ANS: A By age 10 months, children are able to associate meaning with words. The child should be old enough to understand the word “No.” The 10-month-old is too young to understand the purpose of an electrical outlet and is not likely to always associate her father with the word “No.” The father is using both verbal and physical cues to teach safety measures and alert the child to dangerous situations. Physical discipline should be avoided. PTS: 1 DIF: Cognitive Level: Analysis OBJ: Nursing Process: ImplementaNtioUnRSINMGSTCB.:CCOlMient Needs: Health Promotion and Maintenance 15. Sara, age 4 months, was born at 35 weeks' gestation. She seems to be developing normally, but her parents are concerned because she is a “more difficult” baby than their other child, who was term. The nurse should explain that: a. infants' temperaments are part of their unique characteristics. b. infants become less difficult if they are not kept on scheduled feedings and structured routines. c. Sara's behavior is suggestive of failure to bond completely with her parents. d. Sara's difficult temperament is the result of painful experiences in the neonatal period. ANS: A Infant temperament has a strong biologic component. Together with interactions with the environment, primarily the family, the biologic component contributes to the infant's unique temperament. Children perceived as difficult may respond better to scheduled feedings and structured caregiving routines than to demand feedings and frequent changes in routines. Sara's temperament has been created by both biologic and environmental factors. The nurse should provide guidance in parenting techniques that are best suited to Sara's temperament. PTS: 1 DIF: Cognitive Level: Comprehension OBJ: Nursing Process: Implementation MSC: Client Needs: Health Promotion and Maintenance 16. Which information could be given to the parents of a 12-month-old child regarding appropriate play activities for this age? a. Give large push-pull toys for kinesthetic stimulation. b. Place cradle gym across crib to facilitate fine motor skills. c. Provide child with finger paints to enhance fine motor skills. d. Provide stick horse to develop gross motor coordination. ANS: A The 12-month-old child is able to pull to a stand and walk holding on or independently. Appropriate toys for a child of this age include large push-pull toys for kinesthetic stimulation. A cradle gym should not be placed across the crib. Finger paints are appropriate for older children. A 12-month-old child does not have the stability to use a stick horse. PTS: 1 DIF: Cognitive Level: Analysis OBJ: Nursing Process: Planning MSC: Client Needs: Health Promotion and Maintenance 17. Which is an appropriate play activity for a 7-month-old infant to encourage visual stimulation? a. Playing peek-a-boo. b. Playing pat-a-cake. c. Imitating animal sounds. d. Showing how to clap hands. ANS: A Because object permanence is a new achievement, peek-a-boo is an excellent activity to practice this new skill for visual stimulation. Playing pat-a-cake and showing how to clap hands will help with kinesthetic stimulation. Imitating animal sounds will help with auditory stimulation. NURSINGTB.COM PTS: 1 DIF: Cognitive Level: Analysis OBJ: Nursing Process: Planning MSC: Client Needs: Health Promotion and Maintenance 18. At what age should the nurse expect an infant to begin smiling in response to pleasurable stimuli? a. 1 month b. 2 months c. 3 months d. 4 months ANS: B At age 2 months, the infant has a social, responsive smile. A reflex smile is usually present at age 1 month. The 3-month-old can recognize familiar faces. At age 4 months, the infant can enjoy social interactions. PTS: 1 DIF: Cognitive Level: Comprehension OBJ: Nursing Process: Assessment MSC: Client Needs: Health Promotion and Maintenance 19. The mother of a breastfed infant being seen in the clinic for the sixth month checkup is concerned that the infant has begun thumb sucking. How should the nurse respond to the mother's concern? a. Recommend that the mother substitute a pacifier for the infant's thumb. b. Assess the infant for other signs of sensory deprivation. c. Reassure the mother that this behavior is very normal at this age. d. Suggest that the mother breastfeed more often to satisfy sucking needs. ANS: C Sucking is an infant's chief pleasure, and she may not be satisfied by bottle-feeding or breastfeeding alone. During infancy and early childhood, there is no need to restrict nonnutritive sucking. Dental damage does not appear to occur unless the use of the pacifier or finger persists after age 4 to 6 years. The nurse should explore with the mother her feelings about pacifier versus thumb. This is a normal behavior to meet nonnutritive sucking needs. No data support that Latasha has sensory deprivation. PTS: 1 DIF: Cognitive Level: Application OBJ: Nursing Process: Implementation MSC: Client Needs: Health Promotion and Maintenance 20. How should the nurse describe the fact that a 6 month old has 6 teeth? a. Normal tooth eruption. b. Delayed tooth eruption. c. Unusual and dangerous. d. Earlier-than-normal tooth eruption. ANS: D This is earlier than expected. Most infants at age 6 months have two teeth, the lower central incisors. Six teeth at 6 months is not delayed; it is early tooth eruption. Although unusual, it is not dangerous. PTS: 1 DIF: Cognitive Level: Comprehension OBJ: Nursing Process: Assessment MSC: Client Needs: Health Promotion and Maintenance NURSINGTB.COM 21. A mother tells the nurse that she is discontinuing breastfeeding her 5-month-old infant. What should the nurse recommend to be used as substitute for the breastmilk? a. Skim milk b. Whole cow's milk c. Commercial iron-fortified formula d. Commercial formula without iron ANS: C For children younger than 1 year, the American Academy of Pediatrics recommends the use of breast milk. If breastfeeding has been discontinued, iron-fortified commercial formula should be used. Cow's milk should not be used in children younger than 12 months. Maternal iron stores are almost depleted by this age; the iron-fortified formula will help prevent the development of iron deficiency anemia. PTS: 1 DIF: Cognitive Level: Comprehension OBJ: Nursing Process: Implementation MSC: Client Needs: Health Promotion and Maintenance 22. When is the best age for solid food to be introduced into the infant's diet? a. 2 to 3 months b. 4 to 6 months c. When birth weight has tripled d. When tooth eruption has started ANS: B Physiologically and developmentally, the 4 to 6 months old is in a transition period. The extrusion reflex has disappeared, and swallowing is a more coordinated process. In addition, the gastrointestinal tract has matured sufficiently to handle more complex nutrients and is less sensitive to potentially allergenic food. Infants of this age will try to help during feeding. Two to three months is too young. The extrusion reflex is strong, and the infant will push food out with the tongue. No research base indicates that the addition of solid food to bottle-feeding has any benefit. Tooth eruption can facilitate biting and chewing; most infant foods do not require this ability. PTS: 1 DIF: Cognitive Level: Comprehension OBJ: Nursing Process: Planning MSC: Client Needs: Health Promotion and Maintenance 23. The parents of a 4-month-old infant tell the nurse that they are getting a microwave oven and will be able to heat the baby's formula faster. What recommendations should the nurse provide the parents? a. Never heat a bottle in a microwave oven. b. Heat only 10 ounces or more. c. Always leave the bottle top uncovered to allow heat to escape. d. Shake the bottle vigorously for at least 30 seconds after heating. ANS: A Neither infant formula nor breast milk should be warmed in a microwave oven as this may cause oral burns as a result of uneven heating in the container. The bottle may remain cool while hot spots develop in the milk. Warming expressed milk in a microwave decreases the availability of antiinfective properties and causes separation of the fat content. Milk should be warmed in a lukewarm water bath. None of the other options adequately deals with the issue of overheating. NURSINGTB.COM PTS: 1 DIF: Cognitive Level: Analysis OBJ: Nursing Process: Implementation MSC: Client Needs: Health Promotion and Maintenance 24. The parent of a 2 week old asks the nurse if the infant needs fluoride supplements because they plan to exclusively breastfed. What is the nurse's best response? a. “Your infant needs to begin taking them now.” b. “They are not needed if you drink fluoridated water.” c. “Your infant may need to begin taking them at age 6 months.” d. “Your infant can have infant cereal mixed with fluoridated water instead of supplements.” ANS: C Fluoride supplementation is recommended by the American Academy of Pediatrics beginning at age 6 months if the child is not drinking adequate amounts of fluoridated water. The recommendation is to begin supplementation at 6 months, not at 2 weeks. The amount of water that is ingested and the amount of fluoride in the water are evaluated when supplementation is being considered. PTS: 1 DIF: Cognitive Level: Comprehension OBJ: Nursing Process: Planning MSC: Client Needs: Health Promotion and Maintenance 25. A mother tells the nurse that she does not want her infant immunized because of the discomfort associated with injections. The nurse should explain that: a. this cannot be prevented. b. infants do not feel pain as adults do. c. this is not a good reason for refusing immunizations. d. a topical anesthetic, eutectic mixture of local anesthetic (EMLA), will minimize the discomfort. ANS: D Several topical anesthetic agents can be used to minimize the discomfort associated with immunization injections. These include EMLA and vapor coolant sprays. Pain associated with many procedures can be prevented or minimized by using the principles of atraumatic care. With preparation, the injection site can be properly anesthetized to decrease the amount of pain felt by the infant. Infants have the neural pathways to sense pain. Numerous research studies have indicated that infants perceive and react to pain in the same manner as do children and adults. The mother should be allowed to discuss her concerns and the alternatives available. This is part of the informed consent process. PTS: 1 DIF: Cognitive Level: Analysis OBJ: Nursing Process: Implementation MSC: Client Needs: Health Promotion and Maintenance 26. The parents of a 12-month-old child ask the nurse if the child can eat hot dogs. The nurse's reply should be based on what understanding? a. The child is too young to digest hot dogs. b. The child is too young to eat hot dogs safely. c. Hot dogs must be sliced into sections to prevent aspiration. d. Hot dogs must be cut into small, irregular pieces to prevent aspiration. ANS: D Hot dogs are of a consistency, diaNmUeRteSrI,NaGnTdBr.oCuOnMd shape that may cause complete obstruction of the child's airway. If given to young children, the hot dog should be cut into small irregular pieces rather than served whole or in slices. The child's digestive system is mature enough to digest hot dogs. To eat the hot dog safely, the child should be sitting down, and the hot dog should be appropriately cut into irregularly shaped pieces. PTS: 1 DIF: Cognitive Level: Comprehension OBJ: Nursing Process: Implementation MSC: Client Needs: Health Promotion and Maintenance 27. The clinic is lending a federally approved car seat to an infant's family. Where in the car should the nurse explain is the safest place to put the car seat? a. Front facing in back seat. b. Rear facing in back seat. c. Front facing in front seat if an air bag is on the passenger side. d. Rear facing in front seat if an air bag is on the passenger side. ANS: B The rear-facing car seat provides the best protection for an infant's disproportionately heavy head and weak neck. Infants should face the rear from birth to 20 lbs and as close to 1 year of age as possible. The middle of the back seat provides the safest position. Severe injuries and deaths in children have occurred from air bags deploying on impact in the front passenger seat. PTS: 1 DIF: Cognitive Level: Comprehension OBJ: Nursing Process: Planning MSC: Client Needs: Health Promotion and Maintenance 28. A nurse is teaching parents about prevention and treatment of colic. Which should the nurse include in the teaching plan? a. Avoid use of pacifiers. b. Eliminate all secondhand smoke contact. c. Lay infant flat after feeding. d. Avoid swaddling the infant. ANS: B To prevent and treat colic, teach parents that if household members smoke, they should avoid smoking near the infant; smoking activity should preferably be confined to outside of the home. A pacifier can be introduced for added sucking. The infant should be swaddled tightly with a soft, stretchy blanket, and placed in an upright seat after feedings. PTS: 1 DIF: Cognitive Level: Application OBJ: Nursing Process: Implementation MSC: Client Needs: Physiologic Integrity 29. Parent guidelines for relieving colic in an infant include: a. avoiding touching the abdomen. b. avoiding using a pacifier. c. changing the infant's position frequently. d. placing the infant where the family cannot hear the crying. ANS: C Changing the infant's position frequently may be beneficial. The parent can walk holding the infant face down and with the infant's chest across the parent's arm. The parent's hand can support the infant's abdomen, appNlyUiRngSIgNeGnTtlBe.CpOreMssure. Gently massaging the abdomen is effective in some infants. Pacifiers can be used for meeting additional sucking needs. The infant should not be placed where monitoring cannot be done. The infant can be placed in the crib and allowed to cry. Periodically, the infant should be picked up and comforted. PTS: 1 DIF: Cognitive Level: Application OBJ: Nursing Process: Implementation MSC: Client Needs: Physiologic Integrity 30. Which clinical manifestations should cause the nurse to suspect that a child, diagnosed with a digestive disorder, may be demonstrating signs of failure to thrive? a. Avoidance of eye contact. b. An associated malabsorption defect. c. Weight that falls below the 15th percentile. d. Normal achievement of developmental landmarks. ANS: A One of the clinical manifestations of nonorganic failure to thrive is the child's avoidance of eye contact with the health professional. A malabsorption defect would result in a physiologic problem, not behavioral. Weight (but not height) below the 5th percentile is indicative of failure to thrive. Developmental delays, including social, motor, adaptive, and language, exist. PTS: 1 DIF: Cognitive Level: Analysis OBJ: Nursing Process: Assessment MSC: Client Needs: Physiologic Integrity 31. Which consideration should be considered when planning care for an infant diagnosed with failure to thrive? a. Establishing a structured routine and follow it consistently. b. Maintaining a nondistracting environment by not speaking to the infant during feeding. c. Placing the infant in an infant seat during feedings to prevent overstimulation. d. Limiting sensory stimulation and play activities to alleviate fatigue. ANS: A The infant with failure to thrive should have a structured routine that is followed consistently. Disruptions in other activities of daily living can have a great impact on feeding behaviors. Bathing, sleeping, dressing, playing, and feeding are structured. The nurse should talk to the infant by giving directions about eating. This will help the infant maintain focus. Young children should be held while being fed, and older children can sit at a feeding table. The infant should be fed in the same manner at each meal. The infant can engage in sensory and play activities at times other than mealtime. PTS: 1 DIF: Cognitive Level: Application OBJ: Nursing Process: Implementation MSC: Client Needs: Physiologic Integrity 32. What is an important nursing responsibility when dealing with a family experiencing the loss of an infant from sudden infant death syndrome (SIDS)? a. Explain how SIDS could have been predicted and prevented. b. Interview parents in depth concerning the circumstances surrounding the infant's death. c. Discourage parents from making a last visit with the infant. d. Make a follow-up home visit tNoUpRaSreINnGtsTaBs.CsOooMn as possible after the infant's death. ANS: D A competent, qualified professional should visit the family at home as soon as possible after the death and provide the family with printed information about SIDS. An explanation of how SIDS could have been predicted and prevented is inappropriate. SIDS cannot be prevented or predicted. Discussions about the cause will only increase parental guilt. The parents should be asked only factual questions to determine the cause of death. Parents should be allowed and encouraged to make a last visit with their infant. PTS: 1 DIF: Cognitive Level: Application OBJ: Nursing Process: Implementation MSC: Client Needs: Psychosocial Integrity 33. With the goal of preventing plagiocephaly, the nurse should teach new parents to consider which intervention? a. Place the infant prone for 30 to 60 minutes per day. b. Buy a soft mattress. c. Allow the infant to nap in the car safety seat. d. Have the infant sleep with the parents. ANS: A Prevention of positional plagiocephaly may begin shortly after birth by implementing prone positioning or “tummy time” for approximately 30 to 60 minutes per day when the infant is awake. Soft mattresses or sleeping with parents (co-sleeping) are not recommended because they put the infant at a higher risk for a sudden infant death incident. To prevent plagiocephaly, prolonged placement in car safety seats should be avoided. PTS: 1 DIF: Cognitive Level: Application OBJ: Nursing Process: Implementation MSC: Client Needs: Physiologic Integrity 34. The parents of a 3-month-old infant report that their infant sleeps supine (face up) but is often prone (face down) while awake. The nurse's response should be based on what knowledge? a. Unacceptable because of the risk of sudden infant death syndrome (SIDS). b. Unacceptable because it does not encourage achievement of developmental milestones. c. Unacceptable to encourage fine motor development. d. Acceptable to encourage head control and turning over. ANS: D These parents are implementing the guidelines to reduce the risk of SIDS. Infants should sleep on their backs and then be placed on their abdomens when awake to enhance development of milestones such as head control. The face-down position while awake and positioning on the back for sleep are acceptable because they reduce risk of SIDS and allow achievement of developmental milestones. These position changes encourage gross motor, not fine motor development. PTS: 1 DIF: Cognitive Level: Analysis OBJ: Nursing Process: Implementation MSC: Client Needs: Health Promotion and Maintenance NURSINGTB.COM 35. The nurse should teach parents that at what age it is safe to give infants whole milk instead of commercial infant formula? a. 6 months b. 9 months c. 12 months d. 18 months ANS: C The American Academy of Pediatrics does not recommend the use of cow's milk for children younger than 12 months. At 6 and 9 months, the infant should be receiving commercial infant formula or breast milk. At age 18 months, milk and formula are supplemented with solid foods, water, and some fruit juices. PTS: 1 DIF: Cognitive Level: Comprehension OBJ: Nursing Process: Implementation MSC: Client Needs: Health Promotion and Maintenance 36. A parent asks the nurse whether her infant is susceptible to pertussis. The nurse's response should be based on which statement concerning susceptibility to pertussis? a. Neonates will be immune the first few months. b. If the mother has had the disease, the infant will receive passive immunity. c. Children younger than 1 year seldom contract this disease. d. Most children are highly susceptible from birth. ANS: D The acellular pertussis vaccine is recommended by the American Academy of Pediatrics beginning at age 6 weeks. Infants are at greater risk for complications of pertussis. The vaccine is not given after age 7 years, when the risks of the vaccine become greater than those of pertussis. The infant is highly susceptible to pertussis, which can be a life-threatening illness in this age-group. PTS: 1 DIF: Cognitive Level: Application OBJ: Nursing Process: Implementation MSC: Client Needs: Health Promotion and Maintenance MULTIPLE RESPONSE 1. In teaching parents about appropriate pacifier selection, the nurse should explain that a pacifier should have which characteristics? (Select all that apply.) a. Easily grasped handle b. One-piece construction c. Ribbon or string to secure to clothing d. Soft, pliable material e. Sturdy, flexible material ANS: A, B, E A good pacifier should be easily grasped by the infant. One-piece construction is necessary to avoid having the nipple and guard separate. The material should be sturdy and flexible. An attached ribbon or string and soft, pliable material are not characteristics of a good pacifier. PTS: 1 DIF: Cognitive Level: Comprehension OBJ: Nursing Process: Planning MSC: Client Needs: Safe and Effective Care Environment NURSINGTB.COM 2. In terms of gross motor development, what would the nurse educate the parents to expect a 5- month-old infant to do? (Select all that apply.) a. Roll from abdomen to back. b. Put feet in mouth when supine. c. Roll from back to abdomen. d. Sit erect without support. e. Move from prone to sitting position. ANS: A, B Rolling from abdomen to back and placing the feet in the mouth when supine are developmentally appropriate for a 5-month-old infant. Rolling from back to abdomen is developmentally appropriate for a 6-month-old infant. An 8-month-old infant should be able to sit erect without support. A 10-month-old infant can usually move from a prone to a sitting position. PTS: 1 DIF: Cognitive Level: Application OBJ: Nursing Process: Assessment MSC: Client Needs: Health Promotion and Maintenance 3. A nurse is conducting education classes for parents of infants. The nurse plans to discuss sudden infant death syndrome (SIDS). Which risk factors should the nurse include as increasing an infant's risk of a SIDS incident? (Select all that apply.) a. Breastfeeding b. Low Apgar scores c. Male sex d. Birth weight in the 50th or higher percentile e. Recent viral illness ANS: B, C, E Certain groups of infants are at increased risk for SIDS: those with low birth weight, low Apgar scores, or recent viral illness, and those of male sex. Breastfed infants and infants of average or above-average weight are not at higher risk for SIDS. PTS: 1 DIF: Cognitive Level: Comprehension OBJ: Nursing Process: Assessment MSC: Client Needs: Health Promotion and Maintenance 4. Which interventions should the nurse implement when caring for a family of a sudden infant death syndrome (SIDS) infant? (Select all that apply.) a. Allow parents to say goodbye to their infant. b. Once parents leave the hospital, no further follow-up is required. c. Arrange for someone to take the parents home from the hospital. d. Avoid requesting an autopsy of the deceased infant. e. Conduct a debriefing session with the parents before they leave the hospital. ANS: A, C, E An important aspect of compassionate care for parents experiencing a SIDS incident is allowing them to say good-bye to their infant. These are the parents' last moments with their infant, and they should be as quiet, meaningful, peaceful, and undisturbed as possible. Because the parents leave the hospital without their infant, it is helpful to accompany them to the car or arrange for someone else to take them home. A debriefing session may help health care workers who dealt with the family and deceased infant to cope with emotions that are often engendered when a SIDS victim is brought into the acute care facility. An autopsy may clear up possible misconceptions rNeUgRarSdINinGgTtBh.eCOdeMath. When the parents return home, a competent, qualified professional should visit them after the death as soon as possible. PTS: 1 DIF: Cognitive Level: Comprehension OBJ: Nursing Process: Implementation MSC: Client Needs: Psychosocial Integrity MATCHING Place in order the expected sequence of fine motor developmental milestones for an infant, beginning with the first milestone achieved and ending with the last milestone achieved. a. Voluntary palmar grasp b. Reflex palmar grasp c. Puts objects into a container d. Neat pincer grasp e. Builds a tower of two blocks 1. First 2. Second 3. Third 4. Fourth 5. Fifth 1. ANS: B PTS: 1 DIF: Cognitive Level: Analysis OBJ: Nursing Process: Evaluation MSC: Client Needs: Health Promotion and Maintenance NOT: Grasping occurs during the first 2 to 3 months as a reflex and gradually becomes voluntary. By 5 months, infants are able to voluntarily grasp objects. Gradually, the palmar grasp (using the whole hand) is replaced by a pincer grasp (using the thumb and index finger). By 8 to 10 months of age, infants use a crude pincer grasp, and by 11 months, they have progressed to a neat pincer grasp. By 11 months, they put objects into containers and like to remove them. By age 1 year, infants try to build towers of two blocks. 2. ANS: A PTS: 1 DIF: Cognitive Level: Analysis OBJ: Nursing Process: Evaluation MSC: Client Needs: Health Promotion and Maintenance NOT: Grasping occurs during the first 2 to 3 months as a reflex and gradually becomes voluntary. By 5 months, infants are able to voluntarily grasp objects. Gradually, the palmar grasp (using the whole hand) is replaced by a pincer grasp (using the thumb and index finger). By 8 to 10 months of age, infants use a crude pincer grasp, and by 11 months, they have progressed to a neat pincer grasp. By 11 months, they put objects into containers and like to remove them. By age 1 year, infants try to build towers of two blocks. 3. ANS: D PTS: 1 DIF: Cognitive Level: Analysis OBJ: Nursing Process: Evaluation MSC: Client Needs: Health Promotion and Maintenance NOT: Grasping occurs during the first 2 to 3 months as a reflex and gradually becomes voluntary. By 5 months, infants are able to voluntarily grasp objects. Gradually, the palmar grasp (using the whole hand) is replaced by a pincer grasp (using the thumb and index finger). By 8 to 10 months of age, infants use a crude pincer grasp, and by 11 months, they have progressed to a neat pincer grasp. By 11 months, they put objects into containers and like to remove them. By age 1 year, infants try to build towers of two blocks. 4. ANS: C PTS: 1 DIF: Cognitive Level: Analysis OBJ: Nursing Process: Evaluation MSC: Client Needs: Health Promotion and Maintenance NOT: Grasping occurs during the first 2 to 3 months as a reflex and gradually becomes voluntary. By 5 months, infants are able to voluntarily grasp objects. Gradually, the palmar grasp (using the whole hand) is replaced by a pincer grasp (using the thumb and index finger). By 8 to 10 months of age, infants use a crude pincer grasp, and by 11 months, they have progressed to a neat pincer grasp. By 11 months, they put objects into containNerUsRaSnIdNlGikTeBt.oCrOeMmove them. By age 1 year, infants try to build towers of two blocks. 5. ANS: E PTS: 1 DIF: Cognitive Level: Analysis OBJ: Nursing Process: Evaluation MSC: Client Needs: Health Promotion and Maintenance NOT: Grasping occurs during the first 2 to 3 months as a reflex and gradually becomes voluntary. By 5 months, infants are able to voluntarily grasp objects. Gradually, the palmar grasp (using the whole hand) is replaced by a pincer grasp (using the thumb and index finger). By 8 to 10 months of age, infants use a crude pincer grasp, and by 11 months, they have progressed to a neat pincer grasp. By 11 months, they put objects into containers and like to remove them. By age 1 year, infants try to build towers of two blocks. Chapter 32: The Toddler and Family Perry: Maternal Child Nursing Care, 6th Edition MULTIPLE CHOICE 1. Which factor is most important in predisposing toddlers to frequent infections such as otitis media, tonsillitis, and upper respiratory tract infections? a. Respirations are abdominal. b. Pulse and respiratory rates are slower than those in infancy. c. Defense mechanisms are less efficient than those during infancy. d. The presence of short, straight internal ear/throat structures and large tonsil/adenoid lymph tissue. ANS: D Toddlers continue to have the short, straight internal ear canal of infants. The lymphoid tissue of the tonsils and adenoids continues to be relatively large. These two anatomic conditions combine to predispose the toddler to frequent infections. The abdominal respirations and lowered pulse and respiratory rate of toddlers do not affect their susceptibility to infection. The defense mechanisms are more efficient compared with those of infancy. PTS: 1 DIF: Cognitive Level: Comprehension OBJ: Nursing Process: Assessment MSC: Client Needs: Physiologic Integrity 2. What are the psychosocial developmental tasks of toddlerhood? a. Development of a conscience. b. Recognition of sex differencesN.URSINGTB.COM c. Ability to get along with age mates. d. Ability to withstand delayed gratification. ANS: D If the need for basic trust has been satisfied, toddlers can give up dependence for control, independence, and autonomy. One of the tasks that the toddler is concerned with is the ability to withstand delayed gratification. Development of a conscience occurs during the preschool years. The recognition of sex differences occurs during the preschool years. The ability to get along with age mates develops during the preschool and school-age years. PTS: 1 DIF: Cognitive Level: Comprehension OBJ: Nursing Process: Assessment MSC: Client Needs: Psychosocial Integrity 3. A parent of an 18 month old tells the nurse that the child says “no” to everything and has rapid mood swings. If scolded, the child shows anger and then immediately wants to be held. What is the nurse's best interpretation of this behavior? a. This is normal behavior for the child's age. b. This is unusual behavior for the child's age. c. The child is not effectively coping with stress. d. The child is showing the need for more attention. ANS: A Toddlers use distinct behaviors in the quest for autonomy. They express their will with continued negativity and the use of the word “no.” Children at this age also have rapid mood swings. The nurse should reassure the parents that their child is engaged in expected behavior for an 18 month old. PTS: 1 DIF: Cognitive Level: Analysis OBJ: Nursing Process: Assessment MSC: Client Needs: Health Promotion and Maintenance 4. The nurse is planning care for a 17-month-old child. According to Piaget, in what stage would the nurse expect the child to be? a. Trust b. Preoperations c. Secondary circular reaction d. Tertiary circular reaction ANS: D The 17 month old is in the fifth stage of the sensorimotor phase: tertiary circular reactions. The child uses active experimentation to achieve previously unattainable goals. Trust is Erikson's first stage. Preoperation is the stage of cognitive development, usually present in older toddlers and preschoolers. Secondary circular reactions last from about ages 4 to 8 months. PTS: 1 DIF: Cognitive Level: Comprehension OBJ: Nursing Process: Assessment MSC: Client Needs: Health Promotion and Maintenance 5. Which statement describes a toddler's cognitive development at age 20 months? a. Searches for an object only if he or she sees it being hidden. b. Realizes that “out of sight” is NnoUtRoSuINt GofTrBe.aCcOhM. c. Puts objects into a container but cannot take them out. d. Understands the passage of time such as “just a minute” and “in an hour.” ANS: B At this age the child is in the final sensorimotor stage. Children will now search for an object in several potential places, even though they saw only the original hiding place. Children have a more developed sense of objective permanence. They will search for objects even if they have not seen them hidden. Putting an object in a container but being unable to take it out indicates tertiary circular reactions. An embryonic sense of time exists; although toddlers may behave appropriately to time-oriented phrases, their sense of timing is exaggerated. PTS: 1 DIF: Cognitive Level: Comprehension OBJ: Nursing Process: Assessment MSC: Client Needs: Health Promotion and Maintenance 6. Although a 14 month old received a shock from an electrical outlet recently, the parents find the child about to place a paper clip in another outlet. What is the best interpretation of this behavior? a. Her cognitive development is delayed. b. This is typical behavior because toddlers are not very developed. c. This is typical behavior because of inability to transfer knowledge to new situations. d. This is not typical behavior because toddlers should know better than to repeat an act that caused pain. ANS: C During the tertiary circular reactions stage, children have only a rudimentary sense of the classification of objects. The appearance of an object denotes its function for these children. The slot of an outlet is for putting things into. Her cognitive development is appropriate for her age and represents typical behavior for a toddler. Only some awareness exists of a causal relation between events. PTS: 1 DIF: Cognitive Level: Comprehension OBJ: Nursing Process: Assessment MSC: Client Needs: Health Promotion and Maintenance 7. Two toddlers are playing in a sandbox when one child suddenly grabs a toy from the other child. What is the best interpretation of this behavior? a. This is typical behavior because toddlers are aggressive. b. This is typical behavior because toddlers are egocentric. c. Toddlers should know that sharing toys is expected of them. d. Toddlers should have the cognitive ability to know right from wrong. ANS: B Play develops from the solitary play of infancy to the parallel play of toddlers. The toddler plays alongside other children, not with them. This typical behavior of the toddler is not intentionally aggressive. Shared play is not within their cognitive development. Toddlers do not conceptualize shared play. Because the toddler cannot view the situation from the perspective of the other child, it is okay to take the toy. Therefore, no right or wrong is associated with taking a toy. PTS: 1 DIF: Cognitive Level: Comprehension OBJ: Nursing Process: AssessmentNURSINMGSTCB.:CCOlMient Needs: Health Promotion and Maintenance 8. Which statement about toilet training is correct? a. Bladder training is usually accomplished before bowel training. b. Wanting to please the parent helps motivate the child to use the toilet. c. Watching older siblings use the toilet confuses the child. d. Children must be forced to sit on the toilet when first learning. ANS: B Voluntary control of the anal and urethral sphincters is achieved sometime after the child is walking. The child must be able to recognize the urge to let go and to hold on. The child must want to please the parent by holding on rather than pleasing self by letting go. Bowel training precedes bladder training. Watching older siblings provides role modeling and facilitates imitation for the toddler. The child should be introduced to the potty chair or toilet in a nonthreatening manner. PTS: 1 DIF: Cognitive Level: Comprehension OBJ: Nursing Process: Implementation MSC: Client Needs: Health Promotion and Maintenance 9. Which characteristic best describes the gross motor skills of a 24-month-old child? a. Skips b. Rides tricycle c. Broad jumps d. Walks up and down stairs ANS: D The 24-month-old child can go up and down stairs alone with two feet on each step. Skipping and the ability to broad jump are skills acquired at age 3. Tricycle riding is achieved at age 4. PTS: 1 DIF: Cognitive Level: Comprehension OBJ: Nursing Process: Assessment MSC: Client Needs: Health Promotion and Maintenance 10. In the clinic waiting room, a nurse observes a parent showing an 18-month-old child how to make a tower out of blocks. In this situation the nurse should recognize that fact about this task. a. Blocks at this age are used primarily for throwing. b. Toddlers are too young to imitate the behavior of others. c. Toddlers are capable of building a tower of blocks. d. Toddlers are too young to build a tower of blocks. ANS: C This is a good parent-child interaction. The 18 month old is capable of building a tower of 3 or 4 blocks. The ability to build towers of blocks usually begins at age 15 months. With ongoing development, the child is able to build taller towers. At this age, children imitate others around them and no longer throw blocks. PTS: 1 DIF: Cognitive Level: Comprehension OBJ: Nursing Process: Assessment MSC: Client Needs: Health Promotion and Maintenance 11. The parents of a newborn say that their toddler “hates the baby and suggested that we put the baby in the trash can so the trash truck could take him away.” What is the nurse's best response to the parent's concern? a. “Let's see if we can figure outNwUhRySIyNoGuTrBto.CdOdMler hates the new baby.” b. “That's a strong statement to come from such a small child.” c. “Let's refer your child to counseling to work this hatred out. It's not a normal response.” d. “That is a normal response to the birth of a sibling. Let's look at ways to deal with this.” ANS: D The arrival of a new infant represents a crisis for even the best-prepared toddler. Toddlers have their entire schedule and routines disrupted because of the new family member. The nurse should work with parents on ways to involve the toddler in the newborn's care and help focus attention on the toddler. The toddler does not hate the infant. This is an expected response to the changes in routines and attention that affect the toddler. This is a normal response. The toddler can be provided with a doll to tend to its needs when the parent is performing similar care for the newborn. PTS: 1 DIF: Cognitive Level: Analysis OBJ: Nursing Process: Planning MSC: Client Needs: Psychosocial Integrity 12. A toddler's parent asks the nurse for suggestions on dealing with temper tantrums. What intervention is the most appropriate recommendation? a. Punish the child with an age appropriate punishment. b. Leave the child alone until the tantrum is over. c. Ignore the behavior, provided that it is not injurious. d. Explain to child that this is wrong. ANS: C The parent should be told that the best way to deal with temper tantrums is to ignore the behaviors, provided that the actions are not dangerous to the child. Tantrums are common during this age-group as the child becomes more independent and increasingly complex tasks overwhelm him or her. The parents and caregivers need to have consistent and developmentally appropriate expectations. Punishment and explanations will not be beneficial. PTS: 1 DIF: Cognitive Level: Application OBJ: Nursing Process: Implementation MSC: Client Needs: Psychosocial Integrity 13. A parent asks the nurse about how to respond to negativism in toddlers. What is the most appropriate recommendation? a. Punish the child. b. Provide more attention. c. Ask child not always to say “no.” d. Reduce the opportunities for a “no” answer. ANS: D The nurse should suggest to the parent that questions should be phrased with realistic choices rather than “yes” or “no” answers. This provides a sense of control for the toddler and reduces the opportunity for negativism. Negativism is not an indication of stubbornness or insolence and should not be punished. The negativism is not a function of attention; the child is testing limits to gain an understanding of the world. The toddler is too young to be asked to not always say “no.” NURSINGTB.COM PTS: 1 DIF: Cognitive Level: Analysis OBJ: Nursing Process: Planning MSC: Client Needs: Health Promotion and Maintenance 14. The parents of a 2 year old tell the nurse that they are concerned because the toddler has started to use “baby talk” since the arrival of their new baby. The nurse should recommend which intervention for the parents? a. Ignore the “baby talk.” b. Explain to the toddler that “baby talk” is for babies. c. Tell the toddler frequently, “You are a big kid now.” d. Encourage the toddler to practice more advanced patterns of speech. ANS: A The baby talk is a sign of regression in the toddler. It should be ignored, while praising the child for developmentally appropriate behaviors. Regression is children's way of saying that they are expressing stress. The parents should not introduce new expectations and should allow the child to master the developmental tasks without criticism. PTS: 1 DIF: Cognitive Level: Application OBJ: Nursing Process: Planning MSC: Client Needs: Health Promotion and Maintenance 15. Parents tell the nurse that their toddler eats little at mealtimes, only sits at the table with the family briefly, and wants snacks “all the time.” The nurse should recommend what intervention to the parents? a. Give her planned, frequent, and nutritious snacks. b. Offer rewards for eating at mealtimes. c. Avoid snacks so she is hungry at mealtimes. d. Explain to her in a firm manner what is expected of her. ANS: A Most toddlers exhibit a physiologic anorexia in response to the decreased nutritional requirement associated with the slower growth rate. Parents should assist the child to develop healthy eating habits. The toddler is often unable to sit through a meal. Frequent nutritious snacks are a good way to ensure proper nutrition. To help with developing healthy eating habits, food should not be used as positive or negative reinforcement for behavior. The child may develop habits of overeating or eat nonnutritious foods in response. PTS: 1 DIF: Cognitive Level: Application OBJ: Nursing Process: Planning MSC: Client Needs: Health Promotion and Maintenance 16. A father tells the nurse that his toddler wants the same plate and cup used at every meal, even if they go to a restaurant. The nurse should explain that this is a result of what factor? a. A sign that the child is spoiled. b. A way to exert unhealthy control. c. Regression, common at this age. d. Ritualism, common at this age. ANS: D The child is exhibiting the ritualism that is characteristic at this age. Ritualism is the need to maintain sameness and reliability. It provides a sense of comfort to the toddler. It will dictate certain principles in feeding practices, including rejecting a favorite food because it is served in a different container. This does not indicate a child who has unreasonable expectations or a need to exert control, but rather noNrUmRaSlIdNeGvTeBlo.CpOmMent. Toddlers use ritualistic behaviors to maintain necessary structure in their lives. This is not regression, which is a retreat from a present pattern of functioning. PTS: 1 DIF: Cognitive Level: Analysis OBJ: Nursing Process: Assessment MSC: Client Needs: Health Promotion and Maintenance 17. Developmentally, most children at age 12 months demonstrate what behavior? a. Use a spoon adeptly. b. Relinquish the bottle voluntarily. c. Eat the same food as the rest of the family. d. Reject all solid foods in preference to the bottle. ANS: C By age 12 months, most children are eating the same food that is prepared for the rest of the family. Using a spoon usually is not mastered until age 18 months. The parents should be engaged in weaning a child from a bottle if that is the source of liquid. Toddlers should be encouraged to drink from a cup at the first birthday and weaned totally by 14 months. The child should be weaned from a milk/formula-based diet to a balanced diet that includes iron-rich sources of food. PTS: 1 DIF: Cognitive Level: Comprehension OBJ: Nursing Process: Assessment MSC: Client Needs: Health Promotion and Maintenance 18. What is the most effective way to clean a toddler's teeth? a. Child to brush regularly with toothpaste of his or her choice. b. Parent to stabilize the chin with one hand and brush with the other. c. Parent to brush the mandibular occlusive surfaces, leaving the rest for the child. d. Parent to brush the front labial surfaces, leaving the rest for the child. ANS: B For young children, the most effective cleaning of teeth is done by the parents. Different positions can be used if the child's back is to the adult. The adult should use one hand to stabilize the chin and the other to brush the child's teeth. The child can participate in brushing, but for a thorough cleaning adult intervention is necessary. PTS: 1 DIF: Cognitive Level: Application OBJ: Nursing Process: Implementation MSC: Client Needs: Health Promotion and Maintenance 19. The nurse is discussing with a parent group the importance of fluoride for healthy teeth. What recommendation should the nurse make to the parents? a. Use fluoridated mouth rinses in children older than 1 year. b. Have children brush teeth with fluoridated toothpaste unless fluoride content of water supply is adequate. c. Give fluoride supplements to breastfed infants beginning at age 1 month. d. Determine whether water supply is fluoridated. ANS: D The decision about fluoride supplementation cannot be made until it is known whether the water supply contains fluoride and the amount. It is difficult to teach this age-group to spit out the mouthwash. Swallowing fluoridated mouthwashes can contribute to fluorosis. Fluoridated toothpaste is still indicated, but very small amounts are used. Fluoride supplementation is not recommended until after age 6 moNnUthRsS.INGTB.COM PTS: 1 DIF: Cognitive Level: Application OBJ: Nursing Process: Implementation MSC: Client Needs: Health Promotion and Maintenance 20. An appropriate recommendation in preventing tooth decay in young children would include which intervention? a. Substitute raisins for candy. b. Serve sweets after a meal. c. Use honey or molasses instead of refined sugar. d. Serve sweets between meals. ANS: B Sweets should be consumed with meals so the teeth can be cleaned afterward. This decreases the amount of time that the sugar is in contact with the teeth. Raisins, honey, and molasses are highly cariogenic and should be avoided. PTS: 1 DIF: Cognitive Level: Application OBJ: Nursing Process: Planning MSC: Client Needs: Health Promotion and Maintenance 21. What is the leading cause of death during the toddler period? a. Injuries b. Infectious diseases c. Congenital disorders d. Childhood diseases ANS: A Injuries are the single most common cause of death in children ages 1 through 4 years. It is the period of highest death rate from injuries of any childhood age-group except adolescence. Infectious and childhood diseases are less common cause of deaths in this age-group. Congenital disorders are the second leading cause of death in this age-group. PTS: 1 DIF: Cognitive Level: Comprehension OBJ: Nursing Process: Planning MSC: Client Needs: Health Promotion and Maintenance 22. What is the rationale for the nurse recommends to parents that peanuts are not a good snack food for toddlers? a. They are low in nutritive value. b. They are very high in sodium. c. They cannot be entirely digested. d. They can be easily aspirated. ANS: D Foreign-body aspiration is common during the second year of life. Although they chew well, children at this age may have difficulty with large pieces of food such as meat and whole hot dogs and with hard foods such as nuts or dried beans. Peanuts have many beneficial nutrients but should be avoided because of the risk of aspiration in this age-group. The sodium level may be a concern, but the risk of aspiration is more important. Many foods pass through the gastrointestinal tract incompletely digested. This is not necessarily detrimental to the child. PTS: 1 DIF: Cognitive Level: Application OBJ: Nursing Process: Implementation MSC: Client Needs: Health Promotion and Maintenance NURSINGTB.COM 23. The parent of a 16 month old asks, “What is the best way to keep our toddler from getting into our medicines at home?” The nurse should provide which advice? a. “All medicines should be locked securely away.” b. “The medicines should be placed in high cabinets.” c. “Chris just needs to be taught not to touch medicines.” d. “Medicines should not be kept in the homes of small children.” ANS: A The major reason for poisoning in the home is improper storage. Toddlers can climb, unlatch cabinets, and obtain access to high-security places. For medications, only a locked cabinet is safe. Toddlers can climb by using furniture. High places are not a deterrent to an exploring toddler. Toddlers are not able to generalize as dangerous all of the different forms of medications that may be available in the home. Teaching them not to touch medicines is not feasible. Many parents require medications for chronic illnesses. They must be taught safe storage for their home and when they visit other homes. PTS: 1 DIF: Cognitive Level: Application OBJ: Nursing Process: Implementation MSC: Client Needs: Health Promotion and Maintenance 24. Which is the most fatal type of burn in the toddler age-group? a. Flame burn from playing with matches. b. Scald burn from high-temperature tap water. c. Hot object burn from cigarettes or irons. d. Electric burn from electrical outlets. ANS: A Flame burns from matches and lighters represent one of the most fatal types of burns in the toddler age-group. Scald burns from water, hot object burns from cigarettes or irons, and electric burns from outlets are all significant causes of burn injury. The child should be protected from these causes by reducing the temperature of the hot water in the home, keeping objects such as cigarettes and irons away from children, and placing protective guards over electrical outlets when not in use. PTS: 1 DIF: Cognitive Level: Comprehension OBJ: Nursing Process: Planning MSC: Client Needs: Health Promotion and Maintenance 25. The mother of an 18-month-old child is concerned because the child's appetite has decreased. Which is the best response for the nurse to make to the mother? a. “It is important for your toddler to eat three meals a day and nothing in between.” b. “It is not unusual for toddlers to eat less during this developmental stage.” c. “Be sure to increase your child's milk consumption, which will improve nutrition.” d. “Giving your child a multivitamin supplement daily will increase your toddler's appetite.” ANS: B Toddlers need small, frequent meals. Nutritious selection throughout the day, rather than quantity, is more important with this age-group. Physiologically, growth slows and appetite decreases during the toddler period. Milk consumption should not exceed 16 to 24 ounces daily. Juice should be limited to 4 to 6 ounces per day. Increasing the amount of milk will only further decrease solid food intake. Supplemental vitamins are important for all children, but they do not increase appetite. NURSINGTB.COM PTS: 1 DIF: Cognitive Level: Application OBJ: Nursing Process: Implementation MSC: Client Needs: Physiologic Integrity 26. Which toy is the most developmentally appropriate for an 18- to 24-month-old child? a. A push-pull toy b. Nesting blocks c. A bicycle with training wheels d. A computer ANS: A Push-pull toys encourage large muscle activity and are appropriate for toddlers. Nesting blocks are more appropriate for a 12- to 15-month-old child. A bicycle with training wheels is appropriate for a preschool or young school-age child. A computer can be appropriate as early as the preschool years. PTS: 1 DIF: Cognitive Level: Application OBJ: Nursing Process: Teaching/Learning MSC: Client Needs: Health Promotion and Maintenance 27. What is the primary purpose of a transitional object? a. It helps the parents deal with the guilt when leaving the child. b. It keeps the child quiet at bedtime. c. It is effective in decreasing anxiety in the toddler. d. It decreases negativism and tantrums in the toddler. ANS: C Decreasing anxiety, particularly separation anxiety, is the function of a transitional object; it provides comfort to the toddler in stressful situations and helps make the transition from dependence to autonomy. A decrease in parental guilt (distress) is an indirect benefit of a transitional object. A transitional object may be part of a bedtime ritual, but it may not keep the child quiet at bedtime. A transitional object does not significantly affect negativity and tantrums, but it can comfort a child after tantrums. PTS: 1 DIF: Cognitive Level: Comprehension OBJ: Nursing Process: Implementation MSC: Client Needs: Health Promotion and Maintenance 28. Which comment indicates that the mother of a toddler needs further teaching about dental care? a. “We use well water so I give my toddler fluoride supplements.” b. “My toddler brushes his teeth with my help.” c. “My child will not need a dental checkup until his permanent teeth come in.” d. “I use a small nylon bristle brush for my toddler's teeth.” ANS: C Children should first see the dentist 6 months after the first primary tooth erupts and no later than age 30 months. Toddlers need fluoride supplements when they use a water supply that is not fluorinated. Toddlers also require supervision with dental care. The parent should finish brushing areas not reached by the child. A small nylon bristle brush works best for cleaning toddlers' teeth. PTS: 1 DIF: Cognitive Level: Analysis OBJ: Nursing Process: Evaluation MSC: Client Needs: Physiologic IntNegUriRtySINGTB.COM 29. What information should a nurse provide a mother who is concerned about preventing sleep problems in her 2-year-old child? a. Have the child always sleep in a quiet, darkened room. b. Provide high-carbohydrate snacks before bedtime. c. Communicate with the child's daytime caregiver about eliminating the afternoon nap. d. Use a night-light in the child's room. ANS: D The preschooler has a great imagination. Sounds and shadows can have a negative effect on sleeping behavior. Night-lights provide the child with the ability to visualize the environment and decrease the fear felt in a dark room. A dark, quiet room may be scary to a preschooler. High-carbohydrate snacks increase energy and do not promote relaxation. Most 2 year olds take one nap each day. Many give up the habit by age 3. Insufficient rest during the day can lead to irritability and difficulty sleeping at night. PTS: 1 DIF: Cognitive Level: Application OBJ: Nursing Process: Implementation MSC: Client Needs: Physiologic Integrity 30. A 16 months old, falls down a few stairs and then gets up and “scolds” the stairs as if they caused the fall. This is an example of which of the following? a. Animism b. Ritualism c. Irreversibility d. Delayed cognitive development ANS: A Animism is the attribution of life-like qualities to inanimate objects. By scolding the stairs, the toddler is attributing human characteristics to them. Ritualism is the need to maintain sameness and reliability. It provides a sense of comfort to the toddler. Irreversibility is the inability to reverse or undo actions initiated physically. Steven is acting in an age-appropriate manner. PTS: 1 DIF: Cognitive Level: Comprehension OBJ: Nursing Process: Assessment MSC: Client Needs: Health Promotion and Maintenance 31. Which should the nurse expect for a toddler's language development at age 18 months? a. Vocabulary of 25 words b. Increasing level of comprehension c. Use of phrases d. Approximately one third of speech understandable ANS: B During the second year of life, level of comprehension and understanding of speech increases and is far greater than the child's vocabulary. This is also true for bilingual children, who are able to achieve this linguistic milestone in both languages. The 18-month-old child has a vocabulary of 10 or more words. At this age, the child does not use one-word sentences or phrases. The child has a limited vocabulary of single words that are comprehensible. PTS: 1 DIF: Cognitive Level: Comprehension OBJ: Nursing Process: AssessmentNURSINMGSTCB.:CCOlMient Needs: Health Promotion and Maintenance 32. Parents have been using a rearward-facing, convertible car seat since their baby was born. The parents should be taught that most car seats can be safely switched to the forward-facing position when the child reaches which age? a. 1 year b. 2 years c. 3 years d. 4 years ANS: B It is now recommended that all infants and toddlers ride in rear-facing car safety seats until they reach the age of 2 years or the height or weight recommended by the car seat manufacturer. Children 2 years old and older who have outgrown the rear-facing height or weight limit for their car safety seat should use a forward-facing car safety seat with a harness up to the maximum height or weight recommended by the manufacturer. One year is too young to switch to a forward-facing position. PTS: 1 DIF: Cognitive Level: Comprehension OBJ: Nursing Process: Implementation MSC: Client Needs: Safe and Effective Care Environment MULTIPLE RESPONSE 1. Which are characteristics of the physical development of a 30-month-old child? (Select all that apply.) a. Birth weight has doubled. b. Primary dentition is complete. c. Sphincter control is achieved. d. Anterior fontanel is open. e. Length from birth is doubled. ANS: B, C Usually by age 30 months, the primary dentition of 20 teeth is completed, and the child has sphincter control in preparation for bowel and bladder control. A doubling of birth weight, opening of the anterior fontanel, and doubling of length are not characteristic of the physical development of a 30-month-old child. PTS: 1 DIF: Cognitive Level: Comprehension OBJ: Nursing Process: Assessment MSC: Client Needs: Health Promotion and Maintenance 2. Motor vehicle injuries are a significant threat to young children. Knowing this, the nurse plans a teaching session with a toddler's parents on car safety. Which will she teach? (Select all that apply.) a. Secure in a rear-facing, upright, car safety seat. b. Place the car safety seat in the rear seat, behind the driver's seat. c. Harness safety straps should be fit snugly. d. Place the car safety seat in the front passenger seat equipped with an air bag. e. After the age of 2 years, toddlers can be placed in a forward-facing car seat. ANS: A, C, E Toddlers younger than 2 years shoNuUldRSbIeNsGeTcBu.rCeOdMin a rear-facing, upright, approved car safety seat. After the age of 2 years, a forward-facing car seat can be used. Harness straps should be adjusted to provide a snug fit. The car safety seat should be placed in the middle of the rear seat. Children younger than 13 years should not ride in a front passenger seat that is equipped with an air bag. PTS: 1 DIF: Cognitive Level: Comprehension OBJ: Nursing Process: Planning MSC: Client Needs: Health Promotion and Maintenance 3. The nurse is assessing parental knowledge of temper tantrums. Which are true statements regarding temper tantrums? (Select all that apply.) a. Temper tantrums are a common response to anger and frustration in toddlers. b. Temper tantrums often include screaming, kicking, throwing things, and head banging. c. Parents can effectively manage temper tantrums by giving in to the child's demands. d. Children having temper tantrums should be safely isolated and ignored. e. Parents can learn to anticipate times when tantrums are more likely to occur. ANS: A, B, D, E Temper tantrums are a common response to anger and frustration in toddlers. They occur more often when toddlers are tired, hungry, bored, or excessively stimulated. A nap prior to fatigue or a snack if mealtime is delayed will be helpful in alleviated the times when tantrums are most likely to occur. Tantrums may include screaming, kicking, throwing things, biting themselves, or banging their head. Effective management of tantrums includes safely isolating and ignoring the child. The child should learn that nothing is gained by having a temper tantrum. Giving into the child's demands only increases the behavior. PTS: 1 DIF: Cognitive Level: Comprehension OBJ: Nursing Process: Assessment MSC: Client Needs: Health Promotion and Maintenance 4. A nurse is planning care for a hospitalized toddler in the preoperational thinking stage. Which characteristics should the nurse expect in this stage? (Select all that apply.) a. Concrete thinking b. Egocentrism c. Animism d. Magical thinking e. Ability to reason ANS: B, C, D The characteristics of preoperational thinking that occur for the toddler include egocentrism (views everything in relation to self), animism (believes that inert objects are alive), and magical thinking (believes that thinking something causes that event). Concrete thinking is seen in school-age children and ability to reason is seen with adolescents. PTS: 1 DIF: Cognitive Level: Application OBJ: Nursing Process: Planning MSC: Client Needs: Health Promotion and Maintenance NURSINGTB.COM 5. Which gross motor milestones should the nurse assess in an 18-month-old child? (Select all that apply.) a. Jumps in place with both feet b. Takes a few steps on tiptoe c. Throws ball overhand without falling d. Pulls and pushes toys e. Stands on one foot momentarily ANS: A, C, D An 18-month-old child can jump in place with both feet, throw a ball overhand without falling, and pull and push toys. Taking a few steps on tiptoe and standing on one foot momentarily are not acquired until 30 months of age. PTS: 1 DIF: Cognitive Level: Comprehension OBJ: Nursing Process: Assessment MSC: Client Needs: Health Promotion and Maintenance Chapter 33: The Preschooler and Family Perry: Maternal Child Nursing Care, 6th Edition MULTIPLE CHOICE 1. Which accomplishment would the nurse expect of a healthy 3-year-old child? a. Jump rope b. Ride a two-wheel bicycle c. Skip on alternate feet d. Balance on one foot for a few seconds ANS: D Three year olds are able to accomplish the gross motor skill of balancing on one foot. Jumping rope, riding a two-wheel bike, and skipping on alternate feet are gross motor skills of 5-year-old children. PTS: 1 DIF: Cognitive Level: Comprehension OBJ: Nursing Process: Assessment MSC: Client Needs: Health Promotion and Maintenance 2. In terms of fine motor development, what could the 3-year-old child be expected to do? a. Tie shoelaces b. Use scissors or a pencil very well c. Draw a person with 7 to 9 parts d. Copy (draw) a circle. ANS: D Three-year-olds are able to accomNpUliRshSItNhGe TfiBn.Ce OmMotor skill of drawing a circle. Tying shoelaces, using scissors or a pencil very well, and drawing a person with multiple parts are fine motor skills of 5-year-old children. PTS: 1 DIF: Cognitive Level: Comprehension OBJ: Nursing Process: Assessment MSC: Client Needs: Health Promotion and Maintenance 3. In terms of cognitive development, the preschooler would be expected to engage in what behavior? a. Use magical thinking b. Think abstractly c. Understand conservation of matter d. Be able to comprehend another person's perspective ANS: A Preschoolers' thinking is often described as magical thinking. Because of their egocentrism and transductive reasoning, they believe that thoughts are all-powerful. Abstract thought does not develop until school-age years. The concept of conservation is the cognitive task of school-age children ages 5 to 7 years. Five year olds cannot understand another's perspective. PTS: 1 DIF: Cognitive Level: Comprehension OBJ: Nursing Process: Assessment MSC: Client Needs: Health Promotion and Maintenance 4. What is descriptive of the preschooler's understanding of time? a. Has no understanding of time b. Associates time with events c. Can tell time on a clock d. Uses terms like “yesterday” appropriately ANS: B In a preschooler's understanding, time has a relation with events such as, “We'll go outside after lunch.” Preschoolers develop an abstract sense of time at age 3 years. Children can tell time on a clock at age 7 years. Children do not fully understand use of time-oriented words until age 6 years. PTS: 1 DIF: Cognitive Level: Comprehension OBJ: Nursing Process: Assessment MSC: Client Needs: Health Promotion and Maintenance 5. The nurse is caring for a hospitalized 4 year old. The parents tell the nurse that they will be back to visit at 6 PM. When the child asks the nurse, “when my parents are coming”, what is the nurse's best response? a. “They will be here soon.” b. “They will come after dinner.” c. “Let me show you on the clock when 6 PM is.” d. “I will tell you every time I see you how much longer it will be.” ANS: B A 4 year old understands time in relation to events such as meals. Children perceive “soon” as a very short time. The nurse may lose the child's trust if his parents do not return in the time he perceives as “soon.” Children cannot read or use a clock for practical purposes until age 7 years. This answer assumes that the child understands the concept of hours and minutes, which is not developed until age 5 or 6 years. NURSINGTB.COM PTS: 1 DIF: Cognitive Level: Analysis OBJ: Nursing Process: Assessment MSC: Client Needs: Health Promotion and Maintenance 6. A 4 year old is hospitalized with a serious bacterial infection. The child tells the nurse that, “I am sick because I was bad.” What is the nurse's best interpretation of this comment? a. It is a sign of stress. b. It is common at this age. c. It is suggestive of maladaptation. d. It is suggestive of excessive discipline at home. ANS: B Preschoolers cannot understand the cause and effect of illness. Their egocentrism makes them think that they are directly responsible for events, making them feel guilt for things outside of their control. Children of this age show stress by regressing developmentally or acting out. Maladaptation is unlikely. This comment does not imply excessive discipline at home. PTS: 1 DIF: Cognitive Level: Analysis OBJ: Nursing Process: Assessment MSC: Client Needs: Health Promotion and Maintenance 7. In terms of language and cognitive development, which behavior is expected of a 4-year-old child? a. Thinking in abstract terms. b. Following simple commands. c. Understanding conservation of matter. d. Comprehending another person's perspective. ANS: B Children ages 3 to 4 years can give and follow simple commands. Children cannot think abstractly at age 4 years. Conservation of matter is a developmental task of the school-age child. A 4-year-old child cannot comprehend another's perspective. PTS: 1 DIF: Cognitive Level: Comprehension OBJ: Nursing Process: Assessment MSC: Client Needs: Health Promotion and Maintenance 8. Which type of play is most typical of the preschool period? a. Solitary b. Parallel c. Associative d. Team ANS: C Associative play is group play in similar or identical activities but without rigid organization or rules. Solitary play is that of infants. Parallel play is that of toddlers. School-age children play in teams. PTS: 1 DIF: Cognitive Level: Comprehension OBJ: Nursing Process: Assessment MSC: Client Needs: Health Promotion and Maintenance 9. Why are imaginary playmates beneficial to the preschool child? a. Take the place of social interactions. b. Take the place of pets and other toys. c. Become friends in times of lonNeUliRnSeIsNsG. TB.COM d. Accomplish what the child has already successfully accomplished. ANS: C One purpose of an imaginary friend is to be a friend in time of loneliness. Imaginary friends do not take the place of social interactions but may encourage conversation. Imaginary friends do not take the place of pets or toys. They accomplish what the child is still attempting, not what has already been accomplished. PTS: 1 DIF: Cognitive Level: Comprehension OBJ: Nursing Process: Assessment MSC: Client Needs: Health Promotion and Maintenance 10. Which characteristics best describes the language of a 3-year-old child? a. Asks meanings of words b. Follows directional commands c. Can describe an object according to its composition d. Talks incessantly, regardless of whether anyone is listening ANS: D Because of the dramatic vocabulary increase at this age, 3 year olds are known to talk incessantly, regardless of whether anyone is listening. A 4 to 5 year old asks lots of questions and can follow simple directional commands. A 6 year old can describe an object according to its composition. PTS: 1 DIF: Cognitive Level: Comprehension OBJ: Nursing Process: Assessment MSC: Client Needs: Health Promotion and Maintenance 11. By what age would the nurse expect that most children could understand prepositional phrases such as “under,” “on top of,” “beside,” and “in back of”? a. 18 months b. 24 months c. 3 years d. 4 years ANS: D At 4 years, children can understand directional phrases. Children 18 to 24 months and 3 years of age are too young. PTS: 1 DIF: Cognitive Level: Comprehension OBJ: Nursing Process: Assessment MSC: Client Needs: Health Promotion and Maintenance 12. What skill that the nurse should expect a 5-year-old child to be able to master? a. Tie shoelaces. b. Use a knife to cut meat. c. Hammer a nail. d. Make change from a quarter. ANS: A Tying shoelaces is a fine motor task typical of 5 year olds. Using a knife to cut meat is a fine motor task of a 7 year old. Hammering a nail and making change from a quarter are fine motor tasks of an 8 to 9 year old. PTS: 1 DIF: CognitiNveURLSevINelG: TCBo.mCOprMehension OBJ: Nursing Process: Assessment MSC: Client Needs: Health Promotion and Maintenance 13. The nurse is guiding parents in selecting a day care facility for their child. When making the selection, it is especially important to focus on which consideration? a. Structured learning environment. b. Socioeconomic status of children. c. Cultural similarities of children. d. Teachers knowledgeable about development. ANS: D A teacher knowledgeable about development will structure activities for learning. A structured learning environment is not necessary at this age. Socioeconomic status is not the most important factor in selecting a preschool. Preschool is about expanding experiences with others; cultural similarities are not necessary. PTS: 1 DIF: Cognitive Level: Analysis OBJ: Nursing Process: Planning MSC: Client Needs: Health Promotion and Maintenance 14. The parent of a 4 year old tells the nurse that the child believes “monsters and the boogeyman” are in the bedroom at night. What is the nurse's best suggestion for coping with this problem? a. Insist that the child sleep with his parents until the fearful phase passes. b. Suggest involving the child to find a practical solution such as a night-light. c. Help the child understand that these fears are illogical. d. Tell the child frequently that monsters and the boogeyman do not exist. ANS: B A night-light shows a child that imaginary creatures do not lurk in the darkness. Letting the child sleep with parents or telling the child that these creatures do not exist will not get rid of the fears. A 4 year old is in the preconceptual age and cannot understand logical thought. PTS: 1 DIF: Cognitive Level: Application OBJ: Nursing Process: Planning MSC: Client Needs: Health Promotion and Maintenance 15. Preschoolers' fears can best be dealt with by which intervention? a. Actively involving them in finding practical methods to deal with the frightening experience. b. Forcing them to confront the frightening object or experience in the presence of their parents. c. Using logical persuasion to explain away their fears and help them recognize how unrealistic the fears are. d. Ridiculing their fears so they understand that there is no need to be afraid. ANS: A Actively involving the child in finding practical methods to deal with the frightening experience is the best way to deal with fears. Forcing a child to confront fears may make the child more afraid. Preconceptual thought prevents logical understanding. Ridiculing fears does not make them go away. PTS: 1 DIF: Cognitive Level: Application OBJ: Nursing Process: Implementation MSC: Client Needs: Health Promotion and Maintenance NURSINGTB.COM 16. What is a normal characteristic of the language development of a preschool-age child? a. Lisp b. Stammering c. Echolalia d. Repetition without meaning ANS: B Stammering and stuttering are normal dysfluencies in preschool-age children. Lisps are not a normal characteristic of language development. Echolalia and repetition are traits of toddlers' language. PTS: 1 DIF: Cognitive Level: Comprehension OBJ: Nursing Process: Assessment MSC: Client Needs: Health Promotion and Maintenance 17. During the preschool period, what should the emphasis of injury prevention be placed on? a. Constant vigilance and protection b. Punishment for unsafe behaviors c. Education for safety and potential hazards d. Limitation of physical activities ANS: C Education for safety and potential hazards is appropriate for preschoolers because they can begin to understand dangers. Constant vigilance and protection is not practical at this age since preschoolers are becoming more independent. Punishment may make children scared of trying new things. Limitation of physical activities is not appropriate. PTS: 1 DIF: Cognitive Level: Application OBJ: Nursing Process: Assessment MSC: Client Needs: Health Promotion and Maintenance 18. Acyclovir is given to children with chickenpox for what purpose? a. Minimize scarring b. Decrease the number of lesions c. Prevent aplastic anemia d. Prevent spread of the disease ANS: B Acyclovir decreases the number of lesions, shortens duration of fever, and decreases itching, lethargy, and anorexia; however, it does not prevent scarring. Preventing aplastic anemia is not a function of acyclovir. Only quarantine of the infected child can prevent the spread of disease. PTS: 1 DIF: Cognitive Level: Comprehension OBJ: Nursing Process: Implementation MSC: Client Needs: Physiologic Integrity 19. When is a child with chickenpox considered to be no longer contagious? a. When fever is absent b. When lesions are crusted c. 24 hours after lesions erupt d. 8 days after onset of illness ANS: B NURSINGTB.COM When the lesions are crusted, the chickenpox is no longer contagious. This may be a week after onset of disease. The child is still contagious once the fever has subsided and after the lesions erupt, and may or may not be contagious any time after 6 days as long as all lesions are crusted over. PTS: 1 DIF: Cognitive Level: Comprehension OBJ: Nursing Process: Assessment MSC: Client Needs: Physiologic Integrity 20. The nurse is performing an assessment on a child and notes the presence of Koplik's spots. In which communicable disease are Koplik's spots present? a. Rubella b. Measles (rubeola) c. Chickenpox (varicella) d. Exanthema subitum (roseola) ANS: B Koplik's spots are small, irregular red spots with a minute, bluish white center found on the buccal mucosa 2 days before the systemic rash of measles appears. Koplik's spots are not present with rubella, varicella, or roseola. PTS: 1 DIF: Cognitive Level: Application OBJ: Nursing Process: Assessment MSC: Client Needs: Physiologic Integrity 21. A common characteristic of those who sexually abuse children is that they: a. pressure the victim into secrecy. b. are usually unemployed and unmarried. c. are unknown to victims and victims' families. d. have many victims that are each abused only once. ANS: A Sex offenders may pressure the victim into secrecy, regarding the activity as a “secret between us” that other people may take away if they find out. Abusers are often employed upstanding members of the community. Most sexual abuse is committed by men and persons who are well known to the child. Abuse is often repeated with the same child over time. The relationship may start insidiously without the child realizing that sexual activity is part of the offer. PTS: 1 DIF: Cognitive Level: Comprehension OBJ: Nursing Process: Assessment MSC: Client Needs: Psychosocial Integrity 22. Which statement, made by a 4-year-old child's father, demonstrates an understanding about the care of the preschooler's teeth? a. “Because the ‘baby teeth' are not permanent, they are not important to the child.” b. “My son can be encouraged to brush his teeth after I have thoroughly cleaned his teeth.” c. “My son's ‘permanent teeth' will begin to come in at 4 to 5 years of age.” d. “Fluoride supplements can be discontinued when my son's ‘permanent teeth' erupt.” ANS: B NURSINGTB.COM Toddlers and preschoolers lack the manual dexterity to remove plaque adequately, so parents must assume this responsibility. Deciduous teeth are important because they maintain spacing and play an important role in the growth and development of the jaws and face and in speech development. Secondary teeth erupt at about 6 years of age. If the family does not live in an area where fluoride is included in the water supply, fluoride supplements should be continued. PTS: 1 DIF: Cognitive Level: Application OBJ: Nursing Process: Evaluation MSC: Client Needs: Physiologic Integrity 23. A 4-year-old child tells the nurse, “I do not want another blood sample drawn because I need all my insides, and I do not want anyone taking them out.” Which is the nurse's best interpretation of this statement? a. Child is being overly dramatic. b. Child has a disturbed body image. c. Preschoolers have poorly defined body boundaries. d. Preschoolers normally have a good understanding of their bodies. ANS: C Preschoolers have little understanding of body boundaries, which leads to fears of mutilation. The child is not capable of being dramatic at 4 years of age. She truly has fear. Body image is just developing in the school-age child. Preschoolers do not have good understanding of their bodies. PTS: 1 DIF: Cognitive Level: Analysis OBJ: Nursing Process: Assessment MSC: Client Needs: Health Promotion and Maintenance 24. Parents tell the nurse that they found their 3-year-old daughter and a male cousin of the same age inspecting each other closely as they used the bathroom. Which is the most appropriate recommendation the nurse should make? a. Punish children so this behavior stops. b. Neither condone nor condemn the curiosity. c. Allow children unrestricted permission to satisfy this curiosity. d. Get counseling for this unusual and dangerous behavior. ANS: B Three year olds become aware of anatomic differences and are concerned about how the other “works.” Such exploration should not be condoned or condemned. Children should not be punished for this normal exploration. Encouraging the children to ask questions of the parents and redirecting their activity are more appropriate than giving permission. Exploration is age-appropriate and not dangerous behavior. PTS: 1 DIF: Cognitive Level: Application OBJ: Nursing Process: Implementation MSC: Client Needs: Health Promotion and Maintenance 25. Which common childhood communicable disease may cause severe defects in the fetus when it occurs in its congenital form? a. Erythema infectiosum b. Roseola c. Rubeola d. Rubella ANS: D NURSINGTB.COM Rubella causes teratogenic effects on the fetus. There is a low risk of fetal death to those in contact with children affected with fifth disease. Roseola and rubeola are not dangerous to the fetus. PTS: 1 DIF: Cognitive Level: Comprehension OBJ: Nursing Process: Assessment MSC: Client Needs: Physiologic Integrity 26. Which is the causative agent of scarlet fever? a. Enteroviruses b. Corynebacterium organisms c. Scarlet fever virus d. Group A -hemolytic streptococci (GABHS) ANS: D GABHS infection causes scarlet fever. Enteroviruses do not cause the same complications. Corynebacterium organisms cause diphtheria. Scarlet fever is not caused by a virus. PTS: 1 DIF: Cognitive Level: Knowledge OBJ: Nursing Process: Assessment MSC: Client Needs: Physiologic Integrity 27. Which is probably the most important criterion on which to base the decision to report suspected child abuse? a. Inappropriate parental concern for the degree of injury b. Absence of parents for questioning about child's injuries c. Inappropriate response of child d. Incompatibility between the history and injury observed ANS: D Conflicting stories about the “accident” are the most indicative red flags of abuse. Inappropriate response of caregiver or child may be present, but is subjective. Parents should be questioned at some point during the investigation. PTS: 1 DIF: Cognitive Level: Application OBJ: Nursing Process: Assessment MSC: Client Needs: Psychosocial Integrity MULTIPLE RESPONSE 1. Which play patterns does a 3-year-old child typically display? (Select all that apply.) a. Imaginary play b. Parallel play c. Cooperative play d. Structured play e. Associative play ANS: A, B, C, E Children between ages 3 and 5 years enjoy parallel and associative play. Children learn to share and cooperate as they play in small groups. Play is often imitative, dramatic, and creative. Imaginary friends are common around age 3 years. Structured play is typical of school-age children. PTS: 1 DIF: CognitiNveURLSevINelG: TCBo.mCOprMehension OBJ: Nursing Process: Assessment MSC: Client Needs: Health Promotion and Maintenance 2. In terms of language and cognitive development, a 4-year-old child would be expected to have which traits? (Select all that apply.) a. Think in abstract terms b. Sexual curiosity c. Understand conservation of matter d. Use sentences of eight words e. Tell exaggerated stories ANS: B, E Children 4 years of age demonstrate sexual curiosity and tell exaggerated stories. Children cannot think abstractly at age 4 years. Conservation of matter is a developmental task of the school-age child. Five-year-old children use sentences with eight words with all parts of speech. PTS: 1 DIF: Cognitive Level: Application OBJ: Nursing Process: Assessment MSC: Client Needs: Health Promotion and Maintenance Chapter 34: The School-Age Child and Family Perry: Maternal Child Nursing Care, 6th Edition MULTIPLE CHOICE 1. Which statement accurately describes physical development of a child during the school-age years? a. The child's weight almost triples. b. A child grows an average of 2 inches/year. c. Few physical differences are apparent among children at the end of middle childhood. d. Fat gradually increases, which contributes to the child's heavier appearance. ANS: B In middle childhood, growth in height and weight occur at a slower pace. Between the ages of 6 and 12 years, children grow 2 inches/year. In middle childhood, children's weight will almost double; they gain 3 kg/year. At the end of middle childhood, girls grow taller and gain more weight than boys. Children take on a slimmer look with longer legs in middle childhood. PTS: 1 DIF: Cognitive Level: Comprehension OBJ: Nursing Process: Assessment MSC: Client Needs: Health Promotion and Maintenance 2. Generally what is the earliest age at which puberty begins? a. 13 years in girls, 13 years in boys b. 11 years in girls, 11 years in boys c. 10 years in girls, 12 years in bNoUysRSINGTB.COM d. 12 years in girls, 10 years in boys ANS: C Puberty signals the beginning of the development of secondary sex characteristics. This begins in girls earlier than in boys. Usually a 2-year difference occurs in the age at onset. Girls and boys do not usually begin puberty at the same age; girls usually begin earlier than boys do. PTS: 1 DIF: Cognitive Level: Comprehension OBJ: Nursing Process: Assessment MSC: Client Needs: Health Promotion and Maintenance 3. Which statement describes the cognitive abilities of school-age children? a. Have developed the ability to reason abstractly b. Become capable of scientific reasoning and formal logic c. Progress from making judgments based on what they reason to making judgments based on what they see d. Have the ability to classify, group and sort, and hold a concept in their minds while making decisions based on that concept ANS: D In Piaget's stage of concrete operations, children have the ability to group and sort and make conceptual decisions. Children cannot reason abstractly until late adolescence. Scientific reasoning and formal logic are skills of adolescents. Making judgments on what the child sees versus what he or she reasons is not a developmental skill. PTS: 1 DIF: Cognitive Level: Comprehension OBJ: Nursing Process: Assessment MSC: Client Needs: Health Promotion and Maintenance 4. What describes moral development in younger school-age children? a. The standards of behavior now come from within themselves. b. They do not yet experience a sense of guilt when they misbehave. c. They know the rules and behaviors expected of them but do not understand the reasons behind them. d. They no longer interpret accidents and misfortunes as punishment for misdeeds. ANS: C Children who are ages 6 and 7 years know the rules and behaviors expected of them but do not understand the reasons for them. Young children do not believe that standards of behavior come from within themselves but that rules are established and set down by others. Younger school-age children learn standards for acceptable behavior, act according to these standards, and feel guilty when they violate them. Misfortunes and accidents are viewed as punishment for bad acts. PTS: 1 DIF: Cognitive Level: Comprehension OBJ: Nursing Process: Assessment MSC: Client Needs: Health Promotion and Maintenance 5. Which statement characterizes moral development in older school-age children? a. They are able to judge an act by the intentions that prompted it rather than just by the consequences. b. Rules and judgments become more absolute and authoritarian. c. They view rule violations in an isolated context. d. They know the rules but cannoNtUuRnSdIeNrGstTaBn.dCOthMe reasons behind them. ANS: A Older school-age children are able to judge an act by the intentions that prompted the behavior rather than just by the consequences. Rules and judgments become less absolute and authoritarian. Rule violation is likely to be viewed in relation to the total context in which it appears. Both the situation and the morality of the rule itself influence reactions. PTS: 1 DIF: Cognitive Level: Comprehension OBJ: Nursing Process: Assessment MSC: Client Needs: Health Promotion and Maintenance 6. An 8-year-old girl tells the nurse that she has cancer because God is punishing her for “being bad.” She shares her concern that, if she dies, she will go to hell. How should the nurse interpret this belief? a. It is a belief common at this age. b. It is a belief that forms the basis for most religions. c. The belief is suggestive of excessive family pressure. d. The statement suggests a failed attempt to develop a conscience. ANS: A Children at this age may view illness or injury as a punishment for a real or imagined mystique. The belief in divine punishment is common at this age. PTS: 1 DIF: Cognitive Level: Analysis OBJ: Nursing Process: Diagnosis MSC: Client Needs: Psychosocial Integrity 7. What is the role of the peer group in the life of school-age children? a. Gives them an opportunity to learn dominance and hostility. b. Allows them to remain dependent on their parents for a longer time. c. Decreases their need to learn appropriate sex roles. d. Provides them with security as they gain independence from their parents. ANS: D Peer-group identification is an important factor in gaining independence from parents. Through peer relationships, children learn ways to deal with dominance and hostility. They also learn how to relate to people in positions of leadership and authority and explore ideas and the physical environment. Peer-group identification helps in gaining independence rather than remaining dependent. A child's concept of appropriate sex roles is influenced by relationships with peers. PTS: 1 DIF: Cognitive Level: Comprehension OBJ: Nursing Process: Assessment MSC: Client Needs: Health Promotion and Maintenance 8. A group of boys ages 9 and 10 years have formed a “boys-only” club that is open to neighborhood and school friends who have skateboards. How should this behavior be interpreted? a. Behavior that encourages bullying and sexism. b. Behavior that reinforces poor peer relationships. c. Characteristic of social development of this age. d. Characteristic of children who later are at risk for membership in gangs. ANS: C One of the outstanding characterisNtiUcRs SoIfNmGTidBd.CleOcMhildhood is the creation of formalized groups or clubs. Peer-group identification and association are essential to a child's socialization. Poor relationships with peers and a lack of group identification can contribute to bullying. A boys-only club does not have a direct correlation with later gang activity. PTS: 1 DIF: Cognitive Level: Analysis OBJ: Nursing Process: Assessment MSC: Client Needs: Health Promotion and Maintenance 9. Which statement is descriptive of the play of school-age children? a. Individuality in play is better tolerated than at earlier ages. b. Knowing the rules of a game gives an important sense of belonging. c. They like to invent games, making up the rules as they go. d. Team play helps children learn the universal importance of competition and winning. ANS: B Play involves increased physical skill, intellectual ability, and fantasy. Children form groups and cliques and develop a sense of belonging to a team or club. At this age, children begin to see the need for rules. Conformity and ritual permeate their play. Their games have fixed and unvarying rules, which may be bizarre and extraordinarily rigid. With team play, children learn about competition and the importance of winning, an attribute highly valued in the United States. PTS: 1 DIF: Cognitive Level: Comprehension OBJ: Nursing Process: Assessment MSC: Client Needs: Health Promotion and Maintenance 10. What is the characteristic of dishonest behavior in children ages 8 to 10 years? a. Cheating during games is now more common. b. Lying results from the inability to distinguish between fact and fantasy. c. They may steal because their sense of property rights is limited. d. They may lie to meet expectations set by others that they have been unable to attain. ANS: D Older school-age children may lie to meet expectations set by others to which they have been unable to measure up. Cheating usually becomes less frequent as the child matures. In this age-group, children are able to distinguish between fact and fantasy. Young children may lack a sense of property rights; older children may steal to supplement an inadequate allowance, or it may be an indication of serious problems. PTS: 1 DIF: Cognitive Level: Comprehension OBJ: Nursing Process: Assessment MSC: Client Needs: Health Promotion and Maintenance 11. A 9 year old often comes to the school nurse complaining of stomach pains. The teacher says that the child has lately been somewhat aggressive and stubborn in the classroom. What should the school nurse recognize as the possible trigger for these behaviors? a. Signs of stress b. Developmental delay c. A physical problem causing emotional stress d. Lack of adjustment to the school environment ANS: A Signs of stress include stomach paNinUsRoSrINhGeTadBa.CcOheM, sleep problems, bed-wetting, changes in eating habits, aggressive or stubborn behavior, reluctance to participate, or regression to early behaviors. This child is exhibiting signs of stress, not developmental delay, a physical problem, or lack of adjustment. PTS: 1 DIF: Cognitive Level: Analysis OBJ: Nursing Process: Assessment MSC: Client Needs: Physiologic Integrity 12. Which statement best describes fear in school-age children? a. They are increasingly fearful for body safety. b. Most of the new fears that trouble them are related to school and family. c. They should be encouraged to hide their fears to prevent ridicule by peers. d. Those who have numerous fears need continuous protective behavior by parents to eliminate these fears. ANS: B During the school-age years, children experience a wide variety of fears, but new fears related predominantly to school and family bother children during this time. During the middle-school years, children become less fearful of body safety than they were as preschoolers. Parents and other persons involved with children should discuss their fear with them individually or as a group activity. Sometimes school-age children hide their fears to avoid being teased. Hiding the fears does not end them and may lead to phobias. PTS: 1 DIF: Cognitive Level: Comprehension OBJ: Nursing Process: Assessment MSC: Client Needs: Psychosocial Integrity 13. The father of 12 year old tells the nurse that he is concerned about his child getting “fat.” The child's body mass index for age is at the 60th percentile. What is the most appropriate nursing action to address the father's concern? a. Reassure the father that his child is not “fat.” b. Reassure the father that the weight is just a growing child. c. Suggest a low-calorie, low-fat diet, and provide food suggestions. d. Explain that this is typical of the growth pattern of children at this age. ANS: D This is a characteristic pattern of growth in preadolescent boys, in which the growth in height has slowed in preparation for the pubertal growth spurt but weight is still gained. This should be reviewed with both the father and child, and a plan should be developed to maintain physical exercise and a balanced diet. Saying that the child is not “fat” is false reassurance. The child's weight is high for his/her height. The child needs to maintain his/her physical activity. The father is concerned; an explanation is required. A nutritional diet with physical activity should be sufficient to maintain his balance. PTS: 1 DIF: Cognitive Level: Application OBJ: Nursing Process: Implementation MSC: Client Needs: Health Promotion and Maintenance 14. The school nurse has been asked to begin teaching sex education in the 5th grade. Which statement should be the foundation for the information the nurse should present? a. Children in 5th grade are too young for sex education. b. Children should be discouraged from asking too many questions. c. Correct terminology should be reserved for children who are older. d. Sex can be presented as a normNaUlRpSaINrtGoTfBg.rCoOwMth and development. ANS: D When sex information is presented to school-age children, sex should be treated as a normal part of growth and development. Fifth graders are usually 10 to 11 years old. This age is not too young to speak about physiologic changes in their bodies. They should be encouraged to ask questions. Preadolescents need precise and concrete information. PTS: 1 DIF: Cognitive Level: Analysis OBJ: Nursing Process: Implementation MSC: Client Needs: Health Promotion and Maintenance 15. What is an important consideration for the school nurse who is planning a class on bicycle safety to consider? a. Most bicycle injuries involve collision with an automobile. b. Head injuries are the major causes of bicycle-related fatalities. c. Children should wear bicycle helmets if they ride on paved streets. d. Children should not ride double unless the bicycle has an extra-large seat. ANS: B The most important aspect of bicycle safety is to encourage the rider to use a protective helmet. Head injuries are the major cause of bicycle-related fatalities. Although motor vehicle collisions do cause injuries to bicyclists, most injuries result from falls. The child should always wear a properly fitted helmet approved by the U.S. Consumer Product Safety Commission. Children should not ride double. PTS: 1 DIF: Cognitive Level: Analysis OBJ: Nursing Process: Implementation MSC: Client Needs: Health Promotion and Maintenance 16. When teaching injury prevention during the school-age years, the nurse should include what topic? a. Teaching the need to fear strangers. b. Teaching basic rules of water safety. c. Avoiding letting children cook in microwave ovens. d. Cautioning children against engaging in competitive sports. ANS: B Water safety instruction is an important source of injury prevention at this age. The child should be taught to swim, select safe and supervised places to swim, swim with a companion, check sufficient water depth for diving, and use an approved flotation device. Teach stranger safety, not fearing strangers. This includes not going with strangers, not having personalized clothing in public places, having children tell parents if anyone makes them uncomfortable, and teaching children to say “no” in uncomfortable situations. Teach children safe cooking methods. Caution against engaging in hazardous sports, such as those involving trampolines. PTS: 1 DIF: Cognitive Level: Application OBJ: Nursing Process: Implementation MSC: Client Needs: Health Promotion and Maintenance 17. The ability to mentally understand that 1 + 3 = 4 and 4  3 = 1 occurs in which stage of cognitive development? a. Concrete operations stage b. Formal operations stage c. Intuitive thought stage d. Preoperations stage ANS: A NURSINGTB.COM By 7 to 8 years of age, the child is able to retrace a process (reversibility) and has the skills necessary for solving mathematical problems. This stage is called concrete operations. The formal operations stage deals with abstract reasoning and does not occur until adolescence. Thinking in the intuitive stage is based on immediate perceptions. A child in this stage often solves problems by random guessing. In preoperational thinking, the child is usually able to add 1 + 3 = 4 but is unable to retrace the process. PTS: 1 DIF: Cognitive Level: Comprehension OBJ: Nursing Process: Assessment MSC: Client Needs: Health Promotion and Maintenance 18. Which activity is most appropriate for developing fine motor skills in the school-age child? a. Drawing b. Singing c. Soccer d. Swimming ANS: A Activities such as drawing, building models, and playing a musical instrument increase the school-age child's fine motor skills. Singing is an appropriate activity for the school-age child, but it does not increase fine motor skills. The school-age child needs to participate in group activities to increase both gross motor skills and social skills, but group activities do not increase fine motor skills. Swimming is an activity that also increases gross motor skills. PTS: 1 DIF: Cognitive Level: Analysis OBJ: Nursing Process: Evaluation MSC: Client Needs: Health Promotion and Maintenance 19. Which comment is most developmentally typical of a 7-year-old boy? a. “I am a Power Ranger, so don't make me angry.” b. “I don't know whether I like Mary or Joan better.” c. “My mom is my favorite person in the world.” d. “Jimmy is my best friend.” ANS: D School-age children form friendships with peers of the same sex, those who live nearby, and other children who have toys that they enjoy sharing. Magical thinking is developmentally appropriate for the preschooler. Opposite-sex friendships are not typical for the 7-year-old child. Seven-year-old children socialize with their peers, not their parents. PTS: 1 DIF: Cognitive Level: Analysis OBJ: Nursing Process: Assessment MSC: Client Needs: Health Promotion and Maintenance 20. Identify the statement that is the most accurate about moral development in the 9-year-old school-age child. a. Right and wrong are based on physical consequences of behavior. b. The child obeys parents becauNseURofSIfNeaGrToBf.CpOuMnishment. c. The school-age child conforms to rules to please others. d. Parents are the determiners of right and wrong for the school-age child. ANS: C The 7- to 12-year-old child bases right and wrong on a good-boy or good-girl orientation in which the child conforms to rules to please others and avoid disapproval. Children 4 to 7 years of age base right and wrong on consequences, the most important consideration for this age-group. Parents determine right and wrong for the child younger than 4 years of age. PTS: 1 DIF: Cognitive Level: Comprehension OBJ: Nursing Process: Assessment MSC: Client Needs: Health Promotion and Maintenance 21. Which behavior by parents or teachers will best assist the child in negotiating the developmental task of industry? a. Identifying failures immediately and asking the child's peers for feedback b. Structuring the environment so the child can master tasks c. Completing homework for children who are having difficulty in completing assignments d. Decreasing expectations to eliminate potential failures ANS: B The task of the caring teacher or parent is to identify areas in which a child is competent and to build on successful experiences to foster feelings of mastery and success. Structuring the environment to enhance self-confidence and to provide the opportunity to solve increasingly more complex problems will promote a sense of mastery. Asking peers for feedback reinforces the child's feelings of failure. When teachers or parents complete children's homework for them, it sends the message that they do not trust the children to do a good job. Providing assistance and suggestions and praising their best efforts are more appropriate. Decreasing expectations to eliminate failures will not promote a sense of achievement or mastery. PTS: 1 DIF: Cognitive Level: Application OBJ: Nursing Process: Implementation MSC: Client Needs: Health Promotion and Maintenance 22. A nurse is assessing an older school-age child recently admitted to the hospital. Which assessment indicates that the child is in an appropriate stage of cognitive development? a. The child's addition and subtraction ability b. The child's ability to classify c. The child's vocabulary d. The child's play activity ANS: B The ability to classify things from simple to complex and the ability to identify differences and similarities are cognitive skills of the older school-age child; this demonstrates use of classification and logical thought processes. Subtraction and addition are appropriate cognitive activities for the young school-age child. Vocabulary is not as valid an assessment of cognitive ability as is the child's ability to classify. Play activity is not as valid an assessment of cognitive function aNsUiRs SthINeGaTbBil.iCtyOMto classify. PTS: 1 DIF: Cognitive Level: Application OBJ: Nursing Process: Evaluation MSC: Client Needs: Health Promotion and Maintenance 23. A child has an avulsed (knocked-out) tooth. In which medium should the nurse instruct the parents to place the tooth for transport to the dentist? a. Cold milk b. Cold water c. Warm salt water d. A dry, clean jar ANS: A An avulsed tooth should be placed in a suitable medium for transport, either cold milk or saliva (under the child's or parent's tongue). Cold milk is a more suitable medium for transport than cold water, warm salt water, or a dry, clean jar. PTS: 1 DIF: Cognitive Level: Application OBJ: Nursing Process: Implementation MSC: Client Needs: Physiologic Integrity 24. A nurse is teaching parents of first-grade children general guidelines to assist their children in adapting to school. Which statement by the parents indicates they understand the teaching? a. “We will only meet with the teacher if problems occur.” b. “We will discourage hobbies so our child focuses on schoolwork.” c. “We will plan a trip to the library as often as possible.” d. “We will expect our child to make all As in school.” ANS: C General guidelines for parents to help their child in school include sharing an interest in reading. The library should be used frequently and books the child is reading should be discussed. Hobbies should be encouraged. The parents should not expect all As. They should focus on growth more than grades. PTS: 1 DIF: Cognitive Level: Analysis OBJ: Nursing Process: Implementation MSC: Client Needs: Health Promotion and Maintenance 25. Parents of a 12-year-old child ask the clinic nurse, “How many hours of sleep should our child get each night?” The nurse should respond that 12-year-old children need _ hours of sleep at night. a. 8 b. 9 c. 10 d. 11 ANS: B School-age children usually do not require naps, but they do need to sleep approximately 11 hours at age 5 years and 9 hours at age 12 years each night. PTS: 1 DIF: Cognitive Level: Application OBJ: Nursing Process: Implementation MSC: Client Needs: Health Promotion and Maintenance 26. A nurse planning care for a school-age child should take into account that which thought process is seen at this age? a. Animism b. Magical thinking c. Ability to conserve d. Thoughts are all-powerful ANS: C NURSINGTB.COM One cognitive task of school-age children is mastering the concept of conservation. At an early age (5 to 7 years), children grasp the concept of reversibility of numbers as a basis for simple mathematics problems (e.g., 2 + 4 = 6 and 6  4 = 2). They learn that simply altering their arrangement in space does not change certain properties of the environment, and they are able to resist perceptual cues that suggest alterations in the physical state of an object. Animism, magical thinking, and believing that thoughts are all-powerful are thought processes seen in preschool children. PTS: 1 DIF: Cognitive Level: Comprehension OBJ: Nursing Process: Planning MSC: Client Needs: Health Promotion and Maintenance MULTIPLE RESPONSE 1. Which demonstrates the school-age child's developing logic in the stage of concrete operations? (Select all that apply.) a. The school-age child is able to recognize that he can be a son, brother, or nephew at the same time. b. The school-age child understands the principles of adding, subtracting, and reversibility. c. The school-age child understands the principles of adding, subtracting, and reversibility. d. The school-age child has thinking that is characterized by egocentrism and animism. ANS: A, B, C The school-age child understands that the properties of objects do not change when their order, form, or appearance does. Conservation occurs in the concrete operations stage. Comprehension of class inclusion occurs as the school-age child's logic increases. The child begins to understand that a person can be in more than one class at the same time. This is characteristic of concrete thinking and logical reasoning. The school-age child is able to understand principles of adding, subtracting, and the process of reversibility, which occurs in the stage of concrete operations. Thinking that is characterized by egocentrism and animism occurs in the intuitive thought stage, not the concrete operations stage of development. PTS: 1 DIF: Cognitive Level: Analysis OBJ: Nursing Process: Evaluation MSC: Client Needs: Health Promotion and Maintenance 2. Peer victimization is becoming a significant problem for school-age children and adolescents in the United States. Parents should be educated regarding signs that a child is being bullied. These might include: (Select all that apply.) a. The child spends an inordinate amount of time in the nurse's office. b. Belongings frequently go missing or are damaged. c. The child wants to be driven to school. d. School performance improvesN. URSINGTB.COM e. The child freely talks about his or her day. ANS: A, B, C Signs that may indicate a child is being bullied are similar to signs of other types of stress and include nonspecific illness or complaints, withdrawal, depression, school refusal, and decreased school performance. Children expressed fear of going to school or riding the school bus, and their belongings often are damaged or missing. Very often, children will not talk about what is happening to them. PTS: 1 DIF: Cognitive Level: Planning OBJ: Nursing Process: Assessment MSC: Client Needs: Psychosocial Integrity 3. A nurse is planning care for a 7-year-old child hospitalized with osteomyelitis. Which activities should the nurse plan to bring from the playroom for the child? (Select all that apply.) a. Paper and some paints b. Board games c. Jack-in-the-box d. Stuffed animals e. Computer games ANS: A, B, E School-age children become fascinated with complex board, card, or computer games that they can play alone, with a best friend, or with a group. They also enjoy sewing, cooking, carpentry, gardening, and creative activities such as painting. Jack-in-the-box and stuffed animals would be appropriate for a toddler or preschool child. PTS: 1 DIF: Cognitive Level: Application OBJ: Nursing Process: Planning MSC: Client Needs: Health Promotion and Maintenance 4. A nurse teaches parents that team play is important for school-age children. Which abilities can children develop by experiencing team play? (Select all that apply.) a. Achieve personal goals over group goals. b. Learn complex rules. c. Experience competition. d. Learn about division of labor. ANS: B, C, D Team play helps stimulate cognitive growth because children are called on to learn many complex rules, make judgments about those rules, plan strategies, and assess the strengths and weaknesses of members of their own team and members of the opposing team. Team play can also contribute to children's social, intellectual, and skill growth. Children work hard to develop the skills needed to become team members, to improve their contribution to the group, and to anticipate the consequences of their behavior for the group. Team play teaches children to modify or exchange personal goals for goals of the group; it also teaches them that division of labor is an effective strategy for attaining a goal. PTS: 1 DIF: Cognitive Level: Comprehension OBJ: Nursing Process: Implementation MSC: Client Needs: Health Promotion and Maintenance NURSINGTB.COM MATCHING Because school-age children have developed increased muscular coordination and can apply cognitive ability to their behavior, the number of injuries in middle childhood is diminished compared to early childhood. Even so, injuries still occur near home and school. The most effective means of prevention is education for both the child and family regarding the hazards of risk-taking and the improper use of equipment. Please match the developmental ability of the school-age child with the injury for which he or she is at risk. a. Motor vehicle accidents b. Drowning c. Burns d. Poisoning e. Bodily damage 1. Is apt to overdo 2. Confidence exceeds physical capacity 3. Is excited by speed and motion 4. Enjoys trying new things 5. May be easily influenced by peers 1. ANS: B PTS: 1 DIF: Cognitive Level: Application OBJ: Nursing Process: Assessment MSC: Client Needs: Physiologic Integrity NOT: Physically active school-age children are highly susceptible to all of these injuries. Drowning is a high risk when the child over extends him/herself while in the water. As the child is increasingly involved in activities away from home, influenced by peers, and excited to try new things, risk for injury will be present. It is important that the nurse emphasize injury prevention for all school-age children. 2. ANS: E PTS: 1 DIF: Cognitive Level: Application OBJ: Nursing Process: Assessment MSC: Client Needs: Physiologic Integrity NOT: Physically active school-age children are highly susceptible to all of these injuries. Risk for injury increases as the child's confidence exceeds his or her capacities. As the child is increasingly involved in activities away from home, influenced by peers, and excited to try new things, risk for injury will be present. It is important that the nurse emphasize injury prevention for all school-age children. 3. ANS: A PTS: 1 DIF: Cognitive Level: Application OBJ: Nursing Process: Assessment MSC: Client Needs: Physiologic Integrity NOT: Physically active school-age children are highly susceptible to all of these injuries. The child's interest in speed and motion increases the risk for motor vehicle accidents. As the child is increasingly involved in activities away from home, influenced by peers, and excited to try new things, risk for injury will be present. It is important that the nurse emphasize injury prevention for all school-age children. 4. ANS: C PTS: 1 DIF: Cognitive Level: Application OBJ: Nursing Process: Assessment MSC: Client Needs: Physiologic Integrity NOT: Physically active school-age children are highly susceptible to all of these injuries. As the child is increasingly involved in activities away from home, influenced by peers, and excited to try new things, risk for burn injuries will increase. It is important that the nurse emphasize injury prevention for all school-age children. 5. ANS: D PTS: 1 DIF: Cognitive Level: Application OBJ: Nursing Process: Assessment MSC: Client Needs: Physiologic Integrity NOT: Physically active school-age children are highly susceptible to all of these injuries. As the child is increasingly involved in activities NawUaRySfIrNoGmThBo.CmOeM, influenced by peers, and excited to try new things, risk for poisoning injuries will be present. It is important that the nurse emphasize injury prevention for all school-age children. Chapter 35: The Adolescent and Family Perry: Maternal Child Nursing Care, 6th Edition MULTIPLE CHOICE 1. In girls, what is the initial indication of puberty? a. Menarche b. Growth spurt c. Growth of pubic hair d. Breast development ANS: D In most girls, the initial indication of puberty is the appearance of breast buds, an event known as the larche. The usual sequence of secondary sexual characteristic development in girls is breast changes, rapid increase in height and weight, growth of pubic hair, appearance of axillary hair, menstruation, and abrupt deceleration of linear growth. PTS: 1 DIF: Cognitive Level: Knowledge OBJ: Nursing Process: Assessment MSC: Client Needs: Health Promotion and Maintenance 2. The mean age of menarche in the United States is: a. 11.5 years. b. 12.5 years. c. 13.5 years. d. 14 years. ANS: B NURSINGTB.COM The average age of menarche is 12 years and 4 months in North American girls, with a normal range of 10.5 to 15 years. PTS: 1 DIF: Cognitive Level: Knowledge OBJ: Nursing Process: Assessment MSC: Client Needs: Health Promotion and Maintenance 3. By what age should concerns about pubertal delay be considered in boys? a. 12 to 12.5 years b. 12.5 to 13 years c. 13 to 13.5 years d. 13.5 to 14 years ANS: D Concerns about pubertal delay should be considered for boys who exhibit no enlargement of the testes or scrotal changes by 13.5 to 14 years of age. Ages younger than 13.5 years are too young for initial concern. PTS: 1 DIF: Cognitive Level: Knowledge OBJ: Nursing Process: Assessment MSC: Client Needs: Health Promotion and Maintenance 4. A 14 years old, mentions that he now has to use deodorant but never had to before. The nurse's response should be based on what knowledge? a. Eccrine sweat glands in the axillae become fully functional during puberty. b. Sebaceous glands become extremely active during puberty. c. New deposits of fatty tissue insulate the body and cause increased sweat production. d. Apocrine sweat glands reach secretory capacity during puberty. ANS: D The apocrine sweat glands, nonfunctional in children, reach secretory capacity during puberty. They secrete a thick substance as a result of emotional stimulation that, when acted on by surface bacteria, becomes highly odoriferous. They are limited in distribution and grow in conjunction with hair follicles in the axillae, genital and anal areas, and other areas. Eccrine sweat glands are present almost everywhere on the skin and become fully functional and respond to emotional and thermal stimulation. Sebaceous glands become extremely active at this time, especially those on the genitals and the “flush” areas of the body, such as face, neck, shoulders, upper back, and chest. This increased activity is important in the development of acne. New deposits of fatty tissue insulate the body and cause increased sweat production, but this is not the etiology of apocrine sweat gland activity. PTS: 1 DIF: Cognitive Level: Comprehension OBJ: Nursing Process: Implementation MSC: Client Needs: Health Promotion and Maintenance 5. According to Erikson, what is the psychosocial task of adolescence? a. Intimacy b. Identity c. Initiative d. Independence ANS: B Traditional psychosocial theory holds that the developmental crises of adolescence lead to the formation of a sense of identity. InNtUimRaScINyGisTBth.CeOdMevelopmental stage for early adulthood. Initiative is the developmental stage for early childhood. Independence is not one of Erikson's developmental stages. PTS: 1 DIF: Cognitive Level: Comprehension OBJ: Nursing Process: Assessment MSC: Client Needs: Health Promotion and Maintenance 6. According to Piaget, the adolescent is in the fourth stage of cognitive development with a focus on what area? a. Formal operations b. Concrete operations c. Conventional thought d. Postconventional thought ANS: A Cognitive thinking culminates with capacity for abstract thinking. This stage, the period of formal operations, is Piaget's fourth and last stage. The concrete operations stage usually develops between ages 7 and 11 years. Conventional and postconventional thought refer to Kohlberg's stages of moral development. PTS: 1 DIF: Cognitive Level: Comprehension OBJ: Nursing Process: Assessment MSC: Client Needs: Health Promotion and Maintenance 7. Which aspect of cognition develops during adolescence? a. Capability to use a future time perspective. b. Ability to place things in a sensible and logical order. c. Ability to see things from the point of view of another. d. Progress from making judgments based on what they see to making judgments based on what they reason. ANS: A Adolescents are no longer restricted to the real and actual. They are also concerned with the possible; they think beyond the present. During concrete operations (between ages 7 and 11 years), children exhibit the ability to place things in a sensible and logical order, the ability to see things from another's point of view, and the ability to make judgments based on what they reason rather than just what they see. PTS: 1 DIF: Cognitive Level: Comprehension OBJ: Nursing Process: Assessment MSC: Client Needs: Health Promotion and Maintenance 8. The parents of a 14 year old express concerns about the number of hours their child spends with friends. The nurse explains that peer relationships become more important during adolescence because: a. adolescents dislike their parents. b. adolescents no longer need parental control. c. they provide adolescents with a feeling of belonging. d. they promote a sense of individuality in adolescents. ANS: C The peer group serves as a strong support to teenagers, providing them with a sense of belonging and strength and power. During adolescence, the parent-child relationship changes from one of protection-dependency to one of mutual affection and quality. Parents continue to play an important role in personalNaUnRdShINeaGltThB-.rCeOlaMted decisions. The peer group forms the transitional world between dependence and autonomy. PTS: 1 DIF: Cognitive Level: Comprehension OBJ: Nursing Process: Planning MSC: Client Needs: Health Promotion and Maintenance 9. An adolescent boy tells the nurse that he has recently had homosexual feelings. The nurse's response should be based on what knowledge? a. This indicates that the adolescent is homosexual. b. This indicates that the adolescent will become homosexual as an adult. c. The adolescent should be referred for psychotherapy. d. The adolescent should be encouraged to share his feelings and experiences. ANS: D These adolescents are at increased risk for health-damaging behaviors, not because of the sexual behavior itself, but because of society's reaction to the behavior. The nurse's first priority is to give the young man permission to discuss his feelings about this topic, knowing that the nurse will maintain confidentially, appreciate his feelings, and remain sensitive to his need to talk it. In recent studies among self-identified gay, lesbian, and bisexual adolescents, many of the adolescents report changing their self-labels one or more times during their adolescence. PTS: 1 DIF: Cognitive Level: Comprehension OBJ: Nursing Process: Implementation MSC: Client Needs: Health Promotion and Maintenance 10. Matt, aged 14 years, seems to be always eating, although his weight is appropriate for his height. What is the best explanation for this situation? a. This is normal because of increase in body mass. b. This is abnormal and suggestive of future obesity. c. His caloric intake would have to be excessive. d. He is substituting food for unfilled needs. ANS: A In adolescence, nutritional needs are closely related to the increase in body mass. The peak requirements occur in the years of maximal growth. The caloric and protein requirements are higher than at almost any other time of life. This describes the expected eating pattern for young adolescents as long as weight and height are appropriate; obesity and substitution of food for unfilled needs are not concerns. PTS: 1 DIF: Cognitive Level: Comprehension OBJ: Nursing Process: Assessment MSC: Client Needs: Health Promotion and Maintenance 11. Which predisposes the adolescent to feel an increased need for sleep? a. An inadequate diet b. Rapid physical growth c. Decreased activity that contributes to a feeling of fatigue d. The lack of ambition typical of this age-group ANS: B During growth spurts, the need for sleep is increased. Rapid physical growth, the tendency toward overexertion, and the overall increased activity of this age contribute to fatigue. None of the other options have the same or greater influence of sleep needs as so physical growth. NURSINGTB.COM PTS: 1 DIF: Cognitive Level: Comprehension OBJ: Nursing Process: Assessment MSC: Client Needs: Health Promotion and Maintenance 12. What is the most common cause of death in the adolescent age-group? a. Drownings b. Firearms c. Drug overdoses d. Motor vehicles ANS: D The leading cause of all adolescent deaths in the United States is motor vehicle accidents. Drownings, firearms, and drug overdoses are major concerns in adolescence but do not cause the majority of deaths. PTS: 1 DIF: Cognitive Level: Comprehension OBJ: Nursing Process: Assessment MSC: Client Needs: Safe and Effective Care Environment 13. The nurse is completing a health history with a 16-year-old male. He informs the nurse that he has started using smokeless tobacco after he plays baseball. Which information regarding smokeless tobacco would be most correct for the nurse to provide to this teen? a. This form of nicotine is not addicting. b. It has been proven to be carcinogenic. c. It is a much easy habit to stop. d. It is a safe alternative to cigarette smoking. ANS: B Smokeless tobacco is a popular substitute for cigarettes and poses serious health hazards to children and adolescents. Smokeless tobacco is associated with cancer of the mouth and jaw. Smokeless tobacco is just as addictive as cigarettes. Although teens believe that it is easy to stop using smokeless tobacco, this is not the case. A popular belief is that smokeless tobacco is a safe alternative to cigarettes; this has been proven incorrect. Half of all teens who use smokeless tobacco agree that it poses significant health risks. PTS: 1 DIF: Cognitive Level: Application OBJ: Nursing Process: Implementation MSC: Client Needs: Physiologic Integrity 14. A 14-year-old boy and his parents are concerned about bilateral breast enlargement. The nurse's discussion of their concerns should be based on what understanding? a. This is usually a benign and temporary condition. b. This is usually caused by Klinefelter's syndrome. c. Administration of estrogen effectively reduces gynecomastia. d. Administration of testosterone effectively reduces gynecomastia. ANS: A The male breast responds to hormonal changes. Some degree of bilateral or unilateral breast enlargement occurs frequently in boys during puberty. Klinefelter syndrome is a chromosomal condition that affects male physical and cognitive development. This is not a manifestation of Klinefelter's syndrome. Administration of estrogen or testosterone will have no effect on the reduction of breast tissue and may aggravate the condition. PTS: 1 DIF: CognitiNveURLSevINelG: TCBo.mCOprMehension OBJ: Nursing Process: Implementation MSC: Client Needs: Physiologic Integrity 15. How may anorexia nervosa best be described? a. Occurring most frequently in adolescent males. b. Occurring most frequently in adolescents from lower socioeconomic groups. c. Resulting from a posterior pituitary disorder. d. Resulting in severe weight loss in the absence of obvious physical causes. ANS: D The etiology of anorexia remains unclear, but a distinct psychologic component is present. The diagnosis is based primarily on psychologic and behavioral criteria. Anorexia nervosa is observed more commonly in adolescent girls and young women. It does not occur most frequently in adolescents from a lower socioeconomic group. In reality, anorexic adolescents are often from families of means who have high parental expectations for achievement. Anorexia is a psychiatric disorder. PTS: 1 DIF: Cognitive Level: Comprehension OBJ: Nursing Process: Assessment MSC: Client Needs: Physiologic Integrity 16. The weight loss of anorexia nervosa is often triggered by what event? a. Sexual abuse b. School failure c. Independence from family d. Traumatic interpersonal conflict ANS: D Weight loss may be triggered by a typical adolescent crisis such as the onset of menstruation or a traumatic interpersonal incident; situations of severe family stress such as parental separation or divorce; or circumstances in which the young person lacks personal control, such as being teased, changing schools, or entering college. There may in fact be a history of sexual abuse; however, this is not the trigger. These adolescents are often overachievers who are successful in school, not failures in school. The adolescent is most often enmeshed with his or her family. PTS: 1 DIF: Cognitive Level: Comprehension OBJ: Nursing Process: Assessment MSC: Client Needs: Psychosocial Integrity 17. Which statement is most descriptive of bulimia during adolescence? a. Strong sense of control over eating behavior b. Feelings of elation after the binge-purge cycle c. Profound lack of awareness that the eating pattern is abnormal d. Weight that can be normal, slightly above normal, or below normal ANS: D Individuals with bulimia are of normal weight or more commonly slightly above normal weight. Those who also restrict their intake can become severely underweight. Behavior related to this eating disorder is secretive, frenzied, and out of control. These cycles are followed by self-deprecating thoughts and a depressed mood. These young women are keenly aware that this eating pattern is abnormal. PTS: 1 DIF: Cognitive Level: Comprehension OBJ: Nursing Process: Diagnosis MSC: Client Needs: Psychosocial InNteUgRriStyINGTB.COM 18. The nurse is caring for an adolescent diagnosed with acute cocaine toxicity. Initial data collection should include what focus? a. Mode of administration. b. Actual content of the drug. c. Function the drug plays in the adolescent's life. d. Adolescent's level of interest in rehabilitation. ANS: A When the drug is questionable or unknown, every effort must be made to determine the type, amount of drug taken, the mode and time of administration, and factors relating to the onset of presenting symptoms. Because the actual content of most street drugs is highly questionable, this information would be difficult to obtain. It is helpful to know the pattern of use but not essential during this emergency. This is an inappropriate time for an evaluation about the level of interest in rehabilitation. PTS: 1 DIF: Cognitive Level: Comprehension OBJ: Nursing Process: Assessment MSC: Client Needs: Psychosocial Integrity 19. Which statement best describes Tanner staging? a. Predictable stages of puberty that are based on chronologic age. b. Staging of puberty based on the initiation of menarche and nocturnal emissions. c. Predictable stages of puberty that are based on primary and secondary sexual characteristics. d. Staging of puberty based on the initiation of primary sexual characteristics. ANS: C Tanner sexual-maturing ratings are based on the development of stages of primary and secondary sexual characteristics. Tanner stages are not based on chronologic age. The age at which an adolescent enters puberty is variable. The puberty stage in girls begins with breast development. The puberty stage in boys begins with genital enlargement. Primary sexual characteristics are not the sole basis of Tanner staging. PTS: 1 DIF: Cognitive Level: Comprehension OBJ: Nursing Process: Assessment MSC: Client Needs: Health Promotion and Maintenance 20. Which behavior suggests appropriate psychosocial development in the adolescent? a. The adolescent seeks validation for socially acceptable behavior from older adults. b. The adolescent is self-absorbed and self-centered and has sudden mood swings. c. Adolescents move from peers and enjoy spending time with family members. d. Conformity with the peer group increases in late adolescence. ANS: B During adolescence, energy is focused within. Adolescents concentrate on themselves in an effort to determine who they are or who they will be. Adolescents are likely to be impulsive and impatient. Parents often describe their teenager as being “self-centered” or “lazy.” The peer group validates acceptable behavior during adolescence. Adolescents move from family and enjoy spending time with peers. Adolescents also spend time alone; they need this time to think and concentrate on themselves. Conformity becomes less important in late adolescence. PTS: 1 DIF: Cognitive Level: Comprehension OBJ: Nursing Process: AssessmentNURSINMGSTCB.:CCOlMient Needs: Health Promotion and Maintenance 21. The parents of a 15-year-old girl are concerned that their adolescent spends too much time looking in the mirror. Which statement is the most appropriate for the nurse to make? a. “Your teenager needs clearer and stricter limits about her behavior.” b. “Your teenager needs more responsibility at home.” c. “During adolescence this behavior is not unusual.” d. “The behavior is abnormal and needs further investigation.” ANS: C Egocentric and narcissistic behavior is normal during this period of development. The teenager is seeking a personal identity. Stricter limits are not an appropriate response for a behavior that is part of normal development. More responsibility at home is not an appropriate response for this situation. The behavior is normal and needs no further investigation. PTS: 1 DIF: Cognitive Level: Application OBJ: Nursing Process: Assessment MSC: Client Needs: Health Promotion and Maintenance 22. Which statement is the most appropriate advice to give parents of a 16 year old who is rebellious? a. “You need to be stricter so that your teen stops trying to test the limits.” b. “You need to collaborate with your child and set limits that are perceived as being reasonable.” c. “Increasing your teen's involvement with peers will improve his/her self-esteem.” d. “Allow your teenager to choose the type of discipline that is used in your home.” ANS: B Allowing teenagers to choose between realistic options and offering consistent and structured discipline typically enhances cooperation and decreases rebelliousness. Structure helps adolescents to feel more secure and assists them in the decision-making process. Setting stricter limits typically does not decrease rebelliousness or decrease testing of parental limits. Increasing peer involvement does not typically increase self-esteem. PTS: 1 DIF: Cognitive Level: Application OBJ: Nursing Process: Implementation MSC: Client Needs: Health Promotion and Maintenance 23. Which statement by the nurse is most appropriate to a 15 year old whose friend has mentioned suicide? a. “Tell your friend to come to the clinic immediately.” b. “You need to gather details about your friend's suicide plan.” c. “Your friend's threat needs to be taken seriously and immediate help for your friend is important.” d. “If your friend mentions suicide a second time, you will want to get your friend some help.” ANS: C Suicide is the third most common cause of death among American adolescents. A suicide threat from an adolescent serves as a dramatic message to others and should be taken seriously. Adolescents at risk should be targeted for supportive guidance and counseling before a crisis occurs. Instructing a 15 year old to tell a friend to come to the clinic immediately provides the teen with limited information and does not address the concern. It is important to determine whether a person threatening suicide has a plan of action; however, the best information for the 15 year olNdUtRoShINavGeTiBs.CthOaMt all threats of suicide should be taken seriously and immediate help is important. Taking time to gather details or waiting until the teen discusses it a second time may be too late. PTS: 1 DIF: Cognitive Level: Application OBJ: Nursing Process: Evaluation MSC: Client Needs: Psychosocial Integrity 24. When planning care for adolescents, the nurse should consider which intervention? a. Teach parents first, and they, in turn, will teach the teenager. b. Provide information for their long-term health needs because teenagers respond best to long-range planning. c. Maintain the parents' role by providing explanations for treatment and procedures to the parents only. d. Give information privately to adolescents about how they can manage the specific problems that they identify. ANS: D Problems that teenagers identify and are interested in are typically the problems that they are the most willing to address. Confidentiality is important to adolescents. Adolescents prefer to confer privately (without parents) with the nurse and health care provider. Teenagers are socially and cognitively at the developmental stage where the health care provider can teach them. The nurse must keep in mind that teenagers are more interested in immediate health care needs than in long-term needs. PTS: 1 DIF: Cognitive Level: Application OBJ: Nursing Process: Planning MSC: Client Needs: Health Promotion and Maintenance 25. A 17 year old tells the nurse that he/she is not having sex because it would make his/her parents very angry. This response indicates that the adolescent has a developmental lag in which area? a. Cognitive development b. Moral development c. Psychosocial development d. Psychosexual development ANS: B The appropriate moral development for a 17 year old would include evidence that the teenager has internalized a value system and does not depend on parents to determine right and wrong behaviors. Adolescents who remain concrete thinkers may never advance beyond conformity to please others and avoid punishment. Cognitive development is related to moral development, but it is not the pivotal point in determining right and wrong behaviors. Identity formation is the psychosocial development task. Energy is focused within the adolescent, who exhibits behavior that is self-absorbed and egocentric. Although a task during adolescence is the development of a sexual identity, the teenager's dependence on the parents' sanctioning of right or wrong behavior is more appropriately related to moral development. PTS: 1 DIF: Cognitive Level: Analysis OBJ: Nursing Process: Evaluation MSC: Client Needs: Health Promotion and Maintenance 26. What is the first sign of pubertal change seen with boys? a. Testicular enlargement b. Facial hair c. Scrotal enlargement d. Voice deepens ANS: A NURSINGTB.COM The first sign of pubertal changes in boys is testicular enlargement in response to testosterone secretion, which usually occurs in Tanner stage 2. Slight pubic hair is present and the smooth skin texture of the scrotum is somewhat altered. As testosterone secretion increases, the penis, testes, and scrotum enlarge. During Tanner stages 4 and 5, rising levels of testosterone cause the voice to deepen and facial hair appears at the corners of the upper lip and chin. PTS: 1 DIF: Cognitive Level: Comprehension OBJ: Nursing Process: Implementation MSC: Client Needs: Health Promotion and Maintenance 27. A young adolescent tells the nurse, “I feel gawky.” The nurse should explain that this occurs in adolescents because of what physiological event? a. Growth of the extremities and neck precedes growth in other areas. b. Growth is in the trunk and chest. c. The hip and chest breadth increases. d. The growth spurt occurs earlier in girls. ANS: A Growth in length of the extremities and neck precedes growth in other areas, and, because these parts are the first to reach adult length, the hands and feet appear larger than normal during adolescence. Increases in hip and chest breadth take place in a few months, followed several months later by an increase in shoulder width. These changes are followed by increases in length of the trunk and depth of the chest. This sequence of changes is responsible for the characteristic long-legged, gawky appearance of early adolescent children. The growth spurt occurs earlier in girls than in boys but this does not address the child's concern. PTS: 1 DIF: Cognitive Level: Application OBJ: Nursing Process: Implementation MSC: Client Needs: Health Promotion and Maintenance 28. Which hormone is responsible for the growth of beard, mustache, and body hair in the males? a. Estrogen b. Pituitary hormone c. Androgen d. Progesterone ANS: C Beard, mustache, and body hair on the chest, upward along the linea alba, and sometimes on other areas (e.g., back and shoulders) appears in males and is androgen dependent. Estrogen and progesterone are produced by the ovaries in the female and do not contribute to body hair appearance in the male. The pituitary hormone does not have any relationship to body hair appearance in the male. PTS: 1 DIF: Cognitive Level: Analysis OBJ: Nursing Process: Evaluation MSC: Client Needs: Health Promotion and Maintenance 29. A nurse is caring for an adolescenNt UhoRsSpINitGalTizBe.CdOfoMr cellulitis. The nurse notes that the adolescent experiences many “mood swings” throughout the day. The nurse interprets this behavior as: a. requiring a referral to a mental health counselor. b. requiring some further laboratory testing. c. normal behavior. d. related to feelings of depression. ANS: C Adolescents vacillate in their emotional states between considerable maturity and child-like behavior. One minute they are exuberant and enthusiastic; the next minute they are depressed and withdrawn. Because of these mood swings, adolescents are frequently labeled as unstable, inconsistent, and unpredictable, but the behavior is normal. The behavior would not require a referral to a mental health counselor or further laboratory testing. The mood swings do not indicate depression. PTS: 1 DIF: Cognitive Level: Comprehension OBJ: Nursing Process: Assessment MSC: Client Needs: Health Promotion and Maintenance 30. Young people diagnosed with anorexia nervosa are often described as possessing what personal characteristic? a. Independent b. Disruptive c. Conforming d. Low achieving ANS: C Individuals with anorexia nervosa are described as perfectionist, academically high achievers, conforming, and conscientious. Independent, disruptive, and low achieving are not part of the behavioral characteristics of anorexia nervosa. PTS: 1 DIF: Cognitive Level: Comprehension OBJ: Nursing Process: Assessment MSC: Client Needs: Psychosocial Integrity 31. Which symptoms should the nurse expect to observe during the physical assessment of an adolescent girl with severe weight loss and disrupted metabolism associated with anorexia nervosa? a. Dysmenorrhea and oliguria b. Tachycardia and tachypnea c. Heat intolerance and increased blood pressure d. Lowered body temperature and brittle nails ANS: D Symptoms of anorexia nervosa include lower body temperature, severe weight loss, decreased blood pressure, dry skin, brittle nails, altered metabolic activity, and presence of lanugo hair. Amenorrhea, rather than dysmenorrhea, and cold intolerance are manifestations of anorexia nervosa. Bradycardia, rather than tachycardia, may be present. PTS: 1 DIF: Cognitive Level: Comprehension OBJ: Nursing Process: Assessment MSC: Client Needs: Physiologic Integrity: Physiologic Adaptation NURSINGTB.COM 32. Which statement is most descriptive of central nervous system stimulants? a. They produce strong physical dependence. b. They can result in strong psychologic dependence. c. Withdrawal symptoms are life threatening. d. Acute intoxication can lead to coma. ANS: B Central nervous system stimulants such as amphetamines and cocaine produce a strong psychologic dependence. This class of drugs does not produce strong physical dependence and can be withdrawn without much danger. Acute intoxication leads to violent, aggressive behavior, or psychotic episodes characterized by paranoia, uncontrollable agitation, and restlessness. PTS: 1 DIF: Cognitive Level: Comprehension OBJ: Nursing Process: Assessment MSC: Client Needs: Psychosocial Integrity 33. A school nurse is conducting a class with adolescents on suicide. Which true statement about suicide should the nurse include in the teaching session? a. A sense of hopelessness and despair are a normal part of adolescence. b. Gay and lesbian adolescents are at a particularly high risk for suicide. c. Problem-solving skills are of limited value to the suicidal adolescent. d. Previous suicide attempts are not an indication of risk for completed suicides. ANS: B A significant number of teenage suicides occur among homosexual youths. Gay and lesbian adolescents who live in families or communities that do not accept homosexuality are likely to suffer low self-esteem, self-loathing, depression, and hopelessness as a result of a lack of acceptance from their family or community. At-risk teenagers include those who are depressed, have poor problem-solving skills, or use drugs and alcohol. History of previous suicide attempt is a serious indicator for possible suicide completion in the future. PTS: 1 DIF: Cognitive Level: Application OBJ: Nursing Process: Teaching/Learning MSC: Client Needs: Psychosocial Integrity 34. Which is the most commonly used method in completed suicides? a. Firearms b. Drug overdose c. Self-inflected laceration d. Carbon monoxide poisoning ANS: A Firearms are the most commonly used instruments in completed suicides among both males and females. For adolescent boys, firearms are followed by hanging and overdose. For adolescent females, overdose and strangulation are the next most common means of completed suicide. The most common method of suicide attempt is overdose or ingestion of potentially toxic substances such as drugs. The second most common method of suicide attempt is self-inflicted laceration. Carbon monoxide poisoning is not one of the more frequent forms of suicide completion. PTS: 1 DIF: Cognitive Level: Knowledge OBJ: Nursing Process: AssessmentNURSINMGSTCB.:CCOlMient Needs: Psychosocial Integrity 35. Which is the most significant factor in distinguishing those who commit suicide from those who make suicidal attempts or threats? a. Social isolation b. Level of stress c. Degree of depression d. Desire to punish others ANS: A Social isolation is a significant factor in distinguishing adolescents who will kill themselves from those who will not. It is also more characteristic of those who complete suicide than of those who make attempts or threats. Level of stress, degree of depression, and desire to punish others are contributing factors in suicide, but they are not the most significant factor in distinguishing those who complete suicide from those who attempt suicide. PTS: 1 DIF: Cognitive Level: Comprehension OBJ: Nursing Process: Assessment MSC: Client Needs: Psychosocial Integrity MATCHING Place in correct order the sequence of maturational changes for girls. Begin with the first change seen, sequencing to the last change. a. Growth of pubic hair b. Rapid increase in height and weight c. Breast changes d. Menstruation e. Appearance of axillary hair 1. First change 2. Second change 3. Third change 4. Fourth change 5. Fifth change 1. ANS: C PTS: 1 DIF: Cognitive Level: Analysis OBJ: Nursing Process: Evaluation MSC: Client Needs: Health Promotion and Maintenance NOT: The usual sequence of maturational changes for girls is breast changes, rapid increase in height and weight, growth of pubic hair, appearance of axillary hair, and then menstruation, which usually begins 2 years after the first signs. 2. ANS: B PTS: 1 DIF: Cognitive Level: Analysis OBJ: Nursing Process: Evaluation MSC: Client Needs: Health Promotion and Maintenance NOT: The usual sequence of maturational changes for girls is breast changes, rapid increase in height and weight, growth of pubic hair, appearance of axillary hair, and then menstruation, which usually begins 2 years after the first signs. 3. ANS: A PTS: 1 DIF: Cognitive Level: Analysis OBJ: Nursing Process: Evaluation MSC: Client Needs: Health Promotion and Maintenance NOT: The usual sequence of maturational changes for girls is breast changes, rapid increase in height and weight, growth of pubic hair, appearance of axillary hair, and then menstruation, which usually begins 2 years after the first signs. 4. ANS: E PTS: 1 DIF: Cognitive Level: Analysis OBJ: Nursing Process: Evaluation NURSINMGSTCB.:CCOlMient Needs: Health Promotion and Maintenance NOT: The usual sequence of maturational changes for girls is breast changes, rapid increase in height and weight, growth of pubic hair, appearance of axillary hair, and then menstruation, which usually begins 2 years after the first signs. 5. ANS: D PTS: 1 DIF: Cognitive Level: Analysis OBJ: Nursing Process: Evaluation MSC: Client Needs: Health Promotion and Maintenance NOT: The usual sequence of maturational changes for girls is breast changes, rapid increase in height and weight, growth of pubic hair, appearance of axillary hair, and then menstruation, which usually begins 2 years after the first signs. Chapter 36: Impact of Chronic Illness, Disability, or End-of-Life Care for the Child and Family Perry: Maternal Child Nursing Care, 6th Edition MULTIPLE CHOICE 1. The nurse case manager is planning a care conference about a young child who has complex health care needs and will soon be discharged home. Whom should the nurse invite to the conference? a. Family and nursing staff b. Social worker, nursing staff, and primary care physician c. Family and key health professionals involved in child's care d. Primary care physician and key health professionals involved in child's care ANS: C A multidisciplinary conference is necessary for coordination of care for children with complex health needs. The family and key health professionals who are involved in the child's care are included. The nursing staff can address the nursing care needs of the child with the family, but other involved disciplines must be included. The family must be included in the discharge conferences, which allow them to determine what education they will require and the resources needed at home. A member of the nursing staff must be included to review the nursing needs of the child. PTS: 1 DIF: Cognitive Level: Analysis OBJ: Nursing Process: Planning MSC: Client Needs: Safe and Effective Care Environment NURSINGTB.COM 2. Lindsey, age 5 years with a diagnosis of cerebral palsy, will be starting kindergarten next month and will be placed in a special education classroom. The parents are tearful when telling the nurse about this and state that they did not realize that their child's disability was so severe. How should the nurse interpret this parental response? a. This is a sign that parents are in denial. b. This is a normal anticipated time of parental stress. c. The parents need to learn more about cerebral palsy. d. The parents are used to having expectations that are too high. ANS: B Parenting a child with a chronic illness can be very stressful for parents. There are anticipated times that parental stress increases. One of these identified times is when the child begins school. Nurses can help parents recognize and plan interventions to work through these stressful periods. The parents are not in denial; they are responding to the child's placement in school. The parents are not exhibiting signs of a knowledge deficit or expectations that are too high; this is their first interaction with the school system with this child. PTS: 1 DIF: Cognitive Level: Analysis OBJ: Nursing Process: Assessment MSC: Client Needs: Psychosocial Integrity 3. Which behavior is considered an approach behavior in parents of chronically ill children? a. Inability to adjust to a progression of the disease or condition. b. Anticipation of future problems and seeking guidance and answers. c. Looking for new cures without a perspective toward possible benefit. d. Failing to recognize seriousness of child's condition despite physical evidence. ANS: B Approach behaviors are coping mechanisms that result in a family's movement toward adjustment and resolution of the crisis of having a child with a chronic illness or disability. The parents who anticipate future problems and seek guidance and answers are demonstrating approach behaviors. They are demonstrating positive actions in caring for their child. Avoidance behaviors include being unable to adjust to a progression of the disease or condition, looking for new cures without a perspective toward possible benefit, and failing to recognize the seriousness of the child's condition despite physical evidence. These behaviors would suggest that the parents are moving away from adjustment or adaptation in the crisis of a child with chronic illness or disability. PTS: 1 DIF: Cognitive Level: Analysis OBJ: Nursing Process: Assessment MSC: Client Needs: Psychosocial Integrity 4. Families progress through various stages of reactions when a child is diagnosed with a chronic illness or disability. After the shock phase, a period of adjustment usually follows that may be characterized by what reaction? a. Anger b. Overprotectiveness c. Social reintegration d. Guilt ANS: B For most families, the adjustment phase is accompanied by several responses that are normally part of the adjustment process. Overprotectiveness, rejection, denial, or gradual acceptance are common reactions.NTUhReSIiNniGtiTaBl .dCiOagMnosis of a chronic illness or disability often is often met with intense emotion and characterized by guilt and anger. Social reintegration is the culmination of the adjustment process. PTS: 1 DIF: Cognitive Level: Comprehension OBJ: Nursing Process: Planning MSC: Client Needs: Psychosocial Integrity 5. The nurse comes into the room of a child who was just diagnosed with a chronic disability. The child's parents begin to yell at the nurse about a variety of concerns. What is the nurse's best response? a. “What is really wrong?” b. “Being angry is only natural.” c. “Yelling at me will not change things.” d. “I will come back when you settle down.” ANS: B Parental anger after the diagnosis of a child with a chronic disability is a common response. One of the most common targets for parental anger is members of the staff. The nurse should recognize the common response of anger to the diagnosis and allow the family to express their feelings and emotions. “What is really wrong?” “Yelling at me will not change things,” and “I will come back when you settle down” are all possible responses, but they are not addressing the parent's need to express their anger effectively. PTS: 1 DIF: Cognitive Level: Analysis OBJ: Nursing Process: Assessment MSC: Client Needs: Psychosocial Integrity 6. A common parental reaction to a child with special needs is parental overprotection. Parental behavior suggestive of this includes which behavior? a. Attempting to avoid frustrating situations. b. Providing consistent, strict discipline. c. Forcing child to help self, even when not capable. d. Encouraging social and educational activities not appropriate to child's level of capability. ANS: A Parental overprotection is manifested by the parents' fear of letting the child achieve any new skill, avoiding all discipline, and catering to the child's every desire to prevent frustration. The overprotective parents usually do not set limits and or institute discipline, and they usually prefer to remain in the role of total caregiver. They do not allow the child to perform self-care or encourage the child to try new activities. PTS: 1 DIF: Cognitive Level: Analysis OBJ: Nursing Process: Assessment MSC: Client Needs: Psychosocial Integrity 7. Most parents of children with special needs tend to experience chronic sorrow. How may chronic sorrow be characterized? a. Lack of acceptance of the child's limitation. b. Lack of available support to prevent sorrow. c. Periods of intensified sorrow when experiencing anger and guilt. d. Periods of intensified sorrow and loss that occur in waves over time. ANS: D NURSINGTB.COM Chronic sorrow is manifested by feelings of sorrow and loss that recur in waves over time. The sorrow is in response to the recognition of the child's limitations. The family should be assessed in an ongoing manner to provide appropriate support as the needs of the family change. The sorrow is not preventable. The chronic sorrow occurs during the reintegration and acknowledgment stage. PTS: 1 DIF: Cognitive Level: Analysis OBJ: Nursing Process: Diagnosis MSC: Client Needs: Psychosocial Integrity 8. Which intervention will encourage a sense of autonomy in a toddler with disabilities? a. Avoiding separation from family during hospitalization b. Encouraging age appropriate independence in as many areas as possible c. Exposing child to pleasurable experiences as much as possible d. Helping parents learn special care needs of their child ANS: B Encouraging the toddler to be independent encourages a sense of autonomy. The child can be given choices about feeding, dressing, and diversional activities, which will provide a sense of control. Avoiding separation from family during hospitalization and helping parents learn special care needs of their child should be practiced as part of family-centered care. They do not particularly foster autonomy. Exposing the child to pleasurable experiences, especially sensory ones, is a supportive intervention. It does not particularly support autonomy. PTS: 1 DIF: Cognitive Level: Application OBJ: Nursing Process: Implementation MSC: Client Needs: Psychosocial Integrity 9. The feeling of guilt that the child “caused” the disability or illness is especially critical in which child? a. Toddler b. Preschooler c. School-age child d. Adolescent ANS: B Preschoolers are most likely to be affected by feelings of guilt that they caused the illness/disability or are being punished for wrongdoings. Toddlers are focused on establishing their autonomy. The illness will foster dependency. The school-age child will have limited opportunities for achievement and may not be able to understand limitations. Adolescents are faced with the task of incorporating their disabilities into their changing self-concept. PTS: 1 DIF: Cognitive Level: Comprehension OBJ: Nursing Process: Assessment MSC: Client Needs: Psychosocial Integrity 10. The father, of a 9 year old diagnosed with several physical disabilities, explains to the nurse that his child concentrates on what he/she can do rather than cannot do and is as independent as possible. How should the nurse's best interpret this statement? a. The father is experiencing denial. b. The father is expressing his own views. c. The child is using an adaptive coping style. d. The child is using a maladaptive coping style. ANS: C NURSINGTB.COM The father is describing a well-adapted child who has learned to accept physical limitations. These children function well at home, at school, and with peers. They have an understanding of their disorder that allows them to accept their limitations, assume responsibility for care, and assist in treatment and rehabilitation. The father is not denying the child's limitations or expressing his own views. This is descriptive of an adaptive coping style. PTS: 1 DIF: Cognitive Level: Analysis OBJ: Nursing Process: Assessment MSC: Client Needs: Psychosocial Integrity 11. The nurse, talking with the tearful parent of a child newly diagnosed with a chronic illness, asks, “Who do you talk with when something is worrying you?” What is the purpose of this statement? a. Inappropriate, because parent is so upset. b. A diversion of the present crisis to similar situations with which parent has dealt. c. An intervention to find someone to help parent. d. Part of assessing parent's available support system. ANS: D This question will provide information about the marital relationship (does the parent speak to the spouse?), alternate support systems, and ability to communicate. These are very important data for the nurse to obtain and an appropriate part of an accurate assessment. By assessing these areas, the nurse can facilitate the identification and use of community resources as needed. The nurse is obtaining information to help support the parent through the diagnosis. The parent is not in need of additional parenting help at this time. PTS: 1 DIF: Cognitive Level: Analysis OBJ: Nursing Process: Assessment MSC: Client Needs: Psychosocial Integrity 12. The nurse, providing support to parents of a child newly diagnosed with a chronic disability, notices that they keep asking the same questions. How should the nurse respond to best meet their needs? a. Patiently continue to answer questions. b. Kindly refer them to someone else to answer their questions. c. Recognize that some parents cannot understand explanations. d. Suggest that they ask their questions when they are not upset. ANS: A Diagnosis is one of the anticipated stress points for parents. The parents may not hear or remember all that is said to them. The nurse should continue to provide the kind of information that they desire. This is a particularly stressful time for the parents; the nurse can play a key role in providing necessary information. Parents should be provided with oral and written information. The nurse needs to work with the family to ensure understanding of the information. The parents require information at the time of diagnosis. Other questions will arise as they adjust to the information. NURSINGTB.COM PTS: 1 DIF: Cognitive Level: Application OBJ: Nursing Process: Implementation MSC: Client Needs: Psychosocial Integrity 13. The parents of a child born with disabilities ask the nurse for advice about discipline. The nurse's response should be based on what knowledge concerning discipline? a. Appropriate disciple is essential for the child. b. It may be too difficult to implement appropriate discipline for a special-needs child. c. Discipline is not needed unless the child becomes problematic. d. Discipline is best achieved with punishment for misbehavior. ANS: A Discipline is essential for the children with disabilities. It provides boundaries within which to test their behavior and teaches them socially acceptable behaviors. It is not too difficult to implement discipline with a special-needs child. The nurse should teach the parents ways to manage the child's behavior before it becomes problematic. Punishment is not effective in managing behavior. PTS: 1 DIF: Cognitive Level: Analysis OBJ: Nursing Process: Implementation MSC: Client Needs: Psychosocial Integrity 14. An 8 year old will soon be able to return to school after an injury that resulted in several severe, chronic disabilities. What is the most appropriate action by the school nurse to help assure a smooth transition back to school? a. Recommending that the child's parents attend school at first to prevent teasing b. Preparing the child's classmates and teachers for changes they can expect c. Referring the child to a school where the children have chronic disabilities similar to hers d. Discussing with both the child and the parents the fact that classmates will not likely be as accepting as before ANS: B Attendance at school is an important part of normalization for the child. The school nurse should prepare teachers and classmates about her condition, abilities, and special needs. A visit by the parents can be helpful, but unless the classmates are prepared for the changes, it alone will not prevent teasing. The child's school experience should be normalized as much as possible. Children need the opportunity to interact with healthy peers and engage in activities with groups or clubs composed of similarly affected persons. Children with special needs are encouraged to maintain and reestablish relationships with peers and participate according to their capabilities. PTS: 1 DIF: Cognitive Level: Application OBJ: Nursing Process: Implementation MSC: Client Needs: Psychosocial Integrity 15. A 16 year old diagnosed with a chronic illness has recently become rebellious and is taking risks such as missing doses of his medication. What information should the nurse provide the parents to help explain their child's behavior? a. The child at this age requires more discipline. b. At this age, children need more socialization with peers. c. This behavior is seen as a normal part of adolescence. d. This is how the child is askingNfUoRr SmINoGreTpBa.CreOnMtal involvement in managing stress. ANS: C Risk taking, rebelliousness, and lack of cooperation are normal parts of adolescence. If the parents increase the amount of discipline, he will most likely be more rebellious. Socialization with peers should be encouraged as a part of adolescence. It is a normal part of adolescence during which the young adult is establishing independence. PTS: 1 DIF: Cognitive Level: Application OBJ: Nursing Process: Implementation MSC: Client Needs: Psychosocial Integrity 16. At what age do most children have an adult concept of death as being inevitable, universal, and irreversible? a. 4 to 5 years b. 6 to 8 years c. 9 to 11 years d. 12 to 16 years ANS: C By age 9 to 11 years, children have an adult concept of death. They realize that it is inevitable, universal, and irreversible. Preschoolers and young school-age children are too young to have an adult concept of death. Adolescents have a mature understanding of death. PTS: 1 DIF: Cognitive Level: Comprehension OBJ: Nursing Process: Assessment MSC: Client Needs: Psychosocial Integrity 17. Which is the most descriptive of a school-age child's reaction to death? a. Is very interested in funerals and burials b. Has little understanding of words such as forever c. Imagines the deceased person to be still alive d. Has an idealistic view of the world and criticizes funerals as barbaric ANS: A The school-age child is very interested in postdeath services and may be inquisitive about what happens to the body. School-age children have an established concept of forever and have a deeper understanding of death in a concrete manner. Toddler may imagine the deceased person to be still alive. Adolescents may respond to death with an idealistic view of the world and criticize funerals as barbaric. PTS: 1 DIF: Cognitive Level: Analysis OBJ: Nursing Process: Assessment MSC: Client Needs: Psychosocial Integrity 18. At what developmental period do children have the most difficulty coping with death, particularly if it is their own? a. Toddlerhood b. Preschool c. School-age d. Adolescence ANS: D Because of their mature understanding of death, remnants of guilt and shame, and issues with deviations from normal, adolescents have the most difficulty coping with death. Toddlers and preschoolers are too young to have difficulty coping with their own death. They will fear separation from parents. School-aNgeURchSiIlNdGreTnBw.CiOllMfear the unknown, such as the consequences of the illness and the threat to their sense of security. PTS: 1 DIF: Cognitive Level: Comprehension OBJ: Nursing Process: Assessment MSC: Client Needs: Psychosocial Integrity 19. A school-age child is diagnosed with a life-threatening illness. The parents want to protect their child from knowing the seriousness of the illness. What information should the nurse provide to these parents? a. This will help the child cope effectively by denial. b. This attitude is helpful to give parents time to cope. c. Terminally ill children know when they are seriously ill. d. Terminally ill children usually choose not to discuss the seriousness of their illness. ANS: C The child needs honest and accurate information about the illness, treatments, and prognosis. Children, even at a young age, realize that something is seriously wrong and that it involves them. The nurse should help parents understand the importance of honesty. The child will know that something is wrong because of the increased attention of health professionals. This would interfere with denial as a form of coping. Parents may need professional support and guidance from a nurse or social worker in this process. Children will usually tell others how much information they want about their condition. PTS: 1 DIF: Cognitive Level: Analysis OBJ: Nursing Process: Implementation MSC: Client Needs: Psychosocial Integrity 20. The parents of a child who has just died ask to be left alone so that they can rock their child one more time. In response to their request, what intervention should the nurse implement? a. Grant their request. b. Assess why they feel that this is necessary. c. Discourage this because it will only prolong their grief. d. Kindly explain that they need to say good-bye to their child now and leave. ANS: A The parents should be allowed to remain with their child after the death. The nurse can remove all of the tubes and equipment and offer the parents the option of preparing the body. This is an important part of the grieving process and should be allowed if the parents desire it. It is important for the nurse to ascertain if the family has any special needs. None of the other options adequately meet the parent's need to grieve. PTS: 1 DIF: Cognitive Level: Application OBJ: Nursing Process: Implementation MSC: Client Needs: Psychosocial Integrity 21. The nurse is talking with the parents of a child who died 6 months ago. They sometimes still “hear” the child's voice and have trouble sleeping. They describe feeling “empty” and depressed. The nurse should recognize that: a. these are normal grief responses. b. the pain of the loss is usually less by this time. c. these grief responses are more typical of the early stages of grief. d. this grieving is essential until the pain is gone and the child is gradually forgotten. NURSINGTB.COM ANS: A These are normal grief responses. The process of grief work is lengthy and resolution of grief may take years, with intensification during the early years. The child will never be forgotten by the parents. PTS: 1 DIF: Cognitive Level: Comprehension OBJ: Nursing Process: Assessment MSC: Client Needs: Psychosocial Integrity 22. At the time of a child's death, the nurse tells his mother, “We will miss him so much.” What does this statement indicate about the nurse? a. Pretending to be experiencing grief. b. Expressing personal feelings of loss. c. Denying the mother's sense of loss. d. Talking when listening would be better. ANS: B The death of a patient is one of the most stressful aspects of a critical care or oncology nurse. Nurses experience reactions similar to those of family members because of their involvement with the child and family during the illness. Nurses often have feelings of personal loss when a patient dies. The nurse is experiencing a normal grief response to the death of a patient. There is no implication that the mother's loss is minimized. The nurse is validating the worth of the child. PTS: 1 DIF: Cognitive Level: Analysis OBJ: Nursing Process: Implementation MSC: Client Needs: Psychosocial Integrity 23. Which is the most appropriate response to a school-age child who asks if she can talk to her dying sister? a. “You need to speak loudly so she can hear you.” b. “Holding her hand would be better because at this point she can't hear you.” c. “Although she can't hear you, she can feel your presence so sit close to her.” d. “Even though she will probably not answer you, she can still hear what you say to her.” ANS: D Hearing is the last sense to cease before death. Talking to the dying child is important both for the child and for the family. There is no evidence that the dying process decreases hearing acuity; therefore, the sister should speak at a normal volume. The sibling should be encouraged to speak to the child, as well as sit close to the bed and hold the child's hand. PTS: 1 DIF: Cognitive Level: Application OBJ: Nursing Process: Implementation MSC: Client Needs: Psychosocial Integrity 24. Which represents a common best practice in the provision of services to children with chronic or complex conditions? a. Care is focused on the child's chronologic age. b. Children with complex conditions are integrated into regular classrooms. c. Disabled children are less likely to be cared for by their families. d. Children with complex conditions are placed in residential treatment facilities. ANS: B Normalization refers to behaviorsNaUndRSiInNteGrTvBen.CtiOoMns for people with disabilities to integrate into society by living life as people without a disability would. For children, normalization includes attending school and being integrated into regular classrooms. This affords the child the advantages of learning with a wide group of peers. Care is necessarily focused on the child's developmental age. Home care by the family is considered best practice. The nurse can assist families by assessing social support systems, coping strategies, family cohesiveness, and family and community resources. PTS: 1 DIF: Cognitive Level: Comprehension OBJ: Nursing Process: Assessment MSC: Client Needs: Safe and Effective Care Environment: Management of Care 25. Which is the most appropriate nursing intervention to promote normalization in a school-age child with a chronic illness? a. Give child as much control as possible. b. Ask child's peer to make child feel normal. c. Convince child that nothing is wrong with him or her. d. Explain to parents that family rules for the child do not need to be the same as for healthy siblings. ANS: A The school-age child who is ill may be forced into a period of dependency. To foster normalcy, the child should be given as much control as possible. It is unrealistic to expect one individual to make the child feel normal. The child has a chronic illness. It would be unacceptable to convince the child that nothing is wrong. The family rules should be similar for each of the children in a family. Resentment and hostility can arise if different standards are applied to each child. PTS: 1 DIF: Cognitive Level: Application OBJ: Nursing Process: Implementation MSC: Client Needs: Psychosocial Integrity 26. Which term best describes a multidisciplinary approach to the management of a terminal illness that focuses on symptom control and support? a. Dying care b. Curative care c. Restorative care d. Palliative care ANS: D This is one of the definitions of palliative care. The goal of palliative care is the achievement of the highest possible quality of life for patients and their families. Curative care would infer providing a cure for the disease or disorder while restorative care involves measures to regain past abilities. Dying care generally refers to the care of an individual in the final stage of life. PTS: 1 DIF: Cognitive Level: Comprehension OBJ: Nursing Process: Assessment MSC: Client Needs: Psychosocial Integrity 27. Which best describes how preschoolers react to the death of a loved one? a. The preschooler is too young tNoUhRaSvIeNGa TcBon.CcOeMpt of death. b. A preschooler is likely to feel guilty and responsible for the death. c. Grief is acute but does not last long at this age. d. Grief is usually expressed in the same way in which the adults in the preschooler's life are expressing grief. ANS: B Because of egocentricity, the preschooler may feel guilty and responsible for the death. Preschoolers usually have some understanding of the meaning of death. Death is seen as a departure or some kind of sleep and they have no understanding of the permanence of death. None of the other statements accurately describe the usually preschoolers reaction to death. PTS: 1 DIF: Cognitive Level: Comprehension OBJ: Nursing Process: Assessment MSC: Client Needs: Psychosocial Integrity 28. A cure is no longer possible for a young child with cancer. The nursing staff recognizes that the goal of treatment must shift from cure to palliation. Which is an important consideration at this time? a. The family is included in the decision to shift the goals of treatment. b. The decision must be made by the health professionals involved in the child's care. c. The family needs to understand that palliative care takes place in the home. d. The decision should not be communicated to the family because it will encourage a sense of hopelessness. ANS: A When the child reaches the terminal stage, the nurse and physician should explore the family's wishes. The family should help decide what interventions will occur as they plan for their child's death. None of the other options address the parent's need to be involved effectively in their child's care. PTS: 1 DIF: Cognitive Level: Application OBJ: Nursing Process: Implementation MSC: Client Needs: Psychosocial Integrity 29. Which statement made by the nurse would indicate a correct understanding of palliative care? a. “Palliative care serves to hasten death and make the process easier for the family.” b. “Palliative care provides pain and symptom management for the child.” c. “The goal of palliative care is to place the child in a hospice setting at the end of life.” d. “The goal of palliative care is to act as the liaison between the family, child, and other health care professionals.” ANS: B The primary goal of palliative care is to provide pain and symptom management, not to hasten death or place the child in a hospice setting. Palliative care is provided by a multidisciplinary team whose goal it is to provide active total care for patients whose disease is no longer responding to curative treatment. PTS: 1 DIF: Cognitive Level: Analysis OBJ: Nursing Process: Planning MSC: Client Needs: Safe and Effective Care Environment: Management of Care 30. A nurse is planning palliative care for a child with severe pain. Which should the nurse expect to be prescribed for pain relief? a. Opioids as needed b. Opioids on a regular schedule NURSINGTB.COM c. Distraction and relaxation techniques d. Nonsteroidal antiinflammatory drugs ANS: B Pain medications for children in palliative care should be given on a regular schedule, and extra doses for breakthrough pain should be available to maintain comfort. Opioid drugs such as morphine should be given for severe pain, and the dose should be increased as necessary to maintain optimal pain relief. Techniques such as distraction, relaxation techniques, and guided imagery should be combined with drug therapy to provide the child and family strategies to control pain. Nonsteroidal antiinflammatory drugs are not sufficient to manage severe pain for children in palliative care. PTS: 1 DIF: Cognitive Level: Application OBJ: Nursing Process: Planning MSC: Client Needs: Physiologic Integrity: Basic Care and Comfort MULTIPLE RESPONSE 1. What should the nurse identify as major fears in the school-age child who is hospitalized with a chronic illness? (Select all that apply.) a. Altered body image b. Separation from peer group c. Bodily injury d. Mutilation e. Being left alone ANS: C, D, E Bodily injury, mutilation, and being left alone are all major fears of the school age. Altered body image and separation from peers are major fears in the adolescent. PTS: 1 DIF: Cognitive Level: Application OBJ: Nursing Process: Assessment MSC: Client Needs: Health Promotion and Maintenance 2. Which describe avoidance behaviors a parent may exhibit when learning that his or her child has a chronic condition? (Select all that apply.) a. Refuses to agree to treatment b. Shares burden of disorder with others c. Verbalizes possible loss of child d. Withdraws from outside world e. Punishes self because of guilt and shame ANS: A, D, E A parent who refuses to agree to treatment, withdraws from the outside world, and punishes self because of guilt and shame is exhibiting avoidance coping behaviors. A parent who shares the burden of disorder with others and verbalizes possible loss of child is exhibiting approach coping behaviors. PTS: 1 DIF: Cognitive Level: Analysis OBJ: Nursing Process: Evaluation MSC: Client Needs: Psychosocial Integrity 3. Which are appropriate statementsNthUeRnSuINrsGeTsBh.oCuOlMd make to parents after the death of their child? (Select all that apply.) a. “We feel so sorry that we couldn't save your child.” b. “Your child isn't suffering anymore.” c. “I know how you feel.” d. “You're feeling all the pain of losing a child.” e. “You are still young enough to have another baby.” ANS: A, D By saying, “We feel so sorry that we couldn't save your child,” the nurse is expressing personal feeling of loss or frustration, which is therapeutic. Stating, “You're feeling all the pain of losing a child,” focuses on a feeling, which is therapeutic. The statement, “Your child isn't suffering anymore,” is a judgmental statement, which is nontherapeutic. “I know how you feel” and “You're still young enough to have another baby” are statements that give artificial consolation and are nontherapeutic. PTS: 1 DIF: Cognitive Level: Application OBJ: Nursing Process: Communication, Documentation MSC: Client Needs: Psychosocial Integrity 4. A nurse is caring for a child who is near death. Which physical signs indicate the child is approaching death? (Select all that apply.) a. Body feels warm b. Tactile sensation decreasing c. Speech becomes rapid d. Change in respiratory pattern e. Difficulty swallowing ANS: B, D, E Physical signs of approaching death include tactile sensation beginning to decrease, a change in respiratory pattern, and difficulty swallowing. Even though there is a sensation of heat, the body feels cool, not warm, and speech becomes slurred, not rapid. PTS: 1 DIF: Cognitive Level: Comprehension OBJ: Nursing Process: Assessment MSC: Client Needs: Physiologic Integrity: Basic Care and Comfort NURSINGTB.COM Chapter 37: Impact of Cognitive or Sensory Impairment on the Child and Family Perry: Maternal Child Nursing Care, 6th Edition MULTIPLE CHOICE 1. When should a child diagnosed with cognitive impairment be referred for stimulation and educational programs? a. As young as possible. b. As soon as they have the ability to communicate in some way. c. At age 3 years, when schools are required to provide services. d. At age 5 or 6 years, when schools are required to provide services. ANS: A The child's education should begin as soon as possible. Considerable evidence exists that early-intervention programs for children with disabilities are valuable for cognitively impaired children. The early intervention may facilitate the child's development of communication skills. States are encouraged to provide early-intervention programs from birth under Public Law 101-476, the Individuals with Disabilities Act. PTS: 1 DIF: Cognitive Level: Comprehension OBJ: Nursing Process: Implementation MSC: Client Needs: Psychosocial Integrity 2. What is the major consideration when selecting toys for a child who is cognitively impaired? a. Safety b. Age appropriateness c. Ability to provide exercise d. Ability to teach useful skills ANS: A NURSINGTB.COM Safety is the primary concern in selecting recreational and exercise activities for all children. This is especially true for children who are cognitively impaired. Age appropriateness, the ability to provide exercise, and the ability to teach useful skills are all factors to consider in the selection of toys, but safety is of paramount importance. PTS: 1 DIF: Cognitive Level: Analysis OBJ: Nursing Process: Implementation MSC: Client Needs: Safe and Effective Care Environment 3. Which intervention is focused on facilitating socialization of the cognitively impaired child? a. Provide age-appropriate toys and play activities. b. Provide peer experiences such as Special Olympics when older. c. Avoid exposure to strangers who may not understand cognitive development. d. Emphasize mastery of physical skills because they are delayed more often than verbal skills. ANS: B The acquisition of social skills is a complex task. Children of all ages need peer relationships. Parents should enroll the child in preschool. When older, the child should have peer experiences similar to other children, such as group outings, Boy or Girl Scouts, and Special Olympics. Providing age-appropriate toys and play activities is important, but peer interactions will facilitate social development. Parents should expose the child to strangers so the child can practice social skills. Verbal skills are delayed more than physical skills. PTS: 1 DIF: Cognitive Level: Analysis OBJ: Nursing Process: Implementation MSC: Client Needs: Psychosocial Integrity 4. Which is the most common congenital anomaly associated with Down syndrome? a. Hypospadias b. Pyloric stenosis c. Septal defects d. Congenital hip dysplasia ANS: C Congenital heart malformations, primarily septal defects, are very common congenital anomalies in Down syndrome. Hypospadias, pyloric stenosis, and congenital hip dysplasia are not frequent congenital anomalies associated with Down syndrome. PTS: 1 DIF: Cognitive Level: Comprehension OBJ: Nursing Process: Assessment MSC: Client Needs: Physiologic Integrity 5. A 9 year old diagnosed with Down syndrome is mainstreamed into a regular third-grade class for part of the school day. His mother asks the school nurse about programs such as Cub Scouts that he might join. The nurse's recommendation should be based on what knowledge? a. Programs such as Cub Scouts NarUeRiSnIaNpGpTroBp.CriOaMte for children who are cognitively impaired. b. Children with Down syndrome have the same need for socialization as other children. c. Children with Down syndrome socialize better with children who have similar disabilities. d. Parents of children with Down syndrome encourage programs such as scouting because they deny that their children have disabilities. ANS: B Children of all ages need peer relationships. Children with Down syndrome should have peer experiences similar to those of other children, such as group outings, Cub Scouts, and Special Olympics, which can all help children with cognitive impairment to develop socialization skills. Although all children should have an opportunity to form a close relationship with someone of the same developmental level, it is appropriate for children with disabilities to develop relationships with children who do not have disabilities. The parents are acting as advocates for their child. PTS: 1 DIF: Cognitive Level: Comprehension OBJ: Nursing Process: Implementation MSC: Client Needs: Psychosocial Integrity 6. A newborn assessment shows separated sagittal suture, oblique palpebral fissures, depressed nasal bridge, protruding tongue, and transverse palmar creases. These findings are most suggestive of: a. microcephaly. b. Down syndrome. c. cerebral palsy. d. fragile X syndrome. ANS: B These are characteristics associated with Down syndrome. The infant with microcephaly has a small head. Cerebral palsy is a diagnosis not usually made at birth. No characteristic physical signs are present. The infant with fragile X syndrome has increased head circumference; long, wide, and/or protruding ears; long, narrow face with prominent jaw; hypotonia; and high, arched palate. PTS: 1 DIF: Cognitive Level: Comprehension OBJ: Nursing Process: Assessment MSC: Client Needs: Physiologic Integrity 7. The child diagnosed with Down syndrome should be evaluated for which characteristic before participating in some sports? a. Hyperflexibility b. Cutis marmorata c. Atlantoaxial instability d. Speckling of iris (Brushfield's spots) ANS: C Children with Down syndrome are at risk for atlantoaxial instability. Atlantoaxial instability (AAI) is characterized by excessive movement at the junction between the atlas (C1) and axis (C2) as a result of either a bony or ligamentous abnormality. Before participating in sports that put stress on the head and neck, a radiologic examination should be done. Although hyperflexibility, cutis marmorata,NaUndRSBINruGsThBfi.eClOdM's spots are characteristics of Down syndrome, they do not affect the child's ability to participate in sports. PTS: 1 DIF: Cognitive Level: Comprehension OBJ: Nursing Process: Assessment MSC: Client Needs: Physiologic Integrity 8. Which action is contraindicated when a child diagnosed with Down syndrome is hospitalized? a. Determine the child's vocabulary for specific body functions. b. Assess the child's hearing and visual capabilities. c. Encourage parents to leave the child alone at night. d. Have meals served at the child's usual mealtimes. ANS: C The child with Down syndrome needs routine schedules and consistency. Having familiar people present, especially parents, helps to decrease the child's anxiety. To communicate effectively with the child, it is important to know the child's particular vocabulary for specific body functions. Children with Down syndrome have a high incidence of hearing loss and vision problems and should have hearing and vision assessed whenever they are in a health care facility. Meals should be served at the usual mealtimes because routine schedules and consistency are important to children with Down syndrome. PTS: 1 DIF: Cognitive Level: Analysis OBJ: Nursing Process: Planning MSC: Client Needs: Psychosocial Integrity 9. Which statement accurately describes fragile X syndrome? a. It is a chromosome defect affecting only females. b. It is a chromosome defect that follows the pattern of X-linked recessive disorders. c. It is the second most common genetic cause of cognitive impairment. d. It is the most common cause of noninherited cognitive impairment. ANS: C Fragile X syndrome is the most common inherited cause of cognitive impairment and the second most common genetic cause of cognitive impairment after Down syndrome. Fragile X primarily affects males and follows the pattern of X-linked dominant disorders with reduced penetrance. PTS: 1 DIF: Cognitive Level: Comprehension OBJ: Nursing Process: Assessment MSC: Client Needs: Physiologic Integrity 10. Distortion of sound and problems in discrimination are characteristic of which type of hearing loss? a. Conductive b. Sensorineural c. Mixed conductive-sensorineural d. Central auditory imperceptive ANS: B Sensorineural hearing loss, also known as perceptive or nerve deafness, involves damage to the inner ear structures or the auditory nerve. It results in distortion of sounds and problems in discrimination. Conductive hearing loss involves mainly interference with loudness of sound. Mixed conductive-sensorineural hearing loss manifests as a combination of both sensorineural and conductive loss. The central auditory imperceptive category includes all hearing losses that do not demonstrate defects inNthUeRScIoNnGdTuBct.iCoOnMor sensory structures. PTS: 1 DIF: Cognitive Level: Comprehension OBJ: Nursing Process: Assessment MSC: Client Needs: Physiologic Integrity 11. What term is used to identify the most common type of hearing loss, which results from interference of transmission of sound to the middle ear? a. Conductive b. Sensorineural c. Mixed conductive-sensorineural d. Central auditory imperceptive ANS: A Conductive or middle ear hearing loss is the most common type. It results from interference of transmission of sound to the middle ear, most often from recurrent otitis media. Sensorineural, mixed conductive-sensorineural, and central auditory imperceptive are less common types of hearing loss. PTS: 1 DIF: Cognitive Level: Comprehension OBJ: Nursing Process: Assessment MSC: Client Needs: Physiologic Integrity 12. A parent asks the nurse why a developmental assessment is being conducted for a child during a routine well-child visit. The nurse answers based on what knowledge about such routine developmental assessments? a. Not necessary unless the parents request them. b. The best method for early detection of cognitive disorders. c. Frightening to parents and children and should be avoided. d. Valuable in measuring intelligence in children. ANS: B Early detection of cognitive disorders can be facilitated through assessment of development at each well-child examination. Developmental assessment is a component of all well-child examinations; however, they are not intended to measure intelligence. Developmental assessments are not frightening when the parent and child are educated about the purpose of the assessment. PTS: 1 DIF: Cognitive Level: Application OBJ: Nursing Process: Implementation MSC: Client Needs: Health Promotion and Maintenance 13. The nurse is talking with a 10 year old who wears bilateral hearing aids. The left hearing aid is making an annoying whistling sound that the child cannot hear. What is the most appropriate nursing action to address this issue? a. Ignore the sound. b. Ask the child to reverse the hearing aids. c. Suggest that the child reinsert the hearing aid. d. Suggest that the child raise the volume of the hearing aid. ANS: C The whistling sound is acoustic feedback. The nurse should have the child remove the hearing aid and reinsert it, making sure that no hair is caught between the ear mold and the ear canal. Ignoring the sound and suggesting that he raise the volume of the hearing aid would be annoying to others. The hearing aids are molded specifically for each ear. NURSINGTB.COM PTS: 1 DIF: Cognitive Level: Application OBJ: Nursing Process: Implementation MSC: Client Needs: Physiologic Integrity 14. An implanted ear prosthesis for children with sensorineural hearing loss is a(n): a. hearing aid. b. cochlear implant. c. auditory implant. d. amplification device. ANS: B Cochlear implants are surgically implanted, and they provide a sensation of hearing for individuals who have severe or profound hearing loss of sensorineural origin. Hearing aids are external devices for enhancing hearing. An auditory implant does not exist. An amplification device is an external device for enhancing hearing. PTS: 1 DIF: Cognitive Level: Comprehension OBJ: Nursing Process: Assessment MSC: Client Needs: Physiologic Integrity 15. Which action best facilitates lipreading by the hearing-impaired child? a. Speaking at an even rate b. Exaggerating pronunciation of words c. Avoiding using facial expressions d. Repeating in exactly the same way if child does not understand ANS: A The child should be helped to learn and understand how to read lips by speaking at an even rate. Exaggerating word pronunciation, avoiding facial expressions, and repeating words are characteristics of communication that would interfere with the child's comprehension of the spoken word. PTS: 1 DIF: Cognitive Level: Application OBJ: Nursing Process: Implementation MSC: Client Needs: Physiologic Integrity 16. Prevention of hearing impairment in children is a major goal for the nurse. How can this be best achieved? a. Being involved in immunization clinics for children. b. Assessing a newborn for hearing loss. c. Answering parents' questions about hearing aids. d. Participating in hearing screening in the community. ANS: A Childhood immunizations can eliminate the possibility of acquired sensorineural hearing loss from rubella, mumps, or measles encephalitis. Assessing a newborn for hearing loss, answering parents' questions about hearing aids, and participating in community hearing screenings are screening interventions to identify the presence of hearing loss, not prevention. PTS: 1 DIF: Cognitive Level: Comprehension OBJ: Nursing Process: Implementation MSC: Client Needs: Health Promotion and Maintenance 17. Which term refers to opacity of the crystalline lens that prevents light rays from entering the eye and refracting on the retina? a. Myopia b. Amblyopia c. Cataract d. Glaucoma ANS: C NURSINGTB.COM A cataract refers to opacity of the crystalline lens that prevents light rays from entering the eye and refracting on the retina. Myopia, or nearsightedness, refers to the ability to see objects clearly at close range but not at a distance. Amblyopia, or lazy eye, is reduced visual acuity in one eye. Glaucoma is a group of eye diseases characterized by increased intraocular pressure. PTS: 1 DIF: Cognitive Level: Comprehension OBJ: Nursing Process: Diagnosis MSC: Client Needs: Health Promotion and Maintenance 18. A nurse would suspect possible visual impairment in a child who displays: a. excessive rubbing of the eyes. b. rapid lateral movement of the eyes. c. delay in speech development. d. lack of interest in casual conversation with peers. ANS: A Excessive rubbing of the eyes is a clinical manifestation of visual impairment. Rapid lateral movement of the eyes, delay in speech development, and lack of interest in casual conversation with peers are not associated with visual impairment. PTS: 1 DIF: Cognitive Level: Comprehension OBJ: Nursing Process: Assessment MSC: Client Needs: Physiologic Integrity 19. The school nurse is caring. What emergency treatment is appropriate for a child with a penetrating eye injury? a. Applying a regular eye patch. b. Applying a Fox shield to the affected eye and any type of patch to the other eye. c. Applying ice until the physician is seen. d. Irrigating the eye copiously with a sterile saline solution. ANS: B The nurse's role in a penetrating eye injury is to prevent further injury to the eye. A Fox shield (if available) should be applied to the injured eye, and a regular eye patch to the other eye to prevent bilateral movement. Applying a regular eye patch or ice until the physician is seen, or irrigating the eye with a copious amount of sterile saline, may cause more damage to the eye. PTS: 1 DIF: Cognitive Level: Application OBJ: Nursing Process: Implementation MSC: Client Needs: Physiologic Integrity 20. A father calls the emergency department nurse saying that his child's eyes burn after getting some dishwasher detergent in them. What should the nurse recommend before the child is transported to the emergency department? a. Keep the eyes closed. b. Apply cold compresses. c. Irrigate eyes copiously with tap water for 20 minutes. d. Prepare a normal saline solution (salt and water) and irrigate eyes for 20 minutes. ANS: C NURSINGTB.COM The first action is to flush the eyes with clean tap water. This will rinse the detergent from the eyes. Keeping the eyes closed and applying cold compresses may allow the detergent to do further harm to the eyes during transport. Normal saline is not necessary. The delay during preparation can allow the detergent to cause continued injury to the eyes. PTS: 1 DIF: Cognitive Level: Application OBJ: Nursing Process: Implementation MSC: Client Needs: Physiologic Integrity 21. What intervention should the nurse implement when noting gross bleeding in a child's eye after being hit in the eye? a. Apply a Fox shield. b. Instruct the adolescent to apply ice for 24 hours. c. Have adolescent rest with eye closed and heat applied. d. Notify parents that adolescent needs to see an ophthalmologist. ANS: D The parents should be notified that the adolescent must see an ophthalmologist as soon as possible. Applying a Fox shield, instructing the adolescent to apply ice for 24 hours, and having the adolescent rest with the eye closed and heat applied may cause further damage. PTS: 1 DIF: Cognitive Level: Application OBJ: Nursing Process: Implementation MSC: Client Needs: Physiologic Integrity 22. A parent whose child has been diagnosed with a cognitive deficit should be counseled about what fact related to intellectual impairment? a. Is usually due to a genetic defect. b. Is likely caused by a variety of factors. c. Is rarely due to first-trimester events. d. Is usually caused by parental intellectual impairment. ANS: B There are a multitude of causes for intellectual impairment. In most cases, a specific cause has not been identified. Only a small percentage of children with intellectual impairment are affected by a genetic defect. One third of children with intellectual impairment are affected by first-trimester events. Intellectual impairment can be transmitted to a child only if the parent has a genetic disorder. PTS: 1 DIF: Cognitive Level: Comprehension OBJ: Nursing Process: Planning MSC: Client Needs: Health Promotion and Maintenance 23. A nurse is providing a parent information regarding autism spectrum disorder (ASD). Which statement made by the parent indicates understanding of the teaching? a. “Autism is characterized by periods of remission and exacerbation.” b. “The onset of autism usually occurs before toddler stage.” c. “Children with autism have imitation and gesturing skills.” d. “Autism can be treated effectively with medication.” ANS: B The onset of ASD is now frequently diagnosed in toddlers because of their atypical development is being recognized early. Autism does not have periods of remission and exacerbation. Autistic children lacNkUiRmSiItNatGivTeB.sCkOilMls. Medications are of limited use in children with autism. PTS: 1 DIF: Cognitive Level: Analysis OBJ: Nursing Process: Evaluation MSC: Client Needs: Health Promotion and Maintenance 24. What should the nurse keep in mind when planning to communicate with a child who is diagnosed with an autism spectrum disorder (ASD)? a. The child has normal verbal communication. b. The child is expected to use sign language. c. The child may exhibit monotone speech and echolalia. d. The child is not listening if he/she is not looking at the nurse. ANS: C Children with autism have abnormalities in the production of speech, such as a monotone voice or echolalia, or inappropriate volume, pitch, rate, rhythm, or intonation. The child has impaired verbal communication and abnormalities in the production of speech. Some autistic children may use sign language, but it is not assumed. Children with autism often are reluctant to initiate direct eye contact. PTS: 1 DIF: Cognitive Level: Comprehension OBJ: Nursing Process: Planning MSC: Client Needs: Psychosocial Integrity 25. A child with autism is hospitalized with asthma. The nurse should plan care so that the: a. parents' expectations are met. b. child's routine habits and preferences are maintained. c. child is supported through the autistic crisis. d. parents need not be at the hospital. ANS: B Children with autism are often unable to tolerate even slight changes in routine. The child's routine habits and preferences are important to maintain. Focus of care is on the child's needs rather than on the parent's desires. Autism is a lifelong condition. The presence of the parents is almost always required when an autistic child is hospitalized. PTS: 1 DIF: Cognitive Level: Application OBJ: Nursing Process: Planning MSC: Client Needs: Psychosocial Integrity 26. An adolescent male visits his primary care provider complaining of difficulty with his vision. When the nurse asks the adolescent to explain what visual deficits he/she is experiencing, the adolescent states, “I am having difficulty seeing distant objects; they are less clear than things that are close.” What disorder does the nurse suspect the adolescent has? a. Hyphema b. Astigmatism c. Amblyopia d. Myopia ANS: D Myopic patients have the ability to see near objects more clearly than those at a distance; it is caused by the image focusing beyond the retina. Hyphema includes hemorrhage in the anterior chamber and is not a refractive disorder. Astigmatism is caused by an abnormal curvature of the cornea or lens. Amblyopia is a problem of reduced visual acuity not correctable by refraction. NURSINGTB.COM PTS: 1 DIF: Cognitive Level: Application OBJ: Nursing Process: Assessment MSC: Client Needs: Physiologic Integrity 27. Which statement by a parent about a child's conjunctivitis indicates that further teaching is needed? a. “I'll have separate towels and washcloths for each family member.” b. “I'll notify my doctor if the eye gets redder or the drainage increases.” c. “When the eye drainage improves, we'll stop giving the antibiotic ointment.” d. “After taking the antibiotic for 24 hours, my child can return to school.” ANS: C The antibiotic should be continued for the full prescription. Maintaining separate towels and washcloths will prevent the other family members from acquiring the infection. If the infection proliferates, the physician should be contacted. The child should be kept home from school or day care until the child receives the antibiotic for 24 hours. PTS: 1 DIF: Cognitive Level: Application OBJ: Nursing Process: Evaluation MSC: Client Needs: Physiologic Integrity 28. Which teaching guideline helps prevent eye injuries during sports and play activities? a. Restrict helmet use to those who wear eyeglasses or contact lenses. b. Discourage the use of goggles with helmets. c. Wear eye protection when participating in high risk sports such as paintball. d. Wear a face mask when playing any sport or playing roughly. ANS: C High risk sports such as paintball can cause penetrating eye injuries. Eye protection should be worn. All children who participate in sports should be protected by the appropriate headgear. Goggles and helmets can and should be used concurrently. A face mask does not prevent damage to the child's head. PTS: 1 DIF: Cognitive Level: Application OBJ: Nursing Process: Implementation MSC: Client Needs: Health Promotion and Maintenance 29. The teaching plan for the parents of a 3-year-old child with amblyopia should include which instruction? a. Apply a patch to the child's eyeglass lenses. b. Apply a patch only during waking hours. c. Apply a patch over the “bad” eye to strengthen it. d. Cover the “good” eye completely with a patch. ANS: D Treatment for amblyopia (lazy eye) requires that the “good” eye is patched to force the child to use the “bad” eye, thus strengthening the muscles. The patch should always be applied directly to the child's face, not to eyeglasses. The patch should be left in place even when the child is sleeping. Covering the “bad” eye will not contribute to strengthening it. The “good” eye should be patched. PTS: 1 DIF: Cognitive Level: Application OBJ: Nursing Process: Planning MSC: Client Needs: Physiologic Integrity NURSINGTB.COM 30. The nurse assesses a toddler for excessive tearing and corneal haziness to confirm which medical diagnosis? a. Viral conjunctivitis b. Paralytic strabismus c. Congenital cataract d. Infantile glaucoma ANS: D Excessive tearing and corneal haziness are indicative of glaucoma. Because the child is younger than 3 years of age, it would be classified as “infantile.” Discharge is noted with conjunctivitis. Corneal haziness is not a symptom of conjunctivitis. Paralytic strabismus is caused by weakness or paralysis of one or more of the extraocular muscles. Neither tearing nor corneal haziness is a symptom of paralytic strabismus. Congenital cataract will present as an opacity, but not excessive tearing. PTS: 1 DIF: Cognitive Level: Application OBJ: Nursing Process: Assessment MSC: Client Needs: Physiologic Integrity 31. The nurse is discussing sexuality with the parents of an adolescent with moderate cognitive impairment. Which should the nurse consider when dealing with this issue? a. Sterilization is recommended for any adolescent with cognitive impairment. b. Sexual drive and interest are limited in individuals with cognitive impairment. c. Individuals with cognitive impairment need a well-defined, concrete code of sexual conduct. d. Sexual intercourse rarely occurs unless the individual with cognitive impairment is sexually abused. ANS: C Adolescents with moderate cognitive impairment may be easily persuaded and lack judgment. A well-defined, concrete code of conduct with specific instructions for handling certain situations should be laid out for the adolescent. Permanent contraception by sterilization presents moral and ethical issues and may have psychologic effects on the adolescent. It may be prohibited in some states. The adolescent needs to have practical sexual information regarding physical development and contraception. Cognitively impaired individuals may desire to marry and have families. The adolescent needs to be protected from individuals who may make intimate advances. PTS: 1 DIF: Cognitive Level: Application OBJ: Nursing Process: Implementation MSC: Client Needs: Psychosocial Integrity 32. A nurse is preparing a teaching session for parents on the prevention of childhood hearing loss. The nurse identify what as being the most common cause of hearing impairment in children? a. Auditory nerve damage b. Congenital ear defects c. Congenital rubella d. Chronic otitis media ANS: D Chronic otitis media is the most common cause of hearing impairment in children. It is essential that appropriate measures be instituted to treat existing infections and prevent recurrences. Auditory nerve damaNgUe,RcSoINngGeTnBit.CalOeMar defects, and congenital rubella are rarer causes of hearing impairment. PTS: 1 DIF: Cognitive Level: Comprehension OBJ: Nursing Process: Implementation MSC: Client Needs: Health Promotion and Maintenance 33. The nurse is talking to the parent of a 13-month-old child. The mother states, “My child does not make noises like ‘da' or ‘na' like my sister's baby, who is only 9 months old.” Which statement by the nurse would be most appropriate to make? a. “I am going to request a referral to a hearing specialist.” b. “You should not compare your child to your sister's child.” c. “I think your child is fine, but we will check again in 3 months.” d. “You should ask other parents what noises their children made at this age.” ANS: A By 11 months of age, a child should be making well-formed syllables such as “da” or “na” and should be referred to a specialist if not. “You should not compare your child to your sister's child,” “I think your child is fine, but we will check again in 3 months,” and “You should ask other parents what noises their children made at this age” are not appropriate statements to make to the parent. PTS: 1 DIF: Cognitive Level: Analysis OBJ: Nursing Process: Implementation MSC: Client Needs: Physiologic Integrity: Reduction of Risk Potential 34. A nurse is preparing to perform a dressing change on a 6-year-old child with mild cognitive impairment (CI) who sustained a minor burn. Which strategy should the nurse use to prepare the child for this procedure? a. Verbally explain what will be done. b. Have the child watch a video on dressing changes. c. Demonstrate a dressing change on a doll. d. Explain the importance of keeping the burn area clean. ANS: C Children with CI have a marked deficit in their ability to discriminate between two or more stimuli because of difficulty in recognizing the relevance of specific cues. However, these children can learn to discriminate if the cues are presented in an exaggerated, concrete form and if all extraneous stimuli are eliminated. Therefore, demonstration is preferable to verbal explanation, and learning should be directed toward mastering a skill rather than understanding the scientific principles underlying a procedure. Watching a video would require the use of both visual and auditory stimulation and might produce overload in the child with mild CI. Explaining the importance of keeping the burn area clean would be too abstract for the child. PTS: 1 DIF: Cognitive Level: Application OBJ: Nursing Process: Implementation MSC: Client Needs: Physiologic Integrity: Basic Care and Comfort 35. A child with autism spectrum disorder (ASD) is admitted to the hospital with pneumonia. The nurse should plan which priority intervention when caring for the child? a. Maintain a structured routine and keep stimulation to a minimum. b. Place the child in a room withNaUrRoSoImNmGTaBte.CoOfMthe same age. c. Maintain frequent touch and eye contact with the child. d. Take the child frequently to the playroom to play with other children. ANS: A Providing a structured routine for the child to follow is key in the management of ASD. Decreasing stimulation by using a private room, avoiding extraneous auditory and visual distractions, and encouraging the parents to bring in possessions the child is attached to may lessen the disruptiveness of hospitalization. Because physical contact often upsets these children, minimum holding and eye contact may be necessary to avoid behavioral outbursts. Children with ASD need to be introduced slowly to new situations, with visits with staff caregivers kept short whenever possible. The playroom would be too overwhelming with new people and situations and should not be a priority of care. PTS: 1 DIF: Cognitive Level: Application OBJ: Nursing Process: Implementation MSC: Client Needs: Psychosocial Integrity MULTIPLE RESPONSE 1. Which interventions should the nurse plan when caring for a child with a visual impairment? (Select all that apply.) a. Touch the child upon entering the room before speaking. b. Keep items in the room in the same location. c. Describe the placement of the eating utensils on the meal tray. d. Use color examples to describe something to a child who has been blind since birth. e. Identify noises for the child. ANS: B, C, E Keep all items in the room in the same location and order. Describing how many steps away something is or the placement of eating utensils on a tray are both useful tactics. Identify noises for the child because children who are visually impaired or blind often have difficulty establishing the source of a noise. Never touch the child without identifying yourself and explaining what you plan to do. When describing objects or the environment to a child who is blind or visually impaired use familiar terms. If the child has been blind since birth, color has no meaning. PTS: 1 DIF: Cognitive Level: Application OBJ: Nursing Process: Implementation MSC: Client Needs: Physiologic Integrity 2. Which assessment findings help confirm a diagnosis of Down syndrome? (Select all that apply.) a. High-arched, narrow palate b. Protruding tongue c. Long, slender fingers d. Transverse palmar crease e. Hypertonic muscle tone ANS: A, B, D The assessment findings of Down syndrome include high-arched, narrow palate; protruding tongue; and transverse palmar creases. The fingers are stubby and the muscle tone is hypotonic, not hypertonic. NURSINGTB.COM PTS: 1 DIF: Cognitive Level: Comprehension OBJ: Nursing Process: Assessment MSC: Client Needs: Physiologic Integrity 3. A nurse is instructing a nursing assistant on techniques to facilitate lipreading with a hearing-impaired child who lip-reads. Which techniques should the nurse include? (Select all that apply.) a. Speak at eye level. b. Stand at a distance from the child. c. Speak words in a loud tone. d. Use facial expressions while speaking. e. Keep sentences short. ANS: A, D, E To facilitate lipreading for a hearing-impaired child who can lip-read, the speaker should be at eye level, facing the child directly or at a 45-degree angle. Facial expressions should be used to assist in conveying messages, and the sentences should be kept short. The speaker should stand close to the child, not at a distance. Using a loud tone while speaking will not facilitate lipreading. PTS: 1 DIF: Cognitive Level: Application OBJ: Nursing Process: Implementation MSC: Client Needs: Safe and Effective Care Environment: Management of Care Chapter 38: Family-Centered Care of the Child During Illness and Hospitalization Perry: Maternal Child Nursing Care, 6th Edition MULTIPLE CHOICE 1. What represents the major stressor of hospitalization for children from middle infancy throughout the preschool years? a. Separation anxiety b. Loss of control c. Fear of bodily injury d. Fear of pain ANS: A The major stress for children from infancy through the preschool years is separation anxiety, also called anaclitic depression. This is a major stressor of hospitalization. Loss of control, fear of bodily injury, and fear of pain are all stressors associated with hospitalization. However, separation from family is a primary stressor in this age-group. PTS: 1 DIF: Cognitive Level: Comprehension OBJ: Nursing Process: Assessment MSC: Client Needs: Health Promotion and Maintenance 2. When a preschool child is hospitalized without adequate preparation, what is the child may likely see hospitalization as? a. Punishment b. Threat to child's self-image c. An opportunity for regressionNURSINGTB.COM d. Loss of companionship with friends ANS: A If a toddler is not prepared for hospitalization, a typical preschooler fantasy is to attribute the hospitalization to punishment for real or imagined misdeeds. Threat to child's self-image and loss of companionship with friends are reactions typical of school-age children. Regression is a response characteristic of toddlers when threatened with loss of control. PTS: 1 DIF: Cognitive Level: Comprehension OBJ: Nursing Process: Planning MSC: Client Needs: Health Promotion and Maintenance 3. Because of their striving for independence and productivity, which age-group of children is particularly vulnerable to events that may lessen their feeling of control and power? a. Infants b. Toddlers c. Preschoolers d. School-age children ANS: D When a child is hospitalized, the altered family role, physical disability, loss of peer acceptance, lack of productivity, and inability to cope with stress usurp individual power and identity. This is especially detrimental to school-age children, who are striving for independence and productivity and are now experiencing events that lessen their control and power. Infants, toddlers, and preschoolers, although affected by loss of power are not as significantly affected as are school-age children. PTS: 1 DIF: Cognitive Level: Comprehension OBJ: Nursing Process: Diagnosis MSC: Client Needs: Health Promotion and Maintenance 4. A 10 year old, who needs to have another intravenous (IV) line started, keeps telling the nurse, “Wait a minute,” and, “I'm not ready.” How should the nurse interpret these requests? a. This is normal behavior for a school-age child. b. This behavior is usually not seen past the preschool years. c. The child thinks the nurse is punishing her. d. The child has successfully manipulated the nurse in the past. ANS: A This school-age child is attempting to maintain control. The nurse should provide the girl with structured choices about when the IV will be inserted. This can be characteristic behavior when an individual needs to maintain some control over a situation. The child is trying to have some control in the hospital experience. None of the other options accurately interprets the child's statement. PTS: 1 DIF: Cognitive Level: Analysis OBJ: Nursing Process: Implementation MSC: Client Needs: Health Promotion and Maintenance 5. A 6 year old, hospitalized again bNecUaRuSsIeNoGfTaBc.ChOroMnic illness, is told by school-age siblings that, “We are sick of Mom always sitting with you in the hospital and playing with you. It is not fair that you get everything and we have to stay with the neighbors.” What is the nurse's best assessment of the cause of the siblings' resentment? a. The siblings are immature and probably spoiled. b. Jealousy and resentment are common reactions to the illness or hospitalization of a sibling. c. The family has ineffective coping mechanisms to deal with chronic illness. d. The siblings need to better understand the patient's illness and needs. ANS: B Siblings experience loneliness, fear, worry, anger, resentment, jealousy, and guilt. The siblings experience stress equal to that of the hospitalized child. These are not uncommon responses by normal siblings. There is no evidence that the family has maladaptive coping or that the siblings lack understanding. PTS: 1 DIF: Cognitive Level: Analysis OBJ: Nursing Process: Assessment MSC: Client Needs: Psychosocial Integrity 6. What is an appropriate nursing intervention to minimize separation anxiety in a hospitalized toddler? a. Provide for privacy b. Encourage parents to room in c. Explain procedures and routines d. Encourage contact with children the same age ANS: B A toddler experiences separation anxiety secondary to being separated from the parents. To avoid this, the parents should be encouraged to room in as much as possible. Maintaining routines and ensuring privacy are helpful interventions, but they would not substitute for the parents. Contact with same-aged children would not substitute for having the parents present. PTS: 1 DIF: Cognitive Level: Analysis OBJ: Nursing Process: Assessment MSC: Client Needs: Health Promotion and Maintenance 7. Latasha, age 8 years, is being admitted to the hospital from the emergency department with an injury from falling off her bicycle. What intervention will help an 8 year old most in adjusting to a hospital admission? a. Explain hospital schedules such as mealtimes. b. Use terms such as “honey” and “dear” to show a caring attitude. c. Explain when parents can visit and why siblings cannot come to see her. d. Orient her parents, because she is young, to her room and hospital facility. ANS: A School-age children need to have control of their environment. The nurse should offer explanations or prepare the child for experiences that are unavoidable. The nurse should refer to the child by the preferred name. Telling the child about all of the limitations of visiting does not help him/her adjust to the hospital. At the age of 8 years, the child and parents should be oriented to the environment. PTS: 1 DIF: Cognitive Level: Application OBJ: Nursing Process: ImplementaNtioUnRSINMGSTCB.:CCOlMient Needs: Health Promotion and Maintenance 8. What intervention should the nurse implement when a 5 year old tells the nurse, “I need a Band-Aid” after having an injection. a. Apply a Band-Aid. b. Ask why he/she wants a Band-Aid. c. Explain why a Band-Aid is not needed. d. Show he/her that the bleeding has already stopped. ANS: A Children in this age-group still fear that their insides may leak out at the injection site, even if the bleeding has stopped. Provide the Band-Aid. No explanation should be required. PTS: 1 DIF: Cognitive Level: Application OBJ: Nursing Process: Implementation MSC: Client Needs: Health Promotion and Maintenance 9. Emma, age 3 years, is being admitted for about 1 week of hospitalization. The parents of a 3 year old being admitted tell the nurse that they are going to buy their child “a lot of new toys to help during the hospital.” The nurse's reply should be based on an understanding of comfort measures for that age-group? a. New toys do make hospitalization easier. b. New toys are usually better than older ones for children of this age. c. At this age children often need the comfort and reassurance of familiar toys from home. d. Buying new toys for a hospitalized child is a maladaptive way to cope with parental guilt. ANS: C Parents should bring favorite items from home to be with the child. Young children associate inanimate objects with significant people; they gain comfort and reassurance from these items. New toys will not serve the purpose of familiar toys and objects from home. The parents may experience some guilt as a response to the hospitalization, but there is no evidence that it is maladaptive. PTS: 1 DIF: Cognitive Level: Comprehension OBJ: Nursing Process: Assessment MSC: Client Needs: Health Promotion and Maintenance 10. The nurse is doing a prehospitalization orientation for a 7 year old, who is scheduled for cardiac surgery. As part of the preparation, the nurse explains that he/she will not be able to talk until the endotracheal tube is removed. What is the assessment of this explanation? a. It is unnecessary. b. It is the surgeon's responsibility. c. It is too stressful for a young child. d. It is an appropriate part of the child's preparation. ANS: D This is a necessary part of preoperative preparation that will help reduce the anxiety associated with surgery. If the child wakes and is not prepared for the inability to speak, she will be even more anxious. It is a joint responsibility of nursing, medical staff, and child life personnel. This is a necessary component of preparation that will help reduce the anxiety associated with surgery. PTS: 1 DIF: CognitiNveURLSevINelG: TABn.aClOysMis OBJ: Nursing Process: Implementation MSC: Client Needs: Health Promotion and Maintenance 11. The nurse is caring for an adolescent hospitalized after a bicycle accident. Which statement by the adolescent would be expected about separation anxiety? a. “I wish my parents could spend the night with me while I am in the hospital.” b. “I think I would like for my siblings to visit me but not my friends.” c. “I hope my friends don't forget about visiting me.” d. “I will be embarrassed if my friends come to the hospital to visit.” ANS: C Loss of peer-group contact may pose a severe emotional threat to an adolescent because of loss of group status; friends' visiting is an important aspect of hospitalization for an adolescent and would be very reassuring. Adolescents may welcome the opportunity to be away from their parents. The separation from siblings may produce reactions from difficulty coping to a welcome relief. PTS: 1 DIF: Cognitive Level: Analysis OBJ: Nursing Process: Planning MSC: Client Needs: Health Promotion and Maintenance 12. Which situation poses the greatest challenge to the nurse working with a child and family? a. Twenty-four-hour observation b. Emergency hospitalization c. Outpatient admission d. Rehabilitation admission ANS: B Emergency hospitalization involves: (1) limited time for preparation both for the child and family, (2) situations that cause fear for the family that the child may die or be permanently disabled, and (3) a high level of activity, which can foster further anxiety. Although preparation time may be limited with a 24-hour observation, this situation does not usually involve the acuteness of the situation and the high levels of anxiety associated with emergency admission. Outpatient admission generally involves preparation time for the family and child. Because of the lower level of acuteness in this setting, anxiety levels are not as high. Rehabilitation admission follows a serious illness or disease. This type of unit may resemble a home environment, which decreases the child's and family's anxiety. PTS: 1 DIF: Cognitive Level: Comprehension OBJ: Nursing Process: Planning MSC: Client Needs: Safe and Effective Care Environment 13. What is the primary disadvantage associated with outpatient and day facility care? a. Increased cost b. Increased risk of infection c. Lack of physical connection to the hospital d. Longer separation of the child from family ANS: C Outpatient and day facility care do not provide extended care; therefore, a child requiring extended care must be transferred to the hospital, causing increased stress to the child and parents. Outpatient care decreases cost and reduces the risk of infection. Outpatient care also minimizes separation of the child from family. PTS: 1 DIF: CognitiNveURLSevINelG: TCBo.mCOprMehension OBJ: Nursing Process: Implementation MSC: Client Needs: Safe and Effective Care Environment 14. What is the best action for the nurse to take when a 5-year-old child who requires another 2 days of intravenous (IV) antibiotics cries, screams, and resists having the IV restarted? a. Exit the room and leave the child alone until he stops crying. b. Tell the child big boys and girls “don't cry.” c. Let the child decide which color arm board to use with the IV. d. Administer a narcotic analgesic for pain to quiet the child. ANS: C Giving the preschooler some choice and control, while maintaining boundaries of treatment, supports the child's coping skills. Leaving the child alone robs the child of support when a coping difficulty exists. Crying is a normal response to stress. The child needs time to adjust and support to cope with unfamiliar and painful procedures during hospitalization. Although administration of a topical analgesic is indicated before restarting the child's IV, a narcotic analgesic is not indicated. PTS: 1 DIF: Cognitive Level: Application OBJ: Nursing Process: Implementation MSC: Client Needs: Psychosocial Integrity 15. During the first 4 days of hospitalization, an 18 month old cried inconsolably when his/her parents left and he/she refused the staff's attention. Now the nurse observes that the child appears to be “settled in” and unconcerned about seeing his/her parents. How should the nurse interpret this change in behavior? a. The child has successfully adjusted to the hospital environment. b. The child has transferred their trust to the nursing staff. c. The child may be experiencing detachment, which is the third stage of separation anxiety. d. Because the child is “at home” in the hospital now, seeing his mother frequently will only start the cycle again. ANS: C Detachment is a behavioral manifestation of separation anxiety. Superficially it appears that the child has adjusted to the loss and transferred his trust to the nursing staff. Detachment is a sign of resignation, not contentment. Parents should be encouraged to be with their child. If parents restrict visits, they may begin a pattern of misunderstanding the child's cues and not meeting his needs. PTS: 1 DIF: Cognitive Level: Analysis OBJ: Nursing Process: Diagnosis MSC: Client Needs: Health Promotion and Maintenance 16. A 14-year-old boy is being admitted to the hospital for an appendectomy. Which roommate should the nurse assign with this patient? a. A 4-year-old boy who is first day postappendectomy surgery. b. A 6-year-old boy with pneumonia. c. A 15-year-old boy admitted with a vaso-occlusive sickle cell crisis. d. A 12-year-old boy with celluliNtiUs.RSINGTB.COM ANS: C When a child is admitted, nurses follow several fairly universal admission procedures. The minimum considerations for room assignment are age, sex, and nature of the illness. Age-grouping is especially important for adolescents. The 14-year-old boy being admitted to the unit after appendectomy surgery should be placed with a noninfectious child of the same sex and age. The 15-year-old child with sickle cell is the best choice. The 4-year-old boy who is postappendectomy is too young, and the child with pneumonia is too young and possibly has an infectious process. The 12-year-old boy with cellulitis is the right age, but he has an infection (cellulitis). PTS: 1 DIF: Cognitive Level: Application OBJ: Nursing Process: Implementation MSC: Client Needs: Safe and Effective Care Environment: Management of Care 17. A nurse is preparing to complete an admission assessment on a 2-year-old child who is sitting on the parent's lap. Which technique should the nurse implement to complete the physical examination? a. Ask the parent to place the child in the hospital crib. b. Take the child and parent to the examination room. c. Perform the examination while the child is on the parent's lap. d. Ask the child to stand by the parent while completing the examination. ANS: C The nurse should complete the examination while the child is on the parent's lap. For young children, particularly infants and toddlers, preserving parent-child contact is the best means of decreasing the need for or stress of restraint. The entire physical examination can be done in a parent's lap with the parent hugging the child for procedures such as an otoscopic examination. Placing the child in the crib, taking the child to the examination room, or asking the child to stand by the parent would separate the child from the parent and cause anxiety. PTS: 1 DIF: Cognitive Level: Application OBJ: Nursing Process: Implementation MSC: Client Needs: Psychosocial Integrity 18. A school-age child, admitted for intravenous antibiotic therapy for osteomyelitis, reports difficulty in going to sleep at night. Which intervention should the nurse implement to assist the child in going to sleep at bedtime? a. Request a prescription for a sleeping pill. b. Allow the child to stay up late and sleep late in the morning. c. Create a schedule similar to the one the child follows at home. d. Plan passive activities in the morning and interactive activities right before bedtime. ANS: C Many children obtain significantly less sleep in the hospital than at home; the primary causes are a delay in sleep onset and early termination of sleep because of hospital routines. One technique that can minimize the disruption in the child's routine is establishing a daily schedule. This approach is most suitable for noncritically ill school-age and adolescent children who have mastered the concept of time. It involves scheduling the child's day to include all those activities that are important to the child and nurse, such as treatment procedures, schoolwork, exercise,NtUelReSvIiNsiGoTnB, p.ClaOyMroom, and hobbies. The school-age child with osteomyelitis would benefit from a schedule similar to the one followed at home. Requesting a prescription for a sleeping pill would be inappropriate, and allowing the child to stay up late and sleep late would not be keeping the child in a routine followed at home. Passive activities in the morning and interactive activities at bedtime should be reversed; it would be better to keep the child active in the morning hours and plan quiet activities at bedtime. PTS: 1 DIF: Cognitive Level: Application OBJ: Nursing Process: Planning MSC: Client Needs: Health Promotion and Maintenance 19. A previously “potty-trained” 30-month-old child has reverted to wearing diapers while hospitalized. The nurse should reassure the parents based on what knowledge concerning regressive behaviors? a. Regression is seen during hospitalization. b. Developmental delays occur because of the hospitalization. c. The child is experiencing urinary urgency because of hospitalization. d. The child was too young to be “potty-trained.” ANS: A Regression is expected and normal for all age-groups when hospitalized. Nurses should assure the parents this is temporary and the child will return to the previously mastered developmental milestone when back home. This does not indicate a developmental delay. The child should not be experiencing urinary urgency because of hospitalization and this would not be normal. Successful “potty-training” can be started at 2 years of age if the child is ready. PTS: 1 DIF: Cognitive Level: Application OBJ: Nursing Process: Teaching/Learning MSC: Client Needs: Health Promotion and Maintenance 20. The nurse needs to take a blood pressure on the child playing in the playroom. Which is the appropriate procedure for obtaining the blood pressure? a. Take the blood pressure in the playroom. b. Ask the child to come to the exam room to obtain the blood pressure. c. Ask the child to return to his or her room for the blood pressure, then escort the child back to the playroom. d. Document that the blood pressure was not obtained because the child was in the playroom. ANS: C The playroom is a safe haven for children, free from medical or nursing procedures. The child can be returned to his or her room for the blood pressure and then escorted back to the playroom. The examination room is reserved for painful procedures that should not be performed in the child's hospital bed. Documenting that the blood pressure was not obtained because the child was in the playroom is inappropriate. PTS: 1 DIF: Cognitive Level: Application OBJ: Nursing Process: Implementation MSC: Client Needs: Psychosocial Integrity 21. A nurse in the emergency department is assessing a 5-year-old child with symptoms of pneumonia and a fever of 102° F. Which intervention can the nurse implement to promote a sense of control for the child? a. None, this is an emergency anNdUthReSIcNhGilTdBs.hCoOuMld not participate in care. b. Allow the child to hold the digital thermometer while taking the child's blood pressure. c. Ask the child if it is OK to take a temperature in the ear. d. Have parents wait in the waiting room. ANS: B The nurse should allow the child to hold the digital thermometer while taking the child's blood pressure. Unless an emergency is life threatening, children need to participate in their care to maintain a sense of control. Because emergency departments are frequently hectic, there is a tendency to rush through procedures to save time. However, the extra few minutes needed to allow children to participate may save many more minutes of useless resistance and uncooperativeness during subsequent procedures. The child may not give permission, if asked, for a procedure that is necessary to be performed. It is better to give choices such as, “Which ear do you want me to do your temperature in?” instead of, “Can I take your temperature?” Parents should remain with their child to help with decreasing the child's anxiety. PTS: 1 DIF: Cognitive Level: Application OBJ: Nursing Process: Implementation MSC: Client Needs: Health Promotion and Maintenance MULTIPLE RESPONSE 1. A child has just been unexpectedly admitted to the intensive care unit after abdominal surgery. The nursing staff has completed the admission process, and the child's condition is beginning to stabilize. When speaking with the parents, the nurses should expect which stressors to be evident? (Select all that apply.) a. Unfamiliar environment b. Usual day-night routine c. Strange smells d. Provision of privacy e. Inadequate knowledge of condition and routine ANS: A, C, E Intensive care units, especially when the family is unprepared for the admission, are a strange and unfamiliar place. There are many pieces of unfamiliar equipment, and the sights and sounds are much different from a general hospital unit. Also, with the child's condition being more precarious, it may be difficult to keep the parents updated and knowledgeable about what is happening. Lights are usually on around the clock, seriously disrupting the diurnal rhythm. There is usually little privacy available for families in intensive care units. PTS: 1 DIF: Cognitive Level: Analysis OBJ: Nursing Process: Assessment MSC: Client Needs: Psychosocial Integrity 2. What is an age-appropriate nursing intervention to facilitate psychologic adjustment for an adolescent expected to have a prolonged hospitalization? (Select all that apply.) a. Encourage parents to bring in homework and schedule study times. b. Allow the adolescent to wear street clothes. c. Involve the parents in care. d. Follow home routines. NURSINGTB.COM e. Encourage parents to bring in favorite foods. ANS: A, B, E Encouraging parents to bring in homework, street clothes, and favorite foods are all developmentally appropriate approaches to facilitate adjustment and coping for an adolescent who will be experiencing prolonged hospitalization. Involving parents in care and following home routines are important interventions for the preschool child who is in the hospital. Adolescents do not need parents to assist in their care. They are used to performing independent self-care. Adolescents may want their parents to be nearby, or they may enjoy the freedom and independence from parental control and routines. PTS: 1 DIF: Cognitive Level: Application OBJ: Nursing Process: Implementation MSC: Client Needs: Health Promotion and Maintenance 3. A nurse plans therapeutic play time for a hospitalized child. Which are the benefits of therapeutic play? (Select all that apply.) a. Serves as method to assist disturbed children b. Allows the child to express feelings c. The nurse can gain insight into the child's feelings d. The child can deal with concerns and feelings e. Gives the child a structured play environment ANS: B, C, D Therapeutic play is an effective, nondirective modality for helping children deal with their concerns and fears, and at the same time, it often helps the nurse gain insights into children's needs and feelings. Play and other expressive activities provide one of the best opportunities for encouraging emotional expression, including the safe release of anger and hostility. Nondirective play that allows children freedom for expression can be tremendously therapeutic. Play therapy is a structured therapy that helps disturbed children. It should not be confused with therapeutic play. PTS: 1 DIF: Cognitive Level: Comprehension OBJ: Nursing Process: Planning MSC: Client Needs: Psychosocial Integrity 4. A child is being discharged from an ambulatory care center after an inguinal hernia repair. Which discharge interventions should the nurse implement? (Select all that apply.) a. Discuss dietary restrictions. b. Hold any analgesic medications until the child is home. c. Send a pain scale home with the family. d. Suggest the parents fill the prescriptions on the way home. e. Discuss complications that may occur. ANS: A, C, E The discharge interventions a nurse should implement when a child is being discharged from an ambulatory care center should include dietary restrictions being very specific and giving examples of “clear fluids” or what is meant by a “full liquid diet.” The nurse should give specific information on pain control and send a pain scale home with the family. All complications that may occur after an inguinal hernia repair should be discussed with the parents. The pain medication, as prescribed, should be given before the child leaves the building, and prescriptions shouldNbUeRfSilIlNeGdTaBn.dCOgMiven to the family before discharge. PTS: 1 DIF: Cognitive Level: Application OBJ: Nursing Process: Implementation MSC: Client Needs: Health Promotion and Maintenance Chapter 39: Pediatric Variations of Nursing Interventions Perry: Maternal Child Nursing Care, 6th Edition MULTIPLE CHOICE 1. What should the nurse consider when having consent forms signed for surgery and procedures on children? a. Only a parent or legal guardian can give consent. b. The person giving consent must be at least 18 years old. c. The risks and benefits of a procedure are part of the consent process. d. A mental age of 7 years or older is required for a consent to be considered “informed.” ANS: C The informed consent must include the nature of the procedure, benefits and risks, and alternatives to the procedure. In special circumstances such as emancipated minors, the consent can be given by someone younger than 18 years without the parent or legal guardian. A mental age of 7 years is too young for consent to be informed. PTS: 1 DIF: Cognitive Level: Comprehension OBJ: Nursing Process: Planning MSC: Client Needs: Safe and Effective Care Environment 2. The nurse is planning how to best prepare a 4-year-old child for some diagnostic procedures. What guideline should the nurse consider when preparing a preschooler for a diagnostic procedure? a. Planning for a short teaching sNeUssRioSInNoGfTaBb.CouOtM30 minutes. b. Telling the child that procedures are never a form of punishment. c. Keeping equipment out of the child's view. d. Using correct scientific and medical terminology in explanations. ANS: B Illness and hospitalization may be viewed as punishment in preschoolers. Always state directly that procedures are never a form of punishment. Teaching sessions for this age-group should be 10 to 15 minutes in length. Demonstrate the use of equipment and allow the child to play with miniature or actual equipment. Explain the procedure and how it affects the child in simple terms. PTS: 1 DIF: Cognitive Level: Application OBJ: Nursing Process: Implementation MSC: Client Needs: Health Promotion and Maintenance 3. The most appropriate nursing action to implement when a preschooler being prepped for outpatient surgery refused to allow the parent to remove his/her underwear? a. Allow the child to wear their underpants. b. Discuss to the mother why this is important. c. Ask the mother to explain to her child why he/she must remove the underwear. d. Explain in a kind, matter-of-fact manner that this is hospital policy. ANS: A It is appropriate for the child to leave his/her underpants on. This allows his/her some measure of control during the foot surgery. The mother should not be required to make the child more upset. The child is too young to understand what hospital policy means. PTS: 1 DIF: Cognitive Level: Application OBJ: Nursing Process: Implementation MSC: Client Needs: Health Promotion and Maintenance 4. Using knowledge of child development, what is the best approach when preparing a toddler for a procedure? a. Avoid asking the child to make choices. b. Demonstrate the procedure on a doll. c. Plan for the teaching session to last about 20 minutes. d. Show necessary equipment without allowing child to handle it. ANS: B Prepare toddlers for procedures by using play. Demonstrate on a doll, but avoid the child's favorite doll because the toddler may think the doll is really “feeling” the procedure. In preparing a toddler for a procedure, the child is allowed to participate in care and help whenever possible. Teaching sessions for toddlers should be about 5 to 10 minutes. Use a small replica of the equipment and allow the child to handle it. PTS: 1 DIF: Cognitive Level: Application OBJ: Nursing Process: Planning MSC: Client Needs: Health Promotion and Maintenance 5. The nurse is cleaning multiple facial abrasions on a 9-year-old who was brought to the emergency department by his/her mother. When the child begins crying and screaming loudly, what intervention should the nurse implement to best manage this situation? a. Calmly ask the child to be quiNetUeRr.SINGTB.COM b. Suggest that his/her mother help the child to relax. c. Tell the child it is okay to cry and scream. d. Suggest that he/she talk to his/her mother as a form of distraction. ANS: C The child should be allowed to express feelings of anger, anxiety, fear, frustration, or any other emotion. The child needs to know that it is all right to cry. There is no reason for him to be quieter. He is too upset and needs to be able to express his feelings. PTS: 1 DIF: Cognitive Level: Application OBJ: Nursing Process: Implementation MSC: Client Needs: Health Promotion and Maintenance 6. The nurse monitoring a child for signs and symptoms of malignant hyperthermia should be alert for which early sign of this disorder? a. Apnea b. Bradycardia c. Muscle rigidity d. Decreased blood pressure ANS: C Early signs of malignant hyperthermia include tachycardia, increasing blood pressure, tachypnea, mottled skin, and muscle rigidity. Apnea is not a sign of malignant hyperthermia. Tachycardia, not bradycardia, is an early sign of malignant hyperthermia. Increased, not decreased, blood pressure is characteristic of malignant hyperthermia. PTS: 1 DIF: Cognitive Level: Comprehension OBJ: Nursing Process: Assessment MSC: Client Needs: Physiologic Integrity 7. The nurse is caring. What skin care interventions for an unconscious child should be included in the plan of care? a. Avoiding use of pressure reduction on the bed. b. Massaging reddened bony prominences to prevent deep tissue damage. c. Using drawsheet to move child in bed to reduce friction and shearing injuries. d. Avoiding rinsing skin after cleansing with mild antibacterial soap to provide a protective barrier. ANS: C A drawsheet should be used to move the child in the bed or onto a gurney to reduce friction and shearing injuries. Do not drag the child from under the arms. Bony prominences should not be massaged if reddened. Deep tissue damage can occur. Pressure-reduction devices should be used to redistribute weight instead. The skin should be cleansed with mild nonalkaline soap or soap-free cleaning agents for routine bathing. PTS: 1 DIF: Cognitive Level: Application OBJ: Nursing Process: Implementation MSC: Client Needs: Physiologic Integrity 8. What is an appropriate intervention to encourage food and fluid intake in a hospitalized child? a. Force child to eat and drink to combat caloric losses. b. Discourage participation in noneating activities until caloric intake is sufficient. c. Administer large quantities of flavored fluids at frequent intervals and during meals. d. Give high-quality foods and snNaUcRkSsIwNGheTnBe.CvOerMchild expresses hunger. ANS: D Small, frequent meals and nutritious snacks should be provided for the child. Favorite foods such as peanut butter and jelly sandwiches, fruit yogurt, cheese, pizza, macaroni, and cheese should be available. Forcing a child to eat only meets with rebellion and reinforces the behavior as a control mechanism. Large quantities of fluid may decrease the child's hunger and further inhibit food intake. PTS: 1 DIF: Cognitive Level: Application OBJ: Nursing Process: Implementation MSC: Client Needs: Physiologic Integrity 9. A 3 year old has a 102° F fever associated with a viral illness that has not responded to acetaminophen. The nurse's action should be based on what knowledge about fevers in children? a. Fevers such as this are common with viral illnesses. b. Seizures are common in children when antipyretics are ineffective. c. Fever over 102° F indicates greater severity of illness. d. Fever over 102° F indicates a probable bacterial infection. ANS: A Most fevers are of brief duration, have limited consequences, and are viral. Little evidence supports the use of antipyretic drugs to prevent febrile seizures. Neither the increase in temperature nor its response to antipyretics indicates the severity or etiology of infection. PTS: 1 DIF: Cognitive Level: Comprehension OBJ: Nursing Process: Diagnosis MSC: Client Needs: Physiologic Integrity 10. What intervention is appropriate when administering tepid water or sponge baths prescribed for hyperthermia in children? a. Add isopropyl alcohol to the water. b. Direct a fan on the child in the bath. c. Stop the bath if the child begins to chill. d. Continue the bath for 5 minutes. ANS: C Environmental measures such as sponge baths can be used to reduce temperature if tolerated by the child and if they do not induce shivering. Shivering is the body's way of maintaining the elevated set point. Compensatory shivering increases metabolic requirements above those already caused by the fever. Ice water and isopropyl alcohol are inappropriate, potentially dangerous solutions. Fans should not be used because of the risk of the child developing vasoconstriction, which defeats the purpose of the cooling measures. Little blood is carried to the skin surface, and the blood remains primarily in the viscera to become heated. The child is placed in a tub of tepid water for 20 to 30 minutes. PTS: 1 DIF: Cognitive Level: Application OBJ: Nursing Process: Implementation MSC: Client Needs: Physiologic Integrity 11. The nurse wore gloves during a dressing change. When the gloves are removed, the nurse should perform which initial action? a. Wash hands thoroughly. b. Check the gloves for leaks. c. Rinse gloves in disinfectant soNluUtRioSnIN. GTB.COM d. Apply new gloves before touching the next patient. ANS: A When gloves are worn, the hands are washed thoroughly after removing the gloves because both latex and vinyl gloves fail to provide complete protection. Gloves should be disposed of after use and hands should be thoroughly washed again before new gloves are applied. PTS: 1 DIF: Cognitive Level: Application OBJ: Nursing Process: Implementation MSC: Client Needs: Safe and Effective Care Environment 12. The nurse gives an injection in a patient's room. Which method should the nurse use to dispose of the needle? a. Dispose of syringe and needle in a rigid, puncture-resistant container in patient's room. b. Dispose of syringe and needle in a rigid, puncture-resistant container in an area outside of patient's room. c. Cap needle immediately after giving injection and dispose of in proper container. d. Cap needle, break from syringe, and dispose of in proper container. ANS: A All needles (uncapped and unbroken) are disposed of in a rigid, puncture-resistant container located near the site of use. Consequently, these containers should be installed in the patient's room. The uncapped needle should not be transported to an area distant from use. PTS: 1 DIF: Cognitive Level: Application OBJ: Nursing Process: Implementation MSC: Client Needs: Safe and Effective Care Environment 13. Frequent urine testing for specific gravity and glucose are required on a 6-month-old infant. How should the nurse collect small amounts of urine for these tests? a. Apply a urine-collection bag to the perineal area. b. Tape a small medicine cup to the inside of the diaper. c. Aspirate urine from cotton balls inside the diaper with a syringe. d. Aspirate urine from a superabsorbent disposable diaper with a syringe. ANS: C To obtain small amounts of urine, use a syringe without a needle to aspirate urine directly from the diaper. If diapers with absorbent material are used, place a small gauze dressing or cotton balls inside the diaper to collect the urine, and aspirate the urine with a syringe. For frequent urine sampling, the collection bag would be too irritating to the child's skin. Taping a small medicine cup to the inside of the diaper is not feasible; the urine will spill from the cup. Diapers with superabsorbent gels absorb the urine, so there is nothing to aspirate. PTS: 1 DIF: Cognitive Level: Application OBJ: Nursing Process: Implementation MSC: Client Needs: Physiologic Integrity 14. What is an important nursing consideration when performing a bladder catheterization on a young boy? a. Use clean technique, not Standard Precautions. b. Insert 2% lidocaine lubricant into the urethra. c. Lubricate catheter with water-NsoUlRuSbIlNe GluTbBr.iCcOanMt such as K-Y Jelly. d. Delay catheterization for 20 minutes while anesthetic lubricant is absorbed. ANS: B The anxiety, fear, and discomfort experienced during catheterization can be significantly decreased by preparing the child and parents, selecting the correct catheter, and using appropriate insertion technique. Generous lubrication of the urethra before catheterization and use of lubricant containing 2% lidocaine may reduce or eliminate the burning and discomfort associated with this procedure. Catheterization is a sterile procedure, and Standard Precautions for body-substance protection should be followed. Water-soluble lubricants do not provide appropriate local anesthesia. Catheterization should be delayed only 2 to 3 minutes. This provides sufficient local anesthesia for the procedure. PTS: 1 DIF: Cognitive Level: Application OBJ: Nursing Process: Implementation MSC: Client Needs: Physiologic Integrity 15. What is the most appropriate statement for the nurse to make to a 5-year-old child who is undergoing a venipuncture? a. “You must hold still or I'll have someone hold you down. This is not going to hurt.” b. “This will hurt like a pinch. I'll get someone to help hold your arm still so it will be over fast and hurt less.” c. “Be a big boy and hold still. This will be over in just a second.” d. “I'm sending your mother out so she won't be scared. You are big, so hold still and this will be over soon.” ANS: B Honesty is the best approach. Children should be told what sensation they will feel during a procedure. A 5-year-old child should not be expected to hold still, and assistance ensures safety to everyone. Telling the child that “This will be over in just a second” is not supportive or honest. Parents should be encouraged to remain with the child unless they are extremely uncomfortable doing so. PTS: 1 DIF: Cognitive Level: Analysis OBJ: Nursing Process: Implementation MSC: Client Needs: Physiologic Integrity 16. What important consideration in providing atraumatic care should the nurse consider when preforming a venipuncture on a 6-year-old child? a. Use an 18-gauge needle if possible. b. If not successful after four attempts, have another nurse try. c. Restrain the child only as needed to perform venipuncture safely. d. Show the child equipment to be used before procedure. ANS: C Restrain the child only as needed to perform the procedure safely; an alternative would be the use of therapeutic hugging. Use the smallest gauge needle that permits free flow of blood. A two-try-only policy is desirable, in which two operators each have only two attempts. If insertion is not successful after four punctures, alternative venous access should be considered. Keep all equipment out of sight until used. PTS: 1 DIF: Cognitive Level: Application OBJ: Nursing Process: ImplementaNtioUnRSINMGSTCB.:CCOlMient Needs: Physiologic Integrity 17. The nurse administering a bitter oral medication to an infant or small child should mix the medication with what substance? a. A bottle of formula or milk. b. Any food the child is going to eat. c. A teaspoon of jam or ice cream. d. Large amounts of water to dilute medication sufficiently. ANS: C Mix the drug with a small amount (about 1 teaspoon) of sweet-tasting substance. This will make the medication more palatable to the child. The medication should be mixed with only a small amount of food or liquid. If the child does not finish drinking/eating, it is difficult to determine how much medication was consumed. Medication should not be mixed with essential foods and milk. The child may associate the altered taste with the food and refuse to eat in future. PTS: 1 DIF: Cognitive Level: Application OBJ: Nursing Process: Implementation MSC: Client Needs: Physiologic Integrity 18. When liquid medication is given to a crying 10-month-old infant, which approach minimizes the possibility of aspiration? a. Administering the medication with a syringe (without needle) placed along the side of the infant's tongue. b. Administering the medication as rapidly as possible with the infant securely restrained. c. Mixing the medication with the infant's regular formula or juice and administering by bottle. d. Keeping the child upright with the nasal passages blocked for a minute after administration. ANS: A Administer the medication with a syringe without needle placed alongside of the infant's tongue. The contents are administered slowly in small amounts, allowing the child to swallow between deposits. Medications should be given slowly to avoid aspiration. The medication should be mixed with only a small amount of food or liquid. If the child does not finish drinking/eating, it is difficult to determine how much medication was consumed. Essential foods also should not be used. The child may associate the altered taste with the food and refuse to eat in future. Holding the child's nasal passages increases the risk of aspiration. PTS: 1 DIF: Cognitive Level: Application OBJ: Nursing Process: Implementation MSC: Client Needs: Physiologic Integrity 19. Guidelines for intramuscular administration of medication in school-age children include what instruction? a. Inject medication as rapidly as possible. b. Insert the needle quickly, using a dart-like motion. c. Penetrate the skin immediately after cleansing the site, before skin has dried. d. Have the child stand, if possible, and if he or she is cooperative. ANS: B The needle should be inserted quickly in a dart-like motion at a 90-degree angle unless contraindicated. Inject medicationNsUsRloSwINlyG.TABl.CloOwMskin preparation to dry completely before skin is penetrated. Place the child in a lying or sitting position. PTS: 1 DIF: Cognitive Level: Comprehension OBJ: Nursing Process: Implementation MSC: Client Needs: Physiologic Integrity 20. When teaching a mother how to administer eyedrops, where should the nurse instruct to place them? a. In the conjunctival sac that is formed when the lower lid is pulled down. b. Carefully under the upper eyelid while it is gently pulled upward. c. On the sclera while the child looks to the side. d. Anywhere as long as drops contact the eye's surface. ANS: A The lower lid is pulled down, forming a small conjunctival sac. The solution or ointment is applied to this area. The medication should not be administered directly on the eyeball. PTS: 1 DIF: Cognitive Level: Comprehension OBJ: Nursing Process: Implementation MSC: Client Needs: Physiologic Integrity 21. A 2-year-old child comes to the emergency department demonstrating signs of dehydration and hypovolemic shock. Which best explains why an intraosseous infusion is started? a. It is less painful for small children. b. Rapid venous access is not possible. c. Antibiotics must be started immediately. d. Long-term central venous access is not possible. ANS: B In situations in which rapid establishment of systemic access is vital and venous access is hampered, such as peripheral circulatory collapse and hypovolemic shock, intraosseous infusion provides a rapid, safe lifesaving alternative. The procedure is painful, and local anesthesia and systemic analgesia are given. Antibiotics could be given when vascular access is obtained. Long-term central venous access is time-consuming, and intraosseous infusion is used in an emergency situation. PTS: 1 DIF: Cognitive Level: Comprehension OBJ: Nursing Process: Planning MSC: Client Needs: Physiologic Integrity 22. When caring for a child with an intravenous infusion, the nurse should include which intervention in the plan of care? a. Using a macrodropper to facilitate reaching the prescribed flow rate. b. Avoid restraining the child to prevent undue emotional stress. c. Changing the insertion site every 24 hours. d. Observing the insertion site frequently for signs of infiltration. ANS: D The nursing responsibility for intravenous therapy is to calculate the amount to be infused in a given length of time, set the infusion rate, and monitor the apparatus frequently, at least every 1 to 2 hours, to make certain that the desired rate is maintained, the integrity of the system remains intact, the site remains intact (free of redness, edema, infiltration, or irritation), and the infusion does not stop. A minidropper (60 drops/mL) is the recommended intravenous tubing in pediatrics. The intravenous site should be protected. This may require soft restraints on the child. Insertion sites do notNnUeRedSINtoGbTeBc.ChOanMged every 24 hours unless a problem is found with the site. Frequent change exposes the child to significant trauma. PTS: 1 DIF: Cognitive Level: Application OBJ: Nursing Process: Implementation MSC: Client Needs: Physiologic Integrity 23. It is important to make certain that sensory connectors and oximeters are compatible since wiring that is incompatible increases the risk of which injury? a. Hyperthermia b. Electrocution c. Pressure necrosis d. Burns under sensors ANS: D It is important to make certain that sensor connectors and oximeters are compatible. Wiring that is incompatible can generate considerable heat at the tip of the sensor, causing second- and third-degree burns under the sensor. Incompatibility would cause a local irritation or burn, not hyperthermia. A low voltage is used, which should not present risk of electrocution. Pressure necrosis can occur from the sensor being attached too tightly, but this is not a problem of incompatibility. PTS: 1 DIF: Cognitive Level: Comprehension OBJ: Nursing Process: Implementation MSC: Client Needs: Physiologic Integrity 24. What intervention should the nurse implement when suctioning a child with a tracheostomy? a. Encouraging the child to cough to raise the secretions before suctioning. b. Selecting a catheter with a diameter three-fourths as large as the diameter of the tracheostomy tube. c. Ensuring that each pass of the suction catheter take no longer than 10 seconds. d. Allowing the child to rest after every 5 times the suction catheter is passed. ANS: C Suctioning should require no longer than 10 seconds per pass. Otherwise the airway may be occluded for too long. If the child is able to cough up secretions, suctioning may not be indicated. The catheter should have a diameter one-half the size of the tracheostomy tube. If it is too large, it might block the child's airway. The child is allowed to rest for 30 to 60 seconds after each aspiration to allow oxygen tension to return to normal. Then the process is repeated until the trachea is clear. PTS: 1 DIF: Cognitive Level: Analysis OBJ: Nursing Process: Implementation MSC: Client Needs: Physiologic Integrity 25. A child is receiving total parenteral nutrition (TPN; hyperalimentation). At the end of 8 hours, the nurse observes the solution and notes that 200 mL/8 hr is being infused rather than the ordered amount of 300 mL/8 hr. The nurse should adjust the rate so that how much will infuse during the next 8 hours? a. 200 mL b. 300 mL c. 350 mL d. 400 mL ANS: B The TPN infusion rate should notNbUeRinScINreGaTsBed.CoOrMdecreased without the practitioner being informed because alterations in rate can cause hyperglycemia or hypoglycemia. Knowing this will result in the infusion rate being set to the original prescribed flow rate. PTS: 1 DIF: Cognitive Level: Application OBJ: Nursing Process: Implementation MSC: Client Needs: Physiologic Integrity 26. In preparing to give “enemas until clear” to a young child, the nurse should select which solution? a. Tap water b. Normal saline c. Oil retention d. Fleet solution ANS: B Isotonic solutions should be used in children. Saline is the solution of choice. Plain water is not used. This is a hypotonic solution and can cause rapid fluid shift, resulting in fluid overload. Oil-retention enemas will not achieve the “until clear” result. Fleet enemas are not advised for children because of the harsh action of the ingredients. The osmotic effects of the Fleet enema can result in diarrhea, which can lead to metabolic acidosis. PTS: 1 DIF: Cognitive Level: Application OBJ: Nursing Process: Implementation MSC: Client Needs: Physiologic Integrity 27. What nursing action is appropriate for specimen collection? a. Follow sterile technique for specimen collection. b. Sterile gloves are worn if the nurse plans to touch the specimen. c. Use Standard Precautions when handling body fluids. d. Avoid wearing gloves in front of the child and family. ANS: C Standard Precautions should always be used when handling body fluids. Specimen collection is not always a sterile procedure. Gloves should be worn if there is a chance the nurse will be contaminated. The choice of sterile or clean gloves will vary according to the procedure or specimen. The child and family should be educated in the purpose of glove use, including the fact that gloves are used with every patient, so that they will not be offended or frightened. PTS: 1 DIF: Cognitive Level: Application OBJ: Nursing Process: Implementation MSC: Client Needs: Physiologic Integrity 28. What information should the nurse include when teaching parents how to care for a child's gastrostomy tube at home? a. Never turn the gastrostomy button. b. Clean around the insertion site daily with soap and water. c. Expect some leakage around the button. d. Remove the tube for cleaning once a week. ANS: B The skin around the tube insertion site should be cleaned with soap and water once or twice daily. The gastrostomy button should be rotated in a full circle during cleaning. Leakage around the tube should be reported to the physician. A gastrostomy tube is placed surgically. It is not removed for cleaning. NURSINGTB.COM PTS: 1 DIF: Cognitive Level: Application OBJ: Nursing Process: Implementation MSC: Client Needs: Physiologic Integrity 29. Which nursing action is the most appropriate when applying a face mask to a child prescribed oxygen therapy? a. Set the oxygen flow rate at less than 6 L/min. b. Make sure the mask fits properly. c. Keep the child warm. d. Remove the mask for 5 minutes every hour. ANS: B A properly fitting face mask is essential for adequate oxygen delivery. The oxygen flow rate should be greater than 6 L/min to prevent rebreathing of exhaled carbon dioxide. Oxygen delivery through a face mask does not affect body temperature. A face mask used for oxygen therapy is not routinely removed. PTS: 1 DIF: Cognitive Level: Comprehension OBJ: Nursing Process: Implementation MSC: Client Needs: Physiologic Integrity 30. What critical information should the nurse incorporate into care when using restraints on a child? a. Use the least restrictive type of restraint. b. Tie knots securely so they cannot be untied easily. c. Secure the ties to the mattress or side rails. d. Remove restraints every 4 hours to assess skin. ANS: A When restraints are necessary, the nurse should institute the least restrictive type of restraint. Knots must be tied so that they can be easily undone for quick access to the child. The ties are never tied to the mattress or side rails. They should be secured to a stable device, such as the bed frame. Restraints are removed every 2 hours to allow for range of motion, position changes, and assessment of skin integrity. PTS: 1 DIF: Cognitive Level: Application OBJ: Nursing Process: Implementation MSC: Client Needs: Physiologic Integrity 31. A 6-year-old child is hospitalized for intravenous (IV) antibiotic therapy. He eats little on his “regular diet” trays. He tells the nurse that all he wants to eat is pizza, tacos, and ice cream. Which is the best nursing action? a. Request these favorite foods for him. b. Identify healthier food choices that he likes. c. Explain that he needs fruits and vegetables. d. Reward him with ice cream at the end of every meal that he eats. ANS: A Loss of appetite is a symptom common to most childhood illnesses. To encourage adequate nutrition, favorite foods should be requested for the child. Even though these substances are not nutritious, they can provide necessary fluid and calories and can be supplemented with additional fruits and vegetables. Ice cream and other desserts should not be used as rewards or punishment. PTS: 1 DIF: CognitiNveURLSevINelG: TABp.pClOicMation OBJ: Nursing Process: Implementation MSC: Client Needs: Physiologic Integrity 32. What procedure is recommended to facilitate a heelstick on an ill neonate to obtain a blood sample? a. Apply cool, moist compresses. b. Apply a tourniquet to the ankle. c. Elevate the foot for 5 minutes. d. Wrap foot in a warm washcloth. ANS: D Before the blood sample is taken, the heel is heated with warm moist compresses for 5 to 10 minutes to dilate the blood vessels in the area. Cooling causes vasoconstriction, making blood collection more difficult. A tourniquet is used to constrict superficial veins. It will have an insignificant effect on capillaries. Elevating the foot will decrease the blood in the foot available for collection. PTS: 1 DIF: Cognitive Level: Application OBJ: Nursing Process: Assessment MSC: Client Needs: Physiologic Integrity 33. After collecting blood by venipuncture in the antecubital fossa, what intervention should the nurse implement in order to assure control of any bleeding? a. Keep arm extended while applying a bandage to the site. b. Keep arm extended, and apply pressure to the site for a few minutes. c. Apply a bandage to the site, and keep the arm flexed for 10 minutes. d. Apply a gauze pad or cotton ball to the site, and keep the arm flexed for several minutes. ANS: B Applying pressure to the site of venipuncture stops the bleeding and aids in coagulation. Pressure should be applied before a bandage is applied. PTS: 1 DIF: Cognitive Level: Application OBJ: Nursing Process: Implementation MSC: Client Needs: Physiologic Integrity: Reduction of Risk Potential 34. Which is the preferred site for intramuscular injections in infants? a. Deltoid b. Dorsogluteal c. Rectus femoris d. Vastus lateralis ANS: D The preferred site for infants is the vastus lateralis. The deltoid and dorsogluteal sites are used for older children and adults. The rectus femoris is not a recommended site. PTS: 1 DIF: Cognitive Level: Comprehension OBJ: Nursing Process: Planning MSC: Client Needs: Physiologic Integrity 35. What nursing consideration is related to the administration of oxygen (O2) in an infant? a. Humidify the oxygen if the infant can tolerate it. b. Assess the infant to determine how much oxygen should be given. c. Arterial oxygen saturation (SaNOU2)RSreINadGiTnBgs.CaOrMe used to guide O2 therapy. d. Direct the oxygen flow so that it blows directly into the infant's face in a hood. ANS: C Pulse oximetry is a continuous, noninvasive method of determining arterial oxygen saturation (SaO2) to guide oxygen therapy. Oxygen is drying to the tissues. Oxygen should always be humidified when delivered to a patient. A child receiving oxygen therapy should have the oxygen saturation monitored at least as frequently as vital signs. Oxygen is a medication, and it is the responsibility of the practitioner to modify dosage as indicated. Humidified oxygen should not be blown directly into an infant's face. PTS: 1 DIF: Cognitive Level: Comprehension OBJ: Nursing Process: Implementation MSC: Client Needs: Physiologic Integrity 36. When administering a gavage feeding to a school-age child, the nurse should implement what intervention to assure safety? a. Lubricate the tip of the feeding tube with Vaseline to facilitate passage. b. Check the placement of the tube by inserting 20 mL of sterile water. c. Administer feedings over 5 to 10 minutes. d. Position the child on the right side after administering the feeding. ANS: D Position the child with the head elevated about 30 degrees and on the right side or abdomen for at least 1 hour. This is in the same manner as after any infant feeding to minimize the possibility of regurgitation and aspiration. Insert a tube that has been lubricated with sterile water or water-soluble lubricant. With a syringe, inject a small amount of air into the tube, while simultaneously listening with a stethoscope over the stomach area. Feedings should be administered via gravity flow and take from 15 to 30 minutes to complete. PTS: 1 DIF: Cognitive Level: Application OBJ: Nursing Process: Implementation MSC: Client Needs: Physiologic Integrity MULTIPLE RESPONSE 1. The nurse is preparing for the admission of an infant who will have several procedures performed. In which situation is informed consent required? (Select all that apply.) a. Catheterized urine collection b. Intravenous (IV) line insertion c. Oxygen administration d. Lumbar puncture e. Bone marrow aspiration ANS: D, E Informed consent is required for invasive procedures that involve risk to a child, such as a lumbar puncture, chest tube insertion, and bone marrow aspirations. Catheterized urine collection, IV line insertion, and oxygen administration all fall under this category. PTS: 1 DIF: Cognitive Level: Application OBJ: Nursing Process: Planning MSC: Client Needs: Safe and EffectNivUeRCSaIrNeGETnBv.iCroOnMment 2. The advantages of the ventrogluteal muscle as an injection site in young children include which of the following? (Select all that apply.) a. Less painful than vastus lateralis b. Free of important nerves and vascular structures c. Cannot be used when child reaches a weight of 20 lbs d. Increased subcutaneous fat, which increases drug absorption e. Easily identified by major landmarks ANS: A, B, E Less painful, free of important nerves and vascular structures, and easily identifiable are advantages of the ventrogluteal muscle. The major disadvantage is lack of familiarity by health professionals and controversy over whether the site can be used before weight bearing. Cannot be used when a child is 20 lbs or more and increased subcutaneous fat are not advantages of the ventrogluteal muscle as an injection site in young children. PTS: 1 DIF: Cognitive Level: Comprehension OBJ: Nursing Process: Implementation MSC: Client Needs: Physiologic Integrity COMPLETION 1. A child with congestive heart failure is placed on a maintenance dosage of digoxin. The dosage is 0.07 mg/kg/day, and the child's weight is 7.2 kg. The physician prescribes the digoxin to be given once a day by mouth. Each dose will be mg. Record your answer using one decimal place. ANS: 0.5 Calculate the dosage by weight: 0.07 mg/day  7.2 kg = 0.5 mg/day. PTS: 1 DIF: Cognitive Level: Analysis OBJ: Nursing Process: Implementation MSC: Client Needs: Physiologic Integrity MATCHING The nurse is preparing to insert a nasogastric tube into a 4-year-old child for intermittent suctioning after abdominal surgery. Place in correct sequence the steps for inserting a nasogastric tube. a. Lubricate the nasogastric tube with water-soluble lubricant. b. Tape the nasogastric tube securely to the child's face. c. Check the placement of the tube by aspirating stomach contents. d. Place the child in the supine position with head slightly hyperflexed. e. Insert the nasogastric tube through the nares. f. Measure the tube from the tip of the nose to the earlobe to the midpoint between the xiphoid process and the umbilicus. 1. Step one 2. Step two 3. Step three 4. Step four 5. Step five 6. Step six NURSINGTB.COM 1. ANS: D PTS: 1 DIF: Cognitive Level: Application OBJ: Nursing Process: Implementation MSC: Client Needs: Physiologic Integrity NOT: First step is the appropriate patient positioning. 2. ANS: F PTS: 1 DIF: Cognitive Level: Application OBJ: Nursing Process: Implementation MSC: Client Needs: Physiologic Integrity NOT: Second step is to measure tube for appropriate insertion length. 3. ANS: A PTS: 1 DIF: Cognitive Level: Application OBJ: Nursing Process: Implementation MSC: Client Needs: Physiologic Integrity NOT: Third step involves lubricating the tube in preparation for insertion to facilitate insertion. 4. ANS: E PTS: 1 DIF: Cognitive Level: Application OBJ: Nursing Process: Implementation MSC: Client Needs: Physiologic Integrity NOT: Fourth step is the insertion of the tube 5. ANS: C PTS: 1 DIF: Cognitive Level: Application OBJ: Nursing Process: Implementation MSC: Client Needs: Physiologic Integrity NOT: Fifth step the tube should be checked for correct placement to prevent aspiration. 6. ANS: B PTS: 1 DIF: Cognitive Level: Application OBJ: Nursing Process: Implementation MSC: Client Needs: Physiologic Integrity NOT: Sixth step involves appropriate securing of the tube to the child's face. NURSINGTB.COM Chapter 40: The Child With Respiratory Dysfunction Perry: Maternal Child Nursing Care, 6th Edition MULTIPLE CHOICE 1. Which statement best describes why children have fewer respiratory tract infections as they grow older? a. The amount of lymphoid tissue decreases. b. Repeated exposure to organisms causes increased immunity. c. Viral organisms are less prevalent in the population. d. Secondary infections rarely occur after viral illnesses. ANS: B Children have increased immunity after exposure to a virus. The amount of lymphoid tissue increases as children grow older. Viral organisms are not less prevalent, but older children have the ability to resist invading organisms. Secondary infections after viral illnesses include Mycoplasma pneumoniae and groups A and B streptococcal infections. PTS: 1 DIF: Cognitive Level: Comprehension OBJ: Nursing Process: Assessment MSC: Client Needs: Physiologic Integrity 2. Instructions for decongestant nose drops should include what recommendation? a. Avoiding use for more than 3 days. b. Keeping drops to use again for nasal congestion. c. Administering drops until nasal congestion subsides. d. Administering drops after feedNiUngRsSIaNnGdTaBt .bCeOdMtime. ANS: A Vasoconstrictive nose drops should not be used for more than 3 days to avoid rebound congestion. Drops should be discarded after one illness because they may become contaminated with bacteria. Vasoconstrictive nose drops can have a rebound effect after 3 days of use. Drops administered before feedings are more helpful. PTS: 1 DIF: Cognitive Level: Comprehension OBJ: Nursing Process: Implementation MSC: Client Needs: Physiologic Integrity 3. When caring for an infant with an upper respiratory tract infection and elevated temperature, which appropriate nursing intervention should the nurse implement? a. Give tepid water baths to reduce fever. b. Encourage food intake to maintain caloric needs. c. Have child wear heavy clothing to prevent chilling. d. Give small amounts of favorite fluids frequently to prevent dehydration. ANS: D Preventing dehydration by small frequent feedings is an important intervention in the febrile child. Tepid water baths may induce shivering, which raises temperature. Food should not be forced; it may result in the child vomiting. The febrile child should be dressed in light, loose clothing. PTS: 1 DIF: Cognitive Level: Application OBJ: Nursing Process: Implementation MSC: Client Needs: Physiologic Integrity 4. When should the parent of an infant with nasopharyngitis be instructed to notify the health care professional? a. Becomes fussy b. Has a cough c. Has a fever over 99° F d. Shows signs of an earache ANS: D If an infant with nasopharyngitis has a fever over 101° F, there is early evidence of respiratory complications. Irritability and a slight fever are common in an infant with a viral illness. Cough can be a sign of nasopharyngitis. PTS: 1 DIF: Cognitive Level: Application OBJ: Nursing Process: Implementation MSC: Client Needs: Physiologic Integrity 5. When it is generally recommended that a child being treated for acute streptococcal pharyngitis may return to school? a. When the sore throat is better b. If no complications develop c. After taking antibiotics for 24 hours d. After taking antibiotics for 3 days ANS: C After children have taken antibiotics for 24 hours, even if the sore throat persists, they are no longer contagious to other children. Complications may take days to weeks to develop. PTS: 1 DIF: CognitiNveURLSevINelG: TCBo.mCOprMehension OBJ: Nursing Process: Implementation MSC: Client Needs: Physiologic Integrity 6. An 8-year-old child is diagnosed with influenza, probably type A disease. What intervention should be included in the plan of care? a. Clear liquid diet for hydration b. Aspirin to control fever c. Oseltamivir to reduce symptoms d. Antibiotics to prevent bacterial infection ANS: C Oseltamivir may reduce symptoms related to influenza type A if administered within 48 hours of onset of symptoms. It is effective against type A or B. A clear liquid diet is not necessary for influenza, but maintaining hydration is important. Aspirin is not recommended in children because of increased risk of Reye's syndrome. Acetaminophen or ibuprofen is a better choice. Preventive antibiotics are not indicated for influenza unless there is evidence of a secondary bacterial infection. PTS: 1 DIF: Cognitive Level: Application OBJ: Nursing Process: Implementation MSC: Client Needs: Physiologic Integrity 7. A nurse providing care to a child diagnosed with chronic otitis media with effusion (OME) will assess for which sign/symptom? a. Fever as high as 40° C (104° F) b. Severe pain in the ear c. Nausea and vomiting d. A feeling of fullness in the ear ANS: D OME is characterized by an immobile or orange-discolored tympanic membrane and nonspecific complaints of fullness in the ear. OME does not generally cause severe pain. Fever and severe pain may be signs of AOM. Nausea and vomiting are associated with otitis media. PTS: 1 DIF: Cognitive Level: Application OBJ: Nursing Process: Diagnosis MSC: Client Needs: Physiologic Integrity 8. An infant's parents ask the nurse about preventing otitis media (OM). What intervention should the nurse recommend? a. Avoid tobacco smoke b. Use nasal decongestant c. Avoid children with OM d. Bottle-feed or breastfeed in supine position ANS: A Eliminating tobacco smoke from the child's environment is essential for preventing OM and other common childhood illnesses. Nasal decongestants are not useful in preventing OM. Children with uncomplicated OM are not contagious unless they show other upper respiratory infection symptoms. Children should be fed in an upright position to prevent OM. PTS: 1 DIF: Cognitive Level: Application OBJ: Nursing Process: Implementation MSC: Client Needs: Physiologic Integrity NURSINGTB.COM 9. Which type of croup is always considered a medical emergency? a. Laryngitis b. Epiglottitis c. Spasmodic croup d. Laryngotracheobronchitis (LTB) ANS: B Epiglottitis is always a medical emergency needing antibiotics and airway support for treatment. Laryngitis is a common viral illness in older children and adolescents with hoarseness and upper respiratory infection symptoms. Spasmodic croup is treated with humidity. LTB may progress to a medical emergency in some children. PTS: 1 DIF: Cognitive Level: Comprehension OBJ: Nursing Process: Assessment MSC: Client Needs: Physiologic Integrity 10. The nurse encourages the mother of a toddler with acute laryngotracheobronchitis (LTB) to stay at the bedside as much as possible. What is the nurse's primary rationale for this action? a. Mothers of hospitalized toddlers often experience guilt. b. The mother's presence will reduce anxiety and ease the child's respiratory efforts. c. Separation from the mother is a major developmental threat at this age. d. The mother can provide constant observations of the child's respiratory efforts. ANS: B The family's presence will decrease the child's distress. The mother may experience guilt, but this is not the best answer. Although separation from the mother is a developmental threat for toddlers, the main reason to keep parents at the child's bedside is to ease anxiety and therefore respiratory effort. The child should have constant cardiorespiratory monitoring and noninvasive oxygen saturation monitoring, but the parent should not play this role in the hospital. PTS: 1 DIF: Cognitive Level: Application OBJ: Nursing Process: Implementation MSC: Client Needs: Psychosocial Integrity 11. A school-age child has had an upper respiratory tract infection for several days and then began having a persistent dry, hacking cough that was worse at night. The cough has become productive in the past 24 hours. This assessment is most suggestive of what respiratory airway disorder? a. Bronchitis b. Bronchiolitis c. Viral-induced asthma d. Acute spasmodic laryngitis ANS: A Bronchitis is characterized by these symptoms and occurs in children older than 6 years. Bronchiolitis is rare in children older than 2 years. Asthma is a chronic inflammation of the airways that may be exacerbated by a virus. Acute spasmodic laryngitis occurs in children between 3 months and 3 years. PTS: 1 DIF: Cognitive Level: Comprehension OBJ: Nursing Process: Diagnosis MSC: Client Needs: Physiologic Integrity NURSINGTB.COM 12. The nurse is caring for a child diagnosed with acute respiratory distress syndrome (ARDS) associated with sepsis. What nursing intervention should be included in the plan of care? a. Force fluids b. Monitor pulse oximetry c. Institute seizure precautions d. Encourage a high-protein diet ANS: B Monitoring cardiopulmonary status is an important evaluation tool in the care of the child with ARDS. Maintenance of vascular volume and hydration is important and should be done parenterally. Seizures are not a side effect of ARDS. Adequate nutrition is necessary, but a high-protein diet is not helpful. PTS: 1 DIF: Cognitive Level: Application OBJ: Nursing Process: Implementation MSC: Client Needs: Physiologic Integrity 13. The nurse is caring for a child with carbon monoxide (CO) poisoning associated with smoke inhalation. What intervention is essential in this child's care? a. Monitor pulse oximetry b. Monitor arterial blood gases (ABGs) c. Administer oxygen if respiratory distress develops d. Administer oxygen if child's lips become bright, cherry red ANS: B Arterial blood gases (ABGs) and COHb levels are the best way to monitor CO poisoning. PaO2 monitored with pulse oximetry may be normal in the case of CO poisoning. Oxygen at 100% should be given as quickly as possible, not only if respiratory distress or other symptoms develop. PTS: 1 DIF: Cognitive Level: Comprehension OBJ: Nursing Process: Assessment MSC: Client Needs: Physiologic Integrity 14. A child has a chronic, nonproductive cough and diffuse wheezing during the expiratory phase of respiration. This suggests which respiratory condition? a. Asthma b. Pneumonia c. Bronchiolitis d. Foreign body in the trachea ANS: A Children with asthma usually have these chronic symptoms. Pneumonia appears with an acute onset and fever and general malaise. Bronchiolitis is an acute condition caused by respiratory syncytial virus. Foreign body in the trachea will manifest with acute respiratory distress or failure and maybe stridor. PTS: 1 DIF: Cognitive Level: Comprehension OBJ: Nursing Process: Diagnosis MSC: Client Needs: Physiologic Integrity 15. It is now recommended that children with asthma who are taking long-term inhaled steroids should be assessed frequently to monitor for what increased risk? a. Cough b. Osteoporosis c. Slowed growth d. Cushing's syndrome ANS: C NURSINGTB.COM The growth of children on long-term inhaled steroids should be assessed frequently to assess for systemic effects of these drugs. Cough is prevented by inhaled steroids. No evidence exists that inhaled steroids cause osteoporosis. Cushing's syndrome is caused by long-term systemic steroids. PTS: 1 DIF: Cognitive Level: Comprehension OBJ: Nursing Process: Diagnosis MSC: Client Needs: Physiologic Integrity 16. -Adrenergic agonists and methylxanthines are often prescribed for a child with an asthma attack for what resulting action? a. Liquefaction of secretions b. Dilation of the bronchioles c. Reduction of inflammation of the lungs d. Reduction of existing infection ANS: B These medications work to dilate the bronchioles in acute exacerbations. These medications do not liquefy secretions or reduce infection. Corticosteroids and mast cell stabilizers reduce inflammation in the lungs. PTS: 1 DIF: Cognitive Level: Comprehension OBJ: Nursing Process: Implementation MSC: Client Needs: Physiologic Integrity 17. A parent whose two school-age children diagnosed with exercise-induced bronchospasm (EIB) asks the nurse in what sports, if any, they can participate. The nurse should recommend which sport? a. Soccer b. Running c. Swimming d. Basketball ANS: C Swimming is well tolerated in children with EIB because they are breathing air fully saturated with moisture and because of the type of breathing required in swimming. Exercise-induced bronchospasm is more common in sports that involve endurance, such as soccer, running, and basketball. Prophylaxis with medications may be necessary. PTS: 1 DIF: Cognitive Level: Application OBJ: Nursing Process: Planning MSC: Client Needs: Physiologic Integrity 18. Which statement expresses accurately the genetic implications of cystic fibrosis (CF)? a. If it is present in a child, both parents are carriers of this defective gene. b. It is inherited as an autosomal dominant trait. c. It is a genetic defect found primarily in non-Caucasian population groups. d. There is a 50% chance that siblings of an affected child also will be affected. ANS: A CF is an autosomal recessive geneNiUnRhSeIrNitGedTBfr.ComOMboth parents and is found primarily in Caucasian populations. An autosomal recessive inheritance pattern means that there is a 25% chance that a sibling will be infected but a 50% chance a sibling will be a carrier. PTS: 1 DIF: Cognitive Level: Comprehension OBJ: Nursing Process: Assessment MSC: Client Needs: Physiologic Integrity 19. What are the earliest recognizable clinical manifestations of cystic fibrosis (CF)? a. Meconium ileus b. History of poor intestinal absorption c. Foul-smelling, frothy, greasy stools d. Recurrent pneumonia and lung infections ANS: A The earliest clinical manifestation of CF is a meconium ileus, which is found in about 10% of children with CF. Clinical manifestations include abdominal distention, vomiting, failure to pass stools, and rapid development of dehydration. History of malabsorption is a later sign that manifests as failure to thrive. Foul-smelling stools and recurrent respiratory infections are later manifestations of CF. PTS: 1 DIF: Cognitive Level: Comprehension OBJ: Nursing Process: Assessment MSC: Client Needs: Physiologic Integrity 20. Cystic fibrosis (CF) is suspected in a toddler. Which test is essential in establishing this diagnosis? a. Bronchoscopy b. Serum calcium c. Urine creatinine d. Sweat chloride test ANS: D A sweat chloride test result greater than 60 mEq/L is diagnostic of CF. Although bronchoscopy is helpful for identifying bacterial infection in children with CF, it is not diagnostic. Serum calcium is normal in children with CF. Urine creatinine is not diagnostic of CF. PTS: 1 DIF: Cognitive Level: Comprehension OBJ: Nursing Process: Assessment MSC: Client Needs: Physiologic Integrity 21. A child diagnosed with cystic fibrosis is prescribed recombinant human deoxyribonuclease (rhDNase). What information should be included in the medication education provided the child and family? a. May cause mucus to thicken b. May cause minor voice alterations c. Is given subcutaneously d. Is not indicated for children younger than 12 years ANS: B Two of the only adverse effects of rhDNase are voice alterations and laryngitis. rhDNase decreases viscosity of mucus, is given in an aerosolized form, and is safe for children younger than 12 years of age. PTS: 1 DIF: CognitiNveURLSevINelG: TABp.pClOicMation OBJ: Nursing Process: Implementation MSC: Client Needs: Physiologic Integrity 22. Pancreatic enzymes are administered to the child with cystic fibrosis. What information should be included in patient education concerning the administration of these enzymes? a. Do not administer pancreatic enzymes if the child is receiving antibiotics. b. Decrease dose of pancreatic enzymes if the child is having frequent, bulky stools. c. Administer pancreatic enzymes between meals if at all possible. d. Pancreatic enzymes can be swallowed whole or sprinkled on a small amount of food taken at the beginning of a meal. ANS: D Enzymes may be administered in a small amount of cereal or fruit or swallowed whole at the beginning of a meal, not between meals. Pancreatic enzymes are not contraindicated with antibiotics. The dose of enzymes should be increased if the child is having frequent, bulky stools. PTS: 1 DIF: Cognitive Level: Application OBJ: Nursing Process: Implementation MSC: Client Needs: Physiologic Integrity 23. In providing nourishment for a child with cystic fibrosis (CF), what diet consideration should be stressed to both the child and caregivers? a. Diet should be high in carbohydrates and protein. b. Diet should be high in easily digested carbohydrates and fats. c. Most fruits and vegetables are not well tolerated. d. Fats and proteins must be greatly curtailed. ANS: A Children with CF require a well-balanced, high-protein, high-calorie diet because of impaired intestinal absorption. Enzyme supplementation helps digest foods; other modifications are not necessary. A well-balanced diet containing fruits and vegetables is important. Fats and proteins are a necessary part of a well-balanced diet. PTS: 1 DIF: Cognitive Level: Application OBJ: Nursing Process: Implementation MSC: Client Needs: Physiologic Integrity 24. Abdominal thrusts are recommended for airway obstruction in children older than: a. 1 year. b. 4 years. c. 8 years. d. 12 years. ANS: A Abdominal thrusts are recommended for airway obstruction in children older than 1 year. In children younger than 1 year, back blows and chest thrusts are administered. PTS: 1 DIF: Cognitive Level: Comprehension OBJ: Nursing Process: Implementation MSC: Client Needs: Physiologic Integrity 25. What nursing intervention should be included in the plan of care for a young child diagnosed with pneumonia? a. Monitor for abdominal pain b. Encourage the child to lie on tNheURuSnIaNffGeTcBte.CdOsMide c. Administer analgesics d. Place the child in the Trendelenburg position ANS: A The pain of pneumonia may be referred to the abdomen in young children. Lying on the affected side may promote comfort by splinting the chest and reducing pleural rubbing. Analgesics are not indicated. Children should be placed in a semierect position or position of comfort. PTS: 1 DIF: Cognitive Level: Application OBJ: Nursing Process: Implementation MSC: Client Needs: Physiologic Integrity 26. What distinguishing manifestation of spasmodic croup should parents be taught to identify? a. Wheezing is heard audibly b. It has a harsh, barky cough c. It is bacterial in nature d. The child has a high fever ANS: B Spasmodic croup is viral in origin, is usually preceded by several days of symptoms of upper respiratory tract infection, and often begins at night. It is marked by a harsh, metallic, barky cough; sore throat; inspiratory stridor; and hoarseness. Wheezing is not a distinguishing manifestation of croup. It can accompany conditions such as asthma or bronchiolitis. A high fever is not usually present. PTS: 1 DIF: Cognitive Level: Application OBJ: Nursing Process: Assessment MSC: Client Needs: Physiologic Integrity 27. Which intervention for treating croup at home should be taught to parents? a. Have a decongestant available to give the child when an attack occurs. b. Have the child sleep in a dry room. c. Take the child outside if air is cool and moist. d. Give the child an antibiotic at bedtime. ANS: C Taking the child into the cool, humid, night air may relieve mucosal swelling and improve symptoms. Decongestants are inappropriate for croup, which affects the middle airway level. A dry environment may contribute to symptoms. Croup is caused by a virus. Antibiotic treatment is not indicated. PTS: 1 DIF: Cognitive Level: Application OBJ: Nursing Process: Implementation MSC: Client Needs: Health Promotion and Maintenance 28. Which information should the nurse stress to workers at a day care center about respiratory syncytial virus (RSV)? a. RSV is transmitted through particles in the air. b. RSV can live on skin or paper for up to a few seconds after contact. c. RSV can survive on nonporous surfaces for about 60 minutes. d. Frequent hand washing can decrease the spread of the virus. ANS: D Meticulous hand washing can decNreUaRseSItNhGe TsBpr.CeaOdMof organisms. RSV infection is not airborne. It is acquired mainly through contact with contaminated surfaces. RSV can live on skin or paper for up to 1 hour and on cribs and other nonporous surfaces for up to 6 hours. PTS: 1 DIF: Cognitive Level: Application OBJ: Nursing Process: Implementation MSC: Client Needs: Safe and Effective Care Environment 29. Which vitamin supplements are necessary for children with cystic fibrosis? a. Vitamin C and calcium b. Vitamins B6 and B12 c. Magnesium d. Vitamins A, D, E, and K ANS: D Fat-soluble vitamins are poorly absorbed because of deficient pancreatic enzymes in children with cystic fibrosis; therefore, supplements are necessary. Vitamin C and calcium are not fat soluble. Vitamins B6 and B12 are not fat-soluble vitamins. Magnesium is a mineral, not a vitamin. PTS: 1 DIF: Cognitive Level: Comprehension OBJ: Nursing Process: Assessment MSC: Client Needs: Physiologic Integrity 30. Why do infants and young children quickly have respiratory distress in acute and chronic alterations of the respiratory system? a. They have a widened, shorter airway. b. There is a defect in their sucking ability. c. The gag reflex increases mucus production. d. Mucus and edema obstruct small airways. ANS: D The airway in infants and young children is narrower, not wider, and respiratory distress can occur quickly because mucus and edema can cause obstruction to their small airways. Sucking is not necessarily related to problems with the airway. The gag reflex is necessary to prevent aspiration. It does not produce mucus. PTS: 1 DIF: Cognitive Level: Comprehension OBJ: Nursing Process: Assessment MSC: Client Needs: Physiologic Integrity 31. A nurse is charting that a hospitalized child has labored breathing. Which medical term describes labored breathing? a. Dyspnea b. Tachypnea c. Hypopnea d. Orthopnea ANS: A Dyspnea is labored breathing. Tachypnea is rapid breathing. Hypopnea is breathing that is too shallow. Orthopnea is difficulty breathing except in upright position. PTS: 1 DIF: Cognitive Level: Knowledge OBJ: Nursing Process: Assessment MSC: Client Needs: Physiologic Integrity NURSINGTB.COM 32. Parents have understood teaching about prevention of childhood otitis media if they make which statement? a. “We will only prop the bottle during the daytime feedings.” b. “Breastfeeding will be discontinued after 4 months of age.” c. “We will place the child flat right after feedings.” d. “We will be sure to keep immunizations up to date.” ANS: D Parents have understood the teaching about preventing childhood otitis media if they respond they will keep childhood immunizations up to date. The child should be maintained upright during feedings and after. Otitis media can be prevented by exclusively breastfeeding until at least 6 months of age. Propping bottles is discouraged to avoid pooling of milk while the child is in the supine position. PTS: 1 DIF: Cognitive Level: Analysis OBJ: Nursing Process: Implementation MSC: Client Needs: Health Promotion and Maintenance 33. An 18-month-old child is seen in the clinic is diagnosed with acute otitis media (AOM). Oral amoxicillin is prescribed. Which statement made by the parent indicates a correct understanding of the instructions? a. “I should administer all the prescribed medication.” b. “I should continue medication until the symptoms subside.” c. “I will immediately stop giving medication if I notice a change in hearing.” d. “I will stop giving medication if fever is still present in 24 hours.” ANS: A Antibiotics should be given for their full course to prevent recurrence of infection with resistant bacteria. Symptoms may subside before the full course is given. Hearing loss is a complication of AOM. Antibiotics should continue to be given. Medication may take 24 to 48 hours to make symptoms subside. It should be continued. PTS: 1 DIF: Cognitive Level: Analysis OBJ: Nursing Process: Implementation MSC: Client Needs: Physiologic Integrity 34. The nurse is assessing a child with acute epiglottitis. Examining the child's throat by using a tongue depressor might precipitate which symptom or condition? a. Inspiratory stridor b. Complete obstruction c. Sore throat d. Respiratory tract infection ANS: B If a child has acute epiglottitis, examination of the throat may cause complete obstruction and should be performed only when immediate intubation can take place. Stridor is aggravated when a child with epiglottitis is supine. Sore throat and pain on swallowing are early signs of epiglottitis. Epiglottitis is caused by Haemophilus influenzae in the respiratory tract. PTS: 1 DIF: Cognitive Level: Comprehension OBJ: Nursing Process: Assessment MSC: Client Needs: Physiologic Integrity 35. A nurse is conducting an in-service on asthma. Which statement is the most descriptive of bronchial asthma? NURSINGTB.COM a. There is heightened airway reactivity. b. There is decreased resistance in the airway. c. The single cause of asthma is an allergic hypersensitivity. d. It is inherited. ANS: A In bronchial asthma, spasm of the smooth muscle of the bronchi and bronchioles causes constriction, producing impaired respiratory function. In bronchial asthma, there is increased resistance in the airway. There are multiple causes of asthma, including allergens, irritants, exercise, cold air, infections, medications, medical conditions, and endocrine factors. Atopy or development of an immunoglobulin E (IgE)-mediated response is inherited but is not the only cause of asthma. PTS: 1 DIF: Cognitive Level: Comprehension OBJ: Nursing Process: Implementation MSC: Client Needs: Physiologic Integrity 36. A child with cystic fibrosis (CF) receives aerosolized bronchodilator medication. When should this medication be administered? a. Before chest physiotherapy (CPT) b. After CPT c. Before receiving 100% oxygen d. After receiving 100% oxygen ANS: A Bronchodilators should be given before CPT to open bronchi and make expectoration easier. Aerosolized bronchodilator medications are not helpful when used after CPT. Oxygen administration is necessary only in acute episodes with caution because of chronic carbon dioxide retention. PTS: 1 DIF: Cognitive Level: Application OBJ: Nursing Process: Implementation MSC: Client Needs: Physiologic Integrity MULTIPLE RESPONSE 1. The nurse is caring for a 10-month-old infant diagnosed with respiratory syncytial virus (RSV) bronchiolitis. Which interventions should be included in the child's care? (Select all that apply.) a. Administer antibiotics b. Administer cough syrup c. Encourage infant to drink 8 ounces of formula every 4 hours d. Institute cluster care to encourage adequate rest e. Place on noninvasive oxygen monitoring ANS: C, D, E Hydration is important in children with RSV bronchiolitis to loosen secretions and prevent shock. Clustering of care promotes periods of rest. The use of noninvasive oxygen monitoring is recommended. PTS: 1 DIF: Cognitive Level: Application OBJ: Nursing Process: Implementation MSC: Client Needs: Physiologic Integrity NURSINGTB.COM 2. Which information should the nurse teach families about reducing exposure to pollens and dust? (Select all that apply.) a. Replace wall-to-wall carpeting with wood and tile floors b. Use an air conditioner c. Put dust-proof covers on pillows and mattresses d. Keep humidity in the house above 60% e. Keep pets outside ANS: A, B, C Carpets retain dust. To reduce exposure to dust, carpeting should be replaced with wood, tile, slate, or vinyl. These floors can be cleaned easily. For anyone with pollen allergies, it is best to keep the windows closed and to run the air conditioner. Covering mattresses and pillows with dust-proof covers will reduce exposure to dust. A humidity level above 60% promotes dust mites. It is recommended that household humidity be kept between 40% and 50% to reduce dust mites inside the house. Keeping pets outside will help to decrease exposure to dander, but will not affect exposure to pollen and dust. PTS: 1 DIF: Cognitive Level: Analysis OBJ: Nursing Process: Implementation MSC: Client Needs: Health Promotion and Maintenance Chapter 41: The Child With Gastrointestinal Dysfunction Perry: Maternal Child Nursing Care, 6th Edition MULTIPLE CHOICE 1. Nurses must be alert for increased fluid requirements when a child presents with which possible concern? a. Fever b. Mechanical ventilation c. Congestive heart failure d. Increased intracranial pressure (ICP) ANS: A Fever leads to great insensible fluid loss in young children because of increased body surface area relative to fluid volume. Respiratory rate influences insensible fluid loss and should be monitored in the mechanically ventilated child. Congestive heart failure is a case of fluid overload in children. ICP does not lead to increased fluid requirements in children. PTS: 1 DIF: Cognitive Level: Application OBJ: Nursing Process: Assessment MSC: Client Needs: Physiologic Integrity 2. Which type of dehydration results from water loss in excess of electrolyte loss? a. Isotonic dehydration b. Isosmotic dehydration c. Hypotonic dehydration d. Hypertonic dehydration ANS: D NURSINGTB.COM Hypertonic dehydration results from water loss in excess of electrolyte loss. This is the most dangerous type of dehydration. It is caused by feeding children fluids with high amounts of solute. Isotonic dehydration occurs in conditions in which electrolyte and water deficits are present in balanced proportion. Isosmotic dehydration is another term for isotonic dehydration. Hypotonic dehydration occurs when the electrolyte deficit exceeds the water deficit, leaving the serum hypotonic. PTS: 1 DIF: Cognitive Level: Comprehension OBJ: Nursing Process: Assessment MSC: Client Needs: Physiologic Integrity 3. An infant is brought to the emergency department with poor skin turgor, sunken fontanel, lethargy, and tachycardia. This is suggestive of which condition? a. Overhydration b. Dehydration c. Sodium excess d. Calcium excess ANS: B These clinical manifestations indicate dehydration. Symptoms of overhydration are edema and weight gain. Regardless of extracellular sodium levels, total body sodium is usually depleted in dehydration. Symptoms of hypocalcemia are a result of neuromuscular irritability and manifest as jitteriness, tetany, tremors, and muscle twitching. PTS: 1 DIF: Cognitive Level: Comprehension OBJ: Nursing Process: Assessment MSC: Client Needs: Physiologic Integrity 4. What is a common cause of acute diarrhea? a. Hirschsprung's disease b. Antibiotic therapy c. Hypothyroidism d. Meconium ileus ANS: B Acute diarrhea is a sudden increase in frequency and change in consistency of stools and may be associated with antibiotic therapy. Hirschsprung's disease, hypothyroidism, and meconium ileus are usually manifested with constipation rather than diarrhea. PTS: 1 DIF: Cognitive Level: Comprehension OBJ: Nursing Process: Assessment MSC: Client Needs: Physiologic Integrity 5. The viral pathogen that frequently causes acute diarrhea in young children is: a. Giardia organisms. b. Shigella organisms. c. Rotavirus. d. Salmonella organisms. ANS: C Rotavirus is the most frequent viral pathogen that causes diarrhea in young children. Giardia and Salmonella are bacterial pathogens that cause diarrhea. Shigella is a bacterial pathogen that is uncommon in the United StNaUteRsS. INGTB.COM PTS: 1 DIF: Cognitive Level: Comprehension OBJ: Nursing Process: Assessment MSC: Client Needs: Physiologic Integrity 6. A stool specimen from a child with diarrhea shows the presence of neutrophils and red blood cells. This is most suggestive of which condition? a. Protein intolerance b. Parasitic infection c. Fat malabsorption d. Bacterial gastroenteritis ANS: D Neutrophils and red blood cells in stool indicate bacterial gastroenteritis. Protein intolerance is suspected in the presence of eosinophils. Parasitic infection is indicated by eosinophils. Fat malabsorption is indicated by foul-smelling, greasy, bulky stools. PTS: 1 DIF: Cognitive Level: Comprehension OBJ: Nursing Process: Assessment MSC: Client Needs: Physiologic Integrity 7. Therapeutic management of the child with acute diarrhea and dehydration usually begins with what intervention? a. Clear liquids b. Adsorbents such as kaolin and pectin c. Oral rehydration solution (ORS) d. Antidiarrheal medications such as paregoric ANS: C ORS is the first treatment for acute diarrhea. Clear liquids are not recommended because they contain too much sugar, which may contribute to diarrhea. Adsorbents are not recommended and neither are antidiarrheal because they do not get rid of pathogens. PTS: 1 DIF: Cognitive Level: Application OBJ: Nursing Process: Assessment MSC: Client Needs: Physiologic Integrity 8. A young child is brought to the emergency department with severe dehydration secondary to acute diarrhea and vomiting. Therapeutic management of this child will begin with which intervention? a. Intravenous fluids b. Oral rehydration solution (ORS) c. Clear liquids, 1 to 2 ounces at a time d. Administration of antidiarrheal medication ANS: A Intravenous fluids are initiated in children with severe dehydration. ORS is acceptable therapy if the dehydration is not severe. Diarrhea is not managed by using clear liquids by mouth. These fluids have a high carbohydrate content, low electrolyte content, and high osmolality. Antidiarrheal medications are not recommended for the treatment of acute infectious diarrhea. PTS: 1 DIF: Cognitive Level: Application OBJ: Nursing Process: Implementation MSC: Client Needs: Physiologic Integrity 9. Constipation has recently becomeNaUpRrSoIbNlGemTBf.oCrOaMschool-age child who is being treated for seasonal allergies. The nurse should focus the assessment on what possibly related factor? a. Diet b. Allergies c. Antihistamines d. Emotional factors ANS: C Constipation may be associated with drugs such as antihistamines, antacids, diuretics, opioids, antiepileptics, and iron. Because this is the only known recent change in her habits, the addition of antihistamines is most likely the etiology of the diarrhea, rather than diet, allergies, or emotional factors. With a change in bowel habits, the presence and role of any recently prescribed medications should be assessed. PTS: 1 DIF: Cognitive Level: Comprehension OBJ: Nursing Process: Assessment MSC: Client Needs: Physiologic Integrity 10. Therapeutic management of most children with Hirschsprung's disease is primarily: a. daily enemas. b. low-fiber diet. c. permanent colostomy. d. surgical removal of affected section of bowel. ANS: D Most children with Hirschsprung's disease require surgical rather than medical management. Surgery is done to remove the aganglionic portion of the bowel, relieve obstruction, and restore normal bowel motility and function of the internal anal sphincter. Preoperative management may include enemas and low-fiber, high-calorie, high-protein diet until the child is physically ready for surgery. The colostomy that is created in Hirschsprung's disease is usually temporary. PTS: 1 DIF: Cognitive Level: Comprehension OBJ: Nursing Process: Implementation MSC: Client Needs: Physiologic Integrity 11. A 4-month-old infant diagnosed with gastroesophageal reflux disease (GERD) is thriving without other complications. What should the nurse suggest to minimize reflux? a. Place in Trendelenburg position after eating. b. Thicken formula with rice cereal. c. Give continuous nasogastric tube feedings. d. Give larger, less frequent feedings. ANS: B Giving small frequent feedings of formula combined with 1 teaspoon to 1 tablespoon of rice cereal per ounce of formula has been recommended. Milk thickening agents have been shown to decrease the number of episodes of vomiting and increase the caloric density of the formula. This may benefit infants who are underweight as a result of GERD. Placing the child in Trendelenburg position would increase the reflux. Continuous nasogastric feedings are reserved for infants with severe reflux and failure to thrive. Smaller, more frequent feedings are recommended in reflux. PTS: 1 DIF: Cognitive Level: Application OBJ: Nursing Process: ImplementaNtioUnRSINMGSTCB.:CCOlMient Needs: Physiologic Integrity 12. What is the primary purpose of prescribing a histamine receptor antagonist for an infant diagnosed with gastroesophageal reflux? a. Prevent reflux b. Prevent hematemesis. c. Reduce gastric acid production. d. Increase gastric acid production. ANS: C The mechanism of action of histamine receptor antagonists is to reduce the amount of acid present in gastric contents and may prevent esophagitis. None of the remaining options are modes of action of histamine receptor antagonists but rather desired effects of medication therapy. PTS: 1 DIF: Cognitive Level: Comprehension OBJ: Nursing Process: Implementation MSC: Client Needs: Physiologic Integrity 13. Which clinical manifestation would most suggest acute appendicitis? a. Rebound tenderness b. Bright red or dark red rectal bleeding c. Abdominal pain that is relieved by eating d. Abdominal pain that is most intense at McBurney point ANS: D Pain is the cardinal feature. It is initially generalized and usually periumbilical. The pain localizes to the right lower quadrant at McBurney point. Rebound tenderness is not a reliable sign and is extremely painful to the child. Abdominal pain that is relieved by eating and bright or dark red rectal bleeding are not signs of acute appendicitis. PTS: 1 DIF: Cognitive Level: Comprehension OBJ: Nursing Process: Assessment MSC: Client Needs: Physiologic Integrity 14. When caring for a child with probable appendicitis, the nurse should be alert to recognize what sign of perforation? a. Bradycardia b. Anorexia c. Sudden relief from pain d. Decreased abdominal distention ANS: C Signs of peritonitis, in addition to fever, include sudden relief from pain after perforation. Tachycardia, not bradycardia, is a manifestation of peritonitis. Anorexia is already a clinical manifestation of appendicitis. Abdominal distention usually increases in addition to an increase in pain (usually diffuse and accompanied by rigid guarding of the abdomen). PTS: 1 DIF: Cognitive Level: Comprehension OBJ: Nursing Process: Assessment MSC: Client Needs: Physiologic Integrity 15. Which statement is most descriptive of Meckel's diverticulum? a. It is more common in females than in males. b. It is acquired during childhood. c. Intestinal bleeding may be milNdUoRrSpINroGfTusBe.C. OM d. Medical interventions are usually sufficient to treat the problem. ANS: C Blood stools are often a presenting sign of Meckel's diverticulum. It is associated with mild-to-profuse intestinal bleeding. It is twice as common in males as in females, and complications are more frequent in males. Meckel's diverticulum is the most common congenital malformation of the gastrointestinal tract and is present in 2% of the general population. The standard therapy is surgical removal of the diverticulum. PTS: 1 DIF: Cognitive Level: Comprehension OBJ: Nursing Process: Assessment MSC: Client Needs: Physiologic Integrity 16. What condition is characterized by a chronic inflammatory process that may involve any part of the gastrointestinal (GI) tract from mouth to anus? a. Crohn's disease b. Ulcerative colitis c. Meckel's diverticulum d. Irritable bowel syndrome ANS: A The chronic inflammatory process of Crohn's disease involves any part of the GI tract from the mouth to the anus but most often affects the terminal ileum. Ulcerative colitis, Meckel's diverticulum, and irritable bowel syndrome do not affect the entire GI tract. PTS: 1 DIF: Cognitive Level: Comprehension OBJ: Nursing Process: Assessment MSC: Client Needs: Physiologic Integrity 17. What is used to treat moderate-to-severe inflammatory bowel disease? a. Antacids b. Antibiotics c. Corticosteroids d. Antidiarrheal medications ANS: C Corticosteroids such as prednisone and prednisolone are used in short bursts to suppress the inflammatory response in inflammatory bowel disease. Antacids and antidiarrheals are not drugs of choice to treat the inflammatory process of inflammatory bowel disease. Antibiotics may be used as adjunctive therapy to treat complications. PTS: 1 DIF: Cognitive Level: Comprehension OBJ: Nursing Process: Implementation MSC: Client Needs: Physiologic Integrity 18. Bismuth subsalicylate may be prescribed for a child with a peptic ulcer to effect what result? a. Eradicate Helicobacter pylori b. Coat gastric mucosa c. Treat epigastric pain d. Reduce gastric acid production ANS: A This combination of drug therapy is effective in the treatment and eradication of H. pylori. It does not bring about any of the results. NURSINGTB.COM PTS: 1 DIF: Cognitive Level: Comprehension OBJ: Nursing Process: Implementation MSC: Client Needs: Physiologic Integrity 19. The best chance of survival for a child with cirrhosis is: a. liver transplantation. b. treatment with corticosteroids. c. treatment with immune globulin. d. provision of nutritional support. ANS: A The only successful treatment for end-stage liver disease and liver failure may be liver transplantation, which has improved the prognosis for many children with cirrhosis. Liver transplantation has revolutionized the approach to cirrhosis. Liver failure and cirrhosis are indications for transplantation. Liver transplantation reflects the failure of other medical and surgical measures, such as treatment with corticosteroids or immune globulin and nutritional support, to prevent or treat cirrhosis. PTS: 1 DIF: Cognitive Level: Comprehension OBJ: Nursing Process: Assessment MSC: Client Needs: Physiologic Integrity 20. What is the earliest clinical manifestation of biliary atresia? a. Jaundice b. Vomiting c. Hepatomegaly d. Absence of stooling ANS: A Jaundice is the earliest and most striking manifestation of biliary atresia. It is first observed in the sclera and may be present at birth, but is usually not apparent until ages 2 to 3 weeks. Vomiting is not associated with biliary atresia. Hepatomegaly and abdominal distention are common but occur later. Stools are large and lighter in color than expected because of the lack of bile. PTS: 1 DIF: Cognitive Level: Comprehension OBJ: Nursing Process: Assessment MSC: Client Needs: Physiologic Integrity 21. The nurse, caring for a neonate with a suspected tracheoesophageal fistula, should include what intervention into the plan of care? a. Elevating the head to facilitate secrete drainage. b. Elevating the head for feedings only. c. Feeding glucose water only. d. Avoiding suctioning unless the infant is cyanotic. ANS: A When a newborn is suspected of having tracheoesophageal fistula, the most desirable position is supine with the head elevated on an inclined plane of at least 30 degrees to maintain an airway and facilitate drainage of secretions. It is imperative that any source of aspiration be removed at once; oral feedings are withheld. Feeding of fluids should not be given to infants suspected of having tracheoesophageal fistulas. The oral pharynx should be kept clear of secretion by oral suctioning. This is to avoid the cyanosis that is usually the result of laryngospasm caused by overflow of saliva into the larynx. NURSINGTB.COM PTS: 1 DIF: Cognitive Level: Application OBJ: Nursing Process: Implementation MSC: Client Needs: Physiologic Integrity 22. Which type of hernia has an impaired blood supply to the herniated organ? a. Hiatal hernia b. Incarcerated hernia c. Omphalocele d. Strangulated hernia ANS: D A strangulated hernia is one in which the blood supply to the herniated organ is impaired. A hiatal hernia is the intrusion of an abdominal structure, usually the stomach, through the esophageal hiatus. An incarcerated hernia is a hernia that cannot be reduced easily. Omphalocele is the protrusion of intraabdominal viscera into the base of the umbilical cord. The sac is covered with peritoneum and not skin. PTS: 1 DIF: Cognitive Level: Comprehension OBJ: Nursing Process: Assessment MSC: Client Needs: Physiologic Integrity 23. The nurse is caring for an infant with suspected pyloric stenosis. Which clinical manifestation would indicate pyloric stenosis? a. Abdominal rigidity and pain on palpation b. Rounded abdomen and hypoactive bowel sounds c. Visible peristalsis and weight loss d. Distention of lower abdomen and constipation ANS: C Visible gastric peristaltic waves that move from left to right across the epigastrium are observed in pyloric stenosis, as is weight loss. Abdominal rigidity and pain on palpation, and rounded abdomen and hypoactive bowel sounds, are usually not present. The upper abdomen is distended, not the lower abdomen. PTS: 1 DIF: Cognitive Level: Application OBJ: Nursing Process: Assessment MSC: Client Needs: Physiologic Integrity 24. What is the most appropriate nursing action when a child with a probable intussusception has a normal, brown stool? a. Notify the practitioner b. Measure abdominal girth c. Auscultate for bowel sounds d. Take vital signs, including blood pressure ANS: A Passage of a normal brown stool indicates that the intussusception has reduced itself. This is immediately reported to the practitioner, who may choose to alter the diagnostic/therapeutic plan of care. PTS: 1 DIF: Cognitive Level: Analysis OBJ: Nursing Process: Implementation MSC: Client Needs: Physiologic Integrity 25. An important nursing consideration in the care of a child with celiac disease is to facilitate which intervention? NURSINGTB.COM a. Refer to a nutritionist for detailed dietary instructions and education. b. Help the child and family understand that diet restrictions are usually only temporary. c. Teach proper hand washing and Standard Precautions to prevent disease transmission. d. Suggest ways to cope more effectively with stress to minimize symptoms. ANS: A The main consideration is helping the child adhere to dietary management. Considerable time is spent in explaining to the child and parents the disease process, the specific role of gluten in aggravating the condition, and those foods that must be restricted. Referral to a nutritionist would help in this process. The most severe symptoms usually occur in early childhood and adult life. Dietary avoidance of gluten should be lifelong. Celiac disease is not transmissible or stress related. PTS: 1 DIF: Cognitive Level: Application OBJ: Nursing Process: Planning MSC: Client Needs: Physiologic Integrity 26. What is the major focus of the therapeutic management for a child with lactose intolerance? a. Compliance with the medication regimen b. Providing emotional support to family members c. Teaching dietary modifications d. Administration of daily normal saline enemas ANS: C Simple dietary modifications are effective in the management of lactose intolerance. Symptoms of lactose intolerance are usually relieved after instituting a lactose-free diet. Medications are not typically ordered in the management of lactose intolerance. Providing emotional support to family members is not specific to this medical condition. Diarrhea is a manifestation of lactose intolerance. Enemas are contraindicated for this alteration in bowel elimination. PTS: 1 DIF: Cognitive Level: Comprehension OBJ: Nursing Process: Planning MSC: Client Needs: Physiologic Integrity 27. What food choice by the parent of a 2-year-old child with celiac disease indicates a need for further teaching? a. Oatmeal b. Rice cake c. Corn muffin d. Meat patty ANS: A The child with celiac disease is unable to fully digest gluten, the protein found in wheat, barley, rye, and oats. Oatmeal contains gluten and is not an appropriate food selection. Rice is an appropriate choice because it does not contain gluten. Corn is digestible because it does not contain gluten. Meats do not contain gluten and can be included in the diet of a child with celiac disease. PTS: 1 DIF: Cognitive Level: Application OBJ: Nursing Process: Evaluation MSC: Client Needs: Physiologic Integrity NURSINGTB.COM 28. Which description of a stool is characteristic of intussusception? a. Ribbon-like stools b. Hard stools positive for guaiac c. “Currant jelly” stools d. Loose, foul-smelling stools ANS: C With intussusception, passage of bloody mucus-coated stools occurs. Pressure on the bowel from obstruction leads to passage of “currant jelly” stools. Ribbon-like stools are characteristic of Hirschsprung's disease. Stools will not be hard. Loose, foul-smelling stools may indicate infectious gastroenteritis. PTS: 1 DIF: Cognitive Level: Comprehension OBJ: Nursing Process: Assessment MSC: Client Needs: Physiologic Integrity 29. What should the nurse stress in a teaching plan for the mother of an 11-year-old diagnosed with ulcerative colitis? a. Preventing the spread of illness to others b. Nutritional guidance and preventing constipation c. Teaching daily use of enemas d. Coping with stress and avoiding triggers ANS: D Coping with the stress of chronic illness and the clinical manifestations associated with ulcerative colitis (diarrhea, pain) are important teaching foci. Avoidance of triggers can help minimize the impact of the disease and its effect on the child. Ulcerative colitis is not infectious. Although nutritional guidance is a priority teaching focus, diarrhea is a problem with ulcerative colitis, not constipation. Daily enemas are not part of the therapeutic plan of care. PTS: 1 DIF: Cognitive Level: Application OBJ: Nursing Process: Implementation MSC: Client Needs: Physiologic Integrity 30. Careful hand washing before and after contact can prevent the spread of which condition in day care and school settings? a. Irritable bowel syndrome b. Ulcerative colitis c. Hepatic cirrhosis d. Hepatitis A ANS: D Hepatitis A is spread person to person, by the fecal-oral route, and through contaminated food or water. Good hand washing is critical in preventing its spread. The virus can survive on contaminated objects for weeks. Irritable bowel syndrome is the result of increased intestinal motility and is not contagious. Ulcerative colitis and cirrhosis are not infectious. PTS: 1 DIF: Cognitive Level: Comprehension OBJ: Nursing Process: Implementation MSC: Client Needs: Physiologic Integrity 31. A mother shares with the clinic nurse that she has been giving her 4 year old the antidiarrheal drug loperamide. What conclusionNUshRoSuINldGtThBe.CnuOrMse arrive at based on knowledge of this classification of drugs? a. Not indicated b. Indicated because it slows intestinal motility c. Indicated because it decreases diarrhea d. Indicated because it decreases fluid and electrolyte losses ANS: A Antimotility medications are not recommended for the treatment of acute infectious diarrhea. These medications have adverse effects and toxicity, such as worsening of the diarrhea because of slowing of motility and ileus, or a decrease in diarrhea with continuing fluid losses and dehydration. Antidiarrheal medications are not recommended in infants and small children. PTS: 1 DIF: Cognitive Level: Analysis OBJ: Nursing Process: Implementation MSC: Client Needs: Physiologic Integrity 32. Which vaccine is now recommended for the immunization of all newborns? a. Hepatitis A vaccine b. Hepatitis B vaccine c. Hepatitis C vaccine d. Hepatitis A, B, and C vaccines ANS: B Universal vaccination for hepatitis B is now recommended for all newborns. A vaccine is available for hepatitis A, but it is not yet universally recommended. No vaccine is currently available for hepatitis C. Only hepatitis B vaccine is recommended for newborns. PTS: 1 DIF: Cognitive Level: Comprehension OBJ: Nursing Process: Evaluation MSC: Client Needs: Health Promotion and Maintenance 33. An infant diagnosed with pyloric stenosis experiences excessive vomiting that can result in which condition? a. Hyperchloremia b. Hypernatremia c. Metabolic acidosis d. Metabolic alkalosis ANS: D Infants with excessive vomiting are prone to metabolic alkalosis from the loss of hydrogen ions. Chloride ions and sodium are lost with vomiting. Metabolic alkalosis, not acidosis, is likely. PTS: 1 DIF: Cognitive Level: Comprehension OBJ: Nursing Process: Assessment MSC: Client Needs: Physiologic Integrity MULTIPLE RESPONSE 1. The nurse, caring for an infant whose cleft lip was repaired, should include which interventions into the infant's postoperative plan of care? (Select all that apply.) a. Postural drainage NURSINGTB.COM b. Petroleum jelly to the suture line c. Elbow restraints d. Supine and side-lying positions e. Mouth irrigations ANS: B, C Apply petroleum jelly to the operative site for several days after surgery. Elbows are restrained to prevent the child from accessing the operative site for up to 7 to 10 days. The child should be positioned on back or side or in an infant seat. Postural drainage is not indicated. This would increase the pressure on the operative site when the child is placed in different positions. Mouth irrigations would not be indicated. PTS: 1 DIF: Cognitive Level: Analysis OBJ: Nursing Process: Planning MSC: Client Needs: Physiologic Integrity 2. Which statements regarding hepatitis B are correct? (Select all that apply.) a. Hepatitis B cannot exist in a carrier state. b. Hepatitis B can be prevented by hepatitis B virus vaccine. c. Hepatitis B can be transferred to an infant of a breastfeeding mother. d. The onset of hepatitis B is insidious. e. Immunity to hepatitis B occurs after one attack. ANS: B, C, D, E The vaccine elicits the formation of an antibody to the hepatitis B surface antigen, which is protective against hepatitis B. Hepatitis B can be transferred to an infant of a breastfeeding mother, especially if the mother's nipples are cracked. The onset of hepatitis B is insidious. Immunity develops after one exposure to hepatitis B. Hepatitis B can exist in a carrier state. PTS: 1 DIF: Cognitive Level: Comprehension OBJ: Nursing Process: Diagnosis MSC: Client Needs: Physiologic Integrity 3. Which interventions should a nurse implement when caring for a child with hepatitis? (Select all that apply.) a. Provide a well-balanced, low-fat diet. b. Schedule playtime in the playroom with other children. c. Teach parents not to administer any over-the-counter medications. d. Arrange for home schooling because the child will not be able to return to school. e. Instruct parents on the importance of good hand washing. ANS: A, C, E The child with hepatitis should be placed on a well-balanced, low-fat diet. Parents should be taught to not give over-the-counter medications because of impaired liver function. Hand hygiene is the most important preventive measure for the spread of hepatitis. The child will be in contact isolation in the hospital, so playtime with other hospitalized children is not scheduled. The child will be on contact isolation for a minimum of 1 week after the onset of jaundice. After that period, the child will be allowed to return to school. PTS: 1 DIF: Cognitive Level: Application OBJ: Nursing Process: Implementation MSC: Client Needs: Physiologic Integrity 4. The nurse is preparing to care for NanURinSfIaNnGtTrBet.uCrOnMing from pyloromyotomy surgery. Which prescribed orders should the nurse anticipate implementing? (Select all that apply.) a. Nothing by mouth for 24 hours b. Administration of analgesics for pain c. Ice bag to the incisional area d. Intravenous (IV) fluids continued until tolerating fluids by mouth e. Clear liquids as the first feeding ANS: B, D, E Feedings are usually instituted soon after a pyloromyotomy surgery, beginning with clear liquids and advancing to formula or breast milk as tolerated. IV fluids are administered until the infant is taking and retaining adequate amounts by mouth. Appropriate analgesics should be given round the clock because pain is continuous. Ice should not be applied to the incisional area as it vasoconstricts and would reduce circulation to the incisional area and impair healing. PTS: 1 DIF: Cognitive Level: Application OBJ: Nursing Process: Planning MSC: Client Needs: Physiologic Integrity 5. A nurse is conducting dietary teaching on high-fiber foods for parents of a child with constipation. Which foods should the nurse include as being high in fiber? (Select all that apply.) a. White rice b. Avocados c. Whole grain breads d. Bran pancakes e. Raw carrots ANS: C, D, E High-fiber foods include whole grain breads, bran pancakes, and raw carrots. Unrefined (brown) rice is high in fiber but white rice is not. Raw fruits, especially those with skins or seeds, other than ripe banana or avocados are high in fiber. PTS: 1 DIF: Cognitive Level: Comprehension OBJ: Nursing Process: Implementation MSC: Client Needs: Health Promotion and Maintenance 6. A mother who intended to breastfeed has given birth to an infant with a cleft palate. Which nursing interventions should be included in the plan of care? (Select all that apply.) a. Giving medication to suppress lactation. b. Encouraging and helping mother to breastfeed. c. Teaching mother to feed breast milk by gavage. d. Recommending use of a breast pump to maintain lactation until infant can suck. ANS: B, D The mother who wishes to breastfeed may need encouragement and support because the defect does present some logistical issues. The nipple must be positioned and stabilized well back in the infant's oral cavity so that the tongue action facilitates milk expression. The suction required to stimulate milk, absent initially, may be useful before nursing to stimulate the let-down reflex. Because breastfeeding is an option, if the mother wishes to breastfeed, medications should not be given to suppress lactation. Because breastfeeding can usually be accomplished, gavage feedings are not indicated. NURSINGTB.COM PTS: 1 DIF: Cognitive Level: Application OBJ: Nursing Process: Implementation MSC: Client Needs: Physiologic Integrity Chapter 42: The Child With Cardiovascular Dysfunction Perry: Maternal Child Nursing Care, 6th Edition MULTIPLE CHOICE 1. Which complication should the nurse asses for when caring for a child post cardiac catheterization? a. Cardiac arrhythmia b. Hypostatic pneumonia c. Congestive heart failure d. Rapidly increasing blood pressure ANS: A Because a catheter is introduced into the heart, a risk exists of catheter-induced arrhythmias occurring during the procedure. These are usually transient. Hypostatic pneumonia, congestive heart failure, and rapidly increasing blood pressure are not risks usually associated with cardiac catheterization. PTS: 1 DIF: Cognitive Level: Application OBJ: Nursing Process: Assessment MSC: Client Needs: Physiologic Integrity 2. José is 4 year old. Preoperative teaching for a 4-year-old child scheduled for a cardiac catheterization should be done with what primary consideration in mind? a. Directed at his parents because he is too young to understand. b. Detailed in regard to the actual procedures so he will know what to expect. c. Done several days before the pNrUoRceSdINuGreTsBo.CtOhaMt he will be prepared. d. Adapted to his level of development so that he can understand. ANS: D Preoperative teaching should always be directed at the child's stage of development. The caregivers also benefit from the same explanations. The parents may ask additional questions, which should be answered, but the child needs to receive the information based on developmental level. This age-group does not understand in-depth descriptions. Preschoolers should be prepared close to the time of the cardiac catheterization. PTS: 1 DIF: Cognitive Level: Application OBJ: Nursing Process: Implementation MSC: Client Needs: Health Promotion and Maintenance 3. The nurse is caring for a school-age girl who has had a cardiac catheterization. The child tells the nurse that her bandage is “too wet.” The nurse finds the bandage and bed soaked with blood. What is the most appropriate initial nursing action? a. Notify the physician. b. Apply a new bandage with more pressure. c. Place the child in the Trendelenburg position. d. Apply direct pressure above the catheterization site. ANS: D If bleeding occurs, direct continuous pressure is applied 2.5 cm (1 inch) above the percutaneous skin site to localize pressure over the vessel puncture. Notifying the physician and applying a new bandage with more pressure can be done after pressure is applied. The nurse can have someone else notify the physician while the pressure is being maintained. The Trendelenburg position would not be helpful; it would increase the drainage from the lower extremities. PTS: 1 DIF: Cognitive Level: Application OBJ: Nursing Process: Implementation MSC: Client Needs: Physiologic Integrity 4. Which defect results in increased pulmonary blood flow? a. Pulmonic stenosis b. Tricuspid atresia c. Atrial septal defect d. Transposition of the great arteries ANS: C Atrial septal defect results in increased pulmonary blood flow. Blood flows from the left atrium (higher pressure) into the right atrium (lower pressure) and then to the lungs via the pulmonary artery. Pulmonic stenosis is an obstruction to blood flowing from the ventricles. Tricuspid atresia results in decreased pulmonary blood flow. Transposition of the great arteries results in mixed blood flow. PTS: 1 DIF: Cognitive Level: Comprehension OBJ: Nursing Process: Assessment MSC: Client Needs: Physiologic Integrity 5. Which structural defects constitute tetralogy of Fallot? a. Pulmonic stenosis, ventricularNsUepRtSaIlNdGeTfBec.Ct,OoMverriding aorta, right ventricular hypertrophy b. Aortic stenosis, ventricular septal defect, overriding aorta, right ventricular hypertrophy c. Aortic stenosis, atrial septal defect, overriding aorta, left ventricular hypertrophy d. Pulmonic stenosis, ventricular septal defect, aortic hypertrophy, left ventricular hypertrophy ANS: A Tetralogy of Fallot has these four characteristics: pulmonary stenosis, ventricular septal defect, overriding aorta, and right ventricular hypertrophy. There is pulmonic stenosis but not aortic stenosis in tetralogy of Fallot. Right ventricular hypertrophy, not left ventricular hypertrophy, is present in tetralogy of Fallot. There is a ventricular septal defect, not an atrial septal defect, and overriding aorta, not aortic hypertrophy, is present. PTS: 1 DIF: Cognitive Level: Comprehension OBJ: Nursing Process: Assessment MSC: Client Needs: Physiologic Integrity 6. What is best described as the inability of the heart to pump an adequate amount of blood to the systemic circulation at normal filling pressures? a. Pulmonary congestion b. Congenital heart defect c. Heart failure d. Systemic venous congestion ANS: C The definition of heart failure is the inability of the heart to pump an adequate amount of blood to the systemic circulation at normal filling pressures to meet the metabolic demands of the body. Pulmonary congestion is an excessive accumulation of fluid in the lungs. Congenital heart defect is a malformation of the heart present at birth. Systemic venous congestion is an excessive accumulation of fluid in the systemic vasculature. PTS: 1 DIF: Cognitive Level: Comprehension OBJ: Nursing Process: Assessment MSC: Client Needs: Physiologic Integrity 7. Which term is used to describe a clinical manifestation of the systemic venous congestion that can occur with congestive heart failure? a. Tachypnea b. Tachycardia c. Peripheral edema d. Pale, cool extremities ANS: C Peripheral edema, especially periorbital edema, is a clinical manifestation of systemic venous congestion. Tachypnea is a manifestation of pulmonary congestion. Tachycardia and pale, cool extremities are clinical manifestations of impaired myocardial function. PTS: 1 DIF: Cognitive Level: Analysis OBJ: Nursing Process: Assessment MSC: Client Needs: Physiologic Integrity 8. What beneficial effect is achieved by administering digoxin? a. Decreases edema b. Decreases cardiac output c. Increases heart size d. Increases venous pressure ANS: A NURSINGTB.COM Digoxin has a rapid onset and is useful in increasing cardiac output, decreasing venous pressure, and as a result decreasing edema. Heart size is decreased by digoxin. PTS: 1 DIF: Cognitive Level: Comprehension OBJ: Nursing Process: Implementation MSC: Client Needs: Physiologic Integrity 9. Which drug is an angiotensin-converting enzyme (ACE) inhibitor? a. Captopril b. Furosemide c. Spironolactone d. Chlorothiazide ANS: A Capoten is an ACE inhibitor. Lasix is a loop diuretic. Aldactone blocks the action of aldosterone. Diuril works on the distal tubules. PTS: 1 DIF: Cognitive Level: Comprehension OBJ: Nursing Process: Implementation MSC: Client Needs: Physiologic Integrity 10. What is a common sign of digoxin toxicity? a. Seizures b. Vomiting c. Bradypnea d. Tachycardia ANS: B Vomiting is a common sign of digoxin toxicity. Seizures are not associated with digoxin toxicity. The child will have a slower heart rate, not respiratory rate. PTS: 1 DIF: Cognitive Level: Comprehension OBJ: Nursing Process: Implementation MSC: Client Needs: Physiologic Integrity 11. The parents of a young child with congestive heart failure tell the nurse that they are “nervous” about giving digoxin. The nurse's response should be based on knowing what information? a. It is a safe, frequently used drug. b. It is difficult to either overmedicate or undermedicate with digoxin. c. Parents lack the expertise necessary to administer digoxin. d. Parents must learn specific, important guidelines for administration of digoxin. ANS: D Digoxin has a narrow therapeutic range. The margin of safety between therapeutic, toxic, and lethal doses is very small. Specific guidelines are available for parents to learn how to administer the drug safely and monitor for side effects. Digoxin is a frequently used drug, but it has a narrow therapeutic range. Very small amounts of the liquid are given to infants, which makes it easy to overmedicate or undermedicate. Parents may lack the necessary expertise to administer the drug at first, but wiNthURdiSsIcNhGaTrgBe.CpOreMparation they should be prepared to administer the drug safely. PTS: 1 DIF: Cognitive Level: Analysis OBJ: Nursing Process: Implementation MSC: Client Needs: Physiologic Integrity 12. As part of the treatment for congestive heart failure, the child takes the diuretic furosemide. As part of teaching home care, the nurse encourages the family to give the child foods such as bananas, oranges, and leafy vegetables. These foods are recommended because they are high in what electrolyte? a. Chlorides b. Potassium c. Sodium d. Zinc ANS: B Diuretics that work on the proximal and distal renal tubules contribute to increased losses of potassium. The child's diet should be supplemented with potassium. PTS: 1 DIF: Cognitive Level: Comprehension OBJ: Nursing Process: Planning MSC: Client Needs: Physiologic Integrity 13. An 8-month-old infant becomes hypercyanotic while blood is being drawn. What should be the nurse's first action? a. Assess for neurologic defects b. Place the child in the knee-chest position c. Begin cardiopulmonary resuscitation d. Prepare the family for imminent death ANS: B The first action is to place the infant in the knee-chest position. Blow-by oxygen may be indicated. Neurologic defects are unlikely. The child should be assessed for airway, breathing, and circulation. Often calming the child and administering oxygen and morphine can alleviate the hypercyanotic spell; cardiopulmonary resuscitation is not necessary, and death is unlikely. PTS: 1 DIF: Cognitive Level: Application OBJ: Nursing Process: Implementation MSC: Client Needs: Physiologic Integrity 14. The nurse is caring for a child with persistent hypoxia secondary to a cardiac defect. The nurse recognizes that a risk of cerebrovascular accidents (strokes) exists. An important objective to decrease this risk is to achieve what result? a. Minimize seizures b. Prevent dehydration c. Promote cardiac output d. Reduce energy expenditure ANS: B In children with persistent hypoxia, polycythemia develops. Dehydration must be prevented in hypoxemic children because it potentiates the risk of strokes. Minimizing seizures, promoting cardiac output, and reducing energy expenditure will not reduce the risk of cerebrovascular accidents. PTS: 1 DIF: CognitiNveURLSevINelG: TABn.aClOysMis OBJ: Nursing Process: Implementation MSC: Client Needs: Physiologic Integrity 15. Parents of a 3-year-old child diagnosed with congenital heart disease are afraid to let their child play with other children because of possible overexertion. The nurse's reply should be based on what knowledge? a. The child needs opportunities to play with peers. b. The child needs to understand that peers' activities are too strenuous. c. Parents can meet all the child's needs. d. Constant parental supervision is needed to avoid overexertion. ANS: A The child needs opportunities for social development. Children usually limit their activities if allowed to set their own pace and regulate their activities. The child will limit activities as necessary. Parents must be encouraged to seek appropriate social activities for the child, especially before kindergarten. The child needs to have activities that foster independence. PTS: 1 DIF: Cognitive Level: Analysis OBJ: Nursing Process: Implementation MSC: Client Needs: Psychosocial Integrity 16. What should the nurse consider when preparing a school-age child and the family for heart surgery? a. Not showing unfamiliar equipment b. Letting child hear the sounds of an electrocardiograph monitor c. Avoiding mentioning postoperative discomfort and interventions d. Explaining that an endotracheal tube will not be needed if the surgery goes well ANS: B The child and family should be exposed to the sights and sounds of the intensive care unit. All positive, nonfrightening aspects of the environment are emphasized. The child should be shown unfamiliar equipment, and its use should be demonstrated on a doll. Carefully prepare the child for the postoperative experience, including intravenous lines, incision, and endotracheal tube. PTS: 1 DIF: Cognitive Level: Analysis OBJ: Nursing Process: Implementation MSC: Client Needs: Health Promotion and Maintenance 17. Seventy-two hours after cardiac surgery, a young child has a temperature of 37.7° C (101° F). The nurse should initially implement which intervention? a. Keep the child warm with blankets b. Apply a hypothermia blanket c. Record the temperature on nurses' notes d. Report findings to physician ANS: D In the first 24 to 48 hours after surgery, the body temperature may increase to 37.7° C (100° F) as part of the inflammatory response to tissue trauma. If the temperature is higher or an elevated temperature continues after this period, it is most likely a sign of an infection and immediate investigation is indicated. Blankets should be removed from the child to keep the temperature from increasing. A hypothermia blanket is not indicated for this level of temperature. The temperature should be recorded, but the physician must be notified for evaluation. NURSINGTB.COM PTS: 1 DIF: Cognitive Level: Analysis OBJ: Nursing Process: Assessment MSC: Client Needs: Physiologic Integrity 18. What is an important nursing consideration when suctioning a young child who has had heart surgery? a. Perform suctioning at least every hour. b. Suction for no longer than 30 seconds at a time. c. Administer supplemental oxygen before and after suctioning. d. Expect symptoms of respiratory distress when suctioning. ANS: C If suctioning is indicated, supplemental oxygen is administered with a manual resuscitation bag before and after the procedure to prevent hypoxia. Suctioning should be done only as indicated, not on a routine basis. The child should be suctioned for no more than 5 seconds at one time. Symptoms of respiratory distress are to be avoided by using the appropriate technique. PTS: 1 DIF: Cognitive Level: Application OBJ: Nursing Process: Implementation MSC: Client Needs: Physiologic Integrity 19. The nurse is caring for a child after heart surgery. What intervention should the nurse implement immediately if evidence is found of cardiac tamponade? a. Increase analgesia b. Apply warming blankets c. Immediately report this to the physician d. Encourage the child to cough, turn, and breathe deeply ANS: C If evidence is noted of cardiac tamponade (blood or fluid in the pericardial space constricting the heart), the physician is notified immediately of this life-threatening complication. Increasing analgesia may be done before the physician drains the fluid, but the physician must be notified. Warming blankets are not indicated at this time. Encouraging the child to cough, turn, and breathe deeply should be deferred until after the evaluation by the physician. PTS: 1 DIF: Cognitive Level: Analysis OBJ: Nursing Process: Implementation MSC: Client Needs: Physiologic Integrity 20. What intervention should be implemented prior to the removal of a child's chest tubes? a. Explain that it is not painful. b. Explain that only a Band-Aid will be needed. c. Administer analgesics before the procedure. d. Educate the patient to expect bright red drainage for several hours after removal. ANS: C It is appropriate to prepare the child for the removal of chest tubes with analgesics. Short-acting medications can be used that are administered through an existing intravenous line. It is not a pain-free procedure. A sharp, momentary pain is felt, and this should not be misrepresented to the child. A petroleum gauze/airtight dressing is needed. Little or no drainage should be found on removal. PTS: 1 DIF: Cognitive Level: Analysis OBJ: Nursing Process: Planning MSC: Client Needs: Physiologic IntNegUriRtySINGTB.COM 21. What is the most common causative agent of bacterial endocarditis? a. Staphylococcus albus b. Streptococcus hemolyticus c. Staphylococcus albicans d. Streptococcus viridans ANS: D Staphylococcus viridans is the most common causative agent in bacterial (infective) endocarditis. Staphylococcus albus, Streptococcus hemolyticus, and Staphylococcus albicans are not common causative agents. PTS: 1 DIF: Cognitive Level: Comprehension OBJ: Nursing Process: Assessment MSC: Client Needs: Physiologic Integrity 22. What term is used to describe the painful, tender, pea-sized nodules that may appear on the pads of the fingers or toes in cases of bacterial endocarditis? a. Osler's nodes b. Janeway lesions c. Subcutaneous nodules d. Aschoff's nodules ANS: A Osler's nodes are red, painful, intradermal nodes found on pads of the phalanges in bacterial endocarditis. Janeway lesions are painless hemorrhagic areas on palms and soles in bacterial endocarditis. Subcutaneous nodules are nontender swellings located over bony prominences, commonly found in rheumatic fever. Aschoff's nodules are small nodules composed of cells and leukocytes found in the interstitial tissues of the heart in rheumatic myocarditis. PTS: 1 DIF: Cognitive Level: Comprehension OBJ: Nursing Process: Assessment MSC: Client Needs: Physiologic Integrity 23. What is the primary nursing intervention necessary to prevent bacterial endocarditis? a. Institute measures to prevent dental procedures. b. Counsel parents of high risk children about prophylactic antibiotics. c. Observe children for complications such as embolism and heart failure. d. Encourage restricted mobility in susceptible children. ANS: B The objective of nursing care is to counsel the parents of high risk children about both the need for prophylactic antibiotics for dental procedures and the necessity of maintaining excellent oral health. The child's dentist should be aware of the child's cardiac condition. Dental procedures should be done to maintain a high level of oral health. Prophylactic antibiotics are necessary. Observing for complications and encouraging restricted mobility in susceptible children should be done, but maintaining good oral health and using prophylactic antibiotics are most important. PTS: 1 DIF: Cognitive Level: Application OBJ: Nursing Process: Assessment MSC: Client Needs: Physiologic Integrity 24. What is a common, serious complNicUaRtiSoInNGofTrBh.CeuOmMatic fever? a. Seizures b. Cardiac arrhythmias c. Pulmonary hypertension d. Cardiac valve damage ANS: D Cardiac valve damage is the most significant complication of rheumatic fever. Seizures, cardiac arrhythmias, and pulmonary hypertension are not common complications of rheumatic fever. PTS: 1 DIF: Cognitive Level: Comprehension OBJ: Nursing Process: Assessment MSC: Client Needs: Physiologic Integrity 25. What is a major clinical manifestation of rheumatic fever? a. Polyarthritis b. Osler's nodes c. Janeway spots d. Splinter hemorrhages of distal third of nails ANS: A Polyarthritis is swollen, hot, red, and painful joints. The affected joints will change every 1 to 2 days. Primarily the large joints are affected. Osler's nodes, Janeway spots, and splinter hemorrhages are characteristic of infective endocarditis. PTS: 1 DIF: Cognitive Level: Analysis OBJ: Nursing Process: Assessment MSC: Client Needs: Physiologic Integrity 26. When discussing hyperlipidemia with a group of adolescents, the nurse should explain that high levels of what substance are thought to protect against cardiovascular disease? a. Cholesterol b. Triglycerides c. Low-density lipoproteins (LDLs) d. High-density lipoproteins (HDLs) ANS: D HDLs contain very low concentrations of triglycerides, relatively little cholesterol, and high levels of proteins. It is thought that HDLs protect against cardiovascular disease. Cholesterol, triglycerides, and LDLs do not protect against cardiovascular disease. PTS: 1 DIF: Cognitive Level: Application OBJ: Nursing Process: Implementation MSC: Client Needs: Physiologic Integrity 27. What is the leading cause of death after heart transplantation? a. Infection b. Rejection c. Cardiomyopathy d. Congestive heart failure ANS: B The posttransplantation course is complex. The leading cause of death after cardiac transplantation is rejection. Infection is a continued risk secondary to the immunosuppression necessary to prevent rejection. CaNrdUioRmSIyNoGpTaBth.CyOiMs one of the indications for cardiac transplant. Congestive heart failure is not a leading cause of death. PTS: 1 DIF: Cognitive Level: Comprehension OBJ: Nursing Process: Assessment MSC: Client Needs: Physiologic Integrity 28. When caring for the child with Kawasaki disease, the nurse should understand that principle of care? a. The child's fever is usually responsive to antibiotics within 48 hours. b. The principal area of involvement is the joints. c. Aspirin is contraindicated. d. Therapeutic management includes administration of gamma globulin and aspirin. ANS: D High-dose intravenous gamma globulin and aspirin therapy are indicated to reduce the incidence of coronary artery abnormalities when given within the first 10 days of the illness. The fever of Kawasaki disease is unresponsive to antibiotics and antipyretics. Involvement of mucous membranes and conjunctiva, changes in the extremities, and cardiac involvement are seen. PTS: 1 DIF: Cognitive Level: Comprehension OBJ: Nursing Process: Implementation MSC: Client Needs: Physiologic Integrity 29. What is one of the most frequent causes of hypovolemic shock in children? a. Myocardial infarction b. Blood loss c. Anaphylaxis d. Congenital heart disease ANS: B Blood loss and extracellular fluid loss are two of the most frequent causes of hypovolemic shock in children. Myocardial infarction is rare in a child; if it occurred, the resulting shock would be cardiogenic, not hypovolemic. Anaphylaxis results in distributive shock from extreme allergy or hypersensitivity to a foreign substance. Congenital heart disease tends to contribute to hypervolemia, not hypovolemia. PTS: 1 DIF: Cognitive Level: Comprehension OBJ: Nursing Process: Assessment MSC: Client Needs: Physiologic Integrity 30. What type of shock is characterized by a hypersensitivity reaction causing massive vasodilation and capillary leaks, which may occur with drug or latex allergy? a. Neurogenic shock b. Cardiogenic shock c. Hypovolemic shock d. Anaphylactic shock ANS: D Anaphylactic shock results from extreme allergy or hypersensitivity to a foreign substance. Neurogenic shock results from loss of neuronal control, such as the interruption of neuronal transmission that occurs from a spinal cord injury. Cardiogenic shock is decreased cardiac output. Hypovolemic shock is a reduction in the size of the vascular compartment, decreasing blood pressure, and low central venous pressure. NURSINGTB.COM PTS: 1 DIF: Cognitive Level: Comprehension OBJ: Nursing Process: Assessment MSC: Client Needs: Physiologic Integrity 31. Which clinical changes occur as a result of septic shock? a. Hypothermia b. Increased cardiac output c. Vasoconstriction d. Angioneurotic edema ANS: B Increased cardiac output, which results in warm, flushed skin, is one of the manifestations of septic shock. Fever and chills are characteristic of septic shock. Vasodilation is more common in septic shock. Angioneurotic edema occurs as a manifestation in anaphylactic shock. PTS: 1 DIF: Cognitive Level: Analysis OBJ: Nursing Process: Assessment MSC: Client Needs: Physiologic Integrity 32. A child is brought to the emergency department experiencing an anaphylactic reaction to a bee sting. While an airway is being established, what medication should the nurse prepare for immediate administration? a. Diphenhydramine b. Dopamine c. Epinephrine d. Calcium chloride ANS: C After the first priority of establishing an airway, epinephrine is the drug of choice. Diphenhydramine is not a strong enough antihistamine for this severe a reaction. Dopamine and calcium chloride are not appropriate drugs for this type of reaction. PTS: 1 DIF: Cognitive Level: Comprehension OBJ: Nursing Process: Assessment MSC: Client Needs: Physiologic Integrity 33. Which postoperative intervention should be questioned for a child after a cardiac catheterization? a. Continue intravenous (IV) fluids until the infant is tolerating oral fluids. b. Check the dressing for bleeding. c. Assess peripheral circulation on the affected extremity. d. Keep the affected leg flexed and elevated. ANS: D The child should be positioned with the affected leg straight for 4 to 6 hours after the procedure. IV fluid administration continues until the child is taking and retaining adequate amounts of oral fluids. The insertion site dressing should be observed frequently for bleeding. The nurse should also look under the child to check for pooled blood. Peripheral perfusion is monitored after catheterization. Distal pulses should be palpable, although they may be weaker than in the contralateral extremity. PTS: 1 DIF: Cognitive Level: Analysis OBJ: Nursing Process: Implementation MSC: Client Needs: Physiologic Integrity 34. In which situation is there the greaNtUesRtSrIiNskGTthBa.Ct aOMnewborn infant will have a congenital heart defect (CHD)? a. Trisomy 21 detected on amniocentesis b. Family history of myocardial infarction c. Father has type 1 diabetes mellitus d. Older sibling born with Turner's syndrome ANS: A The incidence of congenital heart disease is approximately 50% in children with trisomy 21 (Down syndrome). A family history of congenital heart disease, not acquired heart disease, increases the risk of giving birth to a child with CHD. Infants born to mothers who are insulin dependent have an increased risk of CHD. Infants identified as having certain genetic defects, such as Turner's syndrome, have a higher incidence of CHD. PTS: 1 DIF: Cognitive Level: Application OBJ: Nursing Process: Assessment MSC: Client Needs: Health Promotion and Maintenance 35. Which intervention should be included in the plan of care for an infant with the nursing diagnosis of Excess Fluid Volume related to congestive heart failure? a. Weigh the infant every day on the same scale at the same time. b. Notify the physician when weight gain exceeds more than 20 g/day. c. Put the infant in a car seat to minimize movement. d. Administer digoxin as ordered by the physician. Excess fluid volume may not be overtly visible. Weight changes may indicate fluid retention. Weighing the infant on the same scale at the same time each day ensures consistency. An excessive weight gain for an infant is an increase of more than 50 g/day. With fluid volume excess, skin will be edematous. The infant's position should be changed frequently to prevent undesirable pooling of fluid in certain areas. Digoxin is used in the treatment of congestive heart failure to improve cardiac function. Diuretics will help the body get rid of excess fluid. PTS: 1 DIF: Cognitive Level: Application OBJ: Nursing Process: Implementation MSC: Client Needs: Physiologic Integrity 36. The nurse assessing a premature newborn infant auscultates a continuous machinery-like murmur. This finding is associated with which congenital heart defect? a. Pulmonary stenosis b. Patent ductus arteriosus c. Ventricular septal defect d. Coarctation of the aorta ANS: B The classic murmur associated with patent ductus arteriosus is a machinery-like one that can be heard throughout both systole and diastole. A systolic ejection murmur that may be accompanied by a palpable thrill is a manifestation of pulmonary stenosis. The characteristic murmur associated with ventricular septal defect is a loud, harsh, holosystolic murmur. A systolic murmur that is accompanied by an ejection click may be heard on auscultation when coarctation of the aorta is present. PTS: 1 DIF: Cognitive Level: Application OBJ: Nursing Process: Assessment MSC: Client Needs: Physiologic Integrity NURSINGTB.COM 37. What is an expected assessment finding in a child with coarctation of the aorta? a. Orthostatic hypotension b. Systolic hypertension in the lower extremities c. Blood pressure higher on the left side of the body d. Disparity in blood pressure between the upper and lower extremities ANS: D The classic finding in children with coarctation of the aorta is a disparity in pulses and blood pressures between the upper and lower extremities. Orthostatic hypotension is not present with coarctation of the aorta. Systolic hypertension may be detected in the upper extremities. The left arm may not accurately reflect systolic hypertension because the left subclavian artery can be involved in the coarctation. PTS: 1 DIF: Cognitive Level: Application OBJ: Nursing Process: Assessment MSC: Client Needs: Physiologic Integrity 38. A child with pulmonary atresia exhibits cyanosis with feeding. On reviewing this child's laboratory values, the nurse is not surprised to notice which abnormality? a. Polycythemia b. Infection c. Dehydration d. Anemia Polycythemia is a compensatory response to chronic hypoxia. The body attempts to improve tissue oxygenation by producing additional red blood cells and thereby increases the oxygen-carrying capacity of the blood. Infection is not a clinical consequence of cyanosis. Although dehydration can occur in cyanotic heart disease, it is not a compensatory mechanism for chronic hypoxia. It is not a clinical consequence of cyanosis. Anemia may develop as a result of increased blood viscosity. Anemia is not a clinical consequence of cyanosis. PTS: 1 DIF: Cognitive Level: Analysis OBJ: Nursing Process: Assessment MSC: Client Needs: Physiologic Integrity 39. What is the nurse's first action when planning to teach the parents of an infant with a congenital heart defect (CHD)? a. Assess the parents' anxiety level and readiness to learn b. Gather literature for the parents c. Secure a quiet place for teaching d. Discuss the plan with the nursing team ANS: A Any effort to organize the right environment, plan, or literature is of no use if the parents are not ready to learn or have high anxiety. Decreasing their level of anxiety is often needed before new information can be processed. A baseline assessment of prior knowledge should be taken into consideration before developing any teaching plan. Locating a quiet place for meeting with parents is appropriate; however, an assessment should be done before any teaching is done. Discussing a teaching plan with the nursing team is appropriate after an assessment of the parents' knowledge and readiness. PTS: 1 DIF: Cognitive Level: Application OBJ: Nursing Process: Planning MSC: Client Needs: Health PromotiNonUaRnSdINMGaTiBnt.eCnOaMnce 40. A nurse is teaching an adolescent about essential hypertension. The nurse knows that which of the following is correct? a. Primary hypertension should be treated with diuretics as soon as it is detected. b. Congenital heart defects are the most common cause of primary hypertension. c. Primary hypertension may be treated with weight reduction. d. Primary hypertension is not affected by exercise. ANS: C Essential hypertension in children may be treated with weight reduction and exercise programs. If ineffective, pharmacologic intervention may be needed. Primary hypertension is considered an inherited disorder. PTS: 1 DIF: Cognitive Level: Comprehension OBJ: Nursing Process: Evaluation MSC: Client Needs: Health Promotion and Maintenance 41. What is the initial goal for the treatment of secondary hypertension? a. Weight control and diet b. Treating the underlying disease c. Administration of digoxin d. Administration of -adrenergic receptor blockers Identification of the underlying disease should be the first step in treating secondary hypertension. Weight control and diet are nonpharmacologic treatments for primary hypertension. Digoxin is indicated in the treatment of congestive heart failure. -Adrenergic receptor blockers are indicated in the treatment of primary hypertension. PTS: 1 DIF: Cognitive Level: Comprehension OBJ: Nursing Process: Implementation MSC: Client Needs: Physiologic Integrity 42. The nurse is preparing an adolescent for discharge after a cardiac catheterization. Which statement by the adolescent would indicate a need for further teaching? a. “I should avoid tub baths but may shower.” b. “I have to stay on strict bed rest for 3 days.” c. “I should remove the pressure dressing the day after the procedure.” d. “I may attend school but should avoid exercise for several days.” ANS: B Encourage rest and quiet activities for the first 3 days and avoid strenuous exercise. The child does not need to be on strict bed rest for 3 days. Showers are recommended; children should avoid a tub bath. The pressure dressing is removed the day after the catheterization and replaced by an adhesive bandage to keep the area clean. PTS: 1 DIF: Cognitive Level: Analysis OBJ: Nursing Process: Implementation MSC: Client Needs: Physiologic Integrity 43. Surgical closure of the ductus arteriosus would bring about what desired effect? a. Stop the loss of unoxygenated blood to the systemic circulation. b. Decrease the edema in legs and feet. c. Increase the oxygenation of blNoUodR.SINGTB.COM d. Prevent the return of oxygenated blood to the lungs. ANS: D The ductus arteriosus allows blood to flow from the higher-pressure aorta to the lower-pressure pulmonary artery, causing a right-to-left shunt. If this is surgically closed, no additional oxygenated blood (from the aorta) will return to the lungs through the pulmonary artery. The aorta carries oxygenated blood to the systemic circulation. Because of the higher pressure in the aorta, blood is shunted into the pulmonary artery and the pulmonary circulation. Edema in the legs and feet is usually a sign of heart failure. This repair would not directly affect the edema. Increasing the oxygenation of blood would not interfere with the return of oxygenated blood to the lungs. PTS: 1 DIF: Cognitive Level: Comprehension OBJ: Nursing Process: Planning MSC: Client Needs: Physiologic Integrity: Physiologic Adaptation 44. A nurse is teaching nursing students the physiology of congenital heart defects. Which defect results in decreased pulmonary blood flow? a. Atrial septal defect b. Tetralogy of Fallot c. Ventricular septal defect d. Patent ductus arteriosus Tetralogy of Fallot results in decreased blood flow to the lungs. The pulmonic stenosis increases the pressure in the right ventricle, causing the blood to go from right to left across the interventricular septal defect. Atrial and ventricular septal defects and patent ductus arteriosus result in increased pulmonary blood flow. PTS: 1 DIF: Cognitive Level: Comprehension OBJ: Nursing Process: Assessment MSC: Client Needs: Physiologic Integrity: Physiologic Adaptation 45. The nurse is talking to a parent of an infant with heart failure about feeding the infant. Which statement about feeding the child is correct? a. “You may need to increase the caloric density of your infant's formula.” b. “You should feed your baby every 2 hours.” c. “You may need to increase the amount of formula your infant eats with each feeding.” d. “You should place a nasal oxygen cannula on your infant during and after each feeding.” ANS: A The metabolic rate of infants with heart failure is greater because of poor cardiac function and increased heart and respiratory rates. Their caloric needs are greater than those of the average infants, yet their ability to take in the calories is diminished by their fatigue. Infants with heart failure should be fed every 3 hours; a 2-hour schedule does not allow for enough rest, and a 4-hour schedule is too long. Fluids must be carefully monitored because of the heart failure. Infants do not require supplemental oxygen with feedings. PTS: 1 DIF: Cognitive Level: Analysis OBJ: Nursing Process: Planning MSC: Client Needs: Physiologic IntNegUriRtyS:INPGhyTsBio.CloOgMic Adaptation 46. The nurse is admitting a child with rheumatic fever. Which therapeutic management should the nurse expect to implement? a. Administering penicillin b. Avoiding salicylates (aspirin) c. Imposing strict bed rest for 4 to 6 weeks d. Administering corticosteroids if chorea develops ANS: A The goal of medical management is the eradication of the hemolytic streptococci. Penicillin is the drug of choice. Salicylates can be used to control the inflammatory process, especially in the joints, and reduce the fever and discomfort. Bed rest is recommended for the acute febrile stage, but it does not need to be strict. The chorea is transient and will resolve without treatment. PTS: 1 DIF: Cognitive Level: Application OBJ: Nursing Process: Implementation MSC: Client Needs: Physiologic Integrity 47. Which action by the school nurse is important in the prevention of rheumatic fever? a. Encourage routine cholesterol screenings. b. Conduct routine blood pressure screenings. c. Refer children with sore throats for throat cultures. d. Recommend salicylates instead of acetaminophen for minor discomforts. ANS: C Nurses have a role in prevention—primarily in screening school-age children for sore throats caused by group A -hemolytic streptococci. They can achieve this by actively participating in throat culture screening or by referring children with possible streptococcal sore throats for testing. Cholesterol and blood pressure screenings do not facilitate the recognition and treatment of group A -hemolytic streptococci. Salicylates should be avoided routinely because of the risk of Reye's syndrome after viral illnesses. PTS: 1 DIF: Cognitive Level: Application OBJ: Nursing Process: Planning MSC: Client Needs: Physiologic Integrity: Reduction of Risk Potential 48. A preschool child is scheduled for an echocardiogram. Parents ask the nurse whether they can hold the child during the procedure. The nurse should answer with which response? a. “You will be able to hold your child during the procedure.” b. “Your child can be active during the procedure, but can't sit in your lap.” c. “Your child must lie quietly; sometimes a mild sedative is administered before the procedure.” d. “The procedure is invasive so your child will be restrained during the echocardiogram.” ANS: C Although an echocardiogram is noninvasive, painless, and associated with no known side effects, it can be stressful for children. The child must lie quietly in the standard echocardiographic positions; crying, nursing, being held, or sitting up often leads to diagnostic errors or omissions. Therefore, infants and young children may need a mild sedative; older children benefit from psychologic preparation for the test. The distraction of a video or movie is often helpful. NURSINGTB.COM PTS: 1 DIF: Cognitive Level: Application OBJ: Nursing Process: Implementation MSC: Client Needs: Physiologic Integrity 49. The nurse is caring for an infant diagnosed with congestive heart disease (CHD). The nurse should plan which intervention to decrease cardiac demands? a. Organize nursing activities to allow for uninterrupted sleep b. Allow the infant to sleep through feedings during the night c. Wait for the infant to cry to show definite signs of hunger d. Discourage parents from rocking the infant ANS: A The infant requires rest and conservation of energy for feeding. Every effort is made to organize nursing activities to allow for uninterrupted periods of sleep. Whenever possible, parents are encouraged to stay with their infant to provide the holding, rocking, and cuddling that help children sleep more soundly. To minimize disturbing the infant, changing bed linens and complete bathing are done only when necessary. Feeding is planned to accommodate the infant's sleep and wake patterns. The child is fed at the first sign of hunger, such as when sucking on fists, rather than waiting until he or she cries for a bottle because the stress of crying exhausts the limited energy supply. Because infants with CHD tire easily and may sleep through feedings, smaller feedings every 3 hours may be helpful. PTS: 1 DIF: Cognitive Level: Application OBJ: Nursing Process: Implementation MSC: Client Needs: Physiologic Integrity MULTIPLE RESPONSE 1. Nursing interventions for the child after a cardiac catheterization include which of the following? (Select all that apply.) a. Allow ambulation as tolerated. b. Monitor vital signs every 2 hours. c. Assess the affected extremity for temperature and color. d. Check pulses above the catheterization site for equality and symmetry. e. Remove pressure dressing after 4 hours. f. Maintain a patent peripheral intravenous catheter until discharge. ANS: C, F The extremity that was used for access for the cardiac catheterization must be checked for temperature and color. Coolness and blanching may indicate arterial occlusion. The child should have a patent peripheral intravenous line to ensure adequate hydration. Allowing ambulation, monitoring vital signs every 2 hours, checking pulses, and removing the pressure dressing after 4 hours are interventions that do not apply to a child after a cardiac catheterization. PTS: 1 DIF: Cognitive Level: Application OBJ: Nursing Process: Implementation MSC: Client Needs: Physiologic Integrity 2. Which clinical manifestations would the nurse expect to see as shock progresses in a child and decompensated develops? (Select all that apply.) a. Thirst and diminished urinary output b. Irritability and apprehension NURSINGTB.COM c. Cool extremities and decreased skin turgor d. Confusion and somnolence e. Normal blood pressure and narrowing pulse pressure f. Tachypnea and poor capillary refill time ANS: C, D, F Cool extremities, decreased skin turgor, confusion, somnolence, tachypnea, and poor capillary refill time are beginning signs of decompensated shock. PTS: 1 DIF: Cognitive Level: Analysis OBJ: Nursing Process: Assessment MSC: Client Needs: Physiologic Integrity 3. A nurse is conducting discharge teaching to parents about the care of their infant after cardiac surgery. The nurse instructs the parents to notify the physician if what conditions occur? (Select all that apply.) a. Respiratory rate of 36 breaths/minute at rest b. Appetite slowly increasing c. Temperature above 37.7° C (100° F) d. New, frequent coughing e. Turning blue or bluer than normal ANS: C, D, E The parents should be instructed to notify the physician after their infant's cardiac surgery for a temperature above 37.7° C (100° F); new, frequent coughing; and any episodes of the infant turning blue or bluer than normal. A respiratory rate of 36 breaths/minute at rest for an infant is within normal expectations, and it is expected that the appetite will increase slowly. PTS: 1 DIF: Cognitive Level: Application OBJ: Nursing Process: Implementation MSC: Client Needs: Health Promotion and Maintenance 4. The nurse is conducting discharge teaching about signs and symptoms of heart failure to parents of an infant with a repaired tetralogy of Fallot. Which signs and symptoms should the nurse include? (Select all that apply.) a. Warm flushed extremities b. Weight loss c. Decreased urinary output d. Sweating (inappropriate) e. Anorexia ANS: C, D, E The signs and symptoms of heart failure include decreased urinary output, sweating, and poor feeding. Other signs include pale, cool extremities, not warm and flushed, and weight gain, not weight loss. PTS: 1 DIF: Cognitive Level: Application OBJ: Nursing Process: Implementation MSC: Client Needs: Physiologic Integrity COMPLETION NURSINGTB.COM 1. Which is the acceptable mg/dL level, or below this level, low-density lipoprotein (LDL) cholesterol level for a child from a family with heart disease? Record your answer as a whole number. ANS: 110 The low-density lipoproteins (LDLs) contain low concentrations of triglycerides, high levels of cholesterol, and moderate levels of protein. LDL is the major carrier of cholesterol to the cells. Cells use cholesterol for synthesis of membranes and steroid production. Elevated circulating LDL is a strong risk factor in cardiovascular disease. For children from families with a history of heart disease, the LDL should be <110. PTS: 1 DIF: Cognitive Level: Application OBJ: Nursing Process: Implementation MSC: Client Needs: Physiologic Integrity MATCHING An infant with an unrepaired tetralogy of Fallot defect is becoming extremely cyanotic during a routine blood draw. Which interventions should the nurse implement? Place in order from the highest-priority intervention to the lowest-priority intervention. a. Administer 100% oxygen by blow-by b. Place infant in knee-chest position c. Remain calm d. Give morphine subcutaneously or by an existing intravenous line 1. First priority 2. Second priority 3. Third priority 4. Fourth priority 1. ANS: B PTS: 1 DIF: Cognitive Level: Application OBJ: Nursing Process: Implementation MSC: Client Needs: Physiologic Integrity NOT: Hypercyanotic spells, also referred to as blue spells or tet spells because they are often seen in infants with tetralogy of Fallot, may occur in any child whose heart defect includes obstruction to pulmonary blood flow and communication between the ventricles. The infant becomes acutely cyanotic and hyperpneic because sudden infundibular spasm decreases pulmonary blood flow and increases right-to-left shunting. Because profound hypoxemia causes cerebral hypoxia, hypercyanotic spells require prompt assessment and treatment to prevent brain damage or possibly death. The infant should first be placed in the knee-chest position to reduce blood returning to the heart. Next, 100% oxygen is given to alleviate the hypoxemia. Morphine is next administered to reduce infundibular spasms. Last, the nurse should remain calm. 2. ANS: A PTS: 1 DIF: Cognitive Level: Application OBJ: Nursing Process: Implementation MSC: Client Needs: Physiologic Integrity NOT: Hypercyanotic spells, also referred to as blue spells or tet spells because they are often seen in infants with tetralogy of Fallot, may occur in any child whose heart defect includes obstruction to pulmonary blood flow and communication between the ventricles. The infant becomes acutely cyanotic and hyperpneic because sudden infundibular spasm decreases pulmonary blood flow and increases right-to-left shunting. Because profound hypoxemia causes cerebral hypoxia, hypercyanotic spells require prompt assessment and treatment to prevent brain damage or possibly death. The infant should first be placed in the knee-cheNsUt pRoSsIiNtiGonTBto.CreOdMuce blood returning to the heart. Next, 100% oxygen is given to alleviate the hypoxemia. Morphine is next administered to reduce infundibular spasms. Last, the nurse should remain calm. 3. ANS: D PTS: 1 DIF: Cognitive Level: Application OBJ: Nursing Process: Implementation MSC: Client Needs: Physiologic Integrity NOT: Hypercyanotic spells, also referred to as blue spells or tet spells because they are often seen in infants with tetralogy of Fallot, may occur in any child whose heart defect includes obstruction to pulmonary blood flow and communication between the ventricles. The infant becomes acutely cyanotic and hyperpneic because sudden infundibular spasm decreases pulmonary blood flow and increases right-to-left shunting. Because profound hypoxemia causes cerebral hypoxia, hypercyanotic spells require prompt assessment and treatment to prevent brain damage or possibly death. The infant should first be placed in the knee-chest position to reduce blood returning to the heart. Next, 100% oxygen is given to alleviate the hypoxemia. Morphine is next administered to reduce infundibular spasms. Last, the nurse should remain calm. 4. ANS: C PTS: 1 DIF: Cognitive Level: Application OBJ: Nursing Process: Implementation MSC: Client Needs: Physiologic Integrity NOT: Hypercyanotic spells, also referred to as blue spells or tet spells because they are often seen in infants with tetralogy of Fallot, may occur in any child whose heart defect includes obstruction to pulmonary blood flow and communication between the ventricles. The infant becomes acutely cyanotic and hyperpneic because sudden infundibular spasm decreases pulmonary blood flow and increases right-to-left shunting. Because profound hypoxemia causes cerebral hypoxia, hypercyanotic spells require prompt assessment and treatment to prevent brain damage or possibly death. The infant should first be placed in the knee-chest position to reduce blood returning to the heart. Next, 100% oxygen is given to alleviate the hypoxemia. Morphine is next administered to reduce infundibular spasms. Last, the nurse should remain calm. Chapter 43: The Child With Hematologic or Immunologic Dysfunction Perry: Maternal Child Nursing Care, 6th Edition MULTIPLE CHOICE 1. What is the primary result of anemia? a. Increased blood viscosity. b. Depressed hematopoietic system. c. Presence of abnormal hemoglobin. d. Decreased oxygen-carrying capacity of blood. ANS: D Anemia is a condition in which the number of red blood cells or hemoglobin concentration is reduced below the normal values for age. This results in a decreased oxygen-carrying capacity of blood. Increased blood viscosity is usually a function of too many cells or of dehydration, not of anemia. A depressed hematopoietic system or abnormal hemoglobin can contribute to anemia, but the definition depends on the deceased oxygen-carrying capacity of the blood. PTS: 1 DIF: Cognitive Level: Comprehension OBJ: Nursing Process: Assessment MSC: Client Needs: Physiologic Integrity 2. Several blood tests are ordered for a preschool child with severe anemia. She is crying and upset because she remembers the venipuncture done at the clinic 2 days ago. The nurse should explain that: a. venipuncture discomfort is very brief. b. only one venipuncture will beNnUeeRdSeINd.GTB.COM c. topical application of local anesthetic can eliminate venipuncture pain. d. most blood tests on children require only a finger puncture because a small amount of blood is needed. ANS: C Preschool children are very concerned about both pain and the loss of blood. When preparing the child for venipuncture, a topical anesthetic will be used to eliminate any pain. This is a very traumatic experience for preschool children. They are concerned about their bodily integrity. A local anesthetic should be used, and a bandage should be applied to maintain bodily integrity. A promise that only one venipuncture will be needed should not be made in case multiple attempts are required. Both finger punctures and venipunctures are traumatic for children. Both require preparation. PTS: 1 DIF: Cognitive Level: Application OBJ: Nursing Process: Implementation MSC: Client Needs: Health Promotion and Maintenance 3. What is the most appropriate nursing diagnosis for a child diagnosed with moderate anemia? a. Activity intolerance related to generalized weakness b. Decreased cardiac output related to abnormal hemoglobin c. Risk for injury related to depressed sensorium d. Risk for Injury related to dehydration and abnormal hemoglobin ANS: A The basic pathology in anemia is the decreased oxygen-carrying capacity of the blood. The nurse must assess the child's activity level (response to the physiologic state). The nursing diagnosis would reflect the activity intolerance. In generalized anemia no abnormal hemoglobin may be present. Only at a level of very severe anemia does cardiac output become altered. No decreased sensorium exists until profound anemia occurs. Dehydration and abnormal hemoglobin are not usually part of anemia. PTS: 1 DIF: Cognitive Level: Analysis OBJ: Nursing Process: Diagnosis MSC: Client Needs: Physiologic Integrity 4. Which statement best explains why iron deficiency anemia is common during toddlerhood? a. Milk is a poor source of iron. b. Iron cannot be stored during fetal development. c. Fetal iron stores are depleted by age 1 month. d. Dietary iron cannot be started until age 12 months. ANS: A Children between the ages of 12 and 36 months are at risk for anemia because cow's milk is a major component of their diet, and it is a poor source of iron. Iron is stored during fetal development, but the amount stored depends on maternal iron stores. Fetal iron stores are usually depleted by age 5 to 6 months. Dietary iron can be introduced by breastfeeding, iron-fortified formula, and cereals during the first 12 months of life. PTS: 1 DIF: Cognitive Level: Analysis OBJ: Nursing Process: Assessment MSC: Client Needs: Physiologic Integrity 5. When teaching the mother of a 9-month-old infant about administering liquid iron preparations, the nurse should incNluUdReStIhNaGt TinBf.oCrOmMation? a. They should be given with meals. b. They should be stopped immediately if nausea and vomiting occur. c. Adequate dosage will turn the stools a tarry green color. d. Preparation should be allowed to mix with saliva and bathe the teeth before swallowing. ANS: C The nurse should prepare the mother for the anticipated change in the child's stools. If the iron dose is adequate, the stools will become a tarry green color. The lack of the color change may indicate insufficient iron. The iron should be given in two divided doses between meals, when the presence of free hydrochloric acid is greatest. Iron is absorbed best in an acidic environment. Vomiting and diarrhea may occur with iron administration. If these occur, the iron should be given with meals, and the dosage reduced and gradually increased as the child develops tolerance. Liquid preparations of iron stain the teeth. They should be administered through a straw, and the mouth rinsed after administration. PTS: 1 DIF: Cognitive Level: Analysis OBJ: Nursing Process: Implementation MSC: Client Needs: Physiologic Integrity 6. Iron dextran is ordered for a young child with severe iron deficiency anemia. What nursing consideration should be considered? a. Administering with meals b. Administering between meals c. Injecting deeply into a large muscle d. Massaging injection site for 5 minutes after administration of drug ANS: C Iron dextran is a parenteral form of iron. When administered intramuscularly, it must be injected into a large muscle using the Z-track method. Iron dextran is for intramuscular or intravenous administration; it is not taken orally. The site should not be massaged to prevent leakage, potential irritation, and staining of the skin. The administration has no relationship to food since it is not being given orally. PTS: 1 DIF: Cognitive Level: Comprehension OBJ: Nursing Process: Implementation MSC: Client Needs: Physiologic Integrity 7. What intervention should the nurse share with parents on how to prevent iron deficiency anemia in a healthy, term, breastfed infant? a. Iron (ferrous sulfate) drops after age 1 month b. Iron-fortified commercial formula can be used by ages 4 to 6 months c. Iron-fortified solid foods are introduced at 3 months d. Iron-fortified infant cereal can be introduced at approximately 6 months of age ANS: D Breast milk supplies inadequate iron for growth and development after age 5 months. Supplementation is necessary at this time. Iron supplementation or the introduction of solid foods in a breastfed baby is not indicated. Introducing iron-fortified infant cereal at 2 months should be done only if the mother is choosing to discontinue breastfeeding. PTS: 1 DIF: Cognitive Level: Application OBJ: Nursing Process: ImplementaNtioUnRSINMGSTCB.:CCOlMient Needs: Physiologic Integrity 8. What term is used to identify the condition in which the normal adult hemoglobin is partly or completely replaced by abnormal hemoglobin? a. Aplastic anemia b. Sickle cell anemia c. Thalassemia major d. Iron deficiency anemia ANS: B Sickle cell anemia is one of a group of diseases collectively called hemoglobinopathies, in which normal adult hemoglobin is replaced by abnormal hemoglobin. Aplastic anemia is a lack of cellular elements being produced. Hemophilia refers to a group of bleeding disorders in which there is deficiency of one of the factors necessary for coagulation. Iron deficiency anemia affects size and depth of color of hemoglobin and does not involve abnormal hemoglobin. PTS: 1 DIF: Cognitive Level: Comprehension OBJ: Nursing Process: Assessment MSC: Client Needs: Physiologic Integrity 9. Which statement most accurately describes the pathologic changes of sickle cell anemia? a. Sickle-shaped cells carry excess oxygen b. Sickle-shaped cells decrease blood viscosity c. Increased red blood cell destruction occurs d. Decreased red blood cell destruction occurs ANS: C The clinical features of sickle cell anemia are primarily the result of increased red blood cell destruction and obstruction caused by the sickle-shaped red blood cells. Sickled red cells have decreased oxygen-carrying capacity and transform into the sickle shape in conditions of low oxygen tension. When the sickle cells change shape, they increase the viscosity in the area where they are involved in the microcirculation. PTS: 1 DIF: Cognitive Level: Application OBJ: Nursing Process: Diagnosis MSC: Client Needs: Physiologic Integrity 10. Which clinical manifestation should the nurse expect when a child diagnosed with sickle cell anemia experiences an acute vaso-occlusive crisis? a. Circulatory collapse b. Cardiomegaly, systolic murmurs c. Hepatomegaly, intrahepatic cholestasis d. Painful swelling of hands and feet, painful joints ANS: D A vaso-occlusive crisis is characterized by severe pain in the area of involvement. If in the extremities, painful swelling of the hands and feet is seen; if in the abdomen, severe pain resembles that of acute surgical abdomen; and if in the head, stroke and visual disturbances occur. Circulatory collapse results from sequestration crises. Cardiomegaly, systolic murmurs, hepatomegaly, and intrahepatic cholestasis result from chronic vaso-occlusive phenomena. PTS: 1 DIF: Cognitive Level: Comprehension OBJ: Nursing Process: Diagnosis MSC: Client Needs: Physiologic Integrity NURSINGTB.COM 11. The parents of a child hospitalized with sickle cell anemia tell the nurse that they are concerned about narcotic analgesics causing addiction. The nurse should provide what explanation in response to the parent's concern? a. Narcotics are often ordered but not usually needed. b. Narcotics rarely cause addiction when they are medically indicated. c. Narcotics are given as a last resort because of the threat of addiction. d. Narcotics are used only if other measures such as ice packs are ineffective. ANS: B The pain of sickle cell anemia is best treated by a multidisciplinary approach. Mild-to-moderate pain can be controlled by ibuprofen and acetaminophen. When narcotics are indicated, they are titrated to effect and given around the clock. Patient-controlled analgesia reinforces the patient's role and responsibility in managing the pain and provides flexibility in dealing with pain. Few if any patients who receive opioids for severe pain become behaviorally addicted to the drug. Narcotics are often used because of the severe nature of the pain of vaso-occlusive crisis. Ice is contraindicated because of its vasoconstrictive effects. PTS: 1 DIF: Cognitive Level: Application OBJ: Nursing Process: Implementation MSC: Client Needs: Physiologic Integrity 12. Which statement correctly describes -thalassemia major (Cooley's anemia)? a. All formed elements of the blood are depressed. b. Inadequate numbers of red blood cells are present. c. Increased incidence occurs in families of Mediterranean extraction. d. Increased incidence occurs in persons of West African descent. ANS: C Individuals who live near the Mediterranean Sea and their descendants have the highest incidence of thalassemia. An overproduction of red cells occurs. Although numerous, the red cells are relatively unstable. Sickle cell disease is common in blacks of West African descent. PTS: 1 DIF: Cognitive Level: Comprehension OBJ: Nursing Process: Assessment MSC: Client Needs: Physiologic Integrity 13. Chelation therapy is begun on a child with -thalassemia major with what expected result? a. Treatment of the disease. b. Elimination of excess iron. c. Decreasing the risk of hypoxia. d. Managing nausea and vomiting. ANS: B A complication of the frequent blood transfusions in thalassemia is iron overload. Chelation therapy with deferoxamine (an iron-chelating agent) is given with oral supplements of vitamin C to increase iron excretion. Chelation therapy treats the side effects of disease management. Decreasing the risk of hypoxia and managing nausea and vomiting are not the purposes of chelation therapy. PTS: 1 DIF: Cognitive Level: Comprehension OBJ: Nursing Process: Implementation MSC: Client Needs: Physiologic Integrity 14. In which condition are all the formNeUdReSlIeNmGeTnBt.sCoOfMthe blood simultaneously depressed? a. Aplastic anemia b. Sickle cell anemia c. Thalassemia major d. Iron deficiency anemia ANS: A Aplastic anemia refers to a bone marrow-failure condition in which the formed elements of the blood are simultaneously depressed. Sickle cell anemia is a hemoglobinopathy in which normal adult hemoglobin is partly or completely replaced by abnormal sickle hemoglobin. Thalassemia major is a group of blood disorders characterized by deficiency in the production rate of specific hemoglobin globin chains. Iron deficiency anemia results in a decreased amount of circulating red cells. PTS: 1 DIF: Cognitive Level: Comprehension OBJ: Nursing Process: Assessment MSC: Client Needs: Physiologic Integrity 15. As related to inherited disorders, which statement is descriptive of most cases of hemophilia? a. Autosomal dominant disorder causing deficiency in a factor involved in the blood-clotting reaction b. X-linked recessive inherited disorder causing deficiency of platelets and prolonged bleeding c. X-linked recessive inherited disorder in which a blood-clotting factor is deficient d. Y-linked recessive inherited disorder in which the red blood cells become moon shaped ANS: C The inheritance pattern in 80% of all of the cases of hemophilia is X-linked recessive. The two most common forms of the disorder are factor VIII deficiency (hemophilia A or classic hemophilia), and factor IX deficiency (hemophilia B or Christmas disease). The disorder involves coagulation factors, not platelets. The disorder does not involve red cells or the Y chromosome. PTS: 1 DIF: Cognitive Level: Comprehension OBJ: Nursing Process: Diagnosis MSC: Client Needs: Physiologic Integrity 16. What name is given to identify an acquired hemorrhagic disorder that is characterized by excessive destruction of platelets? a. Aplastic anemia b. Thalassemia major c. Disseminated intravascular coagulation d. Immune thrombocytopenia ANS: D Immune thrombocytopenia is an acquired hemorrhagic disorder characterized by an excessive destruction of platelets, discolorations caused by petechiae beneath the skin, and a normal bone marrow. Aplastic anemia refers to a bone marrow failure condition in which the formed elements of the blood are simultaneously depressed. Thalassemia major is a group of blood disorders characterized by deficiency in the production rate of specific hemoglobin globin chains. Disseminated intravascular coagulation is characterized by diffuse fibrin deposition in the microvasculature, consumption of coagulation factors, and endogenous generation of thrombin and plasma. NURSINGTB.COM PTS: 1 DIF: Cognitive Level: Comprehension OBJ: Nursing Process: Assessment MSC: Client Needs: Physiologic Integrity 17. Which condition is caused by a virus that primarily infects a specific subset of T lymphocytes, the CD4+ T-cells? a. Wiskott-Aldrich syndrome b. Idiopathic thrombocytopenic purpura (ITP) c. Acquired immunodeficiency syndrome (AIDS) d. Severe combined immunodeficiency disease ANS: C AIDS is caused by the human immunodeficiency virus, which primarily attacks the CD4+ T-cells. Wiskott-Aldrich syndrome, ITP, and severe combined immunodeficiency disease are not viral illnesses. PTS: 1 DIF: Cognitive Level: Comprehension OBJ: Nursing Process: Assessment MSC: Client Needs: Physiologic Integrity 18. A young child with human immunodeficiency virus is receiving several antiretroviral drugs. What is the expected outcome of these drug therapies? a. Cure the disease b. Delay disease progression c. Prevent spread of disease d. Treat Pneumocystis jiroveci pneumonia ANS: B Although not a cure, these antiviral drugs can suppress viral replication, preventing further deterioration of the immune system, and delay disease progression. At this time cure is not possible. These drugs do not prevent the spread of the disease. Pneumocystis jiroveci prophylaxis is accomplished with antibiotics. PTS: 1 DIF: Cognitive Level: Comprehension OBJ: Nursing Process: Implementation MSC: Client Needs: Physiologic Integrity 19. Which immunization should be given with caution to children infected with human immunodeficiency virus? a. Influenza b. Varicella c. Pneumococcus d. Inactivated poliovirus ANS: B The children should be carefully evaluated before giving live viral vaccines such as varicella, measles, mumps, and rubella. The child must be immunocompetent and not have contact with other severely immunocompromised individuals. Influenza, pneumococcus, and inactivated poliovirus are not live vaccines. PTS: 1 DIF: Cognitive Level: Application OBJ: Nursing Process: Implementation MSC: Client Needs: Physiologic Integrity 20. An 8-year-old girl is receiving a bNloUoRdStIrNaGnTsfBu.sCiOonMwhen the nurse notes that she has developed precordial pain, dyspnea, distended neck veins, slight cyanosis, and a dry cough. These manifestations are most suggestive of what associated adverse reaction? a. Air embolism b. Allergic reaction c. Hemolytic reaction d. Circulatory overload ANS: D The signs of circulatory overload include distended neck veins, hypertension, crackles, dry cough, cyanosis, and precordial pain. Signs of air embolism are sudden difficulty breathing, sharp pain in the chest, and apprehension. Allergic reactions are manifested by urticaria, pruritus, flushing, asthmatic wheezing, and laryngeal edema. Hemolytic reactions are characterized by chills, shaking, fever, pain at infusion site, nausea, vomiting, tightness in chest, flank pain, red or black urine, and progressive signs of shock and renal failure. PTS: 1 DIF: Cognitive Level: Analysis OBJ: Nursing Process: Assessment MSC: Client Needs: Physiologic Integrity 21. The parents of a child diagnosed with aplastic anemia tell the nurse that a bone marrow transplant (BMT) may be necessary. What should the nurse recognize as important when discussing this with the family? a. BMT should be done at time of diagnosis. b. Parents and siblings of child have a 25% chance of being a suitable donor. c. Finding a suitable donor involves matching antigens from the human leukocyte antigen (HLA) system. d. If BMT fails, chemotherapy or radiotherapy must be continued. ANS: C The most successful BMTs come from suitable HLA-matched donors. The timing of a BMT depends on the disease process involved. It usually follows intensive high-dose chemotherapy and/or radiation therapy. Usually parents only share approximately 50% of the genetic material with their children. A one-in-four chance exists that two siblings will have two identical haplotypes and will be identically matched at the HLA loci. Discussing the continuation of chemotherapy or radiotherapy is not appropriate when planning the BMT. That decision will be made later. PTS: 1 DIF: Cognitive Level: Analysis OBJ: Nursing Process: Implementation MSC: Client Needs: Physiologic Integrity 22. What is the priority nursing intervention for a child hospitalized with hemarthrosis resulting from hemophilia? a. Immobilization and elevation of the affected joint b. Administration of acetaminophen for pain relief c. Assessment of the child's response to hospitalization d. Assessment of the impact of hospitalization on the family system ANS: A Immobilization and elevation of the joint will prevent further injury until bleeding is resolved. Although acetaminophen may help with pain associated with the treatment of hemarthrosis, it is not the priority nursing intervention. Assessment of a child's response to hospitalization is relevant to all hospitalized children; however, in this situation, psychosocial concerns are secondary to physiologic concernsN.UARSpIrNioGrTitBy.CnOurMsing concern for this child is the management of hemarthrosis. Assessing the impact of hospitalization on the family system is relevant to all hospitalized children; however, it is not the priority in this situation. PTS: 1 DIF: Cognitive Level: Application OBJ: Nursing Process: Implementation MSC: Client Needs: Physiologic Integrity 23. What is the most common mode of transmission of human immunodeficiency virus (HIV) in the pediatric population? a. Perinatal transmission b. Sexual abuse c. Blood transfusions d. Poor hand washing ANS: A Infected women can transmit the virus to their infants across the placenta during pregnancy, at delivery, and through breastfeeding. Cases of HIV infection from sexual abuse have been reported; however, perinatal transmission accounts for most pediatric HIV infections. In the past some children became infected with HIV through blood transfusions; however, improved laboratory screening has significantly reduced the probability of contracting HIV from blood products. Poor hand washing is not an etiology of HIV infection. PTS: 1 DIF: Cognitive Level: Comprehension OBJ: Nursing Process: Planning MSC: Client Needs: Physiologic Integrity 24. The nurse is planning activity for a 4-year-old child with anemia. Which activity should the nurse plan for this child? a. Game of “hide and seek” in the children's outdoor play area b. Participation in dance activities in the playroom c. Puppet play in the child's room d. A walk down to the hospital lobby ANS: C Because the basic pathologic process in anemia is a decrease in oxygen-carrying capacity, an important nursing responsibility is to assess the child's energy level and minimize excess demands. The child's level of tolerance for activities of daily living and play is assessed, and adjustments are made to allow as much self-care as possible without undue exertion. Puppet play in the child's room would not be overly tiring. Hide and seek, dancing, and walking to the lobby would not conserve the anemic child's energy. PTS: 1 DIF: Cognitive Level: Application OBJ: Nursing Process: Implementation MSC: Client Needs: Physiologic Integrity MULTIPLE RESPONSE 1. A school-age child is admitted in vaso-occlusive sickle cell crisis. The child's care should include which intervention? (Select all that apply.) a. Correction of acidosis b. Adequate hydration c. Pain management d. Administration of heparin e. Replacement of factor VIII ANS: B, C NURSINGTB.COM The management of crises includes adequate hydration, minimizing energy expenditures, pain management, electrolyte replacement, and blood component therapy if indicated. The acidosis will be corrected as the crisis is treated. Heparin and factor VIII are not indicated in the treatment of vaso-occlusive sickle cell crisis. PTS: 1 DIF: Cognitive Level: Analysis OBJ: Nursing Process: Implementation MSC: Client Needs: Physiologic Integrity 2. The nurse is caring for a child with aplastic anemia. Which nursing diagnoses are appropriate? (Select all that apply.) a. Acute Pain related to vaso-occlusion b. Risk for Infection related to inadequate secondary defenses or immunosuppression c. Ineffective Protection related to thrombocytopenia d. Ineffective Tissue Perfusion related to anemia e. Ineffective Protection related to abnormal clotting ANS: B, C, D These are appropriate nursing diagnosis for the nurse planning care for a child with aplastic anemia. Aplastic anemia is a condition in which the bone marrow ceases production of the cells it normally manufactures, resulting in pancytopenia. The child will have varying degrees of the disease depending on how low the values are for absolute neutrophil count (affecting the body's response to infection), platelet count (putting the child at risk for bleeding), and absolute reticulocyte count (causing the child to have anemia). Acute Pain related to vaso-occlusion is an appropriate nursing diagnosis for sickle cell anemia for the child in vaso-occlusive crisis, but it is not applicable to a child with aplastic anemia. Ineffective Protection related to abnormal clotting is an appropriate diagnosis for a child with hemophilia. PTS: 1 DIF: Cognitive Level: Application OBJ: Nursing Process: Planning MSC: Client Needs: Physiologic Integrity 3. The nurse is planning care for a school-age child admitted to the hospital with hemophilia. Which interventions should the nurse plan to implement for this child? (Select all that apply.) a. Fingersticks for blood work instead of venipunctures b. Avoidance of intramuscular (IM) injections c. Acetaminophen for mild pain control d. Soft toothbrush for dental hygiene e. Administration of packed red blood cells ANS: B, C, D Nurses should take special precautions when caring for a child with hemophilia to prevent the use of procedures that may cause bleeding, such as IM injections. The subcutaneous route is substituted for IM injections whenever possible. Venipunctures for blood samples are usually preferred for these children. There is usually less bleeding after the venipuncture than after finger or heel punctures. Neither aNsUpiRriSnINnGoTr Ba.nCyOaMspirin-containing compound should be used. Acetaminophen is a suitable aspirin substitute, especially for controlling mild pain. A soft toothbrush is recommended for dental hygiene to prevent bleeding from the gums. Packed red blood cells are not administered. The primary therapy for hemophilia is replacement of the missing clotting factor. The products available are factor VIII concentrates. PTS: 1 DIF: Cognitive Level: Application OBJ: Nursing Process: Implementation MSC: Client Needs: Physiologic Integrity: Physiologic Adaptation 4. Parents of a school-age child with hemophilia ask the nurse, “Which sports are recommended for children with hemophilia?” Which sports should the nurse recommend? (Select all that apply.) a. Soccer b. Swimming c. Basketball d. Golf e. Bowling ANS: B, D, E Because almost all persons with hemophilia are boys, the physical limitations in regard to active sports may be a difficult adjustment, and activity restrictions must be tempered with sensitivity to the child's emotional and physical needs. Use of protective equipment, such as padding and helmets, is particularly important, and noncontact sports, especially swimming, walking, jogging, tennis, golf, fishing, and bowling, are encouraged. Contact sports such as soccer and basketball are not recommended. PTS: 1 DIF: Cognitive Level: Application OBJ: Nursing Process: Teaching/Learning MSC: Client Needs: Health Promotion and Maintenance 5. Which should the nurse teach about prevention of sickle cell crises to parents of a preschool child with sickle cell disease? (Select all that apply.) a. Limit fluids at bedtime. b. Notify the health care provider if a fever of 38.5° C (101.3° F) or greater occurs. c. Give penicillin as prescribed. d. Use ice packs to decrease the discomfort of vaso-occlusive pain in the legs. e. Notify the health care provider if your child begins to develop symptoms of cold. ANS: B, C, E The most important issues to teach the family of a child with sickle cell anemia are to (1) seek early intervention for problems, such as a fever of 38.5° C (101.3° F) or greater; (2) give penicillin as ordered; (3) recognize signs and symptoms of splenic sequestration, as well as respiratory problems that can lead to hypoxia; and (4) treat the child normally. The nurse emphasizes the importance of adequate hydration to prevent sickling and to delay the adhesion-stasis-thrombosis-ischemia cycle. It is not sufficient to advise parents to “force fluids” or “encourage drinking.” TNhUeRySnINeeGdTsBp.CecOiMfic instructions on how many daily glasses or bottles of fluid are required. Many foods are also a source of fluid, particularly soups, flavored ice pops, ice cream, sherbet, gelatin, and puddings. Increased fluids combined with impaired kidney function result in the problem of enuresis. Parents who are unaware of this fact frequently use the usual measures to discourage bed-wetting, such as limiting fluids at night. Enuresis is treated as a complication of the disease, such as joint pain or some other symptom, to alleviate parental pressure on the child. Ice should not be used during a vaso-occlusive pain crisis because it vasoconstricts and impairs circulation even more. PTS: 1 DIF: Cognitive Level: Application OBJ: Nursing Process: Implementation MSC: Client Needs: Health Promotion and Maintenance MATCHING The nurse has initiated a blood transfusion on a preschool child. The child begins to exhibit signs of a transfusion reaction. Place in order the interventions the nurse should implement, sequencing from the highest priority to the lowest. a. Take the vital signs b. Stop the transfusion c. Notify the practitioner d. Maintain a patent intravenous (IV) line with normal saline 1. First priority 2. Second priority 3. Third priority 4. Fourth priority 1. ANS: B PTS: 1 DIF: Cognitive Level: Application OBJ: Nursing Process: Implementation MSC: Client Needs: Physiologic Integrity NOT: If a blood transfusion reaction of any type is suspected, stop the transfusion, take vital signs, maintain a patent IV line with normal saline and new tubing, notify the practitioner, and do not restart the transfusion until the child's condition has been medically evaluated. 2. ANS: A PTS: 1 DIF: Cognitive Level: Application OBJ: Nursing Process: Implementation MSC: Client Needs: Physiologic Integrity NOT: If a blood transfusion reaction of any type is suspected, stop the transfusion, take vital signs, maintain a patent IV line with normal saline and new tubing, notify the practitioner, and do not restart the transfusion until the child's condition has been medically evaluated. 3. ANS: D PTS: 1 DIF: Cognitive Level: Application OBJ: Nursing Process: Implementation MSC: Client Needs: Physiologic Integrity NOT: If a blood transfusion reaction of any type is suspected, stop the transfusion, take vital signs, maintain a patent IV line with normal saline and new tubing, notify the practitioner, and do not restart the transfusion until the child's condition has been medically evaluated. 4. ANS: C PTS: 1 DIF: Cognitive Level: Application OBJ: Nursing Process: Implementation MSC: Client Needs: Physiologic Integrity NOT: If a blood transfusion reaction of any type is suspected, stop the transfusion, take vital signs, maintain a patent IV line with normal saline and new tubing, notify the practitioner, and do not restart the transfusion until the child's condition has been medically evaluated. NURSINGTB.COM Chapter 44: The Child With Cancer Perry: Maternal Child Nursing Care, 6th Edition MULTIPLE CHOICE 1. Which is most descriptive of the pathophysiology of leukemia? a. Increased blood viscosity occurs. b. Thrombocytopenia (excessive destruction of platelets) occurs. c. Unrestricted proliferation of immature white blood cells (WBCs) occurs. d. First stage of coagulation process is abnormally stimulated. ANS: C Leukemia is a group of malignant disorders of the bone marrow and lymphatic system. It is defined as an unrestricted proliferation of immature WBCs in the blood-forming tissues of the body. Increased blood viscosity may occur secondary to the increased number of WBCs. Thrombocytopenia may occur secondary to the overproduction of WBCs in the bone marrow. The coagulation process is unaffected by leukemia. PTS: 1 DIF: Cognitive Level: Comprehension OBJ: Integrated Process: Nursing Process: Assessment MSC: Client Needs: Physiologic Integrity: Physiologic Adaptation 2. A boy with leukemia screams whenever he needs to be turned or moved. Which is the most probable cause of this pain? a. Edema b. Bone involvement c. Petechial hemorrhages d. Changes within the muscles ANS: B NURSINGTB.COM The invasion of the bone marrow with leukemic cells gradually causes weakening of the bone and a tendency toward fractures. As leukemic cells invade the periosteum, increasing pressure causes severe pain. Edema, petechial hemorrhages, and changes within the muscles would not cause severe pain. PTS: 1 DIF: Cognitive Level: Analysis OBJ: Integrated Process: Nursing Process: Planning MSC: Client Needs: Physiologic Integrity: Physiologic Adaptation 3. Myelosuppression, associated with chemotherapeutic agents or some malignancies such as leukemia can cause bleeding tendencies because of what resulting outcome? a. Decrease in leukocytes b. Increase in lymphocytes c. Vitamin C deficiency d. Decrease in blood platelets ANS: D The decrease in blood platelets secondary to the myelosuppression of chemotherapy can cause an increase in bleeding. The child and family should be alerted to avoid risk of injury. Decrease in leukocytes, increase in lymphocytes, and vitamin C deficiency would not affect bleeding tendencies. PTS: 1 DIF: Cognitive Level: Application OBJ: Integrated Process: Nursing Process: Planning MSC: Client Needs: Physiologic Integrity: Physiologic Adaptation 4. A youngster will receive a bone marrow transplant (BMT) made possible because an older siblings is a histocompatible donor. Which is this type of BMT called? a. Syngeneic b. Allogeneic c. Monoclonal d. Autologous ANS: B Allogeneic transplants are from another individual. Because he and his sibling are histocompatible, the BMT can be done. Syngeneic marrow is from an identical twin. There is no such thing as a monoclonal BMT. Autologous refers to the individual's own marrow. PTS: 1 DIF: Cognitive Level: Comprehension OBJ: Integrated Process: Nursing Process: Planning MSC: Client Needs: Physiologic Integrity: Physiologic Adaptation 5. Which immunization should be considered carefully before being given to a child receiving chemotherapy for cancer? a. Tetanus vaccine NURSINGTB.COM b. Inactivated poliovirus vaccine c. Diphtheria, pertussis, tetanus (DPT) d. Measles, rubella, mumps ANS: D The vaccine used for measles, mumps, and rubella is a live virus and can result in an overwhelming infection. Tetanus vaccine, inactivated poliovirus vaccine, and diphtheria, pertussis, tetanus (DPT) are not live virus vaccines. PTS: 1 DIF: Cognitive Level: Application OBJ: Integrated Process: Nursing Process: Assessment MSC: Client Needs: Physiologic Integrity: Pharmacologic and Parenteral Therapy 6. Which is often administered to prevent or control hemorrhage in a child diagnosed with cancer? a. Nitrosoureas b. Platelets c. Whole blood d. Corticosteroids ANS: B Most bleeding episodes can be prevented or controlled with the administration of platelet concentrate or platelet-rich plasma. Nitrosoureas, whole blood, and corticosteroids would not prevent or control hemorrhage. PTS: 1 DIF: Cognitive Level: Application OBJ: Integrated Process: Nursing Process: Planning MSC: Client Needs: Physiologic Integrity: Pharmacologic and Parenteral Therapy 7. The nurse is administering an IV chemotherapeutic agent to a child diagnosed with leukemia. The child suddenly begins to wheeze and have severe urticaria. Which is the most appropriate nursing action? a. Stop drug infusion immediately. b. Recheck rate of drug infusion. c. Observe child closely for next 10 minutes. d. Explain to child that this is an expected side effect. ANS: A If an allergic reaction is suspected, the drug should be immediately discontinued. Any drug in the line should be withdrawn, and a normal saline infusion begun to keep the line open. Rechecking the rate of drug infusion, observing the child closely for next 10 minutes, and explaining to the child that this is an expected side effect can all be done after the drug infusion is stopped and the child is evaluated. PTS: 1 DIF: Cognitive Level: Application OBJ: Integrated Process: Nursing Process: Implementation MSC: Client Needs: Physiologic Integrity: Pharmacologic and Parenteral Therapy 8. A school-age child diagnosed with leukemia experienced severe nausea and vomiting when receiving chemotherapy for the first time. Which is the most appropriate nursing action to prevent or minimize these reactions with subsequent treatments? a. Encourage drinking large amoNuUntRsSoINf GfaTvBo.rCiOteMfluids. b. Encourage child to take nothing by mouth (remain NPO) until nausea and vomiting subside. c. Administer an antiemetic at least 30 minutes before chemotherapy begins. d. Administer an antiemetic as soon as child has nausea. ANS: C The most beneficial regimen to minimize nausea and vomiting associated with chemotherapy is to administer the antiemetic 30 minutes to an hour before the chemotherapy is begun. The goal is to prevent anticipatory symptoms. Drinking fluids will add to the discomfort of the nausea and vomiting. Waiting until nausea and vomiting subside will help with this episode, but the child will have the discomfort and be at risk for dehydration. Administering an antiemetic as soon as the child has nausea does not prevent anticipatory nausea. PTS: 1 DIF: Cognitive Level: Application OBJ: Integrated Process: Nursing Process: Implementation MSC: Client Needs: Physiologic Integrity: Pharmacologic and Parenteral Therapy 9. A young child diagnosed with leukemia is experiencing anorexia and severe stomatitis. The nurse should suggest that the parents try which intervention? a. Relax any eating pressures b. Firmly insist that child eat normally c. Begin gavage feedings to supplement diet d. Serve foods that are either hot or cold ANS: A A multifaceted approach is necessary for children with severe stomatitis and anorexia. First, the parents should relax eating pressures. The nurse should suggest that the parents try soft, bland foods; normal saline or bicarbonate mouthwashes; and local anesthetics. The stomatitis is a temporary condition. The child can resume good food habits as soon as the condition resolves. PTS: 1 DIF: Cognitive Level: Application OBJ: Integrated Process: Teaching/Learning MSC: Client Needs: Physiologic Integrity: Physiologic Adaptation 10. The nurse is preparing a child for possible alopecia from chemotherapy. Which intervention should be included? a. Explain to child that hair usually regrows in 1 year. b. Advise child to expose head to sunlight to minimize alopecia. c. Explain to child that wearing a hat or scarf is preferable to wearing a wig. d. Explain to child that when hair regrows, it may have a slightly different color or texture. ANS: D Alopecia is a side effect of certain chemotherapeutic agents. When the hair regrows, it may be a different color or texture. The hair usually grows back within 3 to 6 months after cessation of treatment. The head should be protected from sunlight to avoid sunburn. Children should choose the head covering they prefer. PTS: 1 DIF: Cognitive Level: Application OBJ: Integrated Process: Teaching/Learning MSC: Client Needs: Physiologic IntNegUriRtyS:INPGhyTsBio.CloOgMic Adaptation 11. Which is a common clinical manifestation of Hodgkin disease? a. Petechiae b. Bone and joint pain c. Painful, enlarged lymph nodes d. Enlarged, firm, nontender lymph nodes ANS: D Asymptomatic, enlarged, cervical or supraclavicular lymphadenopathy is the most common presentation of Hodgkin disease. Petechiae are usually associated with leukemia. Bone and joint pain are not likely in Hodgkin disease. The enlarged nodes are rarely painful. PTS: 1 DIF: Cognitive Level: Comprehension OBJ: Integrated Process: Nursing Process: Assessment MSC: Client Needs: Physiologic Integrity: Physiologic Adaptation 12. The nurse is admitting a child with a Wilms' tumor. Which is the initial assessment finding associated with this tumor? a. Abdominal swelling b. Weight gain c. Hypotension d. Increased urinary output The initial assessment finding with a Wilms' (kidney) tumor is abdominal swelling. Weight loss, not weight gain, may be a finding. Hypertension occasionally occurs with a Wilms' tumor. Urinary output is not increased, but hematuria may be noted. PTS: 1 DIF: Cognitive Level: Comprehension OBJ: Integrated Process: Nursing Process: Assessment MSC: Client Needs: Physiologic Integrity: Physiologic Adaptation 13. What is the most common clinical manifestation(s) of brain tumors in children? a. Irritability b. Seizures c. Headaches and vomiting d. Fever and poor fine motor control ANS: C Headaches, especially on awakening, and vomiting that is not related to feeding are the most common clinical manifestation(s) of brain tumors in children. Irritability, seizures, and fever and poor fine motor control are clinical manifestations of brain tumors, but headaches and vomiting are the most common. PTS: 1 DIF: Cognitive Level: Comprehension OBJ: Integrated Process: Nursing Process: Assessment MSC: Client Needs: Physiologic Integrity: Physiologic Adaptation 14. A 5 year old is being prepared for surgery to remove a brain tumor. Nursing actions should be based on which statement? a. Removal of tumor will stop the various symptoms. b. Usually the postoperative dresNsiUnRgScINovGeTrBs .tChOeMentire scalp. c. He is not old enough to be concerned about his head being shaved. d. He is not old enough to Comprehension the significance of the brain. ANS: B The child should be told what he will look and feel like after surgery. This includes the size of the dressing. The nurse can demonstrate on a doll the expected size and shape of the dressing. Some of the symptoms may be alleviated by the removal of the tumor, but postsurgical headaches and cerebellar symptoms such as ataxia may be aggravated. Children should be prepared for the loss of their hair, and it should be removed in a sensitive, positive manner if the child is awake. Children at this age have poorly defined body boundaries and little knowledge of internal organs. Intrusive experiences are frightening, especially those that disrupt the integrity of the skin. PTS: 1 DIF: Cognitive Level: Application OBJ: Integrated Process: Teaching/Learning MSC: Client Needs: Health Promotion and Maintenance 15. Which statement appropriately describes a neuroblastoma? a. It is considered to be a “silent” tumor. b. Early diagnosis is usually possible because of the obvious clinical manifestations. c. It is the most common brain tumor in young children. d. It is the most common benign tumor in young children. Neuroblastoma is a silent tumor with few symptoms. In more than 70% of cases, diagnosis is made after metastasis occurs, with the first signs caused by involvement in the nonprimary site. In only 30% of cases is diagnosis made before metastasis. Neuroblastomas are the most common malignant extracranial solid tumors in children. The majority of tumors develop in the adrenal glands or the retroperitoneal sympathetic chain. They are not benign; they metastasize. PTS: 1 DIF: Cognitive Level: Application OBJ: Integrated Process: Teaching/Learning MSC: Client Needs: Physiologic Integrity: Physiologic Adaptation 16. A nurse is conducting a staff in-service on childhood cancers. Which is the primary site of osteosarcoma? a. Femur b. Humerus c. Pelvis d. Tibia ANS: A Osteosarcoma is the most frequently encountered malignant bone cancer in children. The peak incidence is between ages 10 and 25 years. More than half occur in the femur. After the femur, most of the remaining sites are the humerus, tibia, pelvis, jaw, and phalanges. PTS: 1 DIF: Cognitive Level: Comprehension OBJ: Integrated Process: Teaching/Learning MSC: Client Needs: Physiologic Integrity: Reduction of Risk Potential 17. The nurse is taking care of an adoNleUscReSnINt wGTitBh.CoOstMeosarcoma. The parents ask the nurse about treatment. The nurse should make which accurate response about treatment for osteosarcoma? a. Treatment usually consists of surgery and chemotherapy. b. Amputation of the affected extremity is rarely necessary. c. Intensive irradiation is the primary treatment. d. Bone marrow transplantation offers the best chance of long-term survival. ANS: A The optimal therapy for osteosarcoma is a combination of surgery and chemotherapy. Intensive irradiation and bone marrow transplantation are usually not part of the therapeutic management. PTS: 1 DIF: Cognitive Level: Comprehension OBJ: Integrated Process: Nursing Process: Implementation MSC: Client Needs: Physiologic Integrity: Physiologic Adaptation 18. An adolescent with osteosarcoma is scheduled for a leg amputation in 2 days. The nurse's approach should include which action? a. Answering questions with straightforward honesty b. Avoiding discussing the seriousness of the condition c. Explaining that, although the amputation is difficult, it will cure the cancer d. Assisting the adolescent in accepting the amputation as better than a long course of chemotherapy Honesty is essential to gain the child's cooperation and trust. The diagnosis of cancer should not be disguised with falsehoods. The adolescent should be prepared for the surgery so he or she has time to reflect on the diagnosis and subsequent treatment. This allows questions to be answered. To accept the need for radical surgery, the child must be aware of the lack of alternatives for treatment. Amputation is necessary, but it will not guarantee a cure. Chemotherapy is an integral part of the therapy with surgery. The child should be informed of the need for chemotherapy and its side effects before surgery. PTS: 1 DIF: Cognitive Level: Application OBJ: Integrated Process: Teaching/Learning MSC: Client Needs: Health Promotion and Maintenance MULTIPLE RESPONSE 1. Which expected appearance will the nurse explain to parents of an infant returning from surgery after an enucleation was performed to treat retinoblastoma? (Select all that apply.) a. A lot of drainage will come from the affected socket. b. The face may be edematous or ecchymotic. c. The eyelids will be sutured shut for the first week. d. There will be an eye pad dressing taped over the surgical site. e. The implanted sphere is covered with conjunctiva and resembles the lining of the mouth. ANS: B, D, E After enucleation surgery, the parents are prepared for the child's facial appearance. An eye patch is in place, and the child's face may be edematous or ecchymotic. Parents often fear seeing the surgical site because thNeyURimSIaNgGinTeBa.CcOaMvity in the skull. A surgically implanted sphere maintains the shape of the eyeball, and the implant is covered with conjunctiva. When the eyelids are open, the exposed area resembles the mucosal lining of the mouth. The dressing, consisting of an eye pad taped over the surgical site, is changed daily. The wound itself is clean and has little or no drainage. So expecting a lot of drainage is not accurate to tell parents. The eyelids are not sutured shut after enucleation surgery. PTS: 1 DIF: Cognitive Level: Application OBJ: Integrated Process: Teaching/Learning MSC: Client Needs: Health Promotion and Maintenance 2. The treatment of brain tumors in children consists of which therapies? (Select all that apply.) a. Surgery b. Bone marrow transplantation c. Chemotherapy d. Stem cell transplantation e. Radiation f. Myelography ANS: A, C, E Treatment for brain tumors in children may consist of surgery, chemotherapy, and radiotherapy alone or in combination. Bone marrow and stem cell transplantation therapies are used for leukemia, lymphoma, and other solid tumors where myeloablative therapies are used. Myelography is a radiographic examination after an intrathecal injection of contrast medium. It is not a treatment. PTS: 1 DIF: Cognitive Level: Comprehension OBJ: Integrated Process: Nursing Process: Assessment MSC: Client Needs: Physiologic Integrity: Physiologic Adaptation 3. A clinic nurse is conducting a staff in-service for other clinic nurses about signs and symptoms of a rhabdomyosarcoma tumor. Which should be included in the teaching session? (Select all that apply.) a. Bone fractures b. Abdominal mass c. Sore throat and ear pain d. Headache e. Ecchymosis of conjunctiva ANS: B, C, E The initial signs and symptoms of rhabdomyosarcoma tumors are related to the site of the tumor and compression of adjacent organs. Some tumor locations, such as the orbit, manifest early in the course of the illness. Other tumors, such as those of the retroperitoneal area, only produce symptoms when they are relatively large and compress adjacent organs. Unfortunately, many of the signs and symptoms attributable to rhabdomyosarcoma are vague and frequently suggest a common childhood illness, such as “earache” or “runny nose.” An abdominal mass, sore throat and eNarUpRaSiInN,GaTnBd.CeOccMhymosis of conjunctiva are signs of a rhabdomyosarcoma tumor. Bone fractures would be seen in osteosarcoma, and a headache is a sign of a brain tumor. PTS: 1 DIF: Cognitive Level: Application OBJ: Integrated Process: Teaching/Learning MSC: Client Needs: Physiologic Integrity: Reduction of Risk Potential Chapter 45: The Child With Genitourinary Dysfunction Perry: Maternal Child Nursing Care, 6th Edition MULTIPLE CHOICE 1. Which diagnostic test allows visualization of the renal parenchyma and renal pelvis without exposure to external beam radiation or radioactive isotopes? a. Renal ultrasound b. Computed tomography c. Intravenous pyelography d. Voiding cystourethrography ANS: A The transmission of ultrasonic waves through the renal parenchyma allows visualization of the renal parenchyma and renal pelvis without exposure to external beam radiation or radioactive isotopes. Computed tomography uses external radiation, and sometimes contrast media are used. Intravenous pyelography uses contrast medium and external radiation for x-ray films. Contrast medium is injected into the bladder through the urethral opening for voiding cystourethrography. External radiation for x-ray films is used before, during, and after voiding. PTS: 1 DIF: Cognitive Level: Comprehension OBJ: Nursing Process: Assessment MSC: Client Needs: Physiologic Integrity 2. Which diagnostic finding is present when a child has primary nephrotic syndrome? a. Hyperalbuminemia b. Positive ASO titer c. Leukocytosis d. Proteinuria ANS: D NURSINGTB.COM Large amounts of protein are lost through the urine as a result of an increased permeability of the glomerular basement membrane. Hypoalbuminemia is present because of loss of albumin through the defective glomerulus and the liver's inability to synthesize proteins to balance the loss. ASO titer is negative in a child with primary nephrotic syndrome. Leukocytosis is not a diagnostic finding in primary nephrotic syndrome. PTS: 1 DIF: Cognitive Level: Comprehension OBJ: Nursing Process: Assessment MSC: Client Needs: Physiologic Integrity 3. Which factor predisposes a child to urinary tract infections? a. Increased fluid intake b. Short urethra in young girls c. Prostatic secretions in males d. Frequent emptying of the bladder ANS: B The short urethra in females provides a ready pathway for invasions of organisms. Increased fluid intake and frequent bladder emptying offer protective measures against urinary tract infections. Prostatic secretions have antibacterial properties that inhibit bacteria. PTS: 1 DIF: Cognitive Level: Comprehension OBJ: Nursing Process: Assessment MSC: Client Needs: Physiologic Integrity 4. What should the nurse recommend to prevent urinary tract infections in young girls? a. Wearing cotton underpants b. Limiting bathing as much as possible c. Increasing fluids; decreasing salt intake d. Cleansing the perineum with water after voiding ANS: A Cotton underpants are preferable to nylon underpants. No evidence exists that limiting bathing, increasing fluids, decreasing salt intake, or cleansing the perineum with water decreases urinary tract infections in young girls. PTS: 1 DIF: Cognitive Level: Comprehension OBJ: Nursing Process: Implementation MSC: Client Needs: Physiologic Integrity 5. The nurse understands that hypospadias refers to what urinary anomaly? a. Absence of a urethral opening. b. Penis shorter than usual for age. c. Urethral opening along dorsal surface of penis. d. Urethral opening along ventral surface of penis. ANS: D Hypospadias is a congenital condition in which the urethral opening is located anywhere along the ventral surface of the penis. The urethral opening is present, but not at the glans. Hypospadias does not refer to theNsiUzReSoINf tGhTeBp.CenOiMs. When the urethral opening is along the dorsal surface of the penis, it is known as epispadias. PTS: 1 DIF: Cognitive Level: Comprehension OBJ: Nursing Process: Assessment MSC: Client Needs: Physiologic Integrity 6. What is the term used to identify when the meatal opening is located on the dorsal surface of the penis? a. Chordee b. Phimosis c. Epispadias d. Hypospadias ANS: C Phimosis is the narrowing or stenosis of the preputial opening of the foreskin. Chordee is the ventral curvature of the penis. Epispadias is the meatal opening on the dorsal surface of the penis. Hypospadias is a congenital condition in which the urethral opening is located anywhere along the ventral surface of the penis. PTS: 1 DIF: Cognitive Level: Comprehension OBJ: Nursing Process: Assessment MSC: Client Needs: Physiologic Integrity 7. An objective of care for the child with nephrosis is what desired outcome? a. Reduced blood pressure b. Reduced excretion of urinary protein c. Increased excretion of urinary protein d. Increased ability of tissues to retain fluid ANS: B The objectives of therapy for the child with nephrosis include reduction of the excretion of urinary protein, reduction of fluid retention, prevention of infection, and minimizing of complications associated with therapy. Blood pressure is usually not elevated in nephrosis. Increased excretion of urinary protein and increased ability of tissues to retain fluid are part of the disease process and must be reversed. PTS: 1 DIF: Cognitive Level: Comprehension OBJ: Nursing Process: Planning MSC: Client Needs: Physiologic Integrity 8. What intervention is a component of the therapeutic management of nephrosis? a. Corticosteroids b. Antihypertensive agents c. Long-term diuretics d. Increased fluids to promote diuresis ANS: A Corticosteroids are the first line of therapy for nephrosis. Response is usually seen within 7 to 21 days. Antihypertensive agents and long-term diuretic therapy are usually not necessary. A diet that has fluid and salt restrictions may be indicated. PTS: 1 DIF: Cognitive Level: Comprehension OBJ: Nursing Process: Implementation MSC: Client Needs: Physiologic Integrity 9. What is a common side effect of cNoUrtRicSoINstGerToBi.dCOthMerapy? a. Fever b. Hypertension c. Weight loss d. Increased appetite ANS: D Side effects of corticosteroid therapy include an increased appetite. Fever is not a side effect of therapy. It may be an indication of infection. Hypertension is not usually associated with initial corticosteroid therapy. Weight gain, not weight loss, is associated with corticosteroid therapy. PTS: 1 DIF: Cognitive Level: Comprehension OBJ: Nursing Process: Implementation MSC: Client Needs: Physiologic Integrity 10. The nurse closely monitors the temperature of a child diagnosed with nephrosis. The purpose of this is to detect an early sign of what undesirable outcome? a. Infection b. Hypertension c. Encephalopathy d. Edema ANS: A Infection is a constant source of danger to edematous children and those receiving corticosteroid therapy. An increased temperature could be an indication of an infection, but it is not an indication of hypertension or edema. Encephalopathy is not a complication usually associated with nephrosis. The child will most likely have neurologic signs and symptoms. PTS: 1 DIF: Cognitive Level: Application OBJ: Nursing Process: Assessment MSC: Client Needs: Physiologic Integrity 11. The diet of a child with nephrosis usually includes requirement? a. High protein b. Salt restriction c. Low fat d. High carbohydrate ANS: B Salt is usually restricted (but not eliminated) during the edema phase. The child has very little appetite during the acute phase. Favorite foods are provided (with the exception of high-salt ones) in an attempt to provide nutritionally complete meals. PTS: 1 DIF: Cognitive Level: Application OBJ: Nursing Process: Assessment MSC: Client Needs: Physiologic Integrity 12. A child is admitted with acute glomerulonephritis. The nurse would expect the urinalysis during this acute phase to show: a. bacteriuria and hematuria. b. hematuria and proteinuria. c. bacteriuria and increased specific gravity. d. proteinuria and decreased specNiUfiRc SgIrNaGviTtBy..COM ANS: B Urinalysis during the acute phase characteristically shows hematuria and proteinuria. Bacteriuria and changes in specific gravity are not usually present during the acute phase. PTS: 1 DIF: Cognitive Level: Comprehension OBJ: Nursing Process: Assessment MSC: Client Needs: Physiologic Integrity 13. What is the most appropriate nursing diagnosis for the child with acute glomerulonephritis? a. Risk for Injury related to malignant process and treatment. b. Deficient Fluid Volume related to excessive losses. c. Excess Fluid Volume related to decreased plasma filtration. d. Excess Fluid Volume related to fluid accumulation in tissues and third spaces. ANS: C Glomerulonephritis has a decreased filtration of plasma. The decrease in plasma filtration results in an excessive accumulation of water and sodium that expands plasma and interstitial fluid volumes, leading to circulatory congestion and edema. No malignant process is involved in acute glomerulonephritis. A fluid volume excess is found. The fluid accumulation is secondary to the decreased plasma filtration, not fluid accumulation. PTS: 1 DIF: Cognitive Level: Analysis OBJ: Nursing Process: Diagnosis MSC: Client Needs: Physiologic Integrity 14. What should the nurse include in a teaching plan for the parents of a child with vesicoureteral reflux? a. The importance of taking prophylactic antibiotics b. Suggestions for how to maintain fluid restrictions c. The use of bubble baths as an incentive to increase bath time d. The need for the child to hold urine for 6 to 8 hours ANS: A Prophylactic antibiotics are used to prevent urinary tract infections (UTIs) in a child with vesicoureteral reflux, although this treatment plan has become controversial. Fluids are not restricted when a child has vesicoureteral reflux. In fact, fluid intake should be increased as a measure to prevent UTIs. Bubble baths should be avoided to prevent urethral irritation and possible UTI. To prevent UTIs, the child should be taught to void frequently and never resist the urge to urinate. PTS: 1 DIF: Cognitive Level: Application OBJ: Nursing Process: Implementation MSC: Client Needs: Physiologic Integrity 15. What is the most common cause of acute renal failure in children? a. Pyelonephritis b. Tubular destruction c. Urinary tract obstruction d. Severe dehydration ANS: D The most common cause of acute renal failure in children is dehydration or other causes of poor perfusion that may respond to restoration of fluid volume. Pyelonephritis and tubular destruction are not common causeNsUoRf SaIcNuGteTBre.CnOalMfailure in children. Obstructive uropathy may cause acute renal failure, but it is not the most common cause. PTS: 1 DIF: Cognitive Level: Comprehension OBJ: Nursing Process: Assessment MSC: Client Needs: Physiologic Integrity 16. The nurse, caring for a child with acute renal failure, should recognize event as a sign of hyperkalemia? a. Dyspnea b. Seizure c. Oliguria d. Cardiac arrhythmia ANS: D Hyperkalemia is the most common threat to the life of the child. Signs of hyperkalemia include electrocardiographic anomalies such as prolonged QRS complex, depressed ST segments, peaked T waves, bradycardia, or heart block. Dyspnea, seizure, and oliguria are not manifestations of hyperkalemia. PTS: 1 DIF: Cognitive Level: Comprehension OBJ: Nursing Process: Assessment MSC: Client Needs: Physiologic Integrity 17. When a child diagnosed with chronic renal failure, the progressive deterioration produces a variety of clinical and biochemical disturbances that eventually are manifested in the clinical syndrome known as what? a. Uremia b. Oliguria c. Proteinuria d. Pyelonephritis ANS: A Uremia is the retention of nitrogenous products, producing toxic symptoms. Oliguria is diminished urine output. Proteinuria is the presence of protein, usually albumin, in the urine. Pyelonephritis is an inflammation of the kidney and renal pelvis. PTS: 1 DIF: Cognitive Level: Comprehension OBJ: Nursing Process: Diagnosis MSC: Client Needs: Physiologic Integrity 18. What major complication is noted in a child with chronic renal failure? a. Hypokalemia b. Metabolic alkalosis c. Water and sodium retention d. Excessive excretion of blood urea nitrogen ANS: C Chronic renal failure leads to water and sodium retention, which contributes to edema and vascular congestion. Hyperkalemia, metabolic acidosis, and retention of blood urea nitrogen are complications of chronic renal failure. PTS: 1 DIF: Cognitive Level: Analysis OBJ: Nursing Process: Assessment MSC: Client Needs: Physiologic Integrity 19. Which clinical manifestation wouNldUbReSIsNeGenTBin.CaOcMhild with chronic renal failure? a. Hypotension b. Massive hematuria c. Hypokalemia d. Unpleasant “uremic” breath odor ANS: D Children with chronic renal failure have a characteristic breath odor resulting from the retention of waste products. Hypertension may be a complication of chronic renal failure. With chronic renal failure, little or no urine output occurs. Hyperkalemia is a concern in chronic renal failure. PTS: 1 DIF: Cognitive Level: Comprehension OBJ: Nursing Process: Assessment MSC: Client Needs: Physiologic Integrity 20. One of the clinical manifestations of chronic renal failure is uremic frost. Which best describes this term? a. Deposits of urea crystals in urine b. Deposits of urea crystals on skin c. Overexcretion of blood urea nitrogen d. Inability of body to tolerate cold temperatures ANS: B Uremic frost is the deposition of urea crystals on the skin, not in the urine. The kidneys are unable to excrete blood urea nitrogen, leading to elevated levels. There is no relation between cold temperatures and uremic frost. PTS: 1 DIF: Cognitive Level: Comprehension OBJ: Nursing Process: Assessment MSC: Client Needs: Physiologic Integrity 21. Calcium carbonate is given with meals to a child with chronic renal disease. The purpose of this is to achieve which desired result? a. Prevent vomiting b. Bind phosphorus c. Stimulate appetite d. Increase absorption of fat-soluble vitamins ANS: B Oral calcium carbonate preparations combine with phosphorus to decrease gastrointestinal absorption and the serum levels of phosphate; serum calcium levels are increased by the calcium carbonate, and vitamin D administration is necessary to increase calcium absorption. Calcium carbonate does not prevent vomiting, stimulate appetite, or increase the absorption of fat-soluble vitamins. PTS: 1 DIF: Cognitive Level: Comprehension OBJ: Nursing Process: Implementation MSC: Client Needs: Physiologic Integrity 22. The diet of a child with chronic renal failure is usually characterized as: a. high in protein. b. low in vitamin D. c. low in phosphorus. NURSINGTB.COM d. supplemented with vitamins A, E, and K. ANS: C Dietary phosphorus is controlled to prevent or control the calcium/phosphorus imbalance by the reduction of protein and milk intake. Protein should be limited in chronic renal failure to decrease intake of phosphorus. Vitamin D therapy is administered in chronic renal failure to increase calcium absorption. Supplementation with vitamins A, E, and K is not part of dietary management in chronic renal disease. PTS: 1 DIF: Cognitive Level: Comprehension OBJ: Nursing Process: Implementation MSC: Client Needs: Physiologic Integrity 23. The nurse is caring for an adolescent who has just started dialysis. The child seems always angry, hostile, or depressed. What should the nurse contribute this behavior to? a. Neurologic manifestations that occur with dialysis b. Physiologic manifestations of renal disease c. Adolescents having few coping mechanisms d. Adolescents often resenting the control and enforced dependence imposed by dialysis ANS: D Older children and adolescents need control. The necessity of dialysis forces the adolescent into a dependent relationship, which results in these behaviors. Neurologic manifestations that occur with dialysis and physiologic manifestations of renal disease are a function of the age of the child, not neurologic or physiologic manifestations of the dialysis. Adolescents do have coping mechanisms, but they need to have some control over their disease management. PTS: 1 DIF: Cognitive Level: Analysis OBJ: Nursing Process: Diagnosis MSC: Client Needs: Health Promotion and Maintenance 24. What is an advantage of peritoneal dialysis? a. Treatments are done in hospitals. b. Protein loss is less extensive. c. Dietary limitations are not necessary. d. Parents and older children can perform treatments. ANS: D Peritoneal dialysis is the preferred form of dialysis for parents, infants, and children who wish to remain independent. Parents and older children can perform the treatments themselves. Treatments can be done at home. Protein loss is not significantly different. The dietary limitations are necessary, but they are not as stringent as those for hemodialysis. PTS: 1 DIF: Cognitive Level: Comprehension OBJ: Nursing Process: Planning MSC: Client Needs: Physiologic Integrity 25. Which statement is descriptive of renal transplantation in children? a. It is an acceptable means of treatment after age 10 years. b. It is preferred means of renal replacement therapy in children. c. Children can receive kidneys oNnUlyRSfIrNoGmToBt.hCeOrMchildren. d. The decision for transplantation is difficult since a relatively normal lifestyle is not possible. ANS: B Renal transplantation offers the opportunity for a relatively normal lifestyle versus dependence on dialysis and is the preferred means of renal replacement therapy in end-stage renal disease. It can be done in children as young as age 6 months. Both children and adults can serve as donors for renal transplant purposes. PTS: 1 DIF: Cognitive Level: Comprehension OBJ: Nursing Process: Assessment MSC: Client Needs: Physiologic Integrity 26. Which intervention is appropriate when examining a male infant for cryptorchidism? a. Cooling the examiner's hands b. Taking a rectal temperature c. Eliciting the cremasteric reflex d. Warming the room ANS: D Cryptorchidism is the failure of one or both testes to descend normally through inguinal canal. For the infant's comfort, the infant should be examined in a warm room with the examiner's hands warmed. Testes can retract into the inguinal canal if the infant is upset or cold. Examining the infant with cold hands is uncomfortable for the infant and likely to cause the infant's testes to retract into the inguinal canal. It may also cause the infant to be uncooperative during the examination. A rectal temperature yields no information about cryptorchidism. Testes can retract into the inguinal canal if the cremasteric reflex is elicited. This can lead to an incorrect diagnosis. PTS: 1 DIF: Cognitive Level: Application OBJ: Nursing Process: Implementation MSC: Client Needs: Physiologic Integrity 27. The narrowing of preputial opening of foreskin is referred to as what? a. Chordee b. Phimosis c. Epispadias d. Hypospadias ANS: B Phimosis is the narrowing or stenosis of the preputial opening of the foreskin. Chordee is the ventral curvature of the penis. Epispadias is the meatal opening on the dorsal surface of the penis. Hypospadias is a congenital condition in which the urethral opening is located anywhere along the ventral surface of the penis. PTS: 1 DIF: Cognitive Level: Comprehension OBJ: Nursing Process: Assessment MSC: Client Needs: Physiologic Integrity 28. The nurse is admitting a school-agNeUcRhSiIlNdGinTBa.cCuOteMrenal failure with reduced glomerular filtration rate. Which urine test is the most useful clinical indication of glomerular filtration rate? a. pH b. Osmolality c. Creatinine clearance d. Protein level ANS: C The most useful clinical indication of glomerular filtration is the clearance of creatinine. It is a substance that is freely filtered by the glomerulus and secreted by the renal tubule cells. The pH and osmolality are not estimates of glomerular filtration. Although protein in the urine demonstrates abnormal glomerular permeability, it is not a measure of filtration rate. PTS: 1 DIF: Cognitive Level: Comprehension OBJ: Nursing Process: Assessment MSC: Client Needs: Physiologic Integrity 29. The nurse is conducting an assessment on a school-age child with urosepsis. Which assessment finding should the nurse expect? a. Fever with a positive blood culture b. Proteinuria and edema c. Oliguria and hypertension d. Anemia and thrombocytopenia ANS: A Symptoms of urosepsis include a febrile urinary tract infection coexisting with systemic signs of bacterial illness; blood culture reveals the presence of a urinary pathogen. Proteinuria and edema are symptoms of minimal change nephrotic syndrome. Oliguria and hypertension are symptoms of acute glomerulonephritis. Anemia and thrombocytopenia are symptoms of hemolytic uremic syndrome. PTS: 1 DIF: Cognitive Level: Comprehension OBJ: Nursing Process: Assessment MSC: Client Needs: Physiologic Integrity 30. A mother asks the nurse what would be the first indication that acute glomerulonephritis is improving. The nurse's best response should be to identify which occurrence? a. Blood pressure will stabilize. b. Child will have more energy. c. Urine will be free of protein. d. Urinary output will increase. ANS: D An increase in urinary output may signal resolution of the acute glomerulonephritis. If blood pressure is elevated, stabilization usually occurs with the improvement in renal function. The child having more energy and the urine being free of protein are related to the improvement in urinary output. PTS: 1 DIF: Cognitive Level: Application OBJ: Nursing Process: Implementation MSC: Client Needs: Physiologic Integrity 31. The nurse is teaching the parent about the diet of a child experiencing severe edema associated with acute glomerulonephritis. Which information should the nurse include in the teaching? NURSINGTB.COM a. “You will need to decrease the number of calories in your child's diet.” b. “Your child's diet will need an increased amount of protein.” c. “You will need to avoid adding salt to your child's food.” d. “Your child's diet will consist of low-fat, low-carbohydrate foods.” ANS: C For most children, a regular diet is allowed, but it should contain no added salt. The child should be offered a regular diet with favorite foods. Severe sodium restrictions are not indicated. PTS: 1 DIF: Cognitive Level: Application OBJ: Nursing Process: Implementation MSC: Client Needs: Physiologic Integrity 32. A preschool child is being admitted to the hospital with dehydration and a urinary tract infection (UTI). Which urinalysis result should the nurse expect with these conditions? a. WBC <1; specific gravity 1.008 b. WBC <2; specific gravity 1.025 c. WBC >2; specific gravity 1.016 d. WBC >2; specific gravity 1.030 ANS: D The white blood cell (WBC) count in a routine urinalysis should be <1 or 2. Over that amount indicates a urinary tract inflammatory process. The urinalysis specific gravity for children with normal fluid intake is 1.016 to 1.022. When the specific gravity is high, dehydration is indicated. A low specific gravity is seen with excessive fluid intake, distal tubular dysfunction, or insufficient antidiuretic hormone secretion. PTS: 1 DIF: Cognitive Level: Analysis OBJ: Nursing Process: Assessment MSC: Client Needs: Physiologic Integrity 33. The nurse is conducting teaching for an adolescent being discharged to home after a renal transplantation. The adolescent needs further teaching if which statement is made? a. “I will report any fever to my primary health care provider.” b. “I am glad I only have to take the immunosuppressant medication for 2 weeks.” c. “I will observe my incision for any redness or swelling.” d. “I won't miss doing kidney dialysis every week.” ANS: B The immunosuppressant medications are taken indefinitely after a renal transplantation, so they should not be discontinued after 2 weeks. Reporting a fever and observing an incision for redness and swelling are accurate statements. The adolescent is correct in indicating dialysis will not need to be done after the transplantation. PTS: 1 DIF: Cognitive Level: Analysis OBJ: Nursing Process: Implementation MSC: Client Needs: Physiologic Integrity 34. The nurse is teaching parents of a child with chronic renal failure (CRF) about the use of recombinant human erythropoietin (rHuEPO) subcutaneous injections. Which statement indicates the parents have understNooUdRSthINe GteTaBc.hCiOnMg? a. “These injections will help with the hypertension.” b. “We're glad the injections only need to be given once a month.” c. “The red blood cell count should begin to improve with these injections.” d. “Urine output should begin to improve with these injections.” ANS: C Anemia in children with CRF is related to decreased production of erythropoietin. Recombinant human erythropoietin (rHuEPO) is being offered to these children as thrice-weekly or weekly subcutaneous injections and is replacing the need for frequent blood transfusions. The parents understand the teaching if they say that the red blood cell count will begin to improve with these injections. PTS: 1 DIF: Cognitive Level: Analysis OBJ: Nursing Process: Implementation MSC: Client Needs: Physiologic Integrity 35. A school-age child with chronic renal failure is admitted to the hospital with a serum potassium level of 5.2 mEq/L. Which prescribed medication should the nurse plan to administer? a. Spironolactone b. Sodium polystyrene sulfonate c. Lactulose d. Calcium carbonate ANS: B Normal serum potassium levels in a school-age child are 3.5 to 5 mEq/L. Sodium polystyrene sulfonate is administered to reduce serum potassium levels. Spironolactone is a potassium-sparing diuretic and should not be used if the serum potassium is elevated. Lactulose is administered to reduce ammonia levels in patients with liver disease. Calcium carbonate may be prescribed as a calcium supplement, but it will not reduce serum potassium levels. PTS: 1 DIF: Cognitive Level: Application OBJ: Nursing Process: Implementation MSC: Client Needs: Physiologic Integrity MULTIPLE RESPONSE 1. What are the primary clinical manifestations of acute glomerulonephritis? (Select all that apply.) a. Oliguria b. Hematuria c. Proteinuria d. Hypertension e. Bacteriuria ANS: A, B, C, D The principal feature of acute glomerulonephritis include oliguria, edema, hypertension and circulatory congestion, hematuria, and proteinuria. Bacteriuria is not a principal feature of acute glomerulonephritis. PTS: 1 DIF: Cognitive Level: Application OBJ: Nursing Process: AssessmentNURSINMGSTCB.:CCOlMient Needs: Physiologic Integrity 2. The nurse is caring for an infant with a suspected urinary tract infection. Which clinical manifestations would be observed? (Select all that apply.) a. Vomiting b. Jaundice c. Failure to gain weight d. Swelling of the face e. Back pain f. Persistent diaper rash ANS: A, C, F Vomiting, failure to gain weight, and persistent diaper rash are clinical manifestations observed in an infant with a urinary tract infection. Jaundice, swelling of the face, and back pain would not be observed in an infant with a urinary tract infection. PTS: 1 DIF: Cognitive Level: Analysis OBJ: Nursing Process: Assessment MSC: Client Needs: Physiologic Integrity 3. A child with secondary enuresis who reports of dysuria or urgency should be evaluated for what condition? (Select all that apply.) a. Hypocalciuria b. Nephrotic syndrome c. Glomerulonephritis d. Urinary tract infection (UTI) e. Diabetes mellitus ANS: D, E Complaints of dysuria or urgency from a child with secondary enuresis suggest the possibility of a UTI. If accompanied by excessive thirst and weight loss, these symptoms may indicate the onset of diabetes mellitus. An excessive loss of calcium in the urine (hypercalciuria) can be associated with complaints of painful urination, urgency, frequency, and wetting. Nephrotic syndrome is not usually associated with complaints of dysuria or urgency. Glomerulonephritis is not a likely cause of dysuria or urgency. PTS: 1 DIF: Cognitive Level: Analysis OBJ: Nursing Process: Evaluation MSC: Client Needs: Physiologic Integrity 4. A school-age child is admitted to the hospital with acute glomerulonephritis and oliguria. Which dietary menu items should be allowed for this child? (Select all that apply.) a. Apples b. Bananas c. Cheese d. Carrot sticks e. Strawberries ANS: A, D, E Moderate sodium restriction and even fluid restriction may be instituted for children with acute glomerulonephritis. Foods with substantial amounts of potassium and sodium are generally restricted during the period of oliguria. Apples, carrot sticks, and strawberries would be items low in sodium and allowed. Bananas are high in potassium and cheese is high in sodium. Those items would be restricted. NURSINGTB.COM PTS: 1 DIF: Cognitive Level: Application OBJ: Nursing Process: Planning MSC: Client Needs: Physiologic Integrity 5. A school-age child has been admitted to the hospital diagnosed with minimal-change nephrotic syndrome. Which clinical manifestations should the nurse expect to assess? (Select all that apply.) a. Weight loss b. Generalized edema c. Proteinuria > 2+ d. Fatigue e. Irritability ANS: B, C, D, E The disease is suspected on the basis of clinical manifestations that include generalized edema, steadily gaining weight; appearing edematous; and then becoming anorexic, irritable, and less active. The hallmark of this syndrome is proteinuria (higher than 2+ on urine dipstick). PTS: 1 DIF: Cognitive Level: Application OBJ: Nursing Process: Assessment MSC: Client Needs: Physiologic Integrity Chapter 46: The Child With Cerebral Dysfunction Perry: Maternal Child Nursing Care, 6th Edition MULTIPLE CHOICE 1. Which term is used to describe a child's level of consciousness when the child can be aroused with stimulation? a. Stupor b. Confusion c. Obtundation d. Disorientation ANS: C Obtundation describes a level of consciousness in which the child can be aroused with stimulation. Stupor is a state in which the child remains in a deep sleep, responsive only to vigorous and repeated stimulation. Confusion is impaired decision making. Disorientation is confusion regarding time and place. PTS: 1 DIF: Cognitive Level: Comprehension OBJ: Nursing Process: Assessment MSC: Client Needs: Physiologic Integrity 2. Which term is used when a patient remains in a deep sleep, responsive only to vigorous and repeated stimulation? a. Coma b. Stupor c. Obtundation d. Persistent vegetative state ANS: B NURSINGTB.COM Stupor exists when the child remains in a deep sleep, responsive only to vigorous and repeated stimulation. Coma is the state in which no motor or verbal response occurs to noxious (painful) stimuli. Obtundation describes a level of consciousness in which the child can be aroused with stimulation. Persistent vegetative state describes the permanent loss of function of the cerebral cortex. PTS: 1 DIF: Cognitive Level: Comprehension OBJ: Nursing Process: Assessment MSC: Client Needs: Physiologic Integrity 3. The Glasgow Coma Scale consists of an assessment of what functions? a. Pupil reactivity and motor response. b. Eye opening and verbal and motor responses. c. Level of consciousness and verbal response. d. Intracranial pressure (ICP) and level of consciousness. ANS: B The Glasgow Coma Scale assesses eye opening and verbal and motor responses. Pupil reactivity is not a part of the Glasgow Coma Scale but is included in the pediatric coma scale. Level of consciousness and ICP are not part of the Glasgow Coma Scale. PTS: 1 DIF: Cognitive Level: Comprehension OBJ: Nursing Process: Assessment MSC: Client Needs: Physiologic Integrity 4. The nurse, closely monitoring a child who is unconscious after a fall, notices that the child suddenly has a fixed and dilated pupil. The nurse should interpret this as the indication of what occurrence? a. Eye trauma b. Neurosurgical emergency c. Severe brainstem damage d. Indication of brain death ANS: B The sudden appearance of a fixed and dilated pupil(s) is a neurosurgical emergency. The nurse should immediately report this finding. Although a dilated pupil may be associated with eye trauma, this child has experienced a neurologic insult. Pinpoint pupils or fixed, bilateral pupils for more than 5 minutes are indicative of brainstem damage. The unilateral fixed and dilated pupil is suggestive of damage on the same side of the brain. One fixed and dilated pupil is not suggestive of brain death. PTS: 1 DIF: Cognitive Level: Analysis OBJ: Nursing Process: Assessment MSC: Client Needs: Physiologic Integrity 5. Which test is never performed on a child who is awake? a. Oculovestibular response b. Doll's head maneuver c. Funduscopic examination for papilledema d. Assessment of pyramidal tract lesions ANS: A The oculovestibular response (calNorUicRSteINstG) TinBv.CoOlvMes the instillation of ice water into the ear of a comatose child. The caloric test is painful and is never performed on a child who is awake or one who has a ruptured tympanic membrane. Doll's head maneuver, funduscopic examination, and assessment of pyramidal tract lesions can be performed on children who are awake. PTS: 1 DIF: Cognitive Level: Comprehension OBJ: Nursing Process: Implementation MSC: Client Needs: Physiologic Integrity 6. The nurse is preparing a school-age child for a computed tomography (CT) scan to assess cerebral function. When preparing the child for the scan, which statement should the nurse include? a. “Pain medication will be given.” b. “The scan will not hurt.” c. “You will be able to move once the equipment is in place.” d. “Unfortunately no one can remain in the room with you during the test.” ANS: B For CT scans, the child will not be allowed to move and must be immobilized. It is important to emphasize to the child that at no time is the procedure painful. Pain medication is not required; however, sedation is sometimes necessary. Someone is able to remain with the child during the procedure. PTS: 1 DIF: Cognitive Level: Application OBJ: Nursing Process: Implementation MSC: Client Needs: Health Promotion and Maintenance 7. Which neurologic diagnostic test gives a visualized horizontal and vertical cross section of the brain at any axis? a. Nuclear brain scan b. Echoencephalography c. Computed tomography (CT) scan d. Magnetic resonance imaging (MRI) ANS: C A CT scan provides visualization of the horizontal and vertical cross sections of the brain at any axis. A nuclear brain scan uses a radioisotope that accumulates where the blood-brain barrier is defective. Echoencephalography identifies shifts in midline structures of the brain as a result of intracranial lesions. MRI permits visualization of morphologic features of target structures and tissue discrimination that is unavailable with any other techniques. PTS: 1 DIF: Cognitive Level: Comprehension OBJ: Nursing Process: Assessment MSC: Client Needs: Physiologic Integrity 8. What is the priority nursing intervention when a child is unconscious after a fall? a. Establish an adequate airway b. Perform neurologic assessment c. Monitor intercranial pressure d. Determine whether a neck injury is present ANS: A Respiratory effectiveness is the primary concern in the care of the unconscious child. Establishing an adequate airway is always the first priority. A neurologic assessment and determination of neck injury are pNeUrfRoSrmINeGdTaBf.tCeOr Mbreathing and circulation are stabilized. Intracranial, not intercranial, pressure is monitored if indicated after airway, breathing, and circulation are maintained. PTS: 1 DIF: Cognitive Level: Comprehension OBJ: Nursing Process: Implementation MSC: Client Needs: Physiologic Integrity 9. Which drug would be used to treat a child who has increased intracranial pressure (ICP) resulting from cerebral edema? a. Mannitol b. Epinephrine hydrochloride c. Atropine sulfate d. Sodium bicarbonate ANS: A For increased ICP, mannitol, an osmotic diuretic, administered intravenously, is the drug used most frequently for rapid reduction. Epinephrine, atropine sulfate, and sodium bicarbonate are not used to decrease ICP. PTS: 1 DIF: Cognitive Level: Comprehension OBJ: Nursing Process: Implementation MSC: Client Needs: Physiologic Integrity 10. Which statement is most descriptive of a concussion? a. Petechial hemorrhages cause amnesia. b. Visible bruising and tearing of cerebral tissue occur. c. It is a transient, reversible neuronal dysfunction. d. A slight lesion develops remote from the site of trauma. ANS: C A concussion is a transient, reversible neuronal dysfunction with instantaneous loss of awareness and responsiveness resulting from trauma to the head. Petechial hemorrhages along the superficial aspects of the brain along the point of impact are a type of contusion but are not necessarily associated with amnesia. A contusion is visible bruising and tearing of cerebral tissue. Contrecoup is a lesion that develops remote from the site of trauma as a result of an acceleration/deceleration injury. PTS: 1 DIF: Cognitive Level: Comprehension OBJ: Nursing Process: Assessment MSC: Client Needs: Physiologic Integrity 11. Which type of fracture describes traumatic separation of cranial sutures? a. Basilar b. Compound c. Diastatic d. Depressed ANS: C Diastatic skull fractures are traumatic separations of the cranial sutures. A basilar fracture involves the basilar portion of the frontal, ethmoid, sphenoid, temporal, or occipital bone. A compound fracture has the bone exposed through the skin. A depressed fracture has the bone pushed inward, causing pressure on the brain. PTS: 1 DIF: Cognitive Level: Comprehension OBJ: Nursing Process: AssessmentNURSINMGSTCB.:CCOlMient Needs: Physiologic Integrity 12. Which statement best describes a subdural hematoma? a. Bleeding occurs between the dura and the skull. b. Bleeding occurs between the dura and the cerebrum. c. Bleeding is generally arterial, and brain compression occurs rapidly. d. The hematoma commonly occurs in the parietotemporal region. ANS: B A subdural hematoma is bleeding that occurs between the dura and the cerebrum as a result of a rupture of cortical veins that bridge the subdural space. An epidural hemorrhage occurs between the dura and the skull, is usually arterial with rapid brain concussion, and occurs most often in the parietotemporal region. PTS: 1 DIF: Cognitive Level: Comprehension OBJ: Nursing Process: Assessment MSC: Client Needs: Physiologic Integrity 13. An adolescent boy is brought to the emergency department after a motorcycle accident. His respirations are deep, periodic, and gasping. There are extreme fluctuations in blood pressure. Pupils are dilated and fixed. What type of head injury should the nurse suspect? a. Brainstem b. Skull fracture c. Subdural hemorrhage d. Epidural hemorrhage ANS: A Signs of brainstem injury include deep, rapid, periodic or intermittent, and gasping respirations. Wide fluctuations or noticeable slowing of the pulse, widening pulse pressure, or extreme fluctuations in blood pressure are consistent with a brainstem injury. Skull fracture and subdural and epidural hemorrhages are not consistent with these signs. PTS: 1 DIF: Cognitive Level: Comprehension OBJ: Nursing Process: Assessment MSC: Client Needs: Physiologic Integrity 14. A toddler, who fell out of a second-story window, had brief loss of consciousness and vomited 4 times. Since admission, the child has been alert and oriented. The mother asks why a computed tomography (CT) scan is required when the child “seems fine.” The nurse should base the response on the need to monitor for what possible complication? a. A brain injury b. Coma c. Seizures d. Skull fracture ANS: A The child's history of the fall, brief loss of consciousness, and vomiting 4 times necessitate evaluation of a potential brain injury. The severity of a head injury may not be apparent on clinical examination but will be detectable on a CT scan. All the remaining options are a result of varying degrees of brain injury. PTS: 1 DIF: Cognitive Level: Analysis OBJ: Nursing Process: Implementation MSC: Client Needs: Physiologic Integrity 15. The nurse is assessing a child whoNwURasSIjNuGstTaBd.CmOitMted to the hospital for observation after a head injury. What is the most essential part of the nursing assessment to detect early signs of a worsening condition? a. Posturing b. Vital signs c. Focal neurologic signs d. Level of consciousness ANS: D The most important nursing observation is assessment of the child's level of consciousness. Alterations in consciousness appear earlier in the progression of head injury than do alterations of vital signs or focal neurologic signs. Neurologic posturing indicates neurologic damage. Vital signs and focal neurologic signs are later signs of progression when compared with level-of-consciousness changes. PTS: 1 DIF: Cognitive Level: Analysis OBJ: Nursing Process: Assessment MSC: Client Needs: Physiologic Integrity 16. A school-age child has sustained a head injury and multiple fractures after being thrown from a horse. The child's level of consciousness is variable. The parents tell the nurse that they think their child is in pain because of periodic crying and restlessness. What is the most appropriate nursing action in response to the parent's concern? a. Discuss with parents the child's previous experiences with pain. b. Discuss with practitioner what analgesia can be safely administered. c. Explain that analgesia is contraindicated with a head injury. d. Explain that analgesia is unnecessary when child is not fully awake and alert. ANS: B A key nursing role is to provide sedation and analgesia for the child. Consultation with the appropriate practitioner is necessary to avoid conflict between the necessity to monitor the child's neurologic status and to promote comfort and relieve anxiety. Gathering information about the child's previous experiences with pain should be obtained as part of the assessment, but because of the severity of injury, analgesia should be provided as soon as possible. Analgesia can be used safely in individuals who have sustained head injuries and can decrease anxiety and resultant increased intracranial pressure. PTS: 1 DIF: Cognitive Level: Application OBJ: Nursing Process: Implementation MSC: Client Needs: Physiologic Integrity 17. A 5 year old sustained a concussion when falling out of a tree. In preparation for discharge, the nurse is discussing home care with the mother. Which statement made by the mother indicates a correct understanding of the teaching? a. “I should expect my child to have a few episodes of vomiting.” b. “If I notice sleep disturbances, I should contact the physician immediately.” c. “I should expect my child to have some difficulty concentrating for a while.” d. “If I notice diplopia, I will have my child rest for 1 hour.” ANS: C The parents are advised of probably posttraumatic symptoms that may be expected, including difficulty concentrating, and memory impairment. If the child has episodes of vomiting, sleep disturbances, or diplopia, they should be immediately reported for evaluation. NURSINGTB.COM PTS: 1 DIF: Cognitive Level: Analysis OBJ: Nursing Process: Implementation MSC: Client Needs: Physiologic Integrity 18. A 3-year-old child is hospitalized after a near-drowning accident. The child's mother complains to the nurse, “This seems unnecessary when he is perfectly fine.” What is the nurse's best reply? a. “He still needs a little extra oxygen.” b. “I'm sure he is fine, but the doctor wants to make sure.” c. “The reason for this is that complications could still occur.” d. “It is important to observe for possible central nervous system problems.” ANS: C All children who have a near-drowning experience should be admitted to the hospital for observation. Although many children do not appear to have suffered adverse effects from the event, complications such as respiratory compromise and cerebral edema may occur up to 24 hours after the incident. Aspiration pneumonia is a frequent complication that occurs about 48 to 72 hours after the episode. Stating that, “He still needs a little extra oxygen” does not respond directly to the mother's concern. Why is her child still receiving oxygen? The nurse should clarify that different complications can occur up to 24 hours later and that observations are necessary. The nurse should not provide statements that provide unfounded information, like “I'm sure he is fine.” PTS: 1 DIF: Cognitive Level: Application OBJ: Nursing Process: Implementation MSC: Client Needs: Physiologic Integrity 19. The mother of a 1-month-old infant tells the nurse that she worries that her baby will get meningitis like her oldest son did when he was an infant. On what information should the nurse's response be based upon? a. Meningitis rarely occurs during infancy. b. Often a genetic predisposition to meningitis is found. c. Vaccination to prevent all types of meningitis is now available. d. Vaccination to prevent Haemophilus influenzae type B meningitis has decreased the frequency of this disease in children. ANS: D H. influenzae type B meningitis has virtually been eradicated in areas of the world where the vaccine is administered routinely. Bacterial meningitis remains a serious illness in children. It is significant because of the residual damage caused by undiagnosed and untreated or inadequately treated cases. The leading causes of neonatal meningitis are the group B streptococci and Escherichia coli organisms. Meningitis is an extension of a variety of bacterial infections. No genetic predisposition exists. Vaccinations are not available for all of the potential causative organisms. PTS: 1 DIF: Cognitive Level: Comprehension OBJ: Nursing Process: Implementation MSC: Client Needs: Physiologic Integrity 20. What is the common vector reservoir for agents causing viral encephalitis in the United States? a. Tarantula spiders b. Mosquitoes c. Carnivorous wild animals d. Domestic and wild animals ANS: B NURSINGTB.COM Viral encephalitis, not attributable to a childhood viral disease, is usually transmitted by mosquitoes and ticks. The vector reservoir for most agents pathogenic for humans and detected in the United States are mosquitoes and ticks; therefore, most cases of encephalitis appear during the hot summer months. Tarantulas, carnivorous wild animals, and domestic animals are not reservoirs for the agents that cause viral encephalitis. PTS: 1 DIF: Cognitive Level: Comprehension OBJ: Nursing Process: Planning MSC: Client Needs: Physiologic Integrity 21. What action may be beneficial in reducing the risk of Reye's syndrome? a. Immunization against the disease b. Medical attention for all head injuries c. Prompt treatment of bacterial meningitis d. Avoidance of aspirin and ibuprofen for children with varicella or those suspected of having influenza ANS: D Although the etiology of Reye's syndrome is obscure, most cases follow a common viral illness, either varicella or influenza. A potential association exists between aspirin therapy and the development of Reye's syndrome; thus use of aspirin is avoided. No immunization currently exists for Reye's syndrome. Reye's syndrome is not correlated with head injuries or bacterial meningitis. PTS: 1 DIF: Cognitive Level: Comprehension OBJ: Nursing Process: Planning MSC: Client Needs: Physiologic Integrity 22. When taking the history of a child hospitalized with Reye's syndrome, the nurse should not be surprised that a week ago the child had recovered from infectious illness? a. Measles b. Varicella c. Meningitis d. Hepatitis ANS: B Most cases of Reye's syndrome follow a common viral illness such as varicella or influenza. Measles, meningitis, and hepatitis are not associated with Reye's syndrome. PTS: 1 DIF: Cognitive Level: Comprehension OBJ: Nursing Process: Planning MSC: Client Needs: Physiologic Integrity 23. When caring for the child diagnosed with Reye's syndrome, what is the priority nursing intervention? a. Monitor intake and output b. Prevent skin breakdown c. Observe for petechiae NURSINGTB.COM d. Do range-of-motion (ROM) exercises ANS: A Accurate and frequent monitoring of intake and output is essential for adjusting fluid volumes to prevent both dehydration and cerebral edema. Preventing skin breakdown, observing for petechiae, and doing ROM exercises are important interventions in the care of a critically ill or comatose child. Careful monitoring of intake and output is a priority. PTS: 1 DIF: Cognitive Level: Application OBJ: Nursing Process: Implementation MSC: Client Needs: Physiologic Integrity 24. A young child's parents call the nurse after their child was bitten by a raccoon in the woods. The nurse's recommendation should be based on knowing that: a. the child should be hospitalized for close observation. b. no treatment is necessary if thorough wound cleaning is done. c. antirabies prophylaxis must be initiated. d. antirabies prophylaxis must be initiated if clinical manifestations appear. ANS: C Current therapy for a rabid animal bite consists of a thorough cleansing of the wound and passive immunization with human rabies immune globulin (HRIG) as soon as possible. Hospitalization is not necessary. The wound cleansing, passive immunization, and immune globulin administration can be done as an outpatient. The child needs to receive both HRIG and rabies vaccine. PTS: 1 DIF: Cognitive Level: Comprehension OBJ: Nursing Process: Planning MSC: Client Needs: Physiologic Integrity 25. Which type of seizure involves both hemispheres of the brain? a. Focal b. Partial c. Generalized d. Acquired ANS: C Clinical observations of generalized seizures indicate that the initial involvement is from both hemispheres. Focal seizures may arise from any area of the cerebral cortex, but the frontal, temporal, and parietal lobes are most commonly affected. Partial seizures are caused by abnormal electrical discharges from epileptogenic foci limited to a circumscribed region of the cerebral cortex. A seizure disorder that is acquired is a result of a brain injury from a variety of factors; it does not specify the type of seizure. PTS: 1 DIF: Cognitive Level: Comprehension OBJ: Nursing Process: Assessment MSC: Client Needs: Physiologic Integrity 26. What is the initial clinical manifestation of generalized seizures? a. Being confused b. Feeling frightened c. Losing consciousness d. Seeing flashing lights ANS: C NURSINGTB.COM Loss of consciousness is a frequent occurrence in generalized seizures and is the initial clinical manifestation. Being confused, feeling frightened, and seeing flashing lights are clinical manifestations of a complex partial seizure. PTS: 1 DIF: Cognitive Level: Comprehension OBJ: Nursing Process: Assessment MSC: Client Needs: Physiologic Integrity 27. Which type of seizure may be difficult to detect? a. Absence b. Generalized c. Simple partial d. Complex partial ANS: A Absence seizures may go unrecognized because little change occurs in the child's behavior during the seizure except for a period of unconsciousness lasting less than 10 seconds. Generalized, simple partial, and complex partial seizures all have clinical manifestations that are observable. PTS: 1 DIF: Cognitive Level: Comprehension OBJ: Nursing Process: Assessment MSC: Client Needs: Physiologic Integrity 28. Which nursing intervention is appropriate when caring for a child who has experienced a seizure? a. Describe and record the seizure activity observed. b. Restrain the child when seizure occurs to prevent bodily harm. c. Place a tongue blade between the teeth if they become clenched. d. Suction the child during a seizure to prevent aspiration. ANS: A When a child is having a seizure, the priority nursing care is observation of the child and seizure. The nurse then describes and records the seizure activity. The child should not be restrained, and nothing should be placed in his or her mouth. This may cause injury. To prevent aspiration, if possible, the child should be placed on his or her side, facilitating drainage. PTS: 1 DIF: Cognitive Level: Application OBJ: Nursing Process: Implementation MSC: Client Needs: Physiologic Integrity 29. Which clinical manifestations would suggest hydrocephalus in a neonate? a. Bulging fontanel and dilated scalp veins b. Closed fontanel and high-pitched cry c. Constant low-pitched cry and restlessness d. Depressed fontanel and decreased blood pressure ANS: A Bulging fontanel, dilated scalp veins, and separated sutures are clinical manifestations of hydrocephalus in neonates. Closed fontanel and high-pitched cry, constant low-pitched cry and restlessness, and depressed fontanel and decreased blood pressure are not clinical manifestations of hydrocephalus, but all should be referred for evaluation. PTS: 1 DIF: CognitiNveURLSevINelG: TCBo.mCOprMehension OBJ: Nursing Process: Assessment MSC: Client Needs: Physiologic Integrity 30. Which information should the nurse give to a child who is to have magnetic resonance imaging (MRI) of the brain? a. “Your head will be kept from moving during the procedure.” b. “You will have to drink a special fluid before the test.” c. “You will have to lie flat after the test is finished.” d. “You will have electrodes placed on your head with glue.” ANS: A To reduce fear and enhance cooperation during the MRI, the child should be made aware that the head will be restricted to obtain accurate information. Drinking fluids is usually done for neurologic procedures. A child should lie flat after a lumbar puncture, not after an MRI. Electrodes are attached to the head for an electroencephalogram. PTS: 1 DIF: Cognitive Level: Application OBJ: Nursing Process: Planning MSC: Client Needs: Physiologic Integrity 31. The nurse has received report on four children. Which child should the nurse assess first? a. A school-age child in a coma with stable vital signs. b. A preschool child with a head injury and decreasing level of consciousness. c. An adolescent admitted after a motor vehicle accident who is oriented to person and place. d. A toddler in a persistent vegetative state with a low-grade fever. ANS: B The nurse should assess the child with a head injury and decreasing level of consciousness (LOC) first. Assessment of LOC remains the earliest indicator of improvement or deterioration in neurologic status. The next child the nurse should assess is a toddler in a persistent vegetative state with a low-grade fever. The school-age child in a coma with stable vital signs and the adolescent admitted to the hospital who is oriented to his or her surroundings would be of least worry to the nurse. PTS: 1 DIF: Cognitive Level: Analysis OBJ: Nursing Process: Implementation MSC: Client Needs: Safe and Effective Care Environment 32. The nurse is performing a Glasgow Coma Scale (GCS) on a school-age child with a head injury. The child opens eyes spontaneously, obeys commands, and is oriented to person, time, and place. Which is the score the nurse should record? a. 8 b. 11 c. 13 d. 15 ANS: D The GCS consists of a three-part assessment: eye opening, verbal response, and motor response. Numeric values of 1 through 5 are assigned to the levels of response in each category. The sum of these numeric values provides an objective measure of the patient's level of consciousness (LOC). A person with an unaltered LOC would score the highest, 15. The child who opens eyes spontaneously, obeys commands, and is oriented is scored at a 15. NURSINGTB.COM PTS: 1 DIF: Cognitive Level: Comprehension OBJ: Nursing Process: Assessment MSC: Client Needs: Physiologic Integrity 33. The nurse is caring for a child with severe head trauma after a car accident. Which is an ominous sign that often precedes death? a. Papilledema b. Delirium c. Doll's head maneuver d. Periodic and irregular breathing ANS: D Periodic or irregular breathing is an ominous sign of brainstem (especially medullary) dysfunction that often precedes complete apnea. Papilledema is edema and inflammation of the optic nerve. It is commonly a sign of increased intracranial pressure. Delirium is a state of mental confusion and excitement marked by disorientation to time and place. The doll's head maneuver is a test for brainstem or oculomotor nerve dysfunction. PTS: 1 DIF: Cognitive Level: Comprehension OBJ: Nursing Process: Assessment MSC: Client Needs: Physiologic Integrity 34. It is appropriate, when caring for an unconscious child, to implement which intervention? a. Change the child's position infrequently to minimize the chance of increased intracranial pressure (ICP). b. Avoid using narcotics or sedatives to provide comfort and pain relief. c. Monitor fluid intake and output carefully to avoid fluid overload and cerebral edema. d. Give tepid sponge baths to reduce fever because antipyretics are contraindicated. ANS: C Often comatose patients cannot cope with the quantity of fluids that they normally tolerate. Overhydration must be avoided to prevent fatal cerebral edema. The child's position should be changed frequently to avoid complications such as pneumonia and skin breakdown. Narcotics and sedatives should be used as necessary to reduce pain and discomfort, which can increase ICP. Antipyretics are the method of choice for fever reduction. PTS: 1 DIF: Cognitive Level: Application OBJ: Nursing Process: Implementation MSC: Client Needs: Physiologic Integrity 35. When a 10 year old has been hit by a car while riding his bicycle in front of the school, the school nurse immediately assesses airway, breathing, and circulation. What should be the next nursing action? a. Place on side b. Take blood pressure c. Stabilize neck and spine d. Check scalp and back for bleeding ANS: C After determining that the child is breathing and has adequate circulation, the next action is to stabilize the neck and spine to prevent any additional trauma. The child's position should not be changed until the neck and spine are stabilized. Blood pressure is a later assessment. Less urgent, but an important assessment, is inspection of the scalp for bleeding. NURSINGTB.COM PTS: 1 DIF: Cognitive Level: Application OBJ: Nursing Process: Implementation MSC: Client Needs: Physiologic Integrity: Physiologic Adaptation 36. A child is unconscious after a motor vehicle accident. The watery discharge from the nose tests positive for glucose. The nurse should recognize that this suggests what complication? a. Diabetic coma b. Brainstem injury c. Upper respiratory tract infection d. Leaking of cerebrospinal fluid (CSF) ANS: D Watery discharge from the nose that is positive for glucose suggests leaking of CSF from a skull fracture and is not associated with diabetes or respiratory tract infection. The fluid is probably CSF from a skull fracture and does not signify whether the brainstem is involved. PTS: 1 DIF: Cognitive Level: Analysis OBJ: Nursing Process: Assessment MSC: Client Needs: Physiologic Integrity 37. A child has been seizure-free for 2 years. A father asks the nurse how much longer the child will need to take the antiseizure medications. The nurse includes which intervention in the response? a. Medications can be discontinued at this time. b. The child will need to take the drugs for 5 years after the last seizure. c. A stepwise approach will be used to reduce the dosage gradually. d. Seizure disorders are a lifelong problem. Medications cannot be discontinued. ANS: C A predesigned protocol is used to wean a child gradually off antiseizure medications, usually when the child is seizure-free for 2 years and has a normal electroencephalogram. Medications must be gradually reduced to minimize the recurrence of seizures. Seizure medications can be safely discontinued. The risk of recurrence is greatest within the first year. PTS: 1 DIF: Cognitive Level: Application OBJ: Nursing Process: Implementation MSC: Client Needs: Physiologic Integrity MULTIPLE RESPONSE 1. Clinical manifestations of increased intracranial pressure (ICP) in infants are: (Select all that apply.) a. Low-pitched cry b. Sunken fontanel c. Drowsiness d. Irritability e. Distended scalp veins f. Increased blood pressure ANS: C, D, E Drowsiness, irritability, and distended scalp veins are signs of increased ICP in infants. Low-pitched cry, sunken fontanel, and increased blood pressure are not clinical manifestations associated with ICP in infants. NURSINGTB.COM PTS: 1 DIF: Cognitive Level: Analysis OBJ: Nursing Process: Assessment MSC: Client Needs: Physiologic Integrity 2. An infant diagnosed with hydrocephalus is hospitalized for surgical placement of a ventriculoperitoneal shunt. Which interventions should be included in the child's postoperative care? (Select all that apply.) a. Observe closely for signs of infection b. Pump the shunt reservoir to maintain patency c. Administer sedation to decrease irritability d. Maintain Trendelenburg position to decrease pressure on the shunt e. Maintain an accurate record of intake and output f. Monitor for abdominal distention ANS: A, E, F Infection is a major complication of ventriculoperitoneal shunts. Observation for signs of infection is a priority nursing intervention. Intake and output should be measured carefully. Abdominal distention could be a sign of peritonitis or a postoperative ileus. Pumping the shunt reservoir, administering sedation, and maintaining Trendelenburg position are not interventions associated with this condition. PTS: 1 DIF: Cognitive Level: Application OBJ: Nursing Process: Assessment MSC: Client Needs: Physiologic Integrity 3. A nurse should expect which cerebrospinal fluid (CSF) laboratory results on a child diagnosed with bacterial meningitis? (Select all that apply.) a. Elevated white blood cell (WBC) count b. Decreased protein c. Decreased glucose d. Cloudy in color e. Increase in red blood cells (RBCs) ANS: A, C, D The CSF laboratory results for bacterial meningitis include elevated WBC counts, cloudy or milky in color, and decreased glucose. The protein is elevated and there should be no RBCs present. RBCs are present when the tap was traumatic. PTS: 1 DIF: Cognitive Level: Comprehension OBJ: Nursing Process: Assessment MSC: Client Needs: Physiologic Integrity 4. The nurse is caring for a neonate with suspected meningitis. Which clinical manifestations should the nurse prepare to assess if meningitis is confirmed? (Select all that apply.) a. Headache b. Photophobia c. Bulging anterior fontanel d. Weak cry e. Poor muscle tone ANS: C, D, E Assessment findings in a neonate with meningitis include bulging anterior fontanel, weak cry, and poor muscle tone. Headache and photophobia are signs seen in an older child. NURSINGTB.COM PTS: 1 DIF: Cognitive Level: Analysis OBJ: Nursing Process: Assessment MSC: Client Needs: Physiologic Integrity 5. The nurse is monitoring an infant for signs of increased intracranial pressure (ICP). Which are late signs of increased ICP in an infant? (Select all that apply.) a. Tachycardia b. Alteration in pupil size and reactivity c. Increased motor response d. Extension or flexion posturing e. Cheyne-Stokes respirations ANS: B, D, E Late signs of ICP in an infant or child include bradycardia, alteration in pupil size and reactivity, decreased motor response, extension or flexion posturing, and Cheyne-Stokes respirations. PTS: 1 DIF: Cognitive Level: Analysis OBJ: Nursing Process: Assessment MSC: Client Needs: Physiologic Integrity MATCHING A 6-year-old child is having a generalized seizure in the classroom at school. Place in order the interventions the school nurse should implement, starting with the highest-priority intervention and sequencing to the lowest-priority intervention. a. Take vital signs b. Ease child to the floor c. Allow child to rest d. Turn child to the side e. Integrate child back into the school environment 1. First priority 2. Second priority 3. Third priority 4. Fourth priority 5. Fifth priority 1. ANS: B PTS: 1 DIF: Cognitive Level: Application OBJ: Nursing Process: Implementation MSC: Client Needs: Physiologic Integrity NOT: The nurse should ease the child to the floor immediately during a generalized seizure. During (and sometimes after) the generalized seizure, the swallowing reflex is lost, salivation increases, and the tongue is hypotonic. Therefore, the child is at risk for aspiration and airway occlusion. Placing the child on the side facilitates drainage and helps maintain a patent airway. Vital signs should be taken next and the child should be allowed to rest. When feasible, the child is integrated into the environment as soon as possible. 2. ANS: D PTS: 1 DIF: Cognitive Level: Application OBJ: Nursing Process: Implementation MSC: Client Needs: Physiologic Integrity NOT: The nurse should ease the child to the floor immediately during a generalized seizure. During (and sometimes after) the generalized seizure, the swallowing reflex is lost, salivation increases, and the tongue is hypotonic. Therefore, thNeUcRhSilIdNiGsTaBt .rCisOkMfor aspiration and airway occlusion. Placing the child on the side facilitates drainage and helps maintain a patent airway. Vital signs should be taken next and the child should be allowed to rest. When feasible, the child is integrated into the environment as soon as possible. 3. ANS: A PTS: 1 DIF: Cognitive Level: Application OBJ: Nursing Process: Implementation MSC: Client Needs: Physiologic Integrity NOT: The nurse should ease the child to the floor immediately during a generalized seizure. During (and sometimes after) the generalized seizure, the swallowing reflex is lost, salivation increases, and the tongue is hypotonic. Therefore, the child is at risk for aspiration and airway occlusion. Placing the child on the side facilitates drainage and helps maintain a patent airway. Vital signs should be taken next and the child should be allowed to rest. When feasible, the child is integrated into the environment as soon as possible. 4. ANS: C PTS: 1 DIF: Cognitive Level: Application OBJ: Nursing Process: Implementation MSC: Client Needs: Physiologic Integrity NOT: The nurse should ease the child to the floor immediately during a generalized seizure. During (and sometimes after) the generalized seizure, the swallowing reflex is lost, salivation increases, and the tongue is hypotonic. Therefore, the child is at risk for aspiration and airway occlusion. Placing the child on the side facilitates drainage and helps maintain a patent airway. Vital signs should be taken next and the child should be allowed to rest. When feasible, the child is integrated into the environment as soon as possible. 5. ANS: E PTS: 1 DIF: Cognitive Level: Application OBJ: Nursing Process: Implementation MSC: Client Needs: Physiologic Integrity NOT: The nurse should ease the child to the floor immediately during a generalized seizure. During (and sometimes after) the generalized seizure, the swallowing reflex is lost, salivation increases, and the tongue is hypotonic. Therefore, the child is at risk for aspiration and airway occlusion. Placing the child on the side facilitates drainage and helps maintain a patent airway. Vital signs should be taken next and the child should be allowed to rest. When feasible, the child is integrated into the environment as soon as possible. NURSINGTB.COM Chapter 47: The Child With Endocrine Dysfunction Perry: Maternal Child Nursing Care, 6th Edition MULTIPLE CHOICE 1. Which statement best describes idiopathic hypopituitarism? a. Growth is normal during the first 3 years of life. b. Weight is usually more retarded than height. c. Skeletal proportions are normal for age. d. Most of these children have subnormal intelligence. ANS: C In children with idiopathic hypopituitarism, the skeletal proportions are normal. Growth is within normal limits for the first year of life. Height is usually more delayed than weight. Intelligence is not affected by hypopituitarism. PTS: 1 DIF: Cognitive Level: Comprehension OBJ: Nursing Process: Assessment MSC: Client Needs: Physiologic Integrity 2. A child with growth hormone (GH) deficiency is receiving GH therapy. What is the best time for the GH to be administered? a. At bedtime b. After meals c. Before meals d. On arising in the morning ANS: A NURSINGTB.COM Injections are best given at bedtime to more closely approximate the physiologic release of GH. Before or after meals and on arising in the morning are times that do not mimic the physiologic release of the hormone. PTS: 1 DIF: Cognitive Level: Application OBJ: Nursing Process: Implementation MSC: Client Needs: Physiologic Integrity 3. What is the priority nursing goal for a 14 year old diagnosed with Graves’ disease? a. Relieving constipation b. Allowing the adolescent to make decisions about whether or not to take medication c. Verbalizing the importance of monitoring for medication side effects d. Developing alternative educational goals ANS: C Children being treated with propylthiouracil or methimazole must be carefully monitored for side effects of the drug. Because sore throat and fever accompany the grave complication of leukopenia, these children should be seen by a health care practitioner if such symptoms occur. Parents and children should be taught to recognize and report symptoms immediately. The adolescent with Graves’ disease is not likely to be constipated. Adherence to the medication schedule is important to ensure optimal health and wellness. Medications should not be skipped and dose regimens should not be tapered by the child without consultation with the child’s medical provider. The management of Graves’ disease does not interfere with school attendance and does not require alternative educational plans. PTS: 1 DIF: Cognitive Level: Analysis OBJ: Nursing Process: Planning MSC: Client Needs: Health Promotion and Maintenance 4. At what age is sexual development in boys and girls considered to be precocious? a. Boys, 11 years; girls, 9 years b. Boys, 12 years; girls, 10 years c. Boys, 9 years; girls, 8 years d. Boys, 10 years; girls, 9.5 years ANS: C Manifestations of sexual development before age 9 in boys and age 8 in girls are considered precocious and should be investigated. Boys older than 9 years of age and girls older than 8 years of age fall within the expected range of pubertal onset. PTS: 1 DIF: Cognitive Level: Comprehension OBJ: Nursing Process: Diagnosis MSC: Client Needs: Physiologic Integrity 5. A child will start treatment for precocious puberty. This involves injections of which synthetic medication? a. Thyrotropin b. Gonadotropins c. Somatotropic hormone d. Luteinizing hormone–releasing hormone ANS: D Precocious puberty of central origin is treated with monthly subcutaneous injections of luteinizing hormone–releasing horNmUoRnSeIN. TGhTyBr.CotOroMpin, gonadotropin, and somatotropic hormone are not appropriate therapies for precocious puberty. PTS: 1 DIF: Cognitive Level: Comprehension OBJ: Nursing Process: Implementation MSC: Client Needs: Physiologic Integrity 6. Diabetes insipidus is a disorder of which organ? a. Anterior pituitary b. Posterior pituitary c. Adrenal cortex d. Adrenal medulla ANS: B The principal disorder of posterior pituitary hypofunction is diabetes insipidus. The anterior pituitary produces hormones such as growth hormone, thyroid-stimulating hormone, adrenocorticotropic hormone, gonadotropin, prolactin, and melanocyte-stimulating hormone. The adrenal cortex produces aldosterone, sex hormones, and glucocorticoids. The adrenal medulla produces catecholamines. PTS: 1 DIF: Cognitive Level: Comprehension OBJ: Nursing Process: Assessment MSC: Client Needs: Physiologic Integrity 7. The nurse is caring for a child with suspected diabetes insipidus. Which clinical manifestation would be observable? a. Oliguria b. Glycosuria c. Nausea and vomiting d. Polydipsia ANS: D Excessive urination accompanied by insatiable thirst is the primary clinical manifestation of diabetes. These symptoms may be so severe that the child does little other than drink and urinate. Oliguria is decreased urine production and is not associated with diabetes insipidus. Glycosuria is associated with diabetes mellitus. Nausea and vomiting are associated with inappropriate antidiuretic hormone secretion. PTS: 1 DIF: Cognitive Level: Application OBJ: Nursing Process: Assessment MSC: Client Needs: Physiologic Integrity 8. What is a common clinical manifestation of juvenile hypothyroidism? a. Insomnia b. Diarrhea c. Dry skin d. Accelerated growth ANS: C Dry skin, mental decline, and myxedematous skin changes are associated with juvenile hypothyroidism. Children with hypothyroidism are usually sleepy. Constipation is associated with hypothyroidism. Decelerated growth is common in juvenile hypothyroidism. PTS: 1 DIF: Cognitive Level: Comprehension OBJ: Nursing Process: Assessment MSC: Client Needs: Physiologic Integrity NURSINGTB.COM 9. A goiter is an enlargement or hypertrophy of which gland? a. Thyroid b. Adrenal c. Anterior pituitary d. Posterior pituitary ANS: A A goiter is an enlargement or hypertrophy of the thyroid gland. Goiter is not associated with the adrenals or the anterior and posterior pituitaries. PTS: 1 DIF: Cognitive Level: Comprehension OBJ: Nursing Process: Diagnosis MSC: Client Needs: Physiologic Integrity 10. Exophthalmos may occur in children with what diagnosis? a. Hypothyroidism b. Hyperthyroidism c. Hypoparathyroidism d. Hyperparathyroidism ANS: B Exophthalmos (protruding eyeballs) is a clinical manifestation of hyperthyroidism. Hypothyroidism, hypoparathyroidism, and hyperparathyroidism are not associated with exophthalmos. PTS: 1 DIF: Cognitive Level: Comprehension OBJ: Nursing Process: Assessment MSC: Client Needs: Physiologic Integrity 11. The nurse is teaching the parents of a child who is receiving propylthiouracil for the treatment of hyperthyroidism (Graves’ disease). Which statement made by the parent indicates a correct understanding of the teaching? a. “I would expect my child to gain weight while taking this medication.” b. “I would expect my child to experience episodes of ear pain while taking this medication.” c. “If my child develops a sore throat and fever, I should contact the physician immediately.” d. “If my child develops the stomach flu, my child will need to be hospitalized.” ANS: C Children being treated with propylthiouracil must be carefully monitored for the side effects of the drug. Parents must be alerted that sore throat and fever accompany the grave complication of leukopenia. These symptoms should be immediately reported. Weight gain, episodes of ear pain, and stomach flu are not usually associated with leukopenia. PTS: 1 DIF: Cognitive Level: Application OBJ: Nursing Process: Implementation MSC: Client Needs: Physiologic Integrity 12. A child diagnosed with hypoparathyroidism is receiving vitamin D therapy. The parents should be advised to watch for which sign of vitamin D toxicity? a. Headache and seizures b. Physical restlessness and voracious appetite without weight gain c. Weakness and lassitude d. Anorexia and insomnia ANS: C NURSINGTB.COM Vitamin D toxicity can be a serious consequence of therapy. Parents are advised to watch for signs including weakness, fatigue, lassitude, headache, nausea, vomiting, and diarrhea. Renal impairment is manifested through polyuria, polydipsia, and nocturia. Headaches may be a sign of vitamin D toxicity, but seizures are not. Physical restlessness and a voracious appetite with weight loss are manifestations of hyperthyroidism. Anorexia and insomnia are not characteristic of vitamin D toxicity. PTS: 1 DIF: Cognitive Level: Application OBJ: Nursing Process: Implementation MSC: Client Needs: Physiologic Integrity 13. Glucocorticoids, mineralocorticoids, and sex steroids are secreted by which organ? a. Thyroid gland b. Parathyroid glands c. Adrenal cortex d. Anterior pituitary ANS: C These hormones are secreted by the adrenal cortex. The thyroid gland produces thyroid hormone and thyrocalcitonin. The parathyroid glands produce parathyroid hormone. The anterior pituitary produces hormones such as growth hormone, thyroid-stimulating hormone, adrenocorticotropic hormone, gonadotropin, prolactin, and melanocyte-stimulating hormone. PTS: 1 DIF: Cognitive Level: Comprehension OBJ: Nursing Process: Assessment MSC: Client Needs: Physiologic Integrity 14. Chronic adrenocortical insufficiency is also referred to as what? a. Graves’ disease b. Addison’s disease c. Cushing’s syndrome d. Hashimoto’s disease ANS: B Addison’s disease is chronic adrenocortical insufficiency. Graves’ and Hashimoto’s diseases involve the thyroid gland. Cushing’s syndrome is a result of excessive circulation of free cortisol. PTS: 1 DIF: Cognitive Level: Comprehension OBJ: Nursing Process: Diagnosis MSC: Client Needs: Physiologic Integrity 15. A neonate born with ambiguous genitalia is diagnosed with congenital adrenogenital hyperplasia. Therapeutic management includes administration of which medication? a. Vitamin D b. Cortisone c. Stool softeners d. Calcium carbonate ANS: B The most common biochemical defect with congenital adrenal hyperplasia is partial or complete 21-hydroxylase deficiency. With complete deficiency, insufficient amounts of aldosterone and cortisol are produNceUdR,SsIoNGciTrBcu.ClaOtMory collapse occurs without immediate replacement. Vitamin D, stool softeners, and calcium carbonate have no role in the therapy of adrenogenital hyperplasia. PTS: 1 DIF: Cognitive Level: Analysis OBJ: Nursing Process: Implementation MSC: Client Needs: Physiologic Integrity 16. What is the characteristic of the immune-mediated type 1 diabetes mellitus? a. Ketoacidosis is infrequent b. Onset is gradual c. Age at onset is usually younger than 18 years d. Oral agents are often effective for treatment ANS: C The immune-mediated type 1 diabetes mellitus typically has its onset in children or young adults. Peak incidence is between the ages of 10 and 15 years. Infrequent ketoacidosis, gradual onset, and treatment with oral agents are more consistent with type 2 diabetes. PTS: 1 DIF: Cognitive Level: Comprehension OBJ: Nursing Process: Diagnosis MSC: Client Needs: Physiologic Integrity 17. Which symptom is considered a cardinal sign of diabetes mellitus? a. Nausea b. Seizures c. Impaired vision d. Frequent urination ANS: D Hallmarks of diabetes mellitus are glycosuria, polyuria, and polydipsia. Nausea and seizures are not clinical manifestations of diabetes mellitus. Impaired vision is a long-term complication of the disease. PTS: 1 DIF: Cognitive Level: Comprehension OBJ: Nursing Process: Diagnosis MSC: Client Needs: Physiologic Integrity 18. What is the most appropriate intervention for the parents of a 6-year-old girl with precocious puberty? a. Advise the parents to consider birth control for their daughter. b. Explain the importance of having the child foster relationships with same-age peers. c. Assure the child’s parents that there is no increased risk for sexual abuse because of her appearance. d. Counsel parents that there is no treatment currently available for this disorder. ANS: B Despite the child’s appearance, the child needs to be treated according to her chronologic age and to interact with children in the same age-group. An expected outcome is that the child will adjust socially by exhibiting age-appropriate behaviors and social interactions. Advising the parents of a 6 year old to put their daughter on birth control is not appropriate and will not reverse the effects of precocious puberty. Parents need to be aware that there is an increased risk of sexual abuse for a child with precocious puberty. Treatment for precocious puberty is the administration of gonadotropin-releasing hormone blocker, which slows or reverses the development of secondary sexual NchUaRrSaIcNteGrTisBti.CcsOaMnd slows rapid growth and bone aging. PTS: 1 DIF: Cognitive Level: Application OBJ: Nursing Process: Implementation MSC: Client Needs: Health Promotion and Maintenance 19. Type 1 diabetes mellitus is suspected in an adolescent. Which clinical manifestation may be present? a. Moist skin b. Weight gain c. Fluid overload d. Poor wound healing ANS: D Poor wound healing is often an early sign of type 1 diabetes mellitus. Dry skin, weight loss, and dehydration are clinical manifestations of type 1 diabetes mellitus. PTS: 1 DIF: Cognitive Level: Comprehension OBJ: Nursing Process: Diagnosis MSC: Client Needs: Physiologic Integrity 20. A parent asks the nurse why self-monitoring of blood glucose is being recommended for her child with diabetes. The nurse should base the explanation on what information? a. It is a less expensive method of testing. b. It is not as accurate as laboratory testing. c. Children need to learn to manage their diabetes. d. The parents are better able to manage the disease. ANS: C Blood glucose self-management has improved diabetes management and can be used successfully by children from the time of diagnosis. Insulin dosages can be adjusted based on blood sugar results. Blood glucose monitoring is more expensive but provides improved management. It is as accurate as equivalent testing done in laboratories. The ability to self-test allows the child to balance diet, exercise, and insulin. The parents are partners in the process, but the child should be taught how to manage the disease. PTS: 1 DIF: Cognitive Level: Analysis OBJ: Nursing Process: Implementation MSC: Client Needs: Physiologic Integrity 21. The parents of a child who has just been diagnosed with type 1 diabetes ask about exercise. The nurse should provide the parents with what information to address the child’s safety needs? a. Exercise will increase blood glucose. b. Exercise should be restricted. c. Extra snacks are needed before exercise. d. Extra insulin is required during exercise. ANS: C Exercise lowers blood glucose levels, which can be compensated for by extra snacks. Exercise is encouraged and not restricted unless indicated by other health conditions. Extra insulin is contraindicated because exercise decreases blood glucose levels. PTS: 1 DIF: Cognitive Level: Application OBJ: Nursing Process: Planning MSC: Client Needs: Physiologic Integrity NURSINGTB.COM 22. A child eats some sugar cubes after experiencing symptoms of hypoglycemia. This rapid-releasing sugar should be followed by: a. saturated and unsaturated fat. b. fruit juice. c. several glasses of water. d. complex carbohydrate and protein. ANS: D Symptoms of hypoglycemia are treated with a rapid-releasing sugar source followed by a complex carbohydrate and protein. Saturated and unsaturated fat, fruit juice, and several glasses of water do not provide the child with complex carbohydrate and protein necessary to stabilize the blood sugar. PTS: 1 DIF: Cognitive Level: Application OBJ: Nursing Process: Planning MSC: Client Needs: Physiologic Integrity 23. What are the manifestations of hypoglycemia? a. Lethargy b. Thirst c. Nausea and vomiting d. Shaky feeling and dizziness ANS: D Some of the clinical manifestations of hypoglycemia include shaky feelings; dizziness; difficulty concentrating, speaking, focusing, and coordinating; sweating; and pallor. Lethargy, thirst, and nausea and vomiting are manifestations of hyperglycemia. PTS: 1 DIF: Cognitive Level: Comprehension OBJ: Nursing Process: Assessment MSC: Client Needs: Physiologic Integrity 24. The nurse is caring for an 11-year-old boy who has recently been diagnosed with diabetes. What should be included in the teaching plan for daily injections? a. The parents do not need to learn the procedure. b. He is old enough to give most of his own injections. c. Self-injections will be possible when he is closer to adolescence. d. He can learn about self-injections when he is able to reach all injection sites. ANS: B School-age children are able to give their own injections. Parents should participate in learning and giving the insulin injections. He is already old enough to administer his own insulin. The child is able to use thighs, abdomen, part of the hip, and arm. Assistance can be obtained if other sites are used. PTS: 1 DIF: Cognitive Level: Application OBJ: Nursing Process: Implementation MSC: Client Needs: Physiologic Integrity 25. The nurse is discussing various sites used for insulin injections with a child and her family. Which site usually has the fastest rate of absorption? a. Arm b. Leg c. Buttock d. Abdomen ANS: D NURSINGTB.COM The abdomen has the fastest rate of absorption but the shortest duration. The arm has a fast rate of absorption but short duration. The leg has a slow rate of absorption but a long duration. The buttock has the slowest rate of absorption and the longest duration. PTS: 1 DIF: Cognitive Level: Application OBJ: Nursing Process: Implementation MSC: Client Needs: Physiologic Integrity 26. What should a nurse advise the parents of a child with type 1 diabetes mellitus who is not eating as a result of a minor illness? a. Give the child half his regular morning dose of insulin. b. Substitute simple carbohydrates or calorie-containing liquids for solid foods. c. Give the child plenty of unsweetened, clear liquids to prevent dehydration. d. Take the child directly to the emergency department. ANS: B A sick-day diet of simple carbohydrates or calorie-containing liquids will maintain normal serum glucose levels and decrease the risk of hypoglycemia. The child should receive his regular dose of insulin even if he does not have an appetite. If the child is not eating as usual, he needs calories to prevent hypoglycemia. During periods of minor illness, the child with type 1 diabetes mellitus can be managed safely at home. PTS: 1 DIF: Cognitive Level: Application OBJ: Nursing Process: Implementation MSC: Client Needs: Health Promotion and Maintenance 27. Which laboratory finding confirms that a child with type 1 diabetes is experiencing diabetic ketoacidosis? a. No urinary ketones b. Low arterial pH c. Elevated serum carbon dioxide d. Elevated serum phosphorus ANS: B Severe insulin deficiency produces metabolic acidosis, which is indicated by a low arterial pH. Urinary ketones, often in large amounts, are present when a child is in diabetic ketoacidosis. Serum carbon dioxide is decreased in diabetic ketoacidosis. Serum phosphorus is decreased in diabetic ketoacidosis. PTS: 1 DIF: Cognitive Level: Analysis OBJ: Nursing Process: Assessment MSC: Client Needs: Physiologic Integrity 28. A child diagnosed with hypopituitarism is being started on growth hormone (GH) therapy. Nursing considerations should be based on which information? a. Treatment is most successful if it is started during adolescence. b. Treatment is considered successful if children attain full stature by adulthood. c. Replacement therapy requires daily subcutaneous injections. d. Replacement therapy will be required throughout the child’s lifetime. ANS: C Additional support is required forNcUhiRldSIrNenGTwBh.oCOreMquire hormone replacement therapy, such as preparation for daily subcutaneous injections and education for self-management during the school-age years. Young children, obese children, and those who are severely GH deficient have the best response to therapy. When therapy is successful, children can attain their actual or near-final adult height at a slower rate than their peers. Replacement therapy is not needed after attaining final height. They are no longer GH deficient. PTS: 1 DIF: Cognitive Level: Analysis OBJ: Nursing Process: Planning MSC: Client Needs: Physiologic Integrity 29. An adolescent is being seen in the clinic for evaluation of acromegaly. The nurse understands that which event occurs with acromegaly? a. There is a lack of growth hormone (GH) being produced. b. There is excess GH after closure of the epiphyseal plates. c. There is an excess of GH before the closure of the epiphyseal plates. d. There is a lack of thyroid hormone being produced. ANS: B Excess GH after closure of the epiphyseal plates results in acromegaly. A lack of growth hormone results in delayed growth or even dwarfism. Gigantism occurs when there is hypersecretion of GH before the closure of the epiphyseal plates. Cretinism is associated with hypothyroidism. PTS: 1 DIF: Cognitive Level: Comprehension OBJ: Nursing Process: Assessment MSC: Client Needs: Physiologic Integrity 30. The nurse is admitting a toddler with the diagnosis of juvenile hypothyroidism. Which is a common clinical manifestation of this disorder? a. Insomnia b. Diarrhea c. Dry skin d. Accelerated growth ANS: C Dry skin, mental decline, and myxedematous skin changes are associated with juvenile hypothyroidism. Children with hypothyroidism are usually sleepy. Constipation is associated with hypothyroidism. Decelerated growth is common in juvenile hypothyroidism. PTS: 1 DIF: Cognitive Level: Application OBJ: Nursing Process: Assessment MSC: Client Needs: Physiologic Integrity 31. Which clinical manifestation may occur in the child who is prescribed methimazole for the treatment of hyperthyroidism (Graves’ disease)? a. Seizures b. Enlargement of all lymph glands c. Pancreatitis or cholecystitis d. Sore throat or fever ANS: D Children being treated with propylthiouracil or methimazole must be carefully monitored for side effects of the drug. Because sore throat and fever accompany the grave complication of leukopenia, these children should be seen by a health care practitioner if such symptoms occur. Neither seizures, cholecystiNtiUsRnSoIrNpGaTnBc.rCeOaMtitis are associated with the administration of methimazole. Enlargement of the salivary and cervical lymph glands may occur. PTS: 1 DIF: Cognitive Level: Application OBJ: Nursing Process: Implementation MSC: Client Needs: Physiologic Integrity 32. The parent of a child diagnosed with diabetes mellitus asks the nurse when urine testing will be necessary. The nurse should explain that urine testing is necessary for which reason? a. Glucose is needed before administration of insulin. b. Glucose is needed 4 times a day. c. Glycosylated hemoglobin is required. d. Ketonuria is suspected. ANS: D Urine testing is still performed to detect evidence of ketonuria. Urine testing for glucose is no longer indicated for medication administration because of the poor correlation between blood glucose levels and glycosuria. Glycosylated hemoglobin analysis is performed on a blood sample. PTS: 1 DIF: Cognitive Level: Application OBJ: Nursing Process: Teaching/Learning MSC: Client Needs: Physiologic Integrity: Physiologic Adaptation 33. To help the adolescent deal with diabetes, the nurse must consider which characteristic of adolescence? a. Desire to be unique b. Preoccupation with the future c. Need to be perfect and similar to peers d. Need to make peers aware of the seriousness of hypoglycemic reactions ANS: C Adolescence is a time when the individual wants to be perfect and similar to peers. Having diabetes makes adolescents different from their peers. Adolescents do not wish to be unique; they desire to fit in with the peer group and are usually not future oriented. Forcing peer awareness of the seriousness of hypoglycemic reactions would further alienate the adolescent with diabetes since the peer group would likely focus on the differences. PTS: 1 DIF: Cognitive Level: Analysis OBJ: Nursing Process: Implementation MSC: Client Needs: Health Promotion and Maintenance 34. The nurse is implementing care for a school-age child admitted to the pediatric intensive care experiencing symptomology associated with diabetic ketoacidosis (DKA). Which prescribed intervention should the nurse implement first? a. Begin 0.9% saline solution intravenously as prescribed. b. Administer regular insulin intravenously as prescribed. c. Place child on a cardiac monitor. d. Place child on a pulse oximetry monitor. ANS: A All patients with DKA experience dehydration (10% of total body weight in severe ketoacidosis) because of the osmotic diuresis, accompanied by depletion of electrolytes (sodium, potassium, chloride, phosphate, and magnesium). The initial hydrating solution is 0.9% saline solution. Insulin theraNpUyRsShIoNuGldTBb.eCOstMarted after the initial rehydration bolus because serum glucose levels fall rapidly after volume expansion. The child should be placed on the cardiac and pulse oximetry monitors after the rehydrating solution has been initiated. PTS: 1 DIF: Cognitive Level: Analysis OBJ: Nursing Process: Implementation MSC: Client Needs: Physiologic Integrity 35. A nurse is reviewing the laboratory results on a school-age child diagnosed with hypoparathyroidism. Which results are consistent with this condition? a. Decreased serum phosphorus b. Decreased serum calcium c. Increased serum glucose d. Decreased serum cortisol ANS: B The diagnosis of hypoparathyroidism is made on the basis of clinical manifestations associated with decreased serum calcium and increased serum phosphorus. Decreased serum phosphorus would be seen in hyperparathyroidism, elevated glucose in diabetes, and decreased serum cortisol in adrenocortical insufficiency (Addison’s disease). PTS: 1 DIF: Cognitive Level: Comprehension OBJ: Nursing Process: Evaluation MSC: Client Needs: Physiologic Integrity: Physiologic Adaptation MULTIPLE RESPONSE 1. Nursing care of a child diagnosed with syndrome of inappropriate antidiuretic hormone (SIADH) should include which interventions? (Select all that apply.) a. Weigh daily b. Encourage fluids c. Turn frequently d. Maintain nothing by mouth e. Restrict fluids ANS: A, E Increased secretion of ADH causes the kidney to resorb water, which increases fluid volume and decreases serum osmolarity with a progressive reduction in sodium concentration. The immediate management of the child is to restrict fluids. The child should also be weighed at the same time each day. Encouraging fluids, turning frequently, and maintaining nothing by mouth are not associated with SIADH since they are not associated with managing/monitoring for fluid retention. PTS: 1 DIF: Cognitive Level: Application OBJ: Nursing Process: Planning MSC: Client Needs: Physiologic Integrity 2. Which children admitted to the pediatric unit would the nurse monitor closely for development of syndrome of inappropriate antidiuretic hormone (SIADH)? (Select all that apply.) a. A newly diagnosed preschooler with type 1 diabetes. b. A school-age child returning from surgery for removal of a brain tumor. c. An infant with suspected meningitis. d. An adolescent with blunt abdoNmUiRnSaIlNtrGaTuBm.CaOfMollowing a car accident. e. A school-age child with head trauma. ANS: B, C, E The disorder that results from hypersecretion of ADH from the posterior pituitary hormone. Childhood SIADH usually is caused by disorders affecting the central nervous system, such as infections (meningitis), head trauma, and brain tumors. Type 1 diabetes and blunt abdominal trauma are not likely to cause SIADH since do not affect secretion of this hormone. PTS: 1 DIF: Cognitive Level: Analysis OBJ: Nursing Process: Assessment MSC: Client Needs: Physiologic Integrity 3. A child is diagnosed with juvenile hypothyroidism. The nurse should expect to assess which symptoms associated with hypothyroidism? (Select all that apply.) a. Weight loss b. Sleepiness c. Diarrhea d. Puffiness around the eyes e. Spare hair ANS: B, D, E A child diagnosed with juvenile hypothyroidism will display sleepiness; dry, periorbital puffiness; and spare hair growth. Weight loss and diarrhea are signs of hyperthyroidism. 4. The nurse should expect to assess which clinical manifestations in an adolescent with Cushing’s syndrome? (Select all that apply.) a. Hyperglycemia b. Hyperkalemia c. Hypotension d. Cushingoid features e. Susceptibility to infections ANS: A, D, E In Cushing’s syndrome, physiologic disturbances seen are cushingoid features, hyperglycemia, susceptibility to infection, hypertension, and hypokalemia. PTS: 1 DIF: Cognitive Level: Comprehension OBJ: Nursing Process: Assessment MSC: Client Needs: Physiologic Integrity: Physiologic Adaptation 5. A nurse is planning care for a school-age child diagnosed with type 1 diabetes. Which insulin preparations are either rapid or short acting? (Select all that apply.) a. Novolin N b. Lantus c. NovoLog d. Novolin R ANS: C, D Rapid-acting insulin (e.g., NovoLog) reaches the blood within 15 minutes after injection. The insulin peaks 30 to 90 minutes later and may last as long as 5 hours. Short-acting (regular) insulin (e.g., Novolin R) usually rNeaUcRhSeIsNtGhTeBb.lCoOoMd within 30 minutes after injection. The insulin peaks 2 to 4 hours later and stays in the blood for about 4 to 8 hours. Intermediate-acting insulins (e.g., Novolin N) reach the blood 2 to 6 hours after injection. The insulins peak 4 to 14 hours later and stay in the blood for about 14 to 20 hours. Long-acting insulin (e.g., Lantus) takes 6 to 14 hours to start working. It has no peak or a very small peak 10 to 16 hours after injection. The insulin stays in the blood between 20 and 24 hours. PTS: 1 DIF: Cognitive Level: Comprehension OBJ: Nursing Process: Planning MSC: Client Needs: Physiologic Integrity 6. The nurse is caring for a school-age child with hyperthyroidism (Graves’ disease). Which clinical manifestations should the nurse monitor that may indicate a thyroid storm? (Select all that apply.) a. Constipation b. Hypotension c. Hyperthermia d. Tachycardia e. Vomiting ANS: C, D, E A child with a thyroid storm will have severe irritability and restlessness, vomiting, diarrhea, hyperthermia, hypertension, severe tachycardia, and prostration. COMPLETION 1. The clinic nurse is reviewing hemoglobin A1c levels on several children with type 1 diabetes. Hemoglobin A1c levels of less than % are a goal for children with type 1 diabetes. Record your answer as a whole number. ANS: 7 The measurement of glycosylated hemoglobin (hemoglobin A1c) levels is a satisfactory method for assessing control of type 1 diabetes. As red blood cells circulate in the bloodstream, glucose molecules gradually attach to the hemoglobin A molecules and remain there for the lifetime of the red blood cell, approximately 120 days. The attachment is not reversible; therefore, this glycosylated hemoglobin reflects the average blood glucose levels over the previous 2 to 3 months. The test is a satisfactory method for assessing control, detecting incorrect testing, monitoring the effectiveness of changes in treatment, defining patients’ goals, and detecting nonadherence. Hemoglobin A1c levels of less than 7% are a well-established goal at most care centers. PTS: 1 DIF: Cognitive Level: Comprehension OBJ: Nursing Process: Evaluation MSC: Client Needs: Physiologic Integrity NURSINGTB.COM Chapter 48: The Child With Musculoskeletal or Articular Dysfunction Perry: Maternal Child Nursing Care, 6th Edition MULTIPLE CHOICE 1. The nurse is caring for a 4-year-old child immobilized by a fractured hip. Which complication should the nurse monitor for? a. Hypocalcemia b. Decreased metabolic rate c. Positive nitrogen balance d. Increased production of stress hormones ANS: B Immobilization causes a decreased metabolic rate with slowing of all systems and a decreased food intake, leads to hypercalcemia, and causes a negative nitrogen balance secondary to muscle atrophy. A decreased production of stress hormones occurs with decreased physical and emotional coping capacity. PTS: 1 DIF: Cognitive Level: Application OBJ: Nursing Process: Assessment MSC: Client Needs: Physiologic Integrity 2. What effect does immobilization have on the cardiovascular system? a. Venous stasis b. Increased vasopressor mechanism c. Normal distribution of blood volume d. Increased efficiency of orthostNaUtiRc SnIeNuGroTvBa.CscOuMlar reflexes ANS: A Because of decreased muscle contraction, the physiologic effects of immobilization include venous stasis. This can lead to pulmonary emboli or thrombi. A decreased vasopressor mechanism results in orthostatic hypotension, syncope, hypotension, decreased cerebral blood flow, and tachycardia. An altered distribution of blood volume is found, with decreased cardiac workload and exercise tolerance. Immobilization causes a decreased efficiency of orthostatic neurovascular reflexes, with an inability to adapt readily to the upright position and pooling of blood in the extremities in the upright position. PTS: 1 DIF: Cognitive Level: Comprehension OBJ: Nursing Process: Assessment MSC: Client Needs: Physiologic Integrity 3. Which condition can result from the bone demineralization associated with immobility? a. Osteoporosis b. Urinary retention c. Pooling of blood d. Susceptibility to infection ANS: A Bone demineralization leads to a negative calcium balance, osteoporosis, pathologic fractures, extraosseous bone formation, and renal calculi. Urinary retention is secondary to the effect of immobilization on the urinary tract. Pooling of blood is a result of the cardiovascular effects of immobilization. Susceptibility to infection can result from the effects of immobilization on the respiratory and renal systems. PTS: 1 DIF: Cognitive Level: Comprehension OBJ: Nursing Process: Assessment MSC: Client Needs: Physiologic Integrity 4. A young child has just injured an ankle at school. In addition to calling the child's parents, what is the most appropriate immediate action by the school nurse? a. Apply ice b. Observe for edema and discoloration c. Encourage child to assume a comfortable position d. Obtain parental permission for administration of acetaminophen or aspirin ANS: A Soft-tissue injuries should be iced immediately. In addition to ice, the extremity should be rested, be elevated, and have compression applied. Observing for edema and discoloration, encouraging the child to assume a comfortable position, and obtaining parental permission or administration of acetaminophen or aspirin are not immediate priorities. PTS: 1 DIF: Cognitive Level: Analysis OBJ: Nursing Process: Implementation MSC: Client Needs: Physiologic Integrity 5. Which term is used to describe a type of fracture that does not produce a break in the skin? a. Simple b. Compound c. Complicated d. Comminuted ANS: A NURSINGTB.COM If a fracture does not produce a break in the skin, it is called a simple or closed fracture. A compound or open fracture is one with an open wound through which the bone protrudes. A complicated fracture is one in which the bone fragments damage other organs or tissues. A comminuted fracture occurs when small fragments of bone are broken from the fractured shaft and lie in the surrounding tissue. These are rare in children. PTS: 1 DIF: Cognitive Level: Comprehension OBJ: Nursing Process: Assessment MSC: Client Needs: Physiologic Integrity 6. What is an advantage to using a fiberglass cast instead of a plaster cast? a. Is less expensive b. Dries rapidly c. Molds closely to body parts d. Has a smooth exterior ANS: B A synthetic casting material dries in 5 to 30 minutes as compared with a plaster cast, which takes 10 to 72 hours to dry. Synthetic casts are more expensive. Plaster casts mold closer to body parts. Synthetic casts have a rough exterior, which may scratch surfaces. PTS: 1 DIF: Cognitive Level: Application OBJ: Nursing Process: Assessment MSC: Client Needs: Physiologic Integrity 7. The nurse is teaching the parents of a 7-year-old child who has just had a cast applied for a fractured arm with the wrist and elbow immobilized. Which instructions should be included in the teaching? a. Swelling of the fingers is to be expected for the next 48 hours. b. Immobilize the shoulder to decrease pain in the arm. c. Allow the affected limb to hang down for 1 hour each day. d. Elevate casted arm when resting and when sitting up. ANS: D The injured extremity should be kept elevated while resting and in a sling when upright. This will increase venous return. Swelling of the fingers may indicate neurovascular damage and should be reported immediately. Permanent damage can occur within 6 to 8 hours. Joints above and below the cast on the affected extremity should be moved. The child should not engage in strenuous activity for the first few days. Rest with elevation of the extremity is encouraged. PTS: 1 DIF: Cognitive Level: Application OBJ: Nursing Process: Implementation MSC: Client Needs: Physiologic Integrity 8. The nurse uses the palms of the hands when handling a wet cast to achieve what outcome? a. Assess dryness of the cast b. Facilitate easy turning c. Keep the patient's limb balanced d. Avoid indenting the cast ANS: D NURSINGTB.COM Wet casts should be handled by the palms of the hands, not the fingers, to prevent creating pressure points. Assessing dryness, facilitating easy turning, or keeping the patient's limb balanced are not reasons for using the palms of the hand rather than the fingers when handling a wet cast. PTS: 1 DIF: Cognitive Level: Comprehension OBJ: Nursing Process: Implementation MSC: Client Needs: Physiologic Integrity 9. What would cause a nurse to suspect that an infection has developed under a cast? a. Complaint of paresthesia b. Cold toes c. Increased respirations d. “Hot spots” felt on cast surface ANS: D If hot spots are felt on the cast surface, they usually indicate infection beneath the area. This should be reported so a window can be made in the cast to observe the site. The “five Ps” of ischemia from a vascular injury include pain, pallor, pulselessness, paresthesia, and paralysis. Paresthesia is an indication of vascular injury, not infection. Cold toes may be indicative of too tight a cast and need further evaluation. Increased respirations may indicate a respiratory infection or pulmonary emboli. This should be reported, and the child should be evaluated. PTS: 1 DIF: Cognitive Level: Analysis OBJ: Nursing Process: Diagnosis MSC: Client Needs: Physiologic Integrity 10. A child is upset because, when the leg cast is removed, the skin surface is caked with desquamated skin and sebaceous secretions. What should the nurse suggest to remove this material? a. Soak in a bathtub b. Vigorously scrub the leg c. Apply powder to absorb material d. Carefully pick material off of the leg ANS: A Simple soaking in the bathtub is usually sufficient for the removal of the desquamated skin and sebaceous secretions. It may take several days to eliminate the accumulation completely. The parents and child should be advised not to scrub the leg vigorously or forcibly remove this material because it may cause excoriation and bleeding. Oil or lotion, but not powder, may provide comfort for the child. PTS: 1 DIF: Cognitive Level: Application OBJ: Nursing Process: Implementation MSC: Client Needs: Physiologic Integrity 11. Which type of traction uses skin traction on the lower leg and a padded sling under the knee? a. Dunlop b. Bryant's c. Russell d. Buck's extension ANS: C Russell traction uses skin tractionNoUnRtShIeNlGoTwBe.rClOeMg and a padded sling under the knee. The combination of longitudinal and perpendicular traction allows realignment of the lower extremity and immobilizes the hips and knees in a flexed position. Dunlop traction is an upper-extremity traction used for fractures of the humerus. Bryant's traction is skin traction with the legs flexed at a 90-degree angle at the hip. Buck's extension traction is a type of skin traction with the legs in an extended position. It is used primarily for short-term immobilization, before surgery with dislocated hips, for correcting contractures, or for bone deformities such as Legg-Calvé-Perthes disease. PTS: 1 DIF: Cognitive Level: Comprehension OBJ: Nursing Process: Assessment MSC: Client Needs: Physiologic Integrity 12. Four year old, placed in Buck's extension traction for Legg-Calvé-Perthes disease, is crying with pain as the nurse assesses that the skin of the right foot is pale with an absence of pulse. What should the nurse do first? a. Notify the practitioner of the changes noted. b. Give the child medication to relieve the pain. c. Reposition the child and notify the physician. d. Chart the observations and check the extremity again in 15 minutes. ANS: A The absence of a pulse and change in color of the foot must be reported immediately for evaluation by the practitioner. Pain medication and repositioning should be addressed after the practitioner is notified. This is an emergency condition; immediate reporting is indicated. The findings should be documented with ongoing assessment. PTS: 1 DIF: Cognitive Level: Analysis OBJ: Nursing Process: Diagnosis MSC: Client Needs: Physiologic Integrity 13. What is an appropriate nursing intervention when caring for a child in traction? a. Remove adhesive traction straps daily to prevent skin breakdown. b. Assess for tightness, weakness, or contractures in uninvolved joints and muscles. c. Provide active range-of-motion exercises to affected extremity 3 times a day. d. Keep child in one position to maintain good alignment. ANS: B Traction places stress on the affected bone, joint, and muscles. The nurse must assess for tightness, weakness, or contractures developing in the uninvolved joints and muscles. The adhesive straps should be released/replaced only when absolutely necessary. Active, passive, or active with resistance exercises should be carried out for the unaffected extremity only. Movement is expected with children. Each time the child moves, the nurse should check to ensure that proper alignment is maintained. PTS: 1 DIF: Cognitive Level: Application OBJ: Nursing Process: Implementation MSC: Client Needs: Physiologic Integrity 14. The nurse is teaching a family how to care for their infant in a Pavlik harness to treat developmental dysplasia of the hips (DDH). What information should be included? a. Apply lotion or powder to minNimURizSeINsGkiTnBi.rCrOitMation. b. Remove the harness several times a day to prevent contractures. c. Hip stabilization usually occurs within 12 weeks. d. Place a diaper over harness, preferably using a superabsorbent disposable diaper that is relatively thin. ANS: C The harness is worn continuously until the hip is proved stable on both clinical and ultrasound examination, usually within 6 to 12 weeks. Lotions and powders should not be used with the harness. The harness should not be removed, except as directed by the practitioner. A thin disposable diaper can be placed under the harness. PTS: 1 DIF: Cognitive Level: Comprehension OBJ: Nursing Process: Implementation MSC: Client Needs: Physiologic Integrity 15. A neonate is born with mild clubfeet. When the parents ask the nurse how this will be corrected, the nurse should base the explanation on what fact? a. Traction is tried first. b. Surgical intervention is needed. c. Frequent, serial casting is tried first. d. Children outgrow this condition when they learn to walk. ANS: C Serial casting, the preferred treatment, is begun shortly after birth before discharge from the nursery. Successive casts allows for gradual stretching of skin and tight structures on the medial side of the foot. Manipulation and casting of the leg are repeated frequently (every week) to accommodate the rapid growth of early infancy. Surgical intervention is done only if serial casting is not successful. Children do not improve without intervention. PTS: 1 DIF: Cognitive Level: Comprehension OBJ: Nursing Process: Implementation MSC: Client Needs: Physiologic Integrity 16. Which term is used to describe an abnormally increased convex angulation in the curvature of the thoracic spine? a. Scoliosis b. Ankylosis c. Lordosis d. Kyphosis ANS: D Kyphosis is an abnormally increased convex angulation in the curve of the thoracic spine. Scoliosis is a complex spinal deformity usually involving lateral curvature, spinal rotation causing rib asymmetry, and thoracic hypokyphosis. Ankylosis is the immobility of a joint. Lordosis is an accentuation of the cervical or lumbar curvature beyond physiologic limits. PTS: 1 DIF: Cognitive Level: Comprehension OBJ: Nursing Process: Assessment MSC: Client Needs: Physiologic Integrity 17. When does idiopathic scoliosis become most noticeable? a. Newborn period b. When child starts to walk NURSINGTB.COM c. During preadolescent growth spurt d. Adolescence ANS: C Idiopathic scoliosis is most noticeable during the preadolescent growth spurt and is seldom apparent before age 10 years. PTS: 1 DIF: Cognitive Level: Comprehension OBJ: Nursing Process: Assessment MSC: Client Needs: Physiologic Integrity 18. What is the initial method of treating osteomyelitis? a. Joint replacement b. Bracing and casting c. Intravenous antibiotic therapy d. Long-term corticosteroid therapy ANS: C Osteomyelitis is an infection of the bone, most commonly caused by Staphylococcus aureus. The treatment of choice is antibiotics delivered intravenous and then possibly by the oral route. Joint replacement, bracing and casting, and long-term corticosteroids are not indicated for infectious processes. PTS: 1 DIF: Cognitive Level: Comprehension OBJ: Nursing Process: Implementation MSC: Client Needs: Physiologic Integrity 19. An adolescent is scheduled for a leg amputation in 2 days for treatment of osteosarcoma. The nurse's approach should include what intervention? a. Answering questions with straightforward honesty. b. Avoiding discussing the seriousness of the condition. c. Explaining that, although the amputation is difficult, it will cure the cancer. d. Assisting the adolescent in accepting the amputation as better than a long course of chemotherapy. ANS: A Honesty is essential to gain the cooperation and trust of the child. The diagnosis of cancer should not be disguised with falsehoods. The adolescent should be prepared in advance for the surgery so that there is time for reflection about the diagnosis and subsequent treatment. This allows questions to be answered. To accept the need for radical surgery, the child must be aware of the lack of alternatives for treatment. Amputation is necessary, but it will not guarantee a cure. Chemotherapy is an integral part of the therapy with surgery. The child should be informed of the need for chemotherapy and its side effects before surgery. PTS: 1 DIF: Cognitive Level: Application OBJ: Nursing Process: Implementation MSC: Client Needs: Health Promotion and Maintenance 20. Which medication is usually tried first when a child is diagnosed with juvenile idiopathic arthritis (JIA)? a. Aspirin b. Corticosteroids c. Cytotoxic drugs such as methotrexate d. Nonsteroidal antiinflammatoryNUdrRuSgIsNG(NTSBA.CIODMs) ANS: D NSAIDs are the first drugs used in JIA. Naproxen, ibuprofen, and tolmetin are approved for use in children. Aspirin, once the drug of choice, has been replaced by the NSAIDs because they have fewer side effects and easier administration schedules. Corticosteroids are used for life-threatening complications, incapacitating arthritis, and uveitis. Methotrexate is a second-line therapy for JIA. PTS: 1 DIF: Cognitive Level: Comprehension OBJ: Nursing Process: Implementation MSC: Client Needs: Physiologic Integrity 21. What nursing consideration is especially important when caring for a child diagnosed with juvenile idiopathic arthritis (JIA)? a. Apply ice packs to relieve stiffness and pain. b. Administer acetaminophen to reduce inflammation. c. Teach child and family the correct administration of medications. d. Encourage range-of-motion exercises during periods of inflammation. ANS: C The management of JIA is primarily pharmacologic. The family should be instructed regarding administration of medications and the value of a regular schedule of administration to maintain a satisfactory blood level in the body. They need to know that nonsteroidal antiinflammatory drugs should not be given on an empty stomach and to be alert for signs of toxicity. Warm, moist heat is best for relieving stiffness and pain. Acetaminophen does not have antiinflammatory effects. Range-of-motion exercises should not be done during periods of inflammation. PTS: 1 DIF: Cognitive Level: Application OBJ: Nursing Process: Implementation MSC: Client Needs: Physiologic Integrity 22. When infants are seen for fractures, which nursing intervention is a priority? a. No intervention is necessary. It is not uncommon for infants to fracture bones. b. Assess the family's safety practices. Fractures in infants usually result from falls. c. Assess for child abuse. Fractures in infants are often nonaccidental. d. Assess for genetic factors. ANS: C Fractures in infants warrant further investigation to rule out child abuse. Fractures in children younger than 1 year are unusual because of the cartilaginous quality of the skeleton; a large amount of force is necessary to fracture their bones. Infants should be cared for in a safe environment and should not be falling. Fractures in infancy are usually nonaccidental rather than related to a genetic factor. PTS: 1 DIF: Cognitive Level: Application OBJ: Nursing Process: Assessment MSC: Client Needs: Physiologic Integrity 23. Which nursing intervention is appNroUpRrSiaINteGtToBa.sCsOeMss for neurovascular competency in a child suspected of experiencing compartment syndrome? a. The degree of motion and ability to position the extremity. b. The length, diameter, and shape of the extremity. c. The amount of swelling noted in the extremity and pain intensity. d. The skin color, temperature, movement, sensation, and capillary refill of the extremity. ANS: D A neurovascular evaluation includes assessing skin color and temperature, ability to move the affected extremity, degree of sensation experienced, and speed of capillary refill in the extremity. The degree of motion in the affected extremity and ability to position the extremity are incomplete assessments of neurovascular competency. The length, diameter, and shape of the extremity are not assessment criteria in a neurovascular evaluation. Although the amount of swelling is an important factor in assessing an extremity, it is not a criterion for a neurovascular assessment. PTS: 1 DIF: Cognitive Level: Application OBJ: Nursing Process: Assessment MSC: Client Needs: Physiologic Integrity 24. Which interaction is part of the discharge plan for a school-age child with osteomyelitis who is receiving home antibiotic therapy? a. Instructions for a low-calorie diet b. Arrangements for tutoring and schoolwork c. Instructions for a high-fat, low-protein diet d. Instructions for the parent to return the child to team sports immediately ANS: B Promoting optimal growth and development in the school-age child is important. It is important to continue schoolwork and arrange for tutoring if indicated. The child with osteomyelitis should be on a high-calorie, high-protein diet. The child with osteomyelitis may need time for the bone to heal before returning to full activities. PTS: 1 DIF: Cognitive Level: Application OBJ: Nursing Process: Planning MSC: Client Needs: Health Promotion and Maintenance 25. Discharge planning for the child diagnosed with juvenile arthritis includes the need for which intervention? a. Routine ophthalmologic examinations to assess for visual problems. b. A low-calorie diet to decrease or control weight in the less mobile child. c. Avoiding the use of aspirin to decrease gastric irritation. d. Immobilizing the painful joints, which are the results of the inflammatory process. ANS: A The systemic effects of juvenile arthritis can result in visual problems, making routine eye examinations important. Children with juvenile arthritis do not have problems with increased weight and often are anorexic and in need of high-calorie diets. Children with arthritis are often treated with aspirin. Children with arthritis are able to immobilize their own joints. Range-of-motion exercises are important for maintaining joint flexibility and preventing restricted movement in the affected joints. PTS: 1 DIF: CognitiNveURLSevINelG: TABp.pClOicMation OBJ: Nursing Process: Planning MSC: Client Needs: Health Promotion and Maintenance 26. When assessing the child with osteogenesis imperfecta, the nurse should expect to observe clinical feature? a. Discolored teeth b. Below-normal intelligence c. Increased muscle tone d. Above-average stature ANS: A Children with osteogenesis imperfecta have incomplete development of bones, teeth, ligaments, and sclerae. Teeth are discolored because of abnormal enamel. Despite their appearance, children with osteogenesis imperfecta have normal or above-normal intelligence. The child with osteogenesis imperfecta has weak muscles and decreased muscle tone. Because of compression fractures of the spine, the child appears short. PTS: 1 DIF: Cognitive Level: Application OBJ: Nursing Process: Assessment MSC: Client Needs: Physiologic Integrity 27. A 10 year old sustained a fracture in the epiphyseal plate of the right fibula when falling from a tree. When discussing this injury with the child's parents, the nurse should consider which statement? a. Healing is usually delayed in this type of fracture. b. Growth can be affected by this type of fracture. c. This is an unusual fracture site in young children. d. This type of fracture is inconsistent with a fall. ANS: B Detection of epiphyseal injuries is sometimes difficult, but fractures involving the epiphysis or epiphyseal plate present special problems in determining whether bone growth will be affected. Healing of epiphyseal injuries is usually prompt. The epiphysis is the weakest point of the long bones. This is a frequent site of damage during trauma. PTS: 1 DIF: Cognitive Level: Application OBJ: Nursing Process: Teaching/Learning MSC: Client Needs: Physiologic Integrity 28. A 4-year-old child is newly diagnosed with Legg-Calvé-Perthes disease. Nursing considerations should include which action? a. Encouraging normal activity for as long as is possible. b. Explaining the cause of the disease to the child and family. c. Preparing the child and family for long-term, permanent disabilities. d. Teaching the family the care and management of the corrective appliance. ANS: D The family needs to learn the purpose, function, application, and care of the corrective device and the importance of compliance to achieve the desired outcome. The initial therapy is rest and nonweight bearing, which helps reduce inflammation and restore motion. Legg-Calvé-Perthes is a disease with an unknown etiology. A disturbance of circulation to the femoral capital epiphysis produces an ischemic aseptic necrosis of the femoral head. The disease is self-limiting, but the ultimate outcome of therapy depends on early and efficient therapy and the child's age at onseNtU. RSINGTB.COM PTS: 1 DIF: Cognitive Level: Application OBJ: Nursing Process: Planning MSC: Client Needs: Physiologic Integrity: Physiologic Adaptation 29. A nurse is conducting discharge teaching for parents of an infant diagnosed with osteogenesis imperfecta (OI). Further teaching is indicated if the parents make which statement? a. “We will be very careful handling the baby.” b. “We will lift the baby by the buttocks when diapering.” c. “We're glad there is a cure for this disorder.” d. “We will schedule follow-up appointments as instructed.” ANS: C The treatment for OI is primarily supportive. Although patients and families are optimistic about new research advances, there is no cure. The use of bisphosphonate therapy with IV pamidronate to promote increased bone density and prevent fractures has become standard therapy for many children with OI; however, long bones are weakened by prolonged treatment. Infants and children with this disorder require careful handling to prevent fractures. They must be supported when they are being turned, positioned, moved, and held. Even changing a diaper may cause a fracture in severely affected infants. These children should never be held by the ankles when being diapered but should be gently lifted by the buttocks or supported with pillows. Follow-up appointments for treatment with bisphosphonate can be expected. PTS: 1 DIF: Cognitive Level: Analysis OBJ: Nursing Process: Evaluation MSC: Client Needs: Physiologic Integrity MULTIPLE RESPONSE 1. The nurse is caring for an infant with developmental dysplasia of the hips (DDH). Which clinical manifestations should the nurse expect to observe? (Select all that apply.) a. Positive Ortolani sign b. Unequal gluteal folds c. Negative Babinski's sign d. Trendelenburg's sign e. Telescoping of the affected limb f. Lordosis ANS: A, B A positive Ortolani sign and unequal gluteal folds are clinical manifestations of developmental dysplasia of the hips (DDH) seen from birth to 2 to 3 months. Trendelenburg's sign is noted in a child capable of standing alone. Negative Babinski's sign, telescoping of the affected limb, and lordosis are not clinical manifestations of developmental dysplasia of the hips (DDH). PTS: 1 DIF: Cognitive Level: Application OBJ: Nursing Process: Assessment MSC: Client Needs: Physiologic Integrity 2. A school-age child is diagnosed with systemic lupus erythematosus (SLE). The nurse should plan to implement which interventions for this child? (Select all that apply.) a. Instructions to avoid exposureNtUoRsSuInNliGgThBt .COM b. Teaching about body changes associated with SLE c. Preparation for home schooling d. Restricted activity ANS: A, B Key issues for a child with SLE include therapy compliance; body-image problems associated with rash, hair loss, and steroid therapy; school attendance; vocational activities; social relationships; sexual activity; and pregnancy. Specific instructions for avoiding exposure to the sun and ultraviolet B light, such as using sunscreens, wearing sun-resistant clothing, and altering outdoor activities, must be provided with great sensitivity to ensure compliance while minimizing the associated feeling of being different from peers. The child should continue school attendance in order to gain interaction with peers and activity should not be restricted, but promoted. PTS: 1 DIF: Cognitive Level: Application OBJ: Nursing Process: Implementation MSC: Client Needs: Physiologic Integrity 3. The nurse is caring for a preschool child with a cast applied recently for a fractured tibia. Which assessment findings indicate possible compartment syndrome? (Select all that apply.) a. Palpable distal pulse b. Capillary refill to extremity of <3 seconds c. Severe pain not relieved by analgesics d. Tingling of extremity e. Inability to move extremity ANS: C, D, E Indications of compartment syndrome are severe pain not relieved by analgesics, tingling of extremity, and inability to move extremity. A palpable distal pulse and capillary refill to the extremity of <3 seconds are expected findings. PTS: 1 DIF: Cognitive Level: Application OBJ: Nursing Process: Assessment MSC: Client Needs: Physiologic Integrity: Reduction of Risk Potential NURSINGTB.COM Chapter 49: The Child With Neuromuscular or Muscular Dysfunction Perry: Maternal Child Nursing Care, 6th Edition MULTIPLE CHOICE 1. Spastic cerebral palsy is characterized by what presentation? a. Hypertonicity and poor control of posture, balance, and coordinated motion b. Athetosis and dystonic movements c. Wide-based gait and poor performance of rapid, repetitive movements d. Tremors and lack of active movement ANS: A Hypertonicity and poor control of posture, balance, and coordinated motion are part of the classification of spastic cerebral palsy. Athetosis and dystonic movements are part of the classification of dyskinetic/athetoid cerebral palsy. Wide-based gait and poor performance of rapid, repetitive movements are part of the classification of ataxic cerebral palsy. Tremors and lack of active movement may indicate other neurologic disorders. PTS: 1 DIF: Cognitive Level: Comprehension OBJ: Nursing Process: Assessment MSC: Client Needs: Physiologic Integrity 2. The parents of a child diagnosed with cerebral palsy ask the nurse if any drugs can decrease their child's spasticity. The nurse's response should be based on what knowledge? a. Anticonvulsant medications are sometimes useful for controlling spasticity. b. Medications that would be useful in reducing spasticity are too toxic for use with children. NURSINGTB.COM c. Many different medications can be highly effective in controlling spasticity. d. Implantation of a pump to deliver medication into the intrathecal space to decrease spasticity has recently become available. ANS: D Baclofen given intrathecally is best suited for children with severe spasticity that interferes with activities of daily living and ambulation. Anticonvulsant medications are used when seizures occur in children with cerebral palsy. The intrathecal route decreases the side effects of the drugs that reduce spasticity. Few medications are presently available for the control of spasticity. PTS: 1 DIF: Cognitive Level: Analysis OBJ: Nursing Process: Implementation MSC: Client Needs: Physiologic Integrity 3. What is the most common problem for children born with a myelomeningocele? a. Neurogenic bladder b. Intellectual impairment c. Respiratory compromise d. Cranioschisis ANS: A Myelomeningocele is one of the most common causes of neuropathic (neurogenic) bladder dysfunction among children. Risk of intellectual impairment is minimized through early intervention and management of hydrocephalus. Respiratory compromise is not a common problem in myelomeningocele. Cranioschisis is a skull defect through which various tissues protrude. It is not associated with myelomeningocele. PTS: 1 DIF: Cognitive Level: Comprehension OBJ: Nursing Process: Diagnosis MSC: Client Needs: Physiologic Integrity 4. Which problem is most often associated with myelomeningocele? a. Hydrocephalus b. Craniosynostosis c. Biliary atresia d. Esophageal atresia ANS: A Hydrocephalus is an associated anomaly in 80% to 90% of children. Craniosynostosis is the premature closing of the cranial sutures and is not associated with myelomeningocele. Biliary and esophageal atresias are not associated with myelomeningocele. PTS: 1 DIF: Cognitive Level: Comprehension OBJ: Nursing Process: Assessment MSC: Client Needs: Physiologic Integrity 5. A current recommendation to prevent neural tube defects is the administration of what supplement? a. Vitamin A throughout pregnancy b. Multivitamin preparations as soon as pregnancy is suspected c. Folic acid for all women of chNilUdbReSaINriGngTBa.gCeOM d. Folic acid during the first and second trimesters of pregnancy ANS: C The widespread use of folic acid among women of childbearing age is expected to decrease the incidence of spina bifida significantly. Vitamin A and multivitamin preparations do not have a relation to the prevention of spina bifida. Folic acid supplementation is recommended for the preconceptual period and during the pregnancy. Only 42% of women actually follow these guidelines. PTS: 1 DIF: Cognitive Level: Analysis OBJ: Nursing Process: Implementation MSC: Client Needs: Physiologic Integrity 6. How much folic acid is recommended for women of childbearing age? a. 0.1 mg b. 0.4 mg c. 1.5 mg d. 2 mg ANS: B It has been estimated that a daily intake of 0.4 mg of folic acid in women of childbearing age will prevent 50% to 70% of cases of neural tube defects. A dose of 0.1 mg is too low, and 1.5 mg and 2 mg are not recommended dosages of folic acid. PTS: 1 DIF: Cognitive Level: Comprehension OBJ: Nursing Process: Implementation MSC: Client Needs: Physiologic Integrity 7. The nurse is caring for a neonate born with a myelomeningocele. Surgery to repair the defect is scheduled the next day. The most appropriate way to position and feed this neonate is to place him: a. prone and tube feed. b. prone, turn head to side, and nipple feed. c. supine in infant carrier and nipple feed. d. supine, with defect supported with rolled blankets, and nipple feed. ANS: B In the prone position, feeding is a problem. The infant's head is turned to one side for feeding. If the child is able to nipple feed, no indication is present for tube feeding. Before surgery, the infant is kept in the prone position to minimize tension on the sac and risk of trauma. PTS: 1 DIF: Cognitive Level: Application OBJ: Nursing Process: Implementation MSC: Client Needs: Physiologic Integrity 8. The nurse is talking to a parent with a child who has a latex allergy. Which statement by the parent would indicate a correct understanding of the teaching? a. “My child will have an allergic reaction if he comes in contact with yeast products.” b. “My child may have an upset stomach if he eats a food made with wheat or barley.” c. “My child will probably develop an allergy to peanuts.” d. “My child should not eat bananas or kiwis.” ANS: D NURSINGTB.COM There are cross-reactions between latex allergies and a number of foods such as bananas, avocados, kiwi, and chestnuts. Although yeast products, wheat and barley, and peanuts are potential allergens, they are currently not known to cross-react with latex. PTS: 1 DIF: Cognitive Level: Analysis OBJ: Nursing Process: Planning MSC: Client Needs: Physiologic Integrity 9. Latex allergy is suspected in a child with spina bifida. What intervention should be included in the child's plan of care? a. Avoiding using any latex product b. Using only nonallergenic latex products c. Administering medication for long-term desensitization d. Teaching the family about long-term management of asthma ANS: A Care must be taken that individuals who are at high risk for latex allergies do not come in direct or secondary contact with products or equipment containing latex at any time during medical treatment. There are no nonallergenic latex products. At this time desensitization is not an option. The child does not have asthma. The parents must be taught about allergy and the risk of anaphylaxis. PTS: 1 DIF: Cognitive Level: Application OBJ: Nursing Process: Implementation MSC: Client Needs: Physiologic Integrity 10. Which finding should cause the nurse to suspect a diagnosis of spastic cerebral palsy? a. Tremulous movements at rest and with activity b. Positive Babinski reflex c. Writhing, uncontrolled, involuntary movements d. Clumsy, uncoordinated movements ANS: B Spastic cerebral palsy, the most common type of cerebral palsy, will manifest with persistent primitive reflexes, positive Babinski reflex, ankle clonus, exaggerated stretch reflexes, and eventual development of contractures. The child's muscles are very tight and any stimuli may cause a sudden jerking movement. Tremulous movements are characteristic of rigid/tremor/atonic cerebral palsy. Slow, writhing, uncontrolled, involuntary movements occur with athetoid or dyskinetic cerebral palsy. Clumsy movements, loss of coordination, equilibrium, and kinesthetic sense occur in ataxic cerebral palsy. PTS: 1 DIF: Cognitive Level: Application OBJ: Nursing Process: Assessment MSC: Client Needs: Physiologic Integrity 11. A young boy has just been diagnosed with pseudohypertrophic muscular dystrophy. The management plan should include which intervention? a. Recommending genetic counseling b. Explaining that the disease is easily treated c. Suggesting ways to limit the use of muscles d. Assisting the family in finding a nursing facility to provide his care ANS: A Pseudohypertrophic (Duchenne's) muscular dystrophy is inherited as an X-linked recessive gene. Genetic counseling is recomNmUeRnSdINedGfToBr.CpOarMents, female siblings, maternal aunts, and their female offspring. No effective treatment exists at this time for childhood muscular dystrophy. Maintaining optimal function of all muscles for as long as possible is the primary goal. It has been found that children who remain as active as possible are able to avoid wheelchair confinement for a longer time. Assisting the family in finding a nursing facility is inappropriate at the time of diagnosis. When the child becomes increasingly incapacitated, the family may consider home-based care, a skilled nursing facility, or respite care to provide the necessary care. PTS: 1 DIF: Cognitive Level: Application OBJ: Nursing Process: Implementation MSC: Client Needs: Physiologic Integrity 12. What is the most appropriate nursing response to the father of a newborn infant with myelomeningocele who asks about the cause of this condition? a. “One of the parents carries a defective gene that causes myelomeningocele.” b. “A deficiency in folic acid in the father is the most likely cause.” c. “Offspring of parents who have a spinal abnormality are at greater risk for myelomeningocele.” d. “There may be a variety of different causes.” ANS: D The etiology of most neural tube defects is likely multifactorial. There may be a genetic predisposition or a viral origin, and the disorder has been linked to maternal folic acid deficiency; however, the actual cause has not been determined. There is no evidence that children who have parents with spinal problems are at greater risk for neural tube defects. PTS: 1 DIF: Cognitive Level: Application OBJ: Nursing Process: Implementation MSC: Client Needs: Psychosocial Integrity 13. Which statement best describes a myelomeningocele? a. Fissure in the spinal column that leaves the meninges and the spinal cord exposed. b. Herniation of the brain and meninges through a defect in the skull. c. Hernial protrusion of a sac-like cyst of meninges with spinal fluid but no neural elements. d. Visible defect with an external sac-like protrusion containing meninges, spinal fluid, and nerves. ANS: D A myelomeningocele is a visible defect with an external sac-like protrusion, containing meninges, spinal fluid, and nerves. Rachischisis is a fissure in the spinal column that leaves the meninges and the spinal cord exposed. Encephalocele is a herniation of brain and meninges through a defect in the skull, producing a fluid-filled sac. Meningocele is a hernial protrusion of a sac-like cyst of meninges with spinal fluid, but no neural elements. PTS: 1 DIF: Cognitive Level: Comprehension OBJ: Nursing Process: Assessment MSC: Client Needs: Physiologic Integrity 14. Which signs and symptoms are associated with Werdnig-Hoffmann disease? a. Spinal muscular atrophy b. Neural atrophy of muscles NURSINGTB.COM c. Progressive weakness and wasting of skeletal muscle d. Pseudohypertrophy of certain muscle groups ANS: C Werdnig-Hoffmann disease (spinal muscular atrophy type 1) is the most common paralytic form of floppy infant syndrome (congenital hypotonia). It is characterized by progressive weakness and wasting of skeletal muscle caused by degeneration of anterior horn cells. Kugelberg-Welander syndrome is a juvenile spinal muscular atrophy with a later onset. Charcot-Marie-Tooth disease is a form of progressive neural atrophy of muscles supplied by the peroneal nerves. Progressive weakness of the distal muscles of the arms and feet is found. Duchenne's muscular dystrophy is characterized by muscles, especially in the calves, thighs, and upper arms that become enlarged from fatty infiltration and feel unusually firm or woody on palpation. The term pseudohypertrophy is derived from this muscular enlargement. PTS: 1 DIF: Cognitive Level: Comprehension OBJ: Nursing Process: Assessment MSC: Client Needs: Physiologic Integrity 15. Which interventions should the nurse include in the plan of care for the infant awaiting surgical closure of a myelomeningocele sac? a. Open to air b. Covered with a sterile, moist, nonadherent dressing c. Reinforcement of the original dressing if drainage noted d. A diaper secured over the dressing ANS: B Before surgical closure, the myelomeningocele is prevented from drying by the application of a sterile, moist, nonadherent dressing over the defect. The moistening solution is usually sterile normal saline. Dressings are changed frequently (every 2 to 4 hours), and the sac is closely inspected for leaks, abrasions, irritation, and any signs of infection. The sac must be carefully cleansed if it becomes soiled or contaminated. The original dressing would not be reinforced but changed as needed. A diaper is not placed over the dressing because stool contamination can occur. PTS: 1 DIF: Cognitive Level: Application OBJ: Nursing Process: Implementation MSC: Client Needs: Physiologic Integrity 16. The nurse is admitting a school-age child with suspected Guillain-Barré syndrome (GBS). Which nursing intervention is a priority in the care for this child? a. Monitoring intake and output b. Assessing respiratory efforts c. Placing on a telemetry monitor d. Obtaining laboratory studies ANS: B Treatment of GBS is primarily supportive. In the acute phase, patients are hospitalized because respiratory and pharyngeal involvement may require assisted ventilation, sometimes with a temporary tracheotomy. Treatment modalities include aggressive ventilatory support in the event of respiratory compromise, administration of intravenous immunoglobulin (IVIG), and sometimes steroids; plasmapheresis and immunosuppressive drugs may also be used. Monitoring intake and output, teleNmUeRtrSyINmGoTnBi.tCoOriMng, and obtaining laboratory studies may be part of the plan of care but are not the priority. PTS: 1 DIF: Cognitive Level: Analysis OBJ: Nursing Process: Evaluation MSC: Client Needs: Physiologic Integrity 17. The nurse is caring for an intubated infant with botulism. Which health care provider prescriptions should the nurse clarify with the health care provider before implementing? a. Administer 250 mg botulism immune globulin intravenously (BIG-IV) one time. b. Provide total parenteral nutrition (TPN) at 25 mL/hr intravenously. c. Titrate oxygen to keep pulse oximetry saturations greater than 92. d. Administer gentamicin sulfate 10 mg per intravenous piggyback every 12 hours. ANS: D The nurse should clarify the administration of an aminoglycoside antibiotic. Antibiotic therapy is not part of the management of infant botulism because the botulinum toxin is an intracellular molecule, and antibiotics would not be effective; aminoglycosides in particular should not be administered because they may potentiate the blocking effects of the neurotoxin. Treatment consists of immediate administration of botulism immune globulin intravenously (BIG-IV) without delaying for laboratory diagnosis. Early administration of BIG-IV neutralizes the toxin and stops the progression of the disease. The human-derived botulism antitoxin (BIG-IV) has been evaluated and is now available nationwide for use only in infant botulism. Approximately 50% of affected infants require intubation and mechanical ventilation; therefore, respiratory support is crucial, as is nutritional support because these infants are unable to feed. PTS: 1 DIF: Cognitive Level: Analysis OBJ: Nursing Process: Evaluation MSC: Client Needs: Physiologic Integrity MULTIPLE RESPONSE 1. A 14 year old is in the intensive care unit after a spinal cord injury 2 days ago. Which nursing care interventions are needed for this child? (Select all that apply.) a. Monitoring and maintaining systemic blood pressure. b. Administering corticosteroids. c. Minimizing environmental stimuli. d. Discussing long-term care issues with the family. e. Monitoring for respiratory complications. ANS: A, B, E Spinal cord injury patients are phyNsUioRlSoIgNicGaTlBly.ClOabMile, and close monitoring of blood pressure and respirations is required. They may be unstable for the first few weeks after the injury. Corticosteroids are administered to minimize the inflammation present with the injury. Minimizing environmental stimuli and discussing long-term care issues with the family do not apply to providing care for this patient. PTS: 1 DIF: Cognitive Level: Application OBJ: Nursing Process: Implementation MSC: Client Needs: Physiologic Integrity 2. Which assessment findings should the nurse note in a school-age child diagnosed with Duchenne's muscular dystrophy (DMD)? (Select all that apply.) a. Lordosis b. Gower's sign c. Kyphosis d. Scoliosis e. Waddling gait ANS: A, B, E Difficulties in running, riding a bicycle, and climbing stairs are usually the first symptoms noted in DMD. Typically, affected boys have a waddling gait and lordosis, fall frequently, and develop a characteristic manner of rising from a squatting or sitting position on the floor (Gower's sign). Lordosis occurs as a result of weakened pelvic muscles, and the waddling gait is a result of weakness in the gluteus medius and maximus muscles. Kyphosis and scoliosis are not assessment findings with DMD. PTS: 1 DIF: Cognitive Level: Application OBJ: Nursing Process: Assessment MSC: Client Needs: Physiologic Integrity 3. The nurse is conducting discharge teaching with parents of a preschool child with a myelomeningocele, repaired at birth, who is being discharged from the hospital after a urinary tract infection (UTI). Which should the nurse include in the discharge instructions related to management of the child's genitourinary function? (Select all that apply.) a. Continue to perform the clean intermittent catheterizations (CIC) at home. b. Administer the oxybutynin chloride as prescribed. c. Reduce fluid intake in the afternoon and evening hours. d. Monitor for signs of a recurrent UTI. e. Administer furosemide as prescribed. ANS: A, B, D Discharge teaching to prevent renal complications in a child with myelomeningocele include: (1) regular urologic care with prompt and vigorous treatment of infections; (2) a method of regular emptying of the bladder, such as clean intermittent catheterization (CIC) taught to and performed by parents and self-catheterization taught to children; and (3) medications to improve bladder storage and continence, such as oxybutynin chloride and tolterodine. Fluids should not be limited, and furosemide is not used to improve renal function for children with myelomeningocele. PTS: 1 DIF: Cognitive Level: Application OBJ: Nursing Process: Implementation MSC: Client Needs: Physiologic Integrity: Reduction of Risk Potential 4. The nurse is caring for an infant wNiUthRmSIyNeGloTmB.eCnOiMngocele who is scheduled for surgical repair in the morning. Which early signs of infection should the nurse monitor on this infant? (Select all that apply.) a. Temperature instability b. Irritability c. Lethargy d. Bradycardia e. Hypertension ANS: A, B, C The nurse should observe an infant with unrepaired myelomeningocele for early signs of infection, such as temperature instability (axillary), irritability, and lethargy. Bradycardia and hypertension are not early signs of infection in infants. PTS: 1 DIF: Cognitive Level: Comprehension OBJ: Nursing Process: Evaluation MSC: Client Needs: Physiologic Integrity [Show More]

Last updated: 11 months ago

Preview 1 out of 1157 pages

Reviews( 0 )

$17.00

Add to cart

Instant download

Can't find what you want? Try our AI powered Search

OR

GET ASSIGNMENT HELP
36
0

Document information


Connected school, study & course


About the document


Uploaded On

Dec 19, 2022

Number of pages

1157

Written in

Seller


seller-icon
Nolan19

Member since 2 years

10 Documents Sold


Additional information

This document has been written for:

Uploaded

Dec 19, 2022

Downloads

 0

Views

 36

Document Keyword Tags

Recommended For You


$17.00
What is Browsegrades

In Browsegrades, a student can earn by offering help to other student. Students can help other students with materials by upploading their notes and earn money.

We are here to help

We're available through e-mail, Twitter, Facebook, and live chat.
 FAQ
 Questions? Leave a message!

Follow us on
 Twitter

Copyright © Browsegrades · High quality services·